Вы находитесь на странице: 1из 435

9/

i r9
a h
r/ t
s e
/r u
.t c
k a
/: /
s
tt p
h
9/
i r9
a h
r/ t
s e
/r u
.t c
k a
/: /
s
tt p
h
ECHOCARDIOGRAPHY
REVIEW GUIDE

9/
i r9
a h
r/ t
s e
/r u
.t c
k a
/: /
s
tt p
h
This page intentionally left blank
     
9/
i r9
a h
r/ t
s e
/r u
.t c
k a
/: /
s
tt p
h
ECHOCARDIOGRAPHY
REVIEW GUIDE
9/
Companion to the Textbook of Clinical Echocardiography

ri 9
h
THIRD EDITION

/ ta
r
CATHERINE M. OTTO, MD

e
J. Ward Kennedy-Hamilton Endowed Chair in Cardiology

s
Professor of Medicine

/r u
University of Washington School of Medicine
Director, Heart Valve Clinic
University of Washington Medical Center

.t c
Seattle, Washington

a
REBECCA GIBBONS SCHWAEGLER, BS, RDCS

k
Adjunct Professor

/
Diagnostic Ultrasound Department

: /
College of Science and Engineering

s
Seattle University
Cardiac Sonographer

tt p
University of Washington Medical Center
Seattle, Washington

h
ROSARIO V. FREEMAN, MD, MS
Associate Professor of Medicine
Director, Training Programs in Cardiovascular Disease
University of Washington School of Medicine
Director, Echocardiography Laboratory
University of Washington Medical Center
Seattle, Washington
1600 John F. Kennedy Blvd.
Ste 1800
Philadelphia, PA 19103-2899

ECHOCARDIOGRAPHY REVIEW GUIDE, THIRD EDITION ISBN: 978-0-323-22758-2

 
Copyright © 2016, 2011, 2008 by Saunders, an imprint of Elsevier Inc.

/
All rights reserved. No part of this publication may be reproduced or transmitted in any form or by any means,
electronic or mechanical, including photocopying, recording, or any information storage and retrieval system,

9
without permission in writing from the Publisher. Details on how to seek permission, further information about
the Publisher’s permissions policies, and our arrangements with organizations such as the Copyright Clearance
Center and the Copyright Licensing Agency can be found at our website: www.elsevier.com/permissions.

r 9
This book and the individual contributions contained in it are protected under copyright by the Publisher

i
(other than as may be noted herein).

h
Notices

ta
Knowledge and best practice in this field are constantly changing. As new research and experience broaden

r/
our understanding, changes in research methods, professional practices, or medical treatment may become
necessary.
Practitioners and researchers must always rely on their own experience and knowledge in evaluating and

e
using any information, methods, compounds, or experiments described herein. In using such information or
methods, they should be mindful of their own safety and the safety of others, including parties for whom they

s
have a professional responsibility.
With respect to any drug or pharmaceutical products identified, readers are advised to check the most

/r u
current information provided (i) on procedures featured or (ii) by the manufacturer of each product to be
administered and to verify the recommended dose or formula, the method and duration of administration,
and contraindications. It is the responsibility of practitioners, relying on their own experience and knowledge
of their patients, to make diagnoses, to determine dosages and the best treatment for each individual patient,
and to take all appropriate safety precautions.

.t c
To the fullest extent of the law, neither the Publisher nor the authors, contributors, or editors assume any
liability for any injury and/or damage to persons or property as a matter of products liability, negligence,
or otherwise or from any use or operation of any methods, products, instructions, or ideas contained in the
material herein

Library of Congress Cataloging-in-Publication Data

k a
/: /
Otto, Catherine M., author.
Echocardiography review guide : companion to the Textbook of clinical
echocardiography / Catherine M. Otto, Rebecca Gibbons Schwaegler, Rosario V.

s
Freeman. -- Third edition.
p. ; cm.
The third edition of Echocardiography review guide complements the fifth

tt p
edition of Textbook of clinical echocardiography.
ISBN 978-0-323-22758-2 (alk. paper)
I. Schwaegler, Rebecca Gibbons, author. II. Freeman, Rosario V., author.
III. Otto, Catherine M. Textbook of clinical echocardiography. 5th ed.

h
Supplement to (expression): IV. Title.
[DNLM: 1. Echocardiography--methods--Examination Questions. 2.
Echocardiography--methods--Outlines. 3. Heart
Diseases--ultrasonography--Examination Questions. 4. Heart
Diseases--ultrasonography--Outlines. WG 18.2]
RC683.5.U5
616.1’2--dc23
2015007664
Content Strategist: Dolores Melonie
Content Development Specialist: Joanie Milnes
Publishing Services Manager: Anne Altepeter
Senior Project Manager: Doug Turner
Manager, Art and Design: Teresa McBryan

Printed in the United States of America

Last digit is the print number: 9 8 7 6 5 4 3 2 1











Introduction

Echocardiography Review Guide, Third

9/
Each chapter includes the Echo Exam review
guide from the Textbook of Clinical Echocardiogra-

9
Edition: The Companion Workbook for phy for quick reference. In addition, a step-by-step

r
Textbook of Clinical Echocardiography,

i
approach to patient examination is detailed. Infor-
Fifth Edition mation is conveyed in bulleted points, with each set

h
The third edition of Echocardiography Review Guide com- of major principles followed by a list of key points.
plements the fifth edition of Textbook of Clinical Echo- Potential pitfalls are identified and approaches to

a
cardiography, providing a review of basic principles, avoiding errors are provided. Data measurements

t
additional details of data acquisition and interpreta- and calculations are explained with specific exam-

r/
tion, and a step-by-step approach to patient exami- ples. Numerous illustrations with detailed figure
nation for each diagnosis. In addition, self-assessment legends demonstrate each major idea and guide the
questions, with detailed explanations of the correct reader through the teaching points. Self-assessment

e
answers, allow the reader to be more actively involved questions are included with each chapter to help

s
in the leaning process. the reader consolidate the information and identify
This book will be of interest to practicing cardiolo- areas where further study is needed. Along with the

/r u
gists and sonographers as a quick update on echocar- correct answer to each question, a brief discussion
diography and will be of value for cardiology fellows details how that answer was determined and why
and cardiac sonographer students who are mastering the other potential answers are not correct. The
the material for the first time. Cardiac anesthesiolo- questions are based on information presented in

.t c
gists will find helpful information about details of the both the textbook and in the review guide; in effect,
examination and a chapter dedicated to intraopera- this review guide is the “workbook” that accompa-
tive transesophageal echocardiography. In addition, nies the textbook.

a
primary care, emergency department, and intensive Any book on echocardiography is only a supple-
care physicians using point-of-care echocardiography ment to formal training; no book can replace hands-

k
can refer to this book to get started and to improve on training or practical experience. The three of us

/: /
their echocardiography skills. Multiple-choice ques- fully endorse the current standards for educating and
tions provide a review and self-assessment for those training physicians and sonographers in clinical car-
preparing for echocardiography examinations and diac ultrasound as provided by the American Society

s
may be useful in echocardiography laboratories for of Echocardiography, the American Heart Associa-
continuous quality improvement processes. tion, the American College of Cardiology, and the

tt p
The chapters are arranged in the same order as Society for Cardiovascular Anesthesiology. We sup-
those in the Textbook of Clinical Echocardiography, and we port training in accredited programs with formal
recommend that these two books be used in parallel. certification of sonographers and evaluation of physi-
As in the textbook, there are introductory chapters on cian competency. The material in this book reflects

h
basic principles of image acquisition, transthoracic the clinical practice of echocardiography at one point
and transesophageal echocardiography, other echo- in time. Cardiac imaging is a rapidly changing field,
cardiographic modalities, and clinical indications. and we encourage our readers to stay up to date by
Each of the subsequent chapters focuses on a specific reading journals and other online sources and by
clinical diagnosis, including ventricular systolic and attending national meetings and continuing medical
diastolic function, ischemic cardiac disease, cardiomy- education courses.
opathies, valve stenosis and regurgitation, prosthetic
Catherine M. Otto, MD
valves, endocarditis, cardiac masses, aortic disease,
adult congenital heart disease, and procedural trans- Rebecca Gibbons Schwaegler, BS, RDCS
esophageal echocardiography. Rosario V. Freeman, MD, MS

v
Acknowledgments

I t is never possible to fully acknowledge all those who help make a book possible; how-
ever, we would like to thank some of those who helped us along the way. First, the cardiac

9/
9
sonographers at the University of Washington deserve our special appreciation for the excel-

r
lence of their imaging skills and the time they dedicated to acquiring additional images for

i
us and discussing the finer points of data acquisition. These sonographers include Caryn
D’Jang, RDCS; Margaret Fallenreck, RDCS; Michelle Fujioka, RDCS; Carolyn Gardner,

h
RDCS; Yelena Kovalenko, RDCS; Todd Zwink, RDCS; Pamela Clark, RDCS; Sarah Cur-

a
tis, RDCS; Carol Kraft, RDCS; Chris McKenzie, RDCS; Irina Nesterova, RDCS; Hoang

t
Nguyen, RDCS; Amy Owens, RDCS; Joannalyn Shephard, RDCS; and Yu Wang, RDCS.

r/
Special thanks are due the many readers who provided comments and input on the text and
questions. Our appreciation extends to Delores Meloni, Joanie Milnes, and Doug Turner at
Elsevier and to the production team who supported this project and helped us make it a reality.

e
Finally, we all sincerely thank our families, not only our husbands for their unwaver-
ing and continual encouragement, but also the younger members—Vea, Remy, Brendan,

s
Sarah, Claire, Jack, Anna, and Fiona—for their support and patience in the book-writing

/r u
process. This book would not have been possible without their helping us find the time to
bring this project to fruition.
Catherine M. Otto, MD
Rebecca Gibbons Schwaegler, BS, RDCS

.t c
Rosario V. Freeman, MD, MS

k a
/: /
s
tt p
h

vi
Contents

1. Principles of Echocardiographic Image Acquisition and Doppler Analysis, 1

9/
9



r
2. Normal Anatomy and Flow Patterns on Transthoracic Echocardiography, 21

i



3. Transesophageal Echocardiography, 44

h



a
4. Advanced Echocardiographic Modalities, 65

t



r/
5. Clinical Indications and Quality Assurance, 85



6. Left and Right Ventricular Systolic Function, 98

e



s
7. Ventricular Diastolic Filling and Function, 123



/r u
8. Coronary Artery Disease, 142



9. Cardiomyopathies, Hypertensive, and Pulmonary Heart Disease, 168

.t c



10. Pericardial Disease, 193



a
11. Valvular Stenosis, 213



k
12. Valvular Regurgitation, 235

/: /



13. Prosthetic Valves, 267



s
14. Endocarditis, 293

tt p



15. Cardiac Masses and Potential Cardiac Source of Embolus, 313



16. Diseases of the Great Arteries, 334

h



17. The Adult with Congenital Heart Disease, 355



18. Intraoperative and Interventional Echocardiography, 384



vii
Glossary
  
2D = two-dimensional
3D = three-dimensional
E = early diastolic peak velocity

9/
E′ = early diastolic tissue Doppler velocity

9
ECG = electrocardiogram

r
A = late diastolic ventricular filling velocity with echo = echocardiography

i
atrial contraction ED = end-diastole

h
A′ = diastolic tissue Doppler velocity with atrial EDD = end-diastolic dimension
contraction EDV = end-diastolic volume

a
A2C = apical 2-chamber EF = ejection fraction

t
A4C = apical 4-chamber endo = endocardium

r/
AcT = acceleration time epi = epicardium
AF = atrial fibrillation EPSS = E-point septal separation
A-long = apical long-axis EROA = effective regurgitant orifice area

e
A-mode = amplitude mode (amplitude versus depth) ES = end-systole

s
AMVL = anterior mitral valve leaflet ESD = end-systolic dimension
Ao = aortic or aorta ESV = end-systolic volume

/r u
AR = aortic regurgitation ETT = exercise treadmill test
AS = aortic stenosis
ASD = atrial septal defect Δf = frequency shift
ATVL = anterior tricuspid valve leaflet f = frequency

.t c
AV = atrioventricular FL = false lumen
AVA = aortic valve area Fn = near field frequency
AVR = aortic valve replacement Fo = resonance frequency

a
Fs = scattered frequency
BAV = bicuspid aortic valve FSV = forward stroke volume

k
BP = blood pressure Ft = transmitted frequency

/: /
BSA = body surface area
GLS = global longitudinal strain
c = propagation velocity of sound in tissue

s
CAD = coronary artery disease HCM = hypertrophic cardiomyopathy
cm = centimeters HFpEF = heart failure with preserved ejection

tt p
CMR = cardiac magnetic resonance imaging fraction
­
cm/s = centimeters per second HFrEF = heart failure with reduced ejection
CO = cardiac output fraction
cos = cosine HPRF = high pulse repetition frequency

h
CS = coronary sinus HR = heart rate
CSA = cross-sectional area HV = hepatic vein
CT = computed tomography Hz = Hertz (cycles per second)
CW = continuous wave
Cx = circumflex coronary artery I = intensity of ultrasound exposure
IAS = interatrial septum
D = diameter ICD = implantable cardioverter defibrillator
DA = descending aorta ICE = intracardiac echocardiography
dB = decibels IV = intravenous
dP/dt = rate of change in pressure over time IVC = inferior vena cava
DSE = dobutamine stress echocardiography IVCT = isovolumic contraction time
dT/dt = rate of increase in temperature IVRT = isovolumic relaxation time
dyne · s · cm–5 = units of resistance

viii
GLOSSARY ix

kHz = kilohertz PVC = premature ventricular contraction


PVR = pulmonary vascular resistance
L = length PWT = posterior wall thickness
LA = left atrium or left atrial
LAA = left atrial appendage Q = volume flow rate
LAD = left anterior descending coronary artery Qp = pulmonic volume flow rate
LAE = left atrial enlargement Qs = systemic volume flow rate
LCC = left coronary cusp
LMCA = left main coronary artery RA = right atrium or right atrial
LPA = left pulmonary artery RAE = right atrial enlargement
LSPV = left superior pulmonary vein RAO = right anterior oblique
L-TGA = congenitally corrected transposition of RAP = right atrial pressure
the great arteries RCA = right coronary artery
LV = left ventricle or left ventricular RCC = right coronary cusp
LVEDD = left ventricular end-diastolic Re = Reynolds number
dimension RF = regurgitant fraction
LVEDP = left ventricular end-diastolic RFR = regurgitant instantaneous flow rate
pressure RJ = regurgitant jet
LVESD = left ventricular end-systolic dimension Ro = radius of microbubble
LVH = left ventricular hypertrophy ROA = regurgitant orifice area
LVID = left ventricular internal dimension RPA = right pulmonary artery
LVOT = left ventricular outflow tract RSPV = right superior pulmonary vein
LVPW = left ventricular posterior wall RSV = regurgitant stroke volume
RV = right ventricle or regurgitant volume (depends
MAC = mitral annular calcification on context)
MI = myocardial infarction RVE = right ventricular enlargement
M-mode = motion display (depth versus time) RVH = right ventricular hypertrophy
MR = mitral regurgitation RVOT = right ventricular outflow tract
MS = mitral stenosis
MVA = mitral valve area s = second
MVL = mitral valve leaflet SAM = systolic anterior motion
MVR = mitral valve replacement SC = subcostal
SEE = standard error of the estimate
NBTE = nonbacterial thrombotic endocarditis SPPA = spatial peak pulse average
NCC = noncoronary cusp SPTA = spatial peak temporal average
NSTEMI = non–ST-segment elevation myocardial SSN = suprasternal notch
infarction ST = septal thickness
STE = speckle tracking echocardiography
ΔP = pressure gradient STEMI = ST-segment elevation myocardial
PA = pulmonary artery infarction
PAP = pulmonary artery pressure STJ = sinotubular junction
PDA = patent ductus arteriosus or posterior STVL = septal tricuspid valve leaflet
­descending artery (depends on context) SV = stroke volume or sample volume (depends
PE = pericardial effusion on context)
PEP = preejection period SVC = superior vena cava
PET = positron-emission tomography
PFO = patent foramen ovale T½ = pressure half-time
PHT = pressure half time TAPSE = tricuspid annular plane systolic
PISA = proximal isovelocity surface area excursion
PLAX = parasternal long-axis TAVR = transcatheter aortic valve replacement
PM = papillary muscle TEE = transesophageal echocardiography
PMVL = posterior mitral valve leaflet TG = transgastric
PR = pulmonic regurgitation TGA = transposition of the great arteries
PRF = pulse repetition frequency TGC = time gain compensation
PRFR = peak rapid filling rate Th = wall thickness
PS = pulmonic stenosis TL = true lumen
PSAX = parasternal short-axis TN = true negatives
PV = pulmonary vein TOF = tetralogy of Fallot
x GLOSSARY

TP = true positives Symbols Greek Name Used for


TPV = time to peak velocity
TR = tricuspid regurgitation α alpha frequency
TS = tricuspid stenosis γ gamma viscosity
TSV = total stroke volume Δ delta difference
TTE = transthoracic echocardiography
TV = tricuspid valve Ɵ theta angle
λ lambda wavelength
v = velocity μ mu micro-
V = volume or velocity (depends on context)
VAS = ventriculo-atrial septum π pi mathematical constant
Veg = vegetation (~3.14)
Vmax = maximum velocity ƿ rho tissue density
VSD = ventricular septal defect Ϭ sigma wall stress
VTI = velocity-time integral
τ tau time constant of ventricular
relaxation
Z = acoustic impedance
  

Units of Measure

Variable Unit Definition


Amplitude dB Decibels = a logarithmic scale describing the amplitude (“loudness”)
of the sound wave
Angle degrees Degree = (π/180)rad. Example: intercept angle
Area cm2 Square centimeters. A 2D measurement (e.g., end-systolic area) or a
calculated value (e.g., continuity equation valve area)
Frequency (f ) Hz Hertz (cycles per second)
kHz Kilohertz = 1000 Hz
MHz Megahertz = 1 million Hz
Length cm Centimeter (1/100 m)
mm Millimeter (1/1000 m or 1/10 cm)
Mass g Grams. Example: LV mass
Pressure mmHg Millimeters of mercury, 1 mmHg = 1333.2 dyne/cm2, where dyne
measures force in cm · g · s–2
Resistance dyne · s · cm–5 Measure of vascular resistance
Time s Second
ms Millisecond (1/1000 s)
μs Microsecond
Ultrasound intensity W/cm2 Where watt (W) = joule per second and joule = m2 · kg · s–2 (unit of
mW/cm2 energy)
Velocity (v) m/s Meters per second
cm/s Centimeters per second
Velocity-time integral (VTI) cm Integral of the Doppler velocity curve (cm/s) over time (s), in units of cm
Volume cm3 Cubic centimeters
mL Milliliter, 1 mL = 1 cm3
L Liter = 1000 mL
Volume flow rate (Q) Rate of volume flow across a valve or in cardiac output
L/min L/min = liters per minute
mL/s mL/s = milliliters per second
Wall stress unit dyne/cm2 Units of meridional or circumferential wall stress
kdyn/cm2 Kilodynes per cm2
kPa Kilopascals, where 1 kPa = 10 kdyn/cm2
Key Equations

ULTRASOUND PHYSICS
Frequency f = cycles/s = Hz
Wavelength λ = c/f = 1.54/f (MHz)
Doppler equation v = c × ΔF / [2 FT (cosθ)]
Bernoulli equation ΔP = 4V2
LV IMAGING
Stroke volume SV = EDV – ESV
Ejection fraction EF(%) = (SV/EDV) × 100%
Wall stress σ = PR/2Th
DOPPLER VENTRICULAR FUNCTION
Stroke volume SV = CSA × VTI
Rate of pressure rise dP/dt = 32 mmHg / time from 1 to 3 m/s of MR CW jet(sec)
PULMONARY PRESSURES AND RESISTANCE
Pulmonary systolic pressure PAPsystolic = 4(VTR)2 + RAP
PAP (when PS is present) PAPsystolic = [4(VTR)2 + RAP] – ΔPRV–PA
Pulmonary vascular resistance PVR ≅ 10(VTR) / VTIRVOT
AORTIC STENOSIS
Maximum pressure gradient (integrate over ΔPmax = 4(Vmax)2
ejection period for mean gradient)
Continuity equation valve area AVA(cm2) = [π(LVOTD / 2)2 × VTILVOT] / VTIAS–Jet
Simplified continuity equation AVA(cm2) = [π(LVOTD / 2)2 × VLVOT] / VAS–Jet
Velocity ratio Velocity ratio = VLVOT / VAS–Jet
MITRAL STENOSIS
Pressure half-time valve area MVADoppler = 220/T½
AORTIC REGURGITATION
Total stroke volume TSV = SVLVOT = (CSALVOT × VTILVOT)
Forward stroke volume FSV = SVMA = (CSAMA × VTIMA)
Regurgitant volume RVol = TSV – FSV
Regurgitant orifice area ROA = RSV / VTIAR
MITRAL REGURGITATION
Total stroke volume (or 2D or 3D LV stroke volume) TSV = SVMA = (CSAMA × VTIMA)
Forward stroke volume FSV = SVLVOT = (CSALVOT × VTILVOT)
Regurgitant volume RV = TSV – FSV
Regurgitant orifice area ROA = RV / VTIMR
PISA method
Regurgitant flow rate RFR = 2πr2 × Valiasing
Orifice area (maximum) ROAmax = RFR / VMR
Regurgitant volume RV = ROA × VTIMR
AORTIC DILATION
Predicted sinus diameter
Children (<18 years): predicted sinus dimension = 1.02 + (0.98 BSA)
Adults (age 18–40 years): predicted sinus dimension = 0.97 + (1.12 BSA)
Adults (>40 years): predicted sinus dimension = 1.92 + (0.74 BSA)
Ratio = Measured maximum diameter/Predicted maximum diameter
PULMONARY (QP) TO SYSTEMIC (QS) SHUNT RATIO
Qp:Qs = [CSAPA × VTIPA] / [CSALVOT × VTILVOT]

xi
This page intentionally left blank

     
Principles of Echocardiographic Image
1 Acquisition and Doppler Analysis
BASIC PRINCIPLES Color Doppler
ULTRASOUND WAVES AND TISSUE INTERACTION Continuous Wave Doppler
TRANSDUCERS Doppler Artifacts
ULTRASOUND IMAGING BIOEFFECTS AND SAFETY
Basic Principles THE ECHO EXAM
Imaging Artifacts SELF-ASSESSMENT QUESTIONS
DOPPLER
Pulsed Doppler

• 
Refraction (used to focus the ultrasound beam)
BASIC PRINCIPLES • 
Attenuation (loss of signal strength in the tissue)
n nowledge of basic ultrasound principles is
K
needed for interpretation of images and Doppler v  KEY POINTS
data. o  coustic impedance is directly related to tissue
A
n Appropriate adjustment of instrument parameters density and the propagation velocity of ultra-
is needed to obtain diagnostic information. sound in that tissue.
o Ultrasound reflection occurs at smooth tissue
v  KEY POINTS boundaries with different acoustic impedances
o  he appropriate ultrasound modality (two-
T (such as between blood and myocardium).
dimensional [2D] or three-dimensional [3D] Reflection is greatest when the ultrasound’s
imaging, pulsed Doppler, color Doppler, etc.) beam is perpendicular to the tissue interface.
is chosen for each type of clinical information o Ultrasound scattering that occurs with small

required. structures (such as red blood cells) is used to gen-
o Current instrumentation allows modification
 erate Doppler signals. Doppler velocity record-
of many parameters during data acquisition, ings are most accurate when the ultrasound
such as depth, gain, harmonic imaging, wall beam is parallel to the blood flow direction.
filters, etc. o Attenuation is the loss in signal strength due
o Artifacts must be distinguished from anatomic to absorption of ultrasound energy by tissues.
findings on ultrasound images. Tissue penetration is greatest with a lower fre-
o Accurate Doppler measurements depend on
 quency transducer (e.g., 2 to 3 MHz).
details of both blood flow interrogation and o Resolution has three dimensions—axial, lat-

instrument acquisition parameters. eral, and azimuthal. Image resolution is greatest
along the length of the ultrasound beam (axial
resolution) and is best (∼1 mm) with a higher
ULTRASOUND WAVES AND TISSUE frequency transducer (e.g., 5 to 7.5 MHz;
INTERACTION Fig. 1-1).
n  ltrasound waves (Table 1-1) are mechanical
U o Amplitude (“loudness”) is described using the

vibrations with basic descriptors including: logarithmic decibel (dB) scale; a 6 dB change rep-
• 
Frequency (f, cycles per second = Hz, 1000 cycles per resents a doubling or halving of signal amplitude.
second = MHz) o Refraction of ultrasound can result in imag-
Propagation velocity (c, ∼1540 m/s in blood)
•  ing artifacts due to deflection of the ultrasound
• 
Wavelength (λ [in mm] = 1.54/f, where f is trans- beam from a straight path.
ducer frequency in MHz)
• 
Amplitude (decibels or dB)
n Ultrasound waves interact with tissues (Table 1-2) TRANSDUCERS
in four different ways: n  ltrasound transducers use a piezoelectric crystal
U
• 
Reflection (used to create ultrasound images) to alternately transmit and receive ultrasound sig-
• 
Scattering (the basis of Doppler ultrasound) nals (Fig. 1-2).

1
2 CHAPTER 1  Principles of Echocardiographic Image Acquisition and Doppler Analysis

TABLE 1-1 Ultrasound Waves


Definition Examples Clinical Implications
Frequency ( f ) The number of Transducer frequencies are Different transducer frequencies
cycles/s in an ­measured in MHz (1,000,000 are used for specific clinical ap-
ultrasound wave cycles/s). plications because the transmit-
f = cycles s = Hz (Eq 1-1) Doppler signal frequencies ted frequency affects ultrasound
are measured in kHz (1000 tissue penetration, image resolu-
cycles/s). tion, and the Doppler signal.
Velocity of The speed that The average velocity of ultra- The velocity of propagation is simi-
propagation (c) ultrasound travels through sound in soft tissue is about lar in different soft tissues (blood,
tissue 1540 m/s. myocardium, liver, fat, etc.) but
is much lower in lung tissue and
much higher in bone tissue.
Wavelength (λ) The distance Wavelength is shorter with a Image resolution is greatest (∼1
between ultrasound waves: higher frequency transducer mm) with a shorter wavelength
λ = c f = 1.54 f ( MHz) (Eq 1-2) and longer with a lower fre- (higher frequency).
quency transducer. Depth of tissue penetration is
greatest with a longer wavelength
(lower frequency).
Amplitude (dB) Height of the A log scale is used for dB; on A very wide range of amplitudes
ultrasound wave or the dB scale, 80 dB indicates can be displayed using a gray
“loudness” measured in a 10,000-fold increase in am- scale ­display for both imaging
decibels (dB) plitude and 40 dB indicates a and ­spectral Doppler.
100-fold increase in amplitude.

TABLE 1-2 Ultrasound Tissue Interaction


Definition Examples Clinical Implications
Acoustic A characteristic of each Lung has a low density and slow Ultrasound is reflected from
impedance tissue defined by tissue propagation velocity, whereas boundaries between tissues with
(Z) density (ρ) and propagation bone has a high density and fast differences in acoustic impedance
of velocity (c) as: propagation velocity. Soft tissues (e.g., blood vs. myocardium).
Z = ρ × c (Eq 1-3) have smaller differences in tissue
density and acoustic impedance.
Reflection Return of ultrasound signal Reflection is used to generate 2D Reflection is greatest when the
to the transducer from a cardiac images. ultrasound beam is perpendicular
smooth tissue boundary to the tissue interface.
Scattering Radiation of ultrasound in The change in frequency of The amplitude of scattered signals
multiple directions from a signals scattered from moving is 100 to 1000 times less than
small structure, such as blood cells is the basis of reflected signals.
blood cells Doppler ultrasound.
Refraction Deflection of ultrasound Refraction is used in transducer Refraction in tissues results
waves from a straight design to focus the ultrasound in double image artifacts.
path due to differences in beam.
acoustic impedance
Attenuation Loss in signal strength due Attenuation is frequency dependent A lower frequency transducer may be
to absorption of ultrasound with greater attenuation (less needed for apical views or in larger
energy by tissues penetration) at higher frequencies. patients on transthoracic imaging.
Resolution The smallest resolvable Resolution has three Axial resolution is most precise
distance between two dimensions—along the length (as small as 1 mm), so imaging
specular reflectors on an of the beam (axial), lateral measurements are best made
ultrasound image across the image (azimuthal), along the length of the ultrasound
and in the elevational plane. beam.
Principles of Echocardiographic Image Acquisition and Doppler Analysis  CHAPTER 1 3

3.5 MHz 6 MHz

LA

LV
RV

A B
Figure 1-1  The effect of transducer frequency on penetration and resolution. In this transesophageal 4-chamber view recorded at a transmitted
frequency of (A) 3.5 MHz and (B) 6 MHz, the higher frequency transducer provides better resolution—for example, the mitral leaflets (arrow) look thin, but the
depth of penetration of the signal is very poor so the apical half of the LV is not seen. With the lower frequency transducer, improved tissue penetration provides
a better image of the LV apex but image resolution is poorer, with the mitral leaflets looking thicker and less well defined.

n Transducers are configured for specific imag-



ing approaches—transthoracic, transesophageal,
intracardiac, and intravascular (Table 1-3).
n The basic characteristics of a transducer are: Transducers
• 
Transmission frequency (from 2.5 MHz for transtho-
racic to 20 MHz for intravascular ultrasound)
• 
Bandwidth (range of frequencies in the transmitted Scattering from
ultrasound pulse) moving blood cells
• 
Pulse repetition frequency (the number of transmis-
sion-receive cycles per second)
• 
Focal depth (depends on beam shape and focusing) Specular
Reflection reflector
• 
Aperture (size of the transducer face or “footprint”)
• 
Power output

v  KEY POINTS Refraction


Attenuation
o  he time delay between transmission of
T
an ultrasound burst and detection of the Figure 1-2  Diagram of the interaction between ultrasound and body
tissues. Doppler analysis is based on the scattering of ultrasound in all di-
reflected wave indicates the depth of the tissue rections from moving blood cells with a resulting change in frequency of the
reflector. ultrasound received at the transducer. 2D imaging is based on reflection of
o The pulse repetition frequency is an important ultrasound from tissue interfaces (specular reflectors). Attenuation limits the
factor in image resolution and frame rate. depth of ultrasound penetration. Refraction, a change in direction of the ultra-
o A shorter transmitted pulse length results in sound wave, results in imaging artifacts. (From Otto, CM: Textbook of Clinical
Echocardiography, ed 5, Elsevier, 2013, Philadelphia.)
improved depth (or axial) resolution.
o A wider bandwidth provides better resolution
of structures distant from the transducer. o  smaller aperture is associated with a wider
A
o The shape of the ultrasound beam depends beam width; however, the smaller “footprint”
on several complex factors. Each type of trans- may allow improved angulation of the beam in
ducer focuses the beam at a depth appropriate the intercostal spaces. This is most evident clini-
for the clinical application. Some transducers cally with a dedicated nonimaging continuous
allow adjustment of focal depth. wave (CW) Doppler transducer.
4 CHAPTER 1  Principles of Echocardiographic Image Acquisition and Doppler Analysis

TABLE 1-3 Ultrasound Transducers


Definition Examples Clinical Implications
Type Transducer characteristics Transthoracic (adult and pediatric) Each transducer type is optimized for
and configuration Nonimaging CW Doppler a specific clinical application.
Most cardiac transducers 3D echocardiography More than one transducer may be
use phased array of TEE needed for a full examination.
piezoelectric crystals Intracardiac
Transmission The central frequency Transducer frequencies vary from A higher frequency transducer
frequency emitted by the transducer 2.5 MHz for transthoracic echo provides improved resolution but
to 20 MHz for intravascular less penetration.
imaging. Doppler signals are optimal at a lower
transducer frequency than used for
imaging.
Power The amount of ultrasound An increase in transmitted power Excessive power output may result
output energy emitted by the increases the amplitude of the in bioeffects measured by the
transducer reflected ultrasound signals. mechanical and thermal indexes.
Bandwidth The range of frequencies Bandwidth is determined by A wider bandwidth allows improved
in the ultrasound pulse transducer design. axial resolution for structures distant
from the transducer.
Pulse The length of the A higher frequency signal can be A shorter pulse length improves axial
(or burst) transmitted ultrasound transmitted in a shortened pulse resolution.
length signal length compared with a lower
frequency signal.
Pulse The number of The PRF decreases as imaging (or Pulse repetition frequency affects
repetition transmission-receive Doppler) depth increases because image resolution and frame rate
frequency cycles per second of the time needed for the signal to (particularly with color Doppler)
(PRF) travel from and to the transducer.
Focal depth Beam shape and focusing Structures close to the transducer The length and site of a transducer’s
are used to optimize are best visualized with a short focal zone are primarily determined
ultrasound resolution at a focal depth, while distant struc- by transducer design but adjustment
specific distance from the tures are best visualized with a during the exam may be possible.
transducer long focal depth.
Aperture The surface of the A small nonimaging CW Doppler A larger aperture allows a more
transducer face where transducer allows optimal focused beam.
ultrasound is transmitted positioning and angulation of the A smaller aperture allows improved
and received ultrasound beam. transducer angulation on TTE imaging.

ULTRASOUND IMAGING v  KEY POINTS


o  -mode recordings allow identification of very
M
Basic Principles rapid intracardiac motion because the sampling
n The basic ultrasound imaging modalities are: rate is about 1800 times per second compared to
• 
M-mode—a graph of depth versus time a 2D frame rate of 30 frames per second (Fig. 1-3).
• 
2D—a sector scan in a tomographic image plane o Ultrasound imaging resolution is more precise
with real-time motion along the length of the ultrasound beam (axial
• 
3D—multiple simultaneous 2D images or a selected resolution) compared with lateral (side to side)
cutaway real-time image in a 3D display format (see or elevational (“thickness” of the image plane)
Chapter 4) resolution.
n System controls for 2D imaging typically include: o Lateral resolution decreases with increasing dis-
• 
Power output (transmitted ultrasound energy) tance from the transducer (Fig. 1-4).
• 
Gain (amplitude of the received signal) o Harmonic imaging improves endocardial defi-
• 
Time gain compensation (differential gain along the nition and reduces near-field and side-lobe arti-
ultrasound beam) facts (Fig. 1-5).
• 
Depth of the image (affects pulse repetition fre-
quency and frame rate) Imaging Artifacts
• 
Gray scale/dynamic range (degree of contrast in the n Common imaging artifacts result from:
images) • 
A low signal-to-noise ratio
3D/2D M-mode A-mode
Distance Time

Ao Ao

Depth
Ao

LA LA

Distance

Time

Figure 1-3  3D, 2D, M-mode, and A-mode recordings of aortic valve motion. This illustrations shows the relationship between the 3D and 2D long-axis
image of the aortic valve (left) which shows distance in both the vertical and horizontal directions, M-mode recording of aortic root (Ao), LA, and aortic valve
motion, which shows depth versus time (middle) and A-mode recording (right) which shows depth only (with motion seen on the video screen). Spatial relation-
ships are best shown with 3D/2D, but temporal resolution is higher with M-mode and A-mode imaging. (From Otto, CM: Textbook of Clinical Echocardiography,
ed 5, Elsevier, 2013, Philadelphia.)

LA

RA RV
LV

LV
RA
RV
LA

A B
Figure 1-4  Lateral resolution with ultrasound decreases with the distance of the reflector from the transducer. In this TEE image oriented with the
origin of the ultrasound signal at the top of the image (A), thin structures close to the transducer, such as the atrial septum (upper arrow), appear as a dot
because lateral resolution is optimal at this depth. Reflections from more distant structures, such as the ventricular septum (lower arrow), appear as a broad
line due to poor lateral resolution. When the image is oriented with the transducer at the bottom of the image (B), the effects of depth on lateral resolution are
more visually apparent. The standard orientation for echocardiography with the transducer as the top of the image is based on considerations of ultrasound
physics, not on cardiac anatomy.
6 CHAPTER 1  Principles of Echocardiographic Image Acquisition and Doppler Analysis

RVOT

Ao

Figure 1-5  Harmonic imaging compared with fundamental frequency


imaging. Harmonic imaging improves identification of the LV endocardial
border, as seen in this apical 4-chamber view recorded with a 4-MHz LA
transducer using (left) fundamental frequency imaging and (right) harmonic
imaging.

Figure 1-7  Refraction artifact. In this parasternal short-axis image of the


aortic valve (Ao), a refraction artifact results in the appearance of a “second”
aortic valve (arrows), partly overlapping with the actual position of the aortic
valve. RVOT, Right ventricular outflow tract.

LV
o  ignals originating from the edges of the ultra-
S
RV
sound beam or from side lobes can result in
MVR imaging or Doppler artifacts.
o Deviation of the ultrasound beam from a

straight pathway due to refraction in the tissue
results in the structure appearing in the incor-
rect location across the sector scan (Fig. 1-7).
RA o Ultrasound reflected back and forth between two
strong reflectors creates a reverberation artifact.
o Reflected ultrasound signals received at the

transducer are assumed to originate from the
preceding transmitted pulse. Signals from very
deep structures or signals that have been rere-
flected will be displayed at half or twice the
actual depth of origin.
Figure 1-6  Acoustic shadowing and reverberations. This apical 4-chamber
view in a patient with a mechanical mitral valve replacement (MVR) illustrates
the shadowing (dark area, small arrow) and reverberations (white band of
echoes, large arrow) that obscure structures (in this case, the left atrium)
DOPPLER
distal to the valve. n  oppler ultrasound is based on the principle that
D
ultrasound backscattered (FS) from moving red
• 
Acoustic shadowing blood cells will appear higher or lower in frequency
• 
Reverberations than the transmitted frequency (FT) depending on
• 
Beam width the speed and direction of blood flow (v) (Table 1-4).
• 
Lateral resolution n The Doppler equation is:
• 
Refraction v = c (FS −F T ) [2 FT (cos θ )](1-4)

n Range ambiguity
n Electronic processing n A
 ccurate blood flow measurements depend on a
parallel intercept angle (θ) between the ultrasound
v  KEY POINTS beam and direction of blood flow.
o  shadow occurs distal to a strong ultrasound
A n 
There are three basic Doppler modalities: pulsed
reflector because the ultrasound wave does not Doppler, color flow imaging, and continuous wave
penetrate past the reflector (Fig. 1-6). Doppler ultrasound.
Principles of Echocardiographic Image Acquisition and Doppler Analysis  CHAPTER 1 7

TABLE 1-4 Doppler Physics


Definition Examples Clinical Implications
Doppler effect The change in frequency A higher velocity corresponds to a Ultrasound systems display
of ultrasound scattered higher Doppler frequency shift, velo­city, which is calculated us-
from a moving target: ranging from 1 to 20 kHz for intra- ing the Doppler equation, based
v = c × ∆F [ 2 FT ( cosθ )] (Eq 1-5) cardiac flow velocities. on transducer frequency and
the Doppler shift, assuming cos
θ equals 1.
Intercept angle The angle (θ) between the When the ultrasound beam is par- Velocity is underestimated when
direction of blood flow and allel to the direction of blood flow the intercept angle is not parallel.
the ultrasound beam (0° or 180°), cos θ is 1 and can be This can lead to errors in he-
ignored in the Doppler equation. modynamic measurements.
CW Doppler Continuous ultrasound trans­ CW Doppler allows measurements CW Doppler is used to measure
mission with reception of of high velocity signals but does high velocities in valve stenosis
Doppler signals from the entire not localize the depth of origin of and regurgitation.
length of the ultrasound beam the signal.
Pulsed Doppler Pulsed ultrasound transmission Pulsed Doppler samples velocities Pulsed Doppler is used to record
with timing of reception from a specific site but can only low velocity signals at a spe-
determining depth of the measure velocity over a limited cific site, such as LV outflow
backscattered signal range. velocity or LV inflow velocity.
Pulse repetition The number of pulses The PRF is limited by the time The maximum velocity
frequency (PRF) transmitted per second needed for ultrasound to reach measurable with pulsed Doppler
and return from the depth of is about 1 m/s at 6 cm depth.
interest.
PRF determines the maximum
velo­city that can be unambigu-
ously measured.
Nyquist limit The maximum frequency shift The Nyquist limit is displayed as The greater the depth, the lower
(or velocity) measurable with the top and bottom of the velocity the maximum velocity
pulsed Doppler equal to half range with the baseline centered. measurable with pulsed
PRF Doppler.
Signal aliasing The phenomenon that the With aliasing of the LV outflow sig- Aliasing can result in inaccurate
direction of flow for frequency nal, the peak of the velocity curve velocity measurements if not
shifts greater than the Nyquist is “cut off” and appears as flow in recognized.
limit cannot be determined the opposite direction.
Sample volume The intracardiac location where Sample volume depth is determined Sample volume depth and length
the pulsed Doppler signal by the time interval between are adjusted to record the flow
originated transmission and reception. of interest.
Sample volume length is deter-
mined by the duration of the
receive cycle.
Spectral Method used to display Spectral analysis is used for both The velocity scale, baseline
analysis Doppler velocity data vs. pulsed and CW Doppler. position, and time scale of the
time, with gray scale spectral display are adjusted for
indicating amplitude each Doppler velocity signal.

v  KEY POINTS o  he standard Doppler velocity display (or spec-


T
o  he speed (c) of ultrasound in blood is about
T tral recording) shows time on the horizontal axis
1540 m/s. and velocity on the vertical axis with signal ampli-
o Blood flow velocity will be underestimated with tude displayed using a decibel gray scale (Fig. 1-8).
a nonparallel intercept angle; the error is only o Standard Doppler instrument controls are:
6% with an angle of 20° but increases to 50% Power
o output
at a 60° angle. Receiver
o gain (Fig. 1-9)
o When the ultrasound beam is perpendicular to High-pass (“wall”) filters (Fig. 1-10)
o
flow, there is no Doppler shift and blood flow is o Velocity range and baseline shift
not detected, even when present. o Postprocessing options
8 CHAPTER 1  Principles of Echocardiographic Image Acquisition and Doppler Analysis

40dB 2 •/+1/0/ 1 CW:2MHz CW:2MHz

PW Depth= 96mm Sample 1.0 1.0


PW Gate= 2.0mm volume
PW Gain= 7dB depth

Length m/s m/s

PW:2MHz

m/s

2.0 2.0
High filter Low filter

Figure 1-10  Wall filter settings. An aortic outflow signal is recorded with
CW Doppler with the high pass (“wall”) filter set at a high and low level. With
the higher filter, low velocity signals are eliminated as shown by the blank
Pulsed Doppler space adjacent to the baseline (arrow). This tracing enhances identification
1.5 of the maximum velocity and recognition of the valve closing click. At the
lower filter setting, the velocity signals extend to the baseline, making mea-
Figure 1-8  Doppler spectral tracing. LV outflow velocity was recorded surement of time intervals more accurate, but there also is more low velocity
with pulsed Doppler ultrasound from the apex. The sample volume depth noise in the signal, related to motion of cardiac structures.
(time for transmission and reception of the signal) is shown on a small 2D
image with the length (sampling duration) indicated by the pulsed wave (PW)
gate size. The spectral tracing shows time (horizontal axis), velocity (verti- Pulsed Doppler
cal axis), and signal strength (gray scale). The baseline has been shifted
upward to show the entire velocity curve directed away from the transducer. n Pulsed Doppler allows measurement of blood flow
Some diastolic LV inflow is seen above the baseline, directed toward the velocity at a specific intracardiac site.
transducer. n The depth of interrogation (or sample volume) is
determined by the time interval between transmis-
sion and sampling of the backscattered signal.
8:20:04 am
.69 50dB 1 • /+1/0/ 1 0° TE-V5M n Signal aliasing limits the maximum velocity mea-
PW Depth= 16mm 90mm
PW Gate= 2.0mm
7.0MHz
UWMC TEE surable with pulsed Doppler.
PW Gain= 13dB UWMC TEE /V

.69
Lens Temp=37.2°C
Store in progress
v  KEY POINTS
0:08:04
PW:3.5MHz HR= 66bpm
Sweep=100mm/s
o  pulse of ultrasound is transmitted and then
A
the backscattered signal is analyzed at a time
.40 interval corresponding to the transit time from
the depth of interest.
o The pulsed Doppler interrogation line and

sample volume are displayed on the 2D image,
m/s
with the transducer switched to Doppler only
during data recording.
.10
A Update
o Pulse repetition frequency is the number of
8:20:38 am
transmission/receive cycles per second, which
.69 50dB 1 • /+1/0/ 1
PW Depth= 18mm
0° TE-V5M
7.0MHz
7sec
90mm
is determined by the depth of the sample
PW Gate= 1.5mm UWMC TEE volume.
PW Gain= 7dB UWMC TEE /V
Store in progress
o The maximum frequency detectable with inter-
.69
HR= 60bpm
Tape Wait
mittent sampling is one half the pulse repetition
PW:3.5MHz Sweep=100mm/s
frequency (or Nyquist limit).
.40 o The direction of blood flow for frequencies in
excess of the Nyquist limit is ambiguous, a phe-
nomenon called signal aliasing (Fig. 1-11)
o The effective velocity range for pulsed Doppler
m/s
can be doubled by moving the baseline to the
edge of the spectral display.
.20
o The sample volume length can be adjusted to
B Run/Stop
localize the signal (short length) or improve sig-
Figure 1-9  Pulsed Doppler gain setting. The effect of Doppler gain set-
nal strength (long length).
tings are shown for a TEE recording of pulmonary vein inflow. Excess noise is o Pulsed Doppler is used to measure normal

eliminated; then the gain is decreased from 13 dB (A) to 7 dB (B). intracardiac transvalvular flow velocities.
Principles of Echocardiographic Image Acquisition and Doppler Analysis  CHAPTER 1 9

40dB 2 •/+1/0/ 1 o  ariations of the pulsed Doppler principle are


V
PW Depth= 89 mm used to generate color Doppler flow images and
PW Gate= 2.5mm tissue Doppler recordings.
PW Gain= 8dB Sample
volume Color Doppler
n Color Doppler uses the pulsed Doppler principle
PW:2MHz
.60 to generate a 2D image or “map” of blood flow
velocity superimposed on the 2D real-time image
(Table 1-5).
n Color Doppler signals, like all pulsed Doppler
velocity data, are angle dependent and are subject
m/s Baseline to signal aliasing.
n The frame rate for color Doppler imaging depends
on:
• 
Pulse repetition frequency (depth of color sector)
• 
Number of scan lines (width of color sector and scan
.60 Nyquist limit line density)
Figure 1-11  Signal aliasing. LV outflow velocity recorded from the apical • 
Number of pulses per scan line (affects accuracy of
approach with the sample volume on the LV side of the aortic valve. The mean velocity calculation)
spectral tracing is shown in the standard format with the baseline in the
center of the scale and the Nyquist limit at the top and bottom of the scale. v  KEY POINTS
Signal aliasing is present with the top of the LV outflow signal seen in the
reverse channel (arrows). This degree of aliasing is easily resolved by shift- o  olor Doppler is recorded in real-time simulta-
C
ing the baseline, as seen in Figure 1-7. Aliasing with higher velocity flow is neous with 2D imaging.
best resolved using CW Doppler ultrasound.
o F low toward the transducer typically is shown
in red with flow directed away from the trans-
ducer in blue (Fig. 1-12).

TABLE 1-5 Color Doppler Flow Imaging


Definition Examples Clinical Implications
Sampling line Doppler data are displayed Instead of sampling b­ ackscattered A greater number of ­sampling
from multiple sampling signals from one depth (as in lines results in denser
lines across the 2D image pulsed Doppler), signals from ­ ­Doppler data but a slower
multiple depths along the beam frame rate.
are analyzed.
Burst length The number of ultrasound Mean velocity is estimated from A greater number of bursts
bursts along each the average of the backscattered results in more accurate
sampling line signals from each burst. mean velocity estimates but
a slower frame rate.
Sector scan The width of the displayed A greater sector width requires A narrower sector scan ­allows
width 2D and color image more sampling lines or less a greater sampling line
dense velocity data. ­density and faster frame rate.
Sector scan The depth of the displayed The maximum depth of the ­ The minimum depth needed
depth color Doppler image sector scan determines PRF to display the flow of ­interest
(as with pulsed Doppler) and the ­provides the optimal color
Nyquist limit. display.
Color scale The color display of Doppler Most systems use shades of red The color scale can be adjust-
velocity and flow direction for flow toward the transducer ed by shifting the baseline
and blue for flow away from the and adjusting the maximum
transducer. velocity displayed (within the
Nyquist limit).
Variance The degree of variability in Variance typically is displayed as A variance display highlights
the mean velocity estimate a green scale superimposed flow disturbances and high
at each depth along a on the red–blue velocity scale. velocity flow, but even n­ ormal
sampling line Variance can be turned on or off. flows will be d
­ isplayed as
showing v­ ariance if velocity
exceeds the Nyquist limit.
10 CHAPTER 1  Principles of Echocardiographic Image Acquisition and Doppler Analysis

.69 CW:2MHz APX AV


8.0 1 second
toward
Zero baseline
LA
away v
e
AR
.69 Nyquist limit l
o
c
i
t
RA
y

m/s

Figure 1-12  Color Doppler flow mapping. In this TEE image of an atrial
AS
septal defect, the Doppler signal is superimposed on the 2D image using
a color scale for flow toward the transducer in red and flow away from 4.0
the transducer in blue. The color density indicates velocity as shown by the
scale. This scale includes variance as the addition of green to the color scale.
The flow (arrow) from the left atrium (LA) to the right atrium (RA) across the Figure 1-13  CW Doppler recording. The spectral recording of antegrade
septal defect should be blue (away from the transducer) but has aliased to (aortic stenosis, AS) and retrograde flow (aortic regurgitation, AR) across the
red because the velocity exceeds the Nyquist limit of 60 cm/s. aortic valve shows time (horizontal axis in seconds), velocity (vertical axis in
m/s), and signal strength (gray scale). High velocity flow can be measured
without aliasing using CW Doppler as shown in the aortic regurgitant velocity
o  hen velocity exceeds the Nyquist limit, signal
W over 4 m/s in this example.
aliasing occurs so that faster flows toward the
transducer alias from red to blue and vice versa
for flow away from the transducer. o  he CW Doppler signal is recorded as a
T
o The amount of variation in the velocity signal spectral tracing, with the scale and baseline
from each site can be coded on the color scale as adjusted as needed to display the signal of
variance. interest.
o Variance reflects either signal aliasing (high
 o CW Doppler can be recorded with a standard
velocity flow) or the presence of multiple flow transducer with the CW interrogation line
velocities or directions (flow disturbance). shown on the 2D image; however, a dedicated
o Color Doppler is most useful for visualization nonimaging CW transducer is optimal due to a
of spatial flow patterns; for this purpose, exam- higher signal-to-noise ratio and better angula-
iner preference determines the most appropriate tion with a smaller transducer.
color scale. o The lack of range resolution means that the ori-
o For color Doppler measurements, such as vena gin of the CW signal must be inferred from:
contracta width or proximal isovelocity surface Characteristics
o of the signal itself (timing, shape,
area (PISA) measurements, a color scale with- and associated flow signals)
out variance is optimal. Associated 2D imaging and pulsed or color Dop-
o
o The maximum velocity measurable with color pler findings
Doppler is determined by the Nyquist limit, but o  nderestimation of blood flow velocity occurs
U
the baseline can be shifted or the velocity scale when the CW Doppler beam is not parallel to
can be reduced. the flow of interest.
Continuous Wave Doppler Doppler Artifacts
n CW Doppler uses two ultrasound crystals to con- n 
Artifacts with pulsed or CW Doppler spectral
tinuously transmit and receive ultrasound signals. recordings include:
n CW Doppler allows accurate measurement of • 
Underestimation of velocity because of a nonparallel
high flow velocities without signal aliasing. intercept angle
n Signals from the entire length of the ultrasound beam • 
Signal aliasing (with pulsed Doppler)
are included in the spectral CW Doppler recording. • 
Range ambiguity
• 
Beam width artifacts with superimposition of mul-
v  KEY POINTS tiple flow signals
o  W Doppler is used to measure high velocity
C • 
Mirror image artifact (Fig. 1-14)
flows—for example, across stenotic and regur- • 
Transit time effect
gitant valves (Fig. 1-13). • 
Electronic interference
Principles of Echocardiographic Image Acquisition and Doppler Analysis  CHAPTER 1 11

n Artifacts with color Doppler flow imaging (Table n  ignal aliasing (Fig. 1-15)
S
1-6) include: n Electronic interference
n Shadowing resulting in inability to detect flow
abnormalities v  KEY POINTS
n Ghosting from strong reflectors leading to o  he potential for underestimation of velocity is
T
flashes of color across the image plane the most important clinical limitation of Dop-
n Gain too low (loss of true signal) or gain too pler ultrasound.
high (speckle pattern across the image) o Signal aliasing limits measurement of high

n Intercept angle with absence of detectable flow velocities with pulsed Doppler and may confuse
at 90° angle interpretation of color Doppler images.
o Range ambiguity with CW Doppler is obvious.
With pulsed Doppler, range ambiguity occurs
CW:2MHz APEX TV when signals from two times, three times, or

2.0

.74 CW:2MHz 3V2c–S #182


H4.0MHz R3mm
UWMC
UWMC /V
T1/–1/ 0/VV:3
m/s 1/2 CD:2.0MHz
CD Gain = 45
LV
.74 Store in progress
HR= 66bpm

Ao

4.0
Figure 1-14  Doppler artifacts. Appropriate use of instrumentation allows
minimization of many ultrasound artifacts. This recording of the tricuspid
regurgitant jet velocity shows marked channel cross-talk (signal below the Figure 1-15  Color Doppler signal aliasing. In this apical view angulated
baseline that does not correlate with an actual intracardiac flow) from the anteriorly to visualize the aorta, the antegrade flow in the LV outflow tract
diastolic signal across the tricuspid valve. This recording would be improved aliased from blue to orange because the velocity exceeds the Nyquist limit of
by a higher wall filter and lower gain setting. 74 cm/s. Variance is seen because of signal aliasing.

TABLE 1-6 Ultrasound Terminology: Ultrasound Safety


Definition Examples Clinical Implications
Exposure Ultrasound exposure Common measures of intensity are Transducer output and tissue
Intensity (I) depends on power and area: the spatial peak temporal average exposure affect the total ultra-
I = power area = watt cm2 (Eq 1-6) (SPTA) or the spatial peak pulse sound exposure of the patient.
average (SPPA).
Thermal Heating of tissue due to The degree of tissue heating is Total ultrasound exposure
bioeffects absorption of ultrasound affected by tissue density and depends on transducer fre-
energy described by the blood flow. quency, power output, focus,
thermal index (TI) TI is the ratio of transmitted acous- depth, and exam duration.
tic power to the power needed to When the TI exceeds 1, the
increase temperature by 1°  C. benefits of the study should
TI is most important with Doppler be balanced against potential
and color flow imaging. biologic effects.
Cavitation Creation or vibration of small gas- Mechanical index (MI) is the ratio Cavitation or vibration of
filled bodies by the of peak rarefactional pressure to microbubbles occurs with
ultrasound wave the square root of the transducer higher intensity exposure.
frequency. Power output and exposure
MI is most important with 2D imaging. time should be monitored.
12 CHAPTER 1  Principles of Echocardiographic Image Acquisition and Doppler Analysis

more of the depth of the sample volume return • 


Cavitation (the creation or vibration of small gas-
to the transducer during a receive cycle. filled bodies)
o A mirror image artifact is common on spectral n Ultrasound exposure is measured by the:
tracings and may be reduced by lowering power • 
Thermal index (TI, the ratio of transmitted acoustic
output and gain. power to the power needed to increase temperature
o As ultrasound propagates through moving
 by 1° C)
blood, there is a slight change in ultrasound fre- • 
Mechanical index (MI, the ratio of peak rarefac-
quency, called the transit time effect. The transit tional pressure to the square root of transducer
time effect results in slight blurring of the edge frequency)
of the CW Doppler signal, particularly for high
velocity flows. v  KEY POINTS
o Acoustic shadowing can be avoided by using o  he degree of tissue heating depends on the
T
an alternate transducer position—for example, ultrasound energy imparted to the tissue and
transesophageal imaging of a mitral prosthetic on characteristics of the tissue, including tissue
valve. density and blood flow.
o Color ghosting is seen in only one or two frames o The total ultrasound exposure depends on trans-
of the cardiac cycle, whereas blood flow signals ducer frequency, focus, power output, and depth,
demonstrate physiologic timing. as well as the duration of the examination.
o Cavitation or vibration of microbubbles occurs
with higher intensity ultrasound exposure.
BIOEFFECTS AND SAFETY o When the TI or MI exceeds 1, the benefit of
n  wo types of ultrasound bioeffects are important
T the ultrasound examination should be balanced
with diagnostic imaging: against potential biologic effects.
• 
Thermal (heating of tissue due to the interaction of o Power output and exposure time should be moni-
ultrasound energy with tissue) tored during the echocardiographic examination.
Principles of Echocardiographic Image Acquisition and Doppler Analysis  CHAPTER 1 13

THE ECHO EXAM

Basic Principles
Optimization of Echocardiographic Images
Instrument Control Data Optimization Clinical Issues
Transducer • Different transducer types and transmission • A higher transducer frequency provides
­frequencies are needed for specific clinical improved resolution but less penetration.
applications. • A larger aperture provides a more focused
• Transmission frequency is adjusted for tissue beam.
­penetration in each patient and for ultrasound
­modality (Doppler vs. imaging).
Power output • Power output reflects the amount of • Potential bioeffects must be considered.
ultrasound energy transmitted to the tissue. • Exam time and mechanical and thermal
• Higher power output results in greater tissue indexes should be monitored.
­penetration.
Imaging mode • 2D imaging is the clinical standard for most • Optimal measurement of cardiac chambers
­indications. and vessels may require a combination of
• M-mode provides high time resolution imaging modes.
along a single scan line.
• 3D imaging provides appreciation of spatial
­relationships.
Transducer ­ • Acoustic windows allow ultrasound tissue • Optimal patient positioning is essential for
position ­penetration without intervening lung or acoustic access to the heart.
bone tissue. • Imaging resolution is optimal when the
• Transthoracic acoustic windows include ultrasound beam is reflected perpendicular
­parasternal, apical subcostal, and to the tissue interface.
suprasternal. • Doppler signals are optimal when the
• TEE acoustic windows include high ­ultrasound beam is aligned parallel to flow.
esophageal and transgastric.
Depth • Depth is adjusted to show the structure • PRF is higher at shallow depths, which
of interest. ­contributes to improved image resolution.
• PRF depends on ­maximum image depth. • Axial resolution is the same along the entire
length of the ultrasound beam.
• Lateral and elevations resolutions depend on
the 3D shape of the ultrasound beam at each
depth.
Sector width • Standard sector width is 60° but a • Sector width should be adjusted as needed
narrower sector allows a higher scan line to optimize the image.
density and faster frame rate. • Too narrow a sector may miss important
anatomic or Doppler findings.
Gain • Overall gain affects the display of the • Excessive gain obscures border
reflected ultrasound signals. ­identification.
• Inadequate gain results in failure to display
reflections from tissue interfaces.
TGC • TGC adjusts gain differentially along the length • An appropriate TGC curve results in an
of the ultrasound bean to compensate for the ­image with similar brightness proximally
effects of attenuation. and distally in the sector image.
Gray scale/ • Ultrasound amplitude is displayed using • The range of displayed amplitudes is
­dynamic range a decibel scale in shades of gray. adjusted to optimize the image using the
dynamic range or compression controls.
Harmonic imaging • Harmonic frequencies are proportional to • Harmonic imaging improves endocardial
the strength of the fundament frequency but definition and decreased near field and side
­increase with depth of propagation. lobe artifacts.
• Flat structures, such as valves, appear
thicker with harmonic imaging than with
fundamental imaging.
• Axial resolution is reduced.

Continued
14 CHAPTER 1  Principles of Echocardiographic Image Acquisition and Doppler Analysis

Optimization of Echocardiographic Images—cont’d


Instrument Control Data Optimization Clinical Issues
Focal depth • Transducer design parameters that affect • The ultrasound beam is most focused at
focal depth include array pattern, aperture the junction between the near zone and far
size, and acoustic focusing. field of the beam pattern.
• Transducer design allows a longer focal
zone. In some cases, focal zone can be
adjusted during the examination.
Zoom mode • The ultrasound image can be restricted to • Zoom mode is used to examine areas of
a smaller depth range and narrower section. interest identified on standard views.
The maximum depth still determines PRF,
but scan line density and frame rate can be
­optimized in the region of interest.
ECG • The ECG signal is essential for triggering • A noisy signal or low amplitude ECG results
digital cine loop acquisition. in incorrect triggering or inadvertent record-
ing of an incomplete cardiac cycle.

ECG, Electrocardiogram; PRF, pulse repetition frequency; TGC, time gain compensation.

Optimization of Doppler Recordings


Modality Data Optimization Common Artifacts
Pulsed • 2D guided with “frozen” image • Nonparallel angle with under-
• Parallel to flow estimation of velocity
• Small sample volume • Signal aliasing. Nyquist limit =
• Velocity scale at Nyquist limit half pulse repetition frequency
• Adjust baseline for aliasing (PRF)
• Use low wall filters • Signal strength/noise
• Adjust gain and dynamic range
Continuous wave • Dedicated nonimaging transducer • Nonparallel angle with under-
• Parallel to flow estimation of velocity
• Adjust velocity scale so flow fits and fills displayed range • Range ambiguity
• Use high wall filters • Beam width
• Adjust gain and dynamic range • Transit time effect
Color flow • Use minimal depth and sector width for flow of interest • Shadowing
(best frame rate) • Ghosting
• Adjust gain just below random noise • Electronic interference
• Color scale at Nyquist limit
• Decrease 2D gain to optimize Doppler signal
Principles of Echocardiographic Image Acquisition and Doppler Analysis  CHAPTER 1 15

SELF-ASSESSMENT QUESTIONS
Questions 1-5 Question 3:
Which ultrasound imaging interaction best describes
the findings in each of the images below:
A. Reverberation
B. Ring-down
C. Scattering
D. Refraction
E. Attenuation
  

Question 1:

Figure 1-18  Tricuspid regurgitant jet.

Question 4:

Figure 1-16  Parasternal long-axis view.

Question 2:

Figure 1-19  Parasternal long-axis view.

Question 5:

Figure 1-17  High esophageal window, TEE imaging.

Figure 1-20  Apical 4-chamber view.


16 CHAPTER 1  Principles of Echocardiographic Image Acquisition and Doppler Analysis

Question 6 Question 10
M-mode imaging compared to 2D scanned imaging Early closure of the aortic valve in hypertrophic car-
has the greatest favorable effect on: diomyopathy is best demonstrated by which of the
A. Temporal resolution following imaging modalities:



B. Lateral resolution A. Color Doppler






C. Axial resolution B. Three-dimensional






C. M-Mode imaging
Question 7




D. Pulsed Doppler

/



An important feature of the continuous wave nonim- E. Two-dimensional




9
aging transducer is:
A. Signal localization
Question 11

9



B. Spatial resolution The black signal seen on the parasternal long-axis

r



C. Narrow beam width view shown (Fig. 1-22) is best explained by:

i



D. Small aperture size A. Acoustic shadowing







B. Intercept angle

h
Question 8




C. Electronic interference




D. Reverberations

a
Which of the following most affects frame rate during



E. Refraction of the ultrasound beam

t
color Doppler imaging?



r/
A. Burst length



B. Color scale



C. Electronic interference

e



D. Variance setting



E. Intercept angle

s



Question 9

/r u
In this color Doppler image of the aortic arch (Fig.
1-21), the interposed black region between the red
and blue color Doppler shift is the result of:

.t c
LV
A. Acoustic shadowing



B. Intercept angle Ao



C. Electronic interference



D. Signal aliasing

a



E. Flow disruption

k



/: /
LA

s
tt p
Ao
Figure 1-22

h
Question 12
Which of the following is least affected by increasing
width of the 2D scanning sector?
A. Temporal resolution



B. Spatial resolution



C. Axial resolution



Figure 1-21

Principles of Echocardiographic Image Acquisition and Doppler Analysis  CHAPTER 1 17

Questions 13-17 Question 14: Vena contracta for mitral valve regur-
Select the Doppler modality that offers the best diag- gitation severity assessment
nostic data for Questions 13 to 17: Question 15: Pulmonary venous flow reversal for
A. Color Doppler imaging mitral regurgitation severity assessment



B. Pulsed Doppler imaging Question 16: Velocity of the aortic jet in a patient



C. Continuous wave Doppler imaging with severe aortic stenosis



Question 13: Myocardial velocity for evaluation of Question 17: Tricuspid regurgitation velocity in a
LV diastolic function patient with pulmonary hypertension

9/
i r 9
a h
r/ t
s e
/r u
.t c
k a
/: /
s
tt p
h
18 CHAPTER 1  Principles of Echocardiographic Image Acquisition and Doppler Analysis

ANSWERS

Answer 1: D Answer 6: A
Relative differences in acoustic impedance of tissue With M-mode imaging, the transducer sends and
in the pathway of an ultrasound beam lead to partial receives an ultrasound signal along a single scan line,
deflection of ultrasound waves from a straight path. and the time needed to sweep the ultrasound beam
This leads to image generation to the side of the main across a sector is not incurred. Thus, the transmit

/
beam, creating a double image. In the image shown, time/receive cycle is very rapid (about 1800 times
there is an aortic valve artifact, giving the appearance per second) , which improves the temporal resolution

9
of a second aortic valve adjacent to the aortic valve of the image, allowing evaluation of rapidly moving
in the image. structures, such as valve leaflets, dissection flaps, and

9
valve vegetations.

r
Answer 2: C

i
2D ultrasound images are generated from specular Answer 7: D

h
reflection. Reflection of an ultrasound signal is stron- The continuous wave nonimaging transducer has
gest when reflectors are perpendicular to the ultra- a smaller transducer footprint. The resulting small

a
sound beam. Reverberation occurs if strong specular aperture size leads to a wider beam width distal in the

t
reflectors reflect off a portion of the ultrasound sig- far field. The smaller size allows for facilitated manip-

r/
nal between reflectors before returning to the trans- ulation of the transducer between intercostal spaces,
ducer. The received delayed signal is assigned to the which can be useful when optimizing alignment of
original transmitted pulse, leading to display of the the ultrasound beam with blood flow—for example,

e
signal at half or twice the actual depth of the origin. when obtained maximal aortic jet velocity in aor-

s
In this case, reverberation between the two walls of tic stenosis assessment. As with all continuous wave
the ascending aorta lead to an artifact in the ascend- imaging transducers, velocities are sampled along the

/r u
ing aorta which mimics an aortic dissection. length of the ultrasound beam and, therefore, spatial
resolution is poor across the imaging sector and spe-
Answer 3: A cific velocities along the length of the beam cannot
Doppler ultrasound sends a signal that is reflected be localized.

.t c
and backscattered off of small structures, such as
red blood cells. The reflected backscatter frequen- Answer 8: A
cies will be dependent on the speed and direction Burst length represents the number of ultrasound

a
of blood flow. Based on the Doppler equation, bursts transmitted along each sample line in the imag-
higher Doppler frequency shifts are associated with ing sector. A higher number of bursts increases the

k
a higher velocity recording. Peak tricuspid regur- accuracy of mean velocity assessment, but, because

/: /
gitant jet velocity is the highest recorded velocity of increased data acquisition, results in a slower frame
from the Doppler envelope. The highest velocity is rate. Color scale shows the direction and velocity of
recorded when the ultrasound beam is parallel with flow. Shifting the color scale increases identification

s
flow. of lower velocity flow as long as the maximum veloc-
ity is within the Nyquist limit. Beyond the Nyquist

tt p
Answer 4: E limit, there is aliasing of the signal. Electronic inter-
Attenuation describes the loss in ultrasound sig- ference affects image quality and, therefore, resolu-
nal strength due to tissue absorption of ultrasound tion of velocities within the color sector, but does not
energy, resulting in poor image quality in the far field. affect frame rate. Variance displays flow turbulence in

h
Use of a lower frequency transducer aids tissue pen- the color sector and is superimposed on the standard
etration, improving image quality, but will relatively color velocity scale. While variance can be toggled
lower image resolution in the near field. on/off and affects the display of flow turbulence, the
color frame rate is not affected. With Doppler imag-
Answer 5: B ing, the signal is optimized when the ultrasound beam
High amplitude oscillations of the piezoelectric crys- is parallel with flow. If flow is perpendicular to the
tal elements may create an acoustic noise artifact, ultrasound beam (90° intercept angle), this is dis-
termed ring-down artifact, or near field clutter, which limits played as absence of color, or no detectable flow, but
image resolution within 1 to 2 cm of the transducer. does not affect imaging frame rate.
This is commonly seen in apical view images of the
left ventricle, with an artifact in the LV apex. Multiple Answer 9: B
views of the LV are needed to correctly identify the Color Doppler imaging samples blood velocity mov-
artifact. ing towards (displayed as red) or away (displayed
as blue) from the transducer. In this image, flow up
Principles of Echocardiographic Image Acquisition and Doppler Analysis  CHAPTER 1 19

towards the ascending aorta and through the aortic echodensity at the site of calcium with shadowing of
arch is shown. Maximal velocities are obtained when the signal in the distal field. On 2D imaging a parallel
flow is parallel with the transducer. Flow perpen- intercept angle between the structure of interest and
dicular to the transducer, in this case the interposed the ultrasound beam results in image “drop out” as
black region, is recorded as an absent signal. Thus, few signals are reflected from the anatomic structure.
this black region is due to a perpendicular intercept Electronic interference typically has a geometric pat-
angle in this image. Acoustic shadowing occurs when tern and affects the entire 2D image. Reverberations
a strong specular reflector, such as prosthetic valves appear as multiple bright echo-densities distal to the

/
or calcium, blocks ultrasound penetration distal to anatomic structure whereas refraction results in the
the reflector. Electronic interference is displayed as structure of interest appearing lateral to the actual

9
an overlaying artifact which is not associated with location.
the image and may extend beyond tissue borders.

9
Signal aliasing results in flow being displayed as if Answer 12: C

i r
it were due to flow opposite in direction to actual With scanned (2D) imaging, the image sector is
flow. So, flow towards the transducer, by conven- formed by multiple adjacent scan lines where the

h
tion shown in red, would be displayed as blue, and transducer sweeps the ultrasound beam across the
vice versa. Signal aliasing often is seen on subcostal imaging field. Rapid image processing allows for

a
images of the proximal abdominal aorta. However, real-time imaging. Widening the scanning sector

t
in this image flow in the ascending aorta is correctly allows for improved spatial resolution across the

r/
shown towards the transducer in red and flow down imaging field. Because time is incurred sweeping
the descending aorta away from the transducer in the ultrasound beam across the imaging field with
blue. (Notice the right pulmonary artery seen as a 2D imaging, temporal resolution is optimal with

e
blue circle under the arch.) Disruption of flow would M-mode imaging which images only along a single

s
be accompanied by turbulent and disarrayed flow scan line. A wider 2D scanning sector decreases
with aliasing of the color Doppler signal at the point imaging frame rate and decreases temporal reso-

/r u
of disruption, which is not seen on this image of a lution. Axial resolution (longitudinal resolution) is
normal aortic arch. resolution in the direction parallel to the ultrasound
beam. Because resolution is the same at any point
Answer 10: C along an ultrasound beam, axial resolution is not

.t c
M-Mode displays signals obtained along the ultra- affected by scanning sector width.
sound beam. Because M-mode imaging does not
sweep through the imaging sector, the frame rate Answers 13-17

a
is much higher than in 2D or 3D imaging. The Answer 13: B
sampling rate with M-mode is approximately 1800 Answer 14: A

k
times per second compared to approximately 30 Answer 15: B

/: /
to 50 frames per second for 2D imaging; 3D imag- Answer 16: C
ing, which acquires a pyramidal volume data set, is Answer 17: C
even lower. The much higher frame rate seen with Continuous wave (CW) Doppler imaging allows

s
M-mode imaging allows for significantly increased accurate measurement of high velocity flow with-
temporal resolution and discrimination, and would out aliasing of the signal. Clinically, CW Doppler is

tt p
be helpful to identify early aortic valve closure in used whenever a high velocity signal is present—for
hypertrophic cardiomyopathy patients. Color Dop- example, with aortic stenosis, tricuspid regurgitation,
pler imaging does not allow easily for assessment mitral regurgitation, or a ventricular septal defect.
of aortic valve motion. Velocities across the aortic However, with CW Doppler, sampling occurs along

h
valve would be too high to discriminate with pulsed the line of interrogation without localization of the
Doppler imaging. Aliasing of the signal and artifact point of maximum velocity along that line (lack of
would not allow definitive assessment of aortic out- range resolution).
flow timing. The origin of the high velocity signal is inferred
from imaging data or localized using pulsed Doppler
Answer 11: A or color flow imaging.
This 2D image from the parasternal long-axis view Color Doppler imaging is useful for evaluating the
shows moderate mitral annular calcification at the spatial distribution of flow, which is especially helpful
posterior mitral valve annulus, just at the base of in determining the severity and mechanism of regur-
the posterior mitral valve leaflet. Calcium is a strong gitant flow. The width of the color Doppler regurgi-
specular reflector which blocks ultrasound pen- tant jet, the vena contracta, is a reliable measure of
etration distally. Most of the transmitted ultrasound regurgitation severity.
beam reflects from the calcium back to the transducer. Pulsed wave Doppler imaging allows spatial local-
This is shown on the generated image as a bright ization of a velocity signal but is best used for low
20 CHAPTER 1  Principles of Echocardiographic Image Acquisition and Doppler Analysis

velocity signals with a maximum velocity that is below which precludes accurate velocity measurements. Con-
the Nyquist level. Clinical examples of the use of ventional Doppler imaging assesses blood flow velocity
pulsed Doppler include LV inflow across the mitral by measuring signals from moving blood cells. With
valve, pulmonary venous flow, and LV outflow velocity myocardial tissue Doppler imaging, as is used for LV
proximal to the aortic valve (even when aortic steno- diastolic function assessment, pulsed Doppler is used
sis is present). With velocities that exceed the Nyquist to quantify the lower velocity signals of myocardial tis-
limit, aliasing of the pulsed wave Doppler signal occurs, sue motion.

9/
i r 9
a h
r/ t
s e
/r u
.t c
k a
/: /
s
tt p
h
Normal Anatomy and Flow Patterns
2 on Transthoracic Echocardiography
STEP-BY-STEP APPROACH Apical Window
Clinical Data Imaging 4-Chamber, 2-Chamber,

­
Patient Positioning and Long-Axis Views
Instrumentation Principles Doppler Data

9
Data Recording Subcostal Window
Examination Sequence Suprasternal Window

9
Parasternal Window The Echo Report
Long-Axis View Aging Changes on Echocardiography

i r
Right Ventricular Inflow View THE ECHO EXAM
Right Ventricular Outflow View SELF-ASSESSMENT QUESTIOINS
Short-Axis View

a h
t
STEP-BY-STEP APPROACH

r/

o Knowledge of clinical data ensures that the echo



study includes all the pertinent images and Dop­
Step 1: Clinical Data pler data. For example, when a systolic murmur

e
n The indication for the study determines the focus is present, the echo study includes data address­



of the examination. ing all the possible causes for this finding.

s
n Key clinical history and physical examination find­ o Data from previous imaging studies may identify






ings and results of any previous cardiac imaging specific areas of concern (e.g., a pericardial effu­

/r u
studies are noted. sion on chest computed tomography imaging).
o Detailed information about previous cardiac
v KEY POINTS



procedures assists in interpretation of postoper­

.t c
o The goal of the echo study is to answer the ative findings, evaluation of implanted devices



specific question asked by the referring provider. (such as prosthetic valves or percutaneous clo­
o Blood pressure is recorded at the time of the sure devices), and detection of complications.



echo because many measurements vary with Use of precise anatomic terminology aids accu­

a
o



loading conditions. rate communication of imaging results (Table 2 1).

-
/: / k
TABLE 2-1 Terminology for Normal Echocardiographic Anatomy
 
Aorta* Sinuses of Valsalva Ascending aorta Aortic arch
Sinotubular junction Descending thoracic aorta Proximal abdominal aorta

s
Coronary ostia
Aortic valve Right, left, and noncoronary cusps Nodules of Arantius Lambl excrescence

tt p
Mitral valve Scallops of posterior leaflet Anterior and posterior leaflets Chordae (primary, secondary,
(lateral, central, medial) Commissures (medial and lateral) tertiary, basal, and marginal)
Left ventricle Wall segments (see Chapter 8) Base, apex

h
Septum, free wall Medial and lateral papillary muscles
Right ventricle Inflow segment Outflow tract (conus) Anterior, posterior, and conus
­
Moderator band Supraventricular crest papillary muscles
­
Tricuspid value Anterior, septal, and posterior leaflets Chordae Commissures
Right atrium RA appendage Coronary sinus ostium Fossa ovalis
SVC and IVC junctions Crista terminalis Patent foramen ovale
Valve of IVC (Chiari network)
Left atrium LA appendage Superior and inferior right Ridge at junction of LA
Superior and inferior left pulmonary pulmonary veins appendage and left
veins superior pulmonary vein
­
Pericardium Oblique sinus Transverse sinus

IVC, Inferior vena cava; SVC, superior vena cava.


*The term aortic root is used inconsistently, sometimes meaning the aortic sinuses and sometimes meaning the entire segment of the aorta from the annulus to the arch
(including sinuses and ascending aorta).
22 CHAPTER 2  Normal Anatomy and Flow Patterns on Transthoracic Echocardiography

Step 2: Patient Positioning (Fig. 2-1)


n A steep left lateral position provides acoustic access




for parasternal and apical views.
n The subcostal views are obtained when the patient




is supine; if needed, the legs are bent to relax the
abdominal wall.
n Suprasternal notch views are obtained when the

/



patient is supine with the head turned toward
either side.

9
v KEY POINTS

9

Images may be improved with suspended res­

r
o




i
piration, typically at end expiration, but some­

-
times at other phases of the respiratory cycle.

h
o An examination bed with an apical cutout allows




a steeper left lateral position, often providing

a
improved acoustic access for apical views.

t
Imaging can be performed with either hand

r/

o



holding the transducer and with the examiner
on either side of the patient. However, imaging
Figure 2-1 Patient positioning for transthoracic echocardiography.
from the patient’s left side avoids reaching over

e

The patient is positioned in a steep left lateral decubitus position on an
examination bed with a removable section cut out of the mattress to allow the patient and is essential for apical views when

s
placement of the transducer on the apex by the sonographer as shown. the patient’s girth is larger than the arm span of
Ultrasound gel is used to enhance coupling between the transducer face the examiner.

/r u
and the patient’s skin. The sonographer sits on an adjustable chair and
uses the left hand for scanning and the right hand to adjust the instrument
o Prolonged or repetitive imaging requires the



panel. The room is darkened to improve visualization on the ultrasound examiner to learn ergonomic approaches to
instrument display screen. minimize mechanical stress and avoid injury.

.t c
Step 3: Instrumentation Principles (Fig. 2-2)
n A higher transducer frequency provides improved



resolution but less penetration of the ultrasound

a
signal.

/: / k
Patient Power
information output
Alpha numeric Physiologic inputs
keyboard
Transducer Selection ECG
Respiration
size and position

s
TTE TTE
TEE CWD
3 MHz 5 MHz

tt p
IMAGING DOPPLER

M-mode 2D 3D Harmonic
Common controls Pulsed CW Color

h
(used for all modalities)

Depth TGC
Doppler Baseline
Trackball scale
Sector width

Focal High pass Color


depth filter variance
Gain Record
Dynamic Dynamic Color
range range scale

Post- Measurements and Spectral Sample


processing display
calculations sweep speed
volume size

Figure 2-2 Schematic diagram illustrating the typical features of a simplified echocardiographic instrument panel. Many instrument controls affect

different parameters depending on the imaging modality. For example, the trackball is used to adjust the position of the M-mode and Doppler beams, sample
volume depth, and the size and position of the color Doppler box. The trackball also may be used to adjust 2D image depth and sector width and the position
of the zoom box. The gain control adjusts gain for each modality, imaging, pulsed, or continuous wave Doppler. Only a simplified model of an instrument panel
is shown. The transducer choices are examples; other transducers are available depending on the system. In addition to the time gain compensation (TGC)
controls, a lateral control scale may also be present.
Normal Anatomy and Flow Patterns on Transthoracic Echocardiography  CHAPTER 2 23

9/
Figure 2-3 Parasternal long-axis view.


A, Initially images are recorded at a depth

9
of 18 cm to show the structures posterior
(P) to the heart (top). B, Next, images are

r
recorded with the depth decreased to 13 cm

i
and the resolution mode used (note that the
top of the displayed image now is 2 cm from

h
A B the skin) to focus on the aortic and mitral
valves.

ta
r/
n Harmonic imaging is frequently used to improve PW:1.75MHz



image quality, particularly recognition of endothe­ .50

e
lial borders.

s
n Depth, zoom mode, and sector width are adjusted
m/s



to optimize the image and frame rate, depending

/r u
on the structure or flow of interest.
n Gain settings are adjusted to optimize the data



recording while avoiding artifacts.

.t c
v KEY POINTS

o Although the control panel varies for each



instrument, the basic functions are similar for all 1.5

a
ultrasound systems.
The highest frequency that penetrates ade­ Figure 2-4 Pulsed Doppler velocity recording. The spectral display of

k
o



quately to the depth of interest is used for opti­

the pulsed wave (PW) Doppler signal is shown with the baseline shifted

/: /
mal imaging. and velocity scale adjusted to avoid aliasing and to use the full vertical axis
o With harmonic imaging, flat structures, such as to improve measurement accuracy; for example, the signal fits but fills the
graphical display. The horizontal time scale is 100 mm/s, which is standard



valve leaflets, appear thicker than with funda­ for most Doppler recordings.

s
mental imaging.
o Frame rate is higher for a shorter depth or a

tt p



narrower sector; a fast frame rate is especially
important with Doppler color flow imaging.
o Too narrow a sector may miss important ana­ all the structures in the image plane and then



tomic or physiologic findings. at a depth and sector width optimized for the

h
o Excessive gain results in artifacts with both structures of interest (Fig. 2 3).
-



imaging and Doppler, whereas inadequate gain o Additional zoom mode images of normal and



results in data loss. abnormal findings are recorded as needed.
o Spectral pulsed and continuous wave (CW)
Step 4: Data Recording



Doppler data are recorded with the baseline
n Representative images from the echo study are and velocity range adjusted so the flow signal



recorded, usually digitally, to document the find­ fits but fills the vertical axis. The time scale is
ings and for later review and measurement. adjusted to maximize the accuracy of mea­
n Echo images typically include an electrocardio­ surements (usually an x axis of 100 mm/s)
-



gram tracing for timing purposes. (Fig. 2 4).
-
o Color Doppler is recorded after sector width
v KEY POINTS



and depth are adjusted to optimize frame rate,

o Echo images in each view are recorded first and gain is set just below the level that results in



with a depth and sector width that encompasses background speckle.
24 CHAPTER 2  Normal Anatomy and Flow Patterns on Transthoracic Echocardiography

Suprasternal

LV

/
Subcostal Parasternal

9
LA Ao Apical

Figure 2-6 Standard acoustic windows. The positions on the chest wall

9

where ultrasound can reach the cardiac structures without intervening lung

r
or bone include the parasternal, apical, subcostal, and suprasternal notch

i
windows. The parasternal and apical windows typically are optimal with the
patient in a steep left lateral position. For the subcostal window, the patient

h
is supine with the knees flexed to relax the abdominal vasculature. For the
suprasternal notch window, the patient is supine with the head tilted back

a
and to one side.

r/ t
Figure 2-5 Color Doppler with signal aliasing. This color Doppler image acoustic window (parasternal, apical, subcostal,

of left ventricle outflow in an apical long-axis view shows signal aliasing
adjacent to the septum in the subaortic region. Although this appearance and suprasternal) before moving to the next
acoustic window; this approach minimizes the

e
may be due to an asymmetric flow profile, the effects of intercept angle
also may be important. Even if the velocity is identical across the outflow time needed to reposition the patient between

s
tract, compared with the region along the anterior mitral valve leaflet, in the acoustic windows (Fig. 2 6).
region adjacent to the septum the Doppler beam is more parallel to the flow

-
direction. The higher Doppler shift results in signal aliasing. Aliasing at the
o Some examiners prefer to obtain all the imag­

/r u



aortic valve level is expected because the aortic velocity typically exceeds ing data and then obtain all the Doppler data;
the Nyquist limit at this depth (0.76 m/s in this example). this approach allows the Doppler data record­
ing to be tailored to the imaging findings.
With any approach, the examiner may need to go

.t c
o



o The variance mode on the color scale is preferred back to previous acoustic windows at the end of



by many examiners (including the authors) to the examination if additional views or measure­
enhance recognition of abnormal flows. ments are needed based on abnormal findings.

a
o Normal flows may appear “disturbed” on color o The examination sequence also may need to be






flow imaging—for example, when left ventricular modified depending on patient factors (inability

k
outflow velocity exceeds the Nyquist limit and to move, bandages, etc.) or the urgency of the

/: /
signal aliasing occurs (Fig. 2 5). examination.
-
o Conversely, when variance is not used, it is more o Basic measurements are made as the examina­






difficult to distinguish abnormal flows (such as tion is performed (Table 2 2) or during review

s
-
mitral regurgitation) from normal flows (such as of images at completion of the study. Normal
pulmonary vein flow) when both occur in the values for chamber sizes are provided in Chap­

tt p
same anatomic region. ter 6 (see Tables 6 2 and 6 3) and for the aorta
-
-
in Chapter 16 (see Tables 16 1 and 16 2).
Step 5: Examination Sequence
-
-
n In subsequent chapters, the elements of the exami­ Step 6: Parasternal Window

h



nation for each clinical condition are presented in
the order needed for a final diagnosis. Long-Axis View
n Typically, these examination elements are incorpo­ n Many echocardiographers start with the paraster­






rated into a systemic examination sequence. nal long axis view with:
-
• Imaging to show the aortic and mitral valves, left
v KEY POINTS


atrium (LA) and aortic root, the left ventricle (LV)

o There are several approaches to an examina­ base, and the right ventricle (RV) outflow tract



tion sequence; any of these are appropriate if a • Color Doppler to evaluate for aortic and mitral


complete systemic examination is performed. regurgitation
o In some clinical situations, a limited examination n Standard measurements include:






may be appropriate, with the study components • LV end diastolic and end systolic diameters; diastolic


-
-
selected by the referring or performing physician. thickness of the septum and LV inferior lateral wall
-
o The approach suggested here is based on obtain­ just apical from the mitral leaflet tips (Fig. 2 7)



-
ing all data (imaging and Doppler) for each • Aortic diameter at end diastole (Fig. 2 8)


-
-
Normal Anatomy and Flow Patterns on Transthoracic Echocardiography  CHAPTER 2 25

TABLE 2-2 Basic Echo Imaging Measurements

 
Basic Additional
Cardiac Structure Measurements Measurements Technical Details
Left ventricle (LV) • ED dimension • ED volume • 2D imaging is used to ensure measurements







• ES dimension • ES volume are centered and perpendicular to the long







Wall thickness • 2D stroke volume axis of the LV.



• Ejection fraction • M-mode provides superior temporal resolution





/
• LV mass and more accurate identification of endocardial



borders.

9
Left atrium (LA) • AP diameter • LA area • Left atrial anterior-posterior dimension provides






• LA volume a quick screen but may underestimate LA size.

9


• When LA size is important for clinical decision

r


making, measurement of LA volume is helpful.

i
Right ventricle (RV) • Visual estimate • ED RV outflow tract • Quantitation of RV size by echo is challenging






h
of size diameter due to the complex 3D shape of the chamber.
• ED RV length and • Tricuspid annular plane systolic excursion




a
diameter via M-mode is a quantitative measure of RV

t
systolic function.

r/
Right atrium (RA) • Visual estimate • RA size is usually compared to the LA in the




of size apical 4-chamber view.

e
Aorta • ED diameter at • Maximum diameter • With 2D echo, inner-edge to inner-edge






sinuses indexed to expected measurements are more reproducible.

s
dimension • Measurements are made at end-diastole by


• Diameter at multiple convention, but end-systolic measurements

/r u


sites in aorta also may be helpful.
Pulmonary artery • Diameter


AP, Anterior-posterior; ED, end-diastole (onset of the QRS); ES, end-systole (minimum LV volume).

.t c
k a
0 16

/: /
RV

s
Ao

tt p
LV

h
LA

140
116
Figure 2-7 LV M-mode tracing. 2D-guided M-mode recording of the LV at Figure 2-8 Aortic valve M-mode tracing. 2D guided M-mode re-

the mitral chordal level. End-diastolic measurements of wall thickness and

cording of the aortic valve (Ao) and left atrium (LA) allows measure-
cavity dimension are made at the onset of the QRS, as shown. End-systolic ment of aortic root dimension at end-diastole using a leading-edge to
measurements are made at the maximum posterior motion of the septum leading-edge approach; the aortic leaflet separation (arrows); and the left
(when septal motion is normal) or at minimal LV size. The rapid sampling rate atrial maximum anterior-posterior dimension in early diastole. The fine flut-
with M-mode allows more accurate identification of the endocardial border, tering of the aortic valve leaflets is normal.
which is distinguished from chordae or trabeculations as being a continuous
line in diastole, with the steepest slope during systole.
26 CHAPTER 2  Normal Anatomy and Flow Patterns on Transthoracic Echocardiography

ZOOM
PLAX

RV Ao
LV
LV

LA

9/
9
LA

i r
a h
t
A B

r/
Figure 2-9 Mitral valve imaging. A, First the mitral valve is examined at a standard depth (PLAX). B, Then zoom mode (ZOOM) is used to optimize visualiza-

tion of the aortic and mitral valves. The image plane is angled slightly medial and lateral to encompass the medial and lateral aspects of the valve. Some normal

e
thin mitral chords are well seen in this slightly laterally angulated view, extending from the mitral closure plane to the papillary muscle.

s
/r u
.t c
Ao
LV Ao

k a
/: /
LA

s
tt p
Figure 2-10 Ascending aorta. The ascending aorta is visualized by moving the transducer up an interspace from the parasternal long-axis view. Ao, Aorta.

• Left atrial anterior posterior dimension motion in hypertrophic cardiomyopathy or pos­


-
terior buckling in mitral valve prolapse.

h
• Vena contracta width for aortic, mitral, and tricuspid


regurgitation o LA anterior posterior dimension may under­



-
estimate LA enlargement; when clinically
v KEY POINTS indicated, additional measurements for LA

o Images are initially recorded at a depth that volume calculation are made from apical



includes the descending thoracic aorta to detect views.
pleural and pericardial effusions. o The aortic root (sinuses of Valsalva and sino­



o Depth then is reduced to the level of the poste­ tubular junction) is visualized first from the



rior wall for assessment of the size and function standard window and then with the transducer
of the base of the LV and the RV outflow tract. moved up one or more interspaces to visualize
o The aortic and mitral valves are examined with the ascending aorta (Fig. 2 10).



-
zoom mode sweeping through the valve planes o Color Doppler of aortic and mitral valves



from medial to lateral to assess valve anatomy is used to screen for valve regurgitation. If
and motion (Fig. 2 9). more than physiologic regurgitation is pres­
-
o M mode tracings of the mitral valve can aid in ent, further evaluation is needed as discussed



-
timing of leaflet motion, such as systolic anterior in Chapter 12.
Normal Anatomy and Flow Patterns on Transthoracic Echocardiography  CHAPTER 2 27

RV-inflow

RV

PA
RA
Ao
CS

IVC

Figure 2-11  RV inflow view. From the parasternal long-axis view, the im-
age plane is angulated medially to visualize the RV inflow view with the RV,
RA, coronary sinus (CS), inferior vena cava (IVC), and tricuspid valve.

Figure 2-13  RV outflow view. This view, obtained by angulating the trans-
ducer laterally from the parasternal long-axis view, shows the RV outflow
tract, pulmonic valve (arrow), and main pulmonary artery.  Ao, Aorta; PA,
pulmonary artery.

v  KEY POINTS
o  lide apically one interspace if views are not
S
obtained from the standard window.
o Adjust depth to include the RA, RV, and tricus­
pid valve.
o The entrance of the coronary sinus and the
inferior vena cava into the RA are seen in this
view.
o A small amount of tricuspid regurgitation on
color Doppler is seen in most (>80%) normal
Figure 2-12  Tricuspid regurgitant jet velocity. The tricuspid regurgitation individuals and sometimes is referred to as
jet is recorded with CW Doppler from both the parasternal RV inflow view physiologic.
and from the LV apex. Only the highest velocity is reported, in normal sinus o The CW Doppler tricuspid regurgitant jet

rhythm, because the apparent lower velocity signal is due to a nonparal- is recorded from multiple views: the highest
lel intercept angle between the ultrasound beam and regurgitant jet. This
example shows a high velocity jet consistent with severe pulmonary hyper- velocity represents the most parallel intercept
tension. The maximum velocity is measured at the edge of the dense “enve- angle with flow and is used to estimate pulmo­
lope” of flow, avoiding the faint signals due to gain and transit time effects. nary pressure; the lower velocity recordings are
ignored.
Right Ventricular Outflow View
Right Ventricular Inflow View n From the long-axis view, the image plane is angled
n From the long-axis view, the image plane is angled laterally to show the right ventricular outflow view
medially to show the right ventricular inflow view (Fig. 2-13) with:
(Fig. 2-11) with: • 
Imaging of the RV outflow tract, pulmonic valve,
• 
Imaging of the right atrium (RA), tricuspid valve, and main pulmonary artery
and RV • 
Color Doppler evaluation of pulmonic regurgitation
• 
Color Doppler evaluation of tricuspid regur­­gitation (Fig. 2-14)
• 
CW Doppler recording of tricuspid regurgitant jet • 
Pulsed Doppler recording of pulmonary artery flow
velocity (Fig. 2-15)
n Standard measurements include: n Standard measurements include:
• 
Maximum tricuspid regurgitant velocity (­Fig. 2-12) • 
Antegrade velocity in the pulmonary artery
28 CHAPTER 2  Normal Anatomy and Flow Patterns on Transthoracic Echocardiography

assessment of pulmonary pressures and to


v  KEY POINTS exclude pulmonic stenosis or a patent ductus
o  ove cephalad one interspace if views are not
M arteriosus.
obtained from the standard window. o If present, the number and position (atrial,

o Adjust depth to include the RV outflow tract, ventricular, coronary sinus placement) of pacer
main pulmonary artery, and pulmonary artery wires is described.
bifurcation.
o The pulmonic valve often is difficult to visualize in Short-Axis View
adults, but a small amount of pulmonic regurgita­ n From the long-axis view, the image plane is rotated
tion typically is present with a normally function­ 90° to show the short-axis plane with:
ing valve. • 
Imaging and color Doppler at the level of the aortic
o The pulsed Doppler recording of flow in
 valve to evaluate the aortic, tricuspid, and pulmonic
the main pulmonary artery is helpful for valves (Fig. 2-16)

40dB 2 •/+1/0/ 1
PW Depth = 72mm
PW Gate = 2.0mm
PW Gain = 0dB
PV Vmax = 0.58 m/sec
PK Grad = 1.3 mmHg
PW:2MHz
.20

m/s
PA

.80
Figure 2-15  Pulsed Doppler recording of normal flow in the RV out-
flow tract. The pulmonic valve closure click indicates that the sample vol-
ume is on the RV side of the valve. Normal flow shows a smooth velocity
curve that peaks in mid-systole with a velocity less than 1 m/s.

Figure 2-14  Color Doppler of pulmonic valve regurgitation. Color Dop-


pler in a right ventricular outflow view showing a narrow jet of pulmonic
regurgitation (arrow) in diastole. Mild pulmonic regurgitation is seen in about
80% of normal adults.  PA, Pulmonary artery.

Diastole Systole
RVOT

Figure 2-16  Aortic valve imaging. Para- L


sternal short-axis view of a normal trileaflet
aortic valve in diastole (left) and in systole N
RA
(right). The normal positions of the right (R),
left (L) and noncoronary (N) cusps are seen
in diastole. In systole, the open left coronary
cusp often is difficult to see (arrow) because
the leaflet edge is parallel to the ultrasound LA
beam. However, the three commissures of
the open valve are clearly visualized. RVOT,
Right ventricular outflow tract.
Normal Anatomy and Flow Patterns on Transthoracic Echocardiography  CHAPTER 2 29

• 
Three-dimensional (3D) imaging of the aortic o  he atrial septum is seen in the short-axis view
T
valve may be helpful for identification of a bicuspid at the aortic valve level. Color flow imaging
valve may help detect a patent foramen ovale, but
• 
Imaging at the level of the mitral valve for evaluation must be distinguished from normal flow in the
of mitral leaflet anatomy and motion and LV size RA (inflow from the superior and inferior vena
and function (Fig. 2-17) cava and regurgitation across the tricuspid
• 
Imaging at the midpapillary muscle level to evalu­ valve), all of which are adjacent to the atrial
ate global and regional LV size and function septum.
(Fig. 2-18) o Parasternal views of the LV at the papillary
n Standard measurements include: muscle level provide optimal endocardial defi­
• 
M-mode or two-dimensional (2D) measurements nition and are used in conjunction with api­
of the aorta, LA, and LV using the combination of cal views for detection of regional wall motion
long- and short-axis views to ensure the dimensions abnormalities.
are measured in the minor axis of each chamber or
vessel (see Table 2-1) Step 7: Apical Window
v  KEY POINTS Imaging 4-Chamber, ­2-Chamber,
o  he aortic and pulmonic valves normally are
T and Long-Axis Views
perpendicular to each other (that is, when the n 
The apical window usually corresponds to the
aortic valve is seen in short axis, the pulmonic point of maximal impulse and is optimized with
valve is seen in long axis). the patient in a steep left lateral position.
o Zoom mode is used to identify the number of n 
Images are obtained in 4-chamber (Fig. 2-19),
aortic valve leaflets, taking care to visualize the 2-chamber (Fig. 2-20), and long-axis (Fig. 2-21)
leaflets in systole. views to evaluate:
o A bicuspid aortic valve is a common abnor­ • 
LV size, wall thickness, and global and regional sys­
mality, with a prevalence of about 1% of tolic function
the total population, and often is diagnosed • 
RV size, wall thickness, and systolic function
on echocardiography requested for other • 
Anatomy and motion of the mitral and tricuspid
indications. valves
o The coronary artery os may be seen originat­ • 
Left and right atrial size and coronary sinus anatomy
ing in expected positions from the right and left • 
The amount of pericardial fluid, if present
coronary sinuses.

PSAX-LV

PSAX-MV

RV LV

AMVL

LV

PMVL

Figure 2-18  LV short-axis view. Parasternal short-axis view (PSAX)


of the LV at the papillary muscle level. The LV cavity should appear
circular in this view, and an elliptical shape suggests an oblique in-
Figure 2-17  Mitral valve imaging. Parasternal short-axis view of the left tercept angle. This view sometimes requires the transducer be moved
ventricle at the level of the mitral valve showing both anterior and posterior slightly apically from the short-axis view of the aortic valve, instead of
valve leaflets.  AMVL, Anterior mitral valve leaflet; PMVL, posterior mitral just tilting the transducer towards the apex from a fixed position on
valve leaflet; PSAX-MV, parasternal short-axis-mitral valve. the chest wall.
30 CHAPTER 2  Normal Anatomy and Flow Patterns on Transthoracic Echocardiography

LV LV
RV

RA LA Ao

LA

Figure 2-19  Apical 4-chamber view. The transducer is correctly posi- Figure 2-21  Apical long-axis view. This view is obtained by rotating an
tioned over the LV apex as shown by the longer LV length than width and additional 60° counterclockwise to obtain an image similar to the paraster-
ellipsoid shape of the chamber. Foreshortening of this view results in a more nal long-axis view.
spherical appearance of the LV. This older adult has enlargement of both
atrium and some benign thickening (lipomatous hypertrophy) of the atrial
septum. The loss of signal in the mid-segment of the atrial septum is an
artifact because the thin fossa ovalis is parallel to the ultrasound beam at
this point resulting in echo “dropout.”

Figure 2-22  Left atrial volume measurement. Left ventricle volume is


measured in the apical 4-chamber view by tracing the inner edge of the
LV atrial border at end-systole. At the mitral annulus, a straight line from leaflet
insertion to leaflet insertion is used for this calculation. The area is also
traced in the 2-chamber view for a biplane volume calculation.

n Standard measurements include:


LA • 
Visual estimate of LV ejection fraction
• 
A 2D apical biplane or 3D ejection fraction (see
Chapter 5)
• 
Measurement of LA area or volume when clinically
indicated (Fig. 2–22)
• 
Visual estimate of RV size and systolic function and
measurement of basal RV diameter
Figure 2-20  Apical 2-chamber view. This view is obtained by rotating the • 
M-mode measurement of tricuspid annular plane
transducer about 60° counterclockwise from the 4-chamber view. systolic excursion (TAPSE)
Normal Anatomy and Flow Patterns on Transthoracic Echocardiography  CHAPTER 2 31

LV

RV
RV

RA RA

Figure 2-23  Apical view focusing on the right ventricle. RV size and Figure 2-25  Coronary sinus. The entrance of the coronary sinus (arrow)
function are best estimated by centering the RV in the image plane and into the right atrium is visualized by posterior angulation from the apical
adjusting depth and zoom appropriately. 4-chamber view.

.69 the valve annular plane. The RV is best visual­


ized using zoom mode (Fig. 2-23).
o From the 4-chamber view, the image plane is
angled anteriorly to visualize the aortic valve
.69
LV (sometimes called the 5-chamber view); this view is
useful for Doppler recordings, but image quality
RV
is suboptimal at the depth of the aortic valve
from the apical window (Fig. 2-24).
RA o The image plane is angled posteriorly to visu­
Ao
LA alize the length of the coronary sinus and its
entrance into the RA (Fig. 2-25).
o The LA appendage is not well visualized on
transthoracic imaging, and the sensitivity for
Figure 2-24  Anteriorly angulated apical 4-chamber view (or detection of LA thrombus is low. Transesoph­
5-­chamber view). Anterior angulation from the 4-chamber view allows vi- ageal imaging is needed when atrial thrombus
sualization of the LV outflow tract and an oblique view of the aortic (Ao) valve. is suspected.
Laminar flow in the LV outflow tract is demonstrated with color Doppler.
o The descending thoracic aorta is seen in cross
section behind the LA in the long-axis view
and in a longitudinal plane from the 2-chamber
view with lateral angulation.
v  KEY POINTS
o  he three apical views are at approximately 60°
T Doppler Data
of rotation from each other; however, image n 
The apical window provides an intercept angle
planes are based on cardiac anatomy, not exter­ that is relatively parallel to flow for the aortic,
nal reference points, so that slight adjustment mitral, and tricuspid valves. Standard data record­
of transducer position and angulation often is ing includes:
needed to optimize the image. • 
Pulsed Doppler recordings of transmitral flow, pulmo­
o Initial views are recorded at the maximum
 nary vein inflow, and LV outflow (Fig. 2-26)
depth to see all the cardiac chambers and sur­ • 
Color Doppler evaluation of mitral and tricuspid
rounding pericardium. regurgitation
o Evaluation of the LV and RV are based on • 
CW Doppler recordings of mitral, tricuspid, and
images with the depth adjusted to just beyond aortic antegrade flow and regurgitation (Fig. 2-27)
32 CHAPTER 2  Normal Anatomy and Flow Patterns on Transthoracic Echocardiography

40dB 1 •/+1/0/ 1 40dB 2 •/+1/0/ 1


PW Depth = 80mm PW Depth = 96mm
PW Gate = 2.0mm PW Gate = 2.0mm
PW Gain = –5dB PW Gain = 7dB

MV Peak E = 0.78 m/sec


PK Grad = 2.4 mmHg
PW:2MHz PW:2MHz

E
.80 m/s

Pulsed
m/s
A 1.5 A
.69 40dB 2 •/+1/0/ 1 3V2c-S CW:2MHz APX AV
PW Depth = 154mm H4.0MHz R50mm 1.0
PW Gate = 2.0mm UWMC
PW Gain = 10dB UWMC/ V
.69 RUPV A Vmax = 0.33m/sec HR = 56bpm
PK Grad = 0.4 mmHg
PW:2MHz Sweep = 100mm/s
.80
m/s
S D

m/s

.40 a
B
CWD
Figure 2-26  LV diastolic function. A, LV inflow is recorded using pulsed
Doppler with the sample volume positioned at the mitral leaflet tips in dias- 1.5
tole. The typical early (E) diastolic filling velocity and atrial (A) velocity are B
seen. B, Left atrial inflow is recorded with the pulsed Doppler sample vol- Figure 2-27  LV outflow. A, LV outflow is recorded with the Doppler sample
ume in the right superior pulmonary vein in an apical 4-chamber view. The volume on the LV side of the aortic valve either in an anteriorly angulated
normal pattern of systolic (S) and diastolic (D) inflow with a small atrial (a) 4-chamber view or in an apical long-axis view. The normal smooth “en-
flow reversal are seen. velope” of flow with dense signals along the outer edge and few velocity
signals within the curve are seen. Again, the baseline and scale are adjusted
to prevent aliasing and allow accurate measurements. B, Aortic flow velocity
is recorded from an apical approach using CW Doppler. This velocity tracing
n Standard measurements include: includes signals from the entire length of the ultrasound beam so that the
• 
Pulsed Doppler antegrade mitral early diastolic fill­ velocity curve is filled in by lower velocities proximal to the valve. The aortic
ing (E) and atrial filling (A) velocities closing click is seen. In diastole, the relatively broad CW beam intersects the
• 
Pulsed Doppler LV outflow and CW Doppler aortic left ventricular inflow curve (arrow).
flow velocities
• 
Maximum velocity of the tricuspid regurgitant jet
• 
Tissue Doppler RV systolic velocity (a measure of o  he CW Doppler recordings of aortic, mitral,
T
RV systolic function) and tricuspid regurgitation provide data on the
• 
Additional measurements as clinically indicated (see severity of regurgitation (based on the density
specific chapters for each clinical condition) of the signal) and the transvalvular hemody­
namics (based on the shape and density of the
v  KEY POINTS time-velocity curve).
o ransmitral and pulmonary venous inflow
T o Color flow Doppler from the apical approach
velocities are helpful for evaluation of LV dia­ is helpful for evaluation of jet direction and for
stolic dysfunction (see Chapter 7). Pulsed Dop­ visualization of proximal jet geometry (vena
pler tissue velocities of the myocardial septal contracta) and proximal isovelocity surface area
or lateral wall also are helpful for evaluation of (PISA) for mitral regurgitation
diastolic function. o Apical color Doppler is less helpful for aortic
o There is only a small increase in velocity from regurgitation because beam width is greater at
the LV outflow tract to the ascending aorta in the depth of the aortic valve than at the mitral
normal individuals (see Chapter 11). valve.
Normal Anatomy and Flow Patterns on Transthoracic Echocardiography  CHAPTER 2 33

Subcostal

RV

RA RV
LV IVC

LA

Figure 2-28  Subcostal 4-chamber view. This view is useful for evalua- Figure 2-29  Inferior vena cava (IVC). The IVC is examined from the sub-
tion of RV and LV function. This view also is best for evaluation of the atrial costal view with the size and respiratory variation used to estimate right
septum because the ultrasound beam is perpendicular to the septum from atrial pressure, as discussed in Chapter 6.
this transducer position.

Step 8: Subcostal Window


n The subcostal window provides:
• 
An alternate acoustic window for evaluation of LV
and RV systolic function (Fig. 2-28)
• 
An optimal angle to evaluate the interatrial septum
• 
Estimation of RA pressure based on the size and
respiratory variation in the inferior vena cava
(Fig. 2-29)
• 
Pulsed Doppler evaluation of hepatic vein flow (right
atrial inflow) and proximal abdominal aortic flow,
when clinically indicated (Fig. 2-30)

v  KEY POINTS
o  stimation of RA pressure is a standard part
E Figure 2-30  Hepatic vein flow. Pulsed Doppler velocities are recorded
of the examination used to calculate pulmonary from the subcostal view to evaluate right atrial filling when tricuspid regur-
gitation or pericardial disease is of concern.  a, Atrial; D, diastolic wave;
systolic pressure. S, systolic wave; v, ventricular.
o Atrial septal defects often are best visualized on
imaging and with color Doppler using a low
Nyquist setting from the subcostal window.
o Hepatic vein flow patterns are helpful for detec­ n The suprasternal window provides:
tion of severe tricuspid regurgitation (in sinus • 
Images of the aortic arch and proximal descending
rhythm) and for evaluation of pericardial disease. thoracic aorta (Fig. 2-31)
o Descending aortic holodiastolic flow reversal is • 
Pulsed and CW Doppler evaluation of descending
seen with severe aortic regurgitation; persistent aortic flow, when clinically indicated (Fig. 2-32)
holodiastolic antegrade flow is seen with aortic • 
A parallel intercept angle with the aortic velocity in
coarctation. some patients with native or prosthetic aortic valve
disease
Step 9: Suprasternal Window
n The suprasternal window is a standard part of the
v  KEY POINTS
examination of patients with diseases of the aortic o  ortic disease, such as aortic dissection, may be
A
valve or aorta. visualized from this window.
34 CHAPTER 2  Normal Anatomy and Flow Patterns on Transthoracic Echocardiography

• 
Echo findings
• 
Conclusions (with recommendations)
SSN v  KEY POINTS
o  he clinical data section includes the reason for
T
Ao the study, pertinent history and physical exami­
nation findings, cardiac medications, and blood
PA pressure.
o Standard measurements are indicated in the
example (Table 2-3) with additional measure­
ments as clinically indicated.
o The findings section documents what views and
flow were recorded and describes any abnormal
and key normal findings.
o The conclusions indicate the major diagnosis,
associated findings, and pertinent negative find­
ings (depending on the indication for the study).
o When clinically appropriate, specific recom­

mendations are made. These include:
o  he clinical significance of the findings
T
o Recommendations for cardiology evaluation and
periodic follow-up
Figure 2-31  Suprasternal notch (SSN) view. This view shows the as- o  erious unexpected findings are promptly com­
S
cending aorta (Ao), arch, and descending thoracic aorta. A small segment of municated directly to the referring physician.
the right pulmonary artery (PA) is seen in cross section. o When data are not definitive, the findings are
described along with a differential diagnosis to
explain these findings.
40dB 1 •/+1/0/ 1 o Additional diagnostic approaches are recom­
PW Depth = 76mm
PW Gate = 2.0mm
mended as appropriate.
PW Gain = –2dB
Aging Changes on Echocardiography
n Changes in cardiac anatomy and physiology occur
PW:2MHz with age and are evident on echocardiography.
n In general, the LV walls are thicker, diastolic dys­
function is present, the atria are larger, and the
m/s angle between the long axis of the LV and the
aorta is more acute.
n Calcific changes of the aortic and mitral valve are
common but not normal because they are associ­
1.0 ated with an increased risk of adverse cardiovascu­
lar outcomes.

Figure 2-32  Normal pattern of flow in the descending thoracic aorta.


v  KEY POINTS
There is antegrade flow in systole, brief diastolic flow reversal as a result of o  V wall thickness increases by 2 mm between
L
aortic recoil and coronary blood flow, a small amount of antegrade flow in young adulthood and age 70 years.
mid-diastole, and slight reversal just before the next cardiac cycle.
o LV diastolic filling typically shows an E velocity
less than A velocity about age 50 years.
o  n increased systolic velocity with persistent
A o Mild LA enlargement is typical in older patients.
antegrade flow (“runoff ”) in diastole is seen o Aortic diameter increases by 6% between age
with an aortic coarctation. 30 and 70 years.
o Holodiastolic flow reversal in the descending aorta o Aortic valve sclerosis is present in 25% of adults
suggests significant aortic valve regurgitation. older than age 65 years.
o Mitral annular calcification is present in 50%
Step 10: The Echo Report of older adults.
n The echo report consists of four sections: o The angle between the basal LV septum and
• 
Clinical data ascending aorta (the septal “knuckle”) becomes
• 
Measurements more acute (less parallel) with age.
Normal Anatomy and Flow Patterns on Transthoracic Echocardiography  CHAPTER 2 35

TABLE 2-3 Sample Echo Report


Name__________________________ Date of Study_______________________
Age: 45 years Sex: M
Indication: Systolic murmur on auscultation
Cardiac medications: None
Clinical history: Chest pain on exertion, systolic murmur, no prior cardiac procedures
Blood pressure: 118/68 mmHg Heart rate: 60 bpm Rhythm: NSR
Sonographer: BKS Image quality: Excellent

Measurements
Dimensions (cm) Normal Values (men)
Left ventricle
End-systole 3.2 2.1-4.0 cm
End-diastole 5.4 4.2-5.9 cm
Wall thickness (diastole) 0.8 0.6-1.0 cm
2D ejection fraction 65% (estimate) ≥55 %
Left atrium 3.6 3.0-4.0 cm
Aortic sinus 2.9 <4.0 (<2.1 cm/m2)

Doppler flows
Regurgitation Velocity (m/s)
Aortic valve None LVOT 1.0
Ao 1.4
Mitral valve Trace E 1.0
A 0.4
Pulmonary valve Trace
Tricuspid valve Mild Tricuspid regurgitation 2.3
jet (TR jet)
Normal Values
Right atrium pressure estimate (mmHg) 5 0-5
Pulmonary artery (PA) pressure estimate (mmHg) 26 20-30
Findings
Left ventricle Wall thickness, internal dimension, and systolic function are normal with an estimated ejection
fraction of 65%. There are no resting regional wall motion abnormalities.
Left atrium Size is normal.
Aortic valve Trileaflet with normal systolic opening and no regurgitation.
Aortic root Normal dimensions with normal contours of the sinuses of Valsalva.
Mitral valve Normal anatomy and motion with no stenosis and only physiologic regurgitation.
PA pressures Estimated pulmonary systolic pressure is normal at 21-16 mmHg, based on the velocity in the
TR jet and the size and respiratory variation of the inferior vena cava.
Right heart Right ventricular size and systolic function are normal. Tricuspid and pulmonic valves show
normal anatomy and Doppler flows. Right atrial size is normal.
Pericardium No effusion.
Conclusions
1. Normal valve anatomy and function.
2. Normal left ventricle with an estimated ejection fraction of 65%.
3. Normal pulmonary pressures and right heart.
Given these findings, the murmur appreciated on physical examination most likely is a benign flow murmur. Although
resting left ventricular regional function is normal, coronary disease cannot be excluded on a resting study. If there is
concern that chest pain may be due to coronary disease, a stress study should be considered.
Signed: ___________________MD
36 CHAPTER 2  Normal Anatomy and Flow Patterns on Transthoracic Echocardiography

THE ECHO EXAM

The Transthoracic Echo Exam: Core Elements


A Complete Echo Exam Consists of Core Elements + Additional Components
Modality Window View/Signal Basic Measurements
Clinical data Indication for echo Blood pressure at time of echo
Key history and physical exam findings exam
Previous cardiac imaging data
2D imaging Parasternal Long axis LV ED and ES dimensions
Short-axis aortic valve LV ED wall thickness
Short-axis mitral valve Aortic ED sinus dimension
Short-axis LV (papillary muscle level) LA dimension
RV inflow
Apical 4-chamber Visual estimate or biplane ejection
Anteriorly angulated 4-chamber fraction
2-chamber
Long axis
Subcostal 4-chamber
IVC with respiration
Proximal abdominal aorta
Suprasternal Aortic arch
Pulsed Doppler Parasternal Pulmonary artery flow PA velocity
Apical LV inflow E velocity
A velocity
LV outflow LV outflow velocity
Color flow Parasternal Long axis: aortic and mitral valves Color flow to identify regurgitation
Short axis: aortic and pulmonic valves of all four valves. If more than
RV inflow : tricuspid valve mild, measure vena contracta
Apical 4-chamber: mitral and tricuspid valves
Long axis: aortic and mitral valves
CW Doppler Parasternal Tricuspid valve TR-jet velocity
Pulmonic valve
Apical Aortic valve Aortic velocity
Mitral valve
Tricuspid valve TR-jet (pulmonary pressures)

A-velocity, Late diastolic ventricular filling velocity with atrial contraction; E-velocity, early-diastolic peak velocity; ED, end-diastole; ES, end-systole;
IVC, inferior vena cava; PA, pulmonary artery; PE, pericardial effusion; TR, tricuspid regurgitation.
Normal Anatomy and Flow Patterns on Transthoracic Echocardiography  CHAPTER 2 37

THE ECHO EXAM

The Transthoracic Echo: Additional Components


The Transthoracic Echo Exam: Additional Components
Abnormality on Core Elements Additional Echo Exam Components (Chapter)
Reason for Echo Additional Components to Address Specific Clinical Question*
LEFT VENTRICLE
Decreased ejection fraction See Systolic Function (Chapter 6)
Abnormal LV filling velocities See Diastolic Function (Chapter 7)
Regional wall motion abnormality See Coronary Artery Disease (Chapter 8)
Increased wall thickness See Hypertrophic Cardiomyopathy, Restrictive Cardiomyopathy and
­Hypertensive Heart Disease ­(Chapter 9)

VALVES
Imaging evidence for stenosis or an See Valvular Stenosis (Chapter 11)
increased antegrade transvalvular
velocity
Regurgitation greater than mild on See Valvular Regurgitation (Chapter 12)
color flow imaging or CW Doppler
Prosthetic valve See Prosthetic Valves (Chapter 13)
Valve mass or suspected endocarditis See Endocarditis and Cardiac Masses (Chapters 14 and 15)

RIGHT HEART
Enlarged right ventricle See Pulmonary Heart Disease and Congenital Heart Disease
(Chapters 9 and 17)
Elevated TR-jet velocity See Pulmonary Pressures (Chapter 6)

PERICARDIUM
Pericardial effusion See Pericardial Effusion (Chapter 10)
Pericardial thickening See Constrictive Pericarditis (Chapter 10)

GREAT VESSELS
Enlarged aorta See Aortic Disease (Chapter 16)

*The echo exam should always include additional components to address the clinical indication. For example, if the indication is heart failure,
additional components to evaluate systolic and diastolic function are needed even if the Core Elements do not show obvious abnormalities. If the
indication is cardiac source of embolus, the additional components for that diagnosis are required.

Principles of Doppler Quantitation


Method Assumptions/Characteristics Examples of Clinical Applications
Volume flow • Laminar flow • Cardiac output
SV = CSA × VTI • Flat flow profile • Continuity equation for valve area
• CSA and ­VTI ­measured at same site • Regurgitant volume calculations
• Intracardiac shunts, pulmonary to
­systemic flow ratio
Velocity-pressure relationship • Flow limiting orifice • Stenotic valve gradients
ΔP = 4v2 • CW Doppler signal recorded parallel to • Calculation of pulmonary pressures
flow • LV dP/dt
Spatial flow patterns • Proximal flow convergence region • Detection of valve regurgitation and
• Narrow flow stream in orifice (vena ­intracardiac shunts
­contracta) • Level of obstruction
• Downstream flow disturbance • Quantitation of regurgitant severity

CSA, Cross-sectional area; ΔP, pressure gradient; VTI, velocity time integral.
38 CHAPTER 2  Normal Anatomy and Flow Patterns on Transthoracic Echocardiography

SELF-ASSESSMENT QUESTIOINS
Question 1 Question 3
Identify the numbered “spaces” in Fig. 2-33: From the imaging view presented (Fig. 2-35), which
1 __________________ tricuspid valve leaflet is indicated?
2 __________________ A. Lateral
3 __________________ B. Anterior
4 __________________ C. Septal
5 __________________ D. Posterior
E. Medial

Figure 2-33 
Question 2
From the image presented (Fig. 2-34), identify which
echo finding is indicated by the arrow:
A. Eustachian valve
B. Atrial myxoma
C. Crista terminalis
D. Thebesian valve
Figure 2-35 

Figure 2-34 
Normal Anatomy and Flow Patterns on Transthoracic Echocardiography  CHAPTER 2 39

Question 4 Question 6
Based on the image provided (Fig. 2-36), you are Which myocardial segments are typically seen in the
suspicious for a congenital abnormality. In order to apical long-axis view?
exclude associated abnormalities with this condition, __________________________________________
you conclude that you should:
A. Move the transducer to a higher interspace Question 7
B. Rotate the transducer to a short-axis view This Doppler flow signal (Fig. 2-38) is most consistent
C. Move the transducer to a right parasternal window with:
D. Angulate the transducer posteriorly A. LV inflow
B. LV outflow
C. Pulmonary vein flow
D. Pulmonary artery flow
E. Descending aorta flow

2.0

m/s
.50
Figure 2-38 

Question 8
This Doppler tracing (Fig. 2-39) is most consistent
Figure 2-36  with:
A. Aortic stenosis
Question 5 B. Aortic regurgitation
In the short-axis view of the mitral valve provided C. Mitral stenosis
(Fig. 2-37), label the corresponding scallops: D. Mitral regurgitation
  
E. Tricuspid regurgitation
Anterior 1 (A1) ____
Anterior 2 (A2) ____
Anterior 3 (A3) ____
Posterior 1 (P1) ____
Posterior 2 (P2) ____
Posterior 3 (P3) ____
2.0

m/s

a c
b
f
d
e

.60

Figure 2-37  Figure 2-39 


40 CHAPTER 2  Normal Anatomy and Flow Patterns on Transthoracic Echocardiography

Question 9
0
The atrial border tracing for left atrial volume mea­
surement should be traced:
A. In the parasternal long-axis view
B. To include the atrial appendage
C. From the mitral annular plane
D. At end-diastole
Question 10
You are asked to review an echocardiogram of a
patient who is a cardiac transplant recipient. Serial
echocardiograms in the past document an LV end-
diastolic dimension of 4.5 cm. The LV dimension
from today’s study is provided (Fig. 2-40). The most
likely explanation for the change between studies is: 40
A. Measurement error Figure 2-40 
B. Interval LV enlargement
C. Measurement variability
D. Misaligned ultrasound
Normal Anatomy and Flow Patterns on Transthoracic Echocardiography  CHAPTER 2 41

ANSWERS

Answer 1 To confirm a diagnosis of a bicuspid valve, rotating


1 . Right ventricle the probe to a short-axis view would aid in visualizing
2 . Left ventricle the valve cusp anatomy. However, to exclude associ­
3 . Pericardial effusion ated ascending aortic dilation which may be present in
4 . Pleural effusion patients with a bicuspid aortic valve, visualization of the
5 . Descending thoracic aorta ascending aorta should be optimized, with the trans­
This is a parasternal long-axis view of the heart, ducer moved to a higher intercostal space. In Fig. 2-42,
set to an increased depth of over 18 cm. The num­ the transducer was moved to the first intercostal space
bered echolucent space closest to the transducer (1) with improved visualization of the distal portion of
is the RV, and the adjacent chamber (2) is the LV. the ascending aorta. If, despite these maneuvers, the
This patient has a small pericardial effusion which is ascending aorta is not well seen, then another imaging
circumferential, but more prominent posteriorly to modality, such as MRI or cardiac computed tomogra­
the heart (3). The pericardial effusion is easily seen phy, should be performed. Moving the transducer to a
tracking anteriorly to the descending thoracic aorta right parasternal window or angulating posteriorly will
(5), which is imaged in cross section. There is a small not aid in visualization of the ascending aorta.
strip of pericardial fluid seen anterior to the RV as
well. Posterior to the heart is a large left-sided pleu­
ral effusion (4) which is seen tracking posterior to the
descending thoracic aorta.
Answer 2: C
The crista terminalis is the embyologic line of union
between the trabeculated appendage and the RA.
When echo imaging planes go through the mid-
portion of the crista terminalis, as is seen in the
apical 4-chamber view, a prominent junction line
appears as a circular echodensity along the right
atrial wall. The Eustachian valve overlies the infe­
rior vena cava, lies at the junction of the inferior
vena cava and RA, and is variable in size, length,
and prominence in individuals. The function of the
Eustachian valve was to direct intrauterine flow of
oxygenated blood from the inferior vena cava to the
fossa ovalis. An atrial myxoma is a benign cardiac
tumor, most commonly seen in the LA, but can also
be seen in the RA (∼20% to 30% of cases). Echocar­
Figure 2-41 
diographic appearance of myxomas is heterogenous,
typically a multilobular mass with a pedunculated
attachment point on or near the interatrial septum.
The thebesian valve is a membranous structure
which originates at the superior vena cava, at the ori­
fice of the coronary sinus. It is highly variable in size
among individuals and is not commonly seen.
Answer 3: B
There are three leaflets of the tricuspid valve: septal,
anterior, and posterior. From the apical 4-chamber
view, the septal and anterior leaflets are seen. The
only view where the posterior leaflet is seen is the right
ventricular inflow view, where the posterior leaflet is
in the far field (Fig. 2-41).
Answer 4: A
The parasternal long-axis view shows asymmetric clo­
sure of the aortic valve and a bicuspid valve is suspected. Figure 2-42 
42 CHAPTER 2  Normal Anatomy and Flow Patterns on Transthoracic Echocardiography

occur in systole with an ejection curve shape, with


Answer 5 a mid-systolic peak for the pulmonary artery and an
The anterior mitral valve leaflet is larger than the early systolic peak for aortic flow. Descending aorta
posterior leaflet, with the coaptation line between flow also occurs in systole, directed away from the
the leaflets showing a semi-circular appearance in transducer.
short-axis views of the valve. There are three scal­
lops of each valve, numbered 1 through 3. By con­ Answer 8: A
vention, A1 (answer “c”) and P1 (answer “f ”) coapt This is a CW Doppler recording (note velocity scale)
and lie along the lateral aspect of the valve. Similarly, showing a systolic flow signal, directed away from
A3 (answer “a”) and P3 (answer “d”) coapt and lie the transducer with a maximum velocity of 4.8 m/s.
along the medial aspect (closer to the aortic valve). Aortic stenosis is differentiated from mitral regurgi­
A2 is label “b” and P2 is label “e.” It is important tation in the delay after the QRS signal before flow,
to follow anatomical landmarks, as the images appear which corresponds to isovolumic contraction before
“flipped” with transesophageal imaging compared to aortic valve opening. Mitral regurgitation peak veloci­
transthoracic imaging, although enface views of the ties may be in this range when recorded from an
mitral valve by TEE are most commonly obtained via apical window, reflecting the over 100 mmHg pres­
3D imaging (Fig. 2-43). sure difference between the LV and LA in systole. If
mitral stenosis or aortic regurgitation were present,
increased Doppler signal would be present in diastole.
Aortic regurgitation results in a high velocity diastolic
Aortic valve
flow signal, due to the diastolic pressure difference
between the aorta and LV. Tricuspid regurgitation is
longer in duration than aortic stenosis (no isovolumic
contraction).
A1
A2 A3 Answer 9: C
P1 The left atrial border tracing for volume measure­
P3 ment should be performed from an apical window
P2 when the atria are maximally filled, which occurs
at end-systole. Measurements should be taken from
both the apical 2-chamber (Fig. 2-44, A) and the api­
cal 4-chamber (Fig. 2-44, B) views.
Figure 2-43  Care should be taken that images are optimized
and not foreshortened, so volumes are not under­
estimated. The border tracing should follow the
Answer 6 blood-tissue border of the LA and a horizontal line
across the mitral annulus. The atrial area between
The myocardial segments that are typically seen
the mitral annular plane and the mitral leaflet coap­
are basal to mid-anteroseptal and the apical ante­
tation point should be excluded. The left atrial
rior segments, and the basal to mid-inferolateral
appendage and the pulmonary vein ostia should also
and the apical lateral segments. The three apical
be excluded from the atrial area measurement. If
views (4-chamber, 2-chamber, long-axis views) are
either the 2-chamber or 4-chamber view is subopti­
at approximately 60° of rotation from each other.
mal, then a single apical view measurement can be
Nomenclature for the myocardial segments is based
used twice in the left atrial volume calculation. The
on a 17-segment model, as published in the A­ merican
parasternal long-axis view is useful to provide an
Society of Echocardiography recommendations for
anterior-posterior left atrial dimension, but, because
chamber quantification. The apical long-axis view
it is a single linear measurement of the atria, may
images the basal to mid-anteroseptal and the apical
underestimate overall left atrial size. Border tracings
anterior segments. This view also shows the basal to
from the parasternal long-axis view would be inaccu­
mid-inferolateral and the apical lateral segments.
rate as this view does not allow for full visualization
Answer 7: A of the atria.
LV inflow in diastole consists of an early (E) dia­ Answer 10: D
stolic peak with a second peak occurring with atrial
The left ventricular end-diastolic dimension is mea­
(A) contraction. Pulsed Doppler recording of pul­
sured at 5.0 cm on the provided M-mode tracing.
monary vein flow is low velocity with signals for sys­
This is 0.5 cm larger than previous measurements.
tole and diastole, and slight reversal of flow after
Interval change from the prior study or measure­
atrial contraction. Aortic and pulmonary outflow
ment variability should be considered, but only once
Normal Anatomy and Flow Patterns on Transthoracic Echocardiography  CHAPTER 2 43

A B
Figure 2-44 

proper alignment of the M-mode ultrasound beam is


ensured. 2D-guided M-mode allows for spatial align­
ment of the M-mode ultrasound beam to ensure car­
diac dimension measurements that are perpendicular
to the endocardial border. Misaligned M-mode trac­
ings lead to oblique images of the LV and overesti­
mation of cardiac dimensions. The dimension was
measured correctly from the M-mode image, but the
beam itself is misaligned. In this patient, a 2D image
shows misalignment of the M-mode beam (dashed
line), demonstrating the end-diastolic dimension (cross
marks) is actually unchanged from previous studies, at
4.5 cm (Fig. 2-45)
Although 2D imaging improves spatial resolu­
tion, M-mode has significantly improved temporal
resolution, which may allow for better visualiza­
tion of the blood-tissue border if image quality is
suboptimal.

Figure 2-45 
3 Transesophageal Echocardiography
STEP-BY-STEP APPROACH Coronary Arteries
Clinical Data Right Ventricle and Tricuspid Valve
TEE Protocol Right Atrium
Basic Examination Principles Pulmonary Valve and Pulmonary Artery
Imaging Sequence Descending Aorta and Aortic Arch
Left Ventricle The TEE Report
Left Atrium and Atrial Septum THE ECHO EXAM
Mitral Valve SELF-ASSESSMENT QUESTIONS
Aortic Valve and Ascending Aorta

STEP-BY-STEP APPROACH pressure, electrocardiogram, and arterial oxygen


saturation.
Step 1: Clinical Data n Oral suction is used to clear secretions and main-
n In addition to the indication for the study and the tain an open airway.
cardiac history, clinical data establishing the safety n The study is optimally performed with a physician
of the transesophageal echocardiography (TEE) to manipulate the probe and direct the examination,
procedure are needed. a cardiac sonographer to optimize image quality
n The risk of the TEE procedure is related to both and record data, and a nurse to monitor the patient.
moderate sedation and esophageal intubation.
n Informed consent is obtained before the procedure.
v  KEY POINTS
o Endocarditis prophylaxis is not routinely rec-

v  KEY POINTS ommended for TEE.
o I nformed consent includes a description of the o Adequate local anesthesia of the pharynx
procedure with explanation of the expected improves patient comfort and tolerance.
benefits and potential risks. o The specific choice and dose of pharmacologic
o Complications serious enough to interrupt the agents for sedation are based on institutional
procedure occur in less than 1% of cases, and protocols.
the reported mortality rate is less than 1 in o The TEE probe is inserted via a bite block using
10,000. ultrasound gel for lubrication and to provide
o Significant esophageal disease, excessive bleed- acoustic coupling between the ultrasound trans-
ing risk, and tenuous respiratory status are con- ducer and the wall of the esophagus.
traindications to TEE. o The TEE probe is advanced and diagnostic
o The risk of hemodynamic compromise and
 images are obtained in standard image planes
respiratory depression are assessed using stan- by turning the probe and rotating the image
dard preanesthesia protocols and risk levels. plane (Fig. 3-1).
o Risk is higher in patients with impaired respira- o All health care providers involved in the proce-
tory status or a history of sleep apnea. dure use universal precautions to prevent expo-
o Patients typically have no oral intake for at sure to body fluids.
least 6 hours before the procedure, except in
emergencies. Step 3: Basic Examination Principles
o In anticoagulated patients, the level of antico- n Although the TEE study is directed toward
agulation is checked before the TEE to ensure answering the clinical question, a complete sys-
it is in the therapeutic range. temic examination is recorded unless precluded by
the clinical situation.
Step 2: TEE Protocol n Standard tomographic planes are used to evaluate
n The moderate sedation standards at each institu- cardiac chambers and valves.
tion apply to TEE procedures. n The structure or flow of interest is centered in the
n Typically these include having a credentialed health image plane with depth and sector width adjusted
care provider monitor level of consciousness, blood to optimize image quality.

44
Transesophageal Echocardiography  CHAPTER 3 45

Flexion/extension Distance
markers

Transducer

Rotation
angle

Figure 3-1  TEE probe. The transesophageal multiplane transducer is at


the tip of a steerable probe. The probe motion is controlled by the dials, with
the rotational angle of the image plane adjusted with a button.

n  hree-dimensional (3D) imaging using real-time


T
Figure 3-2  Transgastric image planes. From the transgastric position,
zoom mode and acquisition of full volume 3D data- the probe is positioned near the gastroesophageal junction to obtain a
sets is used selectively based on the clinical diagno- short-axis view of the LV or is advanced into the stomach to obtain an
sis and two-dimensional (2D) imaging findings. “apical” view. Transgastric apical images may show a foreshortened LV
because the true left ventricular apex often does not lie on the diaphragm.
v  KEY POINTS (From Otto, CM: Textbook of Clinical Echocardiography, ed 5, Elsevier,
2013, Philadelphia.)
o I n stable patients, a complete TEE examination
is recommended with recording of all standard
image planes and Doppler flows. often is incorporated into the standard exam
o In unstable patients, the examination should sequence shown in the Echo Exam table.
focus on the key diagnostic issues first, with
additional recordings as tolerance and time v  KEY POINTS
allow. o  he probe position is constrained by the posi-
T
o Each cardiac structure is evaluated in at least tion of the esophagus so that optimal views are
two orthogonal views or, ideally, using a rota- not always possible.
tional scan of the structure. o The terms advance and withdraw refer to the ver-
o The structure of interest is centered in the tical motion of the probe in the esophagus and
image plane with transducer frequency, depth, stomach (Fig. 3-2).
and zoom adjusted to optimize visualization. o The term turn refers to manual rotation of the
o With color Doppler, frame rate is optimized by entire probe toward the patient’s right or left side.
decreasing depth and sector width to focus on o The terms flexion and extension refer to motion
the flow of interest. of the tip of the probe in a plane parallel to the
o Only one to two beats of each view are recorded long-axis of the probe, controlled by a large dial
so the examiner can move quickly through the at the base of the probe (Fig. 3-3 and Fig. 3-4).
examination sequence. The total intubation o The term rotation refers to the electronic move-
time for a complete TEE ranges from less than ment of the image plane in a circular fashion,
10 minutes for a relatively normal study to up to controlled by a button on the probe and dis-
30 minutes for complex examinations. played as an angle on the image (Fig. 3-5).
o The exact degree of rotation needed for a spe-
Step 4: Imaging Sequence cific view varies from patient to patient depend-
n The basic imaging sequence suggested in the Echo ing on the relationship between the heart and
Exam: Basic Transesophageal Exam table at the esophagus. The values given here are a starting
end of this chapter is organized by probe posi- point; image planes are adjusted based on car-
tion because this is the most efficient approach to diac anatomy, not specific rotation angles.
examination in most cases. o If a specific view or flow is difficult to obtain,
n The imaging sequence is adjusted to focus on the continue with the examination and return to
key issues in unstable patients. this view later in the study.
n This step-by-step approach describes the evalua- o The specific views and flows recorded depend on
tion of each anatomic structure. This evaluation the clinical indication and the findings of the study.
46 CHAPTER 3  Transesophageal Echocardiography

Figure 3-3  Turning the TEE probe. From a mid-esophageal position, turn-
ing the image plane from left to right provides images of the left pulmonary
veins (purple), aorta and left ventricle (blue), right ventricle (green), and right
Figure 3-5  TEE image plane rotation. Rotation of the image plane start-
atrium with superior and inferior vena cava (yellow). (From Otto, CM: Text-
ing from the 4-chamber view, with the left ventricular apex centered in
book of Clinical Echocardiography, ed 5, Elsevier, 2013, Philadelphia.)
the image, allows a 2-chamber view (see Fig. 3-6) at approximately 60°
rotation and a long-axis view at approximately 120° rotation. Slight repo-
sitioning and angulation of the transducer may be needed as the image
plane is rotated to ensure inclusion of the left ventricular apex in the image.
(From Otto, CM: Textbook of Clinical Echocardiography, ed 5, Elsevier, 2013,
Philadelphia.)

v  KEY POINTS
o  he starting point for a TEE is a 4-chamber
T
view recorded from a high esophageal posi-
tion (0° rotation) at maximum depth to show
the entire LV. Typically the probe is extended
to include as much of the apex as possible
(Fig. 3-6).
o With the probe centered behind the left
atrium (LA) and the LV apex in the center of
the image, the image plane is rotated to about
60° to obtain a 2-chamber view and then
rotated further to about 120° for a long-axis
view.
o The transducer position, angulation, and exact
Figure 3-4  TEE probe angulation. From a high esophageal position degree of rotation are adjusted to optimize each
with the probe at 0° rotation, the transducer tip is extended to obtain a
4-­chamber view or flexed for a short-axis view of the left atrial appendage. view.
(From Otto, CM: Textbook of Clinical Echocardiography, ed 5, Elsevier, 2013, o Regional ventricular function is evaluated as
Philadelphia.) follows:
Lateral
o wall and inferior septum in the 4-chamber
o  lthough modification of the exam sequence
A view
often is necessary, the examiner should quickly Anterior and inferior walls in the 2-chamber view
o
review a checklist of the recorded data before o Anterior septum and the inferior-lateral (or poste-
removing the probe to ensure a complete exam. rior) wall in the long-axis view
o  jection fraction is estimated from these three
E
Step 5: Left Ventricle views.
n The left ventricle (LV) is evaluated in the high esoph- o Quantitative ejection fraction measurements
ageal 4-chamber, 2-chamber, and long-axis views. can be made either using a 3D full volume acqui-
n Additional views of the LV include the transgastric sition with automated border detection or the
short-axis view and the transgastric apical view. biplane approach with tracing of endocardial
Transesophageal Echocardiography  CHAPTER 3 47

LA
LA LA
Ao
RA

LV
RV LV LV

A4C A2C A-long


Figure 3-6  TEE views of the LV. With the LV apex centered in the image plane at 0°, the apical 4-chamber (A4C) view is obtained. When the angle is adjusted
to about 60°, an apical 2-chamber (A2C) view is obtained showing the anterior and inferior LV walls. With further rotation of the image plane, typically to about
120°, an apical long-axis view (A-long) is obtained with the aortic valve and ascending aorta (Ao) and the inferior-lateral (posterior) and anterior septal walls of
the LV. These views are oriented by the cardiac anatomy, not a specific rotation angle. In this patient, obtaining a long-axis view with the ascending aorta and
LV correctly aligned required rotation of the image plane to 146°.

LA

RA

Figure 3-7  3D LV volume measurement. Ejection fraction can be mea-


sured with TEE using the apical biplane method by tracing endocardial bor-
ders at end-diastole and end-systole in 4-chamber (4D) and 2-chamber Figure 3-8  Atrial septum. The atrial septum is examined by centering the
(2D) views or by using semi-automated border detection with 3D imaging. septum in the image plane at 0° rotation and then slowly rotating the image
(From Otto, CM: Textbook of Clinical Echocardiography, ed 5, Elsevier, 2013, plane to 120°. The thin fossa ovalis (between arrows) is clearly seen on
Philadelphia.) this image.

borders at end-diastole and end-systole in


4-chamber and 2-chamber views (Fig. 3-7). n  he pulmonary veins are identified using 2D
T
o An apical LV thrombus may be missed because and color Doppler imaging most easily in the 0°
the apex is in the far field of the TEE image; image plane, although views at 90° also may be
transthoracic imaging is more sensitive for helpful.
detection of apical thrombus.
v  KEY POINTS
Step 6: Left Atrium and Atrial Septum o I mages of the LA are recorded at a shallow
n The body of the LA is evaluated in the high esopha- depth to focus on the structure of interest.
geal 4-chamber, 2-chamber, and long-axis views. o The atrial septum is best examined by cen-
n The LA appendage is imaged in at least two tering the septum in the image plane in the
orthogonal planes at 0° and 90°. 4-chamber view and then slowly rotating the
48 CHAPTER 3  Transesophageal Echocardiography

38dB 3 •/+1/0/ 1
PW Depth= 30mm
PW Gate= 2.0mm
PW Gain= 11db

PW:3.5MHz

.60

m/s

Figure 3-9  LA appendage. Two views of the LA appendage at about 70°


and 160° rotation are obtained simultaneously using biplane imaging. The
typical crescent shape of the appendage is seen, and the normal ridge is .60
seen between the LA appendage and left superior pulmonary view.

Figure 3-11  Atrial fibrillation. Atrial appendage flow in a patient in atrial


fibrillation shows a rapid, irregular low velocity flow pattern.
40dB 3 •/+1/0/ 1
PW Depth= 22mm
PW Gate= 1.0mm
o  The left superior pulmonary vein is located
PW Gain= 14db adjacent to the atrial appendage and enters
the atrium in an oblique anterior to posterior
direction.
PW:3.5MHz o 
The left inferior pulmonary vein, seen by
P P advancing the probe a few centimeters, enters
.60 the atrium in a left lateral to medial direction
A (Fig. 3-12).
o 
T he right pulmonary veins are imaged in
the 0° plane by turning the probe toward
m/s the patient’s right side.
o 
The superior vein enters the atrium in an
anterior-pozsterior direction; the inferior vein
is seen by advancing the probe and enters in a
.60 right lateral to medial direction (Fig. 3-13).
o Pulmonary vein flow is recorded with pulsed
Figure 3-10  Sinus rhythm. Doppler flow patterns in the atrial appendage
Doppler in one or more pulmonary veins,
are recorded with the sample volume in the appendage about 1 cm from the depending on the clinical indication for the
entrance into the LA. In this patient in sinus rhythm, the normal antegrade study (Fig. 3-14).
flow, with a velocity greater than 0.4 m/s after the p-wave, is seen (arrow). o An orthogonal view at 90° also may be helpful,
turning the probe rightward for the right pul-
image plane, keeping the septum centered, monary veins and leftward for the left pulmo-
from 0° to 120° (Fig. 3-8). nary veins (see Fig. 3-12 and 3-13).
o The atrial appendage is imaged using a high
frequency transducer, zoom mode, and a nar- Step 7: Mitral Valve
row sector to improve image resolution (Fig. 3-9). n 
The mitral valve is evaluated starting in the
Simultaneous biplane imaging of the LA append- 4-chamber view and then rotating the image plane
age or sequential images in a rotational scan are slowly to 120° (long-axis view), keeping the valve
recommended for detection of LA appendage centered in the image.
thrombus. n 
Additional views of the mitral valve include the
o F low in the atrial appendage is recorded with transgastric short-axis and 2-chamber view.
a pulsed Doppler sample volume about 1 cm n 
3D imaging of the mitral valve is recommended
from the mouth of the appendage (Figs. 3-10 for myxomatous mitral valve diseases or other dis-
and 3-11). eases of the valve leaflets.
o The pulmonary veins are most easily identified n 
TEE provides optimal evaluation of mitral regur-
using color Doppler with the aliasing velocity gitant severity, allowing a parallel angle between
decreased to about 20 to 30 cm/s. the flow direction and ultrasound beam for CW
Transesophageal Echocardiography  CHAPTER 3 49

LA
LSPV

LAA

LA Figure 3-12  Left pulmonary veins. From the


standard 4-chamber view at 0° (top), the LA ap-
pendage (LAA) and left superior pulmonary vein
(LSPV), with flow into the LA, are visualized by mov-
ing the transducer up in the esophagus and flexing
LSPV the probe tip. There often is a normal prominent
ridge, seen as a rounded mass in this view, be-
tween the atrial appendage and pulmonary vein.
With rotation of the image plane to about 90°, with
the transducer turned toward the patient’s left side,
both left pulmonary veins are visualized. In this
image, superior structures are to the right of the
image and inferior structures to the left. The left in-
ferior pulmonary vein is not clearly seen because it
enters the atrium at a perpendicular angle to the ul-
trasound beam, but color Doppler shows the inflow
signal (arrows). The LSPV enters with the flow di-
rection parallel to the ultrasound beam and is easily
seen with both 2D and color Doppler imaging.

LA LA
RIPV RIPV

RSPV
SVC RSPV

A B C
Figure 3-13  Right pulmonary veins. The right pulmonary veins are identified in the 0° image plane by turning the transducer toward the patient’s right side.
The right inferior pulmonary vein (A) is seen with color Doppler entering the LA at a relatively perpendicular angle to the ultrasound beam. The probe is withdrawn 1
to 2 cm to visualize the right superior pulmonary vein (B), which enters the atrium relatively parallel to the ultrasound beam direction. (C) The right pulmonary veins
also can be imaged in the orthogonal plane by rotating the image plane to a longitudinal view, with the right superior pulmonary vein on the right and the inferior
pulmonary vein on the left.  RIPV, Right inferior pulmonary vein; RSPV, right superior pulmonary vein; SVC, superior vena cava.
50 CHAPTER 3  Transesophageal Echocardiography

Doppler, excellent visualization of the jet origin o 3 D real-time zoom and full volumes imaging
and direction, and accurate measurement of vena are recommended when the valve is abnormal
contracta width and proximal isovelocity surface (Fig. 3-16).
area (PISA) radius. o A second rotational scan is performed using
color Doppler to evaluate for mitral regurgi-
v  KEY POINTS tation. Regurgitation is evaluated based on
o  he image depth is adjusted to just fit the mitral
T
valve on the image. Transducer frequency,
harmonic imaging, and gain are adjusted to
improve the image (Fig. 3-15).
o The mitral valve is first evaluated with 2D
imaging alone to focus on the details of valve
anatomy. A

P3
P1 MVO

P2
S D

a
Figure 3-16  3D imaging of the mitral valve. Mitral valve anatomy is
better appreciated using 3D imaging, with the 3D presentation of the data
providing a more intuitive view with the viewer looking at the left atrial side
Figure 3-14  Pulmonary vein flow. Pulsed Doppler tracing of normal flow of the valve (the surgical view) with the mitral valve orifice (MVO) in diastole
in the left superior pulmonary vein shows systolic (S) and diastolic (D) inflow opening into the left ventricular with the anterior (A) leaflet and three scallops
with a small reversal of flow with atrial (a) contraction. of the posterior leaflet (P1, P2, and P3) well seen.

LA

P3 A2 P2 A2
A2 P2 P1 Ao

LV
LV

A B C
Figure 3-15  Mitral valve. (A) The mitral valve is imaged starting at 0° rotation with the valve centered in the image plane and the depth adjusted to focus on
the valve. The image plane is then slowly rotated, keeping the mitral valve centered, to examine the entire valve apparatus. (B) At about 60° rotation (2-chamber
image plane), the lateral (P1) and medial (P3) scallops of the posterior mitral leaflet and the central segment of the anterior leaflet are typically seen. (C) In the
long-axis view at about 120° rotation, the central segment of the anterior leaflet and the P2 segment of the posterior leaflet are seen.  Ao, Ascending aorta.
Transesophageal Echocardiography  CHAPTER 3 51

measurement of the vena contracta, evaluation o  Regurgitation is evaluated based on vena con-
of pulmonary venous flow pattern, the CW tracta width, calculation of regurgitant vol-
Doppler signal, and quantitative parameters as ume and orifice areas using the PISA method,
discussed in Chapter 12 (Fig. 3-17). and CW Doppler recording of regurgitant
o The transgastric view of the mitral valve offers flow (see Chapter 12).
improved visualization of the subvalvular appa-
ratus, although concurrent evaluation by trans- Step 8: Aortic Valve and Ascending Aorta
thoracic imaging also may be needed. n The aortic valve and proximal ascending aorta
are evaluated in standard long- and short-axis
views.
n Aortic regurgitation is evaluated by color Doppler
in high esophageal views.
v  KEY POINTS
o The aortic valve is best seen in the long-axis

view (at about 120°) and in a short-axis view of
the valve (at about 30° to 50° rotation), using
a shallow depth, high frequency transducer,
zoom mode, and narrow 2D sector (Figs. 3-18
and 3-19).
o From the standard long-axis view, the TEE
probe is turned rightward and leftward to see
the medial and lateral aspects of the valve.
The probe also is withdrawn higher in the
esophagus to see as much of the ascending
aorta as possible.
o From the short-axis view, the probe is slowly
LV
advanced and withdrawn to visualize the areas
immediately inferior and superior to the valve
plane.
o 3D full volume acquisition allows measurement
of cross-sectional area of the aortic annulus
A

.79 49dB 4 •/+1/1/ 1


CW Focus= 42mm
CW Gain= 12db

.79

CW:3.5MHz
Sweep=100
Ao
8.0
MR LV

m/s
B
2.0
Figure 3-17  Mitral regurgitation. A, Color Doppler is used to identify the
presence of mitral regurgitation (MR) and to evaluate severity based on vena Figure 3-18  Aortic valve long-axis view. This view typically is obtained
contracta width (arrow) and by the proximal isovelocity surface area (PISA) at about 120° rotation. The exact rotation angle needed varies between
approach (see Chapter 12). B, The continuous wave Doppler velocity curve ­patients; the image plane is adjusted to the standard image plane based on
also is useful for confirming the identity and evaluating severity of regurgita- anatomy, not a specific rotation angle. Note that the right coronary ostium
tion. is seen in this view.
52 CHAPTER 3  Transesophageal Echocardiography

if needed during evaluation for transcatheter velocity is likely because of a nonparallel


valve implantation (Fig. 3-20). intercept angle between the ultrasound beam
o Aortic regurgitation can be evaluated by color and regurgitant jet (Fig. 3-22).
Doppler, with measurement of vena contracta, o Transthoracic imaging often provides more
although precise quantitation of regurgitant precise quantitation of regurgitant severity.
severity may be difficult on TEE because the
Doppler beam cannot be aligned parallel to
flow (Fig. 3-21).
o CW Doppler of the aortic regurgitant jet
sometimes can be recorded from a trans-
gastric apical view, but underestimation of

LA

NCC L-main

Ao
HR=137

LCC

LV

RCC

Figure 3-19  Aortic valve short-axis view. This view is obtained by cen- Figure 3-21  Aortic regurgitation. Color Doppler in long- and short-
tering the valve in the long-axis image and then rotating the image plane axis images allows detection of aortic regurgitation. Regurgitant sever-
to about 45°. This zoomed image shows the right coronary cusp (RCC), left ity is evaluated by measurement of vena contracta width in the long-axis
coronary cusp (LCC), and noncoronary cusp (NCC) in systole. The left main view. This example shows a narrow jet, consistent with mild regurgitation. 
(L-main) coronary artery is also seen. Ao, Ascending aorta.

V. long axis Short axis

H. long axis 3D planes

Figure 3-20  3D imaging of the LV outflow


tract. Using 3D TEE imaging, the area of
the LV outflow tract can be measured in the
short-axis plane by the 3D volume dataset
(lower right) to obtain vertical (V) and hori-
zontal (H) long-axis views and then move the
orthogonal short-axis plane to a position just
proximal to the aortic valve.
Transesophageal Echocardiography  CHAPTER 3 53

LA

LV

RV
LA Ao

RA

Ao

RPA

Figure 3-23  Left main coronary artery. The left coronary artery is
seen by moving the image plane slightly superior to the aortic valve
Figure 3-22  Transgastric apical view. From a deep transgastric position, short-axis image plane. The left main coronary artery (arrow) ostium
an anteriorly angulated 4-chamber view is obtained by flexion of the probe is seen originating from the aorta just superior to the aortic valve.  Ao,
tip. This image plane does not pass through the true LV apex, with obvious Ascending aorta.
foreshortening of the LV in this image. The ascending aorta (Ao) and right
pulmonary artery (RPA) are seen. 

Step 9: Coronary Arteries


n The left main coronary artery is easily seen in the
short-axis view of the aorta valve (see Fig. 3-19).
n The right coronary artery may be seen in a long-
axis view of the ascending aorta or in the short-
axis view of the aortic valve, but can be identified
in only about 20% of cases (see Fig. 3-18). RA

v  KEY POINTS LV
o  he left main coronary is slightly superior to
T
the aortic valve plane.
RV
o Visualization of the coronary ostium is enhanced
by using a high frequency transducer and zoom
mode.
o The bifurcation of the left main into the left
anterior descending and circumflex coronaries
is frequently visualized, but the more distal ves-
sels are not seen in most patients.
o Identification of the coronary ostium is most
Figure 3-24  TEE view of RV. The RV is seen in the 4-chamber view, but it
important in adolescents and young adults with often is helpful to turn the transducer toward the RV to focus on RV size and
exertional symptoms and in patients with prior systolic function. This patient has moderate RV dilation and systolic dysfunc-
aortic root surgery with coronary reimplanta- tion. Rotation of the image plane allows evaluation of the RV outflow tract in
tion (Fig. 3-23). the short-axis view at the aortic valve level. 

Step 10: Right Ventricle and Tricuspid


Valve
n 
The right ventricle (RV) and tricuspid valve are v  KEY POINTS
evaluated in the high esophageal 4-chamber and o I n the initial TEE 4-chamber images, RV size
RV inflow views (Fig. 3-24). and systolic function are evaluated.
n 
Additional views of the RV and tricuspid valve o The RV also is seen in the short-axis view start-
include the transgastric short-axis view and RV ing at the aortic valve level and slowly advancing
inflow views. the transducer to see the tricuspid valve and RV.
54 CHAPTER 3  Transesophageal Echocardiography

LA

RV
SVC
RA
RA

Figure 3-25  Transgastric view of RV. From the transgastric short-axis Figure 3-26  Bicaval view. A long-axis view of the right atrium is obtained
view, the image plane is rotated to between 60° and 90°. From the 2-cham- with the image plane rotated to 90° and the transducer turned toward the
ber view of the LV, the probe is turned toward the patient’s right side to patient’s right side. The superior vena cava (SVC) enters the atrium near
obtain this view of the RA, tricuspid valve, and RV.  the trabeculated atrial appendage. When the transducer is advanced in the
esophagus, the entrance of the inferior vena cava into the atrium also may
be seen in this view.
o  rom the transgastric short-axis view, the image
F
plane is rotated to 90° and the probe is turned
rightward to obtain a view of the right atrium o The inferior vena cava can be evaluated by

(RA), tricuspid valve, and RV, similar to a trans- advancing the probe slowly toward the gastro-
thoracic RV inflow view (Fig. 3-25). esophageal junction.
o Tricuspid valve anatomy and motion and color o The central hepatic vein enters the inferior vena
Doppler tricuspid regurgitation are evaluated in cava at a perpendicular angle, allowing Doppler
each of these views. recording of hepatic vein flow when indicated.
o A CW Doppler recording of tricuspid regur- o From the standard 4-chamber plane at 0° the
gitant jet velocity may be obtained from the probe is advanced to obtain a low atrial view
esophageal 4-chamber or short-axis view, and the junction of the coronary sinus with the
although underestimation of velocity because RA. The size and flow characteristics of the
of a poor intercept angle is possible. coronary sinus can be evaluated in this view
when needed.
Step 11: Right Atrium
n 
The RA is evaluated in the high esophageal Step 12: Pulmonic Valve and Pulmonary
4-chamber view and in the 90° view of the RA Artery
(Fig. 3-26). n 
The pulmonic valve and pulmonary artery are
n 
Additional views of the RA include a low atrial visualized in a very high esophageal view in the
view, at the level of the coronary sinus, and the 0° image plane or in a 90° image plane with the
transgastric 2-chamber view of the right side of transducer turned toward the left (RV outflow
the heart. view) (Fig. 3-27).
n 
Images of the pulmonic valve may be suboptimal
v  KEY POINTS because the valve is in the far field of the image
o  he RA is visualized by rotating the image
T and it may be obscured by the air-filled bronchus
plane to 90° and turning the probe rightward to at this level of the esophagus.
obtain a longitudinal view of the RA, including
the entrances of the superior and inferior vena v  KEY POINTS
cava. o  he pulmonic valve also may be visualized in
T
o The trabeculated RA appendage may be seen the transgastric short-axis view.
adjacent to the entry of the superior vena cava o Doppler flow in the pulmonary artery can be
into the atrium. recorded from the high esophageal position.
Transesophageal Echocardiography  CHAPTER 3 55

PA
PA

Figure 3-28  Pulmonary artery. The main pulmonary artery (PA) and pul-
Figure 3-27  Pulmonic valve. With the transducer in the high esophageal monary artery bifurcation seen in a very high transesophageal view. This
position, the pulmonary artery (PA) is seen with the image plane rotated to probe position may not be well tolerated in some patients, and this image
90° and the transducer turned slightly toward the patient’s left side. The cannot be obtained in all patients.
anteriorly located pulmonic valve (arrow) is relatively distant from the trans-
ducer, so image quality often is suboptimal.

o  valuation of pulmonic regurgitation with


E
color Doppler is performed in the RV out- DA
flow view. However, transthoracic imaging
of the pulmonic valve often provides more
accurate data.
o The pulmonary artery bifurcation and proxi-
mal right and left pulmonary arteries may be
seen in a high esophageal view, but visual-
ization of more distal pulmonary arteries is Pleural effusion
rarely possible (Fig. 3-28).
o Cardiac magnetic resonance imaging provides
an alternate approach to evaluation of the pul-
monic valve and pulmonary artery.
Step 13: Descending Aorta and Aortic
Arch
n The descending aorta is evaluated in a short-axis
view, starting at the transgastric level, by turning
the probe leftward to identify the vessel and then Figure 3-29  Descending aorta. With the transducer turned toward the
slowly withdrawing the probe to visualize each seg- patient’s left side, the descending thoracic aorta (DA) is imaged in cross
ment of the descending thoracic aorta (Fig. 3-29). section. This patient also has a pleural effusion.
n Once the probe reaches the level of the aortic arch,
the image plane is turned rightward and the probe o I n addition to short-axis images of the descend-
extended to visualize the arch and ascending aorta. ing aorta, the image plane may be rotated to
90° to provide a longitudinal view of the extent
v  KEY POINTS of disease. However, the short-axis view should
o  etween the segment of the ascending aorta visu-
B always be used to ensure that the medial and
alized in the high esophageal long-axis view and lateral aspects of the aorta are examined, which
the aortic arch, there is a segment of the ascend- would be missed in a single longitudinal image
ing aorta that may be missed on TEE imaging. plane.
56 CHAPTER 3  Transesophageal Echocardiography

o Normal structures adjacent to the aorta (con- o Anatomy of the interatrial septum and location







nective tissue, lymph nodes) should not be mis- of the four pulmonary veins
taken for pathologic aortic conditions. o Aortic, mitral, tricuspid, and pulmonic valve




anatomy and function
Step 14: The TEE Report o Abnormalities of the ascending aorta, descending




n The TEE report provides a systematic summary of aorta, or aortic arch



the findings arranged by anatomic structure. o Integration of the data to provide a specific




n The study includes the diagnostic implications of diagnosis (such as “these findings are diag-

/



the findings, notes any limitations of the study, and nostic for endocarditis”) is provided whenever
suggests further evaluation as appropriate. possible.

9
o Any unresolved clinical issues and areas of uncer-
v KEY POINTS




tainty are identified, and specific approaches to

9

The TEE report includes evaluation of: resolving these issues are recommended.

r
o



The TEE report also includes the details of the

i
o LV size and function o






o RV size and function procedure, including informed consent, patient



h
o LA and atrial appendage anatomy and evidence monitoring, medications, and any procedural



for thrombus complications.

ta
e r/
s
/r u
.t c
k a
/: /
s
tt p
h
Transesophageal Echocardiography  CHAPTER 3 57

THE ECHO EXAM

Basic Transesophageal Exam


Basic Transesophageal Echocardiography Exam
Probe Position Rotation Angle Views Focus on
High esophageal 0° 4-chamber • LV size, global and



Set depth to include 60° 2-chamber regional function
LV apex 120° Long-axis • RV size and systolic



function
• LA and RA size



High esophageal Long-axis • Mitral valve



↓ Depth to optimize valves 120° → 0° 2-chamber

i
4-chamber

h
120° Long-axis • Aortic valve



30°-50° Short-axis • Aorta

a


t
0° Depth to show LAA and • LA appendage (resolution



r/
60° pulmonary veins mode, 7 MHz)
90° • Pulmonary veins



0° → 90° Rotational scan • Atrial septum

e


0° 4-chamber • RV

s

90° SVC/IVC view • RA



• SVC and IVC

/r u

0° 4-chamber • Tricuspid valve



60° Short axis • Pulmonic valve and



90° RV outflow pulmonary artery

.t c
3D views 3D real-time and volume Aortic valve • Aortic valve viewed from


acquisition from TEE aortic and LV sides (when
probe position aortic valve pathology is
present)

a
Mitral valve • Mitral valve viewed from

k


LA and LV sides (when

/: /
mitral valve pathology is
present)
Interatrial septum • Interatrial septum

s


viewed from LA side
(when atrial septal

tt p
defect suspected)
­
Transgastric 0° Short-axis • LV wall motion, wall thick-


ness, chamber dimensions

h
• RV size, and function


90° Long-axis • LV and mitral valve


• Turn medially to image


RV and tricuspid valve
Transgastric apical 0° 4-chamber • Useful for antegrade


aortic flow but may still
be nonparallel intercept
angle
Transgastric to high 0° Short-axis descending aorta • Image aorta from the


esophageal diaphragm to aortic arch
IVC, Inferior vena cava; LAA, left atrial appendage; SVC, superior vena cava.
58 CHAPTER 3  Transesophageal Echocardiography

SELF-ASSESSMENT QUESTIONS

Question 1 Question 3
The following Doppler tracing (Fig. 3-30) was acquired In the image provided, name the labeled structures
on TEE imaging. This signal is most consistent with: (Fig. 3-32).
A. Aortic stenosis ________________________________________



B. Aortic regurgitation ________________________________________

/



C. Mitral stenosis ________________________________________



D. Mitral regurgitation ________________________________________

9



E. Tricuspid regurgitation ________________________________________



e

i r 9
h
4.0

a
a
d

r/ t
e
m/s b
c

s
/r u
2.0

Figure 3-30 Figure 3-32


.t c

Question 2 Question 4
You are asked to quantitate mitral regurgitation sever- The structure indicated by the question mark is (Fig.

a
ity in a patient with mitral valve prolapse, and the fol- 3-33):
lowing color Doppler image is obtained (Fig. 3-31). A. Left atrial appendage

k


To complete your evaluation, based on the image pro- B. Left upper pulmonary vein

/: /


vided, you recommend: C. Coronary sinus ostium



A. Record mitral inflow Doppler signal D. Right lower pulmonary vein






B. Color Doppler interrogation of pulmonary veins

s



C. Adjust color Doppler baseline



D. Measure medial to lateral diameter of isoveloc-

tt p



ity hemisphere
?

h LA

LV

Figure 3-31 Figure 3-33




Transesophageal Echocardiography  CHAPTER 3 59

Question 5 Question 6
You are asked to evaluate a patient with a history of a A TEE is ordered in a patient in whom a type A aortic
bioprosthetic aortic valve and suspected aortic regur- dissection is suspected. Imaging of the distal ascend-
gitation. The following image is obtained from a mid- ing aorta is hindered by:
esophageal long-axis view at 130° (Fig. 3-34). A. Ultrasound attenuation in the far-field




Which additional imaging view of the aortic valve B. Interposition of trachea




would aid in visualizing the regurgitant jet? C. Ring-down, or near field clutter artifact

/



A. High-esophageal short-axis view D. Anatomic constraint of esophagus for the TEE






B. Gastric long-axis view probe

9



C. Mid-esophageal 4-chamber view
Question 7



D. Deep gastric short-axis view

9



You are called urgently to the operating room to aid car-

i r
diovascular imaging in a patient after coronary bypass
grafting surgery who has just been taken off the car-

h
diopulmonary bypass pump. The anesthesiologist has
noted new ST segment elevation on the electrocardio-

a
gram monitor. You note heterogeneity in wall motion

t
in the indicated area (arrows, Fig. 3-35) and conclude:

r/

A. Left anterior descending artery distribution


ischemia
B. Cardiogenic shock and global myocardial ischemia

e



C. Left circumflex artery distribution ischemia

s



D. Right coronary artery distribution ischemia



/r u
.t c
A

k a
/: /
s
tt p
Figure 3-35

h
B
Figure 3-34

60 CHAPTER 3  Transesophageal Echocardiography

Question 8 Question 9
A TEE is performed in a critically ill patient who has A TEE is performed to evaluate for endocarditis. After
hypoxia despite mechanical ventilator support. The recording the transgastric view, the probe is withdrawn
image taken from a standard bicaval view is shown in back into the esophagus. In this position, there is resis-
Figure 3-36. Additional imaging with slight clockward tance to further withdrawal of the probe. The next best
rotation of the probe is shown in Figure 3-37. step would be to:
These images are most consistent with: A. Withdraw the probe




A. Anomalous pulmonary vein B. Retroflex the probe

/






B. Sinus venosus septal defect C. Rotate the probe






C. Patent foramen ovale D. Advance the probe

9






D. Thrombus in transit
Question 10



9
A 66-year-old man presents with dyspnea. Electrocar-

i r
diography demonstrates newly diagnosed atrial fibril-
lation and a TEE is ordered to evaluate for left atrial

h
appendage thrombus before direct current cardiover-
sion (Fig. 3-38). What abnormality is identified in the

a
left atrial appendage?

t
A. Reverberation artifact

r/



B. Left atrial appendage thrombus



C. Atrial appendage trabeculation



D. Spontaneous echo contrast

e



s
/r u
Figure 3-36

.t c
k a
/: /
s
tt p
Figure 3-38

h
Figure 3-37

Transesophageal Echocardiography  CHAPTER 3 61

ANSWERS

Answer 1: B The PISA radius is the distance from the hemi-


This is a CW Doppler tracing (not velocity scale) sphere edge to the valve plane, not the medial to
with a maximum velocity about 4 m/s and with flow lateral (parallel to valve plane) diameter of the hemi-
occurring in diastole, directed towards the trans- sphere. The ROA is then calculated using the veloc-
ducer, consistent with aortic regurgitation. Aortic ity time integral from the regurgitant jet. To make

/
stenosis, mitral regurgitation, and tricuspid regur- this measurement, a spectral Doppler tracing of the
gitation might have a similar velocity, but all occur mitral regurgitant signal (not the mitral inflow Dop-

9
in systole. Mitral stenosis would be a lower velocity pler signal) is needed. A semi-quantitative evaluation
diastolic signal, usually directed away from the trans- of regurgitation severity is a demonstration of systolic

9
ducer from a high TEE 4-chamber view. The tracing flow reversal in the pulmonary veins. This requires

i r
was obtained in a patient with a bicuspid aortic valve spectral Doppler imaging, not color Doppler interro-
where the aortic regurgitant jet was directly posteri- gation of pulmonary venous flow.

h
orly (Fig. 3-39).

ta
e r/
s
/r u
4.0

.t c
a
m/s

2.0

/: / k
s
Figure 3-39

tt p
Figure 3-40

Answer 2: C
Assessment of mitral regurgitation severity requires
visualization of the regurgitant jet from multiple

h
views. Quantitation of severity may include calcula-
tion of the regurgitant orifice area (ROA) using the
PISA measurement. To do this, the color Doppler
baseline should be moved in the direction of flow. In this
example, the baseline was moved opposite to regurgi-
tant flow (an error), seen in the upper right indica-
tor with the baseline moved downward. Once a clear
isovelocity hemisphere is identified, the PISA radius
should be measured and the aliasing velocity recorded
(Fig. 3-40).
62 CHAPTER 3  Transesophageal Echocardiography

Answer 3 lower left side of the image, the tricuspid valve is seen,
adjacent to the right ventricle (c). Additional imaging
A. Superior vena cava
from this patient revealed a small secundum atrial sep-



B. Right atrial appendage
tal defect (Fig. 3-41, A and B).



C. Right ventricle



D. Inferior vena cava
Answer 4: B



E. Left atrium



This image is taken from the mid-esophageal win- The entrance of the pulmonary veins to the left
dow slightly rotated from a bicaval view. The indicator atrium is easily seen by TEE. The left upper pulmo-

/
in the upper left hand portion of the image shows it nary vein is typically the easiest to image, and enters

9
is recorded from 126°. The inter-atrial septum bisects the left atrium superior to the left atrial appendage. A
the image. Above the inter-atrial septum is the left ridge of tissue separates the left upper pulmonary vein

9
atrium (e). Entering the right atrium from the right and the atrial appendage. The left atrial appendage

r
side of the image is the superior vena cava (a), and the lies just superior to the posterior mitral valve leaflet

i
inferior vena cava (d). At the lower right side of the long the atrial wall, as seen in this image taken from
image, the right atrial appendage (b) is seen. At the the same patient (Fig. 3-42). The left lower pulmonary

a h
r/ t
s e
/r u
.t c
a
A B

k
Figure 3-41

/: /

s
tt p
LUPV

h
LAA RLPV

RUPV

A B
Figure 3-42

Transesophageal Echocardiography  CHAPTER 3 63

vein is best seen with the probe at 0° and the probe the aorta is obscured due to poor ultrasound penetra-
slightly advanced, where it enters the left atrium at a tion through tracheal air.
horizontal angle from the transducer. The right pul-
monary veins are also seen best with the probe at 0°. Answer 7: A
The coronary sinus drains into the right atrium, adja- The image is taken from a long-axis gastric view of
cent to the IVC, and is best seen in the 0° view with the left ventricle. In this view, the inferior wall is closer
the probe advanced to the gastroesophageal junction. to the transducer and the anterior wall is in the far
field. New wall motion abnormalities in the anterior
Answer 5: B

/
wall are consistent with a left anterior descending
The image shows the aortic valve in long axis. There artery coronary distribution.

9
is significant acoustic shadowing from the valve
occluders onto the left ventricular outflow tract Answer 8: B

9
(LVOT), and the regurgitant jet is not clearly seen. This image is taken from a bicaval view (100° mid-

i r
Most esophageal views of mechanical aortic pros- esophageal view) where the SVC and IVC are seen
theses show acoustic shadowing of the LVOT and at their entry into the right atrium. In this case, color

h
leaflets. Gastric views allow visualization of the valve Doppler flow is shown just proximal to the insertion
prosthesis from a vantage point “below” the LVOT. of the SVC into the left atrium, consistent with a sinus

a
A gastric long-axis view (120° would be needed to venosus atrial septal defect. A patent foramen ovale,

t
image the regurgitant jet in the LVOT (Fig. 3-43). if present, is a common finding, and is located in

r/
In the image below, turbulent flow in the LVOT is the fossa ovalis in the mid-part of the interatrial sep-
seen with color Doppler imaging just below the mitral tum. An anomalous pulmonary vein would not cause
valve (right side of image). From this view, the regur- hypoxia because the pulmonary vein enters into the

e
gitant jet is also aligned parallel to flow for Doppler right atrium (left to right shunt only), with no chan-

s
interrogation. nel that would allow right to left intracardiac flow. An
anomalous pulmonary vein is identified by showing

/r u
the absence of one view entering the left atrium and
then identifying the location of the vein entry into the
right atrium, SVC or IVC. This patient is also inci-
dentally noted to have mild lipomatous hypertrophy

.t c
of the base of the interatrial septum (adjacent to the
SVC), a benign finding, and is not consistent with
intracardiac thrombus.

a
Answer 9: D

k
In the transgastric view, optimization of the short-

/: /
axis view of the left ventricle typically involves flex-
ion of the transducer for superior angulation of the
probe tip. Once completed, flexion of the probe tip

s
should be relaxed before withdrawal back into the
esophagus; otherwise, the tip may be withdrawn in

tt p
the fully flexed/folded position. In this probe posi-
tion, withdrawing, retroflexing, or rotating the probe
further may perforate the esophagus. The esophagus
is too narrow to correct a folded probe within the

h
esophagus. The probe should be readvanced to the
stomach where the tip can be relaxed. If a folded
TEE probe is suspected, chest radiography may be
Figure 3-43 used to confirm the suspicion before further probe
manipulation.

Answer 6: B Answer 10: B
The trachea at the level of the distal ascending In patients with atrial fibrillation undergoing evalu-
aorta anatomically lies between the esophagus and ation for DC cardioversion who have not been on
the aorta. Near-field clutter artifacts and ultrasound chronic anticoagulation, TEE is needed to visual-
attenuation are not prominent factors hindering ize the left atrial appendage. The appendage should
imaging of the aorta. There is no anatomic constraint be visualized from several views. This image was
in the esophagus to view the aorta, which does lie in taken from the 90° mid-esophageal view. The atrial
the imaging sector; rather, imaging of this portion of appendage thrombus is seen in the mid-right hand
64 CHAPTER 3  Transesophageal Echocardiography

part of the image, with a globular echodensity fill- protruding along the lateral wall of the appendage.
ing the distal half of the appendage. Spontaneous Because trabeculations are contiguous with the atrial
echo contrast appears as a mobile, swirling signal, wall, contractile motion of the trabeculations should
and suggests decreased blood velocity or stasis of be seen with atrial activity. Spontaneous echo con-
flow, which has not consolidated into frank throm- trast appears as swirling echodensity within the body
bus. In the image above, spontaneous echo contrast of the appendage and is consistent with low-velocity
is seen in the mid-portion of the left atrium, just flow. A reverberation artifact from the ridge between
above the mitral valve, but not in the appendage. the appendage and the left upper pulmonary vein

/
Often, spontaneous echo contrast does coexist with a are common and often difficult to differentiate from
true appendage thrombus. Atrial trabeculations are a thrombus. Artifact is more likely if the abnormality

9
atrial muscle seen in cross section, commonly seen cannot be demonstrated from multiple image planes.

i r 9
a h
r/ t
s e
/r u
.t c
k a
/: /
s
tt p
h
4 Advanced Echocardiographic Modalities
STRESS ECHOCARDIOGRAPHY Strain and Strain Rate
THREE-DIMENSIONAL ECHOCARDIOGRAPHY Tissue Doppler Velocity Strain Rate and Strain

 
Image Acquisition and Display Speckle Tracking Strain Imaging

/
Examination Protocol Dyssynchrony
Quantitation from 3D Datasets CONTRAST ECHOCARDIOGRAPHY

9
CLINICAL UTILITY INTRACARDIAC ECHOCARDIOGRAPHY
MYOCARDIAL MECHANICS POINT OF CARE ECHOCARDIOGRAPHY

9
Basic Principles THE ECHO EXAM

r
Tissue Doppler Velocities SELF-ASSESSMENT QUESTIONS

h i
ta
r/
to examine the 3D image volume in any tomo-
STRESS ECHOCARDIOGRAPHY graphic plane, typically with three orthogonal
Stress echocardiography is useful for diagnosis and planes used to select the optimal site for mea-

e
n



patient management in patients with: surement (Fig. 4-2).

s
• Coronary artery disease (see Chapter 8) o The simultaneous multiplane mode allows dis-





• Aortic valve stenosis (see Chapter 11) play of two or more 2D image planes in real-

/r u


• Mitral valve stenosis or regurgitation (see Chapters time (Fig. 4-3). 3D color Doppler imaging allows


11 and 12) acquisition of a 3D volume of Doppler data at a
• Hypertrophic cardiomyopathy (see Chapter 9) slower frame rate than 2D imaging.


The specific stress modality depends on the clinical

.t c
diagnosis and type of information needed for decision Examination Protocol
making (Table 4-1). n A systematic protocol for 3D imaging ensures a



complete examination (Table 4-2).

a
n A full volume acquisition of the left ventricle (LV)



THREE-DIMENSIONAL ECHOCARDIOGRAPHY allows quantitation of LV volumes, ejection frac-

k
tion, and regional wall motion.

/: /
Image Acquisition and Display • Standard orientation of 3D-zoom real-time images


n Echocardiographic data can be acquired and dis- for evaluation of valve anatomy facilitate recognition



played in a three-dimensional (3D) format by: of anatomic abnormalities

s
• Integration of data from multiple two-dimensional • The orientation of the aortic valve on 3D imaging is




(2D) images of known spatial location with the right coronary cusp located inferiorly when

tt p
• Use of a transducer that acquires a volume of echo- viewed from either the aortic or LV side of the valve


cardiographic data • The orientation of the mitral valve is with the ante-


• Use of a transducer that simultaneously records rior mitral leaflet at the top of the image when


h
more than one 2D image plane viewed from the LV or left atrium (LA) side of the
• Reconstruction borders traced on 2D images in a 3D valve


image format • The interatrial septum is shown with the right


n 3D echocardiography facilitates recognition of upper pulmonary vein at 1 o’clock when viewed



complex intracardiac spatial relationships. from the LA side. From the right atrium (RA) side,
the superior vena cava is at the 11 o’clock position
v KEY POINTS
 

o Realtime narrow sector 3D imaging provides a Quantitation from 3D Datasets



beat-by-beat image similar to a 2D image but n LV volumes, ejection fraction, and regional wall



with a thicker image plane. motion are measured with 3D imaging.
o Realtime 3D-zoom volume rendered images • A full volume acquisition that includes the entire LV





show a full volume image rotated to show the is obtained.
structure of interest in real time (Fig. 4-1). • Endocardial borders are automatically identified and


o Full-volume gated acquisition volumes are then adjustment as needed by the echocardiographer



used for quantitative analysis with the ability (Fig. 4-4).

65
66 CHAPTER 4  Advanced Echocardiographic Modalities

TABLE 4-1 Clinical Applications of Stress Echocardiography


 
Clinical
Indication Stress Modality Protocol Interpretation
Detection or Exercise • Maximum treadmill exercise provides • Normal wall motion at rest and a





evaluation highest workload, with image ac- regional wall motion abnormality
of coronary quired immediately after exercise; or with stress indicates ischemia.
artery disease • Supine bicycle exercise (allows con- • Abnormal regional wall motion at





/
tinuous imaging). rest that persists with stress indi-
• Compare rest and stress cine loop cates prior infarction.


9
images of the LV in standard views.
Pharmacologic • Dobutamine is infused beginning at • Normal wall motion at rest and a

9




low dose (5 or 10 μg/kg/min), increas- regional wall motion abnormality

r
ing by 10 μg/kg/min every 3 minutes with stress indicate ischemia.

i
to a maximum dose of 40 μg /kg/min • Abnormal regional wall motion at



or target heart rate of 85% maximum rest that persists with stress indi-

h
predicted. cates prior infarction.
• Atropine may also be used to achieve

a


target heart rate.

t
• Comparison of rest versus peak

r/


stress cine loop images of the LV in
standard views.

e
Myocardial Dobutamine • Dobutamine is infused beginning at • Viability is diagnosed when an area




viability stress low dose (5 μg/kg/min) and increasing of hypokinesis or akinesis at rest

s
­
to 10 μg/kg/min. shows improved wall motion at low-
• The stress test may be continued to dose dobutamine.

/r u


evaluate for ischemia as above. • If wall motion again worsens at high-


• Images of the LV in cine loop format er dobutamine doses, ischemia also


at baseline and low-dose dobuta- is present (the biphasic response to
mine (increase in contractility with no stress).

.t c
change in heart rate) are compared.
Postcardiac Dobutamine • Use standard dobutamine stress echo • A new wall motion abnormality with




transplant stress protocol. stress is consistent with inducible
myocardial • Compare rest versus peak stress cine ischemia.

a


ischemia loop images of the LV in standard • Balanced ischemia (equal involve-


k
views. ment of all major coronary arteries)

/: /
or small-vessel disease may be
missed on stress echocardiography.
Low output AS Dobutamine • Measure stroke volume and ejection • Severe AS is present if aortic veloc-




stress fraction as dobutamine is increased ity increases to at least 4 m/s and

s
from 0 to 20 μg/kg/min in 5 μg/kg/min valve area remains less than 1 cm2.
increments. • Failure of stroke volume or EF to

tt p


• Measure AS velocity, mean gradient, increase by at least 20% is termed


and valve area at each stress level. “lack of contractile reserve” and
• Stop for symptoms or when a hemo- connotes a poor clinical outcome.


h
dynamic endpoint is reached.
Mitral valve Exercise stress • Measure TR jet velocity at baseline • The primary goal is to assess peak




disease and at peak exercise stress on maxi- PA pressure with exercise (and
mum treadmill testing or with supine change from baseline), calculated
bicycle exercise. from the TR jet velocity.
• The transmitral pulsed or CW Doppler • With MS, the transmitral velocity




velocity curve also may be evaluated and mean gradient will increase as
at rest and with exercise. expected for the increase in flow
• MR may be evaluated using CW and rate; this measurement is rarely


color Doppler (optional). diagnostically useful.
• With primary MR, severity may


increase with exercise (e.g., with
mitral prolapse) but quantitation at
peak exercise is challenging. The
change in PA pressure is a surrogate
for the increase in regurgitation.
Advanced Echocardiographic Modalities  CHAPTER 4 67

TABLE 4-1 Clinical Applications of Stress Echocardiography—cont’d


Clinical
Indication Stress Modality Protocol Interpretation
Hypertrophic Exercise stress • Supine bicycle stress is preferred for • Latent LV outflow obstruction is
cardiomyop­ evaluation of HCM because it allows present when the resting sub-
athy (HCM) data recording at each stress level. aortic gradient is < 30 mmHg but
• LV outflow velocity is recorded with increases to > 30 mmHg with stress.
pulsed and CW Doppler at baseline • Separating the LV outflow signal
and with stress. from the higher-velocity MR signal
• MR also is evaluated with CW and can be challenging in some cases.
color Doppler. • Useful features in identifying the
origin of the Doppler signal are tim-
ing of flow onset relative to the QRS
signal, shape of the velocity curve,
delineation of a smooth dark edge
to the velocity curve, and recordings
showing separate LV outflow and
MR CW Doppler flow curves.

AS, Aortic stenosis; EF, ejection fraction; HCM, hypertrophic cardiomyopathy; MS, mitral stenosis; MR, mitral regurgitation; PA, pulmonary
artery; TR, tricuspid regurgitation.

Long axis

N L

Short axis
R

Figure 4-1  3D zoom of the aortic valve. On TEE imaging, a 3D real time zoom image of the aortic valve is obtained with the long- and short-axis 2D views
and the 3D view from the aortic side of the leaflet showing the right (R), left (L), and noncoronary (N) leaflets.

• 
A moving 3D surface rendered image is provided • 
A full volume 3D dataset is acquired with several
with color coding for regional systolic function cardiac cycles “stitched” together to form the full
(Fig. 4-5). image.
• 
A graphical display of motion versus time allows • 
Stitch artifact is avoided by having the patient stay
evaluation of regional function and ventricular still and suspend respiration; data acquisition should
synchrony. be repeated if a discontinuity between image seg-
• 
Multiple short-axis views of the LV can be displayed ments is present.
simultaneously to enhance diagnosis of the presence • 
The 3D dataset is examined in the x, y, and z image
and extent of regional wall motion abnormalities planes to identify the valve orifice and then align an
(Fig. 4-6). image plane at the minimum orifice.
n  easurements of stenotic valve orifice areas are
M • 
The valve area then can be measured on the cor-
made from full volume 3D data acquisition. rectly aligned 2D image.
68 CHAPTER 4  Advanced Echocardiographic Modalities

Figure 4-2  3D measurement of atrial septal defect diameters and area. A full volume 3D acquisition is used for quantitative measurements. The
4-chamber (upper left) and bicaval (upper right) image planes are used to measure the diameter of the septal defect in orthogonal views. An en face
view of the entire defect (lower left) allows visualization of the typical oval shape of a secundum atrial septal defect. This view can be used to measure
the area of the defect. A 3D representation of the angles between the image planes is shown in the lower right panel. ASD, Atrial septal defect; LA, left
atrium; RA, right atrium.

LA

Figure 4-3  Biplane imaging. Simultaneous imaging of the left atrial appendage using biplane imaging allows evaluation for atrial thrombus. Normal pec-
tinate muscles in the appendage are seen (arrow).
Advanced Echocardiographic Modalities  CHAPTER 4 69

TABLE 4-2 ASE/EAE Recommendations for a Systematic 3D Study


Transthoracic Image Transesophageal Image Sequence for TEE full Volume
Acquisition Acquisition Image Orientation (Fig. 4-7)
Aortic valve PLAX with and without 60° mid-esophageal short-axis with 2D views at 60° and 120° with
color, narrow angle and without color, zoomed or full aortic valve centered in acquisition
and zoomed volume boxes.
120° mid-esophageal long-axis with Live 3D to optimize gain.
and without color, zoomed or full Full volume acquisition, and then
volume ­rotate 90° clockwise around
y-axis.
Mitral valve PLAX with and without 0°-120° mid-esophageal with and 2D views at 90° and 120° with mitral
color, narrow angle without color, zoomed valve centered in acquisition
and zoomed boxes.
A4C with and without Full volume acquisition, rotate 90°
color, narrow angle counter-clockwise around x-axis
and zoomed and then 90° counter clockwise in
plane so aortic valve is superior.
LV A4C, narrow and wide 0°-120° mid-esophageal view in- Full volume images are used to
angle cluding entire LV, full volume generate a surface rendered image
for quantitation of LV volumes,
ejection fraction, and regional wall
motion. These data are displayed
as a moving 3D surface rendered
image with color coding and as a
time graph.
RV A4C with image tilted 0°-120° mid-esophageal view tilted
to put RV in center of to put RV in center of image, full
image volume
Atrial A4C, narrow angle and 0° with probe rotated toward atrial
septum zoomed septum, zoomed or full volume
Pulmonic RV outflow view with 90° high-esophageal view with and 2D high esophageal view at 0°
valve and without color, without color, zoomed with pulmonic valve ­centered in
narrow angle and 120° mid-esophageal 3-chamber acquisition box.
zoomed view with and without color, Full volume acquisition, rotate
zoomed 90° counter-clockwise around
x-axis, then rotate in plane 180°
counter clockwise so anterior
leaflet is superior.
Tricuspid A4C with and without 0°-30° mid-esophageal 4-chamber TTE 2D views in off-axis A4C view
valve color, narrow angle view with and without color, with tricuspid valve centered in
and zoomed zoomed acquisition boxes.
RV inflow view with an 40° transgastric view with anteflexion Full volume acquisition, rotate 90°
without color, narrow with or without color, zoomed counter-clockwise around x-axis
angle and zoomed and then rotate 45° in plane so
septal leaflet is in the 6 o’clock
position.
From Otto CM: Textbook of Clinical Echocardiography 5 ed, Elsevier, 2013, Philadelphia.
A4C, Apical 4-chamber; ASE, American Society of Echocardiography; EAE, European Association of Echocardiography; PLAX, parasternal long axis.
70 CHAPTER 4  Advanced Echocardiographic Modalities

Figure 4-4  3D LV volume measurement. A full volume acquisition of the LV from a TTE apical window is used. Standard image planes in 4-chamber
(upper left), 2-chamber (upper right), and short-axis (lower left) views allow review and editing of automated border detection. The reconstruction 3D
LV volume is shown in the lower right panel. Data for the entire cardiac cycle are shown as beating images and a graph of LV volume versus time is
shown at the bottom for one cardiac cycle.

Regional (%R-R) Report Page

Tmsv 16-SD* 3.90 %


Tmsv 12-SD 2.77 %
Tmsv 6-SD 2.46 %
Tmsv 16-Dif 14.55 %
Tmsv 12-Dif 9.42 %
Tmsv 6-Dif 6.83 %
Tmsv 3-6 4.93 %
Tmsv 3-5 3.12 %
Tmsv Sel-SD %
Tmsv Sel-Dif %
R-R Time 769 ms

*Excludes segment 17

Figure 4-5  3D analysis of regional ventricular function. Regional wall motion is displayed as a beating 3D heart and as a graph of volume versus time with
color coding for each myocardial segment. TMSV, Time to minimum systolic volume.
Advanced Echocardiographic Modalities  CHAPTER 4 71

Figure 4-6  Multiple simultaneous short-axis views of LV with 3D imaging. A full volume 3D acquisition is used to generate multiple short-axis views of
the LV for evaluation of regional function.

• 
LV outflow tract or aortic annulus area and circum- o  otation is the circular motion of the LV myocar-
R
ference also can be measured by this approach. dium around its long axis, measured in degrees.
o The LV apex and base rotate in opposite direc-
tions during contraction; the absolute differ-
CLINICAL UTILITY ence in rotation between the apex and base is
n Routine use of 3D imaging is recommended for: “twist” and the gradient in rotation angle from
• 
Quantitation of LV volumes and ejection fraction base to apex is “torsion.”
• 
Evaluation of mitral valve anatomy (valve area in
mitral stenosis) Tissue Doppler Velocities
• 
Guidance of transcatheter procedures n Tissue Doppler measures the velocity of myocar-
n 3D imaging also is useful for: dial motion, displayed as a velocity curve for a sin-
• 
Evaluation of mitral valve anatomy before and after gle point or as a color display across the 2D image
mitral valve repair in patients with mitral regurgi­ plane.
tation n Tissue Doppler velocities are measured relative to
• 
Visualization of the size and shape of atrial septal the position of the transducer, like all Doppler sig-
defects in patients undergoing transcatheter closure nals, so that accurate measurements depend on a
(Table 4-3) parallel alignment between the Doppler beam and
direction of motion.

MYOCARDIAL MECHANICS v  KEY POINTS


o  ecording tissue Doppler velocity at a single
R
Basic Principles location, for example, adjacent to the mitral
n 
LV function is incompletely described by simple annulus for evaluation of diastolic function,
measures such as ejection fraction or diastolic fill- provides a standard velocity versus time spectral
ing patterns. display output.
n 
Myocardial strain, strain rate, and measures of o Tissue Doppler velocities also can be displayed
synchrony attempt to provide an integrated, pre- for multiple points across the 2D image using a
cise description of ventricular contraction and color display, analogous to color Doppler flow
relaxation. mapping for blood flow velocities.
o Tissue Doppler signals are high amplitude and
v  KEY POINTS low velocity. Recording requires adjusting the
o  V contraction occurs simultaneously in the
L instrument settings with:
­longitudinal, radial, and circumferential direc- A low
o velocity range (usually +/– 0.2 m/s)
tions. Very
o low gain and wall filter settings
72 CHAPTER 4  Advanced Echocardiographic Modalities

TABLE 4-3 Clinical Applications of 3D Echocardiography


Application 3D Approach Comments
LV function Surface rendered LV volumes, ejection fraction, 3D echo underestimated LV volumes com-
and regional wall motion derived from a gated full pared to MRI data
volume 3D acquisition. Trabeculae and papillary muscles are
included in the LV chamber
RV function Volume rendered images allow visualization of entire 3D measurement of RV volumes and ejec-
RV. tion fraction requires further validation
Surface rendered images may allow measurement of but is a promising approach.
volumes and ejection fraction.
Mitral valve Volume rendered images show mitral valve anatomy 3D TEE is recommended for guidance of
en face from the LA or LV side of the valve. interventional mitral valve procedures.
Accurate measurement of valve areas in mitral 3D TTE or TEE is recommended for clinical
stenosis using 3D guided 2D image planes. evaluation of mitral valve pathology.
Annular shape and dimensions from volumetric
images.
3D color Doppler shows jet origin and direction.
Aortic valve and Volume rendered images obtained from TTE 3D imaging may be helpful in determining
sinuses parasternal or TEE high esophageal views provide the mechanism of aortic regurgitation
optimal spatial resolution. and defining the number of valve leaflets.
Planimetry of aortic valve area is possible on 2D 3D imaging is recommended for guidance
images derived from the 3D full volume dataset. of transcatheter aortic valve implantation.
3D images demonstrate the ovale shape of the aortic
annulus.
Pulmonic valve The pulmonic valve can be imaged using biplane or Routine 3D pulmonic valve imaging is not
and pulmo- real time 3D imaging. recommended.
nary artery
Tricuspid valve 3D volume rendered images of the tricuspid valve are 3D views of the tricuspid valve may be
acquired similar to those for the mitral valve. helpful in determining the mechanism of
valve regurgitation.
LA and RA 3D volume rendered images of the atrial septum are 3D imaging may improve assessment of
helpful for defining the location, size, and shape of LA volume but is not a routine measure-
atrial septal defects and for guiding transcatheter ment.
closure procedures.
Left atrial 3D volume rendered images are helpful in guiding Biplane imaging of the LA appendage is
appendage transcatheter LA appendage closure. useful to evaluate for LA thrombus
3D stress echo- 3D imaging provides simultaneous evaluation of Disadvantages of 3D stress imaging
cardiography wall motion in all myocardial segments, improved include lower frame rates and spatial
visualization of the L apex, and rapid image resolution compared to 2D imaging. Not
acquisition at peak stress. all 3D systems allow side-by-side review
of rest and stress images.
Summarized from Lang RM, et al: EAE/ASE recommendations for image acquisition and display using three-dimensional echocardiography. J Am
Soc Echocardiogr 25:3-46, 2012. See also Otto, CM: Textbook of Clinical Echocardiography, ed 5, Elsevier, 2013, Philadelphia.)

o  issue Doppler velocity is recorded in the api-


T velocity is directed away from the apex with the
cal 4-chamber view with a 2-mm sample volume velocity reflecting LV systolic function (Fig. 4-7).
positioned in the septal myocardium about 1 cm
apical from the mitral annulus. The lateral annu- Strain and Strain Rate
lus can be used if septal motion is abnormal. n 
Strain rate (SR) is the rate of change in myocar-
o The normal tissue Doppler velocity curve shows dial length, normalized for the original length
an early diastolic velocity (E′) toward the apex, calculated from the difference in velocities at two
followed by a late diastolic velocity (A′) reflect- myocardial sites (V1 and V2), and divided by the
ing atrial filling. In systole (S), the myocardial distance (D) between them (units are s−1).
Advanced Echocardiographic Modalities  CHAPTER 4 73

ECG strain rate. Thus, the strain rate curve looks


S
like a mirror image of a tissue Doppler velocity
4 curve (see Fig. 4-7).
With
o Doppler tissue velocity data, strain is cal-
culated by integrating the strain rate curve over

cm/s
0
time.
With speckle tracking echocardiography (STE),
o

T 4 A strain is measured directly from the change in dis-


E
tance between two points in the myocardium.
Tissue doppler o  lobal longitudinal strain (GLS) is a measure of
G
LV systolic function with a normal peak GLS
V3
V2 V2  V1 of about −20% (normal range −16% to −22%)
SR  and is relatively insensitive to changes in load-
V1 D
2 E ing conditions.
o Strain is analogous to ejection fraction (change
in length versus volume over time), and the
S1

0 strain curve is similar in shape to a ventricular


volume curve.
o Regional LV function can be evaluated using
Apical 4C view 2 S strain rate or strain imaging, as well as global
Strain rate LV systolic function.
o Speckle tracking GLS is preferred over tissue
Strain  SR Doppler velocity derived GLS as it is more
0 ED reproducible and is not angle dependent.

A
Tissue Doppler Velocity Strain Rate
10
and Strain
%

20 E n  Using tissue Doppler velocities, strain is typically


measured from the apical view, reflecting longitu-
30 dinal LV function, using 3 sample volumes placed
ES about 12 mm apart.
Strain n The tissue Doppler velocity curves should show a

Figure 4-7  Schematic diagram of the derivation of strain rate and clear signal without aliasing and with avoidance of
strain from myocardial tissue velocities. From the apical view at least blood pool signals.
three Doppler sample volumes are positioned in the myocardium about 12 n The instrument calculates and displays strain rate
mm apart. The three graphs on the right show one cardiac cycle, matched in units of s−1.
for timing as shown by the electrocardiogram (ECG) at the top. The tissue
Doppler tracings show mean velocity versus time with the line colors cor- Speckle Tracking Strain Imaging
responding to each sample volume position. Strain rate (SR) is calculated
for each time point at the change in velocity (V) between each two-sample n Speckle tracking uses small reflectors in the myo-
volume positions, divided by the distance (D) between them. Strain is deter- cardium to track motion, allowing calculation of
mined by integration of the strain rate to generate a curve similar to a left
ventricle volume curve with a rapid decrease in strain during ejection (ED to
LV strain.
ES) and a rapid increase in strain in early diastole (E) with another increase n Unlike tissue Doppler, speckle tracking strain is not
in late diastole after atrial contraction (A). dependent on the angle between the ultrasound
beam and direction of motion.
SR = (V2 − V1 )/D(4-1) v  KEY POINTS
n Strain is a measure of deformation of a mate- o  peckle tracking provides a direct measure of
S
rial, defined as the difference between the origi- strain defined as the change in length of myo-
nal length (Lo) and final length (L), expressed as a cardium relative to the original length.
percent (units are a dimensionless percent) of the o Speckle tracking is displayed as a 2D
original length: color-coded image, alongside a graphic display
Strain = [(L − L0 ) /L0 × 100% (4-2)
of strain for different myocardial segments

(Fig. 4-8).
o Longitudinal strain can be measured from api-
v  KEY POINTS cal views, circumferential strain from short-
o  yocardial shortening (systole) is a negative
M axis views, and radial strain from various 2D
strain rate. Lengthening (diastole) is a positive views.
74 CHAPTER 4  Advanced Echocardiographic Modalities

Figure 4-8  Speckle tracking echocar-


diography showing long-axis strains. The
left panel shows the typical strain pattern
from a normal LV. The right panel shows re-
cordings from a patient with an anterior myo-
cardial infarction. In the apical LV segments
(arrows) there is lengthening during early
systole and there is postsystolic shortening.

Dyssynchrony SALINE CONTRAST


n Dyssynchrony is defined as spatial variation in
the timing of LV contraction, most often seen in
patients with a low ejection fraction.
n Dyssynchrony is qualitatively easily appreciated on
2D imaging; various quantitative measures have
been proposed.
LV
v  KEY POINTS
o  ne measure of dyssynchrony is a difference
O RV
> 130 ms in septal to posterior wall delay,
defined as the interval between the QRS and
the maximum inward motion of the myocar-
dium. This measure is affected by other factors RA
that alter septal motion. LA
o Interventricular dyssynchrony is reflected by a
> 40 ms difference between the LV and right
ventricle (RV) preejection periods (time from
QRS to antegrade aortic or pulmonic flow).
o Tissue Doppler allows 2D display of dyssyn-
chrony and measurement for multiple myocar- Figure 4-9  Right heart saline contrast. Intravenous injection of agitated
dial segments. Dyssynchrony is present when saline provides opacification of the right heart chambers. A small amount
there is a difference of at least 65 ms in the tis- of contrast is seen in the left atrium (arrow) consistent with the presence of
patent foramen ovale.
sue Doppler S-wave peaks between opposing
LV walls in apical 4-chamber or long-axis views.
when transthoracic image quality is subopti-
mal (Fig. 4-10).
CONTRAST ECHOCARDIOGRAPHY o Instrument settings to optimize left heart con-
n Intravenous injection of microbubbles to opacify trast images include:
the cardiac chambers or evaluate myocardial per- A decrease
o in power output (to a mechanical
fusion is called contrast echocardiography. index of about 0.5)
n Agitated saline contrast opacifies the right heart A lower transducer frequency
o
and is used for detection of right to left intracar- o An increase in overall gain and dynamic range
diac shunting based on the appearance of contrast o Focal depth at the mid- or near-field of the
in the left heart. image
n Smaller microbubbles (1 to 5 μm diameter) trans- o  hen microbubble density is too high, exces-
W
verse the pulmonary vasculature, allowing left sive apical contrast results in shadowing of the
heart chamber and myocardial opacification. rest of the ventricle (Fig. 4-11, A)
o A low microbubble density or high mechani-
v  KEY POINTS cal index results in a swirling appearance with
o  he most common use of right-sided contrast is
T inadequate LV opacification (Fig. 4-11, B)
to detect a patent foramen ovale, either on trans- o Left-sided contrast is contraindicated in patients
thoracic or transesophageal imaging (Fig. 4-9). with:
o Left heart contrast typically is used to Right
o to left or bidirectional shunts or
enhance LV endocardial border detection Hypersensitivity
o to echo contrast
Advanced Echocardiographic Modalities  CHAPTER 4 75

BEDSIDE BEDSIDE
Baseline Contrast
SEMI-LLAT SEMI-LLAT

LV
LV

A B
Figure 4-10  Left heart contrast. In this patient with suboptimal endocardial definition in the apical 4-chamber view (left), intravenous injection of a left heart
contrast agent opacifies the left ventricle, providing improved evaluation of ventricular systolic function (right). LLAT, Left lateral view.

Shadowing Swirling

LV LV

A B
Figure 4-11  Contrast imaging artifacts. A, Shadowing of the LV by contrast in the apex is seen. Apical shadowing occurs when the volume or rate of contrast
injection is too high. B, Swirling of ventricular contrast, with poor definition of the endocardium, is seen when the volume of contrast is too low or when the
mechanical index is too high, which results in destruction of microbubbles.

o  aution is needed (with blood pressure, arte-


C
rial oxygen saturation, and ECG monitoring) in INTRACARDIAC ECHOCARDIOGRAPHY
patients with: n I ntracardiac echocardiography (ICE) is performed
Pulmonary
o hypertension or in the cardiac catheterization or electrophysiology
Unstable cardiopulmonary conditions
o laboratory using a small high frequency transducer
o  ssessment of myocardial perfusion by contrast
A (5 to 10 MHz) on the tip of a catheter.
echocardiography is not widely used for clinical n ICE imaging is used to guide percutaneous
diagnosis, although there is ongoing develop- interventions and complex electrophysiology
ment of this approach. procedures.
76 CHAPTER 4  Advanced Echocardiographic Modalities

Patent
o foramen ovale closure
Arrhythmia
o ablation procedures
ICE
Other complex percutaneous procedures
o
o  ome procedures, such as balloon mitral val-
S
RA vuloplasty, can be monitored either by intra-
cardiac, transesophageal, or transthoracic
imaging.

RV POINT OF CARE ECHOCARDIOGRAPHY


n The bedside use of small, portable ultrasound sys-
tems to direct patient care is called “Point of care
echocardiography.”
n The capability of portable instruments ranges

from simple 2D imaging to the full range of func-
tions of a larger ultrasound system.
v  KEY POINTS
o  ppropriate education and training are
A
essential for the appropriate use of handheld
echocardiography.
Figure 4-12  Intracardiac echocardiography (ICE). The catheter-tip o Goals of point of care echocardiography in
transducer is located in the RA with visualization of the trabeculated RA the emergency department are (Table 4-4 and
appendage, tricuspid valve, and RV.
Table 4-5):
Evaluation
o for pericardial effusion
v  KEY POINTS Evaluation
o of overall LV systolic function
o I CE typically is performed by the physician per- Identification of significant right or left ventricu-
o
forming the invasive procedure. lar enlargement
o Manipulation of the transducer-tipped catheter o Visualization of the inferior vena cava as a mea-
requires considerable experience in intracar- sure of central venous pressure
diac procedures. o  ther indications for point of care ultrasound
O
o Images are obtained primarily from the RA, include:
allowing evaluation of the: Cardiology
o counseling outside of health care
Interatrial
o septum facilities (e.g., school screenings)
LA, atrial appendage, and pulmonary veins
o First cardiac evaluation in ambulances
o
o Mitral valve and base of the LV o Interim focused evaluations in outpatients settings
o Tricuspid valve and RV (Fig. 4-12) to follow LV systolic function, size of pericardial
o I ntracardiac echocardiography is used to guide effusion, etc.
procedures including: o Complement to the physical examination in
Atrial
o septal defect closure intensive care patient settings.
Advanced Echocardiographic Modalities  CHAPTER 4 77

TABLE 4-4 Cardiac Mechanics: Approaches and Clinical Applications


Modality Methodology Clinical Applications
Tissue Doppler Measurement of the velocity (cm/s) of • Tissue Doppler myocardial velocities are a standard
imaging motion of the myocardium either as measure of LV diastolic function.
a single point with pulsed Doppler or
over an image plane with color Dop-
pler
Tissue Doppler Tissue Doppler velocities at several sites • SR is a measure of ventricular contractility.
SR and strain or color Doppler across the image are • SR is integrated to determine strain, a measure of
used to measure strain rate: regional myocardial function.
SR = (V2 – V1)/D • Utility of tissue color Doppler is limited by angle
dependence and high signal noise for derived SR and
strain.
Myocardial Strain is measured directly from the • Myocardial STE is angle independent.
speckle motion of myocardial speckles across • STE analysis can be performed after image acquisition.
tracking the 2D image or in 3D as: • STE strain and SR may improve evaluation of LV
strain [(L - Lo)/ Lo] × 100% diastolic function but further validation is needed.
• STE strain and SR can improve accuracy of stress
echocardiography by experts.
Myocardial Multiple 2D, pulsed Doppler, and tissue • The degree of dyssynchrony may predict the re-
dyssynchrony Doppler methods. sponse to biventricular pacer therapy.
LV rotation, Rotation is the circular motion of the • STE-measured abnormalities in LV rotation, twist
twist, and LV myocardium around its long axis, and torsion have been described in patients with
torsion measured in degrees, using STE heart failure, coronary, valve, and pericardial
Twist is the absolute difference in disease.
rotation between the LV base and • Limitations of these measurements include lack of
apex (degrees) standardization of imaging planes and a need to
Torsion is the gradient in rotation angle define normal values.
from base to apex, measured as • Clinical use of this methodology is not currently
degrees per cm recommended.
LV dyssyn- Approaches to measuring interventricular • Currently there is no clear role for echocardiographic
chrony dyssynchrony include M-mode, 2D measures of ventricular dyssynchrony in manage-
tissue Doppler, STE, and 3D echo. ment of heart failure patients.
• In theory, measures of ventricular dyssynchrony
might better predict the response to cardiac resyn-
chronization therapy (biventricular pacing) in patients
with a borderline QRS duration.
From Otto CM: Textbook of Clinical Echocardiography, 5 ed. Elsevier, 2013, Philadelphia.
SR, Strain rate; STE, speckle tracking echocardiography.

TABLE 4-5 Goals of the Focused Cardiac Ultrasound in the Symptomatic Emergency


Department Patient
Assessment for the presence of pericardial effusion
Assessment of global cardiac systolic function
Identification of marked right ventricular and left ventricular enlargement
Intravascular volume assessment
Guidance of pericardiocentesis
Confirmation of transvenous pacing wire placement
From Labovitz AJ, et al. Focused cardiac ultrasound in the emergent setting: a consensus statement of the American Society of Echocardiography
and American College of Emergency Physicians. J Am Soc Echocardiogr 2010;23:1225-30.
78 CHAPTER 4  Advanced Echocardiographic Modalities

THE ECHO EXAM

Advanced Echocardiographic Modalities


Modality Instrumentation Clinical Utility Special Training
3D Echo Volume rendered images • LV volumes, EF, and regional wall • Image acquisition and
Surface rendered LV volumes motion analysis
Simultaneous 2D images • Mitral valve anatomy
• Procedural guidance
Tissue Doppler Tissue Doppler and 2D imaging • Strain rate is a measure of ventricu- • Data acquisition and
strain rate are used to measure strain lar contractility analysis
and strain rate: • Strain rate is integrated to deter- • Clinical interpretation of
SR = (V2 –V1)/D mine strain, a measure of regional data
myocardial function
Myocardial Strain is measured directly from • Myocardial speckle tracking is • Data acquisition and
speckle the motion of myocardial angle independent analysis
tracking speckles as: • Analysis can be performed after • Clinical interpretation of
[(L − L0)/L0] × 100% image acquisition data
Myocardial Multiple 2D pulsed Doppler and • The degree of dyssynchrony is • Data acquisition and
dyssynchrony tissue Doppler methods altered in various disease states analysis
• Clinical interpretation of
data
Contrast echo Microbubbles for right or left • Detection of patent foramen ovale • Intravenous administra-
heart contrast • LV endocardial definition tion of contrast agents
• Knowledge of potential
risks
Intracardiac 5-10 MHz catheter-like intracar- • Interventional procedures (ASD • Invasive cardiology
echo (ICE) diac probe closure) training and experience
• EP procedures
Point of care Small, inexpensive ultrasound • Bedside evaluation by MD for • At least Level 1 echo
ultrasound instruments pericardial effusion, LV global training
and regional function
ASD, Atrial septal defect; EF, ejection fraction; EP, electrophysiology.

American Society of Echocardiography (ASE) and the European Association of


Echocardiography (EAE) Recommendations for the Use of 3D Echocardiography
Clinical Practice Promising Clinical Studies Areas of Active Research
Quantitation of LV volumes and ejection LV mass quantitation LV shape
fraction RV volumes and ejection fraction LV dyssynchrony
Evaluation of mitral valve anatomy Aortic valve anatomy and stenosis Mitral valve regurgitation
Guidance of transcatheter procedures Prosthetic valves
Advanced Echocardiographic Modalities  CHAPTER 4 79

3D Imaging Modalities
Advantages Limitations
Real-time 3D mode—narrow section Rapid acquisition, familiar image planes Narrow sector; entire structure
volume rendered images Image can be rotated does not fit in imaging plane
Helpful with complex cardiac anatomy
Real-time “zoom” volume rendered Shows anatomy in “surgical” views A wider field of view decreases
cropped images Enlarged 3D image of structure of interest spatial and temporal resolution
Full volume gated acquisition for vol- High spatial resolution May be difficult to optimize im-
ume rendered cropped images High temporal resolution age quality for all structures in
Quantitation of LV volumes and ejection the field of view
fraction “Stitch” artifacts occur due to
Provides 3D LV shape and dyssynchrony patient and respiratory motion
Full volume gated acquisition for Accurate measurements of cardiac dimen- Endocardial definition may be
multiple 2D tomographic slices sions suboptimal depending on
More objective and less operator dependent transducer position
than standard 2D imaging
Visualization of all myocardial segments
simultaneously
Simultaneous multiplane 2D imaging Simultaneous images in two defined planes Only two planes visualized
Highest spatial resolution
Highest temporal resolution
3D color Doppler Visualization of 3D geometry of vena con- Slow frame rate with low
tracta and proximal isovelocity surface temporal resolution
area for regurgitant lesions
Location of paravalvular prosthetic leaks
and intracardiac shunts

Level of other training or skills


Physical
exam
Point-of-care
echo Intracardiac
echo (ICE)
Level of echo training

Transthoracic
echo (TTE)
Contrast echo
Stress echo
Transesophageal
echo (TEE)
3D echo
80 CHAPTER 4  Advanced Echocardiographic Modalities

SELF-ASSESSMENT QUESTIONS
Question 1
Exercise echocardiography is a reasonable consider-
ation in patients with:
A. Recent myocardial infarction
B. Symptomatic aortic stenosis
C. Acute aortic dissection
D. Severe systemic hypertension
Question 2
A 54-year-old woman presents with complaints of
episodic dyspnea which can occur both at rest or on
exertion, but is not reliably provoked. She has no car-
diovascular risk factors. TTE demonstrates an ejec-
tion fraction of 64%, and a global longitudinal strain
of −23% (Fig. 4-13). Based on the speckle tracking
echocardiographic image and strain data provided,
what additional testing would likely be most useful in
evaluating her symptoms?
A. Septal myocardial tissue Doppler velocity
Figure 4-13 
B. Pulmonary function testing
C. Measure 3D systolic and diastolic LV volumes
D. Coronary angiography Question 6
A 62-year-old man is referred for cardiac stress testing
Question 3 for symptoms of intermittent chest discomfort when
Which of the following would aid in improving echo- he lifts or carries heavy loads. His past medical his-
cardiographic image quality when using microbubble tory includes tobacco use and treated hypertension.
transpulmonary contrast? His baseline ECG shows normal sinus rhythm with
A. Concentrate microbubble solution criteria for left ventricular hypertrophy.
B. Set focus depth in near field The most appropriate test to obtain would be:
C. Decrease transmit frequency A. Treadmill ECG stress test
D. Increase mechanical index B. Exercise stress echocardiogram
C. Dobutamine stress echocardiogram
Question 4 D. Cardiopulmonary exercise stress test
3D echocardiography is a critical adjunct to standard
2D echo imaging for which of the following clinical Question 7
conditions? Which of the following statements best describes
A. Left ventricular apical thrombus speckle tracking echocardiographic imaging?
B. Primary pulmonary hypertension A. Measures low-level myocardial velocities
C. Ascending aortic aneurysm B. Dependent on angle of interrogation
D. Mitral valve prolapse C. Measures absolute change in length of
myocardium
Question 5 D. Simultaneous measurement across image
TTE was requested in a 72-year-old man with a stroke plane
to evaluate for a patent foramen ovale. Agitated saline
contrast was injected via a right antecubital vein at
rest and again following Valsalva maneuver for a total
of six injections. Several microbubbles were first seen
in the left atrium eight cardiac cycles after injection.
These data are most consistent with:
A. Persistent left superior vena cava
B. Patent foramen ovale
C. Pulmonary arteriovenous malformation
D. Nondiagnostic study
Advanced Echocardiographic Modalities  CHAPTER 4 81

Figure 4-14 

Question 8
Figure 4-15 
A 58-year-old male patient presents for follow-up
after a recent transmural myocardial infarction. TTE
was obtained. Because of poor endocardial definition, Question 10
transpulmonary microbubble contrast was used and A transthoracic study was obtained in a 41-year-
the following apical long-axis view image was obtained old patient with a history of recurrent dizziness
(Fig. 4-14). Based on the image provided, you conclude (Fig. 4-15). The image is most consistent with:
that the infarct involved: A. Perimembranous ventricular septal defect
A. Left anterior descending artery distribution B. Anomalous coronary artery
B. Right coronary artery distribution C. Secundum atrial septal defect
C. Left circumflex coronary artery distribution D. Anomalous pulmonary return
D. Septal perforator distribution E. No intracardiac shunt
Question 9
Match the units to the echocardiographic modality:
A. 1/s
B. Centimeters
C. Dimensionless
D. m/s
1. Strain
2. Tissue Doppler
3. Strain rate
4. Speckle tracking
82 CHAPTER 4  Advanced Echocardiographic Modalities

ANSWERS
clinical suspicion for coronary artery disease. In
Answer 1: A this case, the patient has a paucity of cardiovascu-
In patients with a recent myocardial infarction who lar risk factors, and her symptoms are not classic for
did not undergo angiography, a submaximal stress myocardial ischemia. Global longitudinal strain is a
test is appropriate 3 to 6 days after the initial event, representative aggregate of longitudinal strain from
as it provides valuable prognostic information by risk several myocardial segments, represented in the
stratifying patients on the basis of exercise tolerance lower left hand side of this image as “G.L. Strain.”
and residual ischemia. Echocardiographic imaging Longitudinal strain is the relative motion towards
also allows localization of the culprit vessel and the the transducer, and a negative value in excess of
size of the ischemic territory at risk. Exercise echo- −17% is consistent with normal systolic function.
cardiography should not be done within 72 hours Each of the separate segments is color coded, with
of an acute myocardial infarction in unrevascular- that color-coded line plotted below the 2D imag-
ized patients as it may provoke an arrhythmic event. ing along the time dimension. Although measure-
Other contraindications to exercise testing include ment of 3D LV volumes would allow calculation of
patients with uncontrolled arrhythmias, severe symp- the ejection fraction (EF), we are already told this
tomatic aortic stenosis, and acute aortic dissection. In patient’s EF is normal (64%), and the image shows
patients with severe systemic hypertension (systolic all segments contracting synchronously, which sug-
blood pressure > 200 mmHg), blood pressure should gests normal regional wall motion. In contrast, a
be controlled, if necessary with medications, before 2-­chamber image taken from a different patient
stress testing. (Fig. 4-16) shows left ventricular chamber dilation,
dyssynchronous motion of the myocardial segments,
Answer 2: B and a global longitudinal strain of −4%.
Of the options listed, pulmonary function testing
would be most likely to provide an etiology of the Answer 3: C
patient’s dyspnea. Left atrial size is small, arguing Blood and microbubbles have different densities.
against high left atrial pressures and diastolic dys- The relative change in density causes a change
function. Therefore, answer “A,” septal myocardial in acoustic impedance which reflects transmitted
tissue Doppler velocity, which is used to calculate ultrasound waves back to the transducer. However,
E:e′ ratio would be less helpful. Coronary angiog- microbubbles also are destroyed by strong ultrasound
raphy should only be performed if there is high signals. Ultrasound machine settings which improve

Figure 4-16 
Advanced Echocardiographic Modalities  CHAPTER 4 83

image quality during microbubble contrast by pre- the persistent left superior vena cava) before opacify-
serving bubble integrity include decreasing acous- ing the right atrium.
tic power both by decreasing the mechanical index
(power output) and decreasing ultrasound transmit Answer 6: B
frequency. Image quality is also improved by increas- This patient has several cardiovascular risk factors
ing the focal depth from the transducer, but not to and has exertional chest discomfort so that further
the point where attenuation occurs; usually micro- evaluation for coronary artery disease is indicated.
bubble imaging is optimal with the focal depth set The most appropriate next test is exercise stress
in the mid field. If ultrasound microbubble density echocardiography as the patient is ambulatory and
is too high, as would occur with concentrating the his symptoms are provoked by exercise. Echocar-
microbubble solution, then most of the ultrasound is diographic imaging allows detection of ischemic
reflected back to the transducer with shadowing of myocardium based on the presence of a regional
distal structures. wall motion abnormality with stress, but not at
rest. Exercise stress testing is preferred over phar-
Answer 4: D macologic (dobutamine) stress testing when it can
Improved spatial resolution with 3D echocardio- be performed as additional data are obtained on
graphic imaging allows for better understanding of exercise tolerance and the workload required for
the anatomic relationship of intracardiac structures. symptom provocation. A treadmill ECG test would
Contemporary clinical applications of 3D echocar- not be helpful because this patient has an abnormal
diography primarily include guidance for procedures baseline ECG, which increases the likelihood of a
where spatial relationship is critical, such as imag- false-positive study when the ECG alone is used
ing the interatrial septum for an atrial septal defect to detect ischemia. A cardiopulmonary stress test
closure, balloon placement for mitral valvuloplasty, concurrently measures oxygen consumption with
or anatomic characterization of valve anatomy in the stress test but is not typically used in conjunc-
patients with mitral valve prolapse for surgical plan- tion with imaging for ischemia. Cardiopulmonary
ning. Enface imaging of the mitral valve leaflets is exercise testing is most useful to separate the pulmo-
not possible with either TTE and TEE standard nary from cardiac component of exercise limitation
imaging. Utilization of the 3D pyramidal dataset in patients with symptoms of unclear etiology, to
allows data cropping down to the valve plane for objectively measure exercise capacity in patient with
improved visualization of the leaflets. In patients chronic heart failure or congenital heart disease, and
with mitral valve prolapse, this allows for identifica- to measure exercise tolerance during cardiopulmo-
tion of specific leaflet involvement and optimization nary rehabilitation.
of surgical planning for repair procedures. Reliable
3D spectral Doppler imaging (i.e., to measure pul- Answer 7: D
monary arterial systolic pressures) is not yet avail- Myocardial strain and strain rate provide data on
able. Most LV thrombi are difficult to visualize with ventricular contraction and relaxation. Echocardio-
2D imaging, and the lower frame rate seen with 3D graphic measurement of myocardial strain may be
datasets make use for LV apical thrombus diagnosis obtained by both tissue myocardial Doppler imaging
challenging. or speckle tracking strain imaging. Tissue myocardial
Doppler imaging tracks low level myocardial veloci-
Answer 5: C ties; as such, it is subject to the inherent features of
This was an adequate, diagnostic agitated saline Doppler imaging, and is dependent on the angle of
contrast study. A total of six injections were per- interrogation between the transducer and myocar-
formed, both at rest and following Valsalva maneu- dial motion. Speckle tracking utilizes reflections from
ver. Following release of a Valsalva maneuver, there bright echogenic spots (natural acoustic markers) in
is a transient increase in right atrial pressure rela- the myocardium as they move throughout the car-
tive to left atrial pressure, which would increase the diac cycle. Speckle tracking provides a direct measure
likelihood of transient right to left shunting. Sev- of the change in length of the speckle relative to the
eral injections (>5) are typically needed to ensure original length. This relative (not absolute) measure is
that interatrial shunting, if present, is identified. displayed, often with color mapping, of the multiple,
Although microbubbles are seen in the left atrium, simultaneous measurements which are taken across
this occurred after eight cardiac cycles. This sug- an image plane.
gests transpulmonary transit of microbubbles rather
than intracardiac shunting. A persistent left superior Answer 8: A
vena cava can also be identified with an agitated The image is taken from the apical long-axis view.
saline contrast study, where saline is injected into The left ventricular outflow tract is to the right of
the left antecubital vein and bubbles are seen opaci- the image. In this view, the anterior septum is shown
fying the enlarged coronary sinus (transported via on the right and the inferolateral wall is shown on
84 CHAPTER 4  Advanced Echocardiographic Modalities

the left of the image. Microbubble transpulmonary strain is typically expressed as a percentage. Speckle
contrast opacifies the endocardial border of the LV. tracking follows the motion of small reflectors in the
There is thinning of the LV apex, with a large echo- myocardium, allowing calculation of LV strain at
lucent filling defect in the apex, consistent with an multiple sites, and is usually displayed as a 2D color
apical thrombus. In patients with poor imaging qual- coded image. Because it measures strain, speckle
ity, transpulmonary contrast is commonly needed to tracking is also dimensionless.
better evaluate regional wall motion. Often, it is dif-
ficult to exclude an apical thrombus due to artifact Answer 10: C
(near field clutter) or poor image quality. In these This is an image from an agitated saline contrast
cases, transpulmonary contrast will not penetrate the study, taken from the parasternal short-axis view.
thrombus, and a negative filling defect is seen. This The aortic valve is in the center of the image, the left
distribution of wall motion abnormality is most con- atrium is in the lower right section of the image, and
sistent with a left anterior descending distribution the right atrium and right ventricle are seen opacified
with transmural infarction involving the entire apex. by saline contrast. There is a dark, negative washout
of contrast which originates at the interatrial septum
Answer 9 and goes towards the right atrium. This finding is
Tissue Doppler measures velocity of myocardial consistent with flow of noncontrast blood from left to
motion; velocity is lower than the velocity of blood right across an intracardiac shunt, at the interatrial
flow, but is still typically reported in units of either septum. If left atrial pressure is significantly higher
meters/second or centimeters/second. Strain rate is than right atrial pressure, then right to left shunting
the rate of change in myocardial length, normalized will not occur following agitated saline contrast injec-
for the original length. Strain rate is calculated from tion. Anomalous takeoff of the coronary arteries is
the difference in velocities at two myocardial sites (V1, not diagnosed with an agitated saline contrast study.
V2) divided by the distance between them; the units Anomalous pulmonary vein return would return oxy-
are (m/s)/m. Thus, strain rate simplifies to 1/s or s-1). genated blood from the pulmonary circulation back
Strain is a measure of myocardial deformation. Strain to the right heart, but would not insert at the inter-
of a myocardial segment is defined as the difference atrial septum, so that neither a positive or negative
between baseline myocardial length (L0) and final contrast jet would be seen. A perimembranous VSD
contraction length (L), relative to the original length, would not be seen in this view, but could also cause
or (L − L0)/ L0. The length units in the numerator a negative washout of contrast if present (would be
and denominator cancel each other so that strain seen best from a parasternal long-axis view or apical
measurements are dimensionless. In clinical practice, 4-chamber view).
5 Clinical Indications and Quality Assurance
BASIC PRINCIPLES Murmur
Understand the Reliability of Echocardiography for the Chest Pain
Specific Diagnosis Heart Failure or Dyspnea
Integrate the Clinical Data and the Echocardiographic Palpitations
Findings Embolic Event
Recommend Additional Diagnostic Testing as Appropriate Fever/Bacteremia
DIAGNOSTIC THINKING FOR THE ECHOCARDIOGRAPHER THE ECHO EXAM
ECHOCARDIOGRAPHY FOR COMMON SIGNS AND SELF-ASSESSMENT QUESTIONS
SYMPTOMS

BASIC PRINCIPLES n  xpertise in image acquisition and interpretation


E
n  he diagnostic value of echocardiography for a
T affect the reliability of echocardiographic data.
specific diagnosis depends both on the reliability
of the echocardiographic data and on integration v  KEY POINTS
with other clinical information. o S ensitivity is the percent of patients with the diag-
n The framework for echocardiographic data acquisi- nosis correctly identified by echocardiography.
tion and reporting is a structured diagnostic approach o Specificity is the percent of patients without the diag-
to the question posed by the requesting physician. nosis correctly identified by echocardiography.
o Positive predictive value is the percent of patients
v  KEY POINTS with a positive echocardiogram who actually
o  he echocardiographic study seeks to provide
T have the diagnosis.
the appropriate data for clinical decision-making o Negative predictive value is the percent of patients
depending on the patient’s symptoms, signs, and with a negative echocardiogram who actually
known diagnoses. do not have the diagnosis.
o The list of possible diagnoses that might explain o Accuracy indicates what proportion of all studies
the clinical findings, called the differential diag- indicated a correct diagnosis.
nosis, is mentally constructed at the beginning o The positive and negative predictive value of
of the echocardiographic study. a test depends on the prevalence of disease in
o As the study proceeds, some diagnoses are
 addition to sensitivity and specificity.
excluded while others may be suggested by spe-
cific findings.
o Pertinent positive data include abnormal echo- Disease
ECHO
cardiographic findings. Present Absent
o Pertinent negative data include normal echo-
Positive
cardiographic findings that help narrow the dif- True False predictive
Positive
ferential diagnosis. positives positives value
Step 1: Understand the Reliability of
Negative
Echocardiography for the Specific Negative
False True
predictive
Diagnosis negatives negatives
value
n The accuracy of echocardiography describes the
agreement between an echocardiographic mea-
surement or diagnosis and an external reference Sensitivity Specificity
standard, such as another imaging approach or
clinical outcomes (Fig. 5-1). TP  TN
n The precision of an echocardiographic measure- Accuracy 
All tests
ment is affected by variability in recording, mea- Figure 5-1  Accuracy of a diagnostic test. Sensitivity and specificity in com-
suring, and interpreting the echocardiographic parison with positive and negative predictive value. Predictive values depend on
data. the prevalence of disease in the population. TN, True negatives; TP, true positives.

85
86 CHAPTER 5  Clinical Indications and Quality Assurance

o ach laboratory should periodically review


E Conversely, when the pretest likelihood of
o disease is
the reproducibility of the echocardiographic very high, the failure to demonstrate the disease on
measurements. echocardiography often is a false negative.
o The effect of variability is minimized when images o  ith the threshold approach, the upper thresh-
W
from sequential studies are compared side by side. old is the point where the risk of the test is
higher than the risk of treating the patient; for
Step 2: Integrate the Clinical Data and the example, additional diagnostic testing should
Echocardiographic Findings (Fig. 5-2) not delay surgical intervention for an acute
n The likelihood ratio indicates the probability of ascending aortic dissection.
disease in a patient with a positive or negative o The lower threshold for transthoracic echo-

echocardiographic finding; a positive likelihood cardiography (TTE) occurs only with a very
ratio greater than 10 or a negative likelihood ratio low probability of disease; the major potential
less than 0.1 indicates an excellent diagnostic test. adverse effect is a false-positive result leading to
n Pre- and posttest probability estimates integrate the further inappropriate testing or therapy.
likelihood of disease before the echocardiogram is
performed with the echocardiographic results.
Step 3: Recommend Additional Diagnostic
n The threshold approach to clinical decision-making
Testing as Appropriate
indicates that diagnostic testing, such as echocar- n The interpretation of the echocardiogram lists the
diography, is most helpful in patients where the pertinent positive and negative findings along with
results will change either one of the following: any confirmed diagnoses.
• Therapy n The differential diagnosis of equivocal findings
• Diagnostic strategy is indicated and appropriate additional diagnos-
tic testing is recommended when the echocardio-
v  KEY POINTS graphic results are not diagnostic.
o  he positive likelihood ratio is calculated as the
T
true positive rate divided by the false-positive v  KEY POINTS
rate. The negative likelihood ratio is the false- o chocardiography provides qualitative and
E
negative rate divided by the true-negative rate. quantitative information on cardiac structure
o The pretest probability of disease is the prob- and function and often provides a definitive
ability of disease before echocardiography is diagnosis.
done—for example, consideration of cardiac o Positive findings are more helpful than nega-
risk factors and symptoms in a patient sched- tive findings; for example, a dissection flap seen
uled for stress echocardiography provides an on TTE is diagnostic for a dissection but its
estimate of the probability of coronary artery absence does not exclude this possibility.
disease. o A noncardiac cause for symptoms is likely

o Echocardiography is most helpful when the
 when the appropriate echocardiographic study
pretest likelihood of disease is intermediate and is normal. For example, normal resting echo in
echocardiography has a high accuracy for the a patient with a systolic murmur is consistent
diagnosis. with a benign flow murmur. A normal stress
When
o the pretest likelihood of disease is very low, echo in a patient with chest pain indicates
an abnormal echocardiographic finding often is a that coronary disease is not a likely cause of
false-positive result. symptoms.

Diagnosis
Pretest Sensitivity/specificity
likelihood Posttest likelihood
Accuracy/precision/reproducibility
?Other diagnostic tests

Patient with
suspected or Clinical
Echocardiography Prognosis
known cardiac outcome
Education
disease
Follow-up
Other tests
Figure 5-2  Flow chart illustrating the
impact of echocardiographic results on Information
diagnosis, prognosis, and therapy. The available by
effects of echocardiography on clinical out- echocardiography Therapy
come are the best measure of the usefulness Medical
of the test result. Surgical
Clinical Indications and Quality Assurance  CHAPTER 5 87

o  he echocardiographer often needs to assist in


T • The ability to integrate the echocardiographic find-
choosing the optimal diagnostic approach (e.g., ings with the clinical data
TTE or transesophageal echocardiography n  he Echo Exam section summarizes the approach
T
[TEE], stress echocardiography, contrast study) by anatomic diagnosis.
based on the indication for the study. n Examples of the approach to common clinical

o Additional diagnostic studies, with either
 indications for echocardiography are discussed in
another echocardiographic modality or an the next section.
alternate imaging approach, are recommended
after review of the echocardiographic exam.
ECHOCARDIOGRAPHY FOR COMMON SIGNS
AND SYMPTOMS
DIAGNOSTIC THINKING FOR THE
ECHOCARDIOGRAPHER Murmur
The echocardiographer needs:
n n The echocardiographic differential diagnosis for a
• Clinical data to estimate the pretest likelihood of dis- murmur is based on an anatomic approach with
ease before starting the exam evaluation of all four valves and a search for an
• An understanding of pertinent positive and negative intracardiac shunt (Fig. 5-3).
findings for each clinical indication n Most patients referred to echocardiography for
• Knowledge of the reliability of echocardiography for a murmur on auscultation have a benign flow
each diagnosis murmur.

Stenosis
Murmur ( systolic Vmax)
Aortic valve
Regurgitation
(diastolic flow)
Left-sided
valve disease Stenosis
(prolonged T½)
Mitral valve
Regurgitation
(systolic flow)

VSD High velocity


left-to-right flow

Pulmonary
Intracardiac flow volume
ASD
shunt Low velocity flow
across ASD
Figure 5-3  Flow chart for the echocardio-
Continuous graphic differential diagnosis of a mur-
PDA systolic/diastolic mur. The flow chart is arranged by anatomy
flow in PA because the echocardiographer often is not
provided information about the type of murmur
Stenosis or other clinical findings. The basic echocar-
( systolic Vmax) diographic examination includes measure-
ment of antegrade flows and evaluation for
Pulmonic valve regurgitation of all four valves. Additional
Regurgitation evaluation for murmur includes careful inter-
(diastolic flow) rogation of flow in the pulmonary artery to
Right-sided detect a patent ductus arteriosus or increased
valve disease Stenosis flow due to an atrial septal defect. Flow in the
(prolonged T½) septal region is examined with color and CW
Doppler to exclude a ventricular septal defect.
Tricuspid valve Normal physiologic amounts of mitral and
Regurgitation tricuspid regurgitation are not audible and do
(systolic flow) not explain the presence of a murmur. ASD,
Atrial septal defect; PA, pulmonary artery;
Physiologic PDA, patent ductus arteriosus, T1/2, pressure
Normal echo Flow murmur half time; Vmax, maximum antegrade velocity;
regurgitation
VSD, ventricular septal defect.
88 CHAPTER 5  Clinical Indications and Quality Assurance

v  KEY POINTS v  KEY POINTS


o  he echocardiography request form may not
T o  cute chest pain is a medical emergency
A
specify the type of murmur (e.g., systolic or dia- because the differential diagnosis includes
stolic), so a systemic echocardiographic exam is acute coronary syndromes and aortic dis-
essential. section, both of which require immediate
o Normal physiologic regurgitation rarely
 treatment.
accounts for an audible murmur. o An abnormal echocardiographic finding, such
o The most common pathologic causes for a
 as anterior wall hypokinesis, may prompt fur-
murmur in adults are aortic valve stenosis and ther diagnostic and therapeutic interventions,
mitral valve regurgitation. such as coronary angiography.
o Murmurs typically are due to high-velocity
 o An acute pulmonary embolism, if large

intracardiac flows (e.g., aortic stenosis or mitral enough, may show signs of right heart strain
regurgitation) because low-velocity flows (e.g., and increased pulmonary pressures. A smaller
tricuspid regurgitation with normal pulmo- pulmonary embolism may have no significant
nary pressures) are not usually audible with a findings on echocardiography.
stethoscope. o Even when the transthoracic echocardiogram
o Conditions that cause increased cardiac out- is normal, further evaluation may be needed—
put (e.g., anemia or pregnancy) may cause a for example, TEE or chest computed tomog-
murmur due to high velocity transvalvular raphy (CT) in a patient with suspected aortic
flow in patients with otherwise anatomically dissection.
normal valves. o Normal resting wall motion does not exclude
o Congenital heart disease may first be diag- the possibility of significant coronary artery
nosed in an adult based on finding a mur- disease. Wall motion is abnormal only after
mur. In patients with an atrial septal defect, infarction or with ongoing ischemia—for
the murmur is due to increased pulmonary example, with unstable angina or on stress
blood flow volume, not to flow across the atrial testing.
septum. o The presence of a pericardial effusion is consis-
tent with pericarditis, although not all patients
Chest Pain with pericarditis have an effusion.
n T he echocardiographic differential diagnosis for o In a patient with aortic dissection, pericardial
chest pain is based on the major clinical diag- fluid may be due to rupture of the aorta into the
noses that are of immediate clinical concern pericardial space.
(Fig. 5-4). o With significant left ventricle (LV) outflow

n When the echocardiogram does not establish obstruction, the increase in LV myocardial
a diagnosis, further evaluation may be needed wall stress and oxygen demand results in
emergently. angina-type chest pain.

Stress echo
Coronary Resting wall motion
Coronary
artery disease abnormalities
angiography

Aortic dilation
Aortic
Aortic regurgitation TEE
dissection
Dissection flap
Chest pain

Pericardial effusion
Pericarditis
Signs of tamponade

Aortic stenosis
Structural HCM
heart disease LV outflow velocity

Figure 5-4  Echocardiographic approach to evaluation of chest pain. The primary goal in the acute setting is to exclude life-threatening conditions,
such as an acute coronary syndrome or acute aortic dissection. With both acute and chronic chest pain, further diagnostic evaluation often is needed.
HCM, Hypertrophic cardiomyopathy.
Clinical Indications and Quality Assurance  CHAPTER 5 89

Heart failure

LV size
LV systolic
Ejection fraction See Chapter 9
dysfunction
Regional wall motion

LV hypertrophy
LV diastolic
LV and LA inflows See Chapter 7
dysfunction
Tissue Doppler

Pericardial effusion
Pericardial
Pericardial thickening See Chapter 10
disease
Signs of constriction

Aortic or mitral stenosis See Chapters


Valve
Valve regurgitation 11 and 12
disease
Endocarditis

Pulmonary pressure
Right heart See Chapters
RV size and function
disease 6 and 9
Tricuspid regurgitation

Congenital Right heart enlargement


See Chapter 17
heart disease Intracardiac shunt

Figure 5-5  Echocardiographic approach to patients referred for heart failure. A systemic echocardiographic study will include the 2D views and Doppler
flows to identify each of these possible diagnoses. In addition, the sonographer should mentally “check off” each of these conditions as the exam progresses to
ensure that the entire differential diagnosis is considered. If the echocardiographic study is normal, a noncardiac cause of symptoms is likely.

Heart Failure or Dyspnea o  eart failure occurs in patients with valvular


H
n 
Symptoms of dyspnea, edema, and decreased heart disease, even when LV function is normal,
exercise tolerance are nonspecific, with a wide dif- due to obstruction of blood flow (e.g., mitral ste-
ferential diagnosis that includes cardiac and non- nosis) or elevated pulmonary diastolic pressure
cardiac conditions. (e.g., mitral regurgitation).
n 
Heart failure, defined as the inability of the heart o Pulmonary hypertension due to left heart dis-
to supply adequate blood flow at a normal filling ease, pulmonary vascular disease, or underlying
pressure, is the clinical consequent of several types lung disease leads to right heart failure with a
of heart disease (Fig. 5-5). dilated hypokinetic right ventricle (RV), some-
times called cor pulmonale.
v  KEY POINTS o Heart failure in patients with congenital heart
o  eart failure may be due to systolic dysfunction
H disease may be due to ventricular dysfunction,
(called heart failure with reduced ejection frac- obstructive or regurgitant lesions, or intracar-
tion [HFrEF]) or may occur with a preserved diac shunts.
ejection fraction (HFpEF). o When heart failure is present with a normal
o HFrEF may be due to a cardiomyopathy,
 echocardiographic study, noncardiac causes for
coronary disease with prior infarction, long- the patient’s symptoms should be considered.
standing valvular heart disease, or congenital
heart disease. Palpitations
o HFpEF, due to diastolic dysfunction, is seen with n Palpitations are the patient’s awareness of a force-
hypertensive heart disease, hypertrophic cardio- ful, rapid, or irregular heart rhythm.
myopathy, and infiltrative myocardial disease. n The primary approach to evaluation of palpita-
o Constrictive pericarditis often presents as right tions includes resting and ambulatory electrocar-
heart failure, with ascites and peripheral edema. diogram (ECG) monitoring.
90 CHAPTER 5  Clinical Indications and Quality Assurance

n Echocardiography allows evaluation of any under- left-sided


o cardiac tumors (atrial myxoma, valve
lying anatomic abnormalities associated with the fibroelastoma)
cardiac arrhythmia. o In patients with a systemic embolic event, a
cardiac source must be presumed to the cause
v  KEY POINTS when there is atrial fibrillation, a prosthetic
o chocardiography typically is normal in
E valve, an intracardiac thrombus, or a tumor.
patients with a supraventricular arrhythmia and o A cause-effect relationship between the cardiac
no prior cardiac history. finding and embolic event is more difficult to
o Conditions associated with supraventricular
 establish in individual patients with common
arrhythmias include: conditions, such as a patent foramen ovale or
Ebstein
o anomaly in patients with preexcitation aortic atheroma.
syndromes o Diagnosis of a patent foramen ovale is based on
Prior surgery for congenital heart disease
o demonstration of right to left shunting at rest or
o  trial fibrillation often is associated with hyper-
A after Valsalva maneuver, following right heart
tensive heart disease, mitral valve disease, and agitated saline contrast. TEE is more sensitive
LV systolic dysfunction. than TTE for detection of a patent foramen
o The prevalence of atrial fibrillation increases ovale, which is present in about 30% of normal
with age (present in about 4% of people older individuals.
than 60 years of age).
o Left atrium (LA) thrombi are associated with Fever/Bacteremia
atrial fibrillation but are not reliably visualized n 
Echocardiography is the primary approach to
on TTE; TEE imaging is more accurate for diagnosis of endocarditis in patients with bactere-
diagnosis of atrial thrombi. mia (Fig. 5-6).
o Echocardiography often is abnormal in patients n 
In most patients, TTE is the initial approach, but
with ventricular arrhythmias. Quantitative TEE is more sensitive for detection of valvular
evaluation of LV systolic function is particularly vegetations.
important in these patients. n 
Complications of endocarditis (e.g., abscess, fis-
o Diseases that affect the RV, such as RV dyspla- tula) are best evaluated by TEE.
sia, also may present with palpitations. n 
Patients with an intracardiac device (pacer, defi-
brillator, or chronic indwelling intravascular line
Embolic Event such as a hemodialysis catheter) should undergo
n An intracardiac thrombus or mass may result in a echocardiography to exclude a lead infection.
systemic embolic event.
n Aortic atheroma is present in 20% of patients with
v  KEY POINTS
an embolic event. o  etection of valvular vegetations is a major cri-
D
n The presence of a patent foramen ovale or atrial terion for diagnosis of endocarditis.
septal aneurysm has been associated with an o Specificity of TTE for detection of a vegeta-
increased prevalence of systemic embolic events. tion is high (i.e., the finding of a vegetation is
diagnostic) but sensitivity is low; therefore, fail-
v  KEY POINTS ure to demonstrate vegetations does not rule
o  systematic transthoracic examination is the
A out the diagnosis.
first step in evaluation for a potential cardiac o When bacteremia or other clinical signs of

source of embolus, but TEE is a more sensitive endocarditis are present, TEE is appropriate
diagnostic approach. unless the likelihood of endocarditis is very
o Conditions associated with systemic embolic
 low and image quality is high on transthoracic
events include: imaging.
Atrial
o fibrillation o Both TTE and TEE are appropriate with sus-
LA thrombus
o pected prosthetic valve endocarditis because pos-
Prosthetic heart valves
o terior structures are shadowed by the prosthetic
Valvular

o vegetations (bacterial or nonbacterial valve on transthoracic imaging, whereas anterior
thrombotic endocarditis) structures are shadowed on TEE imaging.
o Patent foramen ovale o Detection of vegetations is enhanced by scan-
o Aortic atheroma ning across the valve, using multiple images
o LV thrombus (e.g., after anterior myocardial
 planes and using nonstandard views, on both
infarction) TTE and TEE imaging.
Clinical Indications and Quality Assurance  CHAPTER 5 91

Fever/bacteremia

Low risk of High risk of endocarditis:


endocarditis Prosthetic valve
Congenital heart disease
Previous endocarditis
Staphylococcal bacteremia
TTE

Positive Equivocal Negative

Persistent fever Suboptimal Good-quality


AV block images images
Persistent staph
bacteremia*

TEE Evaluate for TTE  TEE


other sources
of bacteremia

*Or other signs of paravalvular abscess or persistent infection.


Figure 5-6  Flow chart for a suggested approach to evaluation of patients with fever and/or bacteremia who are referred for echocardiography.
AV, Atrioventricular.
92
THE ECHO EXAM

CHAPTER 5  Clinical Indications and Quality Assurance


Indications for Transthoracic Echocardiography
Clinical Diagnosis Key Echo Findings Limitations of Echo Alternate Approaches
Valvular Heart Disease
Valve stenosis Etiology of stenosis, valve anatomy Possible underestimation of stenosis severity Cardiac cath
Transvalvular ΔP, valve area Possible coexisting coronary artery disease CMR
Chamber enlargement and hypertrophy
LV and RV systolic function
Associated valvular regurgitation
Valve regurgitation Mechanism and etiology of regurgitation TEE indicated for evaluation of mitral regur- Cardiac cath
Severity of regurgitation gitant severity and valve anatomy (esp. CMR
Chamber enlargement before MV repair)
LV and RV systolic function
PA pressure estimate
Prosthetic valve ­function Evidence for stenosis TTE is limited by s­ hadowing and reverbera- Cardiac cath
Detection of regurgitation tions Fluoroscopy
Chamber enlargement TEE is needed for suspected prosthetic MR
Ventricular function due to “masking” of the LA on TTE
PA pressure estimate
Endocarditis Detection of vegetations (TTE sensitivity 70-85%) TEE more sensitive for detection of Blood cultures and clinical findings
Presence and degree of valve dysfunction vegetations (>90%) also are diagnostic criteria for
Chamber enlargement and function A definite diagnosis of endocarditis also endocarditis
Detection of abscess depends on bacteriologic criteria
Possible prognostic implications TEE more sensitive for abscess detection
Coronary Artery Disease
Acute myocardial infarction Segmental wall motion abnormality reflects “myocar- Coronary artery anatomy itself not directly Coronary angio (Cath or CT)
dium at risk” visualized Radionuclide or PET imaging
Global LV function (EF)
Complications:
Acute MR vs. VSD
Pericarditis
LV thrombus, aneurysm, or rupture
RV infarct
Angina Global and segmental LV systolic Resting wall motion may be normal despite Coronary angio (Cath or CT)
Function significant CAD Radionuclide or PET imaging
Exclude other causes of angina (e.g., AS, HCM) Stress echo needed to induce ischemia and ETT
wall ­motion abnormality
Indications for Transthoracic Echocardiography—cont’d
Clinical Diagnosis Key Echo Findings Limitations of Echo Alternate Approaches
Coronary Artery Disease—cont’d
Pre/postrevascularization Assess wall thickening and endocardial Dobutamine stress and/or contrast echo CMR
motion at baseline needed to detect viable but Coronary angio (cath or CT)
Improvement in segmental function postprocedure nonfunctioning myocardium Radionuclide or PET imaging
Contrast echocardiography
End-stage ischemic disease Overall LV systolic function (EF) Coronary angio (cath or CT)
PA pressures Radionuclide or PET imaging
Associated MR CMR for myocardial viability
LV thrombus
RV systolic function

Cardiomyopathy
Dilated Chamber dilation (all four ) Indirect measures of LVEDP Radionuclide EF
LV and RV systolic function (qualitative and EF) Accurate EF may be difficult if image LV and RV angiography
Coexisting atrioventricular valve quality is poor
Regurgitation
PA systolic pressure
LV thrombus
Restrictive LV wall thickness Must be distinguished from constrictive Cardiac cath with direct, simultane-
LV systolic function pericarditis ous RV and LV pressure measure-

Clinical Indications and Quality Assurance  CHAPTER 5


LV diastolic function ment after volume loading
PA systolic pressure CMR
Hypertrophic Pattern and extent of LV hypertrophy Exercise echo needed to detect inducible CMR
Dynamic LVOT obstruction (imaging and Doppler) LVOT obstruction Strain and strain rate imaging
Coexisting MR
Diastolic LV dysfunction

Hypertension
LV hypertrophy Diastolic dysfunction precedes systolic Speckle tracking strain and
LV diastolic dysfunction dysfunction but detection challenging due strain rate imaging
LV systolic function to impact of age and other factors. LV twist and torsion
Aortic valve sclerosis, MAC

Pericardial Disease
Pericardial thickening Diagnosis of tamponade is a hemodynamic Intracardiac pressure measure-
Detection, size, and location of PE and clinical diagnosis ments for tamponade or
2D signs of tamponade physiology Constrictive pericarditis is a difficult diagnosis constriction
Doppler signs of tamponade physiology Not all patients with pericarditis have an CMR or CT to detect
effusion pericardial thickening
Continued

93
94
Indications for Transthoracic Echocardiography—cont’d

CHAPTER 5  Clinical Indications and Quality Assurance


Clinical Diagnosis Key Echo Findings Limitations of Echo Alternate Approaches
Aortic Disease
Aortic dilation Etiology of aortic dilation The ascending aorta is only partially visual- CT
Accurate aortic diameter ­measurements ized on TTE in most patients CMR
Anatomy of sinuses of Valsalva (esp. Marfan syndrome) TEE
Associated aortic regurgitation
Aortic dissection 2D images of ascending aorta, aortic arch, descending TEE more sensitive (97%) and specific Aortography
thoracic, and proximal abdominal aorta (100%) CT
Imaging of dissection “flap” Cannot assess distal vascular beds CMR
Associated aortic regurgitation TEE
Ventricular function
Cardiac Masses
LV thrombus High sensitivity and specificity for diagnosis of LV Technical artifacts can be misleading LV thrombus may not be recog-
thrombus 5-MHz or higher frequency transducer and nized on radionuclide or contrast
Suspect with apical wall motion abnormality or diffuse angulated apical views needed angiography
LV systolic dysfunction
LA thrombus Low sensitivity for detection of LA thrombus, although TEE is needed to detect LA thrombus reli- TEE
specificity is high ability
Suspect with LA enlargement, MV disease
Cardiac tumors Size, location, and physiologic Extracardiac involvement not well seen TEE
consequences of tumor mass Cannot distinguish benign from malignant, CT
or tumor from thrombus CMR
Intracardiac echo

Pulmonary Hypertension
PA pressure estimate Indirect PA pressure measurement Cardiac cath
Evidence of left-sided heart disease to account for Difficult to determine pulmonary vascular
increased PA pressures resistance accurately
RV size and systolic function (cor pulmonale)
Associated TR

Congenital Heart Disease


Detection and assessment of anatomic abnormalities No direct intracardiac pressure measure- CMR with 3D reconstruction
Quantitation of physiologic abnormalities ments Cardiac cath
Chamber enlargement Complicated anatomy may be difficult to TEE
Ventricular function evaluate if image quality is poor 3D Echo
(TEE helpful)
A2C, Apical 2-chamber; Angio, angiography; AS, aortic stenosis; CAD, coronary artery disease; Cath, catheterization; CMR, cardiac magnetic resonance imaging; CT, computed tomography; EF, ejection
fraction; ETT, exercise treadmill test; HCM, hypertrophic cardiomyopathy; LVEDP, left ventricular end-diastolic pressure; LVOT, left ventricular outflow tract; MHz, megahertz; MR, mitral regurgitation; MV,
mitral valve; ΔP, pressure gradient; PA, pulmonary artery; PE, pericardial effusion; PET, positron-emission tomography; PLAX, parasternal long axis; PSAX, parasternal short axis; SC, subcostal; SSN,
suprasternal notch; TR, tricuspid regurgitation; TTE, transthoracic echocardiography; VSD, ventricular septal defect.
Clinical Indications and Quality Assurance  CHAPTER 5 95

SELF-ASSESSMENT QUESTIONS

Select the best diagnostic modality option available During his evaluation he has ongoing, severe pre-
for the clinical scenario presented in each of the 12 cordial chest pain which radiates to his midscapu-
questions from the following list: lar region. A 12-lead ECG performed in the ER
A. Transthoracic echocardiography (TTE) shows sinus rhythm at 120 bpm. There are no isch-
B. Dobutamine stress echocardiography (DSE) emic changes on the ECG tracings, and blood work
C. Transesophageal echocardiography (TEE) obtained is unrevealing.
D. Intracardiac echocardiography (ICE)
E. Exercise stress echocardiography (ESE) Question 7
F. No further diagnostic testing indicated A 50-year-old woman had undergone cardiac trans-
plantation 8 years ago for nonischemic cardiomy-
Question 1 opathy. Two years ago, she had developed exertional
A 26-year-old man with a history of prior endocar- dyspnea, and diagnostic evaluation at that time identi-
ditis and bioprosthetic aortic valve replacement due fied mild to moderate range transplant vasculopathy.
to illicit drug use presents with fever and bacteremia. The patient was treated medically with resolution of
Electrocardiogram demonstrates a prolonged PR symptoms. She is currently asymptomatic and presents
interval compared to previous ECGs. for routine clinical evaluation.
Question 2 Question 8
A 46-year-old asymptomatic woman who received a A 60-year-old man presents to the emergency depart-
mechanical mitral valve replacement for rheumatic ment with diaphoresis and severe precordial chest
mitral valve stenosis 8 years ago presents for routine discomfort which has been waxing and waning for
clinical evaluation. Her last transthoracic echocardio- several hours. His past medical history includes type
gram, performed 2 years ago, demonstrated a normal II diabetes mellitus, hypertension, and dyslipidemia.
prosthetic valve hemodynamic profile without other A 12-lead ECG is consistent with myocardial isch-
significant cardiac abnormalities. emia in the inferior leads.
Question 3 Question 9
A 65-year-old man with ischemic heart disease and A 43-year-old woman presents with dyspnea. Her
a decreased ejection fraction of 28%, scheduled for medical history includes breast cancer, and she has
a biventricular ICD placement with a coronary sinus completed several rounds of chemotherapy and radi-
lead to treat heart failure symptoms. ation treatment. A 12-lead ECG shows sinus tachy-
cardia with a ventricular rate of 125 bpm; there is low
Question 4 voltage throughout the precordial leads.
A 40-year-old man with known asymmetric septal
hypertrophic cardiomyopathy presents with several epi- Question 10
sodes of exertional presyncope. A TTE performed 1 A 93-year-old woman presents for routine clinical
month ago demonstrated a left ventricular outflow tract evaluation. She has no known cardiac history. Her
velocity of 1.8 m/s following Valsalva maneuver with a medical history includes only mild hypertension. On
maximal, diastolic septal wall thickness of 18 mm. physical examination, there is an early peaking sys-
tolic murmur heard at the upper right sternal border
Question 5 which does not radiate. The other heart sounds are
A 17-year-old asymptomatic male undergoing a rou- normal.
tine sports physical in preparation for participation on
an intramural school sports team presents for evalu- Question 11
ation. He had received a screening ECG and was A 49-year-old woman with symptomatic rheumatic
noted to have prominent voltage in the precordial mitral stenosis undergoing evaluation for percutane-
leads (S wave in V3 = 25 mm). ous mitral valvuloplasty.
Question 6 Question 12
A 76-year-old man who presents to the emergency A 65-year-old man with type 2 diabetes mellitus
department following a syncopal episode. His medi- undergoing preoperative evaluation for bilateral fem-
cal history includes hypertension and dyslipidemia. oral artery bypass surgery.
96 CHAPTER 5  Clinical Indications and Quality Assurance

ANSWERS

Answer 1: C TEE during a procedure necessitates intubation and


ventilator support to minimize patient discomfort
Clinical suspicion for endocarditis is high in this patient
with prolonged TEE probe placement during a pro-
with bacteremia, and additional diagnostic testing
cedure. Transthoracic echocardiography is difficult to
is indicated. The presence of atrioventricular nodal
perform concurrently with procedures due to interfer-
block raises concern for a paravalvular abscess with
ence of transducer placement with the sterile field.
intramyocardial extension affecting conduction path-
ways. TTE imaging is inadequate for definitive evalu- Answer 4: E
ation of paravalvular abscess, particularly in a patient This patient presents with presyncope and further
with a prosthetic aortic valve because reverberations diagnostic testing is indicated. Repeat TTE is unlikely
and acoustic shadowing from the prosthetic valve may to demonstrate a significant interval change compared
obscure or limit visualization of a paravalvular abscess. to a prior study only 1 month before. Patients with
An aortic paravalvular abscess may extend into the asymmetric septal hypertrophic cardiomyopathy may
septum, affecting the conduction system, but may develop left ventricular outflow tract obstruction which
also extend into the posterior aortic annulus, adjacent is only manifest with physical activity. If there is con-
to the anterior mitral valve leaflet. With TEE imag- current systolic anterior motion of the mitral valve with
ing, there is no interposed ribs/lung tissue between physical activity, left ventricular outflow obstruction
the transducer and the heart, so that image quality is may be exacerbated. Provoked LVOT gradients reflect
improved and posterior cardiac structures are better true peak stress gradients when measured in real time.
visualized compared with TTE imaging. Exercise may be performed using a standard treadmill
Answer 2: F protocol or with a supine bicycle. With treadmill exer-
This patient has had a postoperative transthoracic echo- cise, echocardiographic data are recorded at baseline
cardiogram that documents normal prosthetic valve and immediately after exercise. Bicycle stress ergometry
function, and she is currently asymptomatic. As per the allows continuous echocardiographic images because
recently published ACC/AHA Guidelines for Management of the patient lies recumbent and, with some systems, an
Valvular Heart Disease, routine surveillance echocardiog- integrated stress bed can be maneuvered to allow for
raphy is not recommended in asymptomatic patients optimal patient positioning. TEE imaging is not optimal
with mechanical prostheses. If the patient develops for patients with hypertrophic cardiomyopathy as TEE
cardiopulmonary symptoms or signs of systemic infec- is a resting study, and the LVOT jet is difficult to align in
tion, echocardiography should be performed. If valve a parallel manner with the transducer due to the physi-
dysfunction or infection is suspected, TTE is typically cal constraints of the esophagus.
initially obtained. However, because of acoustic shad- Answer 5: A
owing of the prosthesis with limited or no penetration
This patient’s abnormal baseline ECG is consistent
of ultrasound through prosthetic material, TEE imag-
with left ventricular hypertrophy.
ing is often also needed to more completely evaluate
Patients with hypertrophic cardiomyopathy who
valve function. With TTE imaging of mitral mechani-
participate in competitive sports are at increased risk
cal prostheses, acoustic shadowing of the left atrium
of sudden cardiac death. Therefore, this patient war-
limits evaluation of the atrial side of the valve; therefore,
rants additional evaluation. TTE allows for visualiza-
mitral regurgitation cannot be excluded. Patients with tion of cardiac size and function, including myocardial
bioprosthetic valve prostheses should undergo surveil- thickness. Competitive athletes may have concentric
lance transthoracic echo 8 to 10 years after placement hypertrophy (not hypertrophic cardiomyopathy) which
to exclude valve degeneration and dysfunction. regresses with abstinence from participation in sports;
Answer 3: D in cases where a diagnosis of hypertrophic cardiomyop-
athy is equivocal, additional imaging modalities, such
Intraprocedural echocardiography (ICE) is commonly
as cardiac magnetic resonance imaging, which allow for
utilized in procedural-based echocardiographic stud-
a more detailed evaluation of the morphologic, func-
ies to aid in real-time catheter and device placement
tional and tissue abnormalities associated with HCM
during the procedure. In this case, ICE aids visualiza-
myocardial fibrosis and scarring may be needed.
tion of the coronary sinus os for lead placement. In
the cardiac catheterization laboratory it is frequently Answer 6: C
used for catheter guidance, such as for optimization This patient has complaints of severe, ongoing chest
of placement before interatrial septal puncture. Visu- discomfort. He also had a syncopal event, but a 12-lead
alization of cardiac structures during a procedure ECG performed during active pain does not show evi-
may also be obtained with TEE, typically performed dence of myocardial ischemia. This clinical presenta-
by cardiac anesthesiologists trained in periprocedural tion is most consistent with an acute aortic dissection.
TEE imaging. However, continuous imaging with
Clinical Indications and Quality Assurance  CHAPTER 5 97

Transesophageal echocardiogram allows for visualiza- presentation, physical examination, and echocardiog-
tion of the proximal ascending aorta, aortic arch, and raphy findings. Patients with hemodynamically signifi-
proximal descending thoracic aorta. Only a small por- cant effusions are typically symptomatic, with relative
tion of the distal ascending is not well seen due to the hypotension, tachycardia, and significantly increased
overlying trachea where air obscures ultrasound pen- central venous pressure. This is evident on TTE with
etration. Transesophageal imaging also allows for visu- a large pericardial effusion with evidence of right-sided
alization of cardiac function to evaluate regional wall chamber collapse; there is respiratory variation in mitral
motion. Transthoracic imaging allows visualization of and tricuspid inflow, and plethora of the inferior vena

/
myocardial function, but visualization of the ascending cava (indicating increased central venous pressure).
aorta is not optimal, particularly for definitive evalua-

9
tion for aortic dissection. If aortic dissection is suspected, Answer 10: F
other tomographic imaging modalities, such as cardiac This patient has cardiac murmur heard best over

9
computed tomography or magnetic resonance imaging, the aortic region. The characteristics of the mur-

r
mur are benign, early peaking with a normal second

i
would allow evaluation of the aorta. Stress echocardiog-
raphy could be considered if acute ischemia is excluded heart sound and are consistent with aortic sclerosis.

h
by laboratory work-up but the pain is ongoing, once Increased antegrade flow over the cardiac valves
other severe causes of acute chest pain are excluded. increases turbulent flow and commonly produces

a
a benign systolic flow murmur. In an asymptomatic
Answer 7: B

t
patient, no further diagnostic testing is indicated. In

r/
Transthoracic echocardiography allows for resting patients with significant stenosis, aortic valve replace-
evaluation of cardiac structures and function. This ment is not recommended for asymptomatic patients.
patient has a history of known transplant vasculopa-

e
thy. Screening for transplant vasculopathy requires Answer 11: C

s
cardiac stress testing. Dobutamine stress testing is Patients with symptomatic rheumatic mitral stenosis
the procedure of choice for noninvasive evaluation are candidates for percutaneous mitral valvuloplasty.

/r u
for transplant vasculopathy given that the autonomic Before valvuloplasty, TEE is indicated to evaluate for
response to exercise is not intact in the denervated, mitral regurgitation and left atrial thrombus. There
transplanted heart. Treadmill stress echocardiography is an increased likelihood of success with favorable
would provide a reasonable option if the study indica- valve anatomy (less leaflet calcification, less leaflet

.t c
tion was to assess patient exercise tolerance. In order thickening, and less involvement of the subvalvular
to ensure adequate achievement of maximal cardiac apparatus). Valvuloplasty may significantly increase
workload, pharmacologic testing is typically needed. regurgitation, particularly if the valve leaflets are

a
During DSE testing, most patients achieve the target thickened or calcified and leaflet tearing occurs dur-
heart rate. Like the sympathetic response to exercise, ing the procedure. Also, during the procedure, the

k
denervated hearts will not have a predictable response catheter is in the LA, where a LA thrombus may

/: /
to parasympathetic stimulation; therefore, response to dislodge if present. Therefore, valvuloplasty is con-
atropine is typically variable to nonexistent. traindicated if there is greater than moderate regur-
gitation or a LA thrombus at baseline. TTE may be
Answer 8: F

s
adequate to evaluate mitral regurgitation severity,
This patient’s clinical presentation is consistent with but is not adequate to exclude LA appendage throm-

tt p
an acute coronary syndrome. Provocative stress testing bus. ICE may be used intraprocedurally during the
(TME, DSE) is not indicated in an actively ischemic valvuloplasty, but the TEE is first indicated to deter-
patient. This patient’s pretest probability of significant mine if the patient is a candidate for the procedure.
coronary flow limiting disease is high given his ongo-

h
ing symptoms, and the patient should proceed directly Answer 12: B
to cardiac catheterization. Transthoracic echocar- Preoperative evaluation for patients with significant
diography would allow for identification of regional cardiovascular risk factors typically includes provoca-
wall motion abnormalities to localize the region of tive stress testing to identify myocardial ischemia.
ischemia, but in this case, the clinical presentation Resting studies such as TTE or TEE do not provide
and data (ECG tracing) are consistent with an ongo- evaluation of the cardiac response to stress, as would
ing acute coronary syndrome. No further diagnostic potentially be encountered intraoperatively. An exer-
echocardiographic testing is indicated, and the patient cise stress study (treadmill or bicycle stress) is pre-
should be sent for urgent coronary angiography. ferred over pharmacologic testing when possible as
it provides an evaluation of the cardiac response to
Answer 9: A physiologic stress. However, given that the patient in
This patient’s clinical history and current presentation this case scenario is undergoing preoperative evalu-
are concerning for cardiac tamponade. TTE is diag- ation for bilateral lower peripheral artery disease, it
nostic for pericardial effusion. Cardiac tamponade is unlikely that the workload achieved would be ade-
is a clinical diagnosis which incorporates the clinical quate to provide a maximal stress study.
Left and Right Ventricular
6 Systolic Function
LEFT VENTRICULAR SYSTOLIC FUNCTION RIGHT VENTRICULAR SYSTOLIC FUNCTION
Measure Left Ventricular Size Evaluate Right Ventricular Chamber Size and Wall
Left Ventricular Chamber Dimensions Thickness

/
Left Ventricular Chamber Volumes Examine the Pattern of Ventricular Septal Motion
Left Ventricular Wall Thickness Estimate Right Ventricular Systolic Contraction

9
Left Ventricular Mass and Wall Stress Calculate Pulmonary Systolic Pressure
Measure Left Ventricular Ejection Fraction Consider the Cause of an Elevated Pulmonary Systolic

9
Evaluate Regional Ventricular Function Pressure

r
Calculate Left Ventricular Stroke Volume and Cardiac Output THE ECHO EXAM

i
Calculate Left Ventricular dP/dt SELF-ASSESSMENT QUESTIONS
Other Measures of Left Ventricular Systolic Function

a h
LEFT VENTRICULAR SYSTOLIC FUNCTION
Step 1: Measure Left Ventricular Size
v  
r/ t
KEY POINTS
LV internal dimensions are measured from the

e
o



parasternal window because the ultrasound beam
Left Ventricular Chamber Dimensions

s
is perpendicular to the blood-myocardial inter-
n Two-dimensional (2D) measurement of left ven- face, providing high axial resolution (Fig. 6-1).

/r u



tricular (LV) minor axis internal dimensions or o The parasternal long-axis view allows verifica-



2D guided M-mode measurement of LV minor tion that measurements are perpendicular to
axis internal dimensions at end-diastole and the long axis of the LV. An oblique angle may
end-systole not be recognized in short-axis views.

.t c
­
k a
/: /
RV

s
tt p
Ao

h
LV

LA

A B
Figure 6-1 2D measurement of LV size. Parasternal long-axis view showing 2D measurement of LV internal dimension at end-diastole (onset of the QRS)

from the septal endocardium to the posterior wall endocardium at the level of the mitral valve chords and myocardial thickness at end-diastole (A) and end-
systole (B). This minor axis dimension is measured perpendicular to the long axis of the LV. Ao, Aorta.

98
Left and Right Ventricular Systolic Function  CHAPTER 6 99

o 2D imaging in long- and short-axis views is provides more accurate identification of the



used to ensure the dimension is measured in the endocardial borders (Fig. 6-3).
minor axis of the ventricle, and not at an oblique o End-diastole measurements are made at the




angle, which would overestimate size (Fig. 6-2). onset of the QRS complex; end-systole mea-
o The rapid sampling rate of M-mode (com- surements are made at the minimum chamber



pared with the slow frame rate of 2D imaging) size, just before aortic valve closure.
o Measurements are made from the leading edge




of the septal endocardial to the leading edge of

/
the posterior LV wall.
o The posterior LV wall is identified on M-mode

9



as the steepest, most continuous line. Identifica-
tion of the endocardial border on 2D images is

9
less reliable.

i r
o Measurements of LV internal dimensions and




wall thickness are made at the level of the mitral

h
valve chords just apical to the mitral leaflet tips.

a
Left Ventricular Chamber Volumes

t
LV volumes are more accurate measures of cham-

r/
n



ber size than linear dimensions; three-dimensional
(3D) measured LV volumes are more accurate
than 2D calculations of volume.

e
n 2D echo measures of LV volumes are based on trac-

s



ing endocardial borders in apical 4-chamber and

­
2-chamber views at end-diastole and end-systole,

/r u
­
with volumes calculated using the biplane method
of disks (Fig. 6-4).
n 3D LV volumes are calculated using semiauto-



mated endocardial border tracing with a full 3D

.t c
reconstruction of the LV at end-diastole and end-
Figure 6-2 Oblique LV dimension measurements. Parasternal long-axis
systole (Fig. 6-5). LV end-diastolic and end-systolic

view showing that a M-mode measurement of LV dimensions, along the
dotted line, would overestimate ventricular size because the sample line is volumes are indexed by dividing by body surface

a
oblique compared to the minor axis dimension, shown by the arrow. area (Tables 6-1 and 6-2).

/: / k
0

s
RV

tt p
h
LV
Endo

140

Figure 6-3 M-mode LV dimensions. When the M-mode bean can be aligned perpendicular to the long axis of the LV, based on 2D long- and short-axis

views, the advantage of the M-mode recording is a high temporal sampling rate. The rapid motion of the septal and posterior wall endocardium allows precise
measurements. The endocardium (Endo) typically is the most continuous line with the steepest slope in systole. Measurement of the end-systolic dimension
(maximal posterior motion of the septum, or minimal LV dimension) is shown by the vertical line.
100 CHAPTER 6  Left and Right Ventricular Systolic Function

A4Cd A2Cd
LV Length 8.63 cm End -diastole LV Length 8.14 cm
LV Area 36.0 cm2 LV Area 33.7 cm2
LV Vol 125 mL LV Vol 117 mL
EDV (BP) 124 mL

9/
i r 9
h
A C

ta
r/
A4Cs A2Cs
LV Length 7.25 cm End- systole LV Length 7.35 cm
LV Area 21.1 cm2 LV Area 21.0 cm2
LV Vol 53 mL LV Vol 58 mL

e
EF (A4C) 58% EF (A2C) 50%

s
ESV (BP) 55 mL
EF (BP) 56%

/r u
.t c
a
D

k
B

/: /
Figure 6-4 2D LV volumes. LV volumes and ejection fraction are calculated based on tracing endocardial borders at end-diastole (A and C) and end-systole


(B and D) in both apical 4-chamber (A and B) and apical 2-chamber views (C and D). Identification of endocardial borders is optimized by playing the cine loop

s
to show endocardial motion.

tt p identified visually by frame-by-frame viewing

h
v KEY POINTS of the images (Fig. 6-6).

o Care is needed to obtain images from a true api- o Volumes are more reflective of the degree of






cal position; use of a steep left lateral decubitus LV dilation than linear dimensions.
position with an apical cutout in the stretcher o The most common limitation of this approach is



allows optimal transducer positioning. a foreshortened apical view, resulting in underesti-
o Depth is adjusted so the mitral annulus just fits mation of ventricular volumes; 3D imaging helps



on the image; gain and processing curves are avoid underestimation of LV volumes (Fig. 6-7).
adjusted to optimize endocardial definition. o Body surface area may not be the ideal measure



o Left-sided echo contrast enhances recognition of body size but is widely used clinically.



of endocardial borders when image quality is
poor. Left Ventricular Wall Thickness
o End-diastolic tracings are made at the onset of n 2D guided M-mode measurement of LV septal






the QRS (first frame on digital cine loop); end and posterior wall thickness at end-diastole
systole is defined as minimal LV volume and is n 2D measurement of LV wall thickness



Left and Right Ventricular Systolic Function  CHAPTER 6 101

4C 2C
Volume (s)
EDV  121.7 mL
ESV  39.2 mL

Calculation (s)
EF  67.8%

/
SV  82.5 mL

SAX 3D

9 9
i r
a h
r/ t
e
Volume (mL)

s
106

/r u
89

72 ES

.t c
55

38
Time (sec)
0.1 0.2 0.3 0.4 0.5 0.6 0.7 0.8 0.9

a
Figure 6-5 3D LV volumes. A full volume acquisition from the apical view is used to derive three orthogonal views—4 chamber (4C), 2 chamber (2C), and

k
short axis (SAX) using the red, green, and blue image planes as defined by the boxes and lines. After the user selects annular and apical points at end-systole
(ES) and end-diastole, endocardial borders are detected (which can be edited as needed) to generate a 3D volume reconstruction. A graph of the 3D-derived

/: /
LV volume over the time frame of one cardiac cycle is shown, with ES indicated by the arrow.

s
tt p
v KEY POINTS Left Ventricular Mass and Wall Stress

o LV wall thickness is measured from the para- n LV wall thickness measurements usually are suf-






sternal window because the ultrasound beam ficient for clinical care.
is perpendicular to the blood-myocardial inter- n LV mass and wall stress can be calculated from 2D

h



face, providing high axial resolution. images and LV pressures, if needed.
o The rapid sampling rate of M-mode (compared
v KEY POINTS



with the slow frame rate of 2D imaging) pro-

vides more accurate identification of the endo- o LV mass is calculated from endocardial and



cardial borders. epicardial border tracing in a short-axis view at
o Wall thickness of both the septum and posterior the papillary muscle level and measurement of



wall is measured at the level of the mitral valve LV length.
chordae at end-diastole. o LV wall stress can be calculated based on trac-



o The septal wall thickness measurement does not ing LV endocardial and epicardial borders and



include trabeculations on the right ventricular measuring LV systolic pressure.
side of the septum and does not mistake the mid- o Wall thickness of both the septum and posterior



septal stripe for the right-sided endocardium. wall is measured at the level of the mitral valve
o The posterior LV wall thickness is measured from chordae at end-diastole.



the endocardium to the posterior epicardium.
102 CHAPTER 6  Left and Right Ventricular Systolic Function

TABLE 6-1 Left Ventricular Dimension Measurements


 
TTE 2D TTE–2D-Guided M-Mode TEE
Transducer position Parasternal Parasternal Transgastric
Image plane Long axis Long and short axis 2-chamber view (rotation
angle 60°-90°)
Measurement position Perpendicular to LV long Perpendicular to LV long axis in Perpendicular to LV long axis

/
in LV chamber axis in center of LV center of LV in center of LV
Biplane imaging or rotation Correct M-line orientation often Ensuring centered

9
between long and short requires moving transducer measurement is more

­
­
axis views helps ensure up an interspace difficult on TEE

­
9
centered measurement

r
Measurement site Just apical to mitral leaflet Just apical to mitral leaflet tips At junction of basal one third

i
along LV length tips (chordal level) (chordal level) and apical two thirds of LV

h
Measurement White-black interface Leading edge to leading edge White-black interface
technique

a
Timing in cardiac cycle

r/ t
End-diastole Onset of QRS Onset of QRS Onset of QRS

Frame just before mitral Frame just before mitral
valve closure, or valve closure, or

e
Maximum LV volume Maximum LV volume
End-systole Minimum LV volume or Minimum LV volume Minimum LV volume, or

s

Frame just before aortic Frame just before aortic
valve closure valve closure

/r u
Advantages Feasible in most patients; High sampling rate facilitates Data can be obtained
­
Measurements can be made identification of endocardium intraoperatively to monitor
­
perpendicular to LV long Reproducible preload

.t c
axis Ultrasound beam is perpen-
dicular to endocardium
from TG view, improving
border recognition

a
Disadvantages Endocardial and epicardial M-line measurements should Image plane may be oblique

k
borders may be difficult to only be made if perpendicular Wall thickness measured in

/: /
accurately identify LV measurement is possible TG short axis view
Slow frame rate compared Requires more attention to
with M-mode transducer and M-line position

s
TG, Transgastric.

tt p
o LV mass and wall stress calculations are mainly n When poor endocardial definition limits mea-






useful for research studies and are rarely needed surement of the ejection fraction a contrast study
for clinical decision-making. should be considered. If the ejection fraction
Color Doppler strain rate imaging (see Chapter cannot be measured, a visual estimate may be

h
o



4) may be helpful as clinical guidelines are devel- reported.
oped. This advanced topic is discussed in other
textbooks (e.g., see Otto: The Practice of Clinical v KEY POINTS

Echocardiography, 4th ed.). o LV ejection fraction (EF) is calculated from end-



diastolic volume (EDV) and end-systolic volume
Step 2: Measure Left Ventricular Ejection (ESV) as:
Fraction
n LV ejection fraction is an essential part of the EF = [(EDV − ESV) /ED] ×100 % (6-1)




echo examination and should be reported based
on 2D or 3D methods whenever possible. o When image quality is suboptimal, left-sided



n LV ejection fraction is measured from 2D images contrast may enhance identification of endo-



using the apical biplane method with tracing of cardial borders (Fig. 6-8).
endocardial borders at end-diastole (ED) and end- o When quantitation of ejection fraction is not



systole in apical 4- and 2-chamber views (see Fig. needed or is limited by image quality, LV ejec-
­
6-6 and 6-7). tion fraction is visually estimated based on
Left and Right Ventricular Systolic Function  CHAPTER 6 103

TABLE 6-2 Left-Sided Heart Chamber Sizes in Adults

 
Abnormal
Normal Mild Moderate Severe
LV minor axis ED Women 3.9-5.3 cm 5.4-5.7 cm 5.8-6.1 cm ≥6.2 cm
dimension (2D or 2.4-3.2 cm/m2 3.3-3.4 cm/m2 3.5-3.7 cm/m2 ≥3.8 cm/m2
guided M-mode) Men 4.2-5.9 cm 6.0-6.3 cm 6.4-6.8 cm ≥6.9 cm
2.2-3.1 cm/m2 3.2-3.4 cm/m2 3.5-3.6 cm/m2 ≥3.7 cm/m2

/
LV ED volumes (2D) Women 56-104 mL 105-117 mL 118-130 mL ≥131 mL
≥202 mL

9
Men 67-155 mL 156-78 mL 179-201 mL
Indexed for BSA 35-75 mL/m2 76-86 mL/m2 87-96 mL/m2 ≥97 mL/m2

9
LV minor axis Indexed for BSA 2.1-4.0

r
ES dimension (2D 1.4-2.1 cm/m2

i
or guided M-mode)

h
LV ES volumes (2D) Women 19-49 mL 50-59 mL 60-69 mL ≥70 mL
Men 22-58 mL 59-70 mL 71-82 mL ≥83 mL

a
Indexed for BSA 12-30 mL/m2 31-36 mL/m2 37-42 mL/m2 ≥43 mL/m2

t
Ejection fraction ≥55% 45%-54% 30%-44% <30%

r/
LV wall thickness Women 0.6-0.9 cm 1.0-1.2 cm 1.3-1.5 cm ≥1.6 cm
Men 0.6-1.0 cm 1.0-1.3 cm 1.4-1.6 cm ≥1.7 cm

e
LV mass Women 66-150 g 151-171 g 172-192 g ≥193 g
(2D method) 44-88 g/m2 89-100g/m2 101-112 g/m2 ≥113 g/m2

s
Men 96-200 g 201-227 g 228-254 g ≥255 g
50-102 g/m2 103-116 g/m2 117-130 g/m2 ≥131 g/m2

/r u
LA diameter Women 2.7-3.8 cm 3.9-4.2 cm 4.3-4.6 cm ≥4.7 cm
LA volume index Men 3.0-4.0 cm 4.1-4.6 cm 4.7-5.2 cm ≥5.2 cm
22 ± 6 mL/m2 29-33 mL/m2 34-39 mL/m2 ≥40 mL/m2

.t c
Data from Lang RM et al. Recommendations for chamber quantification. J Am Soc Echocardiogr. 2005;18:1440-1463. See also Lang RM, Badano
LP, Mor-Avi V, et al: Recommendations for cardiac chamber quantification by echocardiography in adults. J Am Soc Echocardiogr 2015;28:1-39.
BSA, Body surface area; ED, end-diastolic; ES, end-systolic.

k a
parasternal short axis and apical 4-chamber,

/: /
IVCT IVRT
2-chamber, and long-axis views.
o A visual estimate is reported using a descriptive 100
Pressure (mmHg)



scale as follows: Ao

s
o Normal (EF ≥ 55%)



o Mildly reduced (EF 45% to 54%)

tt p
Systole Diastole



o Moderately reduced (EF 30% to 44%) LV



o Severely reduced (EF < 30%)



o Estimates by an experienced observer are very LA
a v



reliable.

h
0
Step 3: Evaluate Regional Ventricular LV inflow
1
Function E
A
n Regional (or segmental) ventricular function is Valve
Velocity (m/s)

clicks



evaluated as detailed in Chapter 8.
n Wall motion and thickening for each myocardial



segment is graded as normal, hypokinetic, aki-
netic, or dyskinetic.
n Any areas of thinning and increased echogenicity



(consistent with scar) are noted (Fig. 6-9). –1
LV outflow
v KEY POINTS Figure 6-6 The cardiac cycle. LV, aortic (Ao), and LA pressures are shown


with the corresponding Doppler LV outflow and inflow velocity curves. The
o The presence of wall motion abnormalities in a isovolumic contraction time (IVCT) represents the time between mitral valve



pattern corresponding to coronary artery perfu- closure and aortic valve opening, while the isovolumic relaxation time (IVRT)
sion suggests ischemic cardiac disease. represents the time between Ao valve closure and mitral valve opening.
104 CHAPTER 6  Left and Right Ventricular Systolic Function

True apex False apex

9/
i r 9
a h
A B

r/ t
e
Figure 6-7 Apical foreshortening. When the transducer is on the true apex of the LV, the chamber is ellipsoid, compared with a foreshortened view (A) where

s
the ventricle appears more spherical with a false apex. LV volumes will be underestimated in a foreshortened view and apical wall motion abnormalities may
be missed. This potential error is avoided by moving the transducer down an interspace and laterally to the true apex (B).

/r u
.t c
k a
/: /
s
LV

tt p
h
LA
A B
Figure 6-8 Contrast enhancement. When endocardial borders are poorly seen (A), left-sided contrast (B) improves visualization of the LV chamber and allows

more accurate measurement of LV volumes.
Left and Right Ventricular Systolic Function  CHAPTER 6 105

IVS, s  1.38 cm
LV Minor, chord, s  7.02 cm
LVPW, s  1.14 cm

Ao

LV

/
LA

9 9
i r
h
A B

a
Figure 6-9 Regional wall motion abnormality. In these parasternal long-axis (A) and short-axis (B) views, the inferior and inferior-lateral (or posterior) LV

t

walls are thin compared with the septum, consistent with a previous inferior myocardial infarction. Ao, Aorta.

r/
In a short-axis view, the inferior wall may nor-

e
o



mally flatten along the diaphragm in diastole

Velocity (cm/s)
s
(with normal systolic motion); this normal pat-
tern should not be mistaken as a wall motion VTI VTI

/r u
abnormality. (cm)
o Optimal endocardial definition is needed to



evaluate regional function; contrast should be Time (s)
used if needed.

.t c
o Wall thickening, as well as endocardial motion,



should be evaluated for each myocardial
CSA(cm2)  3.14(D/2)2
segment.
LV

a
Step 4: Calculate Left Ventricular Stroke

k
Volume and Cardiac Output SV  CSA  VTI

/: /
n Stroke volume calculations are not a routine



part of every examination but are helpful when Figure 6-10 Doppler stroke volume calculation. The cross-sectional
ventricular function is abnormal and when

area (CSA) of flow is calculated as a circle based on a 2D echo diameter

s
valve regurgitation or an intracardiac shunt is (D) measurement. The length of the cylinder of blood ejected through this
present. cross-sectional area on a single beat is the velocity-time integral (VTI) of the

tt p
Doppler curve. Stroke volume (SV) then is calculated as CSA × VTI.
n Stroke volume (SV in cm3 or mL) is the product of



the cross-sectional area of flow (CSA in cm2) mul-
tiplied by the velocity-time integral (VTI in cm) of
flow at that site (Fig. 6-10):

h
v KEY POINTS

SV = CSA × VTI (6-2)
LVOT diameter (D) is measured from a para-

o



n Stroke volume can be calculated at any site where sternal long-axis view in mid-systole, from inner



diameter and velocity can be measured but most edge to inner edge, immediately adjacent to the
often is measured in the LV outflow tract (LVOT), base of the aortic valve leaflets (Fig. 6-11, A).
just proximal to the aortic valve. o CSA is calculated as the area of a circle:



n Cardiac output (CO in L/min) is stroke volume



(mL) times heart rate (beats/min), divided by 1000 CSA = π(radius)2 = 3.14 (D/2)2 (6-4)

mL/L:
o LV outflow velocity is recorded using pulsed Dop-



pler, with a 2- to 3-mm sample volume length,
CO = [SV (mL) × heart rate (beats/min)] /1000mL/L from the apical window with the sample volume
= L/min (6-3) just proximal to the aortic valve (Fig. 6-11, B).

106 CHAPTER 6  Left and Right Ventricular Systolic Function

o A visible aortic valve closing click on the Dop- o Thus, dP/dt is 32 mmHg divided by the time






pler tracing ensures correct sample volume interval in seconds.
placement. o A normal dP/dt is more than 1000 mmHg/s.




o The modal velocity (darkest part of the veloc-
Step 6: Other Measures of Left Ventricular



ity curve) is traced to obtain the velocity-time
integral. Systolic Function
o The velocity-time integral represents the “stroke n Other signs of LV systolic function that are not






distance” or the length of the cylinder of blood independently diagnostic may aid in recognition

/
ejected by the LV on each beat. of abnormal function and prompt quantitative
o A similar approach can be used to calculate evaluation of ventricular function.

9



stroke volume across the mitral annulus or the n These findings include increased E-point sep-




pulmonic valve. tal separation, decreased aortic root anterior-

­
In adults, normal stroke volume is about 80 mL posterior motion, and decreased mitral annular

r
o



i
and normal cardiac output is about 6 L/min. apical motion.
Step 5: Calculate Left Ventricular dP/dt

h
CW:2MHz APX MV MR dP/dt  2461 mmHg/sec
n The rate of rise of ventricle pressure, or change in Vmax 1  1.02 m/sec

a
Vmax 2  3.00 m/sec



pressure (dP) over time (dt), is a load-independent 1.0 t  13 msec

t
measure of ventricular function.

r/
n LV dP/dt can be calculated from the rise in velocity
m/s



of the mitral regurgitant-jet (Fig. 6-12).
n This measurement is useful in selected patients

e
1 m/s



with evidence of ventricular dysfunction or with

s
significant mitral regurgitation.
t

/r u
v KEY POINTS 3 m/s

4.0
o The time interval (dt) between the points on the



mitral regurgitant velocity curve at 1 and 3 m/s
is measured in seconds (Fig. 6-13). Figure 6-12 Mitral regurgitation (MR) for measurement of LV dP/dt. The

.t c

o The pressure difference (dP) between 1 and 3 rate of rise of LV pressure in early systole is calculated by measuring
the time integral between 1 and 3 m/s on the mitral regurgitant Doppler



m/s, calculated using the Bernoulli equation, is: velocity curve. This time (t) in seconds is divided by the pressure differ-
ence corresponding to a change in velocity from 1 to 3 m/s (32 mmHg).

a
4(3)2 − 4(1)2 = 32 mmHg (6-5) In this example, the dP/dt is 32 mmHg divided by 0.013 s (13 ms), which

equals 2461 mmHg/s.

/: / k 40dB 1 •/1/0/ 1

s
LVOTD PW Depth  102mm
PW Gate  2.0mm

tt p PW:2MHz

h LV
Ao
m/s

1.0
A B
Figure 6-11 LV outflow tract diameter (LVOTD) and flow. (A) Diameter is measured in a parasternal long-axis view (for axial resolution) in mid-systole using

zoom mode. The diameter is measured at the base of the open aortic valve leaflets from the inner edge of the septal endocardium to the inner edge of the
anterior mitral leaflet, as shown. (B) The LV outflow velocity curve is recorded from the apical window, so the ultrasound beam is parallel to the direction of flow,
with the 2- to 3-mm sample volume on the LV side of the valve. Appropriate positioning is confirmed by the presence of an aortic valve closing click (arrow)
and no opening click. The Doppler curve should show a narrow band of velocities with a clearly defined peak. The velocity-time integral is measured by tracing
the modal velocity of the systolic flow signal. Ao, Aorta.
Left and Right Ventricular Systolic Function  CHAPTER 6 107

dt  25 ms
v KEY POINTS

o The distance between the most anterior motion



of the mitral leaflet and the most posterior 1 P  4 mmHg
motion of the septum normally is only 0 to 5 dP  32 mmHg

m/s
mm. An increased mitral E-point to septal sepa- 3 P  36 mmHg
ration occurs with LV dilation or systolic dys-
function, aortic regurgitation, or mitral stenosis.

/
5
This finding is best appreciated on M-mode MR-jet
tracings (Fig. 6-14).

9
o The movement of the aortic root in an anterior- dP/dt  32 mmHg/0.025 s  1280 mmHg/s



posterior direction on M-mode reflects the fill-

9
Figure 6-13 Schematic diagram showing measurement of dP/dt from


ing and emptying of the left atrium, which is the mitral regurgitation velocity curve. The points where the velocity

r
reaches 1 m/s and 3 m/s are identified and the time interval (dt) between

i
confined between the aortic root and spine. A
these two points is measured as shown. The pressure difference (dP)
decrease in atrial filling/emptying (e.g., with a between 1 m/s (4 mmHg) and 3 m/s (36 mmHg) is 32 mmHg, so dP/dt

h
low forward stroke volume) results in decreased is calculated as shown. MR-jet, Mitral regurgitation-jet.
motion of the aortic root (Fig. 6-15).

ta
r/
0 0

s e EPSS

/r u
A
.t c B

a
133 180

/: / k
Figure 6-14 E-point septal separation (EPSS). The vertical distance between the maximum anterior motion of the mitral leaflet (E-point) and the maximum

posterior motion of the septum reflects LV size and systolic function. The normal EPSS is less than 5 mm. A larger separation indicates LV dilation or systolic
dysfunction. The EPSS also is increased with aortic regurgitation due to impingement of the regurgitant jet on the anterior mitral leaflet and with mitral stenosis,
due to restricted motion of the mitral leaflet. Examples of a normal (A) and increased (B) E-point separation (due to a low LV ejection fraction) are shown.

s
37 0

tt p
h
Ao

LA S LA
D

A B
141 160
Figure 6-15 Aortic (Ao) root motion. LA filling in systole results in anterior motion of the Ao root because LA expansion is constrained posteriorly by the

spine. An example of normal Ao root motion on M-mode in a patient with normal LA filling and emptying and a normal cardiac output (A) is compared with the
reduced Ao root motion seen in a patient with severe LV dysfunction (B) and reduced LA filling and emptying. Conversely, Ao root motion may be increased
when significant mitral regurgitation is present. D, Diastole; S, systole.
108 CHAPTER 6  Left and Right Ventricular Systolic Function

Systole Diastole

LV
RV

RA LA

A B
Figure 6-16  Mitral annular motion. The distance the mitral annulus moves toward the left ventricular apex in systole, as indicated by the arrows, reflects the
longitudinal shortening of the ventricle. This measurement is similar to the tissue Doppler systolic velocity at the annulus.

o  entricular contraction occurs along the long


V v  KEY POINTS
axis of the ventricle, in addition to circumferen- o  he best views for evaluation of RV size are an
T
tial shortening. The mitral annulus moves api- apical 4-chamber view tilted toward the RV and
cally with longitudinal contraction of the LV, a subcostal 4-chamber view.
with the magnitude of motion reflecting ven- o RV size may be overestimated if the apical view
tricular function. Reduced apical motion of the is foreshortened, if the transducer is medial to
annulus (<8 mm) indicates an ejection fraction the LV apex, or if the free wall of the RV is not
less than 50% (Fig. 6-16). well visualized.
o The subcostal view provides the most reliable
estimate of RV size because the ultrasound
RIGHT VENTRICULAR SYSTOLIC FUNCTION beam is perpendicular to the RV free wall and
(Table 6-3) ventricular septum.
o RV hypertrophy is seen when the RV free wall is
Step 1: Evaluate Right Ventricular more than 5 mm or when the RV wall appears
Chamber Size and Wall Thickness as thick as the LV wall (Fig. 6-19).
n 
Right ventricular (RV) size and wall thickness are
evaluated from multiple views, including parasternal
Step 2: Examine the Pattern of Ventricular
short-axis and RV inflow views, apical 4-chamber Septal Motion
view, and subcostal 4-chamber view (Fig. 6-17). n Ventricular septal motion is evaluated in 2D para-
n 
RV size is graded qualitatively based on the rela- sternal long- and short-axis images.
tive size of the right and left ventricle: n M-mode evaluation of ventricular septal motion
• N ormal (RV < LV, with RV apex more basal than LV may be helpful in some cases.
apex)
• Mildly enlarged (enlarged but RV < LV) v  KEY POINTS
• Moderately enlarged (RV = LV) o  ith right ventricular volume overload, the
W
• Severely enlarged (RV > LV) ventricular septum is flattened in diastole, but
n  uantitative end-diastolic measurements of RV
Q in systole the LV assumes the normal circular
size include (Fig. 6-18): configuration (Fig. 6-20).
• B asal dimension measures at the annulus in the api- o With RV pressure overload, the ventricular sep-
cal 4-chamber view tum remains flattened or reversed in systole so
• Outflow tract distal diameter measured in the para- that the LV assumes a D shape in the short-axis
sternal short-axis view view (Fig. 6-21).
• Outflow track proximal diameter measured in the o The pattern of ventricular septum motion

parasternal long-axis view is also altered by conduction abnormalities,
n  V wall thickness is evaluated qualitatively, or free
R previous cardiac surgery, and pericardial
­
wall thickness can be measured on the subcostal view. disease.
Left and Right Ventricular Systolic Function  CHAPTER 6 109

TABLE 6-3 Right Heart Chamber Sizes in Adults


Abnormal
Normal Mild Moderate Severe
RV ED basal diameter* 2.0-2.8 cm 2.9-3.3 3.4-3.8 ≥3.9
RV ED length* 7.1-7.9 cm 8.0-8.5 8.6-9.1 ≥9.2
RV ED outflow tract diameter† 2.5-2.9 cm 3.0-3.2 3.3-3.5 ≥3.6
RV free wall thickness‡ <0.5 cm
Tricuspid annular excursion from diastole to 1.5-2.0 cm
systole
RV fractional area change (4-chamber view) 32-60% 25-31% 18-34% ≤17%
RA minor axis dimension§ 1.7-2.5 cm/m2 2.6-2.8 2.9-3.1 ≥3.2
Pulmonary artery diameter 1.5-2.1 cm 2.2-2.5 2.6-2.9 ≥3.0

Data from Lang RM, et al. Recommendations for chamber quantification. J Am Soc Echocardiogr. 2005;18:1440-1463.
ED, End-diastolic.
*Measured in 4-chamber view, with length from annulus to apex and diameter at the RV base.
†Measured in short-axis view at the aortic valve level from the RV free wall to the aortic valve (inner edge to inner edge).
‡Measured in subcostal view at R wave peak (end-diastole) at level of tricuspid valve chords.
§Minor axis dimension measured in 4-chamber view.

A4C

LV

RV
RV

RA LA

A B
Figure 6-17  Imaging the RV. Evaluation of RV size and systolic function is performed (A) in the apical 4-chamber (A4C) view (note that the transducer is
correctly located over the LV apex) and (B) in a zoom view with the transducer tilted toward the RV.

Step 3: Estimate Right Ventricular Systolic n  he tricuspid annular plane systolic excursion
T
Contraction (TAPSE) towards the apex from end-diastole to
n 
RV systolic function is assessed from multiple end-systole provides a useful measures of longitu-
views, including parasternal short-axis and RV dinal RV contraction.
inflow views, the apical 4-chamber view, and the n Other measures of RV systolic function include
subcostal 4-chamber view. the fractional area change, the tissue Doppler peak
n 
RV systolic function is graded qualitatively as systolic velocity at the annulus and the pulsed or
­normal, mildly, moderately, or severely reduced. tissue Doppler myocardial performance index.
110 CHAPTER 6  Left and Right Ventricular Systolic Function

RV
LV

RV

LV
LA
RA

Figure 6-20  RV volume overload. With RV volume overload, the RV is


Figure 6-18  RV dimension measurements. In an apical 4-chamber view,
enlarged and septal motion is flat in diastole. However, in systole (shown
RV diameter is measured at end-diastole from the septum to the free wall at
here) the contour of the septum is normal, with a circular shape of the LV
the base of the chamber as shown by the markers.
in short axis.

RV

A B
Figure 6-19  RV wall thickness. The RV free wall normally is thinner than the left ventricular wall, although prominent trabeculations and the moderator band (arrow)
may be appreciated, as seen in a patient with mild RV dilation (A). An increased thickness of the RV free wall (B) is seen in a patient with pulmonary hypertension.
Left and Right Ventricular Systolic Function  CHAPTER 6 111

o If LV systolic function is reduced and the RV


v  KEY POINTS looks similar to the LV, the degree of dysfunc-
o  he best views for evaluation of RV systolic
T tion is similar.
function are an apical 4-chamber view tilted o TAPSE is calculated using a M-mode beam
toward the RV and a subcostal 4-chamber from the apex aligned through the tricuspid
view. annulus (normal ≥1.7 cm) (Fig. 6-22).
o RV systolic function can be graded in compari- o  The tricuspid annular tissue Doppler systolic
son to LV systolic function. velocity is also recorded from the apical window
(Fig. 6-23).

TIS0.5 MI 0.8

Vel 10.7 cm/s


PG 0 mmHg

RV

20
S
10
LV
cm/s

A 10
E
20
60 bpm
Figure 6-23  RV tissue Doppler imaging (TDI). With TDI, the velocity of the
tricuspid annulus is recorded showing the expected systolic (S′) and diastolic
Figure 6-21  RV pressure overload. In contrast to volume overload, RV (E′ and A′) tissue velocities, similar to the left ventricular TDI recordings. The
pressure overload results in septal flattening in both diastole and in systole, peak S′ velocity reflects RV longitudinal shortening.
as seen on this end-systolic image.

Dist 2.27 cm

5
RV LV

10
RA LA

A B
Figure 6-22  Tricuspid annular plane systolic excursion. (A) In an apical 4-chamber view with the transducer moved over the RV apex, an M-line is posi-
tioned through the lateral tricuspid annulus. (B) The vertical distance between the position of the annulus at end-diastole and end-systole (e.g., the motion of
the annulus towards the apex in systole) is measured from the M-mode tracing.
112 CHAPTER 6  Left and Right Ventricular Systolic Function

Step 4: Calculate Pulmonary Systolic v  KEY POINTS


Pressure o I n the absence of pulmonic valve stenosis, right
n Noninvasive calculation of pulmonary systolic ventricular and pulmonary systolic pressures
pressures is possible in more than 80% of trans- are the same.
thoracic echocardiograms. o When pulmonic stenosis is present, pulmonary
n The RV to right atrium (RA) systolic pressure gra- systolic pressure is calculated by subtracting the
dient is calculated from the maximum velocity in RV-to-pulmonary artery gradient from the esti-
the tricuspid regurgitant (TR) jet using the Ber- mated RV systolic pressure.
noulli equation (Fig. 6-24). o A diligent search for the highest tricuspid

ΔPRV − RA = v 4(VTR peak )2(6-6)
regurgitant jet velocity includes CW Doppler
recording from parasternal and apical views.
n  ight atrial pressure (RAP), estimated from the
R The highest signal obtained is the most paral-
size and respiratory variation in the inferior vena lel to jet direction.
cava, is added to this pressure gradient to deter- o Signal strength may be enhanced by reposition-
mine right ventricular systolic pressure (Table 6-4). ing the patient or having the patient hold their
breath at end expiration or in midinspiration.
o The Doppler scale, gain, and wall filters are
TR-jet adjusted to show a gray-scale spectrum with
a dense outer edge and smooth systolic curve
RV (Fig. 6-25).
o Estimation of RAP from the respiratory variation
IVC in the inferior vena cava is only useful in sponta-
RA LV neously breathing patients. In ventilated patients,
a measured central venous pressure is used or an
RAP estimated range of pulmonary pressures is pro-
estimate LA vided (Fig. 6-26).
Step 5: Consider the Cause of an Elevated
PRV-RA  4(TR-jet)2 Pulmonary Systolic Pressure
n Pulmonary hypertension may be due to left-sided
PAPPRV-RA  RAP
heart disease, resulting in an elevated left atrium
(LA) pressure and consequent increase in pulmo-
nary pressure.
Figure 6-24  Estimation of pulmonary artery pressure. The velocity in n Pulmonary hypertension also may be due to a pul-
the tricuspid regurgitant-jet (TR-jet) is used to calculate the RV to RA systolic
pressure difference which then is added to an estimate of right atrial pres-
monary arterial disease, lung disease, or pulmo-
sure (RAP), based on the size and respiratory variation in inferior vena cava nary embolism, or may be due to multiple factors
(IVC) diameter. PAP, Pulmonary artery pressure. with a systemic disease (Table 6-5).

TABLE 6-4 Estimation of Right Atrial Pressure


IVC Diameter* (1-2 cm from RA Change with Respiration
Junction†) or Sniff Estimated RA Pressure
Normal ≤2.1 cm) Decrease by > 50% 0-5 mmHg 3 mmHg
Normal (≤2.1 cm) Decrease by ≤ 50% 5-10 mmHg 8 mmHg
Dilated (>2.1 cm) Decrease by > 50% 10-15 mmHg
Dilated (>2.1 cm) Decrease by ≤ 50% 15-20 mmHg 15 mmHg

Data from Brennan JM, et al. Reappraisal of the use of inferior vena cava for estimating right atrial pressure. J Am Soc Echocardiogr 2007;20:857-61;
Kircher BJ, et al. Noninvasive estimation of right atrial pressure from the inspiratory collapse of the inferior vena cava. Am J Cardiol 1990;66:493-
496; Lang RM, et al. Recommendations for chamber quantification. J Am Soc Echocardiogr 2005;18:1440; Rudksi LG, et al. Guidelines for the
echocardiographic assessment of the right heart in adults. J Am Soc Echocardiogr 2010;23:685-713.
*Inferior vena cava (IVC) diameter is measured just proximal to the entrance of hepatic veins in a subcostal view. Changes in IVC diameter during
the respiratory cycle are not reliable indicators of right atrial (RA) pressure in patients on mechanical ventilation.
†For intermediate values, the RA pressure estimate should be lowered or increased depending on the absence or presence of other signs of elevated

RA pressures including:
• Restrictive right-sided diastolic filling pattern
• Tricuspid E/E′ > 6
• Diastolic flow predominance in the hepatic veins (systolic filling fraction < 55%)
• Dilated RA with bulging of septum towards left atrium
Left and Right Ventricular Systolic Function  CHAPTER 6 113

CW:2MHz APX TV V = –3.66m/s TABLE 6-5 Classification of Pulmonary


2.0 PG = 53.5mmHg Hypertension
Classification Examples
Pulmonary arterial Idiopathic
disease Heritable
Drug or toxin induced
m/s Portal hypertension
Systemic inflammatory
disease
Left heart disease Mitral stenosis
Mitral regurgitation
LV systolic dysfunction
LV diastolic dysfunction
Aortic valve disease
Lung disease or Chronic obstructive lung
hypoxia disease
Sleep apnea
4.0 Interstitial lung disease
Chronic pulmonary Recurrent pulmonary emboli
thromboembolism
Figure 6-25  Tricuspid regurgitant jet recorded with CW Doppler. The Multifactorial Systemic disorders (e.g.,
velocity curve shows a smooth contour with a dark edge and a well-defined vasculitis)
peak velocity. Although these characteristics are consistent with a high Myeloproliferative disorders
signal-to-noise ratio, they do not exclude the possibility of underestimation
Metabolic disorders
of velocity due to a nonparallel intercept angle between the flow direction
and Doppler beam.

Exp. Insp.

RA
IVC

A B
Figure 6-26  Estimation of RA pressure. Zoom views of the inferior vena cava (IVC) from the subcostal window are used to visualize the size of the inferior
vena cava at the caval-RA junction during expiration (1.8 cm in this case) and the change in size during inspiration or with a sniff (>50% in this case) indicating
a RA pressure of 0 to 5 mmHg (see Table 6-4).
114 CHAPTER 6  Left and Right Ventricular Systolic Function

o I f pulmonary systolic pressure is elevated and


v  KEY POINTS there is no obvious left-sided heart disease,
o  he definition of pulmonary arterial hyperten-
T careful evaluation of LV diastolic function is
sion is a mean pulmonary artery pressure of appropriate.
more than 25 mmHg at rest with a pulmonary o An elevation in pulmonary pressure greater

capillary wedge pressure of less than 15 mmHg. than expected for the degree of left heart dis-
o In patients with left-sided heart disease, typically ease suggests primary pulmonary vascular dis-
both LA and pulmonary pressures are elevated. ease or lung disease.
Left and Right Ventricular Systolic Function  CHAPTER 6 115

THE ECHO EXAM

Ventricular Systolic Function


TTE TEE
Indications A standard echo exam includes measures Nondiagnostic TTE
of LV and RV size and global and regional Intraoperative and procedural TEE monitoring.
systolic function. Whenever TEE is performed for other indications.
When available, 3D measurement of LV
­volumes and EF is recommended.
LV size and wall Linear 2D or M-mode LV internal dimensions Linear dimensions can be measured on TG short-
thickness and wall thickness. axis views.
LV volumes calculated from apical biplane LV volumes can be calculated by the 2D biplane
method or a 3D volume acquisition. method or by 3D volumes.
LV ejection fraction 3D volumes or 2D biplane method using 4C 3D volumes or 2D biplane method using TEE 4C
and 2C views, taking care to image from tip and 2C views, taking care to include LV apex by
of LV apex. angulation of the image plane.
LV regional wall Apical 4C, 2C, and long-axis views plus TEE 4C, 2C, and long-axis views plus TG short-
motion ­parasternal long- and short-axis views. axis view.
Apical wall motion may be difficult to assess.
Doppler cardiac LVOT and transmitral flows from apical Transmitral flow in 4C view. PA flow from high TEE
output ­approach. view. LVOT flow sometimes obtained from TG long-
PA flow from parasternal views. axis view but intercept angle may be nonparallel.
LV dP/dt CWD mitral regurgitant jet. CWD mitral regurgitant jet.
RV size and systolic Apical and subcostal 4C views plus TEE 4C view plus transgastric short-axis and RV
function ­parasternal long- and short-axis views. inflow views.
TAPSE.
PA pressure TR-jet recorded from parasternal and apical TR-jet may be recorded on TEE 4C or short-axis
­estimates views with dedicated CWD transducer. views but underestimation may occur due to a
nonparallel intercept angle.

2C, Two chamber; 4C, four chamber; CWD, continuous wave Doppler; EF, ejection fraction; LVOT, left ventricular outflow tract; PA, pulmonary artery;
TAPSE, tricuspid annular plane systolic excursion; TG, transgastric; TR-jet, tricuspid regurgitation-jet.

Technical Details
Parameter Modality View Recording Measurements
Ejection 3D or 2D Apical 4-chamber Adjust depth, optimize endocardial Careful tracing of endocardial
­fraction and 2-chamber definition, harmonic imaging, borders at end-diastole and
contrast if needed end-systole in both views
dP/dt CW Doppler MR-jet, usually Patient positioning and transducer Time interval between 1 m/s
from apex angulation to obtain highest-­ and 3 m/s on Doppler MR
velocity MR-jet, decrease veloc- velocity curve
ity scale, increase sweep speed
PA pressures CW Doppler Parasternal and Patient positioning and transduc- Estimate of RA pressure from
apical er angulation to obtain highest- size and appearance of IVC
velocity TR-jet
Cardiac output 2D and pulsed Parasternal LVOT Ultrasound beam perpendicular to LVOT diameter from in-
Doppler diameter LVOT with systolic diameter. ner edge to inner edge in
Apical LVOT LVOT velocity from ant. angulated mid-systole, adjacent and
velocity-time A4C view with sample volume parallel to aortic valve
integral just on LV side of aortic valve Trace modal velocity of LVOT
spectral Doppler envelope

IVC, Inferior vena cava; LVOT, left ventricular outflow tract; MR, mitral regurgitation; PA, pulmonary artery; TR, tricuspid regurgitation.
116 CHAPTER 6  Left and Right Ventricular Systolic Function

Example RV size and systolic function are graded qualita-


A 68-year-old man with a recent inferior myocardial tively. The findings of a moderately dilated RV with
infarction now is hypotensive. Echocardiography shows: severe systolic dysfunction in this patient are consis-
tent with RV infarction accompanying the inferior LV
LV wall thickness (diastole) 8 mm
infarction, as the coronary artery that supplies the LV
LV end-diastolic dimension 50 mm inferior wall also often supplies the RV free wall.
LV end-systolic dimension 33 mm Right atrial pressure is moderately elevated (estimate
10-15 mmHg) as shown by the <50% respiratory
Apical biplane change in the diameter of a dilated inferior vena cava
  End diastolic volume 106 mL (see Table 6-4).
Pulmonary systolic pressure (PAP) is calculated from the
  End systolic volume 62 mL
tricuspid regurgitant jet velocity (VTR) and estimate of
Time interval between 34 ms right atrial pressure (RAP) as:
1 and 3 m/s on MR-jet
LV segmental wall motion Akinesis of basal and PAP = 4(VTR )2 + RAP = 4(2.7)2 + 10 = 29 + 10
mid-LV segments of = 39 mmHg
inferior nd inferior-
lateral walls This is consistent with mild pulmonary hypertension.
Cardiac output (CO) is calculated using the LVOT
RV size Moderately increased diameter to calculate the circular cross-sectional areas
RV systolic function Severely decreased of flow:
TR-jet velocity (VTR) 2.7 m/s
CSALLVOT = π(LVOTD /2)2 = 3.14(2.3/2)2 = 4.2 cm2
Inferior vena cava diameter 2.2 cm
Inspiratory change >50% Stroke volume across the aortic valve (cm3 = mL),
then is:
LV outflow tract diameter 2.3 cm
(LVOTD)
SVLVOT = (CSALVOT ×VTILVOT )
LVOT velocity time inte- 11 cm = 4.2 cm2 ×11 cm = 46 cm3
gral (VTILVOT)
Heart rate (HR) 88 bpm Cardiac output is:

Body surface area (BSA) 1.8m2 CO = SV×HR = 46 mL × 88 beats/min


= 4020 mL/min or 4.02 L/min

Cardiac index (CI) is:


Interpretation
The left ventricle is normal to small in size, based
on diastolic dimensions and volumes, with a mildly CI = CO/BSA = 4.02L/min/1.8 m2 = 2.23L/min/m2
reduced ejection fraction and regional wall motion
The low cardiac index (normal >2.5 L/min/m2)
abnormalities consistent with a recent inferior myo-
is due to the right ventricular infarction resulting in
cardial infarction. Ejection fraction is calculated
reduced LV preload in combination with mild LV sys-
from the apical biplane volumes as:
tolic function.
As an internal check for consistency of the echo-
EF = (EDV − ESV) /EDV × 100% cardiographic data, stroke volume and cardiac output
= (106 mL − 62 mL) /106 mL×100 % = 42% can also be calculated from the 2D apical biplane vol-
ume data:
Qualitative evaluation of ejection fraction is used only
when image quality is too poor for tracing endocar- SV = EDV − ESV = 106 mL − 62 mL = 44 mL
dial borders. Left ventricular dP/dt is calculated from CO = SV×HR = 44 mL × 88 beats/min
the time interval between 1 and 3 m/s on the MR-jet = 3872 mL/min or 3.87 L/min
signal (dt) as:
The differences between stroke volumes and car-
[ diac outputs calculated by the two methods (Doppler
2] [ 2]
dp/dt = 4(V2 )2 − 4 (V1 ) /dt = 4(3)2 − 4(1) /dt and 2D) are consistent with normal measurement
= [36 − 4 mmHg] /.034 s = 941 mmHg/s error. If significant mitral regurgitant were present,
transaortic Doppler stroke volume would be less than
which is mildly reduced (normal > 1000 mmHg/s). the 2D apical biplane stroke volume (see Chapter 12).
Left and Right Ventricular Systolic Function  CHAPTER 6 117

SELF-ASSESSMENT QUESTIONS
Question 1 Question 3
Calculate left ventricle (LV) stroke volume (SV), car- A 68-year-old woman presents with dyspnea and
diac output (CO), fractional shortening (FS), and ejec- is referred for transthoracic echocardiography. A
tion fraction (EF) in the following patient: spectral Doppler image is obtained from the apical
  
4-chamber view (Fig. 6-28). Based on this image, you
Heart rate (HR) 64 bpm
conclude the etiology of her dyspnea is most likely:
LV end diastolic dimension (LVEDD) 5.3 cm
A. Severe, functional mitral regurgitation
LV end systolic dimension (LVESD) 3.8 cm
B. Critical aortic stenosis
3D LV end systolic volume (LVESV) 50 mL
C. Left ventricular systolic dysfunction
3D LV end diastolic volume (LVEDV) 122 mL
D. Decreased ventricular compliance
LVOT diameter 2.4 cm
E. Pulmonary hypertension
LVOT velocity time integral (VTI) 15 cm
Question 2
A transthoracic study is obtained in a patient with a 2.0
history of non-Hodgkin lymphoma who presents with
decreased exercise tolerance (Fig. 6-27). The image is
MOST consistent with: m/s
A. Pericardial effusion
B. Dilated cardiomyopathy
C. Pericardial constriction
D. Restrictive cardiomyopathy

Cal  20mm
0
6.0

Figure 6-28 

Question 4
For the patient in Question 3, a zoomed view of the
signal is shown (Fig. 6-29). The time interval between
the two marked points is 75 ms.
Calculate the dP/dt. _____________

200 1.0

Figure 6-27 

m/s

3.0

Figure 6-29 
118 CHAPTER 6  Left and Right Ventricular Systolic Function

Question 5
A 56-year-old male patient presents with progressive A.
Ejection fraction calculation is higher
dyspnea. Echocardiography is ordered, and the apical B. Geometric assumptions for ejection fraction

biplane LV tracings provide an LV end-systolic vol- calculation are more accurate
ume of 100 mL and an LV end-diastolic volume of C. Stroke volume calculation is lower
222 mL. 3D imaging was obtained (Fig. 6-30). D. Less foreshortened image planes
Based on the data provided, which of the following E. Improved endocardial border definition
best describes LV function assessment, utilizing 3D
imaging relative to standard 2D imaging?

Volume(s)
Apex EDV  244.0 mL
ESV  110.6 mL

Calculation(s)
EF  54.7%
SV  133.4 mL
S
L A I Regional
Tmsv Sel-SD  ******
Tmsv Sel-Dif  ******
Tmsv Sel-SD  ******

Figure 6-30 

Question 6 67 Dist  1.17cm


T  0.384s
A transthoracic study is obtained in a patient with com- T  156bpm
Slope  3.04cm/s
plaints of dyspnea. The M-mode tracing (Fig. 6-31) was
obtained from an apical 4-chamber view. Based on the
image, you conclude the most likely diagnosis is:
A. Hypertrophic cardiomyopathy
B. Pericardial tamponade
C. Restrictive cardiomyopathy
D. RV systolic dysfunction

164

Figure 6-31 
Left and Right Ventricular Systolic Function  CHAPTER 6 119

Question 7 Question 10
A 30-year-old man with dilated cardiomyopathy pres- Which of the following LEAST affects septal myocar-
ents with decompensated heart failure and volume dial motion?
overload. He is referred for echocardiography. Which A. Restrictive cardiomyopathy
of the following echocardiographic findings is MOST B. Pericardial constriction
likely present? C. Coronary artery bypass graft surgery
A. Mitral dP/dt 1400 mmHg/s D. Paced ventricular rhythm
B. Tricuspid regurgitant peak velocity 2.1 m/s E. Primary pulmonary hypertension
C. Mitral PISA radius 1.0 cm
D. LV end-diastolic indexed volume 70 mL/m2 Question 11
An image from a transthoracic echocardiographic
Question 8 study is shown (Fig. 6-33).
Which of the following is LEAST indicative of LV Based on the image provided, you conclude:
systolic function: A. Right ventricular systolic pressure is 50 mmHg
A. Slope mitral regurgitant Doppler signal B. Pulmonary arterial diastolic pressure is 50 mmHg
B. Left ventricular strain rate C. Right ventricular diastolic pressure is 31 mmHg
C. Mitral E-point septal separation D. Pulmonary arterial systolic pressure is 31 mmHg
D. Mitral inflow color Doppler M-mode
E. Fractional shortening
5.0
Question 9
4.0
A patient with progressive dyspnea is referred for trans-
thoracic echocardiography. The tricuspid regurgitant 3.0
jet is faint, but you would like an assessment of pulmo-
nary pressures. The following image is obtained from 2.0
the parasternal short-axis view (Fig. 6-32). Based on the
1.0
image provided, you conclude that the patient has:
A. Normal pulmonary pressures m/s
B. Moderate pulmonary hypertension
C. Severe pulmonary hypertension 1.0

2.0

3.0
100mm/s 95bpm
Figure 6-33 

Time 103 ms
Slope 565 cm/s2

40

20

cm/s

20

40

60

80

100
100mm/s 63bpm

Figure 6-32 
120 CHAPTER 6  Left and Right Ventricular Systolic Function

ANSWERS

Answer 1 rapidly as a normally functioning LV, and the slope of


the MR-jet decreases. This is not an aortic stenosis jet
The stroke volume (SV) is the volume of blood
as there is no delay in the signal onset for isovolumic
ejected from the LV during systole. SV is calculated
contraction before AV opening. Decreased ventricu-
as the product of the velocity time integral and
lar compliance, as would be seen in severe diastolic
cross-sectional area at the point of interrogation,
dysfunction, cannot be assessed definitively from this
and at the same point in the cardiac cycle. In this
image. Other parameters to assess diastolic function
case, SV across the left ventricular outflow tract in
would be assessment of PW Doppler mitral inflow
systole can be calculated as: areaLVOT × VTILVOT. A
tracings, left atrial size, septal myocardial tissue veloci-
circular orifice is assumed for the LVOT. Therefore,
ties (for E:e′ calculation), and pulmonary venous flow.
for this patient, this calculation is: π (radiusLVOT)2
× VTILVOT or π (2.4/2)2 × 15 = 68 mL. Using the
3D volumes, stroke volume is calculated as LVEDV Answer 4
− LVESV, or 122 mL − 50 mL = 72 mL. Cardiac The dP/dt is 427 mmHg/s. The LV to LA pressure
output (CO) is the product of stroke volume and gradient is calculated based on the Bernoulli equa-
heart rate. In this patient, CO = SV × heart rate = tion (gradient = 4V2). The markers are placed at
68 mL × 64 bpm/(1000 mL/L) = 4.4 L/min. The 1 and 3 m/s, corresponding to an LV to LA pres-
fractional shortening is measured from the paraster- sure difference of 4V3m/s2 − 4V1m/s2 or 36 mmHg –
nal long-axis ventricular dimensions as (LVEDD − 4 mmHg, or a 32 mmHg change in pressure over a
LVESD)/LVEDD = (5.3 − 3.8)/5.3 = 0.28 or 28%, 75 ms time interval. The rate of rise of LV pressure
which is in the normal range. The EF is more com- (dP) over time (dt) is: dP/dt = 32 mmHg/0.075ms =
monly reported than the fractional shortening and is 427 mmHg/s. A normal rate of pressure rise is over
the relative change in diastolic and systolic volumes. 1000 mmHg/s.
This is measured as (LVEDV − LVESV)/LVEDV =
(122 − 50)/122 = 0.59 or 59%. Answer 5: D
Apical biplane calculation of LV ejection fraction
Answer 2: B utilizes geometric assumptions for ventricular cham-
This is an M-mode tracing from the parasternal ber shape. Tracings are taken at end-diastole and
long-axis view of the heart. Left ventricular cham- end-systole in the apical 4-chamber and 2-­chamber
ber size is severely dilated with the marks on the views. 3D imaging does not utilize geometric
vertical axis each representing 2 cm. The tracing assumptions, but rather includes the entire LV in
is taken at the mitral valve leaflet tips (seen along the pyramidal dataset. The asymmetric, “true”
the midportion of the image). The end-diastolic endocardial border is generally larger than the esti-
dimension is ∼9 cm and the end systolic dimension mated LV chamber size obtained by standard 2D
is ∼7 cm. There is a large separation between the imaging. Because the entire left ventricle is imaged
anterior mitral valve leaflet and the anteroseptum in 3D, image planes can be adjusted to ensure that
(increased E-point septal separation) consistent with the LV apex is included, and images are not fore-
severe LV enlargement. A pericardial effusion is not shortened (as can happen with 2D imaging). Fore-
seen. Septal and posterior wall motion is synchro- shortening of the LV excludes the apex from volume
nous throughout the cardiac cycle, not consistent measurements, decreases measured LV volumes,
with either myocardial restriction or pericardial and often erroneously raises ejection fraction calcu-
constriction. lations. However, the imaging frame rate in acquir-
ing a pyramidal data set is lower, and endocardial
Answer 3: C border definition is not as optimal for 3D imaging
The image provided is a spectral Doppler tracing relative to standard 2D imaging relative to imaging.
taken across the mitral valve from an apical 4-chamber In this example, the end-diastolic and end-systolic
view. The signal directed away from the transducer volumes were both higher than the volumes derived
in systole is mitral regurgitation. The jet starts at onset from the apical biplane method. However, the rela-
of the QRS, with a peak velocity of 5.2 m/s, which tive increase in LV volumes is proportional such that
is consistent with a systolic LV to LA gradient of 108 the calculated ejection fraction is the same. For the
mmHg, normal systemic pressures. Jet density is not 2D derived volumes, ejection fraction was (222 mL
dense relative to the LV inflow signal, so it is not con- – 100 mL)/222 mL, or 55%, the same as was calcu-
sistent with severe mitral regurgitation. The initial lated using the 3D volumes. The stroke volume for
slope of the MR-jet (dP/dt) is decreased below 1000 the 3D images was 244 mL – 110 mL = 114 mL,
mmHg/s. In patients with LV systolic dysfunction, and for the 2D volumes 122 mL. The incremental
the LV does not generate a transmitral gradient as difference in stroke volume is relatively comparable.
Left and Right Ventricular Systolic Function  CHAPTER 6 121

decreases to below 1000 mmHg/s. A slope of 1400


mmHg/s is consistent with normal LV systolic dys-
function. An LV end-diastolic indexed volume of
70 mL/m2 is normal. LV volumes are indexed to
body surface area. For both women and men, a value
of ∼75 mL/m2 is consistent with only mild LV dila-
tion and a volume over 100 mL/m2 is consistent with
severe LV dilation.
Answer 8: D
The initial slope of the mitral regurgitant jet (dP/
dt) measures the relative change in transmitral pres-
sure over time. A slope of less than 1000 mmHg/s
is consistent with systolic dysfunction. Color Doppler
M-mode of mitral inflow from the apical 4-chamber
view allows evaluation of diastolic function by evaluat-
ing the velocity of LV inflow. With diastolic dysfunc-
tion, the ventricle is stiffer with less “pull” of blood,
and blood flow will be slower. Flow propagation is the
slope of the M-mode color Doppler signal taken at
the midventricular level (>30 cm/s in a patient with
normal diastolic function). Strain rate is the rate of
change in myocardial length along a particular seg-
Figure 6-34 
ment, normalized for the original length. Strain rate
is calculated from the difference in velocities at two
myocardial sites divided by the distance between
Answer 6: D them. Velocities at myocardial sites with poor systolic
This is an M-mode tracing taken from the apical LV function would be lower than at sites with nor-
4-chamber view with the ultrasound beam directed mal systolic function. E-point septal separation is the
through the tricuspid valve annulus, as shown (Fig. maximal distance between the anterior mitral valve
6-34). The tricuspid annular plane systolic excur- leaflet and LV septum measured from the parasternal
sion (TAPSE) is a measure of RV systolic function. long-axis view obtained using M-mode. A distance
A TAPSE displacement below 1.8 cm, as was seen in over 15 mm is consistent with increased LV cham-
this case (measured at 1.2 cm), is consistent with RV ber size and decreased systolic function. Fractional
systolic dysfunction. With the M-mode ultrasound shortening is measured from the parasternal long-axis
beam positioned at the tricuspid valve annulus, the LV dimensions and is the relative change in diam-
pericardial space and LV are not optimally seen. Peri- eter between the end-systolic (LVESD) and end-dia-
cardial tamponade cannot be evaluated from this view. stolic (LVEDD) LV dimensions, measured as 100 ×
Similarly, the interventricular septum, particularly the (LVEDD – LVESD)/LVEDD.
basal septum, is not fully visualized in the M-mode
ultrasound beam, and hypertrophic cardiomyopathy Answer 9: A
cannot be excluded. The image provided is a pulse wave Doppler inter-
rogation of pulmonary artery flow. The anterograde
Answer 7: C Doppler signal is relatively symmetric with a peak
Functional mitral regurgitation is common in dilated velocity of ∼0.6 m/s. The time from signal onset to
cardiomyopathy. LV dilation leads to apical tethering peak velocity is 103 ms. In patients with normal pul-
of the mitral valve leaflets and poor leaflet coapta- monary arterial systolic pressure, the Doppler signal
tion. With progressive volume overload and worsen- is symmetrical in shape. Pulmonary velocity accelera-
ing LV dilation, mitral regurgitation often worsens. A tion time (AT) is the interval between the onset of flow
proximal isovelocity hemisphere radius in excess of and peak velocity. With pulmonary hypertension, the
1.0 cm is consistent with severe regurgitation. A peak peak velocity occurs earlier, with a time to peak dura-
tricuspid regurgitant jet velocity of 2.1 m/s, utilizing tion that is shortened, and an asymmetric Doppler sig-
the Bernoulli equation provides a right ventricular nal. A normal pulmonary velocity AT is greater than
systolic pressure of 17 mmHg (RVSP = 4 [VTR-jet]2), 100 ms, and an AT consistent with severe pulmonary
which is not consistent with decompensated heart hypertension is less than 60 ms. With severe pulmo-
failure. In patients with LV systolic dysfunction, the nary hypertension there may be mid- to late-systolic
poorly functioning LV does not generate a transmi- notching of the descending portion of the signal as
tral gradient rapidly, and the slope of the MR-jet well. The pulmonary velocity AT may underestimate
122 CHAPTER 6  Left and Right Ventricular Systolic Function

severity of pulmonary hypertension when right ven- occurs with severe pulmonary hypertension, results
tricular flow is increased, such as seen in patients with in septal flattening; there is a leftward septal shift
large left to right shunts. throughout the cardiac cycle.
Answer 10: A Answer 11: C
Restrictive cardiomyopathy is most commonly due to This is a continuous wave Doppler tracing in the
an underlying infiltrative process. This process most right ventricular outflow tract, recording pulmonary
commonly affects diastolic function, with relative regurgitant flow. Assuming there is no obstruction of
preservation of systolic function. Systolic function is flow across the pulmonic valve, the regurgitant veloc-
typically synchronous, unless the infiltrative process ity reflects the pulmonary artery to right ventricular
has affected the electrical conduction pathways. In diastolic pressure gradient. Right ventricular diastolic
pericardial constriction, there is a fixed space for car- pressure can be estimated from the end-diastolic
diac motion. As a consequence, there is often respira- velocity utilizing the Bernoulli equation. In this case,
tory dependent shifting of septal motion from right the end-diastolic velocity is 2.8 m/s, which corre-
to left with transient increases in preload, as occurs lates with a right ventricular end diastolic pressure of
with inspiration. Ventricular conduction abnormali- 31 mmHg. Pulmonary arterial diastolic pressure
ties (LBBB, RBBB or ventricular pacing) alter the could then be estimated by adding the central venous
sequence of ventricular contraction. Initial activa- pressure estimate. This is obtained by evaluating infe-
tion with an RV apical lead will lead to dysynchro- rior vena cava diameter from the subcostal view and
nous contraction of the RV free wall relative to the determining whether there is inspiratory collapse.
septum and LV. Cardiac surgery involving the septum Analogous to this is estimation of right ventricular
can also lead to alteration in the ventricular activation systolic pressure utilizing the peak tricuspid regurgitant
sequence. Pressure overload of the right ventricle, as jet velocity.
7 Ventricular Diastolic Filling and Function
STEP-BY-STEP APPROACH MILD DIASTOLIC DYSFUNCTION (IMPAIRED RELAXATION)
Basic Principles MODERATE DIASTOLIC DYSFUNCTION
Measure Left Ventricular Inflow Velocities (PSEUDO-NORMALIZATION)
Record Left Atrial Inflow SEVERE DIASTOLIC DYSFUNCTION (DECREASED
Record Tissue Doppler at the Mitral Annulus COMPLIANCE)
Measure the Isovolumic Relaxation Time LEFT ATRIAL PRESSURE ESTIMATES
Consider other Useful Measurements THE ECHO EXAM
Integrate the Data SELF-ASSESSMENT QUESTIONS
NORMAL DIASTOLIC FUNCTION
Characteristic Features
Diastolic Filling Versus Diastolic Function

STEP-BY-STEP APPROACH
Basic Principles
n  Diastolic dysfunction often occurs in association
with abnormal imaging findings, for example,
left ventricular hypertrophy or impaired systolic
function.
n Diastolic dysfunction may be the earliest sign of
cardiac disease, often with Doppler findings ante-
dating clinical or imaging signs of dysfunction.
n 
Chronic elevation of left ventricle (LV) diastolic
pressure often leads to left atrium (LA) enlarge-
ment, a key element in evaluation of LV diastolic
dysfunction. Figure 7-1  Normal LV inflow pattern.  LV inflow velocities were record-
ed using pulsed Doppler with the sample volume at the mitral leaflet tips.
Step 1: Measure Left Ventricular A, Atrial; E, early.
Inflow Velocities
n LV inflow velocities are recorded at the mitral leaf-
E
let tips and at the mitral annulus (Fig. 7-1). 1.0
n  Standard measurements are E velocity and decel- Deceleration slope
m/s
eration time, and A velocity and duration (Fig. 7-2). A
n The normal pattern of a higher E than A velocity
is reversed with impaired early diastolic relaxation,
but the pattern may be “pseudo-normalized” with
more severe diastolic dysfunction.
v  KEY POINTS A dur
t
o  V inflow velocities are recorded at the mitral
L
Figure 7-2  Schematic diagram showing basic measurements from
leaflet tips (highest velocity signal) in the apical the LV inflow curve.  The early (E) diastolic peak velocity, the velocity after
4-chamber view using pulsed Doppler with a atrial (A) contraction, the early diastolic deceleration slope, and the duration
sample volume of 2.0 to 2.5 mm in length. of the A velocity (from the recording at the annulus).
o The Doppler scale, baseline, and gain are

adjusted to show a clear velocity curve. Recordings at the annulus are used to measure
o Low wall filter settings allow accurate measure- A duration.
ments that require identification of where the o A transient decrease in preload may unmask an
velocity signal intersects the baseline (Fig. 7-3). impaired relaxation filling pattern in patients
o Recordings at the leaflet tips are used to mea- with superimposed elevated filling pressures.
sure E and A velocity and deceleration slope. This is shown by recording LV inflow at the

123
124 CHAPTER 7  Ventricular Diastolic Filling and Function

40dB 1 •/+1/0/ 1
PW Depth= 110mm
PW Gate= 2.0mm
PW Gain= –3dB

PW:2MHz

.60

Figure 7-4  Normal pulmonary vein flow pattern.  LA inflow is recorded


m/s with the pulsed Doppler sample volume positioned in the right superior pul-
monary vein (RSPV) from an apical 4-chamber approach. With atrial con-
traction, there is a small atrial reversal velocity (a), with a normal pattern of
systolic (S) and diastolic (D) inflow into the atrium.
40dB 4 •/+1/0/ 1
PW Depth= 110mm
PW Gate= 3.0mm
PW Gain= 6dB D
S
0.5
m/s
PW:2MHz adur

.60 a
Figure 7-5  Schematic of measurements for pulmonary vein flow. 
Typical measurements included the atrial (a) reversal peak and duration and
peak systolic (S) and diastolic (D) filling velocities.

o  olor flow imaging may be helpful in locating


C
m/s the pulmonary vein and optimizing sample vol-
ume position. The 2- to 3-mm length sample
Figure 7-3  Effect of wall filter settings.  Example of LV inflow recorded volume should be at least 1 cm into the pulmo-
at the annulus with the wall filters set at a low level (level is set at 1) to allow nary vein (Fig. 7-6).
accurate timing measurements (top). When the wall filter is inappropriately o The Doppler scale, baseline, and gain are

high (level is set at 4), the intersection of the Doppler signal with the baseline adjusted to show a clear spectral signal.
is no longer seen, making accurate measurement difficult (bottom).
o Low wall filter settings allow accurate measure-
ments that require identification of where the
velocity signal intersects the baseline.
mitral leaflet tips while the patient performs a
Valsalva maneuver. Step 3: Record Tissue Doppler at
the Mitral Annulus
Step 2: Record Left Atrial Inflow n Tissue Doppler myocardial velocities are recorded
n LA inflow velocities are recorded in the right supe- just below the mitral annulus from a TTE apical
rior pulmonary vein from an apical 4-chamber approach (Fig. 7-7).
view on transthoracic echocardiography (TTE) or n Standard measurements are the early myocardial
in any pulmonary vein on transesophageal echo- velocity (E′) and atrial myocardial velocity (A′)
cardiography (TEE) (Fig. 7-4). (Fig. 7-8).
n  Standard measurements are peak systolic velocity, n An E′/A′ ratio more than 1.0 is normal, with a
peak diastolic velocity, and the atrial velocity peak reduced ratio indicating impaired early diastolic
and duration (adur) (Fig. 7-5). relaxation.
n A PVa greater than 0.35 m/s and an adur 20 ms n 
A ratio of the transmitral E velocity to the tissue
longer than transmitral A duration indicate an Doppler E′ velocity greater than 15 predicts an LV
elevated LV end-diastolic pressure. end-diastolic pressure more than 15 mmHg.
v  KEY POINTS v  KEY POINTS
o A inflow velocities from the transthoracic
L o In the apical 4-chamber view, a small (2 mm)
approach may be difficult to record due to poor sample volume is positioned in the myocardium
signal strength at the depth of the pulmonary vein. about 1 cm from the mitral annulus. The tissue
Ventricular Diastolic Filling and Function  CHAPTER 7 125

Doppler instrument settings include a velocity


scale of about 0.2 m/s, low gain settings, low
velocity scale, and low wall filters.
o Tissue Doppler recordings at the septal side of
the annulus are more reproducible than signals
LV
from the lateral wall.
o The E′ and A′ velocities are less dependent on
preload than the transmitral flow velocities.
Step 4: Measure the Isovolumic
Relaxation Time
LA n Pulsed Doppler is used to show the time inter-
val between aortic valve closure and mitral valve
opening (the isovolumic relaxation time [IVRT])
SV (Fig. 7-9).
n 
The IVRT normally 50 to 100 ms, is prolonged
with impaired relaxation but is shortened with
severe diastolic dysfunction and reduced compli-
ance (Fig. 7-10).
v  KEY POINTS
o In an anteriorly angulated 4-chamber view, a
Figure 7-6  Identification of the right superior pulmonary vein from 2- to 3-mm sample volume is positioned midway
the transthoracic apical 4-chamber view.  Color Doppler imaging aids in between aortic and mitral valves to show both
positioning the pulsed Doppler sample volume (SV, arrow) about 1 cm into
the pulmonary vein for optimal data quality. LV ejection and LV filling velocity curves. The
wall filters are set at a low level to identify the
end of aortic outflow and onset of mitral inflow
at their intersection with the baseline.
o The time interval is measured in milliseconds (ms).

LV 0.1
RV S
m/s

S.V.
0.1 E′ A′
B

LA
RA

A
Figure 7-7  Schematic diagram of tissue Doppler measurements.  In the apical 4-chamber view, the sample volume is placed about 1 cm apical to the
medial mitral annulus. The typical early (E) and late (A) tissue Doppler velocities are seen in diastole directed away from the transducer (as the ventricle fills).
In systole, there is a velocity component toward the transducer corresponding to systolic contraction of the ventricle.
126 CHAPTER 7  Ventricular Diastolic Filling and Function

Step 5: Consider other Useful Measurements


n  The diastolic slope of the apical color M-mode
recording of LV inflow (the propagation velocity)
reflects the rate of LV diastolic relaxation (Fig. 7-11).
n The rate of decline in velocity of the mitral regur-
gitant jet at end-systole reflects the early diastolic
rate of decline in LV pressure (Fig. 7-12).
v  KEY POINTS
o  ropagation velocity is measured from an api-
P
cal view using a narrow sector, a depth that just
includes the mitral annulus, with the aliasing
velocity set to 0.5 to 0.7 m/s, at a fast (100 to
200 mm/s) sweep speed.
Figure 7-8  Doppler myocardial tissue velocities.  Recordings are made
at the septal side of the mitral annulus using a small sample volume, with o The early diastolic –dP/dt is measured from the
the velocity scale reduced (note that the velocity range is only 0.2 m/s), the mitral regurgitant continuous wave (CW) Doppler
wall filters at a low level (setting = 1), and the gain reduced to a very low curve by measuring the time interval between 3
level (setting = −17 dB) A’, Diastolic late velocity; E’, diastolic early velocity; and 1 m/s and dividing by 32 mmHg (analogous
S, systolic velocity.
to measurement of +dP/dt from the early systolic
part of the mitral regurgitant velocity curve).
Step 6: Integrate the Data
n  easurement of LA size (diameter and/or indexed
M
MV volume) is useful in the assessment of diastolic
function. Chronically elevated LV filling pressure
0 leads to increased LA chamber size.
n Based on integration of data from LA size, LV fill-
ing velocities, LA filling velocities, tissue Doppler,
and IVRT, diastolic dysfunction can be detected
Ao and graded (Fig. 7-13).
IVRT
n The clinical interpretation of the data also takes sev-
Figure 7-9  Isovolumic relaxation time. This time interval is measured eral other factors into consideration, including sever-
from aortic valve (Ao) closure to mitral valve (MV) opening on the Doppler
tracing, corresponding to the phase of the cardiac cycle where LV pressure is ity of mitral regurgitation, LV systolic function, LV
rapidly declining, but LV volume is constant. IVRT, Isovolumic relaxation time. wall thickness, and clinical signs and symptoms.

40dB 2 •/+1/0/ 1
PW Depth= 81mm
LV PW Gate = 2.0mm
PW Gain= –2dB

SV LV IVRT = 88 msec
PW:2MHz

.60

Ao
LA

m/s

A B
.40
Figure 7-10  Example of IVRT measurement.  A, In an apical 4-chamber view angulated anteriorly to include the aortic valve, a pulsed Doppler sample
volume is positioned on the LV side of the anterior mitral leaflet in systole (to record LV outflow) and on the atrial side in diastole (to record LV inflow). B, The
time interval between the end of aortic antegrade flow and the onset of diastolic inflow across the mitral valve is measured. The scale and wall filters have been
adjusted to optimize identification of the onset and end of flow, at their intersection with the baseline. A rapid sweep speed (100 mm/s) is used to improve the
accuracy of the measurement. In this patient, the IVRT is normal at 88 ms (normal 50 to 100 ms) sample volume. SV, Sample volume.
Ventricular Diastolic Filling and Function  CHAPTER 7 127

Cal=20mm NORMAL DIASTOLIC FUNCTION (Fig. 7-14)


0
Characteristic Features
.69
n Normal LA size.
n  Transmitral flow shows an E/A velocity ratio
between 1 and 2.
D n The E deceleration time is 150 to 200 ms.
n The tissue Doppler E′/A′ ratio is 1 to 2.
.69 n The pulmonary vein systolic to diastolic flow ratio
is 1 or more.
S
n 
The pulmonary vein a-velocity is less than 0.35
m/s and duration is less than 20 ms longer than
transmitral A duration.
Diastolic Filling Versus Diastolic Function
180 n  At a higher heart rate (shorter diastolic fill-
Figure 7-11  Color M-mode propagation velocity.  LV inflow is recorded ing time), the A velocity may be increased as
from an apical view using a color Doppler M-mode beam aligned along the it is superimposed on the E deceleration slope
center of the mitral annulus. Thus, the vertical axis is distance from the LA (at (Fig. 7-15).
about 160 mm depth on the scale) to the apex (at the top of the scale) with n  The transmitral E/A ratio decreases with age,
the horizontal axis indicating time, using an electrocardiogram for timing of
the cardiac cycle. Flow toward the transducer in diastole (D) represents LV
reversing at about age 50 years. Similarly, the pul-
filling with the slope of the edge of this signal (line) reflecting the velocity of monary vein diastolic flow declines, so that the sys-
the movement of blood from the annulus to the apex. S, Systole. tolic to diastolic ratio increases with age.
n A higher preload increases the transmitral E veloc-
CW:2MHz APEX MV MR dP/dt = 512 mmHg/sec ity; hypovolemia results in a lower velocity—with
Vmax 1 = 1.00 m/sec
1.0 Vmax 2 = 3.00 m/sec
∆t = 63 msec
Valsalva maneuver, E velocity falls transiently due
to reduced venous return (Fig. 7-16).
m/s n  Increased transmitral volume flow due to mitral
regurgitation increases the transmitral E velocity.
1 m/s
n Atrial contractile function affects LV filling, LA fill-
ing, and tissue Doppler signals (Fig. 7-17).

MILD DIASTOLIC DYSFUNCTION (IMPAIRED


3 m/s RELAXATION) (Fig. 7-18)
3.0
∆t n I ncreased LA diameter and volume.
n Impaired relaxation is typical of mild diastolic dys-
Figure 7-12  Rate of decline in LV pressure (or negative dP/dt ). This
measurement can be made from the mitral regurgitant jet velocity as velocity function due, for example, to hypertensive heart
decelerates, analogous to measurement of positive dP/dt from the rate of disease, ischemic disease, or an early infiltrative
acceleration in velocity. The pressure difference between 1 and 3 m/s (32 cardiomyopathy.
mmHg) is divided by the time interval (in seconds) measured between these
points on the velocity curve at 1 and 3 m/s to give the –dP/dt in mmHg/s.

Normal LV size, wall


thickness, and EF
Figure 7-13  Suggested algorithm for evaluation
of diastolic dysfunction on routine clinical stud-
Normal EF with EF  55% ies.  When LV size, wall thickness, and ejection frac-
LVH or LV dilation tion (EF) are normal, further evaluation of diastolic
Yes function is needed only if there is left atrial enlarge-
No No ment or an abnormal E/A ratio for age. In patients with
Normal LA size E/E  E/E   15 ventricular hypertrophy or dilation with a normal EF,
E  velocity adur  Adur  30 ms diastolic function should be fully evaluated, particularly
Yes DT if there is a clinical concern that diastolic dysfunction
No IVRT may account for symptoms. When the EF is reduced,
Normal E/A for age PVS / PVD the first step is to evaluate for elevated filling pres-
sures. If simple criteria for elevated filling pressures
Yes Yes are not present, a more complete evaluation of diastol-
ic function is appropriate. IVRT, Isovolumic relaxation
Normal diastolic Diastolic dysfunction* Elevated LV filling time; LVH, left ventricular hypertrophy. (From Otto, CM:
function (mild, moderate, severe) pressures Textbook of Clinical Echocardiography, ed 5, Elsevier,
2013, Philadelphia.)
128 CHAPTER 7  Ventricular Diastolic Filling and Function

40dB 2 •/+1/0/ 1 40dB 1 •/+1/0/ 1


PW Depth= 87mm PW Depth= 102mm
PW Gate= 2.0mm Leaflet PW Gate= 2.0mm
PW Gain= 8dB tips PW Gain= –1dB Annulus
MV DT = 191 msec
P1/2 time = 55 msec MV A Dur = 104 msec
PW:2MHz PW:2MHz

.80
E E
1.0
A

m/s
m/s
A .20 B
.69 40dB 2 •/+1/0/ 1 40dB 2 •/+1/0/ 1
PW Depth= 154mm PW Depth= 98mm
PW Gate= 2.0mm Pulmonary PW Gate= 2.0mm
PW Gain= 10dB PW Gain= 10dB
vein
RUPV A Vmax = 0.33 m/sec IVRT
.69
Pk Grad = 0.4 mmHg LV IVRT = 75 msec
PW:2MHz PW:2MHz

.80 .80

S D
MV

m/s m/s

Ao
.40 a .40
C D
Figure 7-14  An example of normal diastolic function.  A, The LV inflow curve at the mitral leaflet tips shows a normal E and A velocity with a deceleration
time (DT) of 191 ms. B, Inflow recorded at the annulus shows the duration of the atrial flow curve (104 ms) is the same as the duration of atrial reversal in
the pulmonary vein recording. C, The pulmonary vein flow also shows normal systolic and diastolic inflow signals. D, The isovolumic relaxation time (IVRT) is
normal at 75 ms. Ao, Aorta; MV, mitral valve.

1.0

m/s

Figure 7-15  Diastolic filling pattern changes with changes in the duration of diastole.  The pattern of LV filling across the mitral valve in this patient with
a variable R-R interval shows an E/A ratio greater than 1 on the longer R-R interval. However, the A velocities (arrows) are higher when superimposed on the E
deceleration slope on the shorter R-R intervals. There is fusion of the E and A velocities on the shortest diastolic intervals.
Ventricular Diastolic Filling and Function  CHAPTER 7 129

PW:2MHz

Sweep=25mm/s

Valsalva
1.0

m/s

Figure 7-16  Diastolic filling pattern changes with changes in preload.  LV inflow recorded at a slow sweep speed during the Valsalva maneuver shows a
gradual reduction in E velocity, due to a relative decrease in LV preload, but no change in A velocity. Thus, E/A ratio is dependent on preload.

PW:2MHz
.80

m/s

.20

Figure 7-17  Effect of PR interval on diastolic filling patterns.  In this patient with third-degree atrioventricular (AV) block, the height of the E velocity var-
ies with the timing of atrial contraction. When atrial contraction occurs in mid- to late diastole, a separate A velocity is seen (vertical arrow), but when atrial
contraction occurs in early diastole, a higher (summated) E velocity is seen (horizontal arrow).

n The decreased rate of early diastolic filling is associ- n I n addition to the findings seen with mild diastolic
ated with a reduced E velocity (reduced E/A ratio), a dysfunction, there is evidence for elevated filling
reduced E′/A′ ratio on tissue Doppler, reduced pul- pressures, including a higher peak (> 0.35 m/s)
monary vein diastolic flow, and a prolonged IVRT. and duration of the pulmonary vein a-velocity, an
n LV filling pressure may be normal with mild dia- increased E/E′ ratio (> 15), and a shorted E veloc-
stolic dysfunction, so pulmonary vein atrial veloc- ity deceleration time.
ity and duration are normal. n The LV filling velocity shows an apparently nor-
mal E/A ratio of 1 to 2 (pseudo-normal) that is
distinguished from a true normal by the tissue
MODERATE DIASTOLIC DYSFUNCTION Doppler showing an E′/A′ less than 1 and a short-
(PSEUDO-NORMALIZATION) (Fig. 7-19) ened E velocity deceleration time.
n  V relaxation is impaired and LV filling pressures
L n The change in the transmitral flow pattern with
are elevated with moderate diastolic dysfunction— Valsalva maneuver also can be used to identify
for example, due to dilated, hypertrophic, or restric- a pseudo-normal transmitral flow pattern; the E
tive cardiomyopathy. velocity decreases with pseudo-normalization.
130 CHAPTER 7  Ventricular Diastolic Filling and Function

31dB 1 •/+1/0/ 1 40dB 1 •/+1/0/ 1


PW Depth= 92mm PW Depth= 82mm
PW Gate= 2.0mm
PW Gate= 2.0mm
PW Gain= 3dB
PW Gain= –1dB

MV DT = 157 msec
Leaflet tips P1/2 Time = 46 msec

PW:2MHz PW:2MHz

1.0 1.0

m/s

m/s
A .20 A

31dB 1 •/+1/0/ 1 30dB 2 •/ 0/1/ 2


PW Depth= 104mm PW Depth= 85mm
PW Gate= 2.0mm PW Gate= 3.0mm
PW Gain= –24dB PW Gain= –19dB

Tissue Doppler
Tissue Doppler
PW:2MHz PW:2MHz

.05 .05

m/s m/s

B .15 B
.15
PW:2MHz
31dB 3 •/+1/0/ 1 1.0
PW Depth= 156mm
PW Gate= 2.0mm
PW Gain= 13dB

Pulmonary vein
PW:2MHz
.80

m/s
m/s
.20
.40 C
C
Figure 7-18  Example of mild diastolic dysfunction with impaired Figure 7-19  Moderate diastolic dysfunction (pseudo-normalization). 
early diastolic relaxation.  Characteristic findings are (A) an E/A ratio less Characteristic findings include (A) a mitral inflow curve with an E/A velocity
than 1 on the LV inflow curve, (B) a tissue Doppler early to late diastolic between 1 and 2 but a relatively steep deceleration time (157 ms), and (B)
velocity ratio less than 1, and (C) a pulmonary vein flow curve with a re- a tissue Doppler E′/A′ less than 1. Typically, the pulmonary vein flow signal
duced diastolic inflow curve but a relatively normal atrial reversal velocity shows greater systolic than diastolic flow and a prolonged duration and in-
and duration. creased velocity of the atrial reversal. However, in this case the pulmonary
venous flow signal (C) does not show these features, suggesting the degree of
diastolic function falls between mild-moderate and moderate (pseudo-normal)
as shown in the classification in the Echo Exam section.
Ventricular Diastolic Filling and Function  CHAPTER 7 131

SEVERE DIASTOLIC DYSFUNCTION LEFT ATRIAL PRESSURE ESTIMATES


(DECREASED COMPLIANCE) (Fig. 7-20) n  xact measurement of LA (or LV filling) pressure
E
n evere diastolic dysfunction is characterized by
S is not possible with echocardiography, but there
decreased compliance, in addition to impaired are several parameters that suggest significant ele-
relaxation, an enlarged LA and an elevated filling vation of LA pressures:
pressure. • Pulmonary vein atrial reversal velocity (PVa) more
n Decreased compliance means there is a greater than 0.35 m/s (Fig. 7-21)
increase in LV pressure for a given increase in LV • Pulmonary vein atrial reversal duration (adur) at least
volume compared with a normal ventricle. 20 ms longer than transmitral A duration (Adur)
n Although the E/A ratio is more than 2 and the recorded at the mitral annulus
E′/A′ ratio is more than 1, severe diastolic dysfunc- • Ratio of transmitral E velocity to myocardial tissue
tion is differentiated from normal by the higher E′ velocity more than 15 (Fig. 7-22)
E/A ratio, shorter IVRT, decreased deceleration • Pulmonary venous diastolic flow deceleration time
time (<150 ms), blunted pulmonary vein systolic less than 175 ms
flow, and increased pulmonary a-wave velocity and • E velocity deceleration time less than 150 ms
duration. • E/A ratio more than 2
n The E′ velocity is very low (<5 cm/s) with severe n  hen more than one parameter is consistent with
W
diastolic dysfunction. elevated LA pressure, the diagnosis is more certain.

45dB 2 •/+1/0/ 1 42dB 1 •/+1/0/ 1


PW Depth = 115mm PW Depth= 113mm
PW Gate= 3.0mm PW Gate= 3.0mm
PW Gain= 5dB PW Gain= –40dB

Leaflet tips Tissue Doppler


PW:1.75MHz PW:1.75MHz
1.5 .10

m/s

m/s

.50 .20
A B

45dB 3 •/+1/0/ 1 45dB 3 •/+1/0/ 1


PW Depth= 116mm PW Depth= 178mm
PW Gate= 3.0mm PW Gate= 3.0mm
PW Gain= 10dB PW Gain= 8dB

Pulmonary vein
PW:1.75MHz PW:1.75MHz HPRF

1.0
.50

S D
m/s
m/s
a

.50 .50
C D
Figure 7-20  Severe diastolic dysfunction.  Characteristic findings include (A) an LV inflow curve with an E/A > 2 and a short deceleration time, (B) a tissue
Doppler E′/A′ more than 1, (C) a short IVRT, and (D) reduced systolic flow compared with diastolic flow in the pulmonary vein with a pulmonary vein a-reversal
that is prolonged (>20 ms longer than transmitral A duration) and increased in velocity (≥0.35 m/s).
132 CHAPTER 7  Ventricular Diastolic Filling and Function

.64 40dB 3 •/+1/0/ 1 40dB 2 •/+1/0/ 1


PW Depth= 154mm PW Depth= 74mm
PW Gate= 3.0mm PW Gate= 2.0mm
PW Gain= 7dB PW Gain= 1dB

RUPV A Vmax = 0.47 m/sec MV Peak E = 1.15 m/sec


.64
Pk Grad = 5.3 mmHg
PW:2MHz PW:2MHz
1.0

1.0

S
D
m/s

a m/s
.50 A
Figure 7-21  Pulmonary vein recording in a patient with hypertrophic
40dB 1 •/+1/0/ 1
cardiomyopathy and severe diastolic dysfunction.  The atrial reversal du- PW Depth= 74mm
ration is prolonged with an elevated velocity of 0.47 m/s, suggesting markedly PW Gate= 2.0mm
elevated left ventricular filling pressures. D, Diastole; S, systole. PW Gain= –17dB

PW:2MHz

n In patients with atrial fibrillation, parameters of .10


diastolic function that do not rely on atrial contrac-
tion may be helpful, including deceleration time,
m/s
the E/E′ ratio, and the IVRT.
n With mitral valve stenosis or regurgitation, evalu-
ation of LV diastolic function and LA pressure
are problematic because transmitral filling reflects
mitral valve hemodynamics, rather than LV dia- .20
stolic function. B
Patterns of diastolic inflow are summarized in
Fig. 7-23 for patients with mild, moderate, and severe Figure 7-22  E/E′ as a marker of an elevated LV filling pressure. The
diastolic dysfunction in comparison to normal filling ratio of the transmitral E velocity to myocardial tissue Doppler velocity is
higher when filling pressures are elevated with a ratio more than 15 indicat-
patterns. ing a significant elevation. In this example, the ratio is 1.15 : 0.15 = 7.7,
suggesting normal filling pressures.
Ventricular Diastolic Filling and Function  CHAPTER 7 133

Diastolic Dysfunction

Normal Mild Moderate Severe

E/A 1–2 E/A < 0.8 E/A 0.8–2.0 E/A ≥ 2


LV inflow (m/s)

E DT

1.0 A

0
DT 150–200 DT > 200 DT 150–200 DT < 150

E/E ′ ≤ 8 E/E ′ ≤ 8 E/E ′ 9–14 E/E ′ ≥ 15


Tissue Doppler (m/s)

0.1
S
0

0.1 A′
E′
E ′ ≥ 0.1 E ′ < 0.08 E ′ < 0.08 E ′ < 0.05
IVRT (m/s)

IVRT 50–100 ≥ 100 60–100 < 60

S=D S>D S<D S<<D


Pulmonary vein (m/s)

PVS PVD
0.1

0
PVA
0.1

PVa < 0.35 PVa < 0.35 PVa ≥ 0.35 Adur – adur ≥ 30
Figure 7-23  Diagram comparing typical Doppler findings in patients with normal, mild, moderate, and severe diastolic dysfunction.  The top row
shows LV inflow with early (E) and atrial (A) phases of diastolic filling, the second row shows tissue Doppler recorded at the septal side of the mitral annulus
with the myocardial early (E′) and atrial (A′) velocities and the expected ratio of E/E′, the third row shows the isovolumic relaxation time (IVRT), and the bottom
row shows the pulmonary vein (PV) inflow pattern with systolic (S) and diastolic (D) antegrade flow and the pulmonary vein atrial (PVa) reversal of flow. (From
Otto, CM: Textbook of Clinical Echocardiography, ed 5, Elsevier, 2013, Philadelphia.)
134
CHAPTER 7  Ventricular Diastolic Filling and Function
THE ECHO EXAM
Diastolic Dysfunction
Quantitation of Diastolic Function
Parameter Modality TTE View TEE View Recording Measurements
LV inflow at leaflet Pulsed Doppler A4C with 2-3 mm sample High TEE 4-chamber view Parallel to flow E = Early diastolic filling velocity (m/s)
tips volume positioned at with sample volume at Normal expiration A = Filling velocity after atrial
mitral leaflet tips leaflet tips Low wall filters ­contraction (m/s)
E/A ratio
DT = Deceleration time (ms)
LV inflow at annulus Pulsed Doppler A4C with 2 mm sample High TEE 4-chamber view Parallel to flow, normal Adur = duration of atrial filling velocity
volume at mitral annulus with sample volume at mitral expiration, low wall filters (ms)
annulus
Myocardial tissue Pulsed Doppler A4C with 2-4 mm sample High TEE 4-chamber Very low gain settings E′ = Early diastolic filling velocity (m/s)
Doppler volume placed within view with 2-3 mm sample Low wall filters A′ = Filling velocity after atrial
basal segment of septal volume placed within ­contraction (m/s)
wall basal segment of septal wall E/E′=ratio of LV inflow E velocity to
tissue Doppler E′ velocity
IVRT Pulsed Doppler Anteriorly angulated A4C High TEE 4-chamber view Clear aortic closing click and IVRT = isovolumic relaxation
with 3-5 mm sample ­angulated towards aortic clear onset of transmitral flow, time (ms)
volume midway between valve with a 3-5 mm sample low wall filters
aortic and mitral valves volume midway between
aortic and mitral valves
Pulmonary venous Pulsed Doppler Right superior pulmonary Left superior pulmonary 2-3 mm sample volume PVS = peak systolic velocity (m/s)
inflow (color to guide vein in A4C view using vein from high TEE view 1-2 cm into pulmonary vein PVD= peak diastolic velocity (m/s)
location) color flow to visualize (all 4 veins can be used) PVa = peak atrial reversal velocity
flow adur = pulmonary vein atrial reversal
duration (ms)

A4C, Apical 4-chamber; TTE, transthoracic echocardiography.


Ventricular Diastolic Filling and Function  CHAPTER 7 135

Classification of Diastolic Dysfunction


Normal Mild (Grade I) Moderate (Grade II)† Severe‡ (Grade III)
Pathophysiology ↓ relaxation ↓ relaxation and ↓ compliance and
↑ LV-EDP ↑ ↑ LV-EDP
E/A ratio 1-2 < 0.8 0.8-2.0 ≥ 2.0
Valsalva ΔE/A < 0.5 ≥ 0.5 ≥ 0.5
DT (ms) 150-200 > 200 150-200 < 150
E′ velocity (cm/s) ≥ 10 <8 <8 <5
E/E′ ratio ≤8 ≤8 9-14 ≥ 15
IVRT (ms) 50-100 ≥ 100 60-100 ≤ 60
Pulm. vein S/D ≅1 S>D S<D S<<D
PVa (m/s) < 0.35 < 0.35* ≥ 0.35 ≥ 0.35
adur - Adur (ms) < 20 < 20* ≥ 30 ≥ 30
LA volume < 34 mL/m2 Mildly enlarged Moderately enlarged Severely enlarged
  

Modified from Nagueh SF, et al. Recommendations for the evaluation of left ventricular diastolic function by echocardiography. J Am Soc Echocar-
diogr 2009;22:107-133; Rakowki H, et al. Canadian consensus recommendations for the measurement and reporting of diastolic dysfunction by
echocardiography. J Am Soc Echocardiogr 1996;9:736-760; Yamada H, et al. Prevalence of left ventricular diastolic dysfunction by Doppler echo-
cardiography. J Am Soc Echocardiogr 2002;15:1238-1244; Redfield MM. Burden of systolic and diastolic ventricular dysfunction in the community.
JAMA 2003;289:194-202; Lester SJ, et al. Unlocking the mysteries of diastolic function. J Am Coll Cardiol 2008;51:679-689.
*Pulmonary veina duration and velocity may be increased if filling pressures are elevated.
†E/A with Valsalva is < 1.0.
‡An additional grade of irreversible severe dysfunction is characterized by the absence of a decrease in E velocity with the strain phase of the

Valsalva maneuver.

Example The E/A ratio is greater than 1 but the E′/A′ ratio
A 62-year-old man with amyloidosis has an echocar- is less than 1, indicating a pattern of pseudonormal-
diogram which shows a symmetric increase in wall ization suggestive of moderate diastolic dysfunction
thickness with an ejection fraction of 52%. The fol- with decreased compliance. Moderate diastolic dys-
lowing parameters of diastolic function are recorded: function is confirmed by the short IVRT and rela-
tively short deceleration time. There also is evidence
E-velocity 1.0 m/s of elevated filling pressures with an equivocal E/E′
A-velocity 0.6 m/s ratio of 14, but a PVa > 0.35 m/s and with the dura-
Deceleration time (DT) 160 ms tion of pulmonary vein atrial flow minus the duration
Adur 130 ms of atrial flow at the mitral annulus greater than 20 ms.
E′ 0.07 m/s
E′/A′ ratio <1
IVRT 40 ms
PVS /PVD <1
PVa 0.4 m/s
adur 155
136 CHAPTER 7  Ventricular Diastolic Filling and Function

SELF-ASSESSMENT QUESTIONS

Question 1 Question 4
A 60-year-old man under evaluation for liver trans- An echocardiogram is ordered and the following
plant undergoes a routine transthoracic echocardio- images of the left atrium are obtained:
gram. On the transmitral Doppler inflow the mitral Body surface area: 1.6 cm2
peak E wave velocity is 1.1 m/s and A-velocity is 0.4 LA area apical 4ch view: 17 cm2
m/s. Data from myocardial tissue Doppler sampling LA area apical 2ch view: 15 cm2
is provided below (Fig. 7-24). This patient’s data are LA length: 5 cm
consistent with: Calculate the indexed left atrial volume: __________
A. Decreased LV compliance
B. Normal myocardial function Question 5
C. Impaired LV relaxation In Question 4, you conclude that diastolic function is
D. Normal LA pressure most likely:
A. Normal diastolic function
20 B. Impaired LV relaxation
C. Restrictive filling
10

Question 6
cm/s
What feature of the color Doppler M-mode tracing
–10 (as shown in Fig. 7-26) is MOST useful for LV dia-
stolic assessment?
–20 A. Signal duration
B. Maximal signal distance from mitral valve
Figure 7-24  C. Slope of signal from mitral valve opening
D. Signal intensity
Question 2
A patient is referred for echocardiography and the fol-
lowing image is obtained (Fig 7-25).
Progression in the Doppler signal across several car- Cal=20mm
0
diac cycles is accounted for by:
A. Shifts in transducer position
B. Change in LV loading
C. Doppler signal optimization
D. Underlying atrial fibrillation

1.0

m/s

180

Figure 7-26 
Figure 7-25 

Question 3
Which of the following, if present, LEAST affects
echocardiographic Doppler assessment of LV dia-
stolic function?
A. Atrial fibrillation
B. Mitral regurgitation
C. Pulmonary hypertension
D. Mitral stenosis
Ventricular Diastolic Filling and Function  CHAPTER 7 137

Question 7
A patient with exertional dyspnea is referred for A.
Calculate the dP/dt:________________________
echocardiography. A continuous wave spectral Dop- B. Calculate the –dP/dt:_____________________
pler sample from the mitral regurgitant jet is shown C. This is consistent with (normal/abnormal) dia-
in Fig. 7-27. stolic function

2.0 2.0

m/s m/s

A B
6.0 6.0

Figure 7-27

Question 8 Question 9
Review the M-mode image shown in Fig. 7-28. Which Regarding echocardiographic assessment of diastolic
feature of the image is MOST helpful in evaluating function, indicate the position for sample volume
LV diastolic pressure? acquisition for each of the following indices (some
A. Late diastole mitral valve motion of the indices may require data from more than one
B. Peak early mitral valve diastolic displacement position) (Fig. 7-29):
C. Duration diastolic leaflet excursion 1. Isovolumic relaxation time
D. E-point septal separation 2. E/A ratio
3. –dP/dt calculation
4. E/E′ ratio
Cal=20mm
5. E wave deceleration time
0 6. Mitral A wave duration
7. Left atrial inflow velocity

A
B

180
C
Figure 7-28 
E D

Figure 7-29 
138 CHAPTER 7  Ventricular Diastolic Filling and Function

Questions 10-12 Question 11:


For the next three questions, match the images shown
in Fig. 7-30, Fig. 7-31, and Fig. 7-32 with the most
likely diagnosis: 1.5
A. Normal diastolic function
B. Impaired LV relaxation
C. Pseudonormal (moderate diastolic dysfunction)
D. Restrictive LV filling

Question 10: m/s

2.0

.10

m/s
m/s

.50

.20

.15

Figure 7-31 
m/s
Question 12:

.20 1.0

Figure 7-30 

m/s

.50

m/s

.50

Figure 7-32 

  
Ventricular Diastolic Filling and Function  CHAPTER 7 139

ANSWERS

Answer 1: A atrial pressure and/or the transmitral gradient will


The transmitral Doppler inflow E velocity is much affect diastolic function assessment. For mitral regur-
higher than A velocity (ratio 1.1/0.4 = 3), consistent gitation, increased transmitral volume flow due to
with either a normal inflow pattern or decreased increases LA pressure and the transmitral E-velocity,
passive LV compliance. The image shown is a tissue limiting Doppler derived diastolic function assess-
Doppler tracing of myocardial velocity sampled from ment using the E/A ratio alone. For mitral stenosis,
the interventricular septum just below the mitral valve the increased pressure gradient at the valve level is
annulus, showing an early diastolic tissue velocity (E′ ) directly due to inflow obstruction. Atrial fibrillation
of 0.06 m/s (below the baseline) and an atrial (A′ ) results in the loss of the atrial contribution to dia-
velocity of ∼0.08 m/s. These data provide evidence stolic filling and absence of the “A” wave. However,
for elevated left atrial pressure, reflected in the ratio in patients with atrial fibrillation, the transmitral E
of early transmitral Doppler inflow velocity (E) to the to myocardial velocity (E′ ) ratio (E:E′ ratio) remains a
myocardial tissue Doppler peak (E′ ) wave velocity of reasonable gauge of LA pressure.
1.1/0.06 = 18. A ratio of over 15 is consistent with
increased left atrial pressure. In impaired LV relax- Answer 4: 27 mL/cm2
ation, LV filling relies more heavily on the latter part The left atrial volume can be calculated from the biplane
of diastole, during atrial contraction with a mitral area length equation utilizing left atrial area traced from
Doppler inflow E/A ratio < 1, with a less pronounced the apical 4-chamber and apical 2-chamber views. Left
early filling mitral E wave velocity, typically less than atrial area tracings should be performed at maximal left
1 m/s. atrial size, typically end systole, with image planes that
avoid atrial foreshortening. The left atrial area tracings
Answer 2: B should be horizontal across the mitral annulus plane,
This is a pulse wave Doppler tracing taken at the level excluding the left atrial appendage and the confluence
of the mitral valve leaflet tips from an apical view. In of the pulmonary veins, and left atrial length is the
the beginning of the Doppler sample shown, the early perpendicular distance from the middle of the mitral
(E) wave in diastole is higher velocity than the late (A) annular plane to the left atrial wall.
wave velocity. At the third cardiac cycle, the patient LA volume = (8⁄3π) * (LA area 4ch) * (LA area 2ch)/
performed a Valsalva maneuver, transiently decreas- LA length. In this case, the calculation is 43 mL.
ing left ventricular preload, and the volume of trans- Indexed LA volume is the LA volume divided by
mitral flow in early diastole was reduced with now a the BSA: 43 mL/1.6cm2 = 27 mL/cm2
relatively larger contribution of atrial contraction to
LV filling. With relaxation of Valsalva, the transmitral Answer 5: A
filling pattern returned to baseline (not shown). The An indexed LA volume of 27 mL/cm2 is normal (nor-
transducer remained fixed, recording mitral inflow mal < 32 mL/cm2). Left atrial pressure is increased
for the duration of the sample provided. The rhythm in patients with diastolic dysfunction, and as a conse-
is not paced nor is it atrial fibrillation, with atrial quence, left atrial size/volume is typically increased.
contraction evidenced by the “A” wave throughout
the sample. The Doppler signal quality is reasonable Answer 6: C
and optimization during sample acquisition was not This is a color Doppler M-mode tracing of transmitral
performed. flow along the center of the mitral annulus taken from
an apical view. With the mitral valve open in diastole,
Answer 3: C flow towards the transducer is recorded. The slope
Diastolic dysfunction with either impaired LV relax- of the color Doppler signal from the point of valve
ation or decreased ventricular compliance impedes opening (color propagation velocity [Vp]) reflects the
LV filling, leading to increased left atrial pressure. velocity of LV inflow from the mitral valve plane to the
Doppler evaluation of diastolic function includes apex. A steep slope is consistent with normal diastolic
evaluation of transmitral filling (early “E” and late “A” function and a flatter slope suggests higher LV diastolic
diastolic filling), as well as tissue myocardial velocities pressure. The maximal signal distance from mitral
in the myocardial wall (E′ and A′, respectively), mea- valve is the distance where LV inflow velocities were
sured 1 cm apically from the mitral valve annulus. detected. Increased turbulent flow could increase the
While increased left atrial pressure may lead to pul- maximal signal distance from mitral valve detected.
monary hypertension, particularly with superimposed The color signal duration corresponds to the time of
intravascular volume overload, the reverse is not true; flow in diastole and the signal intensity reflects the
pulmonary hypertension does not cause LV diastolic turbulence of flow within the jet; both are not mea-
dysfunction. Cardiac abnormalities which affect left sures of LV diastolic function.
140 CHAPTER 7  Ventricular Diastolic Filling and Function

Answer 7 Cal=20mm
0
A. Using panel B, the dP/dt (rate in pressure change)
is calculated from the early acceleration portion
of the mitral regurgitant jet. Using the simpli-
fied Bernoulli equation, pressure gradients are
calculated at 1 m/s and 3 m/s (lines indicated
on the image). The relative change in pressure
gradient is then divided by the time difference
for the velocity to move from 1m/s to 3m/s.
4(3m/s)2-4(1m/s)2/Δt = (36-4)/0.025sec =
1280 mmHg/s
This is consistent with normal systolic function
(normal > 1000 mmHg/s)
B. Analagous to dP/dt, the rate of pressure change
in the deceleration of the mitral regurgitant jet 180
reflects diastolic function. Using panel A, the
−dP/dt (rate in pressure change) is calculated Figure 7-33 
from the deceleration portion of the mitral
regurgitant jet. Again, pressure gradients are 3. –dP/dt calculation (C)
calculated at 1 m/s and 3 m/s (lines indicated 4. E/E′ ratio (A,B)
on the image) using the simplified Bernoulli 5. E wave deceleration time (B)
equation. The relative change in pressure gra- 6. Mitral A wave duration (C)
dient is then divided by the time difference for 7. Left atrial inflow velocity (D)
the velocity to move from 3 m/s to 1 m/s. IVRT is recorded from an anteriorly angulated api-
4(3m/s)2-4(1m/s)2/Δt = (36-4)/0.044 sec = cal 4-chamber view midway between the aortic and
727 mmHg/s mitral valves and is the time duration between aor-
C. This is consistent with abnormal diastolic func- tic valve closure and mitral valve opening. IVRT is
tion (normal >1000 mmHg/s) prolonged in patients with impaired LV relaxation
and is shortened (<50 ms) in patients with decreased
Answer 8: A LV compliance. LV inflow (E and A) velocities are
This is an M-mode tracing taken from the parasternal recorded at the point where inflow occurs, the mitral
long-axis view at the mitral valve leaflet tips. In the mid- leaflet tips (B), and the mitral annulus (C). At the
portion of the image, the anterior and posterior mitral mitral valve tips, the highest, or peak, velocity signal
valve leaflets are seen separating and opening during for both the E and A waves are measured, and the
diastole. Early diastolic motion of the anterior mitral E/A ratio can be calculated. An E:A ratio less than
valve leaflet is concurrent with early left ventricular fill- 1 indicates impaired LV relaxation. An E:A ratio
ing. Mid-diastolic anterior motion of the leaflet is con- greater than 1 may indicate either normal diastolic
current with atrial contraction and later diastolic filling. function or decreased passive LV compliance, and
Just before systole, there is a very late anterior displace- additional data are needed. The early E wave decel-
ment, or “bump” in the motion of the anterior mitral eration time is also measured from tracings taken at
valve leaflet (Fig. 7-33). This “B-bump” is indicative of the mitral valve tips, and is prolonged in patients with
elevated LV end-diastolic pressure. Peak early mitral impaired LV relaxation and shortened (<150 ms) in
valve diastolic anterior displacement and E-point sep- patients with decreased LV compliance. The mitral
tal separation are a reflection of LV chamber size. inflow atrial “A” wave duration is obtained from the
With dilated cardiomyopathy, apical tethering of the mitral annular position (C) and is compared to left
mitral valve leaflet tips may hinder leaflet excursion atrial inflow a-wave duration (pulmonary venous
and increase the distance between the anterior mitral flow) (D) where pulmonary venous a-wave velocities
valve leaflet and the interventricular septum. In dilated less than 0.35 m/s is consistent with normal left atrial
cardiomyopathy, diastolic function is not normal, LV pressure. The –dP/dt is calculated from the decelera-
diastolic pressure may not be significantly elevated of tion slope of the CW mitral regurgitant jet, analogous
volume status is euvolumic. The duration diastolic leaf- to the dP/dt calculation for assessment of LV systolic
let excursion reflects the time in the cardiac cycle spent function. The CW is aligned along the mitral inflow.
in diastole and is not reflective of LV diastolic function. Tissue Doppler myocardial velocity samples from the
septum (A) (E′ velocity) and the mitral E wave velocity
Answer 9 recorded at the tips (B), are a reflection of left atrial
1. Isovolumic relaxation time (E) pressure, with an E:E′ ratio over 15 consistent with
2. E/A ratio (B) increased left atrial pressure.
Ventricular Diastolic Filling and Function  CHAPTER 7 141

Answer 10: A The E wave deceleration slope is shortened, consis-


tent with rapid equalization of transmitral pressure
The top portion of the image (see Fig. 7-30) is of
gradient during mitral valve opening. The bottom
mitral inflow at the level of the mitral valve tips. The
portion of the image shows septal myocardial tissue
sample is E wave dominant with an E/A ratio greater
velocity, E′, reduced at 0.05, indicating decreased
than 1. The peak E wave velocity is ∼1 m/s. The bot-
myocardial motion. The E/E′ ratio of 28 is consistent
tom portion of the image is a tissue Doppler sample
with severely increased left atrial pressure.
from the septum. The peak e′ wave velocity is ∼0.08
m/s. The E/E′ ratio is 12, consistent with normal left Answer 12: B
atrial pressure. The pulmonary venous atrial wave
The LV mitral inflow Doppler image shows reversal
duration is shorter than the mitral atrial duration,
of the peak E/A velocities. The E wave deceleration
suggesting normal LA pressure. The findings in sum
slope is prolonged, consistent with impaired LV relax-
support a diagnosis of normal diastolic function.
ation. The second image shows the pulmonary vein
Answer 11: C inflow. The systolic wave is dominant and the pulmo-
nary atrial reversal wave is prolonged with a high peak
The top portion of the image shows LV inflow
velocity (∼0.4 m/s), implying increased LA pressure.
Doppler with a normal E/A velocity ratio of ∼1.8.
8 Coronary Artery Disease
REVIEW OF CORONARY ANATOMY AND LEFT Complications of Acute Myocardial Infarction
VENTRICULAR WALL SEGMENTS Evaluation of the Patient with Recurrent Chest Pain
STEP-BY-STEP APPROACH after Myocardial Infarction
Stress Echocardiography Evaluation of the Patient with a New Systolic
Basic Principles Murmur after Myocardial Infarction
Prepare for the Stress Echo Evaluation of the Patient with Hypotension or
Evaluate Regional and Global LV Systolic Function Cardiogenic Shock after Myocardial Infarction
at Rest Evaluation of Late Complications of Myocardial
Perform the Stress Protocol Infarction
Evaluate Regional and Global LV ­Systolic Function End-stage Ischemic Disease
at Peak Heart Rate Evaluate Global LV Systolic and Diastolic Function
Monitor Patient Recovery Evaluate Regional LV Systolic Function
Review and Interpretation of the Stress Study Evaluate RV Size and Systolic Function
Acute Coronary Syndromes Estimate Cardiac Hemodynamics
Basic Principles Identify and Evaluate any Associated Valve Disease
Evaluate Regional Ventricular Function THE ECHO EXAM
Estimate or Measure Ejection Fraction SELF-ASSESSMENT QUESTIONS
Consider Alternate Causes of Chest Pain
Evaluate Cardiac Hemodynamics

REVIEW OF CORONARY ANATOMY AND LEFT v  KEY POINTS


VENTRICULAR WALL SEGMENTS o  he ventricle is divided into basal, midventricu-
T
n  valuation of coronary disease by echocardiogra-
E lar, and apical segments, plus the tip of the apex.
phy (echo) is based on visualization of endocardial o A distal coronary stenosis results in apical

motion and wall thickening. abnormalities; a midcoronary lesion results in
n To describe regional myocardial function, the left midventricular and apical wall motion changes;
ventricle (LV) is divided into segments that corre-
spond to the coronary artery blood supply (Fig. 8-1).
n Myocardial infarction results in thinning and
 LCA
­akinesis of the affected regions. With myocardial
ischemia, wall motion may be normal at rest.
n The ostia of the right and left main coronary
arteries often can be identified, but direct visu-
alization of distal coronary anatomy by echo is
limited (Fig. 8-2 and Fig. 8-3).
Ao
1 Four- 2 Two- 3 Long-
chamber chamber axis

LA
4 Base 5 Mid 6 Apex

RCA RCA or Cx
LAD LAD or Cx
Cx RCA or LAD
Figure 8-1  Coronary artery anatomy. Typical coronary artery distribu-
tion of blood flow shown in the apical and parasternal short-axis views. Cx, Figure 8-2  Left main coronary artery. The left main coronary artery
Circumflex; LAD, left anterior descending; RCA, right coronary artery. (From (arrow) is visualized on TTE arising from the aorta anterior to the left atrium
Lang RM, et al. J Am Soc Echocardiogr 2015;28:1-39; Otto, CM: Textbook in a transthoracic parasternal short-axis view just above the aortic valve
of Clinical Echocardiography, ed 5, Elsevier, 2013, Philadelphia) plane. Ao, Aorta; LCA, left main coronary artery.

142
Coronary Artery Disease  CHAPTER 8 143

and proximal coronary disease results in abnor-


malities that extend from the base to the apex. v  KEY POINTS
o In the short-axis plane, the LV is divided into six o  he accuracy of a stress echo correlates with
T
segments: anterior, anterior-lateral, inferior-lateral, the stress load achieved. Typically, the goal is
inferior, inferior-septal, and anterior-septal. a peak heart rate at least 85% of the patient’s
o The left anterior descending coronary supplies maximum predicted heart rate.
the entire anterior wall and anterior septum o Comparison of resting and stress images is facil-
and typically extends to supply the apical seg- itated by acquiring images in a cine loop format,
ment of the inferior septum and the tip of the gated from the onset of the QRS to include the
apex. same number of frames for each image.
o The right coronary artery supplies the basal and o Because ischemia may be induced with the

midventricular segments of the inferior septum stress protocol, appropriate medical supervision
and the entire inferior wall and sometimes sup- and monitoring is essential for patient safety
plies the inferior-lateral wall. and to promptly treat any complications of the
o The circumflex coronary artery supplies the
 procedure.
entire anterior-lateral and inferior-lateral walls.

Baseline echo Resting wall


STEP-BY-STEP APPROACH images motion abnormalities
Stress Echocardiography
Basic Principles Exercise duration
n LV global and regional function is normal at rest,
BP, HR
Exercise test
Symptoms
even when significant coronary artery disease is ECG changes
present.
n With an increase in myocardial oxygen demand,
myocardial ischemia is evidenced by reversible Immediate post Compare baseline
regional hypokinesis or akinesis. or maximal and stress images on
n 
The basis of a stress echo is a comparison of exercise images quad-screen cine loop
images of the LV acquired at rest and after Figure 8-4  Flow chart of treadmill stress echocardiography.  BP, Blood
induction of myocardial ischemia, either with pressure; ECG, electrocardiogram; HR, heart rate. (From Otto, CM: Textbook
exercise or pharmacologic intervention (Fig. 8-4 of Clinical Echocardiography, ed 5, Elsevier, 2013, Philadelphia.)
and Fig. 8-5).

Baseline echo

RCA

Dobutamine infusion
Increase dose
and repeat echo
every 3 min.
Repeat echo

Protocol Ischemia on Intolerable


Ao complete echo, ECG, or side effects
by symptoms

Compare baseline,
low-dose, maximal
Recovery echo dose, and recovery
echo images in
quad-screen cine
loop.
Figure 8-3  Right coronary artery (RCA).  The RCA (arrow) is seen in a Figure 8-5  Flow chart of the protocol for dobutamine stress echocardiog-
transthoracic parasternal short-axis view arising from the aorta (Ao) by slight raphy.  (From Otto, CM: Textbook of Clinical Echocardiography, ed 5, Elsevier,
adjustment of the image plane. 2013, Philadelphia.)
144 CHAPTER 8  Coronary Artery Disease

Step 1: Prepare for the Stress Echo stress is preferred because of the additional
n  The patient is instructed not to take beta-blocking information gained regarding hemodynamics
medications the day before and day of the stress test. and symptoms.
n The reason for the stress study and the patient his- o A pharmacologic (usually dobutamine) stress

tory is reviewed, followed by a directed physical echo is preferred in patients unable to walk on
examination. a treadmill or use a supine bicycle due to ortho-
n Informed consent is obtained for both the exercise pedic or vascular problems and in some spe-
and pharmacologic stress echo. cific patient subgroups, such as those who have
n  Patient monitoring includes continuous 12-lead undergone heart transplantation.
electrocardiogram (ECG) monitoring (with a
recording at each stress stage) and intermittent Step 2: Evaluate Regional and Global Left
blood pressure measurement under the supervi- Ventricular Systolic Function at Rest
sion of a qualified medical professional, in a pro- n V global and regional function is evaluated in
L
cedure room with resuscitation equipment and parasternal long- and short-axis views and in apical
medications readily available. 4-chamber, 2-chamber, and long-axis views (Fig. 8-6).
n When needed, an intravenous line is placed for infu- n The function for each myocardial segment is graded
sion of dobutamine and/or use of contrast echo. as hyperdynamic, normal, hypokinetic, akinetic,
or dyskinetic based on the degree of endocardial
v  KEY POINTS motion and wall thickening (Table 8-1).
o  ny potential contraindications or risk factors
A n Overall LV ejection fraction is visually estimated
for the stress study are identified and discussed or (preferably) measured using the apical biplane
with the referring health care provider before approach.
beginning the test.
o The risks and benefits of the stress echo study v  KEY POINTS
are discussed with the patient, in the context o  he four standard views (with the long-axis
T
of the patient’s medical history and cardiac in apical or parasternal, whichever is best) are
function. recorded in cine loop format. A beat with clear
o Because the cardiac sonographer’s attention
 definition of endocardial borders and optimal
is focused on image acquisition, patient moni- image plane alignment chosen for each view.
toring typically is performed by an additional o Depth is reduced to maximize LV image size,
health care professional. including the mitral annulus but not the left
o The rationale for a pharmacologic versus exer- atrium (LA). The same depth and sector width
cise stress echo is reviewed. Usually, exercise is used for the stress images.

A4C A2C

Figure 8-6  Baseline stress echo images. 


In a cine loop quad-screen format, apical
4-chamber (A4C), apical 2-chamber (A2C)
and apical (or parasternal) long-axis (A-long)
and parasternal short-axis (PSAX) views are
recorded. Several beats are acquired and
the best image saved. The cine loop only A-long PSAX
includes systole, to allow matching with the
stress images. Images are again acquired at
peak stress (with dobutamine) or immediate-
ly after stress (with exercise) in the same im-
age planes. The rest and exercise images are
then matched by view to allow comparison of
the same myocardial segments on side-by-
side views. Because a set number of frames
is recorded and there is little change in the
duration of systole with stress, the rest and
exercise images move at the same speed,
despite the difference in heart rates.
Coronary Artery Disease  CHAPTER 8 145

o If endocardial definition is suboptimal, left- n  upine bicycle exercise provides a lower work-
S
sided echo contrast is used to improve evalua- load, but images can be acquired during exer-
tion of regional endocardial motion (Fig. 8-7). cise using a dedicated stress echo stretcher and
o The ECG leads and gain are adjusted to show bicycle.
a clear signal with an adequate QRS height for
accurate electrocardiography gating (Fig. 8-8). v  KEY POINTS
o  ith treadmill exercise, ensuring the patient
W
Step 3: Perform the Stress Protocol can rapidly move from the treadmill to the echo
Exercise stress stretcher is important.
n Any standard exercise protocol can be used
 o In addition to echo images, the heart rate and
with ECG and blood pressure monitoring. blood pressure response to exercise, patient
n Upright treadmill exercise provides the high- symptoms, arrhythmias, and septal thickness
est workload, but images can only be obtained (ST) segment changes are important clinical
after exercise, so rapid image acquisition is parameters (Fig. 8-9).
essential. o The endpoint for a maximal exercise stress

study is when the patient cannot exercise fur-
ther due to shortness of breath, leg fatigue, or
TABLE 8-1 Qualitative Scale for
other symptoms.
Assessment of Segmental Wall
o The exercise test also is stopped for any decline
Motion on Echocardiography
in blood pressure, significant arrhythmias,
Wall Motion Definition excessive increase in blood pressure, or signifi-
Normal Normal endocardial inward cant ST-segment depression.
motion and wall thickening in
systole.
Hypokinesis Reduced amplitude (<5 mm) and
velocity of endocardial motion
and wall thickening in systole.
Delay in the onset of contrac-
tion and relaxation.
Akinesis Absence of inward ­endocardial
motion (<2 mm) or wall
­thickening in systole.
Dyskinesis Outward motion or bulging of
the segment in systole, usually Figure 8-8  ECG triggering.  Schematic of an appropriate electrocardio-
associated with thin, scarred graphic signal with little noise and a QRS height greater than the T wave,
myocardium. allowing accurate triggering for digital image acquisition and an example
with the T wave equal in height to the QRS signal so that both signals will
Aneurysmal Diastolic contour abnormality trigger image acquisition, resulting in very short cine loops that do not in-
and dyskinesis. clude the full cardiac cycle.

4CH

LV

Figure 8-7  Contrast enhancement. When


LA endocardial definition is suboptimal (left),
echo contrast enhancement is used (right)
to improve identification of the endocardium
(arrows) and evaluation of regional and glob-
al function with stress echocardiography.
146 CHAPTER 8  Coronary Artery Disease

Dobutamine stress o  T-segment depression with dobutamine is not


S
n  A typical dobutamine stress protocol is to infuse diagnostically useful. ST-segment elevation is
intravenous dobutamine beginning at 5 mcg/kg/ rare but is predictive of significant coronary
min (if there are resting wall motion abnormalities) disease.
or 10 mcg/kg/min, with an increase by 10 mcg/ o Both ventricular and atrial arrhythmias may be
kg/min every 3 minutes to a maximum dose of precipitated by dobutamine and require prompt
40 mcg/kg/min. cessation of dobutamine infusion.
n Atropine in increments of 0.25 mg (maximum 1 mg o When needed, the effects of dobutamine can be
total) may be added to achieve target heart rate, if reversed with a rapidly acting intravenous beta
needed. blocker, such as esmolol or metoprolol.
n  The primary endpoint is a heart rate 85% of the o Improvement in wall motion at low-dose dobu-
maximum predicted heart rate for age. tamine of a myocardial segment that is abnor-
n Other endpoints include: mal at rest is evidence for myocardial viability.
• 
Maximum dose allowed by the protocol o A biphasic response is when a myocardial seg-
• 
Definite wall motion abnormality in two or more ment that is abnormal at rest shows increased
adjacent segments wall thickening at low-dose dobutamine (viabil-
• 
Fall in systolic blood pressure to less than 100 mmHg ity) and then worsening of wall motion at high
or increase in blood pressure to more than 200 mmHg dose (ischemia) (Table 8-2).
• 
Diastolic blood pressure more than 120 mmHg
• 
Significant arrhythmia Step 4: Evaluate Regional and Global Left
• 
Patient discomfort Ventricular Systolic Function at Peak Heart
Rate
v  KEY POINTS n  ine loop images of the ventricle are acquired
C
o  areful monitoring of heart rate, blood pres-
C at (or immediately after) peak stress using the
sure, electrocardiographic findings, and symp- same four image planes as the baseline images
toms is needed. (see Fig. 8-6).
o Maximum predicted heart rate is roughly 220 n The image depth, sector width, and electrocardi-
minus the patient’s age. ography gating on the stress images are the same
o About 10% of patients will have a fall in blood as on the baseline images.
pressure, which may necessitate ending the n Rest and exercise images are compared side by side
dobutamine infusion. in the cine loop format (Fig. 8-10).

BASELINE Lead MAX ST Lead


Exercise Stage 1 92 bpm ST @ 10mm/mv ST Exercise Stage 4 166 bpm ST @ 10mm/mv ST
00:00 1.7 80ms postJ Slope 10:20 12.2 220/78 80ms postJ Slope

I aVR V1 V4 I aVR V1 V4
0.6 mm -0.3 0.9 0.9 0.3 0.4 1.4 -1.5
0.6 mV/s -0.8 0.5 0.8 0.9 -1.0 1.1 1.1

II aVL V2 V5 II aVL V2 V5
0.1 0.6 2.8 0.5 -1.3 0.9 3.1 -2.1
0.5 0.2 1.8 0.7 1.0 0.5 3.4 0.1

III aVF V3 V6 III aVF V3 V6


-0.6 -0.3 2.2 0.1 -1.6 -1.4 1.9 -1.8
-0.1 0.2 1.6 0.5 0.2 0.7 3.5 -0.2

Figure 8-9  Exercise stress echocardiogram.  The 12-lead resting ECG leads are shown on the left and the stress ECG on the right. The numbers below
each averaged ECG lead show the amount of ST depression (in mm) and the slope of the ST segment for each lead. In this 42-year-old man with multiple
cardiac risk factors and chest pain symptoms, there is a 1.5- to 2-mm flat ST depression in the inferior and lateral leads consistent with myocardial ischemia.

TABLE 8-2 Patterns of Wall Motion with Dobutamine Stress Echo


Normal Ischemia Stunned or Hibernating Infarction
Baseline Normal Normal Hypokinetic or akinetic Hypokinetic or akinetic
Low dose Normal Normal Improved Hypokinetic or akinetic
High dose Hyperkinetic Hypokinetic or akinetic Hypokinetic or akinetic Hypokinetic or akinetic
Coronary Artery Disease  CHAPTER 8 147

v  KEY POINTS Step 5: Monitor Patient Recovery


o  ith exercise stress, the images are obtained as
W n  The patient is monitored until all symptoms or
quickly as possible after exercise. wall motion abnormalities (if any) resolve and
o With pharmacologic stress, images are acquired heart rate has returned to normal (<100 bpm).
at each dosage stage, as well as at peak dose and n If symptoms or wall motion abnormalities were seen
heart rate. at peak-dose dobutamine, heart rate may be slowed
o If endocardial definition is suboptimal, left-
 more rapidly with a short-acting beta blocker.
sided echo contrast is used to improve evalua-
tion of regional endocardial motion. v  KEY POINTS
o Several cine loops are quickly acquired in each o I schemia is reversible so that stress-induced wall
view with subsequent selection of the best motion abnormalities quickly resolve as heart
image to compare to the baseline images. rate declines.
o Using electrocardiography gating and the same o Poststress images are recorded to document that
cine loop length for rest and exercise images LV global and regional function has returned to
results in a similar timing of contraction on baseline on completion of the study.
both images. o If ischemia is induced, as evidenced by chest
o The normal response to exercise or dobuta- discomfort or wall motion abnormalities, a
mine stress is hyperkinesis of all segments with a short-acting beta blocker, such as esmolol, is
decrease in ventricular chamber size (Fig. 8-11). used to reduce heart rate and relieve symptoms.
o Each myocardial segment is graded as hyperki-
netic, normal, hypokinetic, akinetic, or dyskinetic. Step 6: Review and Interpretation of the
Stress Study
n 
Baseline and stress echocardiographic images are
reviewed in a side-by-side cine loop format using a
Diastole

REST

A
A

Systole

STRESS

B
Figure 8-10  Inducible ischemia. Resting myocardial wall motion was B
normal in this 56-year-old man with exertional chest discomfort. The im-
mediate images after stress with the apical 4-chamber view at end-diastole Figure 8-11  Normal decrease in chamber size with stress. These
(A) and end-systole (B) show akinesis of the apical lateral wall and inferior parasternal short-axis end-systolic images at rest (A) and maximum-dose
septum. These findings are consistent with inducible ischemia in the territory dobutamine (B) demonstrate the normal decrease in ventricular size with
of the distal left anterior descending coronary artery. dobutamine stress.
148 CHAPTER 8  Coronary Artery Disease

systemic approach to grading wall motion for each


myocardial segment.
n The stress study interpretation depends on inte- REST
gration of clinical (symptoms, exercise duration),
hemodynamic (blood pressure, heart rate), electro-
cardiographic (ST changes and arrhythmias), and
ECG data. LV
v  KEY POINTS
o  he stress echo report includes the following
T
minimal elements:
Exercise

o duration or maximum dobutamine/
atropine dose
Heart rate and blood pressure at baseline and
o A
maximal stress
o ECG ST-segment changes or arrhythmias
o Symptoms
STRESS
o Resting global and regional LV systolic function
o Global and regional LV systolic function at maxi-
mal stress
o Integration of these data to indicate study qual-
ity (images and maximum stress achieved), the
likelihood of coronary disease, and the probable
affected vessels
o  he maximum stress achieved is a key element
T
in interpretation; typically the study is consid-
ered nondiagnostic unless the maximum heart
rate is at least 85% of the maximum predicted
heart rate for that patient. B
o An inducible wall motion abnormality is
 Figure 8-12  Abnormal exercise stress study. Example of inducible
defined as hypokinesis or akinesis of a segment ischemia with the apical long-axis view showing normal wall motion on this
that was normal at rest. Failure of a normal seg- end-systolic image at rest, with akinesis of the mid and apical segments of
ment to become hyperkinetic also is evidence of the anterior septum with exercise stress (arrows).
ischemia.
o ECG evidence of an inducible wall motion

abnormality in one or more adjacent segments n  ther causes of acute chest pain that require
O
is consistent with coronary artery disease, with immediate intervention (Table 8-4) are:
the probable affected coronary artery identified • 
Aortic dissection
from the location of the wall motion abnormality • 
Pericarditis
(Fig. 8-12). • 
Pulmonary embolism
o With three-vessel coronary disease, instead of a
regional wall motion abnormality, the only find- Step 1: Evaluate Regional Ventricular
ing may be the absence of hyperkinesis and fail- Function
ure of ventricular size to decrease appropriately. n  A regional wall motion abnormality in a patient
o Symptoms of chest discomfort accompanied
 with chest pain indicates myocardial infarction or
by inducible wall motion abnormalities are ischemia.
consistent with ischemia; symptoms occurring n In a patient with prior coronary disease, it may be
simultaneously with normal regional function difficult to distinguish preexisting regional dysfunc-
suggest noncardiac chest pain. tion from acute dysfunction.
n 
Regional ventricular function may be normal
Acute Coronary Syndromes between episodes of chest pain in patients with
unstable angina.
Basic Principles
n Acute coronary syndromes include patients with:
v  KEY POINTS
• 
ST-segment elevation myocardial infarction (STEMI) o  chocardiographic evaluation of wall motion is
E
• 
Non–ST-segment elevation myocardial infarction most helpful when ECG is nondiagnostic; prompt
(NSTEMI) revascularization is appropriate in patients with
• 
Unstable angina (Table 8-3) ST-elevation myocardial infarction.
Coronary Artery Disease  CHAPTER 8 149

TABLE 8-3 Coronary Artery Disease: Clinical-Echocardiographic Correlates


Coronary Anatomy Clinical Presentation Echocardiographic Findings
Asymptomatic Coronary artery narrowing Stress echo may be A normal stress echo does not
­coronary disease < 70% requested in asymptom- ­exclude atherosclerotic ­coronary
Typically does not cause atic patients at high risk disease but indicates a low
symptoms or myocardial of coronary disease, for ­likelihood of significant ischemia.
ischemia example, to assess risk
before noncardiac surgery
Chronic stable Coronary stenosis ≥ 70% Typical angina on exertion Normal resting LV regional and global
angina Narrowing may be asymp- systolic function.
tomatic at rest but causes Stress echo shows inducible wall mo-
symptoms with exertion tion abnormalities in the distribution
of the affected coronary artery.
Acute coronary Coronary occlusion or Acute chest pain Akinesis or hypokinesis of the
syndrome severe stenosis with Differential diagnosis myocardium supplied by the
rupture of an atheroscle- includes aortic dissection, occluded vessel with normal wall
rotic plaque and luminal pericarditis, AS, HCM thickness.
thrombus With unstable angina, wall m
­ otion may
be normal between pain episodes.
Old myocardial Occluded coronary with Asymptomatic if other Thinning increased echogenicity and
infarction ­attenuated distal vessel, coronary vessels are not akinesis in the distribution of the
­collateral vessels often stenosed affected coronary artery.
are present Heart failure if significant LV Ischemic MR may be present.
dysfunction is present
End-stage ischemic Multiple old coronary Heart failure Dilated LV with severely reduced
disease ­occlusions, small distal ejection fraction.
vessels Areas of akinesis and areas of normal
LV function are present.
LV diastolic dysfunction.
RV systolic function is normal, unless
RV infarction is present.
Ischemic MR may be present.

From Otto, CM: Textbook of Clinical Echo­cardiography, ed 5, Elsevier, 2013, Philadelphia.


AS, Aortic stenosis; HCM, hypertrophic cardiomyopathy; MI, myocardial infarction; MR, mitral regurgitation.

o  ormal regional myocardial function simulta-


N
TABLE 8-4 Medically Urgent Causes
neous with chest pain symptoms indicates a very
of Acute Chest Pain low likelihood of an acute coronary syndrome.
Acute coronary syndrome
Acute ST-elevation myocardial infarction (STEMI) Step 2: Estimate or Measure Ejection
Non–ST-elevation myocardial infarction (NSTEMI) Fraction
Unstable angina n Evaluation of overall LV systolic function is clini-
Aortic dissection
cally useful in management of patients with acute
Pulmonary embolus
Acute pericarditis
chest pain.
n 
Hospitalization and further evaluation often is
Esophageal rupture (Boerhaave syndrome)
needed in patients with a reduced ejection frac-
tion, even when due to causes other than acute
coronary syndrome.
o  remote transmural myocardial infarction
A
results in akinesis, myocardial thinning, and v  KEY POINTS
increased echogenicity, consistent with scar o  easurement of ejection fraction using three-
M
(Fig. 8-13). dimensional (3D) volumetric imaging or the
o However, with a prior non–ST-elevation or
 apical biplane approach is preferred when
reperfused myocardial infarction wall thickness endocardial definition is adequate and there are
may be relatively normal. no time constraints.
150 CHAPTER 8  Coronary Artery Disease

A2C

Figure 8-14  3D imaging of regional wall motion.  A 3D volume acquisi-


tion from the apical window can be used to derive multiple simultaneous
short-axis views of the LV for evaluation of wall motion.

Figure 8-13  Myocardial infarction. The inferior wall is thinned, bright,


and akinetic in the 2-chamber view in a patient with a prior ST-elevation
inferior myocardial infarction. A2C, Apical 2-chamber.

o 3 D volumetric imaging also allows analysis of


regional wall motion (Fig. 8-14 and Fig. 8-15).
o In urgent situations, a visual estimate of ejec-
tion fraction based on parasternal short-axis
and apical 4-chamber, 2-chamber, and long-axis
views is appropriate.
o Left-sided echo contrast may be helpful for defi-
nition of both global and regional LV systolic
function when image quality is suboptimal.
Step 3: Consider Alternate Causes of Chest
Pain Figure 8-15  Analysis of 3D regional function.  Regional function can be
evaluated based on segmental wall motion from 3D imaging.
n The echo may suggest other causes of chest pain
when LV function is normal.
n 
Often additional imaging approaches are needed
for further evaluation when the clinical diagnosis the pericardium) and with numerous systemic
remains unclear. diseases (see Chapter 10).
v  KEY POINTS Step 4: Evaluate Cardiac Hemodynamics
o  vidence of aortic dilation and aortic regurgita-
E n ECG evaluation of cardiac hemodynamics is help-
tion in a patient with acute chest pain prompts ful in selected patients with acute chest pain.
further evaluation for aortic dissection by trans-
esophageal echocardiography (TEE) or cardiac v  KEY POINTS
computed tomographic imaging. o I schemia or infarction often is accompanied by
o Although a pulmonary embolus is rarely
 diastolic dysfunction with evidence of elevated
visualized on echo, findings of pulmonary left atrial pressure on the Doppler LV and LA
hypertension and right ventricle (RV) dila- filling curves (see Chapter 7).
tion or dysfunction suggest this diagnosis be o Pulmonary pressures may be elevated due to
considered. elevated left-sided filling pressures.
o A pericardial effusion is consistent with the
 o Cardiac output can be measured using the LV
diagnosis of pericarditis but also may be seen outflow tract diameter and flow velocity inte-
with acute aortic dissection (with rupture into gral (see Chapter 6).
Coronary Artery Disease  CHAPTER 8 151

TABLE 8-5 Complications of Acute Myocardial Infarction


Incidence Pathophysiology Echocardiographic findings
Pericarditis and 5% Occurs in first 4 days after reperfused acute Small circumferential pericardial
­pericardial ­effusion MI. effusion.
Larger effusion raises concern for
LV rupture.
RV infarction 30-50% of Occlusion of the acute marginal branch of Dilated hypo- or akinetic RV.
patients with the RCA; often associated with ­inferior LV Infarction of adjacent inferior LV
inferior MI myocardial infarction. wall.
Ischemic MR 25% Infarction or ischemia of the papillary muscle Moderate to severe mitral
results in MR. No audible murmur in 50%. ­regurgitation.
More common with inferior-posterior MI. TEE often needed to identify
Papillary muscle rupture is uncommon cause of MR.
but ischemic MR is seen in about ¼ of MI
patients.
Ventricular septal <0.5% Transmural infarction with hemorrhage in Discrete septal defect in area of
defect necrotic zone. Occurs most often 24 h akinesis with left to right flow
­after reperfusion in older women with seen on color and CW Doppler.
single vessel disease.
Free wall rupture 0.8% Transmural infarction with hemorrhage Large pericardial effusion with
and tamponade in necrotic zone. Most likely involves tamponade. Acute fatal event
­inferolateral wall with circumflex or unless temporarily sealed by
LAD occlusion. fibrinous pericardial adhesions.
LV pseudoaneurysm Rare Free wall rupture contained by organized Discrete akinetic dilated area with
thrombus and pericardium. Occurs most a narrow neck between the LV
often with circumflex and RCA occlusion and pseudoaneurysm cavity.
in the basal inferior-posterior walls. Lined with thrombus which may
be mistaken for a thick wall.
LV aneurysm 8-15% Myocardial thinning and dilation due to Thin, bright, dyskinetic LV segment
infarct expansion 24-72 hours after acute with a diastolic contour abnor-
MI. The thin aneurysm walls are composed mality. May have associated
of fibrotic myocardium. Occurs most often thrombus.
at the LV apex with LAD occlusion.
LV thrombus 5-10% Peak incidence 3 days post-MI but may Echogenic mass, distinct from
­occur within hours in areas of infarction myocardium, often protruding
and akinesis, most commonly at the apex. into the chamber, with underlying
akinesis, typically at the apex.
LV systolic Variable Extent of LV regional and global ­systolic Location and size of the regional
­dysfunction dysfunction depends on infarct size, wall motion abnormalities cor-
­timing and success of reperfusion, and respond to infarct size. Overall
medical therapy. ejection fraction also reflects
adverse LV remodeling.

LAD, Left anterior descending coronary artery; MI, myocardial infarction; MR, mitral regurgitation; RCA, right coronary artery.

Complications of Acute Myocardial Step 1: Evaluation of the Patient with


Infarction Recurrent Chest Pain after Myocardial
n  Echo provides rapid, accurate bedside diagnosis Infarction
of mechanical complications of acute myocardial n  Recurrent chest pain after myocardial infarction
infarction (Table 8-5). may be due to recurrent ischemia, pericarditis, or
n Mechanical complications of myocardial infarction noncardiac chest pain.
present as recurrent chest pain, new systolic mur- n ECG evaluation focuses on segmental wall motion
mur, heart failure, cardiogenic shock, or a systemic and detection of a pericardial effusion.
embolic event.
n 
Arrhythmias associated with acute myocardial v  KEY POINTS
infarction may occur in the absence of significant o  omparison of regional wall motion with pre-
C
structural abnor ­malities. vious studies may allow detection of recurrent
152 CHAPTER 8  Coronary Artery Disease

A2C

LV

A B
Figure 8-16  LV rupture.  In this patient with an inferior myocardial infarction, an area of discontinuity in the inferior wall (arrow) is seen in the apical 2-chamber
view with color Doppler showing flow into a narrow-necked pseudo-aneurysm. The myocardial rupture is contained by pericardial adhesions, which form the
wall of the pseudo-aneurysm. A2C, Apical 2-chamber.

ischemia in the periinfarct region or in the dis- resulting in “tethering” of the mitral leaflets
tribution of a different coronary artery. How- with inadequate systolic coaptation (Fig. 8-17).
ever, coronary angiography often is needed for o With partial or complete papillary muscle rup-
definitive diagnosis. ture, acute severe mitral regurgitation occurs,
o The presence of a pericardial effusion is con- with pulmonary edema and cardiogenic shock.
sistent with the diagnosis of pericarditis but o TEE often is needed to define the mechanism
also may be seen with acute aortic dissection and evaluate the severity of ischemic mitral
(with rupture into the pericardium) or with LV regurgitation.
rupture. o Ventricular septal defects that develop after myo-
o LV rupture may present as transient chest pain; cardial infarction are detected using color Doppler
this diagnosis should be considered when peri- showing a flow disturbance on the RV side of the
cardial effusion is present in a patient with a septum. Often the defect can be visualized on two-
history of myocardial infarction particularly if dimensional (2D) imaging. Continuous wave (CW)
the episode of chest pain was accompanied by Doppler interrogation provides information on the
hypotension (Fig. 8-16) LV-to-RV systolic pressure difference (Fig. 8-18).
o With a ventricular septal defect, oxygen satura-
Step 2: Evaluation of the Patient with a New tion is increased from the right atrium (RA) to
Systolic Murmur after Myocardial Infarction the RV due to shunting of oxygenated blood
n  he differential diagnosis of a new murmur that
T from the LV to RV across the ventricular defect.
develops after myocardial infarction is: If a right-sided heart catheter is in position,
• 
Ventricular septal defect due to rupture of the septal measurement of oxygen saturations may be
myocardium, or helpful when the diagnosis is unclear.
• 
Acute mitral regurgitation due to papillary muscle
rupture or dysfunction Step 3: Evaluation of the Patient with
n Imaging focuses on evaluation of segmental wall Hypotension or Cardiogenic Shock after
motion and detection of a pericardial effusion with Myocardial Infarction
Doppler evaluation for the cause of the murmur. n Hypotension after myocardial infarction may be
due to RV infarction, LV systolic dysfunction, or
v  KEY POINTS myocardial rupture with pericardial tamponade.
o  R after myocardial infarction most often is
M n 
Echocardiographic evaluation focuses on evalu-
due to ischemia or infarction of the papillary ation of global left and RV systolic function and
muscle or underlying inferior-lateral LV wall, detection of a pericardial effusion.
Coronary Artery Disease  CHAPTER 8 153

LV

LA

Figure 8-17  Ischemic mitral regurgitation. Imaging (left) shows tethering (arrow) of the posterior leaflet due to traction on the chords by the ischemic
myocardium underlying the papillary muscle, resulting in a “tented” appearance of the closed valve at end-systole and (right) a posterior-laterally directed jet
of mitral regurgitation (arrow) as the anterior leaflet fails to coapt completely with the relatively immobile posterior leaflet.

TTE TEE

LV RV
RV
RV

LV
RA LV
LA

A B C
Figure 8-18  LV pseudo-aneurysm.  In this patient with an anterior myocardial infarction, a localized area of dyskinesis (arrows) is seen in the apical segment
of the septum in the 4-chamber view on TTE (A), with an area of frank discontinuity in the septum (arrow) seen on the transgastric short-axis view on TEE (B).
C, Color Doppler confirms the postmyocardial infarction ventricular septal defect with a narrow jet of flow from the LV into the RV.

v  KEY POINTS with elevated LV filling pressures. Diastolic dys-


o  V infarction often accompanies an inferior
R function may lead to pulmonary congestion, but
myocardial infarction The typical presenta- rarely is it the primary cause of hypo­tension.
tion is hypotension that responds to volume o LV rupture due to myocardial infarction may
loading. Echo shows a dilated hypocontrac- result in acute cardiac tamponade and death.
tile RV, despite normal pulmonary pressures However, in some cases the myocardial rup-
(Fig. 8-19). ture is contained by a pericardial thrombus and
o With a large or recurrent myocardial infarction, adhesions.
LV systolic function may be significantly reduced, o A contained LV rupture is called a pseudo-
resulting in pulmonary edema and hypotension. aneurysm because its wall consists of pericar-
Echo allows measurement (or estimation) of ejec- dium (not myocardium) (Fig. 8-20).
tion fraction and assessment of regional ventricu- o Typical characteristics of a pseudo-aneurysm
lar function. are a narrow neck compared with its widest
o Myocardial ischemia or infarction is typically diameter and an abrupt transition at an acute
accompanied by diastolic dysfunction, often angle between the normal myocardium and the
154 CHAPTER 8  Coronary Artery Disease

aneurysm. Often the pseudo-aneurysm is lined Step 4: Evaluation of Late Complications


with thrombus. of Myocardial Infarction
o Most pseudo-aneurysms require urgent sur-
 n Late complications of myocardial infarction include
gical intervention to repair the ventricular LV aneurysm, thrombus, and systolic dysfunction.
rupture. True aneurysms typically are treated n Echocardiographic examination includes evaluation
medically. of global and regional myocardial function, calcula-
tion of ejection fraction, assessment of diastolic dys-
function, and a diligent search for apical thrombus.
v  KEY POINTS
o  dverse ventricular remodeling after myocardial
A
infarction results in thinning and scar formation
in areas of infarction, overall LV dilation, and a
reduction in ejection fraction. Adverse remodel-
LV ing is prevented, to some extent, by appropriate
medical therapy.
RV o The myocardial thickness in diastole, wall thick-
ening during systole, and endocardial motion
are graded for each myocardial segment.
o An aneurysm is defined as a discrete area
of the LV (usually the apex) with a diastolic
contour abnormality and systolic dyskinesis
(Fig. 8-21).
RA
LA o LV ejection fraction is calculated using the

biplane apical approach. Single plane or
M-mode evaluation of LV function may be inac-
curate due to regional ventricular dysfunction.
o Apical thrombus is best visualized in standard
and oblique apical views. Identification of
thrombus is enhanced by use of a higher-fre-
Figure 8-19  RV infarction.  RV dilation and (in real time) hypokinesis in a
patient with an inferior myocardial infarction is seen in this apical 4-chamber quency transducer and a shallow image depth
view. (Fig. 8-22).

A2C

LV
LV

pA
pA

LA

A B
Figure 8-20  Chronic left ventricle pseudo-aneurysm.  Characteristics include a narrow neck (arrows) relative to the maximum diameter of the pseudo-
aneurysm, as seen in a short-axis view of the LV (A) and in the apical 2-chamber view (B). There is an abrupt transition from the normal myocardial thickness to the
aneurysm and the pseudo-aneurysm has an irregular echodensity consistent with thrombus lining the cavity. A2C, Apical 2-chamber; pA, pseudo-aneurysm.
Coronary Artery Disease  CHAPTER 8 155

o pical trabeculation is distinguished from


A End-stage Ischemic Disease
thrombus by the lack of mobility, linear appear- n End-stage ischemic disease, colloquially called isch-
ance, and attachments to the LV wall. emic cardiomyopathy, has many features in common
o Left-sided ultrasound contrast may be helpful with a dilated cardiomyopathy or ventricular dys-
when it is difficult to distinguish apical trabecu- function due to valvular heart disease (see Echo
lation from thrombi (Fig. 8-23). Exam at the end of the chapter).

A4C A2C

A B
Figure 8-21  Apical aneurysm.  A true aneurysm is characterized by a diastolic contour abnormality with a gradual, smooth transition from normal myocar-
dial thickness to the thin scarred myocardium of the aneurysm and with systolic dyskinesis as shown by the arrows in the apical 4-chamber (A4C) (A) and
2-chamber (A2C) end-diastolic (B) images.

LV

RV

RA LA

A B
Figure 8-22  Apical thrombus.  A, This apical echodensity that protrudes into the chamber in an area of dyskinesis is consistent with an apical thrombus.
B, The zoomed image using a higher-frequency transducer and an oblique image plane through the apex helps confirm that these echoes represent a thrombus,
and not prominent trabeculation or an imaging artifact.
156 CHAPTER 8  Coronary Artery Disease

n  he echocardiographic features most helpful in


T Step 1: Evaluate Global Left Ventricular
diagnosis of end-stage ischemic disease are: Systolic and Diastolic Function
• 
Definite regional wall motion abnormalities with n Overall LV systolic function is evaluated by calcu-
areas of thinning and akinesis (Fig. 8-24) lation of an apical biplane ejection fraction.
• 
Normal RV size and systolic function (unless RV n 
LV diastolic function is evaluated as described in
infarction has occurred) Chapter 7.
• 
Absence of evidence for primary valvular heart disease
v  KEY POINTS
o  alculation of a biplane ejection fraction is
C
performed whenever possible. If endocardial
definition is suboptimal, left-sided contrast echo
provides better visualization of LV function.
o Most patients with end-stage ischemia disease
have diastolic dysfunction, as well as systolic
dysfunction.
o Early in the disease course, diastolic dysfunction
is characterized by impaired relaxation.
o However, as systolic function deteriorates, LV
filling pressures may increase, LV compliance
decreases, and the increased ventricular vol-
umes result in a rightward shift along the LV
diastolic pressure-volume curve.

LV Step 2: Evaluate Regional Left Ventricular


Systolic Function
n Regional function is evaluated by grading each
myocardial segment as normal, hypokinetic, aki-
netic, or dyskinetic.
n 
Any areas of thin, scarred myocardium are
Figure 8-23  Apical trabeculation.  The apical trabeculation seen in this
apical 2-chamber view is distinguished from thrombus by similar echoden- noted.
sity to myocardium, a linear structure that connects to the myocardium at
both ends and the absence of an underlying wall motion abnormality.

A4C A2C

LV
LV

RV

RA

LA LA

Figure 8-24  End-stage ischemic disease.  There is a dilated left ventricle chamber, global hypokinesis, and a low ejection fraction. There is thinning,
increased echogenicity, and dyskinesis (arrows) of the basal inferior septum (apical 4-chamber [A4C] view) and inferior wall (apical 2-chamber [A2C] view) on
these end-systolic images, consistent with old infarction and myocardial scar.
Coronary Artery Disease  CHAPTER 8 157

o  hen RV systolic function appears similar to


W
LV function, the degree of dysfunction is the
same for both ventricles.
o RV function may be impaired in patients with
coronary disease and a prior RV infarction.

LV Step 4: Estimate Cardiac Hemodynamics


n  Pulmonary artery systolic pressure is estimated
RV from the velocity of the tricuspid regurgitant jet
and the appearance and respiratory variation of
the inferior vena cava, as discussed in Chapter 6.
n Evidence for elevated left atrial pressure includes
a prolonged and high-velocity pulmonary vein
RA
a-wave reversal and a high ratio of transmitral
LA flow to tissue Doppler velocity in early diastole
(see Chapter 7).
v  KEY POINTS
o  ulmonary pressures may be elevated due to
P
LV dysfunction, leading to chronic elevation
of LA pressure and consequent pulmonary
hypertension.
Figure 8-25  Normal RV with ischemic disease.  In patients with heart o Evidence for elevated filling pressures may no
failure due to end-stage ischemic disease, RV size and systolic function longer be present after optimization of medical
often are normal (unless concurrent RV infarction has occurred). Compared
with the dilated and hypokinetic left ventricle, the normal RV appears rela- therapy.
tively small and hyperdynamic.
Step 5: Identify and Evaluate any Associated
Valve Disease
• LV dilation and systolic dysfunction due to chronic
v  KEY POINTS valve disease may be difficult to distinguish from
o  discrete area of scarring or dyskinesis is
A a primary cardiomyopathy or end-stage ischemic
consistent with coronary disease, rather than heart disease.
a primary cardiomyopathy. • LV systolic dysfunction of any cause often is
o In both end-stage ischemic disease and primary accompanied by significant mitral valve regurgita-
cardiomyopathy, ventricular function typically tion due to displacement of the papillary muscle,
is best preserved for the inferior-lateral and leaflet tethering, and annular dilation.
lateral basal segments of the LV.
v  KEY POINTS
Step 3: Evaluate Right Ventricular Size o  rimary mitral valve disease is characterized by
P
and Systolic Function abnormalities of the valve leaflets or chordae—
n  ight ventricular size and systolic function are
R for example, myxomatous or rheumatic valve
qualitatively evaluated as discussed in Chapter 6. disease.
n RV systolic function typically is normal in patients o With secondary mitral regurgitation, the mitral
with end-stage ischemic disease, so that the RV valve apparatus is anatomically normal, but geo-
appears small and hypercontractile compared with metric relationships are altered by the dilated LV.
the dilated, hypokinetic LV (Fig. 8-25). o Mitral regurgitation due to ischemic heart dis-
ease may improve with medical or interven-
v  KEY POINTS tional approaches for relief of ischemia.
o  hen the RV is proportionate to the LV, the
W o Coronary angiography may be needed to deter-
degree of RV dilation is similar to the degree of mine the contribution of coronary disease to
LV dilation. the clinical cardiac dysfunction.
158 CHAPTER 8  Coronary Artery Disease

THE ECHO EXAM

Coronary Artery Disease


Echocardiographic Diagnosis of Coronary Disease
Echocardiographic
Modality Clinical Utility Findings Recording Interpretation
Resting regional • Acute coronary • Akinesis or • 2D or 3D imaging of • Use standard
wall motion syndrome ­hypokinesis the LV wall segment
• Chronic CAD of ­infarcted or • Optimize endocardial nomenclature for
acutely ischemic definition location
regions • Use contrast if ­images • Categorize wall
are suboptimal motion as normal,
hypo- or akinetic
• Use 3D display
when possible
Exercise stress • Diagnosis of CAD • Normal wall • Depth that includes • Compare optimal
echocardiography ­motion at rest only LV, optimize baseline and stress
endocardial definition, images in same
use contrast if needed views
Dobutamine stress • Evaluation for • Hypokinesis or • Same depth as • Maximal work load
echocardiography ischemia with akinesis with baseline, optimize affects accuracy
known CAD stress in ischemic endocardial definition, of echo results for
• Diagnosis and segments use contrast if needed ­detection of
evaluation of CAD • Return of normal • Select optimal image ­ischemia
in patients unable wall motion with from series of digital
to exercise rest cine loops
Myocardial viability • Diagnosis of • Biphasic response • Standard DSE • Improvement in
­hibernating on DSE ­protocol with wall thickening at
or stunned ­additional low-dose low dose followed
­myocardium stages by ischemia at
high-dose DSE
is consistent with
viable myocardium
supplied by a
stenosed vessel
Overall LV systolic • All CAD patients • Ejection fraction • 3D Biplane apical • The degree of
function by 2D and 3D ejection fraction reduction in EF
imaging • CWD mitral after acute MI
• dP/dt regurgitant jet depends on infarct
size and success of
reperfusion
LV diastolic function • All CAD patients • Diastolic dysfunc- • Standard approaches • Diastolic
tion and elevated for evaluation of LV dysfunction may
filling pressures diastolic function and be transient with
depend on type filling pressures (see ischemia
and severity of Chapter 7) • End-stage CAD is
CAD associated with
severe diastolic
dysfunction

CAD, Coronary artery disease; DSE, dobutamine stress echocardiography; EF, ejection fraction; MI, myocardial infarction.
Coronary Artery Disease  CHAPTER 8 159

Echo Views for Wall Motion

LAD
2 1 8 7
RCA
RV LV RV Cx
3 6 9
LV 12
Short axis Short axis
(base) (mid-LV)
4 5 10 11

17 17
17
14 16 15 13 16 14

Figure 8-26  Echocardiographic views for


9 12 10 7 11 8 wall motion evaluation.  In the short-axis view,
RV LV LV LV at the base and midventricular levels, the LV is
divided into the anterior (1, 7), anterior-septal
3 6 4 1 5 2 (2, 8), inferior septal (3, 9), inferior (4, 10), inferolat-
eral (5, 11), and anterolateral (6, 12) segments.
In the apical region there are four segments: an-
terior (13), septal (14), inferior (15), and lateral
Ao (16), plus the tip of the apex (17). The territory of
RA LA LA LA the left anterior descending (LAD) artery is indi-
cated in green, the right coronary artery (RCA) in
red, and the left circumflex (Cx) coronary artery
Apical 4-chamber Apical 2-chamber Apical long-axis in yellow.

Differentiation of Left Ventricular Systolic Dysfunction due to End-Stage Ischemic Disease


from Dilated Cardiomyopathy or Chronic Valvular Disease
Findings End-Stage Ischemic Disease Dilated Cardiomyopathy Chronic Valvular Disease
Left-ventricular ejection Moderate-severely depressed Moderate-severely Moderate-severely
fraction ­depressed ­depressed
Segmental wall motion May be present Absent Absent
­abnormalities
Right ventricular systolic Normal Decreased Variable
function
Pulmonary artery pressures Elevated Elevated Elevated
Mitral regurgitation Moderate Moderate Moderate-severe
Aortic regurgitation Not significant Not significant Moderate-severe
160 CHAPTER 8  Coronary Artery Disease

Complications of Acute Myocardial Infarction


Complication Echocardiographic Findings Imaging Approach
Pericardial effusion Small circumferential pericardial effusion. Standard views for evaluation of effusion.
Larger effusion raises concern for LV rupture.
RV infarction Dilated hypo- or akinetic RV. Apical and subcostal views to evaluate RV
Infarction of adjacent inferior LV wall. free wall motion.
Measure TAPSE, DTI S-velocity, fractional
area change.
Ischemic MR Tethering of posterior leaflet with posteriorly Evaluate mitral valve anatomy in standard
­directed MR. views.
Papillary muscle rupture (rare) with mass attached Evaluate and quantitate mitral regurgitant
to flail leaflet. severity (see Chapter 12).
Moderate to severe mitral regurgitation (may TEE and 3D imaging often needed to identify
be ­intermittent, present only during ischemic cause of MR.
­episodes).
Ventricular septal Discrete septal defect in area of akinesis with left Use color Doppler to detect VSD in focal
defect to right flow seen on color and CW Doppler. region of akinesis or when imaging suggests
discontinuity in septum.
CW Doppler confirms velocity and direction
of blood flow.
Free wall rupture Large pericardial effusion with tamponade. Acute Pericardial hematoma or localized effusion
and tamponade fatal event unless temporarily sealed by fibrinous after MI should be promptly reported to
pericardial adhesions. referring MD.
Use color Doppler to search for
communication from LV to pericardial
space; subcostal views may be helpful.
LV pseudoaneurysm Abrupt transition from normal myocardium to Most often located at inferior base of LV.
aneurysm. Parasternal views and apical 2-chamber
Acute angle between myocardium and aneurysm. views are helpful.
Narrow neck. TEE imaging often needed for diagnosis.
Ratio of neck diameter to aneurysm diameter < 0.5.
May be lined with thrombus.
LV aneurysm Thin, bright, dyskinetic LV segment with a dia- Most often located at LV apex.
stolic contour abnormality. May have associated Best seen in apical views or with 3D imaging
thrombus. from apex.
LV thrombus Echogenic mass, distinct from myocardium, often Use high frequency transducer, zoom mode,
protruding into the chamber, with underlying adjust gain and instrument settings; off-
akinesis, typically at the apex. axis lateral apical views may be helpful.
Contrast to opacify the LV may better
demonstrate the thrombus.
Apical thrombi may be missed on TEE.
LV systolic Location and size of the regional wall motion abnor- 3D or 2D biplane ejection fraction
­dysfunction malities correspond to infarct size. Overall ejection ­calculations.
fraction also reflects adverse LV remodeling.

DTI, Doppler tissue imaging; MI, Myocardial infarction; MR, mitral regurgitation; TAPSE, tricuspid annular plane systolic excursion; VSD, ventricular
septal defect.
Coronary Artery Disease  CHAPTER 8 161

SELF-ASSESSMENT QUESTIONS

Question 1
A 56-year-old woman with no medical history pres-
ents with precordial chest discomfort and dyspnea.
Blood pressure is 86/50 mmHg with a heart rate
of 115 bpm. Physical examination reveals jugular
venous distention and bibasilar lung crackles. There
is a systolic murmur at the cardiac apex radiating
toward the left axilla. TTE reveals a hypokinetic infe-
rior wall with an ejection fraction of 40%. There is
moderate, posteriorly directed mitral regurgitation.
The tricuspid regurgitant jet velocity is 3.4 m/s and
the inferior vena cava diameter measures 2.1 cm with
minimal inspiratory collapse. You suspect myocardial
ischemia in which coronary distribution?
A. Anterior septal perforator
B. Left circumflex artery Figure 8-27 
C. Posterior descending artery
D. Left anterior descending artery Question 3
Which of the following is the earliest manifestation
Question 2 of myocardial ischemia detectable during stress echo?
You are evaluating a 64-year-old man who had A. Anginal chest discomfort
an anterior myocardial infarction and underwent B. ECG ischemic changes
placement of a drug-eluting stent in his left anterior C. Regional coronary hypoperfusion
descending artery 1 week ago. He was discharged D. Segmental wall motion abnormality
home on aspirin, clopidogrel, a beta blocker, and a
statin medication. He presents now with dyspnea and Question 4
the following image is obtained (Fig. 8-27). What do A 62-year-old man presents with progressive dyspnea.
you refer the patient for next? The following apical 2-chamber view is obtained on
A. Repeat coronary angiogram TTE (Fig. 8-28). The image is most consistent with:
B. Pericardiocentesis A. Myocardial rupture
C. Pulmonary angiogram B. Chagas disease
D. Intra-aortic balloon pump C. Cardiac sarcoidosis
D. Left ventricular aneurysm
E. Endomyocardial fibrosis

Figure 8-28 
162 CHAPTER 8  Coronary Artery Disease

Question 5 Question 9
Which of the following factors most lowers the sensi- You are asked to evaluate a 68-year-old man who pre-
tivity of a treadmill stress echo? sented to the neurology service following a left middle
A. Transpulmonary microbubble contrast cerebral artery stroke. Past medical history includes
B. Increased LV wall thickness a left anterior descending coronary distribution myo-
C. Increased treadmill-stretcher transfer time cardial infarct 4 weeks ago and the patient underwent
D. Multivessel coronary disease stent placement. TTE is completed (Fig. 8-29). You
E. Prolonged exercise duration refer the patient for:
A. Initiation of vitamin K antagonist
Question 6 B. Patent foramen ovale closure
Which of the following is an appropriate end- C. Surgical pseudoaneurysm repair
point for termination of an exercise treadmill stress D. Cardiac magnetic resonance imaging
study?
A. Achievement of 75% of maximal predicted

heart rate
B. Ventricular bigeminy during exercise stress
C. Increase in systolic blood pressure 30 mmHg
over baseline
D. Onset of exertional dyspnea
Question 7
A 68-year-old woman presents to the emergency
department with precordial chest discomfort that
developed suddenly during a protracted argument
with her estranged son. The patient has hyperten-
sion and is a nonsmoker. Physical examination
reveals flat neck veins. Her blood pressure is 158/98
mmHg. Her lungs are clear and no murmurs are
heard. The ECG shows diffuse ST-elevation and her
troponin is mildly elevated at 2.5 ng/mL (normal <
0.5 ng/mL) with a normal B-type natriuretic pep-
tide level. What would likely be seen on transtho-
racic imaging?
A. Right ventricular free wall hypokinesis
B. Basal segment hyperkinesis
Figure 8-29 
C. Inter-ventricular septal dyskinesis
D. Apical segment hyperkinesis
Question 8
For each of the following exercise protocols for stress
echo, select (a) supine bicycle ergometery stress pre-
ferred or (b) treadmill stress preferred:
1. Peak stress image acquisition
2. Maximal workload achieved
3. Controlled workload during testing
4. Proximal leg fatigue
5. Decreased patient mobility
Coronary Artery Disease  CHAPTER 8 163

Question 10
A 72-year-old man undergoes treadmill stress echo. A.
Left anterior descending
End-systolic images at rest (left) and immediately post- B. Right coronary artery
stress (right) are provided (Fig. 8-30, A and B). Based C. Left main artery
on the images, what coronary distribution is most D. Left circumflex artery
likely affected?

A B
Figure 8-30 

Question 11 Based on the images provided, which of the fol-


A 58-year-old man is preparing for hospital discharge lowing additional findings would you expect on the
following a myocardial infarct. He had presented with echocardiogram?
stuttering chest discomfort over 3 to 4 days and sub- A. Pulmonary to systemic shunt ratio = 2.4
sequently received a drug eluting stent in the right B. Septal E′ velocity = 0.1 m/s
coronary artery. Before discharge, TTE is completed. C. Tricuspid regurgitant velocity = 2.1 m/s
Images from the short-axis view are shown (Fig. 8-31). D. Wall motion score index = 1

BEDSIDE

Figure 8-31 
164 CHAPTER 8  Coronary Artery Disease

Question 12
A 68-year-old man presents for dobutamine stress presented (Fig. 8-32, A and B). The LV response to
echo. Image quality was not optimal and transpul- dobutamine in this case is best described as:
monary microbubble contrast was used. Resting LV A. Akinetic
function is normal with an ejection fraction of 60%. B. Biphasic
The end-systolic images in the apical 4-chamber view C. Hyperdynamic
at rest (left) and at peak dobutamine infusion (right) are D. Tethered

A B
Figure 8-32 
Coronary Artery Disease  CHAPTER 8 165

ANSWERS

Answer 1: C Answer 4: A
This patient’s clinical presentation (with cardiogenic The apical 2-chamber view shows myocardial discon-
shock, chest discomfort, and echocardiographic find- tinuity at the base of the inferior wall. The echolu-
ings) is consistent with an acute coronary syndrome. cency in this region is a contained myocardial rupture
The patient has inferior wall motion abnormalities (pseudoaneurysm). This is a late complication of myo-
and significant mitral regurgitation. Mitral regurgi- cardial infarction and is most common in the basal
tation coincident with ischemia suggests papillary inferior wall following a right coronary artery distri-
muscle dysfunction, which would correlate with dys- bution infarction. Because an LV pseudoaneurysm is
pnea. The posteromedial papillary muscle is typically an LV rupture that has been contained by pericar-
supplied by a single coronary artery, the posterior dium, prompt surgical intervention is needed. Exten-
descending artery. Ischemia and dysfunction of the sion of necrotic tissue to the inferior septum may also
posteromedial papillary muscle tethers the posterior lead to rupture of the interventricular septum and a
leaflet, resulting in posteriorly directed mitral regur- septal defect. This is in contrast to true LV aneurysms,
gitation. In this patient, ischemia has involved the with myocardial thinning and dilation, but with pre-
RV as well, exacerbating hemodynamic instability. In served myocardial continuity. The walls of a true LV
contrast, the anterolateral papillary muscle is typically aneurysm are thinned and scarred myocardium (not
supplied by both the left anterior descending and left pericardium as with a pseudoaneurysm), and there is
circumflex arteries, making it less susceptible to is­che­ a smooth transition from normal to infarcted myocar-
mia. The septal perforator arteries do not supply the dium with a wide neck of the aneurysm. Chagas dis-
papillary muscles. ease is a cause of nonischemic cardiomyopathy, most
commonly seen in Latin America; a clinical manifes-
Answer 2: B tation of Chagas disease is LV apical aneurysm for-
The parasternal long-axis view shows a moderate mation. The inflammatory lesions in cardiac sarcoid
circumferential pericardial effusion. This was a typically occur in a noncoronary distribution, pre-
new effusion which developed after the infarction. dominantly in the endocardium of the anteroseptum
An inflammatory pericardial effusion may compli- and apex of the LV, with focal hypokinesis of affected
cate myocardial infarction and typically presents regions. Endomyocardial fibrosis is characterized by
several days after the infarct. The patient is symp- fibrosis of the left and/or right apices (with akinesis
tomatic (dyspnea) with rapid accumulation of fluid or dyskinesis), often with associated apical thrombi.
in a relatively short time frame. Additional spec-
tral Doppler imaging showed respiratory variation Answer 5: C
in tricuspid and mitral inflow as well as increased Because induced regional wall motion abnormali-
central venous pressure. He was referred for peri- ties may resolve quickly once adequate oxygen sup-
cardiocentesis with drainage of 1100 mL of clear ply is reestablished, rapid post stress imaging to
fluid and resolution of symptoms. identify myocardial dysfunction is critical. Images
should be obtained 90 seconds after completion
Answer 3: C of exercise, ideally within 60 seconds, to increase
The sequential progression of myocardial ischemia the likelihood of detecting abnormalities. We have
during stress echo is initiated by relative regional the patient practice the transfer from the treadmill
hypoperfusion distal to a coronary occlusion pro- to the imaging stretcher before beginning the test to
voked by increased myocardial oxygen demand. ensure the patient understands the sequence of events
With regional hypoperfusion, metabolic changes and is comfortable with the process. Transpulmonary
occur within the affected myocardium. Following microbubble contrast allows for improved endocar-
this, there are alterations in LV diastolic function. dial border definition in patients where image quality
With continued ischemia, there is impaired systolic is suboptimal, which improves the sensitivity of stress
function in the ischemic region. Only with pro- echo. Increased LV wall thickness, or LV hypertrophy,
longed ischemia are characteristic ECG changes, may lead to false-positive ECG findings, but does not
such as horizontal ST depression, and onset of affect echocardiographic identification of myocar-
typical angina. On a standard stress echo, regional dial dysfunction. Exercise duration only affects test
coronary hypoperfusion cannot be visualized so the sensitivity if the patient fails to achieve an adequate
earliest change seen is segmental wall motion abnor- workload; prolonged exercise duration indicates a fit
malities. Although evaluation of diastolic function individual, and does not limit sensitivity of the stress
during stress echo is conceptually possible, practical test. In general, sensitivity of stress echo is higher with
application of diastolic interrogation is difficult to multivessel compared to single vessel coronary disease
routinely implement. because the area of ischemic myocardium is larger.
166 CHAPTER 8  Coronary Artery Disease

The caveat to this is with “balanced” ischemia in all predominance (>90%). Catecholamine excess in the
myocardial beds where a discrete wall motion abnor- setting of microvascular disease has been implicated.
mality may not be evident; instead, there is a lack of Although the wall motion abnormality findings might
hyperdynamic response to stress. be consistent with multi-vessel disease (more than
one epicardial coronary distribution), with diffuse ST-
Answer 6: D segment elevation on ECG that exceeds biomarker
Onset of symptoms which limit patient exercise, such measurement of necrosis, findings are discordant with
as angina or significant dyspnea, should trigger ter- the relatively low serum biomarker values for myocar-
mination of the stress study. Other indications for dial necrosis. The outcome of stress cardiomyopathy
discontinuing a stress study include declines in blood is generally good and LV functional recovery is likely
pressure and ST-segment changes. In contrast, an with supportive care.
increase in systolic blood pressure during exercise
stress of 30 to 40 mmHg over baseline is expected; Answer 8
excessive increases in blood pressure beyond this level 1 . Peak stress image acquisition (a) supine bicycle
should prompt study discontinuation. Ventricular ergometery stress preferred
bigeminy is a hemodynamically stable rhythm and 2 . Maximal workload achieved (b) treadmill stress
should not prompt study discontinuation. For diag- preferred
nostic stress testing, patients should achieve at least 3 . Controlled workload during testing (b) treadmill
85% of their maximal predicted heart rate (calculated stress preferred
as 220 minus the patient’s age). Achievement of only 4 . Proximal leg fatigue (a) supine bicycle ergometery
75% of maximal predicted heart rate would be a non- stress preferred
diagnostic study. 5 . Decreased patient mobility (a) supine bicycle
ergometery stress preferred
Answer 7: B Exercise stress is preferred over nonexercise stressors
This woman’s presentation is consistent with stress as it allows for a contextual understanding of results
cardiomyopathy, an acute cardiac syndrome also relative to patient symptoms and functional capacity.
termed Takotsubo cardiomyopathy with acute chest Treadmill and bicycle ergometry protocols are most
pain or dyspnea following a significant emotional or widely utilized, and overall accuracy is comparable
physiologic stress. The clinical manifestation is tran- as long as adequate patient workload is achieved.
sient akinesis or dyskinesis of the apical and midven- Because patient motion limits imaging during active
tricular segments of the LV which extend beyond a treadmill use, “stress” imaging for treadmill protocols
single epicardial coronary distribution. Basal seg- is performed immediately following exercise cessation
ments are hyperdynamic. An image from this patient’s without a cool-down period, and expeditious imag-
ventriculogram is presented (Fig. 8-33). ing is critical to maximize likelihood of identifying
Angiography in patients with stress cardiomyopa- transient ischemia. Reasonable agility is required for
thy demonstrates an absence of obstructive coronary treadmill protocols as patients need to transition to the
disease. Stress cardiomyopathy has a strong female imaging table to allow time for imaging. In contrast,
bicycle protocols allow for simultaneous image acqui-
sition during active exercise at “peak stress” because
of maintenance of a relatively stationary position
during exercise. Patients performing treadmill exer-
cise typically achieve higher maximal workloads,
increasing likelihood of attaining hemodynamic tar-
gets, and overall workload is more controlled as the
treadmill proceeds at prespecified grade and speeds
with the patient pacing with the treadmill belt. In
contrast, workload for bicycle ergometry protocols is
maintained by graded resistance of the ergometer;
patient voluntary effort is needed to maintain cycle
speed. For bicycle protocols, quadriceps (proximal leg)
fatigue may precede maximal workload; this is partic-
ularly true for supine ergometry where lower extremi-
ties must be supported against gravity.
Answer 9: A
The image shows an apical 4-chamber view of the
LV with a large apical thrombus. The patient’s left
Figure 8-33  anterior descending artery infarction likely resulted
Coronary Artery Disease  CHAPTER 8 167

in anteroapical akinesis. Blood stasis in the akinetic rare complication after a myocardial infarct and typi-
apex can lead to thrombus formation, which can sub- cally occurs several days after the event. Most patients
sequently embolize. Vitamin K antagonist therapy present with acute cardiac symptoms and hypotension
(warfarin) is indicated for 3 to 6 months with a follow- but some patients are relatively asymptomatic initially.
up echocardiogram to monitor for thrombus resolu- The rupture may be evident as a systolic murmur on
tion. An embolic stroke may result from a paradoxical physical examination; echo usually is diagnostic. A
embolus via a patent foramen ovale. However, in septal E′ velocity = 0.1 m/s implies normal septal tis-
this case, therapy for the LV thrombus is indicated, sue velocity. With a septal infarct, myocardial velocity
regardless of whether a patent foramen ovale PFO should be severely reduced, generally less than 0.05
is present. A pseudoaneurysm is a mechanical com- m/s. With the left to right shunt, right ventricular sys-
plication of myocardial infarction where myocardial tolic pressure should be increased whereas a tricuspid
necrosis leads to a contained ventricular rupture. In jet velocity of only 2.1 m/s implies a normal pressure
patients where apical thrombus is suspected, transpul- gradient. The wall motion score index is a quantita-
monary microbubble contrast may help delineate the tive measure of regional wall motion, based on the
thrombus. In this case, the thrombus is well visualized mean wall motion score for all myocardial segments,
without the need for echocontrast. using a grade of 1 and 4 where 1 is normal motion, 2
is hypokinetic, 3 is akinetic, and 4 is dyskinetic; a wall
Answer 10: B motion score index of 1 implies normal LV function
The images provided are from the apical 4-chamber in all segments, and would not be consistent with a
view. There is a pacer lead in the right heart. At rest, myocardial infarction
the chamber size is small and the myocardial walls are
thick along the entire LV, consistent with normal wall Answer 12: A
motion. Following peak stress, the basal two thirds This is an abnormal stress echocardiogram. During
of the inferoseptum is thinned relative to the lateral imaging, microbubble transpulmonary contrast was
wall and apex, reflecting lack of systolic wall thicken- used to aid endocardial border definition. The resting
ing and consistent with ischemia. There is hyperdy- image shows a normal LV contour and wall thickness
namic function of the nonischemic myocardium with consistent with normal endocardial motion. At peak
increased myocardial thickening of the lateral wall and stress, there is a contour abnormality of the apical
a decrease in overall LV chamber size. These findings two thirds of the inferior septum and the entire apex
are most consistent with a right coronary artery distri- with a lack of systolic inward motion. In contrast, the
bution ischemia. The left anterior descending artery basal two thirds of the lateral wall showed increased
serves the anterior wall and LV apex, best seen in the inward motion on the stress image compared to base-
apical 2-chamber view. The myocardial regions sup- line. Subsequent coronary angiography in this patient
plied by the left circumflex artery are the inferolateral demonstrated a 90% right coronary artery occlusion
and anterolateral walls. These are best seen on the and an 80% left anterior descending artery occlusion.
apical 4-chamber and apical long-axis views. A left A biphasic response is the LV response of hibernat-
main coronary occlusion would cause ischemia in ing myocardium to low dose dobutamine where, in
both the left anterior descending artery and left cir- patients with severe LV dysfunction and akinetic
cumflex artery (whose myocardial distribution is not regions at rest, an initial increase in contractility of
shown). With a left main coronary occlusion, the myo- the akinetic zones at very low doses of dobutamine
cardial ischemic burden is significant, and LV dilation (∼5 mcg/kg/min) is followed by akinesis in these
with stress, rather than compensatory hyperdynamic regions at higher infusion doses (∼20 to 30 mcg/kg/min).
function of nonischemic segments, may be seen. A biphasic response is the effect of severely ische­mic
but not infarcted myocardium which responds with
Answer 11: A increased inotropy at a very low dose of dobutamine,
This is a subcostal short-axis view of the heart. The but becomes frankly ischemic at higher dobutamine
interventricular septum is thin, bright, and scarred, doses. A hyperdynamic LV response is increased
consistent with prior infarct. Color Doppler imaging endocardial motion and a decrease in systolic LV
shows flow towards the transducer across the infarct, cavity size, which is normal and consistent with no
consistent with ventricular septal rupture. The ratio impairment in coronary flow. In patients with a prior
of the stroke volume in the pulmonary artery (Qp) to transmural infarct, normal myocardium adjacent to
the stroke volume in the LV outflow tract (Qs) would infarcted tissue may have decreased systolic motion
be increased, representing the additional volume of due to “tethering” from the akinetic, infarcted region.
flow across the defect from the higher pressure LV In this case, resting LV function was normal without
into the RV. Ischemic ventricular septal rupture is a infarction at baseline.
Cardiomyopathies, Hypertensive,
9 and Pulmonary Heart Disease

CARDIOMYOPATHIES Other Cardiomyopathies


General Step-By-Step Approach ADVANCED HEART FAILURE THERAPIES
Measure LV Chamber Size and Systolic Function Posttransplant Heart Disease
Evaluate for the Presence and Pattern of Ventricular Left Ventricular Assist Devices
Hypertrophy HYPERTENSIVE HEART DISEASE
Assess LV Diastolic Function PULMONARY HEART DISEASE
Estimate Pulmonary Artery Pressures Step-by-Step Approach
Evaluate RV Size and Systolic Function Estimate Pulmonary Pressure
Evaluate the Severity of Mitral and Tricuspid Evaluate RV Size and Systolic Function
Regurgitation Evaluate the Severity of Tricuspid Regurgitation
Evaluate LA Size Exclude other Causes of Pulmonary Hypertension
Additional Steps or Right Heart Enlargement
Dilated Cardiomyopathy THE ECHO EXAM
Hypertrophic Cardiomyopathy SELF-ASSESSMENT QUESTIONS
Restrictive Cardiomyopathy

CARDIOMYOPATHIES n  hree-dimensional (3D) volumetric or apical


T
biplane calculation of end-diastolic and end-systolic
General Step-By-Step Approach ventricular volumes (Fig. 9-1 and Fig. 9-2).
An overall approach to patients with a known or sus-
pected cardiomyopathy is reviewed, followed by spe- v  KEY POINTS
cific features of each type of cardiomyopathy. 3D volumetric LV volumes indexed to body size
o
is the most accurate approach for evaluation of
LV dilation because the pattern of dilation may
Step 1: Measure Left Ventricle Chamber Size not be reflected in basal minor axis dimensions.
and Systolic Function o 2D LV internal dimensions are measured from
the parasternal window because the ultrasound
LV chamber size beam is perpendicular to the blood-myocardial
n 
Two-dimensional (2D) guided M-mode measure- interface, providing high axial resolution.
ment of left ventricle (LV) minor axis internal o 2D imaging in long-and short-axis views will
dimensions at end-diastole and end-systole. ensure that the dimension is measured in the

Diastole Systole

A4C A2C

A B C D

Figure 9-1  Dilated cardiomyopathy.  Apical biplane calculation of ejection fraction is based on tracing endocardial borders at end-diastole (A) and end-
systole (B) in the 4-chamber view (A and B) and in the 2-chamber view (C and D). Foreshortened apical views are avoided by positioning the patient in a steep
left lateral position with an apical cutout in the stretcher to allow the transducer to be positioned on the true apex and after moving the transducer down one
or more interspaces. A2C, Apical 2-chamber; A4C, apical 4-chamber.

168
Cardiomyopathies, Hypertensive, and Pulmonary Heart Disease  CHAPTER 9 169

minor axis of the ventricle (not at an oblique LV systolic function


angle, which would overestimate size). n  LV ejection fraction is calculated using 3D vol-
o On 2D images, the white-black (tissue-blood) umes or the 2D apical biplane approach.
interface is used to measure LV dimensions. n LV dP/dt is calculated from the mitral regurgita-
The
o rapid sampling rate of M-mode (compared tion velocity curve (Fig. 9-4).
with the slow frame rate of 2D imaging) provides n Forward cardiac output is measured in the LV out-
more accurate identification of the endocardial flow tract.
borders (Fig. 9-3). n Regional ventricular function is evaluated qualita-
2D guided M-mode measurements are most

o tively as normal, hypokinetic, or akinetic for each
accurate when the ultrasound beam can be myocardial segment (see Chapter 8).
aligned perpendicular to the LV wall of interest. n 
Global longitudinal strain can be measured using
For sequential studies, measurements should be
o speckle tracking imaging.
made with the same method at the same location.

Figure 9-2  3D measurement of LV ejection


fraction in dilated cardiomyopathy. A full
volume acquisition from the apical window
is used to measure endocardial borders in
three orthogonal views to generate the 3D
volume reconstruction. The time-volume curve
is shown at the bottom of the figure with the
calculated volumes and EF in the upper left.
Volumes are increased and EF reduced in this
patient with dilated cardiomyopathy.

Cal20mm M-mode
Long Axis Short Axis 0

RV
Ao

LV
LA
LV

160
Figure 9-3  M-mode tracing of the LV.  The 2D image is used to ensure the M-line is perpendicular to the long axis of the LV in the parasternal long-axis view
and in the middle of the chamber in the short-axis view. The rapid sampling rate of the M-mode recording (time on the horizontal axis) provides more accurate
identification and measurement of septal and posterior wall thickness and ventricular chamber dimensions at end-diastole (onset of QRS) and end-systole
(maximum posterior motion of septum). Ao, Aorta.
170 CHAPTER 9  Cardiomyopathies, Hypertensive, and Pulmonary Heart Disease

MR dP/dt  427 mmHg/sec APEX MV 0


Vmax 1  1.00 m/sec
Vmax 2  3.00 m/sec
t  75 msec

m/s

EPSS
1 m/s

3 m/s

180
5.0

Figure 9-4  Rate of LV pressure increase.  The rate of increase in velocity Figure 9-5  E-point septal separation (EPSS).  M-mode tracing at the
of the mitral regurgitant jet is markedly reduced in early systole. The calculated level of the mitral valve shows a marked increase in the EPSS consistent
dP/dt of 427 mmHg/s indicated severely reduced LV contractility. with severe LV systolic dysfunction. There also is delayed mitral valve closure
(arrow)—an AC shoulder or B-bump—which is a sign of an elevated LV
filling pressure.

v  KEY POINTS (aortic stenosis, mitral regurgitation, or aortic


o Ejection fraction measured by the apical biplane regurgitation) and coronary artery disease.
method is compared with the visually estimated
ejection fraction. v  KEY POINTS
o The traced endocardial borders are reviewed LV dilation and dysfunction due to coronary
o
for accuracy and retraced if the estimated and disease with myocardial infarction or hiber-
measured ejection fractions differ by more than nation can be difficult to distinguish from a
10 ejection fraction units. primary cardiomyopathy.
o Echo-contrast is used to enhance identification o Mitral regurgitation may be a cause or a conse-
of endocardial borders when image quality is quence of LV dilation and dysfunction. When
suboptimal. more than mild mitral regurgitation (vena con-
o Measured ejection fraction is reported when- tracta > 3 mm) is present, careful quantitative
ever possible. The estimated ejection fraction is evaluation may be helpful (see Chapter 12)
reported only if endocardial borders cannot be (Fig. 9-6).
traced accurately. o The degree of aortic valve leaflet opening is
o Ejection fraction is an imperfect measure of reduced when LV dysfunction is present, mak-
contractility because it is affected by loading ing it difficult to separate severe aortic stenosis
conditions. Even so, ejection fraction is useful resulting in LV dysfunction from moderate aor-
for clinical decision making. tic stenosis with coincidental LV dysfunction
o LV dP/dt and forward stroke volume are other (see Chapter 11). (Fig. 9-7).
useful parameters of LV systolic function (see
Chapter 6).
o Indirect qualitative indicators of LV systolic dys-
Step 2: Evaluate for the Presence
function include M-mode findings of reduced and Pattern of Ventricular Hypertrophy
aortic root motion and increased E-point septal
separation (Fig. 9-5). Presence and severity of LV hypertrophy
o Global longitudinal strain may be more indepen- n  2D guided M-mode measurement of LV wall
dent of loading conditions and more sensitive for thickness
detection of early LV systolic dysfunction. n Calculation of LV mass in selected cases

Evaluate for other cause of LV dilation and systolic v  KEY POINTS


dysfunction 2D guided M-mode measurement of LV wall
o
n  A cardiomyopathy is defined as a primary disease thickness at end-systole (onset of the QRS) is
of the myocardium in the absence of coronary or adequate in most cases.
valvular disease. o Calculation of LV mass from traced 2D endo-
n Evaluate for evidence for other causes of LV dila- cardial and epicardial borders is largely limited
tion and dysfunction, specifically valve disease to research applications.
Cardiomyopathies, Hypertensive, and Pulmonary Heart Disease  CHAPTER 9 171

Ao

LV

LA
Figure 9-6  Secondary mitral regurgita-
tion. Mitral regurgitation in a patient with
dilated cardiomyopathy due to tethering of
the leaflets (arrows) with central to posteri-
orly directed regurgitant jet. Ao, Aorta.

0 Cal10mm
14

RV

Ao

LA
LV

180
Figure 9-7  Aortic root motion.  In this M-mode recording in a patient
with acute viral cardiomyopathy, the aortic tracing shows reduced anterior- 138
posterior motion of the root due to a reduced cardiac output. In addition, the
aortic valve leaflets do not remain fully opened during systole due to low Figure 9-8  Concentric LV hypertrophy.  This M-mode tracing shows in-
transaortic flow. Ao, Aorta. creased wall thickness of both the septum and posterior wall.

Pattern of LV hypertrophy diastolic septal thickness in patients with hyper-


n Long-axis, short-axis, and apical views are used to trophic cardiomyopathy.
evaluate the pattern of ventricular hypertrophy o 2D guided M-mode measurements are most
(Fig. 9-8). accurate when the ultrasound beam can be
aligned perpendicular to the LV wall of inter-
v  KEY POINTS est. Otherwise, 2D measurements at end-systole
2D imaging allows evaluation of the pattern of
o are reported.
hypertrophy in all myocardial segments.
o When hypertrophy is concentric, LV wall thick- Step 3: Assess LV Diastolic Function
ness measurements at one site adequately rep- (see Chapter 7)
resent the degree of hypertrophy. n LV and LA inflow patterns (Fig. 9-9)
o When hypertrophy is asymmetric, measure- n Tissue Doppler at the mitral annulus (Fig. 9-10)
ments at key sites are reported, particularly the n Isovolumic relaxation time (Fig. 9-11)
172 CHAPTER 9  Cardiomyopathies, Hypertensive, and Pulmonary Heart Disease

38dB 3 /1/0/1 PW:2MHz LV IVRT  75 msec


PW Depth  97mm
PW Gate  2.5mm
PW Gain  1dB
.60

2.5

m/s

m/s
.40
Figure 9-9  Severe LV diastolic dysfunction. LV diastolic filling in a
Figure 9-11  Isovolumic relaxation time (IVRT). The IVRT, measured
62-year-old patient with a dilated cardiomyopathy showing an increased E/A
from the end of aortic ejection flow to the onset of mitral inflow, is normal
ratio with a steep early diastolic deceleration slope. The E is greater than A in
at 75 ms.
this patient older than age 50, in association with a steep deceleration slope,
suggests severe LV diastolic dysfunction with elevated filling pressures.

CW:2MHz APX TV
38dB 1 /1/0/1
PW Depth  106mm
PW Gate  2.5mm 1.0
PW Gain  27dB

m/s

.15

m/s

3.0
.15
Figure 9-12  Estimates of pulmonary systolic pressure.  This tricuspid
Figure 9-10  Tissue Doppler at the mitral annulus.  This tracing shows regurgitant jet velocity obtained from an apical view (APX TV) has a peak
an E′ less than 0.10 m/s, with an E′ greater than A′, in the same patient as somewhere around 2.5 m/s. This signal is not ideal because the peak is not
Figure 9-9. The ratio of transmitral E velocity to tissue Doppler E′ velocity well defined. This likely is due to the position of the ultrasound beam relative
is 2.2/0.10 = 22, which is severely elevated, consistent with a high left to the jet and the use of color coding (instead of gray scale) for the spectral
atrium pressure. display, which often obscures the peak velocity with superimposed noise.
However, the peak velocity of about 2.4 m/s indicates a RV-to-RA systolic
v  KEY POINTS pressure difference of 23 mmHg.

Systolic LV dysfunction typically is accompa-


o
nied by some degree of diastolic dysfunction. n  ther signs of pulmonary hypertension include
O
o Classification of the degree of diastolic dys- a short time to peak velocity in the pulmonary
function as mild (impaired relaxation) versus artery velocity curve, paradoxical septal motion,
severe (decreased compliance) is used in clinical and a high end-diastolic pulmonic regurgitant
decision making. velocity.
o LV filling pressures are estimated whenever sys-
tolic dysfunction is present. v  KEY POINTS

Pulmonary pressures often are elevated
o
Step 4: Estimate Pulmonary Artery in patients with heart failure due to a
Pressures (see Chapter 6) cardiomyopathy.
n 
Pulmonary systolic pressure is estimated from the o 
Pulmonary pressures may be reduced with
tricuspid regurgitant jet velocity and estimated RA effective medical therapy, in conjunction with a
pressure (Fig. 9-12 and Fig. 9-13). decrease in LA (or LV filling) pressure.
Cardiomyopathies, Hypertensive, and Pulmonary Heart Disease  CHAPTER 9 173

v  KEY POINTS
Mitral and tricuspid regurgiation due to LV
o
SNIFF dilation and dysfunction are common.
o The mechanism of atrioventricular valve regur-
gitation is tethering of the mitral leaflets resulting
in incomplete systolic coaptation. This also has
been described as an increased angle between
the papillary muscles so that the leaflets are
“pulled apart” relative to the mitral annulus
(Fig. 9-15).
IVC o The degree of annular dilation is variable, with
a variable contribution to the degree of mitral
RA regurgitation.
o Mitral regurgitation severity may decrease with
effective therapy for heart failure.
Step 7: Evaluate LA Size (see Chapter 2)
n  A size typically is increased either due to chronic
L
Figure 9-13  RA pressure.  The junction of the inferior vena cava (IVC) and elevation of LV filling pressures or to coexisting
RA is visualized from the subcostal window for estimating RA pressure. In this mitral regurgitation (see Fig. 2-22).
patient, the IVC diameter is about 2.5 cm and collapses less than 50% with
rapid inspiration (SNIFF), consistent with an RA pressure of 10 to 15 mmHg. n LA size can be evaluated qualitatively, using a sim-
ple anterior-posterior dimension, or by calculation
of atrial volume from apical views.
o When pulmonary pressures are elevated dispro-
portionately to the degree of left-sided heart dys- v  KEY POINTS
function, concurrent primary pulmonary disease LA anterior-posterior dimension is measured in
o
or pulmonary thromboembolism may be present. a long-axis view at end-systole (maximum LA
dimension).
Step 5: Evaluate RV Size and Systolic o Although atrial volumes are predictive of clini-
Function cal outcome, simpler measures of atrial size
n RV size and systolic function are assessed from suffice for clinical decision making in many
parasternal, apical, and subcostal views (Fig. 9-14). cases.
n 
Standard measures of RV size and function are o When clinically indicated, LA volume is calcu-
used (see Chapter 6). lated from tracing the LA border at end-systole
in apical 4-chamber and 2-chamber views. LA
v  KEY POINTS volume then is indexed for body size.
RV systolic dysfunction may be due to primary
o o RA size also may be increased and is evaluated
myocardial disease affecting both ventricles or qualitatively (as mild, moderately, or severely
to the effects of pulmonary hypertension. dilated) in comparison to the other cardiac
o Qualitative assessment of RV size and function chambers.
takes into account the degree of LV dilation and
dysfunction. RV lengthwise systolic shortening Additional Steps
and tricuspid annular plane systolic excursion
are reduced when RV dysfunction is present. Dilated Cardiomyopathy
n Evaluate for LV apical thrombus (Fig. 9-16).
Step 6: Evaluate the Severity of Mitral and n  Differentiate from end-stage coronary disease
Tricuspid Regurgitation (see Chapter 12) (Table 9-1).
n  Mitral and tricuspid regurgitation often are pres- n  Differentiate LV dysfunction due to severe mitral
ent in patients with a cardiomyopathy. regurgitation from a primary cardiomyopathy with
n  Regurgitant severity is evaluated using standard secondary mitral regurgitation.
approaches, starting with the color Doppler vena n Ensure an accurate ejection fraction calculation.
contracta and the continuous wave (CW) Doppler
velocity curve. v  KEY POINTS
n The mechanism of regurgitation is evaluated using o 
Decision making for placement of an auto-
2D imaging from multiple views; typically regur- mated implanted defibrillator is based on the
gitation is secondary to ventricular dilation and calculated ejection fraction, with the breakpoint
dysfunction but some patients have concurrent typically at an ejection fraction less than 35%
structural valve disease. after optimization of medical therapy.
174 CHAPTER 9  Cardiomyopathies, Hypertensive, and Pulmonary Heart Disease

RV LV LV

RV

RA
LA

RA

A B
Figure 9-14  Right ventricular size.  The RV is imaged from the apical 4-chamber view (A) by tilting the transducer toward the RV and narrowing the sector (B).

MR

Leaflet tethering
Normal Dilated cardiomyopathy
Figure 9-15  Mitral leaflet tethering.  Schematic diagram showing leaflet tethering with dilated cardiomyopathy due to lateral displacement of the papillary
muscles, resulting in an oblique angle to the mitral annulus. MR, Mitral regurgitation.


o Examination for LV apical thrombus includes 
oFeatures that suggest end-stage coronary dis-
oblique views of the LV apex using a high-­ ease include: definite evidence for myocardial
frequency transducer and shallow image depth. infarction (segmental thinning and akinesis) and
Contrast imaging may be helpful to exclude throm- normal RV size and systolic function. However,
bus when standard imaging is not diagnostic. direct visualization of coronary anatomy by con-
o Transesophageal echocardiography (TEE) is not ventional catheter or computed tomographic
useful to evaluate for apical thrombus due to the angiography typically is needed.
distance of the apex from the transducer and the o Stress echocardiography is difficult to interpret
likelihood that the true apex may be missed from with significant resting LV systolic dysfunction
this approach. with a suboptimal sensitivity and specificity for
o LV systolic dysfunction due to coronary dis- detection of ischemia.
ease and to a primary cardiomyopathy may o Speckle tracking strain imaging is a promising
look similar on echocardiography. Both may approach for more quantitative evaluation of
show wall motion that is best preserved at the global and regional function in patients with a
­inferior-lateral base. cardiomyopathy.
Cardiomyopathies, Hypertensive, and Pulmonary Heart Disease  CHAPTER 9 175

LV
RV
Thrombus
Artifact
RA LA Figure 9-16  LV apical thrombus.  A, In the
apical 4-chamber view, the LV is dilated with
marked thickening along the apical septum
LV suggestive of thrombus. B, Using a higher
transducer frequency and shallow image
depth the laminated thrombus filling the apex
A B is better visualized. At this depth, a ring-down
artifact also is seen overlying the LV chamber.

TABLE 9-1 Cardiomyopathies: Clinical Echocardiographic Correlation


Cardiomyopathy Pathophysiology Clinical Presentation Echocardiographic Findings
Dilated
Idiopathic Primary myocardial dysfunc- Heart failure signs and Dilation of all four chambers with
tion of unknown cause. symptoms. RV and LV systolic dysfunction.
Functional mitral regurgitation may
be present but valve leaflets are
normal.
LV thrombus can occur with severe
LV dysfunction.
Elevated LV filling pressures with
variable elevation in PA pressures.
Familial Inherited primary ­myocardial Heart failure signs and Dilation of all four chambers with
dysfunction. symptoms. RV and LV systolic dysfunction.
Functional mitral regurgitation may
be present but valve leaflets are
normal.
LV thrombus can occur with severe
LV dysfunction.
Elevated LV filling pressures with
variable elevation in PA pressures.
Chagas Protozoan infection due to Acute phase is characterized LV dilation and systolic dysfunction,
Trypanosoma cruzi that af- by fever, myalgias, ranging from mild to severe. Wall
fects the heart, ­esophagus, hepatosplenomegaly, and motion may be regional but not in
and colon. myocarditis. a pattern consistent with coronary
Chronic Chagas heart disease artery disease.
has a high mortality rate Apical abnormalities are common
(44% at 4 years) due to sudden with apical aneurysm in about
death (55-65%), heart failure 5% of asymptomatic patients and
(25-30%), and stroke (10-15%). about 55% of those with heart
failure.
Duchenne MD Inherited myopathic ­disorder Often have asymptomatic LV Echocardiography is consistent
that affects both skeletal dysfunction, likely due to with dilated cardiomyopathy.
and cardiac muscle. limited physical activity. Late
in disease, heart failure and
arrhythmias are seen.
176 CHAPTER 9  Cardiomyopathies, Hypertensive, and Pulmonary Heart Disease

TABLE 9-1 Cardiomyopathies: Clinical Echocardiographic Correlation—cont’d


Cardiomyopathy Pathophysiology Clinical Presentation Echocardiographic Findings
Hypertrophic
Inherited autosomal domi- Wide age range of clinical Asymmetric LV hypertrophy with
nant myocardial disease. presentation. normal systolic function but
Often diagnosed in asymptomatic abnormal diastolic function.
patients on screening echo. About 1/3 have resting dynamic
May present as sudden death outflow obstruction and 1/3
with no previous diagnosis or have a provocable gradient with
with symptoms of heart failure exercise.
and angina.
Restrictive
Amyloid Extracellular tissue deposition Conduction system disease. Increased LV and RV wall thickness
of serum protein subunit Myocardial involvement. with increased myocardial
fibrils—cardiac involvement echogenicity but “sparkling”
in 50% of primary AL appearance is not specific or
amyloidosis (monoclonal sensitive for diagnosis.
light chains) cases but only Progressive diastolic dysfunction.
5% with secondary AA Valve thickening.
amyloidosis. Intracardiac thrombus.
Fabry Inherited X-linked glycolipid Presents in boys under age LV hypertrophy may be asymmetric
storage disease, now 10 years with skin and but in an atypical pattern for
recognized in women as neurologic findings. Women HCM.
well as men. present later in life with Endocardial hyperechoic layer is
unexplained LV ­hypertrophy. typical for Fabry heart disease.
Diagnosis based on plasma About 50% have aortic and
alpha-galactosidase A activity. mitral valve thickening and mild
Conduction system abnormalities regurgitation.
and arrhythmias are common.
Isolated LV non- Rare primary genetic Clinical presentation with heart Deep ventricular trabeculations,
compaction cardiomyopathy failure, angina, arrhythmias, and particularly in the inferior and
thromboembolic events. lateral walls. Color Doppler
shows communication between
intertrabecular recesses and LV
chamber.
Ejection fraction may be reduced.
Ratio of noncompacted to
compacted myocardium > 2:1 at
end systolic in short-axis view.
Takotsubo Catecholamine induced acute Sudden onset chest pain, Apical dilation and systolic
(stress-­ myocardial dysfunction. ­dyspnea, ECG changes, dysfunction resulting in a
induced and elevated cardiac enzymes significant reduction in LV
cardio­ with normal coronary arteries. ejection fraction.
myopathy) Occurs in the setting of intense Pattern of regional myocardial
emotional or physical stress or dysfunction is atypical for
with an acute medical illness. coronary disease.
Over 80% of cases are women, LV systolic function typically returns
typically age 50-75 years. to normal in 1-4 weeks although
recurrences have been reported.
Arrhythmogenic Familial inheritance in at Presents with sudden RV dilation and systolic dysfunction.
RV cardio­ least 30%, most often in cardiac death or Echo findings are nonspecific;
myopathy an autosomal dominant ventricular ­arrhythmias. diagnosis depends on CMR
pattern. Autosomal imaging and electrophysiologic
recessive inheritance also evaluation.
has been described.

From Otto CM: Cardiomyopathies, hypertensive and pulmonary heart disease. In Textbook of Clinical Echocardiography, ed 5. Philadelphia, Elsevier, 2013.
CMR, Cardiac magnetic resonance imaging; ECG, electrocardiogram; HCM, hypertrophic cardiomyopathy; MR, mitral regurgitation; PA, pulmonary
artery.
Cardiomyopathies, Hypertensive, and Pulmonary Heart Disease  CHAPTER 9 177

Hypertrophic Cardiomyopathy n  valuate the end-diastolic wall thickness of the


E
basal posterior wall.
Step 1: LV Hypertrophy n Measure the maximal end-diastolic septal thickness.
n 
Describe the anatomic pattern of hypertrophy Wall thickness measurements should be taken from
(Fig. 9-17). both long-axis and short-axis views of the ventricle.
v  KEY POINTS
o The most common pattern of hypertrophy
involves the ventricular septum with a normal
posterior LV wall.
o Apical hypertrophy may be missed unless echo-
contrast is used to opacify the ventricle, because
the endocardial border in the mid-LV may be
difficult to visualize (Fig. 9-18).
Ao o Even with atypical patterns of hypertro-
LV
phy, the basal posterior wall thickness typi-
cally is normal in patients with hypertrophic
cardiomyopathy.
o Maximal septal thickness is a predictor of sud-

LA den death risk. Septal thickness is measured at


end-diastole, taking care to exclude RV trabec-
ulations from the measurement.
o Screening of first-degree relatives is recom-
mended when hypertrophic cardiomyopathy is
diagnosed, because early detection and treat-
ment can prevent sudden death.

Step 2: Dynamic Subaortic Outflow Obstruction


Figure 9-17  Hypertrophic cardiomyopathy. The parasternal long-axis n Evaluate dynamic subaortic outflow obstruction.
view at end diastole shows marked increased thickness of the interventricu- n Evaluate the mechanism and severity of mitral
lar septum with a normal thickness of the posterior wall. Ao, Aorta. regurgitation.

A B
Figure 9-18  Subaortic obstructions.  Apical 4-chamber view (A) in a young woman with apical hypertrophic cardiomyopathy showing the small chamber
(double arrow) and thick walls at the apex on a diastolic frame. In systole, the long-axis view (B) shows flow acceleration at the midventricular level (arrow),
instead of the subaortic obstruction seen with more typical basal septal hypertrophy.
178 CHAPTER 9  Cardiomyopathies, Hypertensive, and Pulmonary Heart Disease

v  KEY POINTS V  4.32m/s


o Subaortic obstruction is due to systolic anterior PG  74.7mmHg
motion (SAM) of the mitral leaflet and a hyper-
trophied septum (Fig. 9-19). 2.0
o The Doppler velocity curve peaks in late systole,
instead of in mid-systole as seen with valvular
obstruction. (Fig. 9-20).
o The level of obstruction is established with m/s
pulsed or high-pulse repetition frequency Dop-
pler, with CW Doppler used to measure the
maximum velocity.
o The severity of obstruction varies with load-
ing conditions, increasing when afterload is
decreased or when ventricular volume is reduced.
o Exercise testing may be used to evaluate the
change in outflow obstruction with exertion;
latent obstruction is defined as an increase in sub-
aortic velocity to at least 2.7 m/s with exercise.
o Mitral regurgitation may be due to SAM of the
mitral leaflet, resulting in inadequate coapta-
6.0
tion with a typical posterior-directed regurgi-
tant jet (Fig. 9-21). Figure 9-20  Dynamic subaortic obstruction. CW Doppler recording
from an apical approach of outflow velocity in a patient with hypertrophic
cardiomyopathy. This waveform is consistent with dynamic obstruction with
Step 3: Distinguish From Hypertensive Heart Disease a late-peaking high-velocity systolic signal. The level of obstruction is not
and Normal Aging Changes defined using CW Doppler because the signal includes velocities from the
n  Hypertensive heart disease may be mistaken for entire length of the ultrasound beam. A step-by-step pulsed Doppler evalua-
hypertrophic cardiomyopathy. tion moving the sample volume from the left ventricle toward the aortic valve
n LV hypertrophy in a patient with a clinical history
allows localization of the site of obstruction.
of hypertension most often is due to hypertensive
heart disease.

LV

RV

MR

RA

LA LA

Figure 9-21  Secondary mitral regurgitation. Mitral regurgitation in a


Figure 9-19  Mitral systolic anterior motion (SAM).  Hypertrophic cardio- patient with hypertrophic cardiomyopathy and dynamic subaortic obstruc-
myopathy in a parasternal long-axis view, showing SAM of the mitral leaflet tion. The mitral regurgitant posterior-lateral jet direction is demonstrated in
(arrow) causing dynamic subaortic obstruction. this apical 4-chamber view. MR, Mitral regurgitation.
Cardiomyopathies, Hypertensive, and Pulmonary Heart Disease  CHAPTER 9 179

PW:2MHz PW:2MHz
1.5 .05

m/s

.10
m/s
A B
PW:2MHz PW:2MHz

.30
.50

m/s
m/s

.50
.20

C D
Figure 9-22  Diastolic dysfunction.  Evaluation of diastolic function in a patient with a restrictive cardiomyopathy includes transmitral flow (A), tissue Doppler
velocity at the mitral annulus (B), pulmonary vein flow (C), and the isovolumic relaxation time (D). These tracings show impaired diastolic relaxation (transmitral
and tissue Doppler E less than A, prolonged deceleration time, and prolonged IVRT). LA pressure also may be elevated with an E/E′ greater than 30, even though
the pulmonary venous a-wave is low velocity and short in duration.

n With aging, dilation and tortuosity of the ascend- Restrictive cardiomyopathy is characterized by
o
ing aorta result in an increased angle between the predominant diastolic, rather than systolic, dys-
basal septum and aortic root, with bulging of the function so that detailed evaluation of diastolic
septum into the LV outflow tract. function is helpful.
o Restrictive cardiomyopathy and constrictive
v  KEY POINTS pericarditis result in similar changes in ventric-
Hypertension results in concentric LV hypertro-
o ular filling but can be differentiated based on
phy; even the basal posterior wall is thickened. several features (see Table 10-2).
o Dynamic outflow obstruction may occur with o LV systolic dysfunction may also be present,
hypertensive heart disease, but the location of especially late in the disease course.
obstruction is midventricular and systolic ante- o Pulmonary systolic pressure usually is moder-
rior motion occurs in the mitral chordal region, ately to severely elevated.
instead of at the leaflet level.
o Age-related bulging of the basal septum can Other Cardiomyopathies
be difficult to distinguish from hypertrophic n Arrhythmogenic RV dysplasia (ARVD) is a genetic
cardiomyopathy; diagnosis depends on associ- cardiomyopathy with fibrofatty replacement of the
ated echocardiographic and clinical findings, RV resulting in arrhythmias, dilation, and systolic
as well as genetic and family studies. dysfunction.
n  LV noncompaction is characterized by areas of
Restrictive Cardiomyopathy prominent trabeculation and hypokinesis, typically
n Perform a more detailed evaluation of LV diastolic located in the LV apex and mid-LV segments of
function (Fig. 9-22). the lateral and inferior walls (Fig. 9-23).
n Differentiate restrictive cardiomyopathy from con- n  Chagas disease is due to a parasitic infection. It
strictive pericarditis (see Chapter 10). results in apical aneurysm formation in about 50%
of patients and global hypokinesis in those with
v  KEY POINTS advanced disease.
o Restrictive cardiomyopathy is a primary disease n Takotsubo cardiomyopathy is an acute stress-
of the myocardium, often related to an infiltra- related cause of heart failure with apical dilation
tive or inflammatory process. and dyskinesis (apical "ballooning").
180 CHAPTER 9  Cardiomyopathies, Hypertensive, and Pulmonary Heart Disease

LV

RV

RA
LA

RA LA

Figure 9-23  Left ventricle noncompaction. Apical 4-chamber view


showing LV noncompaction, particularly in the apex. An LV web or false ten-
Figure 9-24  After heart transplantation. In this patient with biatrial
don from the lateral wall to septum also is present.
anastomoses at heart transplantation, the enlarged left and right atria,
caused by suturing of the native and donor atrium (arrow shows suture line),
are seen in the apical 4-chamber view.

ADVANCED HEART FAILURE THERAPIES


Left Ventricular Assist Devices
Posttransplant Heart Disease n  End-stage heart failure may be treated acutely or
n  Evaluation of the patient after heart transplanta- chronically with an implanted continuous flow
tion follows the general approach outlined for a device.
patient with a cardiomyopathy. n  The most common type of left ventricular assist
n  Early after heart transplantation, the major issues devices (LVAD) has an inflow cannula in the LV
are surgical (e.g., pericardial effusion) and myo- apex and an outflow cannula in the ascending aorta.
cardial preservation (e.g., RV and LV systolic n  Echocardiography allows detection of LVAD
function). complications.
n Echocardiographic signs of transplant rejection n Measurements of LV size can be used to optimize
include diastolic and systolic LV dysfunction. LVAD flow rates.
n 
After transplant, long-term follow-up of patients
includes dobutamine stress echocardiography for v  KEY POINTS
detection of graft coronary artery disease. The aortic valve remains closed throughout the
o
cardiac cycle in patients with an LVAD.
v  KEY POINTS o Flow signals in the inflow and outflow cannula

Biatrial enlargement is typical after heart
o can be helpful for diagnosis of obstruction of
transplantation, with sometimes massive atrial the cannula (Fig. 9-25).
enlargement with biatrial anastomotic suture o A health care provider experienced in manage-
lines (Fig. 9-24). ment of LVAD patients is present during the
o 
Complications of percutaneous myocardial echocardiographic study if measurements at
biopsy include (1) cardiac perforation result- different LVAD flow rates are needed.
ing in pericardial effusion and tamponade o Evaluation of RV size and systolic function is
and (2) tricuspid valve damage resulting in important when an LVAD is present.
regurgitation.
o 
New systolic dysfunction, even if mild, may
indicate acute rejection and requires prompt HYPERTENSIVE HEART DISEASE
evaluation by the transplant team. n  valuation of the patient with hypertensive heart
E
o 
When dobutamine stress echocardiography disease follows the general approach outlined for a
is performed after heart transplantation, atro- patient with a cardiomyopathy.
pine may not increase heart rate due to cardiac n Blood pressure should be recorded at the time of
denervation. every echocardiographic examination.
Cardiomyopathies, Hypertensive, and Pulmonary Heart Disease  CHAPTER 9 181

LV

LV
Ao

LA

1.0

Figure 9-26  Hypertensive heart disease.  Parasternal long-axis view in a


patient with hypertension showing mild ventricular hypertrophy, aortic valve
sclerosis, and mild mitral annular calcification. Ao, Aorta.
m/s
.20

Figure 9-25  Left ventricular assist device (LVAD).  The LVAD inflow can-
nula (arrow) is seen in the apical view. Pulsed Doppler shows continuous low
velocity flow into the LVAD cannula.

v  KEY POINTS
o Hypertension results in LV hypertrophy with
impaired diastolic relaxation.
o Prolonged poorly controlled hypertension may Ao
eventually result in more severe diastolic dysfunc- LV
tion and in superimposed systolic dysfunction.
o Effective treatment of hypertension results in
regression of LV hypertrophy.
o LV hypertrophy may be accompanied by
dynamic midcavity obstruction due to a small, LA
thick-walled, hyperdynamic ventricle.
o Obstruction may only be present or may
increase with hypovolemia or hyperdynamic
states (such as anemia, fever, sepsis).
o Aortic valve sclerosis and mitral annular calcifi-
cation are typically seen in patients with hyper- Figure 9-27  Basal LV septal prominence. Low parasternal long-axis
tensive heart disease (Fig. 9-26). view in an elderly patient showing prominence of the base of the septum
o Hypertension, like aging, is associated with dila- (sometimes called a septal knuckle; arrow) due to an increased angle be-
tion and increased tortuosity of the ascending tween the long axes of the aorta and LV. Mild mitral annular calcification is
present, but there is no evidence of LV hypertrophy. Ao, Aorta.
aorta, resulting in a more acute angle between
the ventricular septum and the aortic root,
sometimes mistaken for focal basal septal thick-
ening (Fig. 9-27). n  he effects of pulmonary hypertension on the
T
right heart result in pulmonary heart disease (or
cor pulmonale) (Fig. 9-28).
PULMONARY HEART DISEASE n New onset of increased pulmonary pressures in
n  ulmonary hypertension in the absence of signifi-
P a patient with acute chest pain or shortness of
cant left-sided heart disease indicates primary pul- breath suggests the possibility of acute pulmonary
monary or pulmonary vascular disease. embolism.
182 CHAPTER 9  Cardiomyopathies, Hypertensive, and Pulmonary Heart Disease

regurgitant peak velocity (VTR) and the veloc-


ity-time integral of flow in the RV outflow tract
(VTIRVOT) using the equation:

PVR = 10 (VTR /VTIRVOT ) (9-1)

Step 2: Evaluate RV Size and Systolic


RV LV Function
n  The RV responds to chronic pressure overload with
dilation of the chamber, in addition to hypertrophy of
the wall, often accompanied by systolic dysfunction.
n RV size is measured as described in Chapter 6.
RA
n  RV systolic function is evaluated using the tricus-
LA
pid annular plane systolic excursion (TAPSE) and
the Doppler tissue annular systolic velocity.
n The pattern of ventricular septal motion is evalu-
ated in a parasternal short-axis view.
v  KEY POINTS
Visually, if the LV is normal in size, the RV is
o
severely dilated if the 2D area in a 4-chamber
Figure 9-28  Cor pulmonale.  In this patient with severe pulmonary hyper- view is larger than the LV; moderately dilated
tension there is severe RV hypertrophy and dilation and severely reduced RV
systolic function. The RA is severely enlarged, with bulging of the atrial sep-
if equal to the LV; and mildly dilated if greater
tum from right to left, suggesting that RA pressure is higher than LA pressure. than normal but not equal to the LV.
o RV size and systolic function are evaluated based
Step-by-Step Approach on parasternal, apical, and subcostal views.
o RV size and function often are best evaluated
Step 1: Estimate Pulmonary Pressure from the subcostal window because the RV is
n Pulmonary systolic pressure is determined based on seen in oblique image planes from the paraster-
the velocity in the tricuspid regurgitant jet and the nal view and the RV free wall may be difficult to
estimated RA pressure (see Chapter 6) (Fig. 9-29). visualize on apical views.
n 
Additional signs of pulmonary hypertension also o Elevated RV pressure results in flattening of
are evaluated. the ventricular septum during both systole and
diastole, whereas right heart volume overload
v  KEY POINTS results in flattening mostly during diastole.
o 
When 2D echocardiography findings show
right heart dysfunction, a diligent search for the Step 3: Evaluate the Severity of Tricuspid
highest-velocity tricuspid regurgitant-jet is espe- Regurgitation
cially important. n  Pulmonary hypertension often results in dilation
o 
Other findings that suggest pulmonary hyper- of the tricuspid annulus with inadequate leaflet
tension include a short time to peak velocity coaptation and tricuspid regurgitation.
and mid-systolic deceleration of the pulmonary n Tricuspid regurgitation severity is evaluated based
artery velocity curve and paradoxical septal on the vena contracta width of the regurgitant jet,
motion (Fig. 9-30). the density of the CW Doppler velocity curve, and
o 
The velocity of the tricuspid regurgitant-jet the pattern of flow in the hepatic veins.
reflects the systolic pressure difference between
the RV and RA, not the volume of regurgita- v  KEY POINTS
tion. Thus, severe pulmonary hypertension may Vena contracta width is best measured in the
o
be present with only mild tricuspid regurgitant. parasternal RV inflow view; a width greater
o 
Overestimation of pulmonary pressures is than 7 mm indicates severe regurgitation.
avoided by using a gray-scale velocity display, o The density of the CW Doppler signal is com-
increasing the high-pass (or wall) filter and pared with antegrade flow: equal density indi-
adjusting the gain level appropriately. The low- cates severe regurgitation.
intensity linear signals outside the edge of the o The normal hepatic vein pattern of systolic flow
velocity envelope are not included in the veloc- into the RA is reversed when tricuspid regurgi-
ity measurement. tation is severe. However, systolic flow reversal
o 
Pulmonary vascular resistance (PVR in Wood also may be seen when the patient is not in sinus
units) can be estimated from the tricuspid rhythm, even when regurgitation is not severe.
Cardiomyopathies, Hypertensive, and Pulmonary Heart Disease  CHAPTER 9 183

m/s

4.0

Figure 9-29  Pulmonary hypertension.  Tricuspid regurgitant jet recorded with CW Doppler from an apical window. This recording shows a well-defined peak
velocity with a dark band of velocities along the outer edge of the velocity curve, consistent with a good-quality signal. Even though color Doppler was used
to guide placement of the CW Doppler ultrasound beam, a nonparallel intercept angle cannot be excluded with certainty. However, this velocity indicates an
RV-to-RA systolic pressure difference of 70 mmHg, consistent with severe pulmonary hypertension.

PW:2MHz
n Right heart enlargement without severe pulmo-
nary hypertension is seen with volume overload
due to valve regurgitation or a left to right shunt.
m/s
v  KEY POINTS
Left-sided or congenital heart disease results in
o
secondary pulmonary hypertension, which is
1.0
easily distinguished from primary pulmonary
disease.
o Right-sided volume overload in the absence of
Figure 9-30  Mid-systolic notching.  Pulsed Doppler recording of ante- an obvious atrial septal defect or severe right-
grade flow in the pulmonary artery from the parasternal RV outflow view, in sided valve regurgitation prompts TEE exami-
the same patient as Figure 9-29, shows a short time to peak velocity and a nation to exclude a sinus venous atrial septal
mid-systolic notch (arrow) in the velocity curve, which are specific for severe
pulmonary hypertension. defect or partial anomalous pulmonary venous
return.

Step 4: Exclude other Causes of Pulmonary


Hypertension or Right Heart Enlargement
n 
Pulmonary hypertension and right heart enlarge-
ment also may be due to left-sided heart disease or
congenital heart disease.
184 CHAPTER 9  Cardiomyopathies, Hypertensive, and Pulmonary Heart Disease

THE ECHO EXAM

Cardiomyopathies and Hypertensive and Pulmonary Heart Disease


Echo Differential Diagnosis of Heart Failure
Ischemic disease
Valvular disease
Hypertensive heart disease
Cardiomyopathy
 Dilated
 Hypertrophic
 Restrictive
 Other
Pericardial disease
 Constriction
 Tamponade
Pulmonary heart disease

Differentiation of Cause of Increased Wall Thickness


Hypertrophic
Hypertensive Heart Disease Cardiomyopathy Restrictive Cardiomyopathy
LV hypertrophy + + +
Pattern of hypertrophy Concentric Asymmetrical Concentric
Clinical history of + Absent Absent
­hypertension
Outflow obstruction Midventricular cavity Dynamic subaortic Absent
obliteration obstruction
RV hypertrophy Absent May be present +
Pulmonary hypertension Mild Mild Moderate
LV systolic function Normal initially but may be Normal Normal initially but may be
reduced late in disease reduced late in disease
course course
LV diastolic function Abnormal Abnormal Abnormal

+, Present.
Cardiomyopathies, Hypertensive, and Pulmonary Heart Disease  CHAPTER 9 185

Cardiomyopathies: Typical Features


Dilated Hypertrophic Restrictive Athlete’s Heart
LV systolic function Moderately-severely ↓ Normal Normal Normal
LV diastolic function May be abnormal Abnormal Abnormal Normal
LV hypertrophy ↑LV mass due to LV Asymmetrical LV Concentric LV Normal wall thickness
dilation with normal hypertrophy hypertrophy
wall thickness
Chamber dilation All four chambers Left and right atrial Left and right atrial LV dilation
­dilation if MR is dilation
present
Outflow tract Absent Dynamic LV outflow Absent Absent
­obstruction tract obstruction
may be present
LV end-diastolic Elevated Elevated Elevated Normal
pressure
Pulmonary artery Elevated Elevated Elevated Normal
pressures
MR, Mitral regurgitation.

Echo Approach to the Cardiomyopathies


Modality Echo Views and Flows Measurements
Imaging LV size and systolic function LV-EDV, LV-ESV
Apical biplane EF
Degree and pattern of LV hypertrophy LV-mass
Evidence for dynamic outflow tract SAM of the mitral valve
­obstruction Aortic valve midsystolic closure
RV size and systolic function
LA size LA volume
Doppler echo Associated valvular regurgitation Measure vena contracta, quantitate if more
than mild
LV diastolic function Standard diastolic function evaluation with
­classification of severity and estimate
of LV-EDP
LV systolic function dP/dt from MR-jet
Calculation of cardiac output
Pulmonary pressures TR jet and IVC for PA systolic pressure,
Evaluate PR-jet for PA diastolic pressure,
Estimate pulmonary resistance
Color, pulsed, and CW Doppler Maximum outflow tract gradient
to quantitate outflow obstruction
LV-EDV, Left ventricular end-diastolic volume; LV-ESV, left ventricular end-systolic volume; MR-jet, mitral regurgitation jet; PA, pulmonary artery;
PR-jet, pulmonic regurgitation jet; SAM, systolic anterior motion; TR-jet, tricuspid regurgitation jet.
186 CHAPTER 9  Cardiomyopathies, Hypertensive, and Pulmonary Heart Disease

SELF-ASSESSMENT QUESTIONS

Question 1 Question 3
Which of the following is most consistent with LV sys- Which of the following is most consistent with right
tolic dysfunction? ventricular systolic dysfunction?
A. Systolic mitral valve apposition to LV septum A. Tissue Doppler tricuspid annulus systolic veloc-
B. Deceleration slope of mitral regurgitant Dop- ity = 5 cm/s
pler envelope B. Tricuspid annular plane systolic excursion =
C. M-mode imaging of aortic root motion 3.0 cm
D. Aortic valve leaflet opening diameter C. Right ventricular annular diameter = 3.0 cm
D. Tricuspid regurgitant jet acceleration slope =
Question 2 1500 m/s
A 36-year-old man with hypertrophic cardiomyopa-
thy is evaluated with echocardiography. Doppler trac- Question 4
ings are obtained (Fig. 9-31). Which features of the A 66-year-old man is referred for exertional dys-
Doppler signal are the most helpful in distinguishing pnea. Twelve-lead ECG shows diffuse, nonspecific
the LV outflow velocity from mitral regurgitation? ST-T wave abnormalities. A coronary angiogram
A. Duration of flow documented absence of coronary artery disease. An
B. Maximum velocity echocardiogram is obtained (Fig. 9-32). The image is
C. Accompanying diastolic flow velocity consistent with:
D. Color imaging of jet direction A. Ebstein anomaly
B. Takotsubo cardiomyopathy
CW:2MHz C. LV noncompaction
D. Amyloid heart disease

m/s

6.0

A
CW:2MHz

2.0

m/s

Figure 9-32 

8.0
B
Figure 9-31 
Cardiomyopathies, Hypertensive, and Pulmonary Heart Disease  CHAPTER 9 187

Question 5
A 62-year-old patient status post–cardiac transplan- His LV inflow (A), tissue Doppler (B), and LA inflow
tation 12 years ago for a familial cardiomyopathy is velocities (C) are shown (Fig. 9-33 A, B, C). These find-
referred for echocardiography. The following mea- ings are most consistent with:
surements are recorded: A. Transplant rejection
B. Coronary vasculopathy
LV dimension, end- 5.0 cm/3.5 cm
diastole/end-systole
C. Pericardial constriction
Ejection fraction 62% D. Normal heart
LA dimension 5.2 cm
Tricuspid regurgitant jet 2.1 m/s

PW:2MHz Sweep100mm/s

1.0

m/s

PW:2MHz

.20

m/s

B
.20

PW:2MHz

1.0

m/s

Figure 9-33 
188 CHAPTER 9  Cardiomyopathies, Hypertensive, and Pulmonary Heart Disease

Question 6 Question 7
A 42-year-old woman presents for evaluation of Characteristic echocardiographic findings commonly
exertional dyspnea. The following M-mode image is found in hypertensive heart disease include all of the
obtained from a parasternal view (Fig. 9-34). Based following except:
on the finding (arrows) indicated, an additional echo- A. Aortic valve regurgitation
cardiographic finding you would expect to see in this B. Aortic root effacement
patient would be: C. Left atrial enlargement
A. Prominent LV apical trabeculations D. Left ventricular hypertrophy
B. Interventricular septal flattening E. Basal interventricular septal thickening
C. Systolic anterior motion of mitral valve chordae
D. Severe biatrial enlargement

2D / MM
56% 48%
C 52 0
P Off
HPen

10

15
75mm/s 93bpm
Figure 9-34 

Question 8 A.
Mitral valve prolapse
A patient is referred for echocardiography for a newly B. Dilated cardiomyopathy
diagnosed systolic murmur (Fig. 9-35, A, B). These C. Hypertrophic cardiomyopathy
images are most consistent with: D. Aortic stenosis

Cal20mm Cal10mm
0 0

160 140

A B
Figure 9-35 
Cardiomyopathies, Hypertensive, and Pulmonary Heart Disease  CHAPTER 9 189

Question 9 Question 10
A 62-year-old man presents to the emergency depart- A 66-year-old woman with primary pulmonary
ment following cardiac arrest. The ECG shows tachy- hypertension confirmed by right heart catheteriza-
cardia with frequent ectopic beats and non-specific tion is referred for echocardiography to determine if
ST-T wave changes. Troponin-I is elevated at 2.0 ng/ml. there has been interval improvement in pulmonary
A transthoracic echocardiogram is obtained (Fig. 9-36). pressures since starting medical therapy. On her prior
LV ejection fraction is measured at 65% by the apical study, comment was made that peak tricuspid regur-
biplane method without regional wall motion abnormal- gitant jet was faint and unmeasurable. To evaluate RV
ities. Urgent coronary angiography is negative for critical pressure you recommend:
intracoronary lesions. The most likely ­diagnosis is: A. Doppler tracing pulmonary valve
A. Cardiac tamponade B. Qp/Qs measurement
B. Takotsubo cardiomyopathy C. 2D imaging inferior vena cava
C. Aortic dissection D. Doppler tracing pulmonary branches
D. Pulmonary embolism

Figure 9-36 
190 CHAPTER 9  Cardiomyopathies, Hypertensive, and Pulmonary Heart Disease

ANSWERS

Answer 1: C interrogation, nonparallel alignment with flow will


Decreased systolic motion of the aortic root, as imaged lead to underestimation of peak velocities. Tricuspid
from M-mode in the parasternal long-axis view, is an annular plane systolic excursion (TAPSE) measures
indirect indicator of systolic dysfunction. Care should the distance of longitudinal systolic motion of the
be taken to align the M-mode line perpendicular to RV just below the tricuspid valve annulus, and is typi-
the long axis of the aortic root to maximize systolic cally obtained from M-mode imaging from an apical
motion. Moving the M-line apically to the mitral valve 4-chamber view, taking care to align the transducer
leaflet tips allows visualization of mitral valve leaflet parallel with maximal systolic motion. Global RV
systolic motion relative to the inter-ventricular septum. function is inferred from the motion of the segment
Increased separation between the anterior mitral leaf- interrogated, so TAPSE would not be a valid measure
let and septum at end-diastole (E-point septal separation) of regional RV dysfunction. A TAPSE of greater than
suggests LV chamber enlargement and possible systolic 1.8 cm is consistent with normal RV systolic function.
dysfunction. During systole, the mitral valve is closed. Right ventricular annular diameter should be mea-
The rate of increase in velocity (acceleration slope) of sured from an RV focused apical 4-chamber view.
the mitral regurgitant jet is represented by dP/dt, and The upper reference size for RV basal diameter is 4.2
reflects the rate of rise in LV pressure over time. A dP/ cm, so a diameter of 3.0 cm would fall in the normal
dt less than 1000 mmHg/s is consistent with systolic range. Analogous to dP/dt from the mitral regurgitant
dysfunction. The diameter of maximal aortic valve Doppler jet for LV systolic function, RV dP/dt is not
leaflet opening is a poor indicator of systolic function. routinely measured as it is load dependent and is less
Absolute aortic valve opening diameter is variable from accurate with increasing tricuspid regurgitation sever-
person to person, and decreased aortic valve opening ity (commonly coincident with worsening RV function).
occurs with leaflet abnormalities, such as calcification.
Answer 4: C
Answer 2: B The apical 4-chamber view shows prominent LV api-
It can be challenging to separate the velocity signal cal trabeculations consistent with LV noncompaction,
from the mitral regurgitant jet from the LV outflow sig- a congenital defect of myocardial development with
nal in patients with hypertrophic cardiomyopathy when failure of apical trabeculations to compact and solidify.
dynamic LV outflow obstruction is present. All of the Advanced cases of noncompaction may lead to progres-
listed features can be helpful, but maximum velocity, sive systolic dysfunction. Ebstein anomaly is a congeni-
when both signals are recorded, is most reliable. The tal abnormality where the septal tricuspid valve leaflet
mitral regurgitant velocity must be higher than the LV attachment is apically displaced, and may be attached
outflow velocity because the LV to LA pressure differ- or tethered to the RV wall and interventricular septum
ence is always higher than the LV to aortic pressure dif- with atrialization of the RV chamber. Takotsubo cardio-
ference. However, the signals may overlap because the myopathy, or transient apical ballooning syndrome is an acute
jet direction may be similar, with an anteriorly directed cardiac syndrome. Clinical presentation is chest pain or
mitral regurgitant jet due to systolic anterior motion dyspnea following a significant emotional or physiologi-
of the mitral valve. Examination of flow duration may cal stress. The clinical manifestation is transient akinesis
aid in differentiation given that LV outflow starts after of the apical and mid-LV segments with regional wall
isovolumic contraction, whereas mitral regurgitation motion abnormalities that extend beyond a single epi-
starts at mitral valve closure; however, flow duration cardial coronary arterial distribution despite absence
may be misleading if onset of mitral regurgitation is of obstructive coronary disease. Cardiac amyloid is an
delayed (late-systolic flow) following systolic anterior infiltrative disorder caused by amyloid deposition in
motion of the mitral leaflets. The diastolic Doppler myocardial tissue. It is the most typical type of restric-
mitral inflow signal is typically recorded on both LV tive cardiomyopathy. Echocardiographic features of
outflow and mitral valve inflow tracings, so would not cardiac amyloid include a hyperreflective appearance
definitively differentiate between them. With obstruc- of the myocardium, myocardial hypertrophy due to the
tion, turbulent systolic flow in the LV outflow tract deposition, and evidence for decreased LV compliance.
with aliasing of the color Doppler signal often obscures
clear visualization of the mitral regurgitation jet. Answer 5: D
The tissue Doppler sample (B) demonstrates a higher
Answer 3: A myocardial E wave velocity, implying normal myo-
Pulsed tissue Doppler interrogation of the tricuspid cardial motion and a normal E/E′ ratio (normal LA
annulus assesses basal RV function. A peak systolic pressure). Cardiac donor hearts usually come from
velocity (S′) greater than 10 cm/s is consistent with younger, previously healthy donors. Echocardiography
normal RV systolic function. As with all Doppler is utilized to monitor for posttransplant complications.
Cardiomyopathies, Hypertensive, and Pulmonary Heart Disease  CHAPTER 9 191

The Doppler LV inflow pattern (A) seen in this case may result in hypertrophy regression. LV diastolic dys-
is comparable to a young individual, with a relatively function is common with hypertension, leading to
higher E/A ratio. In young hearts, vigorous contrac- increased left atrial pressure and left atrial enlargement.
tion of a normal ventricle “pulls” blood into the LV, With significant hypertension, prominence of the basal
and the majority of LV inflow occurs early in dias- interventricular septum is common. Thought to be due
tole with a relatively small contribution of LV filling to straightening of the proximal ascending aorta, there
by atrial contraction. Left atrial enlargement was the is anterior tethering of the proximal ventricular septum.
result of suturing the native and donor atria rather In these patients, the LV is often oriented vertically rela-
than increased LA pressure. tive to the standard parasternal long-axis view. Septal
Echocardiographic markers of cardiac transplant thickening in hypertensive heart disease is focal, usually
rejection are usually absent in the early stages. How- limited to the base of the septum; significant hemody-
ever, with progression, myocardial inflammation namic obstruction of LV outflow is rare.
may lead to increased wall thickness (hypertrophy),
evidence of diastolic dysfunction with restrictive Answer 8: C
ventricular filling and, in advanced cases, systolic dys- A newly diagnosed systolic murmur is a com-
function. Coronary vasculopathy would manifest with mon clinical indication for a TTE. M-Mode trac-
regional wall motion abnormalities in the myocardial ings across the aortic valve (A) and mitral valve
distributions affected by the vasculopathy. With dif- (B) for this patient are shown. M-mode tracings
fuse microvascular disease, systolic dysfunction may of the mitral valve in patients with significant
be global in nature. Pericardial constriction following obstruction in the LV outflow tract show early
cardiac transplantation is rare given that the native closure of the aortic valve (arrow) as was seen in
pericardium is not resewn following transplant. this case below (Fig. 9-37, A). M-Mode tracings
across septum and mitral valve show a severely
Answer 6: B
The M-mode tracing is consistent with pulmonary
hypertension taken across the RV outflow tract and
pulmonic valve. During systole, there is mid-systolic Cal20mm
notching (arrows, termed flying W) of the posterior 3.0
pulmonary valve cusp. Other characteristic echo find-
ings in severe pulmonary hypertension include RV
chamber enlargement with flattening of the inter-
ventricular septum, RV hypertrophy, and systolic dys-
function. Pulmonary arterial hypertension also results
Ao
in an asymmetrical Doppler envelope and decreased
time to peak velocity or acceleration time to less than
60 ms. Prominent apical trabeculations are seen in
patients with LV noncompaction. Systolic anterior LA
motion of the mitral valve chordae occurs in patients
with dynamic left ventricular outflow tract obstruc-
tion (hypertrophic cardiomyopathy). Severe biatrial
enlargement is a hallmark feature of restrictive car- 160
A
diomyopathy with restricted ventricular diastolic fill- Cal10mm
ing and high atrial pressures. 0

Answer 7: B
Mild calcific changes of the mitral annulus and aortic
SAM
valve are often present in patients with hypertension. RV
The proximal aorta may be mildly dilated, predomi-
nantly at the coronary sinuses, with relative preservation septum
of the sinotubular junction. Dilation and effacement of LV
the aortic sinotubular junction is more consistent with
an aortopathy, such as in Marfan syndrome. Aortic
valve regurgitation may be present if coincident aortic
annular dilation or leaflet calcification cause faulty leaf-
let coaptation. However, regurgitation is typically only
trace or mild in severity. The LV response to increased 140
afterload from hypertension is an increase in LV mass B
and hypertrophy, but effective treatment of hypertension Figure 9-37 
192 CHAPTER 9  Cardiomyopathies, Hypertensive, and Pulmonary Heart Disease

thickened septum and systolic anterior motion of the Answer 10: A


anterior mitral valve leaflet (arrow) (see Fig. 9-37, B). This patient’s peak tricuspid regurgitant jet was faint
These echocardiographic findings are consistent with and unmeasurable, making estimation of RV systolic
hypertrophic cardiomyopathy. A systolic murmur is pressure by echocardiography limited on repeat stud-
common in hypertrophic cardiomyopathy, due to ies. In the absence of pulmonic stenosis, the RV end-
turbulent flow in the LV outflow tract. diastolic pressure can be estimated from the pulmonic
The murmur of mitral valve prolapse is due to regurgitant jet (Fig. 9-38). The RV diastolic pressure is
mitral regurgitation. M-mode tracings of the mitral calculated as 4V2 where (V) is the peak end-diastolic
valve leaflets in mitral valve prolapse show late-­systolic velocity measured from the pulmonic valve Doppler
posterior buckling into the left atrium. Patients with regurgitant jet. Analagous to pulmonary artery systolic
dilated cardiomyopathy often have significant func- pressure estimates, the pulmonary artery diastolic pres-
tional mitral regurgitation due to dilation of the sure is the sum of the RV diastolic pressure and the RA
valve annulus and resultant poor leaflet coaptation. pressure estimate, which is inferior vena cava diam-
M-mode tracings of these patients show enlargement eter at rest and following inspiration. The diameter
of the LV and increased E point septal separation. In of the inferior vena cava, as assessed by 2D imaging,
calcific aortic stenosis, there is thickening and heavy provides an estimate of right atrial pressure (not RV
calcification of the aortic valve. In advanced cases, pressure, the question in this case). The pulmonic vol-
there is poor systolic opening of the leaflets through- ume flow rate (Qp) measurement is the product of the
out systole. The aortic valve leaflets in this M-mode RV outflow tract Doppler velocity time integral and
tracing appear thin and mobile, not consistent with the cross-sectional area at the RV outflow tract. The
aortic stenosis. systemic volume flow rate (Qs) is the product of the LV
outflow tract Doppler velocity time integral and the
Answer 9: D cross-sectional area at the LV outflow tract. Qp/Qs
This parasternal short-axis view shows interventricu- ratio is a calculation of the comparative volume flow
lar septal flattening, consistent with right ventricu- between the RV and LV to quantitate the severity of
lar enlargement. The echocardiogram and clinical an intracardiac shunt. Antegrade Doppler tracing of
presentation are most consistent with pulmonary the pulmonary branches would provide hemodynamic
embolism with acute RV strain. Although increased data for the pressure gradient at the point of interroga-
cardiac biomarkers suggest acute myocardial injury, tion (i.e., pulmonary artery branch stenosis) and would
LV systolic function is preserved without regional not provide data on RV chamber pressure in systole.
wall motion abnormalities, and the troponin is only
minimally elevated. Coronary angiography is nega- V  2.18m/s
tive, excluding a transmural myocardial infarction. PG  19.0mmHg
The clinical presentation of Takotsubo cardiomy- CW Gain  12dB
opathy is chest pain or dyspnea following a signifi-
cant emotional or physiological stress, manifested as
transient akinesis of the apical and mid-LV segments
with regional wall motion abnormalities that extend
beyond a single epicardial coronary arterial distribu-
tion. Normal LV chamber size and systolic function,
as was seen in this case, is not consistent with Takot- 3.0
subo cardiomyopathy. There is no pericardial effu-
sion seen on the image provided to suggest cardiac
tamponade. The clinical presentation could be con-
cordant with aortic dissection, but the echo image is
not consistent. From the parasternal long-axis view,
the proximal portion of the ascending aorta may be
seen. Complications of aortic dissection could include
LV dysfunction, myocardial infarction, or pericardial m/s
effusion. If the aorta is not well seen, other imaging
modalities, such as cardiac computed tomographic 1.0
or, magnetic resonance imaging, or TEE should be
considered. Figure 9-38 
10 Pericardial Disease
STEP-BY-STEP APPROACH Determine if RA Filling Pressures Are Elevated
Pericardial Effusion Perform Echo-guided Pericardiocentesis if Clinically
Record Blood Pressure and Heart Rate Indicated
Evaluate for the Presence of Pericardial Fluid Pericardial Constriction
Evaluate the Distribution of Pericardial Fluid Look for Evidence of Pericardial Thickening
Estimate the Size of the Pericardial Effusion Evaluate for Anatomic Evidence of Constriction
Tamponade Physiology Perform Doppler Studies to Diagnose Constriction
Look for RA Systolic Collapse Distinguish Constrictive Pericarditis from Restrictive
Evaluate RV Diastolic Collapse Cardiomyopathy
Examine for Reciprocal Respiratory Changes in RV THE ECHO EXAM
and LV Volumes SELF-ASSESSMENT QUESTIONS
Evaluate for Reciprocal Respiratory Changes in RV
and LV Filling Velocities

STEP-BY-STEP APPROACH TABLE 10-1 Differential Diagnosis


of Pericardial Disease
Pericardial Effusion
I. Infections
n  There are numerous causes for accumulation of A. Postviral pericarditis
fluid in the pericardial space (Table 10-1). B. Bacterial
n  A pericardial effusion may be asymptomatic or C. Tuberculosis
may be associated with pericarditis or with tam- D. Parasitic (Echinococcus, amebiasis, toxoplasmosis)
ponade physiology. II. Malignant
n Pericarditis is a clinical diagnosis based on the A. Metastatic disease (e.g., lymphoma, melanoma)
triad of typical pericardial pain, a pericardial rub, B. Direct extension (lung carcinoma, breast
­carcinoma)
and diffuse ST elevation on the electrocardiogram
C. Primary cardiac malignancy
(ECG) (Fig. 10-1). III. “Inflammatory”
n 
Tamponade physiology is present when systemic A. Postmyocardial infarction (Dressler syndrome)
blood pressure or cardiac output is reduced due to B. Uremia
compression of the cardiac chambers by the peri- C. Collagen-vascular disease
cardial fluid. D. Postcardiac surgery
E. Radiation
v  KEY POINTS III. Intracardiac-pericardial communications
In patients with pericarditis, the effusion ranges
o A. Blunt or penetrating chest trauma
from absent to large in size. B. Postcatheter procedures (electrophysiology
studies, percutaneous coronary intervention,
o The presence of a pericardial rub does not cor-
valvuloplasty, endomyocardial biopsy)
relate with the size of the effusion. C. Pacer lead or IV line perforation
o In a patient with a large pericardial effusion and D. Left ventricular rupture postmyocardial infarction
hypotension or a low cardiac output, tampon- E. Aortic dissection
ade physiology likely is present, even if other
echocardiographic signs are not seen.
v  KEY POINTS
Step 1: Record Blood Pressure and Heart Hypotension and tachycardia are nonspecific but
o
Rate are seen in patients with tamponade physiology.
n  The first step in echocardiographic evaluation of o To measure pulsus paradoxus, the blood pres-
a patient with suspected pericardial disease is to sure cuff is deflated until the first Korotkoff
measure and record blood pressure and heart rate sound is intermittently heard during expira-
(as for any echocardiographic examination). tion. The cuff is then slowly deflated until the
n Pulsus paradoxus is a decline in the systolic blood Korotkoff sound is heard on every beat. The
pressure by more than 20 mmHg with inspiration difference between these two pressures is
(Fig. 10-2). the paradoxical pulse.

193
194 CHAPTER 10  Pericardial Disease

I aVR V1 V4

II aVL V2 V5

Figure 10-1  ECG in pericarditis. This III aVF V3 V6


12-lead ECG shows diffuse upsloping ST
elevation and PR segment depression, con-
sistent with pericarditis in this 42-year-old
man with a 2-week history of persistent dull
chest pain and a pericardial rub on physical II
examination.

Pulsus Paradoxus A PE is seen anterior to the descending thoracic


o
aorta, whereas a pleural effusion extends poste-
INSP riorly to the descending aorta (Fig. 10-6)
EXP EXP o F luid adjacent to the RA in the apical
Respiration
20 mm Hg 4-­chamber view may be due to pericardial or
BP pleural fluid. The specific diagnosis is based on
BP

evidence of pericardial or pleural fluid in other


views.
o If the pericardial fluid contains thrombus or
fibrinous debris, the effusion may be echogenic
t
instead of echolucent (Fig. 10-7).
Figure 10-2  Pulsus paradoxus.  This schematic shows that with inspira-
tion, systolic blood pressure falls by at least 20 mmHg when pulsus para- Step 3: Evaluate the Distribution
doxus is present. BP, Blood pressure; EXP, expiration; INSP, inspiration. of Pericardial Fluid
n Effusions may be circumferential or loculated so
o A physician should be present for the echocar- that evaluation in multiple views from parasternal,
diographic study when the patient is hemody- apical, and subcostal windows is essential.
namically compromised (i.e., hypotension or n 
Loculation of fluid due to adhesions often is seen
significant tachycardia). after cardiac surgery or trauma or with malignant
effusions.
Step 2: Evaluate for the Presence
of Pericardial Fluid v  KEY POINTS
n An echo-free space adjacent to the heart is consis- Loculated fluid may be missed unless multiple
o
tent with a pericardial effusion (Fig. 10-3). views are examined; sometimes transesophageal
n The pericardial sac extends completely around echocardiography (TEE) is needed to identify
both the left ventricle (LV) and right ventricle (RV), loculated fluid posterior to the LA (Fig. 10-8).
from the apex to the base, and extends around the o Loculated fluid occasionally may be mistaken
right atrium (RA) to the bases of the superior and for a normal cardiac chamber, for example,
inferior vena cava. when loculated fluid compresses the LA or
n 
The pericardial sac extends posterior to the left RA.
atrium (LA), between the pulmonary vein orifices
(the oblique sinus of the pericardium), and there is Step 4: Estimate the Size of the Pericardial
a small cuff of pericardial space around the base Effusion
of the great vessels (the transverse sinus) (Fig. 10-4). n A small amount of pericardial fluid is nor-

mal, appearing as a trivial or absent effusion on
v  KEY POINTS echocardiography.
o An isolated anterior, relatively echo-free space n The volume of an abnormal pericardial effusion
usually is due to the normal epicardial fat pad. ranges from 50 mL to more than 1 L.
With an effusion, the echo-free space usually is n The size of the effusion is qualitatively graded as
seen both anteriorly and posteriorly (Fig. 10-5) small, moderate, or large.
Pericardial Disease  CHAPTER 10 195

Ao
LV LV

PE
LA PE

DA
DA
A B
Figure 10-3  Pericardial effusion.  An echo-free space consistent with a pericardial effusion is seen posterior to the left ventricle in both the parasternal
long-axis view (A) and in the short-axis view at the midventricular level (B). Ao, Aorta; DA, descending aorta; PE, pericardial effusion.

Ao RV
LV
Ao
LA
LV
PE

DA
LA

Figure 10-4  Pericardial fluid in the oblique sinus of the p­ ericardium.  Figure 10-5  Anterior adipose tissue.  When an anterior echo-free space
A small echo-free space is seen posterior to the LA (arrow) in this paraster- (arrow) is seen, as in this parasternal long-axis view, without evidence for
nal long-axis view. This is clearly a pericardial effusion, not pleural fluid, as posterior effusion, the most likely diagnosis is normal epicardial adipose
it tracks anteriorly to the descending aorta. Ao, Aorta; DA, descending aorta; tissue or a “fat pad.” Ao, Aorta.
PE, pericardial effusion.
196 CHAPTER 10  Pericardial Disease

DA

PE

Figure 10-6  Pleural effusion.  A large left pleural effusion is seen in this
parasternal long-axis view. The pleural effusion extends posterior to the de-
scending thoracic aorta. DA, descending aorta. A

SUPINE

Ao

LV

LA PE

B
Figure 10-8  Loculated pericardial effusion. In an apical 4-chamber
orientation, the sector has been narrowed to focus on the right side of the
heart. The RV is small, with a catheter seen in the chamber. A, The area
Figure 10-7  Pericardial mass. In this parasternal long-axis view, the normally occupied by the RA consists primarily of loculated pericardial ef-
pericardial space (arrow) is filled with echo-dense material, consistent with fusion with the RA free wall (arrow) compressed so that is almost touches
hematoma, tumor, or fibrinous debris. Ao, Aorta. the interatrial septum. B, Color Doppler confirms the severe compression of
the RA with a very narrow flow stream into the RV. PE, Pericardial effusion.

v  KEY POINTS
A small effusion on two-dimensional (2D)
o One useful approach is to consider the effusion
o
imaging can be confirmed by the M-mode small if the distance between the epicardium
finding of flat motion of the parietal pericar- and pericardium is less than 0.5 cm, moderate
dium with systolic separation of the epicar- if 0.5 to 2 cm, and large if more than 2 cm (Fig.
dium (Fig. 10-9). 10-10).
o There is no precise approach to estimation of o With loculated effusions, size is described in a sim-
pericardial fluid volume by echocardiography. ilar fashion along with the location of the fluid.
Pericardial Disease  CHAPTER 10 197

Cal10mm
5

139
A B
Figure 10-9  M-mode tracing of pericardial effusion.  A, A very small pericardial effusion is seen on 2D imaging posterior to the LV (arrow). B, The M-mode
tracing demonstrates the small effusion more clearly with flat motion of the parietal pericardium so that there is a more prominent posterior echo-free space
in systole than diastole (arrow).

BEDSIDE

RV
RV
RA LV
RA

LV
LA
LA

A B
Figure 10-10  Size of pericardial effusion.  The size of a pericardial effusion is graded qualitatively, but measurement of the distance between the epi-
cardium and pericardium is helpful. On a subcostal view, both these patients have circumferential pericardial effusion, with a moderate effusion (A) showing
between 0.5 and 2 cm maximal pericardial separation compared to more than 2 cm with a large effusion (arrows) (B).

o Evaluation from the subcostal view is especially


important because this approach often is used n  amponade physiology occurs when pericardial
T
for pericardiocentesis (Fig. 10-11) pressure exceeds intracardiac pressure.
n With tamponade physiology, cardiac output and
Tamponade Physiology blood pressure are reduced due to impaired car-
n 
Pericardial pressure depends on both the volume diac filling due to compression of the cardiac
and rate of accumulation of pericardial fluid. chambers.
198 CHAPTER 10  Pericardial Disease

LV
RV

RA

LA

Figure 10-11  Subcostal view of pericardial effusion. This approach


often is used for drainage of pericardial fluid. In this patient, the effusion
between the liver and right side of the heart is seen. The normal adipose
tissue at the right atrioventricular groove (arrow) often is well seen when a
pericardial effusion is present. If pericardiocentesis is planned, a transducer Figure 10-12  RA systolic collapse.  The RA free wall is examined frame
position where the effusion is closer to the site of needle entry, with less by frame in the apical 4-chamber plane using zoom mode and a narrow
intervening hepatic tissue, is preferred. This effusion is quite small, so many sector. This end-systolic frame shows persistent systolic compression
clinicians would defer pericardiocentesis. (or ­collapse) of the RA free wall consistent with tamponade physiology.

n  ulsus paradoxus is an excessive fall (>20 mmHg)


P Step 1: Look for RA Systolic Collapse
in systolic blood pressure with inspiration, due to n  When intrapericardial pressure exceeds RA pres-
the fall in cardiac output with inspiration. sure, the RA free wall collapses in systole (Fig.
10-12).
v  KEY POINTS n RA free wall inversion for more than one third of
o Tamponade physiology may occur with a rap- systole is sensitive and specific for the diagnosis of
idly accumulating moderate-sized effusion (e.g., tamponade physiology.
with aortic dissection or trauma) but may not
occur even with very large effusions if the rate v  KEY POINTS
of increase in size was gradual. Brief systolic inversion of the RA free wall may
o
o Thin-walled, low-pressure cardiac chambers be seen without tamponade physiology.
(e.g., the RA) are compressed at lower pericar- o The RA free wall is best evaluated in the apical
dial pressures than thicker-walled chambers and subcostal 4-chamber views.
(e.g., the RV). o Zoom mode provides optimal image resolution;
o Cardiac compression is most evident during the a narrow 2D sector improves frame rate.
phase of the cardiac cycle when the chamber o Frame-by-frame analysis, to determine the
pressure is low (e.g., systole for the atrial cham- number of frames with free wall inversion com-
bers, diastole for the RV). pared with the total frames in systole, improves
o With inspiration, intrathoracic pressure falls, the accuracy of this approach.
resulting in increased filling of the right heart.
If total cardiac volume is fixed (as with tam- Step 2: Evaluate RV Diastolic Collapse
ponade physiology), the increased filling of the n 
When intrapericardial pressure exceeds RV dia-
right heart limits filling of the left heart, result- stolic pressure, the RV free wall collapses in dias-
ing in a lower forward stroke volume and blood tole (Fig. 10-13).
pressure.
o If the patient has a low cardiac output or v  KEY POINTS
hypotension and a large pericardial effusion o Frame-by-frame analysis or use of an M-mode
is present, further echocardiographic evalu- cursor through the RV free wall may be help-
ation is not needed; prompt therapy is more ful for evaluation of the timing of RV free wall
appropriate. motion.
Pericardial Disease  CHAPTER 10 199

A B
Figure 10-13  RV diastolic collapse.  Parasternal short-axis views in mid-diastole in a patient with a large pericardial effusion show RV diastolic collapse at the
midventricular (A) and RV outflow tract (B) levels (arrows). The RV chamber is very small, with a convex indentation of the RV free wall by the pericardial effusion.

RV diastolic collapse may be appreciated in


o
parasternal long- and short-axis and in apical Step 4: Evaluate for Reciprocal Respiratory
and subcostal 4-chamber views. Changes in RV and LV Filling Velocities
o RV diastolic collapse is less sensitive, but more n Analogous to the changes in RV and LV volumes
specific, than brief RA systolic collapse for the with respiration, the volume of inflow across the
diagnosis of pericardial tamponade. atrioventricular valves varies with respiration.
o If the RV free wall is thickened due to hypertro- n 
With inspiration, there is an increase (>25%) in
phy or an infiltrative process, diastolic collapse RV diastolic filling; with expiration, LV diastolic
may not occur even with elevated pericardial filling increases by >25% (Fig. 10-14).
pressures.
o RV diastolic collapse will not occur if RV dia- v  KEY POINTS
stolic pressure exceeds pericardial pressure, as o The phase of respiration is recorded (using a
can happen in severe pulmonary hypertension. respirometer) simultaneously with the ECG and
Doppler velocity data.
Step 3: Examine for Reciprocal Respiratory o A slow sweep speed is used to include more
Changes in RV and LV Volumes than one respiratory cycle on the recording.
n An effusion with tamponade physiology results in a o The Doppler sample volume is positioned,
fixed total cardiac volume. and a 2D image is recorded for several beats
n With a fixed total volume, the increase in right- to ensure that the intercept angle between the
sided filling with inspiration is matched by a recip- Doppler beam and the direction of inflow
rocal decrease in left-sided volumes. does not vary significantly with respiration.
n 
Conversely, with expiration there is a relative If there is a significant variation in the inter-
increase in left, compared with right, heart filling. cept angle, observed differences in velocity
with respiration may be an artifact due to
v  KEY POINTS as­
suming a constant angle in the Doppler
The reciprocal changes in right- and left-filling
o equation.
with respiration are best seen on 2D imaging in o With tamponade physiology, RV diastolic filling
a 4-chamber view. increases and LV diastolic filling decreases on
o With inspiration, the ventricular septum shifts the first beat after inspiration.
to the left, followed by a shift toward the right o Evaluation of filling dynamics is challenging, so
with expiration. that an apparent lack of respiratory variation
o An M-mode tracing of the septum from the does not exclude the possibility of tamponade
parasternal window also may be helpful. physiology.
200 CHAPTER 10  Pericardial Disease

1.5
1.0 TV MV

m/s
m/s

A B
Figure 10-14  Inflow velocities with tamponade.  In this patient with a large pericardial effusion, ventricular inflow velocities across the tricuspid valve
(TV) and mitral valve (MV) were recorded at a slow sweep speed simultaneously with a respirometer tracing. The cyan-colored respirometer tracing indicates
inspiration as an upward deflection and expiration as a downward deflection. The TV tracing shows that the inflow velocity increases with inspiration with a peak
velocity of only 0.29 m/s in expiration and 0.75 m/s with inspiration (arrow). There are reciprocal changes in transmitral flow, with the peak velocity decreasing
from 1.2 m/s during expiration to 0.9 m/s on the first beat after inspiration (arrow).

Step 5: Determine if RA Filling Pressures Are


Elevated
n Elevated RA filling pressures are a sensitive, but
not specific, sign of cardiac tamponade.
n 
Echocardiographic evaluation of RA filling
HV
pressure is based on the size and respiratory
variation of the inferior vena cava; a dilated
inferior vena cava without respiratory variation RA
IVC
and with dilated hepatic veins is called plethora
of the inferior vena cava (Fig. 10-15).
v  KEY POINTS
Images of the inferior vena cava are obtained
o
from the subcostal view in spontaneously
breathing patients.
o This method is not applicable in patients on
positive pressure mechanical ventilation.
o There are many other causes for elevated RA
pressures, other than tamponade physiology, so
this finding is interpreted in the context of the
other imaging findings. Figure 10-15  Inferior vena caval (IVC) dilation.  Subcostal view showing a
o Tamponade may be present without plethora of dilated inferior vena cava and hepatic vein in a patient with pericardial tampon-
ade. With inspiration, there was no change in IVC diameter. HV, Hepatic vein.
the inferior vena cava if the patient is hypovolemic.
Step 6: Perform Echo-guided Visualization of the tip of the needle is problem-
o
Pericardiocentesis if Clinically Indicated atic because any segment of the needle passing
n Echocardiography may be used to guide the per- through the 2D image plane may be mistaken
cutaneous pericardiocentesis procedure either to for the tip; therefore three-dimensional (3D)
define the best approach to percutaneous drainage imaging may be helpful.
or to confirm the needle position in the pericardial o The position of the needle is confirmed by
space (Fig. 10-16). injection of a small amount of agitated saline
n 
Echocardiography is used after pericardiocen- to produce a contrast effect.
tesis to assess the amount of residual fluid (Fig.
10-17). Pericardial Constriction
n Pericardial constriction is the result of pericardial
v  KEY POINTS thickening and fibrosis with fusion of the parietal
o Evaluation from parasternal, apical, and sub- and visceral pericardium.
costal approaches demonstrates the depth and n 
T he thickened and rigid pericardium constricts
amount of pericardial fluid relative to the posi- the cardiac chambers, resulting in a limited total
tion of the transducer. cardiac volume and a reduced cardiac output.
Pericardial Disease  CHAPTER 10 201

PE PRE

LV
RV
RV LV

LA
RA

A
Figure 10-16  Large pericardial effusion.  Apical 4-chamber view show-
ing a large circumferential pericardial effusion, with relatively more fluid
posterior and lateral to the left ventricle, as is typical on echocardiography. POST
Fluid is seen adjacent to the right atrium and a small amount of fluid is
seen superior to the left atrium, in the oblique sinus of the pericardium that
extends between the right and left pulmonary veins. PE, Pericardial effusion.

v  KEY POINTS RV
LV

oCommon causes of pericardial constriction
include prior cardiac surgery or trauma, radia-
tion therapy, and recurrent pericarditis.
RA LA
o Like tamponade physiology, the fixed total car-
diac volume with pericardial constriction results
in reciprocal changes in right- and left heart fill-
ing (Fig. 10-18).
o Typically there is no significant pericardial effu-
sion when constrictive pericarditis is present,
although there are rare cases of effusive con-
strictive physiology.
o Clinically the differentiation of constrictive
pericarditis (which is treated by pericardiotomy) B
and restrictive cardiomyopathy (which is treated
Figure 10-17  Echocardiographic monitoring of pericardiocentesis in
medically) is problematic. the cardiac catheterization lab.  At baseline (PRE), a foreshortened apical
view shows a large pericardial effusion with RA compression. After drainage
Step 1: Look for Evidence of Pericardial of 1 L of fluid (POST), repeat imaging shows a much smaller effusion at the
Thickening apex, although some fluid persists adjacent to the RV and RA. Image quality
n Pericardial thickening may be evident on 2D echo-
is suboptimal because the patient is supine, and removal of the fluid resulted
in poorer acoustic access.
cardiography as areas of increased echogenicity in
the pericardial region (Fig. 10-19).
n 
On M-mode tracings, pericardial thickening is
evident as multiple dense parallel lines posterior
to the LV endocardium that persist even with low tomographic (CT) or magnetic resonance imag-
gain settings (Fig. 10-20). ing is preferred when measurement of pericar-
dial thickness is needed.
v  KEY POINTS o Pericardial thickening may be asymmetric so
o Echocardiography is not sensitive for detection that a complete evaluation includes evaluation
of pericardial thickening; cardiac computed from parasternal, apical, and subcostal windows.
202 CHAPTER 10  Pericardial Disease

Cal10mm
36

RV
LV

RV septum

LV
RA LA

EXP INSP PW

Figure 10-18  Pericardial constriction.  Schematic diagram showing an pericardium


apical 4-chamber view at end-expiration (EXP) and during inspiration (INSP)
with pericardial constriction. The increase in RV filling with inspiration results
in a compensatory decrease in LV size, as the total cardiac volume is con- 168
strained by the thickened and adherent pericardium. This results in “septal
shift” with inspiration. Figure 10-20  M-mode of pericardial thickening. A band of multiple
echodensities moving together posterior to the left ventricle on M-mode is
consistent with a thick posterior pericardium that moves with the epicardium
during the cardiac cycle. PW, Posterior wall.

v  KEY POINTS
There are no specific 2D imaging findings in
o
patients with constrictive pericarditis.
o The diagnosis of constrictive pericarditis should
be considered in the appropriate clinical setting
(cardiac symptoms in a patient at risk of con-
LV
strictive disease) if the echocardiogram does not
RV show other causes for the patient’s symptoms.
o Constrictive pericarditis most often is diagnosed
in patients with unremarkable echocardio-
RA LA
graphic images.
Step 3: Perform Doppler Studies
to Diagnose Constriction
n  Reciprocal respiratory changes in RV and LV
diastolic filling, in the absence of a pericardial
effusion, suggest the diagnosis of constrictive peri-
carditis (Fig. 10-21).
Figure 10-19  Pericardial thickening. This 4-chamber view shows n Typically, pulmonary pressures are normal in
marked thickening of the pericardium lateral to the LV (arrows) in a patient patients with constrictive pericarditis but ele-
later diagnosed with constrictive pericarditis. Pericardial thickening is differ- vated (>60 mmHg) in those with restrictive
entiated from effusion by the echogenicity of the pericardial space.
cardiomyopathy.
Tissue Doppler E′ velocity is increased with con-
n 
strictive pericarditis compared to a decreased E′
Step 2: Evaluate for Anatomic Evidence velocity (< 8 cm/s) with restrictive cardiomyopathy.
of Constriction
n Typical findings in patients with pericardial con- v  KEY POINTS
striction are enlarged atria (due to chronically With pericardial constriction, the normal myo-
o
elevated filling pressures) and small ventricles with cardium allows rapid early diastolic filling of the
normal systolic function. chamber with normal relaxation and compli-
n 
M-mode findings in constrictive pericarditis ance. Once the chamber has reached the limit
include reduced posterior motion of the LV pos- of total cardiac volume imposed by the rigid
terior wall endocardium in diastole (<2 mm) and pericardium, ventricular filling abruptly halts.
a brief rapid posterior motion of the ventricular o The ventricular inflow pattern shows a
septum in early diastole. prominent early (E) filling velocity, a normal
Pericardial Disease  CHAPTER 10 203

HR120bpm HR124bpm
TV Sweep50mm/s MV Sweep50mm/s

1.0
1.0

Insp m/s
m/s Insp

Figure 10-21  Inflow velocities with constriction.  A marked (>25%) increase in RV filling is seen across the tricuspid valve (TV) during inspiration (Insp; arrow )
with a reciprocal respiratory decrease (>25%) in LV diastolic filling across the mitral valve (MV).

TABLE 10-2 Comparison of Pericardial Tamponade, Constriction, and Restrictive


Cardiomyopathy
Restrictive
Pericardial Tamponade Constrictive Pericarditis Cardiomyopathy
Hemodynamics
RA pressure ↑ ↑ ↑
RV/LV filling ↑, RV = LV ↑, RV = LV ↑, LV > RV
pressures
Pulmonary artery Normal Mild elevation (35-40 mmHg Moderate-severe elevation
pressures systolic) (≥ 60 mmHg systolic)
RV diastolic > 1/3 peak RV pressure > 1/3 peak RV pressure
­pressure plateau
Diastolic filling Rapid early filling, impaired late Impaired early filling
filling
2D echo Moderate-large PE Pericardial thickening without LV hypertrophy
Inferior vena cava plethora ­effusion Normal systolic function
Doppler echo Reciprocal ­respiratory E > A on LV inflow (1) Early in disease E < A
changes in RV and LV Prominent y descent in hepatic vein on LV inflow
­filling Pulmonary venous flow = prominent (2) Late in disease E > A
a wave, reduced systolic phase (3) Constant IVRT
Respiratory variation in IVRT and in (4) Absence of significant
E velocity respiratory variation
Tissue Doppler ↓ E′ without respiratory ↑ E′ E′ < 8 cm/s with S′< 8
­variation cm/s
Other diagnostic Therapeutic/diagnostic CT or CMR for pericardial thickening Endomyocardial biopsy
tests ­pericardiocentesis

CT, Computed tomography; CMR, cardiac magnetic resonance imaging, IVRT, isovolumic relaxation time; PE, pericardial effusion.

deceleration slope, and a very small atrial con- n  owever, the possibility of these diagnoses
H
tribution to filling (due to elevated end-diastolic often is first suggested by the echocardiographic
LV pressures). findings.
o The RA inflow (hepatic vein) pattern shows a
prominent atrial reversal, with prominent dia- v  KEY POINTS
stolic (and blunted systolic) ventricular filling. RA and LA filling pressures are increased in
o
o Pulmonary pressures are estimated based on both conditions.
the velocity in the tricuspid regurgitant jet and o RV and LV diastolic pressures are equal, even
an estimate of RA pressure. after volume loading, when constrictive pericar-
ditis is present.
Step 4: Distinguish Constrictive Pericarditis o Pulmonary systolic pressure typically
from Restrictive Cardiomyopathy is severely elevated with a restrictive
n 
Echocardiography alone often is inadequate to cardiomyopathy.
distinguish constrictive pericarditis from restrictive o Severe biatrial enlargement is typical with
cardiomyopathy (Table 10-2). restrictive cardiomyopathy.
204 CHAPTER 10  Pericardial Disease

Early diastolic filling is rapid with constrictive


o The LV myocardium is normal with constric-
o
pericarditis and is reduced with restrictive car- tive pericarditis. With restrictive cardiomyopa-
diomyopathy early in the disease course. thy, LV wall thickness often is increased.
o However, with advanced restrictive physiology, o Additional helpful studies are CT and/or car-
ventricular compliance is reduced so that early diac magnetic resonance imaging direct imaging
diastolic filling may appear similar to the pat- of pericardial thickness, cardiac catheterization
tern seen with constrictive pericarditis. for simultaneous measurement of LV and RV
diastolic pressures, and endomyocardial biopsy.
Pericardial Disease  CHAPTER 10 205

THE ECHO EXAM

Pericardial Disease
Pericardial Effusion Constrictive Pericarditis
Views Imaging
Parasternal Pericardial thickening
Apical Normal LV size and systolic function
Subcostal LA enlargement
Distinguish from pleural fluid Flattened diastolic wall motion
Size Abrupt posterior motion of the ventricular septum in
Small (< 0.5 cm) early diastole
Moderate (0.5-2.0 cm) Dilated inferior vena cava and hepatic veins
Large (> 2.0 cm) Septal shifting with inspiration
Diffuse versus loculated
Evaluate for tamponade physiology if moderate or large Doppler
TEE if needed, especially in post-op patients Prominent Y descent on hepatic vein or superior vena
cava flow pattern
LV inflow shows prominent E velocity with a rapid early
diastolic deceleration slope and a small or absent A
Pericardial Tamponade velocity
Increase in LV-IVRT by >20% on first beat after
Clinical Findings
­inspiration
Low cardiac output Respiratory variations in RV/LV diastolic filling (dif-
Elevated venous pressures ference >25%) with inspiratory ↑RV ↓LV filling with
Pulsus paradoxus inspiration
Hypotension Tissue Doppler ↑ E′ > 8 cm/s
2D-Echo Pulmonary venous flow shows prominent a wave and
Moderate-large pericardial effusion blunting of systolic phase
RA systolic collapse (duration greater than a third of IVRT, Isovolumetric relaxation time.
systole)
RV diastolic collapse
Reciprocal respiratory changes in RV and LV volumes LV Pseudoaneurysm
Inferior vena cava plethora
Abrupt transition from normal myocardium to aneurysm
Doppler Acute angle between myocardium and aneurysm
Respiratory variation in RV and LV diastolic filling Narrow neck
Increased RV filling on first beat after inspiration Ratio of neck diameter to aneurysm diameter < 0.5
Decreased LV filling on first beat after inspiration May be lined with thrombus
206 CHAPTER 10  Pericardial Disease

SELF-ASSESSMENT QUESTIONS

Question 1 Question 3
Identify the numbered structures in Figure 10-22: Echocardiography is requested on a patient in the
1. ____________________ emergency department who presents with chest pres-
2. ____________________ sure and hypotension. The findings in Figure 10-24
3. ___________________ are most consistent with:
4. _____________________ A. Aortic dissection
5. _____________________ B. Acute myocardial infarction
6. _____________________ C. Mitral valve chordal rupture
D. Pericardial tamponade
E. Pulmonary embolism

Cal20mm
1 0

3 4

160

Figure 10-24 
Figure 10-22 
Question 4
Question 2 A 72-year-old woman presents with a 6-month history
Identify the numbered structures in Figure 10-23: of progressive pedal edema and exertional dyspnea. Her
1. ________________________ medical history includes coronary disease and hyperten-
2. ________________________ sion. The image (Fig. 10-25) is most consistent with:
3. ________________________ A. Cardiac tamponade
4. ________________________ B. Primary pulmonary hypertension
C. Pericardial constriction
D. Dilated cardiomyopathy
2D
59% Cal10mm PSAX
2 26
C 50
P Low
HGen
1

181
Figure 10-23 
Figure 10-25 
Pericardial Disease  CHAPTER 10 207

Question 5 Question 7
A 55-year-old woman presents with a 3-month history The Doppler flow data as shown in Figure 10-27 were
of progressive pedal edema and exertional dyspnea. obtained in patient with dyspnea. These tracings are
She has no significant past medical history. Data from most consistent with:
her transthoracic echocardiogram are as follows: A. Normal respiratory variation
B. Pericardial constriction
LV end-diastolic volume 100 mL C. Restrictive cardiomyopathy
LV posterior diastolic wall thickness 1.2 cm D. Chronic obstructive pulmonary disease
LV ejection fraction 59%
LA indexed volume 45 mL/m2
Mitral valve E wave velocity 1.7 m/s
Tissue Doppler E′ velocity 0.05 m/s FR 50Hz PW
Inferior vena cava diameter 2.0 cm 15cm 50%
2D 1.6MHz
Tricuspid regurgitant jet velocity 3.6 m/s 64% WF 150Hz
C 50 SV4.0mm
P Low 7.1cm
You conclude that the data are most consistent with: HGen
A. Pericardial constriction 200
B. Dilated cardiomyopathy
C. Restrictive cardiomyopathy 160

D. Chronic obstructive pulmonary disease 120

Question 6 80

A 54-year-old woman with acute myeloid leukemia 40

presents with dyspnea. Based on the image in Figure cm/s


10-26, the next best step in patient management is: 50mm/s 106bpm

A. Pericardiocentesis FR 50Hz PW
B. Thoracentesis 12cm 75%
1.6MHz
C. Ligation of the persistent left superior vena cava 2D
62% WF 125Hz
D. Pericardial stripping C 50
P Low
SV4.0mm
6.3cm
HGen

120
100

80
60

40

20

cm/s

50mm/s 110bpm

Figure 10-27 

Figure 10-26 
208 CHAPTER 10  Pericardial Disease

Question 8 Question 9
A 55-year-old woman becomes hypotensive during A 58-year-old man with amyloidosis is undergoing
a percutaneous coronary intervention and an urgent evaluation for stem cell transplantation. TTE is per-
echocardiogram is obtained (Fig. 10-28); the most formed (as shown in Fig. 10-29). Which of the fol-
likely diagnosis is: lowing clinical conditions would most hinder further
A. Acute myocardial infarction echocardiographic evaluation?



B. Cardiac tamponade A. Pulmonary hypertension






C. Hypovolemia B. Pleural effusion

/






D. Papillary muscle rupture C. Cardiac amyloidosis






9
E. Pericardial hematoma D. Abdominal ascites






9
2D

r
64%

i
C 50
P Low

h
HPen

ta
er/
s
/r u
.t c
Figure 10-29

Figure 10-28

a

FR 101Hz PW
22cm Vel 116 cm/s
PG 5 mmHg 65%

k
2D Vel 66.6 cm/s 1.6MHz
67% PG 2 mmHg WF 275Hz

/: /
Question 10 C 48
P Off
SV3.0mm
12.2cm
HPen
The following pulsed Doppler tracings were recorded
180
(Fig. 10-30). The upper portion of the figure is the

s
Doppler flow across the mitral valve, and the bottom 120
portion of the figure is Doppler flow across the tricuspid

tt p
valve. The most likely diagnosis for this patient is: 60
A. Chronic obstructive pulmonary disease



B. Normal respiratory variation cm/s



C. Pericardial tamponade

h



D. Positive pressure ventilation 25mm/s
60
102bpm



FR 46Hz PW
22cm 65%
2D 1.6MHz
67% WF 225Hz
C 48 SV3.0mm
P Off 11.5cm
HPen
160

120

80

40

cm/s

40
25mm/s 104bpm

Figure 10-30

Pericardial Disease  CHAPTER 10 209

Question 11 Of the following options, what additional finding


A 78-year-old man presents with exertional dyspnea would most likely be present?
6 years after coronary artery bypass grafting. Coro- A. IVRT respiratory variation




nary angiography documents occlusion of one of his B. Indexed LA volume 43 cm3




four bypass grafts. A TTE (Fig. 10-31) is ordered and C. LV dP/dt 800 mmHg/s




compared with a study he had done 5 years earlier. D. Inferior vena cava diameter 1.5 cm




5 years ago Now

9/
.80 .80

i r 9
a h
m/s
m/s

r/ t
e
.20

s
.40

/r u
.80 1.0

.t c
a
m/s
m/s

/: / k
.20
.20

.10

s
.10

tt p
m/s m/s

h .20
.15

Figure 10-31

210 CHAPTER 10  Pericardial Disease

ANSWERS

Answer 1 procedures. This patient is not in cardiac tamponade;


there is no significant pericardial fluid between the
This is a parasternal long-axis view in a patient with
right ventricular free wall and the transducer and fluid
a large pericardial effusion (1) which is also seen pos-
is not seen posterior to the heart. In primary pulmo-
terior to the LV. The RV outflow tract (2) is small and
nary hypertension, the right ventricle is enlarged with
compressed due to tamponade physiology. Left ven-
flattened septal motion in systole and diastole due to

/
tricular (3) size is relatively normal but the left atrium
RV pressure overload. With dilated cardiomyopathy
(5) appears small and underfilled compared to the

9
the LV is enlarged with decreased systolic endocardial
normal sized aorta (4). The position of the effusion
motion of the septum and posterior wall. In this case,
anterior to the descending thoracic aorta (6) confirms

9
the LV at end-diastole is normal at about 5 cm and
that the fluid is pericardial, not pleural.

r
the end-systolic dimension is 3.5 cm for a fractional

i
Answer 2 shortening of 30%, which is normal.

h
The is an apical view of the left ventricle (1) with Answer 5: C
the image plane angulated posterior to the stan-

a
dard 4-chamber view and then rotated to show the This is a patient with restrictive cardiomyopathy.

t
descending thoracic aorta (4) in long axis. Only a Patients with restrictive cardiomyopathy have rela-

r/
small segment of the right ventricle (2) is seen but a tively normal systolic function with significant dia-
pericardial effusion (3) is present. stolic dysfunction, often in the setting of increased LV
wall thickness. This study shows decreased ventricular

e
Answer 3: D compliance and severely elevated LV filling pressure
as reflected in the elevated E wave velocity of 1.7 m/s

s
This M-mode tracing from a parasternal window
shows a pericardial effusion which is larger anteri- and a severely elevated E/E′ of 34. The elevated LV

/r u
orly than posteriorly. The right ventricular free wall filling pressure is also reflected in the severely elevated
motion is abnormal with a smaller RV dimension in indexed LA volume (normal < 30 mL/m2) and pulmo-
diastole than in systole, consistent with RV diastolic nary hypertension, with only mildly increased central
collapse and tamponade physiology. The ventricular venous pressure. LV chamber size (indexed LV vol-

.t c
septal motion is relatively normal but shows changes ume) is normal, with preserved systolic function and
with respiration. The left ventricular chamber is small mild hypertrophy of the chamber walls, also charac-
with normal wall thickening and endocardial motion. teristic of restrictive cardiomyopathy. For pericardial
constriction, elevation in RV filling pressure is more

a
The mitral valve opens in late diastole.
Aortic dissection cannot be diagnosed from this pronounced than the increase in LV filling pressure,

k
tracing as the aorta is not seen at this level. There is evidenced by dilation of the inferior vena cava, which

/: /
no evidence of abnormal LV wall thickness or motion is not seen in this case. Also, in constriction, myocardial
to suggest myocardial infarction and the mitral valve tissue Doppler typically shows a tissue Doppler E′ > 8
motion does not show prolapse or chordal rupture. cm/s and a S′ > 8 cm/s with only mildly increased pul-

s
Pulmonary embolism might result in elevated pulmo- monary pressures. This patient does not have a dilated
nary pressures leading to RV dilation and systolic dys- cardiomyopathy; LV volume would be increased, with

tt p
function, not a small RV as seen here. a decreased ejection fraction. Chronic obstructive pul-
monary disease is not associated with LV diastolic dys-
Answer 4: C function, and the severity of pulmonary hypertension
This is an M-mode tracing taken from the parasternal is greater than expected for this diagnosis.

h
long-axis view. The RV is closer to the transducer and
the interventricular septum is seen between the ventri- Answer 6: B
cles. Superimposed on the M-mode tracing is a tracing This patient has a large left-sided pleural effusion.
from a respirometer in green. During inspiration, the The image presented is an apical long-axis view of the
respirometer shows an upward deflection. Coincident heart. The LV is closest to the transducer. Posterior to
with inspiration, there is septal shifting with a tran- the heart, there is an echolucent space consistent with
sient increase in RV size and a concordant decrease fluid. The circular structure is the descending thoracic
in LV size. The opposite occurs during expiration. aorta seen in cross section. Tracking posterior to the
Respiratory-dependent septal shifting is consistent descending thoracic aorta is a large pleural effusion. A
with a fixed external obstruction to ventricular filling, pleural and pericardial effusion can be differentiated by
as occurs in pericardial constriction. Pericardial con- the tissue planes that bound the fluid collection. Fluid
striction is typically a consequence of cyclic or chronic that tracks anterior to the descending aorta is pericar-
pericardial inflammation and is most commonly seen dial. In this example, there is a trivial pericardial stripe
in inflammatory conditions or postcardiac surgical seen just along the epicardial border, which is normal
Pericardial Disease  CHAPTER 10 211

thickness. The descending aorta might be mistaken for clinical diagnosis based on evidence of hemodynamic
a dilated coronary sinus, as seen in patients with a per- compromise including tachycardia, hypotension, and
sistent left superior vena cava. However, the coronary a pulsus paradoxus. In addition, echo-Doppler find-
sinus is not well seen on this image; it typically is closer ings suggesting tamponade physiology can be helpful
to the atrioventricular groove and slightly superior to in clinical decision making. Echocardiographic find-
the descending aorta in this view. A persistent left supe- ings of hemodynamic significance include respira-
rior vena cava is a normal variant and does not cause tory-dependent variation in ventricular inflow; with
symptoms or require intervention. Pericardial strip- an increase in RV inflow on inspiration and a recipro-

/
ping refers to surgical removal of a thickened pericar- cal decrease in LV inflow with inspiration. As a conse-
dial when pericardial constriction is present. quence, there are also concordant reciprocal changes

9
in ventricular volumes with the respiratory cycle. Sig-
Answer 7: A nificant pulmonary hypertension increases intracavi-

9
This is a pulsed Doppler sample taken across the tary RV pressure, preventing RV compression, and

i r
mitral valve in a patient with a moderate to large peri- thus may obscure classic echocardiographic features
cardial effusion as seen in the small image showing the of tamponade. Cardiac amyloidosis is associated

h
sample volume position. The green line shows inspi- with restrictive LV filling on the transmitral Doppler
ration (up) and expiration (down). With inspiration, tracing, with a high E/A ratio and low tissue myo-

a
negative intrathoracic pressure increases and there is cardial velocity. However, amyloidosis does not cause

t
an increase in RV inflow (tricuspid E wave velocity) respiratory-dependent variation in RV and LV filling

r/
up to 25% variation. With pericardial tamponade or and septal shifting is not present. Extracardiac fluid
constriction, reciprocal respiratory variation in ven- collections such as pleural effusion or abdominal asci-
tricular inflow is greater than 25% between the first tes do not exert external circumferential pressure on

e
beat after inspiration and the first beat after expi- the heart and therefore do not generate echocardio-

s
ration. With exaggerated respiratory effort, as can graphic findings consistent with cardiac tamponade.
occur with chronic obstructive pulmonary disease,
Answer 10: C

/r u
respiratory variation in inflow to the thinner-walled
RV is commonly seen, but without external cardiac These tracings shows mitral (top) and tricuspid (bot-
constraint, reciprocal changes in LV filling are not tom) inflow velocities with the green line indicating
seen. In restrictive cardiomyopathy, there is no exter- inspiration (up) and expiration (down). These find-

.t c
nal constraint on the heart and, although diastolic LV ings are consistent with pericardial tamponade with
function is abnormal, reciprocal respiratory changes a significant (>25%) decrease in the mitral E wave
in ventricular filling are not seen. velocity on first beat after inspiration. The tricus-

a
pid velocities also show an exaggerated change with
Answer 8: E respiration, with decreased LV filling during inspi-

k
This subcostal 4-chamber view shows a large hematoma ration and increased LV filling during expiration.

/: /
in the pericardial space, between the liver and right The mechanism for these changes is that negative
ventricle. This acute, echodense pericardial hematoma intrathoracic pressure with normal inspiration allows
is the most likely cause of hypotension due to decreased increased RV inflow. Because total heart volume is

s
cardiac output, due to compression of the right heart. limited due to compression by the pericardial fluid,
An acute myocardial infarction due to stent thrombosis the increase in RV size results in a decrease in LV

tt p
results in a regional wall motion abnormality of the LV. size and a reduction in LV filling with inspiration.
Cardiac tamponade presents with an echolucent peri- The opposite changes occur during expiration, and
cardial effusion and signs of tamponade physiology. these changes exceed the normal degree of variation
Hypovolemia is diagnosed on echocardiography when in RV and LV inflow with respiration. The changes

h
the LV is small and underfilled. Papillary muscle rup- in LV filling are then reflected in forward cardiac
ture results in acute severe mitral regurgitation. output across the aortic valve (Fig. 10-32) resulting in
the physical examination finding of pulsus paradoxus
Answer 9: A with a 20 mmHg or greater decline in systolic blood
This subcostal 4-chamber view shows a large peri- pressure with inspiration.
cardial effusion. Both RV and LV chamber size are With exaggerated respiratory effort, as can occur
small and there is a large echolucent space anterior with chronic obstructive pulmonary disease, respiratory
and posterior to the heart. The posterior fluid is seen variation in inflow to the thinner-walled RV is com-
between the posterior LV wall and the pericardium monly seen, but without external cardiac constraint,
(seen in the very far field). Symptoms of a pericardial reciprocal changes in LV filling are not seen. In myo-
effusion typically include chest discomfort and dys- cardial restriction, there is no external constraint on the
pnea, but a slowly accumulating effusion may also be heart and, although diastolic LV function is abnormal,
asymptomatic, as in this example. In a patient with reciprocal respiratory changes in ventricular filling are
a large pericardial effusion, cardiac tamponade is a not seen. With positive pressure ventilation, the normal
212 CHAPTER 10  Pericardial Disease

pattern of negative intrathoracic pressure is disrupted. is increased in this patient compared with his base-
Thus, in patients on positive pressure ventilation, line study. On the pulmonary venous tracing (mid-
changes in inferior vena cava size are not reliable for dle), higher LV filling pressure leads to blunting of
estimated right atrial pressure. Similarly, the expected the systolic component of LA filling compared with
respiratory changes in LV and RV filling patterns may baseline. However, because myocardial function is
not be present even when tamponade is present. normal, the E/E′ ratio remains in the normal range,
with a baseline E/E′ of 0.65/1.1 = 6 and a follow-
Answer 11: A up E/E′ of 0.65/0.08 = 8, both of which are in the

/
This patient has Doppler evidence of pericardial normal range and within measurement variability of
constriction, likely a consequence of his prior bypass each other. In constriction, the thickened pericardium

9
grafting surgery. In addition to the findings shown, encases the entire heart, and biventricular size is nor-
respiratory variation in LV/RV inflow and the IVRT mal or only mildly increased; an indexed LA volume

9
would be present. In pericardial constriction, myocar- of 43 cm3 is severely increased and would be more

i r
dial function is normal, with normal LV relaxation typical of restrictive cardiomyopathy. The LV dP/
and ventricular compliance, but diastolic filling of the dt is normal (>1000 mmHg/s) with pericardial con-

h
ventricle is constrained externally by the rigid peri- striction because LV systolic function is normal. Also,
cardium. The early component of diastolic filling, in constriction, return of blood is restricted, with

a
E wave, is normal, but the late atrial contribution, severely increased central venous pressure, and the

t
A wave velocity, is minimal because of elevated LV inferior vena cava would be dilated and plethoric, not

r/
end-diastolic pressure. Therefore, the E/A ratio (top) normal caliber.

FR 17Hz
19cm

s e PW
60%

/r u
2D 1.6MHz
78% WF 75Hz
C 50 SV4.0mm
P Low 10.4cm

.t c
HGen BEDSIDE
CF
62% 20
2.5MHz

a
WF Low cm/s
Med

k
20

/: /
40

60

s
80

tt p
100
25mm/s 66bpm

h
Figure 10-32

11 Valvular Stenosis
AORTIC STENOSIS Evaluate Mitral Valve Morphology
Step-by-Step Approach Evaluate the Severity of Mitral Stenosis
Determine the Etiology of Stenosis Evaluate Mitral Regurgitation

/
Evaluate Stenosis Severity Examine Aortic and Tricuspid Valves for Rheumatic
Evaluate Aortic Regurgitation and Ascending Aorta Involvement

9
Evaluate the Consequences of Chronic LV Pressure Evaluate the Consequences of Mitral Valve
Overload Obstruction

9
Additional Evaluation for Low-output Low-gradient THE ECHO EXAM

r
Aortic Stenosis SELF-ASSESSMENT QUESTIONS

i
MITRAL STENOSIS
Step-by-Step Approach

a h
AORTIC STENOSIS v

r/
KEY POINTS
t
e

Step-by-Step Approach o Calcific changes usually start in the central part

s


of the leaflets, resulting in a three-pointed star-
Step 1: Determine the Etiology of Stenosis shaped orifice.
(Fig. 11-1)

/r u
o Rheumatic aortic valve disease affects the


n Parasternal two-dimensional (2D) images of the commissures and leaflet edges, with a triangu-



valve in long- and short-axis views lar-shaped orifice, and is accompanied by rheu-
n Number of leaflets, mobility, thickness, and matic mitral valve changes.

.t c



calcification o A bicuspid valve may appear trileaflet in dias-


n Level of obstruction: valvular, subvalvular, or tole due to a raphe in one leaflet; the number



supravalvular of leaflets must be visualized when the valve

a
n Three-dimensional (3D) transesophageal echocar- is open in systole, taking care to identify each



diography (TEE) imaging if transthoracic images commissure, and the points where the leaflets

k
nondiagnostic and further information needed attach to the aortic wall.

/: /
(Fig. 11-2)

s
tt p
h LV
Ao

LV
Ao

LA
LA

A B
Figure 11-1 Etiology of aortic valve stenosis. A, With calcific valve disease, there is increased echogenicity of the leaflets, due to calcification and thicken-


ing, with reduced systolic opening of the leaflets. B, In a patient with a congenitally bicuspid, noncalcified valve, the long-axis view shows thin leaflets with
reduced systolic opening due to doming of the leaflets in systole (arrow), as seen by the curve at the tips of the leaflets. Ao, Aorta.

213
214 CHAPTER 11  Valvular Stenosis

TABLE 11-1 Other High-Velocity Systolic

 
Jets that May Be Mistaken for
Aortic Stenosis
Subaortic obstruction (fixed or dynamic)
Mitral reguargitation
Tricuspid regurgitation
Ventricular septal defect
Pulmonic or branch pulmonary artery stenosis

/
Peripheral vascular stenosis (e.g., subclavian artery)

9
From Otto CM: Textbook of Clinical Echocardiography, ed 5. Philadel­
phia, Elsevier, 2013.

i r 9
h
v KEY POINTS


a
Figure 11-2 3D TEE imaging of a bicuspid aortic valve. A 3D view of
Use multiple acoustic windows (apical, supra-


o

t
the aortic valve from the aortic side of the valve clearly shows two leaflets,


with no raphe, and two commissures in systole. sternal, right parasternal) with careful patient

r/
positioning and transducer angulation to avoid
underestimation of velocity.
A dedicated small CW Doppler transducer pro-

e
o


vides the optimal signal-to-noise ratio and allows

s
more precise angulation of the transducer.
4.0 o Decrease gain, increase wall filters, and adjust

/r u


the baseline and scale to optimize identification
of the maximum velocity.
o Use the gray-scale spectral displays because with


some color displays the signal-to-noise ratio is

.t c
m/s poor and the edge of the spectral envelope may
be blurred, leading to overestimation of velocity.
o A smooth velocity curve with a dense outer

a


edge and clear maximum velocity should be
recorded; fine lines at the peak of the curve

k
are due to the transit time effect and are not

/: /
6.0 included in measurements.
o Color Doppler is usually not helpful for jet


direction because the jet is short with postste-

s
Figure 11-3 Aortic jet velocity recorded with CW Doppler. An optimal sig- notic turbulence and because the elevational


nal-to-noise ratio is obtained using a small dedicated transducer; the small foot- plane is not visualized.

tt p
print of this transducer also allows optimal positioning and angulation to align the
ultrasound beam parallel to the direction of the stenotic jet. In this example, the
scale has been adjusted to show both aortic stenosis and regurgitation. The aortic Mean gradient
jet should show a denser signal around the edge and a smooth velocity curve. The n Transaortic pressure gradient (ΔP) is calculated from



velocity (v) using the Bernoulli equation (Fig. 11-4) as:

h
difficulty in identifying the maximum velocity is seen in this example, with fuzzy
linear signals at peak velocity that are due to the transit time effect. Maximum
velocity is measured at the edge of the denser signal, as shown by the arrow.
ΔP = 4υ2 (11-1, A)

o Subvalvular or supravalvular stenosis is distin- Maximum gradient is calculated from maximum velocity


guished from valvular stenosis based on the site
ΔPmax = 4υmax2 (11-1, B)
of the increase in velocity and on the anatomy

of the outflow tract.
v KEY POINTS

Step 2: Evaluate Stenosis Severity o When proximal velocity is greater than 1.0 m/s,


it should be included in the Bernoulli equation,
Aortic Jet Velocity (Fig. 11-3) so that:
n Continuous wave (CW) Doppler gray-scale spectral ( )
ΔP = 4 vmax 2 − vproximal 2 (11-2)



recording of aortic jet velocity (Tables 11-1 and 11-2)

Valvular Stenosis  CHAPTER 11 215

TABLE 11-2 Categories of Stenosis Severity

 
Aortic Stenosis
AS Severity Valve Anatomy and Hemodynamics LV Geometry and Function
At risk of AS Bicuspid valve or aortic sclerosis Normal
Vmax < 2 m/s
Progressive AS Mild AS: Vmax 2.0-2.9 m/s or mean Mild LVH and diastolic dysfunction may
ΔP < 20 mmHg

/
be present.
Moderate AS: Vmax 3.0-3.9 m/s or mean
ΔP 20-39 mmHg

9
Severe AS Severe leaflet calcification with reduced LVH, diastolic dysfunction, systolic

9
systolic motion or congenital AS function usually normal.

r
Vmax ≥ 4 m/s, or Patient may be asymptomatic or

i
Mean ΔP ≥ 40 mmHg symptomatic.

­
Typically AVA ≤ 1.0 cm2

h
Low-output low-gradient severe Severely calcified aortic valve. LV EF < 50%

a
AS with low EF Vmax <4 m/s (rest) DSE – Vmax ≥ 4 m/s with AVA ≤ 1.0 cm2

t
AVA ≤ 1.0 cm2

r/
Low-output low-gradient severe Severely calcified aortic valve. Small LV chamber with increased
AS with normal EF Vmax <4 m/s (rest) relative wall thickness.

­
AVA ≤ 1.0 cm2 Normal EF

e
Indexed AVA ≤ 0.6 cm2/m2 Restrictive diastolic filling.
SV index < 35 mL/m2

s
Measured with normal BP

/r u
Mitral Stenosis
MS Severity Hemodynamics Associated Findings
At risk of MS Rheumatic leaflet changes without

.t c
stenosis
Progressive MS Rheumatic valve disease PA systolic pressure < 30 mmHg
Pressure half time < 150 ms Mild LA enlargement
MVA > 1.5 cm2

a
Severe MS MVA ≤ 1.5 cm2 PA systolic pressure > 30 mmHg

k
Moderate LA enlargement

/: /
Very severe MS MVA ≤ 1.0 cm2 PA systolic pressure > 30 mmHg
Moderate-severe LA enlargement

AS, Aortic stenosis; AVA, aortic valve area; DSE, low dose dobutamine stress echocardiography; EF, ejection fraction; LVH, left ventricular hypertro­

s
-
phy; MS, mitral stenosis; MVA, mitral valve area; PA, pulmonary artery.

tt p
o Mean gradient is calculated by tracing the


velocity curve and averaging instantaneous
gradients over the systolic ejection period

h
Bernoulli Equation
(Fig. 11-5). V2
o Any underestimation of aortic velocity ∆P = 4(V22 – V12)


results in an even greater underestimation in ∆P = 4V2
V1
gradients. Stenotic
valve
AMVL
Continuity equation valve area (Fig. 11-6)
n Aortic valve area (AVA) is calculated as:
Figure 11-4 Fluid dynamics of aortic stenosis. There is laminar low ve-


locity flow on the ventricular side of the valve, a small area of acceleration



AVA = (CSALVOT × VTILVOT )/VTIAS (11-3) into the narrow orifice, and the high velocity jet of flow through the narrowed
valve. The distal flow disturbance is shown by the curved arrows. The instan-

n The simplified continuity equation, which uses taneous pressure gradient (ΔP) across the valve is related to the proximal



maximum velocities instead of velocity-time inte- velocity (V1) and jet velocity (V2) as shown. Because the proximal velocity is
much less than the jet velocity, and usually is less than 1 m/s, the simplified
grals (VTIs), also can be used: Bernoulli equitation uses only jet velocity in the equation. AMVL, Anterior
AVA = (CSALVOT × VLVOT )/Vmax (11-4) mitral valve leaflet. (From Otto, CM: Textbook of Clinical Echocardiography,
ed 5, Elsevier, 2013, Philadelphia.)

216 CHAPTER 11  Valvular Stenosis

CW:2MHz APEX AV v KEY POINTS


o LV outflow tract (LVOT) diameter is measured
2.0



in the parasternal long-axis view in mid-systole
using zoom mode and adjusting gain setting to
optimize the blood-tissue interface (Fig. 11-7).

m/s CONTINUITY EQUATION

9/
9
2.0 cm

i r
Time
AVA
LVOT

h
Time
m/s
m/s

a
1

t
5
VTILVOT 20 cm VTIAS120 cm

r/
AoV VTI = 1.35 m
CSALVOT × VTILVOT = AVA × VTIAS
Vmax = 4.71 m/sec
6.0 AVA = (CSALVOT × VTILVOT)/ VTIAS

e
Pk Grad = 88.7 mmHg
Mn Grad = 56.3 mmHg = (3.14 cm2 × 20 cm)/120 cm

s
Figure 11-5 Mean transaortic pressure gradient. For accurate mea- = 0.5 cm2


surement of aortic velocity and mean gradient, the baseline is moved and Figure 11-6 Continuity equation. The basic principle is that the volume

/r u


the scale is adjusted so that the stenotic signal fills the vertical range of the of flow proximal to and in the narrowed valve must be equal. Flow for one
display. The horizontal axis or “sweep speed” is adjusted to 100 mm/s to cardiac cycle in the LV outflow tract (LVOT) is shown as a cylinder with a di-
allow accurate measurement. The Doppler curve is traced along the outer ameter equal to LVOT diameter. Length is equal to the velocity-time integral
edge of the dark signal to obtain the velocity-time integral (VTI). The instanta- (VTI) of LVOT flow (because the integral of velocity over time is distance, like

.t c
neous pressure gradients over the systolic ejection period are averaged by the traveling in a car). The flow through the orifice is shown as a cylinder with
analysis package to provide the mean systolic gradient. Note that the mean the cross-section equal to aortic valve area (AVA) and length equal to the VTI
gradient is not calculated by using the mean velocity in the Bernoulli equation. of the aortic stenosis (AS) jet. Because the volume of both cylinders is the
same, the equation is solved for AVA as shown.

k a
/: /
LVOTd LVOTv
2.2 cm 0.7 m/s

s
tt p
Ao
LV

h A
LA

B
0.7 m/s

Figure 11-7 Transaortic volume flow rate. A, LV outflow tract diameter (LVOTd) is measured in a parasternal long-axis view in mid-systole from the inner


edge of the septum to the inner edge of the anterior mitral leaflet, immediately adjacent to the aortic valve leaflets (markers). A magnified image allows more
accurate measurement, and typically several beats are measured to ensure a reproducible value. A typical outflow tract diameter is 2.2 to 2.6 cm in adult men
and 2.0 to 2.4 cm in adult women. B, Although LVOT diameter is measured from the parasternal window, to provide axial resolution of the tissue-blood inter-
faces, with ultrasound imaging, LVOT velocity is recorded from the apical window to allow parallel alignment between the ultrasound beam and flow direction.
Pulsed Doppler is used to measure the velocity signal on the ventricular side of the aortic valve, in an anteriorly angulated 4-chamber view (as shown here) or
in a long-axis view. The sample volume length or gate is adjusted to 2 to 3 mm and the sample volume is positioned as close to the valve as possible (often the
closing click is seen), avoiding the small area of flow acceleration immediately adjacent to the stenotic orifice. The sample volume position should correspond to
the site where LVOT diameter was measured. The velocity range and baseline are adjusted so the signal fits but fills the scale, using a fast (100 to 150 mm/s)
horizontal axis scale. A smooth curve with a dense band of velocities (“envelope of flow”) with a well-defined peak velocity should be seen. The midpoint of the
spectral broadening at peak velocity is measured. Ao, Aorta.
Valvular Stenosis  CHAPTER 11 217

o Diameter (D) is measured at the basal insertion


Step 3: Evaluate Aortic Regurgitation and


of the aortic leaflets from inner edge to inner
edge of the aortic annulus. Calculate the circu- Ascending Aorta
lar cross-sectional area: n If regurgitation is significant (vena contracta ≥ 3




mm), evaluate as detailed in Chapter 12.
CSALVOT = 3.14(D/2)2 (11-5)
n Dilation of the ascending aorta may accompany





o LVOT diameter is useful for valve area calcula- aortic stenosis, particularly with a bicuspid valve.


tion but should not be used in isolation for sizing

/
of transcatheter valves because of the complex v KEY POINTS


anatomy of the outflow tract and valve. 3D o Most patients with aortic stenosis have some

9


imaging of the aortic annulus may be helpful in degree (usually mild) of regurgitation.
this situation (Fig. 11-8) With combined moderate stenosis and regurgi-

9
o



LVOT velocity is recorded with pulsed Doppler tation, quantitation of both lesions is needed.

r
o


i
from the apical window with the sample volume o The end-diastolic diameter of the aorta is mea-



positioned just apically from the flow accelera- sured at the sinuses, sinotubular junction, and

h
tion into the valve. An aortic closing click on midascending aorta when aortic valve disease is
the spectral tracing indicates correct sample present (see Chapter 16).

a
volume positioning.

t
Move the baseline, adjust the velocity scale, Step 4: Evaluate the Consequences

r/
o
of Chronic Pressure Overload


and use an expanded time scale for accurate
measurements. n Measure LV size and wall thickness and calculate



Trace the modal systolic velocity (VTILVOT) and ejection fraction as detailed in Chapter 6.

e
o


measure peak velocity (VLVOT). n Evaluate LV diastolic function as detailed in

s



o If LVOT diameter cannot be accurately mea- Chapter 7.


sured, calculate the ratio of LVOT to aortic jet n Evaluate coexisting mitral regurgitation (if vena

/r u


contracta ≥ 3 mm) as detailed in Chapter 12.

velocity:
n Estimate pulmonary pressures as detailed in



Ratio = VLVOT /VMAX (11-6) Chapter 6.
­

.t c
n A ratio less than 0.25 indicates severe stenosis. v KEY POINTS




o Aortic stenosis typically results in concentric LV


hypertrophy.

a
o Systolic function and ejection fraction remain
Short axis


normal in most patients but, occasionally, sys-

k
LVOT
LA tolic dysfunction is identified in an asymptom-

/: /
Ao atic patient.
o Diastolic dysfunction, usually impaired relax-


ation, is common.

s
LV o Pulmonary pressures may be elevated with


longstanding severe aortic stenosis

tt p
3D volume Step 5: Additional Evaluation for Low-output
Low-gradient Aortic Stenosis
n Additional evaluation may be needed when clini-

h



cal evaluation suggests aortic stenosis may be more
severe than indicated by standard measures of ste-
nosis severity (Table 11-3).
n With a calcified aortic valve and a valve area ≤ 1.0


cm2 but a velocity < 4 m/s in conjunction with
Figure 11-8 3D measurement of aortic annulus area and diam-
an LV ejection fraction less than 50%, dobutamine

eter. 3D TEE is used to measure LV outflow tract diameter and area in
stress echo should be considered.

a patient being considered for transcatheter aortic valve implantation. The
long-axis view (upper left) corresponds to the standard 2D measurement of n With a calcified aortic valve and a valve area ≤


annulus diameter (D1) as shown. Using the orthogonal plane (lower left) and 1.0 cm2 but a velocity < 4 m/s and a normal ejec
­
adjusted the 3D image planes shown by the red, green, and blue lines (and tion fraction an integrative approach to evaluation
shown as image planes on the 3D volume image (lower right), a short-axis
view of the annulus is obtained which allows measurement of additional is needed.
diameters (D2 and D3) as well as annular perimeter and area. Ao, Aorta;
LVOT, left ventricular outflow tract.
218 CHAPTER 11  Valvular Stenosis

TABLE 11-3 Possible Causes of

 
Discrepancies in Measures
of Aortic Stenosis Severity 1.0

Severe AS by Velocity or Gradient But Not by Valve


Area (AS Velocity > 4 m/s and AVA > 1.0 cm2) m/s

• LVOT diameter overestimated




• LVOT velocity recorded too close to valve


/
• High transaortic flow rate due to:


• Moderate to severe aortic regurgitation


9
• High output state


• Large body size

9


Severe AS by Valve Area But Not by Velocity or

r
­
Gradient (AS Velocity ≤ 4 m/s and AVA ≤ 1.0 cm2)

i
4.0
• LVOT diameter underestimated


h
• LVOT velocity recorded too far from valve


• Small body size Figure 11-9 Effect of variable heart rate on aortic velocity. When the



cardiac rhythm is irregular, the velocity (and pressure gradient) across a ste-


a
• Low transaortic flow volume due to:
notic valve varies with the length of the R-R interval because of an increased


t
• Low ejection fraction
stroke volume with a longer diastolic filling period. This example shows the

r/


• Small ventricular chamber
variation in aortic jet velocity (at a slow sweep speed to include multiple beats)


• Moderate to severe mitral regurgitation in a patient in atrial fibrillation. Ideally, heart rate should be controlled before


• Moderate to severe mitral stenosis evaluation of stenosis severity is performed. Several beats are then averaged


e
for each measurement. Signal quality in this example is suboptimal so ad-
From Otto CM: Textbook of Clinical Echocardiography, ed 5. Philadel­ ditional efforts to improve patient and/or transducer positioning are needed.

s
phia, Elsevier, 2013.
AS, Aortic stenosis; AVA, aortic valve area; LVOT, left ventricular

/r u
outflow tract.
MITRAL STENOSIS
Step-by-Step Approach

.t c
v KEY POINTS Step 1: Evaluate Mitral Valve Morphology

o The degree of valve calcification (mild, mod- (Fig. 11-11)


erate, severe) is a simple, important parameter n Use long- and short-axis views of the mitral valve

a



that is predictive of clinical outcome. to demonstrate the typical findings of rheumatic
The dimensionless ratio of outflow tract to valve disease:

k
o


aortic jet velocity provides a simple index • Commissural fusion resulting in diastolic doming

/: /


of stenosis severity (normal, 1.0; mild, 0.5; • Chordal shortening and fusion


severe, 0.25). n Evaluate mitral valve leaflet mobility, thickening,



o Planimetry of the valve area can be helpful in calcification, and subvalvular disease.

s


selected cases with excellent images, but caution n Use 3D imaging for better evaluation of the sym-



is needed due to reverberations and shadowing metry of commissural fusion (Fig. 11-12).

tt p
from leaflet calcification.
o Blood pressure should be recorded at the time v KEY POINTS



of the velocity data acquisition; stenosis severity o Rheumatic valve disease is the most common


may be underestimated in hypertensive patients cause of mitral stenosis.

h
so measurement should be repeated after effec- o Rarely, severe mitral annular calcification


tive treatment of hypertension. encroaches on the mitral orifice, but calcific ste-
o With atrial fibrillation, several beats should be nosis is rarely severe.


averaged for each measurement (Fig. 11-9). o In addition to a numerical score, a narrative


o With low output aortic stenosis and a low LV description of valve anatomy is helpful for


ejection fraction, a velocity greater than or deciding on the optimal intervention.
equal to 4 m/s with an aortic valve area less o The extent of commissural calcification and


than or equal to 1.0 cm2 on dobutamine stress asymmetry of leaflet calcification should be noted.
echocardiography is consistent with severe aor- o The subvalvular apparatus may be best seen on


tic stenosis (Fig. 11-10). apical views (and poorly visualized on TEE).
Valvular Stenosis  CHAPTER 11 219

BASELINE STRESS−SEVERE AS STRESS−MODERATE AS


Systolic valve opening

No change Increased
from baseline leaflet opening

HR 70 HR 90 HR 90
0 0 0
LVOT velocity (m/s)

1.0 0.8 m/s 1.0 1.2 m/s 1.0 1.2 m/s


VTI 13 cm VTI 18 cm VTI 18 cm
SV 50 mL SV 68 mL SV 68 mL

0 0 0

1 1 1
AS-Jet velocity (m/s)

2 2 2

3 3 3
3.5 m/s 5.1 m/s 3.8 m/s
4 AVA 0.9 cm2 4 AVA 0.9 cm2 4 AVA 1.2 cm2

5 5 5

Figure 11-10  Low-output low-gradient aortic stenosis (AS).  Changes in aortic valve opening and Doppler flows with dobutamine stress echocardiography
for low-output low-gradient AS. The baseline data show a hypothetical patient with an ejection fraction (EF) of 35% and limited aortic valve systolic opening,
an aortic jet velocity (AS-jet) of 3.5 m/s, and aortic valve area (AVA) of 0.9 cm2. If true, severe AS is present (middle panel), as EF increases from 35% to 45%,
transaortic flow rate increases but aortic opening is fixed, resulting in a marked increase in aortic velocity (and pressure gradient) with no change in valve area.
In a patient with the same baseline data but “pseudo-severe AS,” the increase in EF and transaortic stroke volume “push” the aortic leaflets to open more so
there is a smaller increase in aortic velocity in association with an increase in AVA. Current diagnostic testing relies on Doppler data with dobutamine stress
testing because direct imaging of valve anatomy is not adequate for visualization of the exact systolic orifice. SV, stroke volume; VTI, velocity-time integral. (From
Otto CM, Owens DS: Stress testing for structural heart disease. In Gillam LD, Otto CM. Advanced Approaches in Echocardiography: Practical Echocardiography
Series, Elsevier, 2013, Philadelphia.)

Ao
MVO

LV

LA

A B
Figure 11-11  Rheumatic mitral stenosis.  A, In this parasternal long-axis view, the typical changes of rheumatic mitral stenosis are seen. There is diastolic
doming (arrow) of the anterior mitral leaflet due to commissural fusion. B, The rheumatic mitral valve orifice is imaged in short axis, taking care to scan from
apex toward base to identify the smallest area of the funnel-shaped stenotic orifice. The inner edge of the white-dark interface is traced to obtain valve area.
The distance between the LV wall and the edge of the orifice reflects the degree of commissural fusion. Ao, Aorta; MVO, mitral valve orifice.
220 CHAPTER 11  Valvular Stenosis

Step 2: Evaluate the Severity of Mitral TABLE 11-4 Pitfalls in Evaluation of Mitral


Stenosis Stenosis Severity
Planimetry of valve area (see Fig. 11-11 and Fig. 11-12) Pressure Gradient
n Valve area is measured directly by tracing the orifice • Intercept angle between mitral stenosis jet and
in a short-axis view at the leaflet tips (Table 11-4). ultrasound beam
n 
Mitral valve area may be measured accurately • Beat-to-beat variability in atrial fibrillation
with 2D imaging by experienced sonographers but • Dependence on transvalvular volume flow rate (e.g.,
ensuring measurement at the mitral leaflet tips is exercise, coexisting mitral regurgitation)
more reliable with 3D imaging. 2D Valve Area
• Image orientation
v  KEY POINTS • Tomographic plane
o With 2D imaging, starting in a parasternal • 2D gain settings
short-axis orientation, the image plane is slowly • Intra- and interobserver variability in planimetry of
moved from the apex toward the base to iden- orifice
• Poor acoustic access
tify the orifice of the funnel-shaped stenotic
• Deformed valve anatomy postvalvuloplasty
valve.
o The zoom mode is used to focus on the valve T1⁄2 Valve Area
orifice, with the gain reduced to clearly show • Definition of Vmax and early diastolic slope
the tissue-blood interface. • Nonlinear early diastolic velocity slope
o With 3D imaging, a 3D volume is acquired • Sinus rhythm with a wave superimposed on early
diastolic slope
from the parasternal window, taking care that
• Influence of coexisting aortic regurgitation
the volume includes the orifice and avoiding • Changing LV and LA compliances immediately after
stitch artifact. commissurotomy
o The 3D volume is used to obtain a short-axis
Continuity Equation Mitral Valve Area
view at the leaflet tips using three orthogonal
image planes. • Accurate measurement of transmitral stroke volume
o The inner border of the black-white interface is
From Otto CM: Textbook of Clinical Echocardiography, ed 5. Philadel­
traced to obtain valve area. phia, Elsevier, 2013.

4-chamber 2-chamber

Figure 11-12  3D measurement of mitral


valve area.  Using 3D TEE volumetric data, Short axis 3D volume
images are shown in three planes identified
by green, blue, and red. The plane for each
color box is shown as a line in the other two
views. The blue box at lower left is the first
attempt at a short-axis view of the mitral
orifice. In the 4-chamber view, the lines to
be adjusted as illustrated by the arrows so
the red plane is centered in the orifice and
the blue plane is at the leaflet tips. Similar
adjustments are needed in the 2-chamber
image. These planes are adjusted iteratively
until the minimal orifice at the leaflet tips is
visualized. The short-axis image of the ori-
fice then is traced from the inner edge of the
white-black interface.
Valvular Stenosis  CHAPTER 11 221

The orifice typically is a smooth, elliptical shape


o o Movement of the heart with respiration may
in patients with no prior procedures. result in variation in the Doppler curve due to a
o After percutaneous or surgical valvotomy, the variation in the intercept angle; if so, have the
orifice is more irregular due to splitting of the patient suspend respiration briefly during data
fused commissures. recording.

Mean gradient (Fig. 11-13) Pressure half-time valve area (Fig. 11-14)
n The Doppler velocity curve across the narrowed n The pressure half-time is calculated from the Dop-
mitral orifice is recorded from the apical window. pler curve at the time interval between peak veloc-
n 
Mean gradient is determined using the Bernoulli ity and the peak velocity divided by 1.4.
equation to average the instantaneous pressures n 
The empiric constant 220 is divided by the pres-
gradients over the diastolic filling period. sure half-time (T½ in milliseconds) to estimate
mitral valve area (MVA in cm2):
v  KEY POINTS
The mitral stenosis jet is directed toward the
o MVA = 220/T 1/2 (11-7)
apex, so only minor adjustment of transducer
position and angulation is needed to obtain a
parallel angle between the Doppler beam and
Step 1: Identify the peak early diastolic velocity.
mitral jet; color flow Doppler can help with
alignment. 2.0
o Transducer position and gain are adjusted to
demonstrate a clear outer boundary of the
m/s

velocity curve with a well-defined peak and a 1.0


linear deceleration slope.
o The baseline is moved toward the edge of the
display, the scale adjusted so the Doppler curve
fits but fills the space, and gain and wall filters 1.0 sec
adjusted to decrease signal noise.
o Pulsed or high pulse repetition frequency
(HPRF) Doppler may provide a more clearly Step 2: Draw a line along the diastolic deceleration slope.
defined velocity curve than CW Doppler. 2.0
m/s

.74 40dB 3 •/+1/0/ 1 1.0


PW Depth= 83mm
PW Gate= 2.5mm
PW Gain= 6dB

.74
MV VTI, leaflet tips = 0.63 m
1.0 sec
Mn Grad = 6.0 mmHg
2.0
Step 3: The pressure half time (T½) is the time interval in
milliseconds between the peak velocity and the
point on the deceleration line equal to the peak
velocity divided by 1.4 (the square root of 2).

2.0
m/s

1.4
m/s
1.0
Figure 11-13  Mean transmitral pressure gradient in mitral steno- T½
sis.  The transmitral velocity is recorded with pulsed Doppler (including
HPRF) or CW Doppler if needed to prevent signal aliasing, from an apical
window with the baseline shifted and the velocity scale adjusted so that 1.0 sec
the Doppler velocity fits the vertical axis of the tracing. The time scale is Figure 11-14  Mitral pressure half-time measurement.  In this example,
set at 100 to 150 mm/s with the electrocardiogram included for timing. the pressure half-time is 320 ms. The mitral valve area is 220/320 = 0.7
After a smooth Doppler curve with a narrow band along the outer edge cm2, consistent with severe stenosis.
and a clearly defined peak is obtained, the outer edge of the signal is
traced. The analysis package averages the instantaneous gradients over
the diastolic filling period. This patient is in sinus rhythm, which does not
affect the accuracy of Doppler evaluation of stenosis severity.
222 CHAPTER 11  Valvular Stenosis

Continuity equation valve area


v  KEY POINTS n If further evaluation of mitral stenosis severity is
o The peak velocity occurs at the onset of diastole needed, a continuity equation valve area can be
with flow deceleration in mid-diastole. calculated.
o A clearly defined peak velocity is needed for an n 
Transmitral stroke volume (SV) is divided by the
accurate pressure half-time measurement (Fig. velocity-time integral of the mitral stenosis jet
11-15). (VTIMS) to obtain mitral valve area:
o The diastolic slope should be linear with a
MVA = SV/VTIMS (11-8)
clearly defined edge; if a nonlinear slope is
obtained, the mid-diastolic segment of the
curve should be used for the pressure half-time v  KEY POINTS
calculation (Fig. 11-16). Transmitral stroke volume is determined in the
o
o The pressure half-time may be inaccurate if left LVOT or across the pulmonic valve.
atrium (LA) or LV compliance is abnormal. o This approach is only accurate when there is no
o If an atrial contraction is present, only the early mitral regurgitation.
diastolic portion of the curve is included in the
pressure half-time calculation. Step 3: Evaluate Mitral Regurgitation
(Fig. 11-17)
If regurgitation is significant (vena contracta ≥ 3
n 
mm), evaluate as detailed in Chapter 12.
3V2c-S 4sec
H4.0MHz R14mm
v  KEY POINTS
UWMC
UWMC /V
Pwr= 0dB
o Most patients with rheumatic mitral stenosis
.74
Mlcd=1.9 TIS=1.1 have some degree of mitral regurgitation.
MV DT = 643 msec
P1/2 Time = 186 msec HR= 60bpm
2.0 Sweep=100mm/s
CW:3.5MHz

m/s

m/s

2.0
Figure 11-15  Pressure half-time example.  On the ultrasound system,
the pressure half-time is measured by identifying the peak early diastolic
velocity and then placing a line along the mid-diastolic deceleration slope. Figure 11-16  Nonlinear diastolic velocity curve. TEE imaging in a
The quantitation software then calculates the time interval between the peak 64-year-old woman with rheumatic mitral valve disease shows CW Doppler
gradient and half the peak gradient. The empiric constant 220 is divided by antegrade flow (away from the transducer) across the stenotic mitral valve.
the pressure half-time by to obtain the valve area. The initial diastolic slope (arrow) is steeper than the mid-diastolic slope.
When there is an initial steep decline in velocity (often called a “ski-slope”
pattern) with a flatter mid-diastolic slope, the pressure half-time is measured
along the mid-diastolic portion of the curve, as shown on the second beat,
extrapolating back to the onset of flow.

Ao

LV

LA
Figure 11-17  Coexisting rheumatic
mitral regurgitation.  A, In this patient
with mitral stenosis, the diastolic long-axis
view shows typical diastolic doming due to
commissural fusion. B, In systole, there is
moderate-severe mitral regurgitation with a A B
wide vena contracta. Ao, Aorta.
Valvular Stenosis  CHAPTER 11 223

With combined moderate stenosis and regurgi-


o
tation, quantitation of both lesions is needed. v  KEY POINTS
o The degree of mitral regurgitation may need LA enlargement is usually present in patients
o
to be evaluated by TEE because moderate or with mitral stenosis and is related to the severity
greater regurgitation is a contraindication to and chronicity of mitral valve obstruction.
percutaneous valvotomy. o Patients with rheumatic mitral valve disease
have a high risk of atrial thrombus, even if in
Step 4: Examine Aortic and Tricuspid Valves sinus rhythm.
for Rheumatic Involvement o Pulmonary pressures are elevated passively
n When rheumatic mitral stenosis is present, careful due to the increased LA pressure. In addition,
evaluation of aortic and tricuspid valves is needed reactive pulmonary hypertension is seen with
to detect rheumatic involvement. changes in the pulmonary vasculature that may
persist after relief of mitral stenosis
v  KEY POINTS o RV enlargement and systolic dysfunction in
Rheumatic disease typically affects the mitral
o patients with mitral stenosis may be due to pulmo-
valve first, causing stenosis and/or regurgitation. nary hypertension (pressure overload) or to rheu-
o The aortic valve is affected in about 35% of matic tricuspid regurgitation (volume overload).
patients and the tricuspid valve in about 6% of
patients with rheumatic mitral valve disease. Tricuspid Stenosis
o The appearance of rheumatic disease affecting n  Tricuspid stenosis is uncommon and usually is
the aortic and tricuspid valves is similar to the due to rheumatic tricuspid valve involvement in
mitral valve, with commissural fusion being the patients with mitral stenosis
most consistent feature. n Evaluation of rheumatic tricuspid stenosis is simi-
lar to evaluation of mitral stenosis
Step 5: Evaluate the Consequences of Mitral
Valve Obstruction v  KEY POINTS
n Measure LA size (Fig. 11-18). o 
Rheumatic tricuspid stenosis appears similar
n Evaluate for LA thrombus on TEE if a valve pro- to mitral stenosis, with commissural fusion and
cedure is considered (Fig. 11-19). diastolic bowing of the leaflets.
n  Estimate pulmonary pressures as detailed in o 
Carcinoid disease can cause tricuspid stenosis
Chapter 6. with thickened, shortened leaflets.
n Evaluate right ventricle (RV) size and systolic function. o 
The antegrade tricuspid velocity curve,
recorded from an RV inflow view or an apical
approach, allows measurement of mean gradi-
ent and pressure half-time.
o 
Diastolic pressure gradients may be lower for
the tricuspid, compared with mitral stenosis
(Fig. 11-20).
o 
Planimetry of the stenotic tricuspid orifice
LV rarely is possible with 2D imaging; 3D imaging
RV may be helpful in some cases.

RA LA
LA

LAA

LV

Figure 11-19  LA thrombus.  In this patient with mitral stenosis, the LA is


Figure 11-18  LA enlargement.  The marked degree of LA enlargement in severely enlarged on TEE imaging. Although a thrombus is not seen in the LA ap-
this patient with rheumatic mitral stenosis is seen in an apical 4-chamber pendage, there is marked spontaneous contrast with low velocity swirling blood
view. The LA area appears larger than the LV area in this frame. flow consistent with a high risk of thromboembolism. LAA, Left atrial appendage.
224 CHAPTER 11  Valvular Stenosis

CW:2MHz APX TV
4.0
2.0

m/s

m/s

2.0
4.0
Figure 11-20  Tricuspid stenosis.  Antegrade flow across a stenotic tri-
cuspid valve recorded with CW Doppler from an apical view. The velocity is Figure 11-21  Pulmonic stenosis.  CW Doppler recording of pulmonary
markedly increased, with a mean gradient of 12 mmHg, and the diastolic valve flow showing an antegrade velocity of 3.2 m/s, consistent with a
slope is very flat with a pressure half-time of 400 ms and a valve area of maximum gradient of 41 mmHg, or moderate pulmonic stenosis. Pulmonic
0.6 cm2. regurgitation is seen above the baseline and appears to be moderate based
on the relative density of retrograde versus antegrade flow. Pulmonary pres-
sures are low, based on the low end-diastolic velocity of the regurgitant
signal.
Pulmonic Stenosis (Fig. 11-21)
n The velocity across the pulmonic valve is recorded
using pulsed or CW Doppler from a parasternal
approach. Grading of stenosis severity is based on the
o
n Maximum gradient is calculated using the Ber- maximum transvalvular pressure gradient (mild
noulli equation. < 25 mmHg; moderate 25 to 50 mmHg; severe
n 
Coexisting pulmonic regurgitation is evaluated by > 50 mmHg).
color and CW Doppler. o Pulmonic stenosis often is accompanied by sig-
nificant pulmonic regurgitation, particularly if
v  KEY POINTS there has been a prior surgical or percutaneous
Pulmonic stenosis usually is due to congenital
o procedure.
heart disease and may be an isolated defect or o Branch pulmonary artery stenosis may also be
a component of more complex congenital dis- present and is difficult to evaluate by echocar-
ease, such as tetralogy of Fallot. diography, although evaluation of the proximal
o Visualization of the pulmonic valve is challeng- right and left pulmonary may be possible from
ing on both transthoracic and TEE imaging a high parasternal short-axis view.
in adults; often Doppler data are used to infer
valve pathology.
Valvular Stenosis  CHAPTER 11 225

THE ECHO EXAM

Aortic Stenosis
Aortic Stenosis the echo instrument calculating and then averaging
instantaneous pressure gradients over the systolic
Valve anatomy Calcific ejection period. The simplified method for estimation
Bicuspid (2 leaflets in systole) of mean gradient is:
Rheumatic
ΔP = 2.4(Vmax )2 = 2.4(4.5)2 = 49 mmHg  (11-10)
Stenosis severity Jet velocity (Vmax)
Mean pressure gradient (ΔPmean) In order to correct for transvalvular volume flow
LVOT/AS velocity ratio
rate, the velocity ratio and valve area are calculated:
Velocity ratio is:
Aortic valve area (AVA)
Coexisting AR Qualitative evaluation of severity VLVOT /Vmax = 0.9/4.5 = 0.2
(11-11)
(dimensionless index)
LV response LV hypertrophy
LV dimensions or volumes Aortic valve area is:
LV ejection fraction
AVA = (CSALVOT × VTILVOT ) /VTIAS ‐ Jet  (11-12)
Other findings Pulmonary pressures
Mitral regurgitation Where cross-sectional area (CSA) of the LVOT is:

CSALVOT = π(LVOTD /2)2 = 3.14(2.4/2)2 (11-13)


Example = 4.5 cm2 
A 79-year-old man presents with dyspnea on exer-
tion and is noted to have a 3/6 systolic murmur at Thus:
the base, radiating to the carotids with a single S2 and
diminished carotid upstrokes. AVA = (4.5 cm2 × 16 cm)/83 cm = 0.87 cm2 (11-14)

Echocardiography shows a calcified aortic valve with:
Simplified formula for valve area is:
Aortic jet velocity (Vmax) 4.5 m/s
AVA = (CSALVOT × VLVOT )/Vmax  (11-15)
Velocity time integral (VTIAS) 83 cm
Mean gradient 46 mmHg Thus:
LV outflow tract diameter (LVOTD) 2.4 cm ( )
AVA = 4.5 cm2 × 0.9 cm/s /4.5 cm/s (11-16)
2
LVOT velocity (VLVOT) 0.9 m/s = 0.9 cm 
Velocity time integral (VTILVOT) 16 cm This mean gradient (> 40 mmHg), velocity ratio
(<0.25), and valve area (< 1.0 cm2) are all consistent
The maximum jet velocity of 4.5 m/s indicates severe ste- with severe stenosis.
nosis, which is confirmed by calculation of maximum
and mean pressure gradients.
Maximum pressure gradient is calculated from maxi- Classification of Aortic Stenosis Severity
mum aortic jet velocity (Vmax) as: Mild Severe
Jet velocity (m/s) < 3.0 ≥ 4.0
ΔPmax = 4(Vmax )2 = 4(4.5)2 = 81 mmHg (11-9) Mean gradient (mmHg) < 20 ≥ 40
Velocity ratio > 0.50 < 0.25
Mean pressure gradient is calculated by tracing the
Valve area (cm2) > 1.5 ≤ 1.0
outer edge of the CW Doppler velocity curve, with
226 CHAPTER 11  Valvular Stenosis

Quantitation of Aortic Stenosis Severity


Components Modality View Recording Measurements
LVOT diameter 2D Parasternal long- Adjust depth, optimize Inner edge to inner
(LVOTD) axis endocardial definition, zoom edge of LVOT,
mode ­parallel and adjacent
to aortic valve,
­mid-systole
LVOT flow Pulsed Doppler Apical 4-chamber Sample volume 2-3 mm, Trace modal velocity
VLVOT (anteriorly envelope of flow with defined of spectral velocity
VTILVOT angulated) peak, start with sample volume curve
at valve and move apically
AS-Jet CW Doppler Apical, SSN, other Examination from multiple Measure maximum
Vmax ­windows, careful positioning, velocities at edge
VTIAS-Jet and transducer angulation to of intense velocity
obtain highest velocity signal signal
ΔPmax= 4 (Vmax)2
Continuity equation AVA (cm2) = [π (LVOTD /2)2
× VTILVOT]/ VTIAS-Jet
Simplified continuity AVA (cm2) = [π (LVOTD/2)2
equation × VLVOT]/ VAS-Jet
Velocity ratio Velocity ratio = VLVOT/ VAS-Jet

AS, Aortic stenosis; AVA, aortic valve area; LVOT, LV outflow tract; SSN, suprasternal notch; VLVOT, LVOT velocity; Vmax, maximum velocity; VTI,
velocity-time integral.

Mitral Stenosis Example


A 26-year-old pregnant woman presents with dyspnea
Mitral Stenosis: Echo Approach and is noted to have a diastolic murmur at the apex.
Valve anatomy Valve thickness and mobility
Echocardiography shows rheumatic mitral stenosis
with:
Calcification
Commissural fusion MVA2D 0.8 cm2
Subvalvular involvement Mean ΔP 5 mmHg
Stenosis severity 2D valve area T½ 260 ms
Mean pressure gradient Mitral valve morphology score:
Pressure half-time valve area
Leaflet thickness Mildly increased
Left atrium Size
Mobility Diastolic doming
TEE for thrombus prevalvuloplasty
Calcification Mild at leaflet tips
Co-existing MR Qualitative evaluation of severity
Pulmonary Pulmonary systolic pressure
Subvalvular Moderate chordal
vasculature shortening
Right ventricular size and function
Tricuspid regurgitant 3.1 m/s
Other findings Aortic valve involvement jet velocity
LV size and systolic function Estimated RA pressure 10 mmHg
Mitral regurgitation Mild

Mean pressure gradient is calculated by tracing the outer


edge of the CW Doppler velocity curve, with the echo
instrument calculating and then averaging instanta-
neous pressure gradients over the systolic ejection
period.
Valvular Stenosis  CHAPTER 11 227

Doppler mitral valve area (MVADoppler) is calculated as: Mitral valve morphology was suitable and only
mild mitral regurgitation was present, indicat-
MVADoppler = 220/T 1/2 = 220/260 = 0 85 cm2 (11-17) ing a high likelihood of immediate and long-term
success with balloon mitral valvuloplasty. TEE is
The 2D mitral valve area and the pressure half needed just before mitral valvuloplasty to evaluate
time valve area show reasonable agreement and both for LA thrombus.
are consistent with severe mitral stenosis.
Pulmonary artery pressure (PAP) is:
Classification of Mitral Stenosis Severity
PAP = 4(VTR )2 + RAP = 4(3.1)2 + 10 mmHg (11-18)
= 48 mmHg  Mild Severe Very Severe
Pressure half-time < 150 ≥ 150 ≥ 220
Pulmonary pressure is moderately elevated con-
Valve area (cm2) > 1.5 ≤ 1.5 ≤ 1.0
sistent with a secondary response to severe mitral
stenosis.

Quantitation of Mitral Stenosis Severity


Parameter Modality View Recording Measurements
2D valve area 2D Parasternal short-axis Scan from apex to base to Planimetry of inner
(MVA2D) identify minimal valve area. edge of dark-light
interface
Mean gradient HPRF Doppler Apical 4-chamber Align Doppler beam parallel Trace maximum
(Mean ΔP) or long-axis to MS jet. Adjust angle to velocity of spectral
obtain smooth envelope, velocity curve
clear peak and linear
­deceleration slope.
Pressure half-time HPRF Doppler Apical 4-chamber Same as mean gradient. Place line from maxi-
(T½) or long-axis ­Adjust scale so ­velocity mum velocity along
curve fills the screen. mid-diastolic linear
HPRF Doppler often slope
has less noise than CW MVA = 220/ T½
­Doppler signal.

HPRF, High pulse repetition frequency; MS, mitral stenosis; MVA, mitral valve area.
228 CHAPTER 11  Valvular Stenosis

SELF-ASSESSMENT QUESTIONS

Question 1 Question 4
Match the clinical condition with the phenotypic All of the following are needed for calculation of aor-
manifestation: tic valve area with the continuity equation except:
1 . Noncoronary cusp calcium A. CW Doppler aortic velocity curve
2 . Leaflet edge thickening B. Transaortic mean pressure gradient
3 . Early systolic cusp closure C. LV outflow or aortic annulus diameter
4 . Commissural raphe D. LV outflow velocity time integral
A. Hypertrophic cardiomyopathy
B. Aortic sclerosis Question 5
C. Rheumatic valve disease An asymptomatic patient with rheumatic mitral ste-
D. Congenitally bicuspid valve nosis is seen for routine follow-up. The diastolic flow
curve shows an increased velocity and flat diastolic
Question 2 slope with the following measurements:
A 42-year-old woman presents with palpitations and
is newly diagnosed with atrial fibrillation. A trans- Maximum velocity 2.0 m/s
thoracic echocardiogram documents concentric LV The time interval between maximum velocity and
hypertrophy with preserved systolic function. There is various points on the diastolic flow curve are mea-
an aortic velocity of 4.8 m/s. The patient undergoes sured as follows:
TEE to exclude a LA appendage thrombus before
direct cardioversion (Fig. 11-22). The image obtained 1.8 m/s 190 ms
from the TEE study is consistent with:
A. Supravalvular stenosis 1.4 m/s 225 ms
B. Hypertrophic cardiomyopathy 1.0 m/s 240 ms
C. Aortic stenosis
0.6 m/s 280 ms
D. Subaortic stenosis   

Calculate the mitral valve area ___________

Question 6
The following parasternal long-axis view M-mode
image is consistent with (Fig. 11-23):
A. Mitral stenosis
B. Aortic stenosis
C. Tricuspid stenosis
D. Pulmonic stenosis

Figure 11-22 

Question 3
In an 82-year-old woman with a loud systolic murmur
on exam, you suspect that peak aortic jet velocity is
underestimated on echocardiography. Which of the 180
following views would be most likely to help identify
the peak aortic jet velocity?
A. Left parasternal view Figure 11-23 
B. Subcostal 4-chamber view
C. Suprasternal notch view
D. Parasternal long-axis view
Valvular Stenosis  CHAPTER 11 229

Question 7
A 60-year-old woman with tricuspid valve stenosis A. Average of peak gradient three cardiac cycles
is admitted to the hospital with dyspnea and pedal B. Mean gradient of longest Doppler signal
edema. The transtricuspid Doppler tracing is shown C. Average of mean gradient three cardiac cycles
(Fig. 11-24). In this case, the severity of stenosis is best D. Peak gradient of highest Doppler signal
assessed by measuring:

2.0

m/s

Figure 11-24 

Question 8
V = –3.28m/s
An 82-year-old female patient is referred for progres- PG = 43.0mmHg
sive exertional dyspnea. She has a known history of 1.0
moderate aortic stenosis. A myocardial perfusion
study did not show evidence of myocardial ischemia.
Prior TTE done 2 years ago demonstrated an ejection m/s
fraction of 68% without regional wall motion abnor-
malities; the aortic valve was calcified with peak aortic
jet velocity of 3.6 m/s. She is now referred for a trans-
thoracic echo and the following aortic jet velocity is
obtained (Fig. 11-25). The most appropriate next step
in patient management is:
A. Transesophageal echocardiography
B. Repeat transthoracic echocardiography
C. Dobutamine stress echocardiography
D. Coronary angiography

5.0

Figure 11-25 
230 CHAPTER 11  Valvular Stenosis

Question 9
A 34-year-old woman presents for clinical evalua- A.
Holodiastolic flow reversal abdominal aorta
tion and the following image is obtained (Fig. 11-26). B. Low septal tissue Doppler velocity
Based on this image, which additional echocardio- C. Pulmonary vein systolic flow reversal
graphic finding is likely? D. Dilated inferior vena cava

300

240

180

120

60

cm/s

Figure 11-26 

Question 10
A 88-year-old man presents with aortic stenosis. On Based on these data, do the following calculations:
  
physical examination vital signs reveal a blood pres-
Transaortic stroke volume _________________
sure of 144/80 mmHg and heart rate of 60 bpm. On
Cardiac output _________________
TTE, the following data was obtained:
Continuity equation aortic valve area ___________
Aortic velocity ratio _________________________
LVOT diameter 2.0 cm Overall, the degree of aortic stenosis is __________
LVOT velocity 1.1 cm/s
LVOT VTI 0.26 m
Aortic velocity 4.4 m/s
Aortic VTI 0.97 m
Valvular Stenosis  CHAPTER 11 231

Question 11 Question 12
A 44-year-old woman is referred for echocardiogra- A 82-year-old man with prior ischemic heart disease
phy for new onset atrial fibrillation. She lives in a rural presents for evaluation. Echocardiography demon-
area and has not seen care providers regularly due to strates a heavily calcified aortic valve, an EF of 24%,
lack of insurance. The following image is obtained and regional wall motion abnormalities in the ante-
(Fig. 11-27). The most likely cause of the abnormali- rior wall, inferior wall, and apex. The LV outflow tract
ties seen here is the following: diameter measures 2.4 cm. The following data were
A. Rheumatic valve disease obtained from a dobutamine stress echocardiogram
B. Bacterial endocarditis at an infusion rate of 20 μg/kg/m:
C. Senile calcific valve disease
D. End-stage renal disease
Baseline Dobutamine
Ejection fraction (%) 24 38
LV outflow tract velocity 0.7 1.2
(m/s)
Aortic maximum velocity 3.6 4.4
(m/s)
Mean aortic gradient 32 38
(mmHg)
  

Calculate the aortic valve area (cm2) at baseline and


during dobutamine stress:
Aortic valve area _________ __________
Question 13
The findings in Question 12 are most consistent with:
A. Lack contractile reserve
B. Severe aortic stenosis
C. Moderate aortic stenosis
D. Inadequate test

Figure 11-27 
232 CHAPTER 11  Valvular Stenosis

ANSWERS

Answer 1 variation in peak aortic flow trajectory from per-


1. B—Aortic sclerosis, or “senile calcification,” is a son to person, several views are needed to ensure
slowly progressive process that affects the aortic valve. that the maximum aortic jet velocity is recorded.
In more prominent cases, calcification can lead to Typically, peak aortic jet flow is obtained from the
aortic stenosis with hemodynamic obstruction of aor- suprasternal notch or right parasternal window,
tic outflow. Most commonly, sclerotic changes of the or from the apical 4-chamber view. For both the
aortic valve occur at the base of the cusps, initially in subcostal 4-chamber view and parasternal long-
the noncoronary cusp with progressive involvement axis view, flow across the aortic valve is perpen-
of the other cusps over time. dicular to the transducer, and peak velocity will be
2. C—Rheumatic valve disease is the result of leaflet underestimated.
scarring and chronic inflammation from an autoim-
mune reaction triggered by recurrent infections with Answer 4: B
group A Streptococci. This is manifest by inflamma- Aortic valve area is calculated based on the prin-
tion along leaflet coaptation. Progression can lead to ciple of continuity of flow where stroke volume in
commissural fusion and leaflet retraction, producing a the LV outflow tract is maintained across the aor-
stenotic, triangular valve orifice. tic valve. For the stroke volume calculation, the
3. A—Ejection of blood through the LV outflow tract modal LV outflow tract systolic velocity time inte-
may be impeded in the setting of hypertrophic cardio- gral (VTI) and the modal aortic systolic VTI must
myopathy with significant subaortic obstruction. With be recorded. Additionally, the cross-­sectional area
anterior motion of the anterior mitral valve leaflet of the LV outflow tract is needed for the calcula-
and LV outflow obstruction in middle to late systole, tion. Cross-sectional area is calculated as the area
there is a relative decrease in the pressure gradient of a circle from a diameter measurement in systole
above the obstruction, below the aortic valve, lead- immediately adjacent to the insertion of the aortic
ing to early systolic closure of the aortic valve. This is valve leaflets. Transaortic mean gradient is calcu-
typically best seen with M-mode imaging of the aortic lated by tracing the CW Doppler aortic jet signal
valve from the parasternal long-axis view. with integration of the instantaneous gradients
4. D—A congenitally bicuspid aortic valve may over the systolic ejection period but is not needed
appear as two symmetrical cusps, or asymmetric cusps for calculation of valve areas.
with a raphe or ridge replacing what would have been
a commissure. On short-axis views of the aortic valve Answer 5: 1.0 cm2
in diastole, these valves may appear tricuspid, but are The pressure half time is the time required for the pres-
revealed as bicuspid in systole, with visualization of sure gradient across an obstruction to decrease to half
the raphe. of its maximal value. Velocity is squared in the Bernoulli
equation to calculate pressure gradient, so to calculate
Answer 2: D the velocity on the curve where the gradient is half the
This is a long-axis view of the LV outflow tract, taken maximum gradient, maximum velocity is divided by
at 127°. The aortic valve leaflets appear thin and 1.4 (because 1.4 is the square root of 2). In this case,
mobile but there is a thin membrane in the LV out- 2.0 m/s divided by 1.4 equals 1.4 m/s so the T1⁄2 is 225
flow tract about 1 cm below the aortic valve plane. ms. Then mitral valve area is calculated by the equation
The ascending aorta is normal caliber without evi- 220/T1⁄2, in this case 220/225 = 0.98 cm2. Valve area
dence of supravalvular stenosis. Subaortic obstruc- calculations are only accurate to one decimal point so
tion significantly increases afterload on the LV and this calculation should be rounded up and reported at
there is compensatory concentric LV hypertrophy. 1.0 cm2, consistent with severe mitral stenosis.
Hypertrophic cardiomyopathy typically appears as
asymmetric septal thickening; there is often accom- Answer 6: A
panying systolic anterior motion of the mitral sub- This is an M-mode tracing through the mitral valve
valvular apparatus and valve which may contribute leaflets. In this view, the pulmonic and tricuspid valves
further to a systolic subaortic gradient. are not visualized. At this level, the aortic valve is not
seen. The aortic valve would be seen if the M-mode
Answer 3: C ultrasound beam were moved closer to the base of the
To identify the peak aortic jet velocity, the transducer heart. Mitral valve motion is shown against time on
must be optimally aligned parallel to blood flow. the horizontal axis. Just following systole, the mitral
As described in the Doppler equation, nonparal- valve is closed in the midportion of the image. There
lel alignment of the ultrasound beam with flow is only a small opening of the anterior and posterior
will underestimate peak velocity. Because of slight mitral valve leaflet tips in diastole (arrows) (Fig. 11-28).
Valvular Stenosis  CHAPTER 11 233

0
the future. For the repeat study, Doppler interroga-
tion of the aortic valve should be taken from multiple
views such as apical, suprasternal, high right paraster-
nal and subcostal views, with careful patient position-
ing ensuring careful transducer angulation to align
the beam parallel with flow. On the Doppler record-
ing from the question above, taken from the apical
window, a peak velocity of 3.3 m/s suggested only
moderate range stenosis, but is incongruent with the
prior TTE results and with patient symptoms. In this
case, a higher velocity (>4.3 m/s) was obtained from a
right parasternal window consistent with severe aortic
stenosis.
Answer 9: D
180
This is a transmitral Doppler tracing taken from
Figure 11-28 
an apical window with flow directed towards the
transducer during diastole. The Doppler pattern
is consistent with atrioventricular valve inflow and
the prolonged diastolic deceleration slope consistent
Answer 7: C with mitral stenosis. Obstruction of LV inflow leads
Tricuspid valve stenosis is nearly uniformly caused by to pulmonary hypertension and volume overload,
rheumatic valve disease. The patient is in atrial fibril- resulting in dilation of the inferior vena cava. Holo-
lation with variability in the R to R interval. With a diastolic flow reversal (upstream reversal of flow)
shorter cardiac cycle, less time is spent in diastole, and in the abdominal aorta would be seen with severe
diastolic LV filling is completed in a shorter inter- aortic regurgitation. Similarly, pulmonary vein sys-
val. For these shorter cardiac cycles, the peak early tolic reversal would be seen in the setting of signifi-
inflow velocity is higher than in Doppler signals with cant mitral regurgitation. Low septal tissue Doppler
longer cardiac cycle duration. Because mean gradi- velocity is seen in patients with decreased ventricu-
ent averages the instantaneous gradients over the lar compliance and restrictive LV filling. In this case,
flow duration, the mean gradient with be higher on the increased diastolic transmitral mean gradient is
shorter cycle lengths and lower on long cycle lengths. elevated due to inflow obstruction, not to diastolic
In clinical practice, when significant variation in heart dysfunction.
rate is present, any measurements of peak and mean
gradients are averaged over several cardiac cycles. For Answer 10
mitral or tricuspid stenosis, mean gradients are more The data provide the following information:
representative of stenosis severity than peak gradients.
Answer 8: B LVOT tract diameter 1.96 cm

The original echo report describes normal systolic LVOT peak velocity 1.5 m/s
function and aortic stenosis, with a calcified, immobile LVOT velocity time integral 32 cm
valve. The current aortic Doppler data are incongru-
Aortic valve velocity 4.4 m/s
ent as they are lower than the peak velocity obtained
on the prior study. Given progression in the patient’s Aortic valve velocity time integral 97 cm
symptoms, a repeat transthoracic study is indicated.
TEE is helpful for visualization of valve anatomy and The first step is to calculate the outflow tract volume
allows planimetry of valve area, but Doppler data flow rate:
obtained by TEE are suboptimal as it is difficult to
align the transducer with the jet due to transducer CSALVOT = π(LVOTD /2)2 = 3.14(2.0/2)2 
position constraints in the esophagus. Dobutamine = 3.1 cm2 (11-19)
stress echocardiography may aid in differentiating
low-gradient severe aortic stenosis, but systolic func- Transaortic stroke volume (SV)
tion was preserved on the prior study, making interval = CSALVOT × VTILVOT
development of significant LV dysfunction less likely. = 3.1 cm2 × 32 cm = 99 cm3 = 99 mL (11-20)
The yield of coronary angiography is low at this time
to determine etiology of symptoms given the negative
myocardial stress perfusion study, but would be appro- Cardiac output is SV × heart rate (60 bpm)
priate prior to aortic valve replacement if planned in = 5940 mL/min = 5.94 L/m  (11-21)
234 CHAPTER 11  Valvular Stenosis

The aortic jet is examined from both apical and Answer 12


high right parasternal windows with the highest jet
velocity representing the most parallel intercept angle CSALVOT = π(LVOTD /2)2 = 3.14(2.4/2)2
between the jet and ultrasound beam. The highest = 4.5 cm2 (11-24)
velocity signal is used to measure the velocity time
integral (VTI).
Continuity equation aortic valve area is: Baseline AVA
= CSA
( LVOT × VLVOT /V) AV
= 4.5 cm2 × 0.7 m/s /3.6 m/s
AVA = SV/VTIAS − Jet = 99 cm3 /97 cm
= 0.9 cm2 (11-25)
= 1.0 cm2  (11-22)

The dimensionless index (velocity ratio) is: Dobutamine AVA


= CSA
( LVOT × VLVOT /V
) AV
= 4.5cm2 × 1.2m/s /4.4m/s 
VLVOT /Vmax = 1.1/4.4 = 0.25 (11-23) = 1.2 cm2 (11-26)
In this case, peak aortic jet velocity is greater than
4 m/s (4.4 m/s), consistent with severe aortic stenosis. Answer 13: C
The dimensionless index and aortic valve area also This patient has low-output low-gradient aortic ste-
support a diagnosis of severe stenosis. nosis. In this patient, there is a contractile response to
dobutamine with an increase in stroke volume (LV out-
Answer 11: A flow tract velocity increased from 0.7 m/s to 1.2 m/s)
This parasternal long-axis image shows thickening of and an increase in ejection fraction from 24% to 38%.
both the aortic and mitral valves. There is diastolic There is a concurrent increase in both stroke volume
doming of the anterior mitral valve leaflet, with a and aortic jet velocity, and calculated aortic valve area
“hockey stick” appearance, and the LA is severely increased from 0.9 cm2 to 1.2 cm2. These findings are
enlarged. The aortic valve is thickened with leaflet tip consistent wtih LV systolic dysfunction and moderate
retraction at the coaptation. Atrial fibrillation can be aortic stenosis. In this situation, inadequate forward
a presenting symptom with rheumatic valve disease. stroke volume to fully open the thickened, calcified
In the United States, many patients initially present leaflets in systole may lead to an apparently small aor-
at age 50 to 60 years, with about 80% of cases occur- tic valve orifice area at rest. However, following dobu-
ring in women. In immigrants from countries with a tamine infusion, increases in forward stroke volume
higher prevalence of rheumatic fever, valve disease result in greater leaflet opening and a larger valve
presents at a younger age. Endocarditis results in val- area calculation. A lack of contractile reserve would
vular vegetations, leaflet destruction, and abscess for- lead to no change in cardiac output (ejection fraction)
mation (not commissural fusion). Calcific aortic valve or stroke volume with dobutamine, not the situation
disease affects the body of the leaflet, not the leaflet in this case. The appropriate augmentation in cardiac
edges or commissures, and in this case the mitral valve output suggests this test was diagnostic (adequate). If
is also involved. End-stage renal disease is associated severe aortic stenosis were present, valve area would
with both mitral annular calcification and calcific aor- remain less than 1.0 cm2, with an increase in aortic
tic valve disease. velocity to 4 m/s or higher with dobutamine.
12 Valvular Regurgitation
BASIC PRINCIPLES Evaluate Antegrade Mitral Flow and Stenosis
Vena Contracta Evaluate the Consequences of Chronic Ventricular
Proximal Isovelocity Surface Area Volume Overload
Regurgitant Volume Evaluate other Consequences of Mitral
Regurgitant Orifice Area Regurgitation
Distal Flow Reversals PULMONIC REGURGITATION
CW Doppler Signal Step-By-Step Approach
AORTIC REGURGITATION Determine the Etiology of Regurgitation
Step-by-Step Approach Evaluate the Severity of Pulmonic Regurgitation
Determine the Etiology of Regurgitation Evaluate the Consequences of RV Volume
Determine the Severity of Regurgitation Overload
Evaluate Antegrade Aortic Flow and Stenosis TRICUSPID REGURGITATION
Evaluate the Consequences of Chronic LV Pressure Step-By-Step Approach
and Volume Overload Evaluate the Etiology of Tricuspid Regurgitation
MITRAL REGURGITATION Evaluate the Severity of Tricuspid Regurgitation
Step-By-Step Approach Evaluate the Consequences of RV Volume Overload
Determine the Etiology of Regurgitation THE ECHO EXAM
Determine the Severity of Regurgitation SELF-ASSESSMENT QUESTIONS

BASIC PRINCIPLES n The aliasing velocity on color Doppler flow imag-


ing provides visualization of a contour where all
Vena Contracta (Fig. 12-1) the blood cells have the same velocity (isovelocity)
n 
Narrowest width of the regurgitant jet, measured (Fig. 12-2).
using color Doppler flow imaging. n The shape of this proximal isovelocity contour
typically is a hemisphere so that the cross-sectional
v  KEY POINTS area (CSA) of this surface is 2πr2.
Optimal color flow images show flow accelera-
o
tion proximal to the regurgitant valve and distal
jet expansion in the receiving chamber, with the
vena contracta being the narrow neck between
them.
o Vena contracta measurements are most accu- PISA
rate with:
The flow
o signal in the near field of the image (e.g.,
transthoracic parasternal long-axis views) Regurgitant
orifice
A narrow sector width to optimize frame rate
o
o Zoom mode to increase image size
Small differences in vena contracta width cor-
o
respond to substantial changes in regurgitant
severity grade so that if a precise and accurate
measurement is not possible, other approaches Vena
should be used. contracta
o Three-dimensional (3D) imaging of vena
Figure 12-1  Fluid dynamics of valve regurgitation. Streamlines of
contracta area is a promising new approach blood flow are shown in red. Flow proximal to the regurgitant orifice in-
but currently limited by low temporal creases in velocity as the flow stream narrows into the regurgitant orifice.
resolution. A proximal isovelocity surface area (PISA) is represented by the blue line that
connects points with the same velocity on each stream line. Multiple PISA
Proximal Isovelocity Surface Area are present proximal to the orifice; the PISA seen with color flow depends
on the aliasing velocity of the color scale. The flow stream continues to nar-
n 
Blood flow accelerates proximal to a regurgitant row beyond the orifice with the narrowest point or vena contracta reflecting
orifice. regurgitant severity.

235
236 CHAPTER 12  Valvular Regurgitation

n olume flow rate is cross-sectional area times


V Regurgitant Volume (Fig. 12-3)
velocity (in this case the aliasing velocity): n  Regurgitant volume is the amount of blood that
flows backward across the valve, measured in cm3
( )
Instantaneous flow(rate ) in cm3 /s = or mL.
CSA cm2 × Vailiasing (cm/s) (12-1) n Regurgitant volume can be calculated by subtract-
ing the stroke volume (SV) across a competent
valve (forward SV) from the antegrade volume
v  KEY POINTS flow rate across the regurgitant valve (total SV):
o The proximal isovelocity surface area (PISA) is
best visualized from a window where the ultra- Regurgitant volume = Total SV − Forward SV (12-2)
sound beam is parallel to the flow direction,
typically the apical long-axis or 4-chamber view n Total SV also can be calculated by 2D or 3D echo-
for mitral regurgitation. cardiographic measurement of left ventricle (LV)
o The aliasing velocity is decreased to 30 to stroke volume.
40 cm/s in the direction of flow by shifting
the Doppler baseline, which enhances PISA v  KEY POINTS
visualization. o Transvalvular volume flow rate calculations are
o Both a narrow color sector and zoom mode are based on diameter measurements (using a cir-
used for accurate measurement. cular cross-sectional area) and the velocity-time
o PISA measures instantaneous flow rate (cm3/s). integral (VTI) of flow at that site:
PISA must be integrated over the flow period to
obtain flow volume (cm3 or mL). SV = CSA × VTI = π(D/2)2 × VTI (12-3)
o PISA may be inaccurate when the proximal
flow field is not hemispherical, so this approach Small errors in diameter measurement lead to
o
is more useful for central jets compared with large errors in calculated SV.
eccentric jets. o The largest source of error is ensuring that
o It is more difficult to visualize the PISA when diameter is measured at the same level as the
regurgitation is mild, and it is more difficult VTI recording; this is particularly problematic
to visualize a PISA for aortic, compared with for transmitral volume flow.
mitral, regurgitation. o When both aortic and mitral valves are regur-
o Identification of the valve plane by two- gitant, pulmonic valve flow rate can be used for
dimensional (2D) imaging is critical because forward stroke volume.
the PISA measurement is from the aliasing o 2D LV volumes provide total SV when image
velocity to the valve orifice. planes and endocardial definition are adequate,
but volumes may be underestimated if apical
views are foreshortened.

LV
baseline
shift PISA

Figure 12-2  Proximal isovelocity surface


area.  In an apical 4-chamber view, color Dop-
pler at this depth shows a central regurgitant
jet; this view is helpful for determining the RA
presence and etiology of regurgitation. With the LA
depth reduced and the image zoomed on the
mitral valve, the Doppler aliasing velocity is re-
duced to 30 to 40 cm/s in the direction of flow
(away from the transducer on TTE imaging) and
variance is turned off to visualize the proximal
isovelocity surgical area and vena contracta.
PISA, Proximal isovelocity surface area. (From
Otto, CM: Textbook of Clinical Echocardiogra-
phy, ed 5, Elsevier, 2013, Philadelphia.)
Valvular Regurgitation  CHAPTER 12 237

Regurgitant Orifice Area v  KEY POINTS


n Conceptually, the regurgitant orifice area (ROA) is ROA can be calculated using regurgitant SV
o
the size of the defect in the closed valve that allows calculated by any method.
valve regurgitation. o The continuous wave (CW) recording of the
n The actual anatomy of the regurgitant orifice may regurgitant jet is used to trace the VTI (Fig.
be complex, sometimes with multiple sites of back- 12-4) to go with the regurgitant volume calcula-
flow across the valve. tion from Fig. 12-3.
n The continuity equation applies to both antegrade o ROA also can be estimated using the PISA
and retrograde flow across a valve. approach by dividing the PISA instantaneous
n 
Thus, ROA can be calculated from regurgitant volume flow rate by the maximum regurgitant
stroke volume and the VTI of the regurgitant jet jet velocity:
(RJ) as:
ROA (cm2 ) = PISA (cm3 /s)/VRJ (cm/s) (12-5)
3

ROA (cm )/VTIRJ (cm) (12-4)

o The PISA-estimated ROA reflects the instan-
taneous ROA only; thus, it is most useful for
regurgitation that occurs equally throughout
the flow period.

LVOT Diam  2.00 cm


40dB 2 •/1/0/ 1
PW Depth  114mm
PW Gate  2.0mm
PW Gain  5dB

LVOT VTI = 0.32 m


PW:2MHz HPRF
.50
LV Ao m/s

LA

2.5

Site 2 Diam  2.8 cm 40dB 1 •/1/0/ 1


PW Depth  118mm
PW Gate  2.0mm
PW Gain  3dB
LV
Site 2 VTI = 0.132 m
PW:2MHz
RV

.60

LA
RA

m/s

Figure 12-3  Volume flow at two sites.  In this patient with aortic regurgitation, regurgitant volume is calculated as the difference between total stroke vol-
ume across the aortic valve and forward stroke volume across the mitral valve. The diameter (D, left) and velocity-time integral (right) for transaortic (top) and
transmitral (bottom) flow are shown. Transaortic (total) stroke volume (TSV) is LVOT cross-sectional area (CSA = πr2 = 3.14[2 cm/2]2 = 3.14 cm2) times the VTI
(TSV = 3.14 cm2 × 32 cm = 100 mL). Transmitral (forward) stroke volume (FSV) is π(2.8 cm/2)2 × 13.2 cm = 81 mL. Then regurgitant stroke volume (RSV) is
TSV − FSV or 100 mL − 81 mL = 19 mL. Regurgitant fraction is 19 mL/100 mL × 100% = 19%. These findings suggest mild regurgitation. Ao, Aorta; LVOT,
left ventricular outflow tract; VTI, velocity-time integral.
238 CHAPTER 12  Valvular Regurgitation

CW:2MHz
APEX AV AR VTI  1.81 m
Vmax  4.27 m/sec
o In clinical practice, ROA should be calculated
6.0 PK Grad  72.9 mmHg by more than one method, if possible, to ensure
validity.
Distal Flow Reversals
n The direction of blood flow distal to a regurgitant
valve is reversed from normal when regurgitation
is severe.
m/s • With severe mitral regurgitation, there is systolic flow
reversal in the pulmonary veins.
• With severe aortic regurgitation, there is holodia-
2.0
stolic flow reversal in the aorta (Fig. 12-5).
• With severe tricuspid regurgitation, there is systolic
Figure 12-4  Regurgitant orifice area calculation.  The velocity-time in- flow reversal in the hepatic veins (Fig. 12-6).
tegral of aortic regurgitant flow in the same patient shown in Figure 12-3 is n  his qualitative indicator is integrated with
T
used to calculate regurgitant orifice area (ROA) as the RSV/VTI = 18 cm3/181 other findings in classifying overall regurgitant
cm = 0.1 cm2, consistent with mild regurgitation. VTI, Velocity-time integral.
severity.
v  KEY POINTS
38dB 1 •/1/0/ 1 These findings are more specific when flow
o
PW Depth  73mm reversal is more distal (e.g., abdominal compared
PW Gate  2.5mm with thoracic aorta for aortic regurgitation) and
PW Gain  13dB
more severe (e.g., reversed versus blunted pulmo-
nary vein systolic flow in mitral regurgitation).
o F low reversal is sometimes seen even when
regurgitation is not severe—for example, in
1.0 hepatic and pulmonary veins in nonsinus
rhythms, or in the descending aorta with a pat-
ent ductus arteriosus.
o Flow reversal is best detected with low wall filter
m/s settings, gain reduced to avoid channel cross-
talk and with the scale adjusted to the velocity
range of interest.
o Normal patterns of flow sometimes are mis-
taken for flow reversal.
1.5
In the
o descending aorta, early diastolic flow rever-
sal is normal.
Figure 12-5  Holodiastolic flow reversal in the descending thoracic aorta.  In the hepatic veins, the atrial reversal can be
o
In this patient with moderate to severe aortic regurgitation, a suprasternal prominent and may appear to extend into early
notch window was used to record pulsed Doppler flow in the descending tho- systole.
racic aorta. Holodiastolic flow reversal (arrows) also may be seen with other
causes of diastolic flow exiting the proximal aorta, including a patent ductus
arteriosus or a large arteriovenous fistula in an upper extremity.

38dB 3 •/1/0/ 1
PW Depth  87mm
PW Gate  2.0mm
PW Gain  7dB

HV
1.0
RA
Figure 12-6  Hepatic vein systolic flow
reversal. The hepatic vein flow signal, re- m/s
corded from the subcostal window in the IVC
central hepatic vein (left), shows flow toward
the transducer in systole (right, arrow), also
called systolic flow reversal, when severe tri-
cuspid regurgitation is present. HV, Hepatic
vein; IVC, inferior vena cava. 1.5
Valvular Regurgitation  CHAPTER 12 239

o Recordings are enhanced using gray scale spec-


Continuous Wave Doppler Signal tral analysis with the velocity scale adjusted to
(Fig. 12-7) the range of interest, the wall filters increased
n  The shape of the CW Doppler signal reflects the to improve the signal-to-noise ratio, and gains
instantaneous pressure differences between the lowered to avoid overestimation of velocities.
two chambers.
n The density of the CW Doppler signal, relative to
antegrade flow, reflects the volume of regurgitant AORTIC REGURGITATION
flow. Step-by-Step Approach
v  KEY POINTS Step 1: Determine the Etiology
The diastolic deceleration slope (or pressure half-
o
of Regurgitation
time) of the aortic regurgitant signal is steeper n Aortic regurgitation is due either to disease of the
(shorter) with more severe aortic regurgitation. valve leaflets or abnormalities of the aortic root
o A late systolic decline in velocity with mitral regur- (Fig. 12-8).
gitation reflects a rise in LA pressure, suggestive of n Primary causes of aortic leaflet dysfunction include
a v-wave. bicuspid valve, rheumatic disease, endocarditis, cal-
o Care is needed to ensure the Doppler recording cific disease, and some systemic diseases (Table 12-1).
is made with the ultrasound beam parallel to n 
Aortic root enlargement resulting in aortic regur-
the direction of the regurgitant jet at the vena gitation may be due to Marfan syndrome, familial
contracta. aortic aneurysm, hypertension, or aortic dissection.
o Optimal CW recordings of the regurgitant jets
show a smooth velocity curve with a dense sig- v  KEY POINTS
nal along the outer edge of the spectral signal. o Long- and short-axis images of the aortic
valve allow identification of a bicuspid aortic
valve (two leaflets in systole), rheumatic disease
(commissural fusion), vegetations, and calcific
changes (Fig. 12-9).
100 o Leaflet perforation or fenestration cannot be
visualized but is inferred from the location of the
regurgitant jet orifice identified by color Doppler.
Pressure (mmHg)

Ao o When aortic regurgitation is more than mild,


Doppler flow in the aorta should be measured
at several sites as detailed in Chapter 16. The
transducer is moved cephalad to visualize the
ascending aorta.
o Marfan syndrome is characterized by loss of the
LV normal acute angle at the sinotubular junction.
0 o Systemic inflammatory diseases associated with
AR cause dilation of the aorta and thickening
of the posterior aortic root extending onto the
AR base of the anterior mitral leaflet.
Step 2: Determine the Severity
of Regurgitation
n Regurgitant severity is evaluated using a stepwise
Velocity (m/s)

approach with integration of several types of data.


n 
In addition to Doppler measures of regurgitant
Aortic severity, the cause of regurgitation, and LV size
outflow and systolic function are important parameters in
clinical decision making.

Figure 12-7  Regurgitation hemodynamics.  The shape of the CW Dop- Step 2A: Measurement of vena contracta width is the
pler velocity curve reflects the instantaneous pressure differences between initial step in evaluation of aortic regurgitation (Fig. 12-10)
the aorta and LV in diastole, with the relationship between LV and aortic
pressures (top) and Doppler velocities (bottom) shown for chronic (green) v  KEY POINTS
and acute (blue) aortic regurgitation. Ao, Aorta; AR, aortic regurgitation.
(From Otto, CM: Textbook of Clinical Echocardiography, ed 5, Elsevier, 2013, o With aortic regurgitation, vena contracta usu-
Philadelphia.) ally is best measured in the parasternal long-axis
240 CHAPTER 12  Valvular Regurgitation

FR 18Hz
10cm
2D
71%
C 50
P Off
Res
CF
59%
3.3MHz LA
WF Med
Low

Ao

LV

A
FR 19Hz
12cm
2D
70%
C 50
P Off
Gen
CF
59%
3.3MHz
WF Med
Low

B
Figure 12-8  Aortic regurgitation.  A, TEE imaging shows an eccentric aortic regurgitant jet with a vena contracta of 5 mm. B, Short-axis views shows
noncoaptation of a trileaflet valve due to dilation of the aortic sinuses resulting in aortic regurgitation. Ao, Aorta.

view on transthoracic echocardiography (TTE) Step 2B: Evaluation of diastolic flow reversal in the
or the long-axis view at about 120° rotation on descending aorta is a simple, reliable approach
transesophageal echocardiography (TEE). to evaluation of aortic regurgitant severity
o Vena contracta is measured as the smallest
width of the jet, taking care with eccentric jets v  KEY POINTS
to avoid an oblique diameter measurement. 
oHolodiastolic flow reversal in the proximal
o A vena contracta width less than 0.3 cm indi- abdominal aorta is highly specific for severe
cates mild regurgitation; a vena contracta aortic regurgitation (Fig. 12-11).
width greater than 0.6 cm indicates severe o Holodiastolic flow reversal in the descending
regurgitation. thoracic aorta is seen in some patients with
o Further evaluation is needed when vena moderate aortic regurgitation, as well as those
contracta is 0.3 to 0.6 cm, when images of with severe aortic regurgitation.
the vena contracta are suboptimal, or when o Early diastolic flow reversal in the descending
further quantitation is needed for clinical aorta is normal and should not be mistaken for
decision-making. aortic regurgitation.
TABLE 12-1 Aortic Regurgitation: Clinical Echocardiographic Correlation
Chronic Primary AR Chronic AR due to Aortic Disease Acute AR
Causes Bicuspid aortic valve Marfan syndrome Endocarditis
(examples) Rheumatic valve disease Familial aortic aneurysm Aortic dissection
Calcific valve disease Hypertensive disease Blunt chest trauma
Systemic inflammatory Systemic inflammatory diseases
­diseases
Clinical Asymptomatic diastolic Diastolic murmur on exam New onset heart failure
presentation ­murmur or AR on echocardiography in a Pulmonary edema
and disease Slow disease progression patient with aortic disease Cardiogenic shock
course over many years leads to
dyspnea and decreased
exercise capacity
LV response Progressive severe LV LV dilation depending on AR Normal LV size with normal EF
­dilation severity. Severely elevated LV filling
Some develop irreversible pressures
contractile dysfunction Reduced forward ­cardiac
­without symptoms output
EF remains normal until
late in disease course
but is not an accurate
marker for ­myocardial
­dysfunction.
Valve anatomy Bicuspid aortic valve with Normal aortic leaflet anatomy with Perforated or flail aortic valve
two leaflets stretched leaflets and central leaflet with endocarditis and
Commisural fusion and ­ regurgitation valve destruction.
mitral valve involvement Dilation aortic sinuses or Paravalvular abscess may be
with rheumatic disease ascending aorta present.
Concurrent stenosis with Effacement of the sinotubular Aortic dissection flap resulting
calcific disease junction is typical for Marfan in flail leaflet, undermining of
­syndrome commisural support or distor-
tion of valve anatomy
Key Doppler Vena contracta Vena contracta ­measurement Color Doppler with wide vena
findings ­measurement CW Doppler signal contracta
CW Doppler signal Flow reversal in the ­descending Dense CW Doppler signal with
Flow reversal in the aorta steep deceleration slope
­descending aorta Holodiastolic flow reversal in
the descending aorta
Definition of Vena contracta width Vena contracta width > 0.6 cm Qualitative signs of severe AR
severe AR > 0.6 cm Holodiastolic flow reversal in the are adequate for clinical deci-
Holodiastolic flow reversal proximal abdominal aorta sion making because therapy
in the proximal abdominal Regurgitant volume ≥ 60 mL is directed at the underlying
aorta Regurgitant fraction ≥ 50% disease process
Regurgitant volume ≥ 60 mL Regurgitant orifice area ≥ 0.3 cm2
Regurgitant fraction ≥ 50%
Regurgitant orifice area
≥ 0.3 cm2
Indications for Symptom onset Timing of intervention often depends Urgent surgical intervention for
­intervention LV-ESD > 50 mm on the severity of aortic dilation ascending aortic dissection
with severe LV-EF < 50% and cause of disease, rather than Early surgery for endocarditis
AR AR severity complicated by acute severe AR
Options for Surgical aortic valve Replacement of the aortic sinuses Aortic root replacement for
­intervention replacement and ascending aorta with “sparing” aortic dissection.
Aortic valve repair may be of the aortic valve which is In some cases, the valve can
possible in selected positioned inside the prosthetic be resuspended at the
cases conduit (David procedure) sinotubular junction.
Bentall procedure with a prosthetic Aortic valve replacement
valve-conduit replacement of the
aortic valve and root (with coronary
reimplantation)

From Otto CM: Textbook of Clinical Echocardiography, ed 5. Philadelphia, Elsevier, 2013.


AR, Aortic regurgitation; CW, continuous wave; EF, ejection fraction; ESD, end-systolic dimension; MR, mitral regurgitation.
*Major accepted indications for intervention; guidelines should be consulted for other indications and more details.
242 CHAPTER 12  Valvular Regurgitation

Systole Diastole

Figure 12-9  3D aortic valve imaging.  In the same patient as Fig. 12-8, 3D imaging shows a trileaflet valve in systole (left) with noncoaptation of the leaflets
in diastole (right).

Ao LV Ao

LV

LA

A B
Figure 12-10  Aortic regurgitation vena contracta.  Examples of measuring vena contracta width with a centrally (left) and eccentrically (right) directed jet
of aortic regurgitation. Vena contracta width in aortic regurgitation is best recorded in the parasternal long-axis view using zoom mode to focus on the aortic
valve. The narrowest width of the regurgitant jet is measured, ideally with the proximal flow acceleration and distal jet expansion regions seen. Vena contracta
width is measured perpendicular to the jet direction; with an eccentrically directed jet, this measurement is not perpendicular to the LV outflow tract. Ao, Aorta.

o If holodiastolic aortic flow reversal is seen but In cases with an eccentric posteriorly directed
o
there is no color Doppler evidence of severe aortic regurgitant jet, the best intercept angle
aortic regurgitation, evaluate for a patent duc- may be obtained from the parasternal window.
tus arteriosus, which also causes aortic diastolic o On TEE a transgastric apical view may allow
flow reversal due to flow from the aorta into the recording of the aortic regurgitant jet, but it
pulmonary artery. may not be possible to obtain a parallel inter-
cept angle using TEE.
Step 2C: CW Doppler evaluation of aortic regurgitation o The density of the velocity signal compared
is a standard part of the evaluation with the density of the antegrade signal provides
a qualitative measure of regurgitant severity
v  KEY POINTS (Fig. 12-12).
o Aortic regurgitation usually is best recorded from o In general, a steep diastolic deceleration slope
an apical approach using CW Doppler because (pressure half-time < 200 ms) is consistent with
this window allows parallel alignment between severe regurgitation, whereas a flat slope (>500
the ultrasound beam and regurgitant jet. ms) indicates mild regurgitation. However, some
Valvular Regurgitation  CHAPTER 12 243

APX AV AR Slope  2.4 m/sec2


38dB 1 •/1/0/ 1
8.0 P½ Time  528 msec
PW Depth  101mm Decel Time  1820 msec
PW Gate  2.5mm
PW Gain  13dB

1.5

m/s

4.0
m/s

.50 Figure 12-12  CW Doppler evaluation of aortic regurgitation.  The CW


Doppler signal provides information on (1) the velocity of antegrade flow
Figure 12-11  Proximal abdominal aorta holodiastolic flow rever- (v) reflecting both the volume of flow and coexisting valve stenosis, (2) the
sal.  With severe aortic regurgitation, flow in the proximal abdominal aorta, relative density of the regurgitant signal compared with the density of an-
recorded from the subcostal window, shows antegrade flow in systole with tegrade flow, and (3) the time course of the velocity signal. In this example,
retrograde flow throughout diastole (arrows), reflecting severe backflow the diastolic slope is 2.4 m/s2, with a pressure half-time of 528 ms, and the
across the aortic valve. signal is only slightly less dense than antegrade flow; both these features
are consistent with moderate regurgitation. The systolic velocity indicates
concurrent aortic stenosis.

patients with compensated severe regurgitation increased antegrade volume flow rate across the
have a long pressure half-time. aortic valve in systole.
o Pressure half-time is measured using the same n Thus, in addition to velocity and mean pressure
approach as measurement of pressure half-time gradient, aortic valve area should be calculated
in mitral stenosis (see Chapter 11). using the continuity equation as described in
Chapter 11.
Step 2D: When further quantitation is needed,
regurgitant volume and orifice area can be calculated Step 4: Evaluate the Consequences
of Chronic LV Pressure and Volume Overload
v  KEY POINTS n The LV dilates in response to the chronic load
The most common approach is to calculate
o imposed by aortic regurgitation with the extent of
total stroke volume across the aortic valve and LV dilation reflecting the severity of regurgitation
then subtract forward stroke volume (calculated (Fig. 12-13).
across the mitral or pulmonic valve) to deter- n 
Some patients develop irreversible LV dysfunc-
mine regurgitant volume. tion in the absence of symptoms so that the most
o Regurgitant orifice area is calculated by divid- important parameters to measure on echocardiog-
ing regurgitant volume by the VTI of the CW raphy in patients with chronic severe aortic regur-
aortic regurgitation velocity curve. gitation are LV size and ejection fraction.
o The PISA is often difficult to visualize with aor-
tic regurgitation. v  KEY POINTS
o Methods to calculate regurgitant volume LV end-diastolic and end-systolic dimensions,
o
based on antegrade and retrograde flow in the volumes, and ejection fraction are key measure-
descending aorta have been described but are ments, with direct side-by-side comparison to
not routinely used. previous examinations.
o Guidelines recommend M-mode ventricular
Step 3: Evaluate Antegrade Aortic Flow dimension measurements because of better
and Stenosis endocardial definition due to the high sampling
n Many patients with aortic regurgitation also have rate of M-mode recordings.
some degree of aortic stenosis. o When the M-line cannot be aligned perpen-
n 
However, antegrade aortic velocity is increased in dicular to the long and short axes of the LV,
patients with severe regurgitation because of the 2D measurements can be used, taking care to
244 CHAPTER 12  Valvular Regurgitation

optimally define the endocardium and to cor- mitral annular calcification, and endocarditis
rectly measure the LV minor axis at end-dias- (Fig. 12-14).
tole and end-systole. n LV dilation results in secondary mitral regurgi-
o Indexing LV dimensions and volumes to body tation due to annular dilation and malalignment
surface area is especially important in women of the papillary muscles, resulting in tethering
and smaller patients. or “tenting” of the valve leaflets in systole (Fig.
o With severe aortic regurgitation, LV volumes 12-15).
are increased in direct proportion to the n Ischemic mitral regurgitation may be due to pap-
regurgitant volume; the stroke volume calcu- illary muscle dysfunction, regional dysfunction of
lated using the biplane apical approach is the the inferior-lateral wall, or diffuse LV dysfunction
total stroke volume (forward SV plus regurgi- and dilation.
tant SV). n Mitral regurgitation may be intermittent if

o The LV becomes more spherical in aortic reversible ischemia results in inadequate leaflet
regurgitation patients, so it is especially impor- closure.
tant to ensure that LV dimensions are measured
at the same position on sequential examinations v  KEY POINTS
in each patient. o Mitral valve anatomy is evaluated in multiple
2D and 3D TTE views, including long-axis,
short-axis, and 4-chamber image planes (Fig.
MITRAL REGURGITATION 12-16). If better definition of valve anatomy is
Step-By-Step Approach needed for clinical decision-making, 3D TEE
imaging is recommended (Fig. 12-17).
Step 1: Determine the Etiology o Imaging of the mitral valve allows identification
of Regurgitation of myomatous valve disease, rheumatic disease
n  Mitral regurgitation may be primary (due to (commissural fusion), vegetations, and calcific
abnormalities of the valve leaflets and chordae) changes.
or secondary (due to LV dilation or dysfunction o With myxomatous mitral valve disease, the
with normal leaflets) (Table 12-2). degree of thickening, redundancy, and prolapse
n Primary causes of mitral leaflet and chordal dys- of each leaflet is described.
function include myxomatous mitral valve dis- o The tip of a flail leaflet segment points toward
ease (mitral valve prolapse), rheumatic disease, the roof of the LA in systole; a severely pro-
lapsing segment is curved so that the tip points
toward the LV apex (Fig. 12-18).
o Restricted leaflet motion is characteristic of sec-
ondary mitral regurgitation. The area defined
by the tented leaflets and the annular plane at
end-systole provides an index of the severity of
restricted motion.
o 3D TEE is especially useful for evaluation of
prolapse and chordal rupture in patients with
myxomatous mitral valve disease.
LV
Step 2: Determine the Severity
RV of Regurgitation
n Regurgitant severity is evaluated using a stepwise
approach with integration of several types of data.
RA n 
In addition to Doppler measures of regurgitant
LA
severity, the cause of regurgitation, LV size and sys-
tolic function, LA size, and pulmonary pressures are
important parameters in clinical decision-making.

Step 2A: Measurement of vena contracta width is the


initial step in evaluation of mitral regurgitation
(Fig. 12-19)
v  KEY POINTS
Figure 12-13  LV shape changes with aortic regurgitation.  In this pa-
tient with severe aortic regurgitation, the apical 4-chamber view shows a o 
With mitral regurgitation, vena contracta
dilated LV with increased sphericity (rounded shape of the LV apex). usually is best measured in the parasternal
Valvular Regurgitation  CHAPTER 12 245

TABLE 12-2 Mitral Regurgitation: Clinical Echocardiographic Correlation


Chronic Primary MR Chronic Secondary MR Acute MR
Causes Mitral valve prolapse Dilated cardiomyopathy Endocarditis
­(examples) Rheumatic valve disease Chronic ischemic disease Chordal rupture
Papillary muscle rupture or
dysfunction
Clinical Asymptomatic systolic murmur Systolic murmur on exam and New onset heart failure
­presentation Slow disease progression over MR on echocardiography in Pulmonary edema
and disease many years leads to dyspnea and a patient with chronic heart Cardiogenic shock
course decreased exercise capacity failure
LV response Mild LV dilation LV dilation and dysfunction are Normal LV size with n ­ ormal
Some develop irreversible contrac- due to the underlying disease EF
tile dysfunction without symptoms process Elevated LV filling pressures
EF remains normal until late in Evaluation of dynamic ­mitral Reduced forward cardiac
disease course but is not an valve anatomy allows output
accurate marker for myocardial diagnosis of ­secondary MR
dysfunction
Valve anatomy Typical findings for mitral ­prolapse Tethering of mitral leaflets Mitral leaflet perforation or
or other cause of MR Annular dilation inadequate coaptation due to
TEE provides improved image Normal leaflet thickness and valve destruction
quality anatomy Flail chord or leaflet ­segment
3D imaging is helpful in most cases TEE may be needed for
diagnosis
Key Doppler Vena contracta measurement Vena contracta ­measurement Color Doppler with wide vena
findings CW Doppler signal CW Doppler signal contracta
Quantitation of regurgitant ­severity Quantitation of regurgitant Dense CW Doppler signal with
Calculation of PA systolic ­pressure severity “v-wave”
(exercise testing may be needed) Calculation of PA systolic
pressure
Definition of Vena contracta width ≥ 0.7 cm PISA derived regurgitant Qualitative signs of severe
severe MR Regurgitant volume ≥ 60 mL orifice area ≥ 0.2 cm2 MR are adequate for clinical
Regurgitant fraction ≥ 50% Regurgitant volume ≥ 30 mL decision making
Regurgitant orifice area Regurgitant fraction ≥ 50%
≥ 0.4 cm2
Indications for Symptom onset Persistent severe MR ­causing Urgent surgical intervention
intervention LV-ESD ≥ 40 mm symptoms ­despite optimal usually is needed, except for
with severe LV-EF ≤ 60% heart failure therapy some coronary disease cases
MR* Valve repairability affects timing of where revascularization may
intervention decrease MR severity
Options for Surgical mitral valve repair Medical therapy for heart Treat underlying disease
intervention ­(preferred) failure. process (e.g., coronary
Mitral valve replacement Cardiac resynchronization ­disease, endocarditis)
therapy. Stabilize in ICU with IABP
Transcatheter mitral clip† Urgent surgical valve repair
Mitral annuloplasty ring or replacement

From Otto CM: Textbook of Clinical Echocardiography, ed 5. Philadelphia, Elsevier, 2013.


AR, Aortic regurgitation; EF, ejection fraction; ESD, end-systolic dimension; IABP, intraaortic balloon pump; ICU, intensive care unit; MR, mitral regur-
gitation; PA, pulmonary artery; PISA, proximal isovelocity surface area.
*Major accepted indications for intervention; guidelines should be consulted for other indications and more details.
†Investigational device in the United States.

long-axis view on TTE or the long-axis view at distal jet expansion should be seen to ensure the
about 120° rotation on TEE. narrowest segment of the jet is measured.
o 
Vena contracta is measured as the smallest A vena
o contracta width less than 0.3 cm indi-
width of the jet, taking care with eccentric jets cates mild regurgitation; a vena contracta
to avoid an oblique diameter measurement. width of 0.7 cm or greater indicates severe
Both the proximal acceleration region and the regurgitation.
246 CHAPTER 12  Valvular Regurgitation

HR  83bpm

Ao

LV

LA

Figure 12-14  Primary mitral regurgita-


tion.  In a parasternal long-axis view, myxo-
matous mitral valve disease is present with
posterior leaflet prolapse and mitral regurgi-
tation on this end-systolic frame. Ao, Aorta.

Dist  0.60cm

Ao

LV

LA

Figure 12-15  Secondary mitral regurgi-


tation.  In a parasternal long-axis view in a
patient with a dilated cardiomyopathy, nor-
mal mitral leaflets with “tethering” of leaflet
closure results in mitral regurgitation. Ao,
Aorta.

Further
o evaluation is needed when the vena con-
tracta is between 0.3 and 0.7 cm, when images of v  KEY POINTS
the vena contracta are suboptimal, or when further 
oWith holosystolic regurgitation and a cen-
quantitation is needed for clinical decision-making. tral jet, the PISA approach to quantitation
of severity is appropriate; use of the PISA
Step 2B: Evaluation of jet direction determines the next approach with late systolic regurgitation or an
step in evaluation of mitral regurgitant severity eccentric jet is problematic.
n  Central jets typically are seen with secondary o With an eccentric jet or late systolic regurgita-
mitral regurgitation due to LV dilation. tion, pulsed Doppler measurement of regurgi-
n Ischemic mitral regurgitation often results in an tant volume and orifice area is appropriate.
eccentric posteriorly directed jet. o The duration of regurgitation in systole can
n 
Mitral valve prolapse often results in an eccentric be visualized on frame-by-frame review of the
regurgitant jet with the jet directed away from the cardiac cycle or can be inferred from the CW
affected leaflet (Fig. 12-20). Doppler mitral regurgitation jet signal.
Valvular Regurgitation  CHAPTER 12 247

2D
62%
C 50
P Low Ao
HPen
CF
62% LV
2.5MHz
WF High
Med
LA

A
+38.5

2D
78%
C 51
P Off LA
Res
CF Figure 12-16  Comparison of TTE and TEE
59% evaluation of mitral regurgitation.  A, on
3.3MHz TTE imaging, posterior mitral leaflet prolapse
WF High and a possible flail segment (arrow) are seen
Low
with color Doppler showing anteriorly direct-
ed mitral regurgitation (arrow) with a wide
vena contracta. B, TEE in a long-axis view
shows severe prolapse of the posterior mitral
LV leaflet (arrow) with severe mitral regurgitation
with a wide vena contracta and large proxi-
mal isovelocity surface area with the aliasing
velocity set to 38.5 cm/s in the direction of
B flow. Ao, Aorta.

Step 2C: When clinically indicated, RV and ROA are


calculated
v  KEY POINTS
o With late systolic mitral regurgitation (MR)
or eccentric jets, total stroke volume is cal-
A culated across the mitral valve (SVMR), and
LAA then forward stroke volume (calculated across
the LV outflow tract [SVLVOT] or pulmonic
valve) is subtracted to obtain regurgitant vol-
P2
ume (Fig. 12-21):

RVMR = SVMR − SVLVOT (12-6)

Figure 12-17  3D TEE imaging of a flail posterior mitral leaflet. In o The 2D biplane total LV stroke volume (SV2D)
the same patient as in Figure 12-16, 3D imaging shows ruptured chords
(arrows) prolapsing into the LA in systole at the lateral aspect of the central
can be used instead of transmitral flow to calcu-
posterior leaflet scallop (P2). The mitral valve is viewed from the perspective late regurgitant volume:
of the LA with the anterior (A) mitral leaflet shown superiorly and the LA ap-
pendage at the left of the screen. LAA, Left atrial appendage. RVMR = SV2D − SVLVOT (12-7)
248 CHAPTER 12  Valvular Regurgitation

LV

LA

Ao

LV Ao
Prolapse

LA
Flail

Figure 12-18  Schematic of prolapse versus flail leaflet. The term


prolapse of the mitral leaflet indicates that the chordal connections of the
leaflet to the papillary muscle are intact so that, regardless of the severity of Figure 12-20  Eccentric mitral regurgitant jet on TEE.  Color Doppler in
prolapse, the tip of the leaflet still points toward the LV apex. With chordal a long-axis view shows an eccentric anteriorly directed mitral regurgitant jet.
rupture, the mitral leaflet segment becomes “flail” and the tip of the flail seg- With myxomatous mitral valve disease, the direction of the jet typically is op-
ment points toward the roof of the LA. Ao, Aorta. (From Otto, CM: Textbook of posite the affected leaflet. This anteriorly directed jet confirms that regurgi-
Clinical Echocardiography, ed 5, Elsevier, 2013, Philadelphia.) tation is primarily due to involvement of the posterior valve leaflet. Ao, Aorta.

FR 17Hz
11cm
2D
80%
C 51
P Off
Res LA
CF
59%
3.3MHz
WF High
Low

Figure 12-19  Mitral regurgitation vena LV


contracta width. The vena contracta (be-
tween arrows) is the narrow neck between the
proximal acceleration on the ventricular side of
the valve and jet expansion in the LA. Measure-
ments can be made on TTE or TEE imaging.

o Regurgitant orifice area is calculated by divid- ROA = (PISA × Valiasing )/VMR (12-9)
ing regurgitant volume (RV) by the velocity time
integral (VTIMR) of the CW Doppler mitral o The regurgitant volume can be estimated using
regurgitant velocity curve during systole: the PISA method by multiplying the ROA by
the VTI of the mitral regurgitant jet:
ROA = RV/VTIMR (12-8)
RV = ROA × VTIRJ (12-10)
o The PISA approach provides instantaneous
flow rate, which is divided by the peak o The PISA images are optimized using an alias-
mitral regurgitant velocity (VMR) to estimate ing velocity of 30 to 40 cm/s. The radius (r) of
­regurgitant orifice area: the PISA is measured from the edge of the color
Valvular Regurgitation  CHAPTER 12 249

LVOT
2.0cm LVOT
VTI 12.2cm

PW:2MHz

.20

m/s

.80

MA MA
3.4cm VTI 8cm

PW:2MHz

.80

m/s

.20
Figure 12-21  Calculation of regurgitant volume in a patient with mitral regurgitation.  Regurgitant volume is calculated from the difference between
mitral (bottom) and aortic (top) volume flow rates based on measurement of annular diameters (D, left) and velocity-time integrals (VTI, right). Total (transmitral)
stroke volume (TSV) is the mitral annular cross-sectional area (CSA= πr2 = 3.41[3.4 cm/2]2 = 9.1 cm2) multiplied by the VTI (TSV = 9.1 cm2 × 8 cm = 73 mL).
Transaortic (forward) stroke volume (FSV) is π(2.0 cm/2)2 × 12.2 cm = 38 mL. Then regurgitant stroke volume (RSV) is TSV − FSV or 73 mL − 38 mL = 35 mL.
Regurgitant fraction is 35 mL/78 mL × 100% = 48%. These findings suggest moderate regurgitation. LVOT, Left ventricular outflow tract; MA, mitral annulus
VTI, velocity-time integral.

corresponding to the aliasing velocity to the level is seen in most patients with severe mitral regur-
of the closed leaflets in systole (Fig. 12-22). gitation (Fig. 12-23).
o Recording images for PISA measurement with n The density of the CW Doppler mitral regurgitant
and without color facilitates correct identifica- curve, compared with the density of antegrade
tion of the valve orifice plane. flow, indicates relative mitral regurgitant severity
o With the aliasing velocity set at about 40 cm/s (Fig. 12-24).
and assuming a maximum regurgitant veloc-
ity of 5 m/s, quick estimate of ROA can be v  KEY POINTS
obtained from the PISA radius (in cm) as r2/2. The specific location of systolic flow reversal
o
depends on jet direction, so that TEE imaging
Step 2D: Additional simple measures of regurgitant may be needed to evaluate all four pulmonary
severity include pulmonary vein systolic flow reversal veins; absence of systolic flow reversal on TTE
and the density of the CW Doppler signal does not exclude severe regurgitation.
n 
Reversal or blunting of the normal pattern of o Systolic flow reversal may be present even when
pulmonary venous inflow into the LA in systole regurgitation is not severe in patients with atrial
250 CHAPTER 12  Valvular Regurgitation

FR 23Hz FR 23Hz CW
6.9cm 6.9cm 60%
2D 2D 2.5MHz
65% 65% WF 225Hz
C 48 C 48
LA
P Off P Off
Gen Gen
CF CF 8.0
58% 57%
3.3MHz 3.3MHz 6.0
WF Med Ao WF Med
4.0
Low Low
2.0

m/s
A LV B PAT T: 37.0C
TEE T: 40.4C 75mm/s 48 bpm

Figure 12-22  Proximal isovelocity surface area (PISA) calculation of regurgitant volume and orifice area.  A, The PISA is seen in the long-axis TEE
view in this patient; the best view will vary between patients so careful adjustment of the image plane is needed for optimal visualization. Zoom mode is used
to maximize the image size of the PISA, with a velocity scale without variance and with the baseline moved so that the aliasing velocity (white arrow) in the
direction of flow is about 40 cm/s. In this example, the PISA diameter is 0.7 cm (black arrow, left panel). The instantaneous flow rate is calculated as the surface
area of the PISA (2πr2 = 2 × 3.14 × [0.7 cm]2 = 3.1 cm2) times the aliasing velocity of 40 cm/s (arrow, left panel), which equals 123 cm3/s. B, The maximum
mitral regurgitant velocity recorded at the same time is 6.8 m/s (680 cm/s) (arrow, right frame), then regurgitant orifice area is (123 cm3/s)/680 cm/s = 0.18
cm2). Note that the high mitral regurgitation velocity indicates a severely elevated systolic blood pressure in this patient with a normal aortic valve. Ao, Aorta.

PW:2MHz HPRF APEX MV

.50
D
m/s m/s

1.0

Figure 12-23  Pulmonary vein flow systolic flow reversal. Doppler


pulmonary vein flow is recorded on TTE from the apical 4-chamber view
in the right superior pulmonary vein. Signal strength often is suboptimal,
as in this example; even so, the flow into the LV in diastole (D) can be 6.0
distinguished from the systolic (S) flow reversal due to severe mitral re-
Figure 12-24  CW Doppler recording of the mitral regurgitant velocity. This
gurgitation.
signal is evaluated for (1) the velocity and deceleration curve of the antegrade
flow in diastole, (2) the relative density of the retrograde flow compared with
antegrade flow, and (3) the shape and timing of the regurgitant velocity curve. In
this example, the antegrade flow is normal velocity (<1 m/s) with a steep decel-
arrhythmias or other factors that affect normal eration curve indicating the absence of mitral stenosis. The regurgitant signal is
atrial filling patterns. almost as dense as antegrade flow and is holosystolic, consistent with severe re-
o The CW Doppler mitral regurgitant jet usu- gurgitation. In addition, the falloff in velocity in late systole suggests that LA pres-
ally is best recorded from an apical approach sure is elevated in late systole, consistent with a v-wave and acute regurgitation.
on TTE or a 4-chamber view on TEE because
these windows allow parallel alignment
between the ultrasound beam and regurgitant n  ll patients with severe mitral regurgitation have
A
jet. an elevated antegrade mitral velocity because of
o In cases with an eccentric posteriorly directed the increased antegrade volume flow rate across
regurgitant jet, the best intercept angle may the mitral valve in diastole.
be obtained from the parasternal window or n Mitral stenosis is distinguished from a high volume
­occasionally from a suprasternal approach. flow rate by the mitral pressure half-time.
Step 3: Evaluate Antegrade Mitral Flow Step 4: Evaluate the Consequences
and Stenosis of Chronic Ventricular Volume Overload
n 
Patients with rheumatic mitral regurgitation often n 
The LV dilates in response to the chronic load
have some degree of mitral stenosis. imposed by mitral regurgitation. However, the
Valvular Regurgitation  CHAPTER 12 251

extent of LV dilation is much less than seen with


aortic regurgitation because aortic regurgitation
results in pressure and volume overload, whereas
mitral regurgitation predominantly imposes a vol- RVOT
ume overload.
n The most important parameters to measure

on echocardiography in patients with chronic
severe mitral regurgitation are LV size and ejec-
tion fraction because some patients develop
irreversible LV dysfunction in the absence of PA
symptoms.
v  KEY POINTS
o LV end-diastole and end-systolic dimensions
and volumes should be measured and com-
pared side by side with previous examinations.
o Even a slight increase in systolic size is clinically
significant because the threshold for interven-
tion is close to the upper normal limit for LV
size (> 40 mm).
o Guidelines recommend M-mode ventricular
dimension measurements because of better Figure 12-25  Color Doppler image of pulmonic regurgitation. Pul-
endocardial definition due to the high sampling monic regurgitation is evaluated in the parasternal short-axis view on this
rate of M-mode recordings. diastolic image. The vena contracta is narrow, reflecting mild regurgitation.
o When the M-line cannot be aligned perpen- PA, Pulmonary artery; RVOT, right ventricular outflow tract.
dicular to the long and short axes of the LV, 2D
measurements can be used, taking care to opti-
mally define the endocardium and to correctly
measure the LV minor axis at end-diastole and
PULMONIC REGURGITATION
end-systole. Step-By-Step Approach
o LV end-diastolic and end-systolic volumes and
ejection fraction are measured using 3D imag- Step 1: Determine the Etiology
ing or the 2D apical biplane method. of Regurgitation
o With severe mitral regurgitation, LV vol- n A small amount of pulmonic regurgitation is seen
umes are increased in direct proportion to in most individuals (Fig. 12-25).
the regurgitant volume; stroke volume calcu- n 
Pathologic regurgitation most often is due to con-
lated using the biplane apical approach is the genital heart disease, such as a repaired tetralogy
total stroke volume (forward plus regurgitant of Fallot.
volume).
o Ejection fraction measurement is accurate v  KEY POINTS
in patients with mitral regurgitation. Even a Imaging the pulmonic valve is difficult in adult
o
small decline in ejection fraction has impor- patients.
tant clinical implications for optimal timing o Thickened, deformed leaflets may be seen with
of valve surgery, so precise measurement is congenital pulmonic valve disease (Fig. 12-26).
essential.
o Measurement of LV dP/dt (rate of rise in pres-
Step 2: Evaluate the Severity of Pulmonic
sure) from the mitral regurgitation jet is useful Regurgitation
(see Chapter 6). n  Vena contracta width is helpful for evaluation of
pulmonic regurgitation.
Step 5: Evaluate other Consequences n The density and shape of the CW Doppler wave-
of Mitral Regurgitation form are diagnostic (Fig. 12-27).
n LA enlargement is assessed as described in Chap- n 
Further quantitation of pulmonic regurgitation is
ter 2. rarely needed.
n Pulmonary systolic pressures are estimated as
described in Chapter 6. v  KEY POINTS
n 
RV size and systolic function are evaluated as o Pulmonic regurgitation is low velocity (if pul-
described in Chapter 6. monary diastolic pressure is normal), so the
252 CHAPTER 12  Valvular Regurgitation

color Doppler display may show uniform lami- slope that reaches the baseline at end-diastole
nar flow in diastole in the RV outflow tract (see (see Fig. 12-27, B).
Fig. 12-27, A).
o The CW Doppler curve is especially helpful Step 3: Evaluate the Consequences of Right
for detection of severe pulmonic regurgitation Ventricular Volume Overload
showing a dense signal with a steep deceleration n 
Severe pulmonic regurgitation results in RV dila-
tion and eventual systolic dysfunction.
v  KEY POINTS
43dB 2/1/0/1
CW Focus  59mm 
oEvaluation of RV size and systolic func-
CW Gain  4dB tion by echocardiography is largely based on
qualitative evaluation of 2D images, using a
scale of normal, mild, moderate, and severely
abnormal.
o Sequential studies are helpful in distinguishing
3.0 residual RV dilation or dysfunction after repair
of tetralogy of Fallot from progressive postop-
erative changes.
o Cardiac magnetic resonance imaging allows
quantitation of RV volumes and ejection
m/s fraction.

TRICUSPID REGURGITATION
2.0
Step-By-Step Approach
Figure 12-26  Continuous wave Doppler recording of pulmonic regur- Step 1: Evaluate the Etiology of Tricuspid
gitation. The signal is much less dense than antegrade flow, consistent
with mild regurgitation. A prominent pulmonic closing click (arrow) is fol-
Regurgitation
lowed by an area of signal dropout, probably due to initial competence of n  Tricuspid regurgitation may be due to primary
the valve in early diastole, followed by the typical low-velocity diastolic curve valve disease or may be secondary to annular
of pulmonic regurgitation. The velocity-time course reflects the pulmonary
artery-to-right ventricular pressure difference in diastole. Low-velocity flow dilation.
is consistent with a small pressure gradient and thus normal pulmonary n P rimary causes of tricuspid regurgitation include
diastolic pressures. endocarditis, Ebstein anomaly, rheumatic

CW:2.5MHz

RVOT
2.0
PR

PA m/s

2.0

A B
Figure 12-27  Severe pulmonic valve regurgitation.  A, Color Doppler evaluation of the pulmonic valve in a parasternal short-axis view shows laminar flow
filling the right ventricular outflow tract (RVOT) in diastole, consistent with severe pulmonic regurgitation. Because flow velocities are low, there is little vari-
ance, so that regurgitation may be missed on cine images but is evident on a frame-by-frame review. B, The CW Doppler recording shows that the density of
retrograde flow across the valve (above the baseline) is equal to the density of antegrade flow in systole. In addition, the end-diastolic velocity of the pulmonic
regurgitation approaches zero, indicating equalization of diastolic pressures in the pulmonary artery and RV. PA, Pulmonary artery; PR, pulmonic regurgitation;
RVOT, right ventricular outflow tract.
Valvular Regurgitation  CHAPTER 12 253

­ isease, carcinoid, and myxomatous disease (Fig.


d
12-28).
n Secondary tricuspid regurgitation is seen with

pulmonary hypertension of any cause, including RV
mitral valve disease, pulmonary parenchymal dis-
ease, or primary pulmonary hypertension.
LV
v  KEY POINTS
With Ebstein anomaly there is apical displace-
o
ment (insertion of tricuspid leaflet greater than
LA
10 mm apical from mitral valve leaflets) of one
or more valve leaflets. RA
o Carcinoid results in short, thick, and immobile
valve leaflets.
o Rheumatic tricuspid disease occurs in 20%
to 30% of patients with rheumatic mitral
disease.
o The diagnosis of secondary tricuspid regurgi-
tation is based on the presence of pulmonary
hypertension and the absence of structural
abnormalities of the leaflets.
Step 2: Evaluate the Severity of Tricuspid Figure 12-28  Ebstein anomaly of the tricuspid valve. In the apical
Regurgitation 4-chamber view the tricuspid valve leaflets are apically displaced so that the
portion of the RV between the leaflets and the tricuspid annulus (arrows) has
n Vena contracta width is the key step in evaluation a RA pressure. Severe tricuspid regurgitation results in severe RA and RV
of tricuspid regurgitant severity. enlargement. This patient has an intact atrial septum.
n Density of the CW Doppler velocity curve, relative
to antegrade flow, is also helpful.
n 
Systolic flow reversal in the hepatic veins is seen
with severe tricuspid regurgitation.
v  KEY POINTS
A vena contracta greater than 0.7 cm is spe-
o RV
cific for severe tricuspid regurgitation (Fig.
12-29).
o Vena contracta width is best measured in the
parasternal short-axis or RV inflow view.
o A dense CW Doppler signal is seen with severe
tricuspid regurgitation, but velocity reflects the
RV-to-RA systolic pressure gradient, not regur-
gitant severity (Fig. 12-30).
o Evaluation of hepatic vein flow patterns is
problematic unless sinus rhythm is present.
Step 3: Evaluate the Consequences of RV
Volume Overload
n Severe chronic tricuspid regurgitation is associated RA
with RV enlargement.
n  RV systolic function may be normal or may be
reduced with chronic tricuspid regurgitation.
n RV size and systolic function are evaluated qualita-
tively as described in Chapter 6.
n 
RA size is increased with chronic tricuspid
regurgitation. Figure 12-29  Vena contracta of the tricuspid regurgitant jet.  In the RV
inflow view from the parasternal window, zoom mode is used to maximize
resolution of the proximal jet geometry. The vena contract width of 4 mm is
consistent with moderate regurgitation. Evaluation of the vena contracta is
more accurate than visualization of the size of the flow disturbance in the RA
for quantitation of tricuspid regurgitation.
254 CHAPTER 12  Valvular Regurgitation

CW:2MHz

1.0

m/s

2.0

Figure 12-30  CW Doppler recording of severe tricuspid regurgitation.  There is a dense systolic signal with a low peak velocity. The low velocity reflects
the low-pressure difference between the normal pressure in the RV and the elevated pressure in the RA. The high signal strength (or density on the spectral
tracing) reflects the severity of regurgitation. Flow velocity is related to the pressure gradient as stated in the Bernoulli equation; high volume flow rates increase
velocity only slightly. Severe tricuspid regurgitation most often occurs in the absence of a severely elevated right ventricular (or pulmonary artery) pressure.
Valvular Regurgitation  CHAPTER 12 255

THE ECHO EXAM

Aortic Regurgitation
Aortic Regurgitation The vena contracta width indicates more than mild aortic
regurgitation, but this could be moderate or severe.
Etiology Valve abnormality Holodiastolic flow reversal in the proximal abdominal
Dilated aorta
aorta would be consistent with severe AR. Flow rever-
Severity of Vena contracta width sal in the descending thoracic aorta indicates at least
­regurgitation moderate AR but is less specific for severe AR.
Descending aorta holosystolic flow CW Doppler signal density indicates at least moder-
reversal
ate AR and a deceleration slope > 3 m/s2 but < 5 m/s2
CW Doppler deceleration slope is also consistent with moderate or severe aortic
Calculation of RV, RF and ROA regurgitation.
Co-existing aortic Aortic jet velocity Next, regurgitant volume (RV), regurgitant fraction
stenosis (RF), and regurgitant orifice area (ROA) are calculated.
LV response LV dimensions or volumes Using the LVOT and mitral annulus diameters
LV ejection fraction (MAD), the circular cross-sectional areas of flow are
dP/dt calculated:
Other findings Dilation of sinuses or ascending aorta
Aortic coarctation (with bicuspid CSALVOT = π(LVOTD /2)2 = 3.14(2.7/2)2 = 6.2 cm2
valve)
CSAMA = π(MAD /2)2 = 3.14(3.1/2)2 = 7.5 cm2
RF, Regurgitant fraction; ROA, regurgitant orifice area.
Stroke volume across each valve (cm3 = mL), then is:

Example SVLVOT = (CSALVOT × VTILVOT )


A 37-year-old man presents with an asymptomatic = 6.2 cm2 × 24 cm = 149 cm3
diastolic murmur. Echocardiography shows a bicus- SVMA = (CSAMA × VTIMA ) = 7.5 cm2 × 12 cm = 91 cm3
pid aortic valve with more than mild AR with:
Regurgitant volume (RV) is calculated from transaortic
Vena contracta width 5 mm flow (TSV, total stroke volume) and transmitral flow
(FSV, forward stroke volume), as:
Descending aorta Holodiastolic flow rever-
sal in descending tho- RV = TSV − FSV = 149 mL − 91 mL = 58 mL
racic, but not proximal
abdominal, aorta Regurgitant fraction (RF) is:
CW Doppler AR signal less dense
RV = RSV/TSV × 100 % = 58 mL/149 mL × 100 %
than antegrade flow = 39 %
VTIAR 204 cm
Regurgitant orifice area (ROA) is:
LVOT diameter (LVOTD) 2.7 cm
VTILVOT 24 cm ROA = RSV/VTIAR = 58 cm3 /204 cm = 0.28 cm2
Mitral annulus (MA) 3.1 cm
­diameter The RV, RF, and ROA all are consistent with moder-
VTIMA 12 cm ate (but nearly severe) aortic regurgitation.

Quantitative Evaluation of Aortic Regurgitant Severity


Parameter Mild Moderate Severe
Jet width/left ventricular outflow tract <25% 25-64% ≥65%
Vena contracta (cm) <0.3 0.3-0.6 >0.6
Regurgitant volume (mL/beat) <30 30-59 ≥60
Regurgitant fraction (%) <30 30-49 ≥50
Regurgitant orifice area (cm2) <0.1 0.1-0.29 ≥0.3
256 CHAPTER 12  Valvular Regurgitation

Quantitation of Aortic Regurgitation Severity


Parameter Modality View Recording Measurements and Calculations
Vena contracta width Color flow Parasternal Angulate, decrease Narrowest segment of regurgitant jet
­imaging long-axis depth, narrow sector, between proximal flow conver-
zoom gence and distal jet expansion
Descending aortic Pulsed Doppler Subcostal Sample volume 2-3 mm, Evidence for holodiastolic flow
­diastolic flow reversal and SSN decrease wall filters, reversal
adjust scale
CW Doppler signal CW Doppler Apical Careful positioning and Compare signal intensity of retro-
­(intensity, slope, VTI) transducer angulation grade to antegrade flow, measure
to obtain clear signal slope along edge of dense signal
Volume flow at two 2D and pulsed Parasternal LVOT diameter and VTI, TSV  = SVLVOT = (CSALVOT × VTILVOT)
sites (RV, RF, ROA) ­Doppler (2D) and Mitral annulus diameter FSV  = SVMA = (CSAMA × VTIMA)
apical and VTI RV  = TSV- FSV
ROA = RSV/VTIAR

AR, Aortic regurgitation; FSV, forward stroke volume; LVOT, left ventricular outflow tract; MA, mitral annulus; RF, regurgitant fraction;
ROA, regurgitant orifice fraction; RV, regurgitant volume; SSN, suprasternal notch; TSV, total stroke volume.

Mitral Regugitation Example


A 52-year-old man with a dilated cardiomyopathy
Mitral Regurgitation: Echo Approach presents with worsening heart failure symptoms.
Cause Primary valve disease Echocardiography shows a dilated LV with an
Secondary (functional) ­ejection fraction of 32% and a central jet of MR
Severity of Vena contracta width with:
regurgitation Jet direction (central, eccentric)
CW Doppler signal Vena contracta width 8 mm
Calculation of RV, RF, and ROA CW Doppler MR signal as dense as
Central jet: PISA method antegrade flow with
Eccentric jet: Volume flow at two no evidence for a
sites
v-wave
Pulmonary vein flow reversal
LV response LV dimensions or volumes dP/dt = 840 mmHg/s
LV ejection fraction VMR = 4.6 m/s
dP/dt
Pulmonary Pulmonary systolic pressure VTIMR = 130 cm
vasculature RV size and systolic function PISA radius 1.0 cm
Other findings Left atrial size Aliasing velocity 30 cm/s
PISA, Proximal isovelocity surface area; ROA, regurgitant orifice area; Right superior p
­ ulmonary Systolic flow reversal
RF, regurgitant fraction; RV, regurgitant volume. vein
The vena contracta width indicates severe mitral
regurgitation.
CW Doppler signal density indicates moderate to
severe MR and the absence of a v-wave suggests a
chronic disease process. The dP/dt is < 1000 mmHg/s
consistent with decreased LV contractility.
Color flow indicates a central jet so the proximal isovelocity
surface area (PISA) method can be used to quantitate
regurgitant severity.
The PISA is calculated from the radius measure-
ment as:

PISA = 2πr2 = 2π(1.0 cm)2 = 6.3 cm2


Valvular Regurgitation  CHAPTER 12 257

The maximum instantaneous regurgitant flow rate (RFR) Regurgitant volume over the systolic flow period
is calculated from PISA and the aliasing velocity can be estimated as:
(Valiasing) as:
RV = ROA × VTIMR = 0.41 cm2 × 130 cm
2
RFR = PISA × Valiasing = 6.3 cm × 30cm/s = 189cm /s 3 = 53 cm3 or mL

Maximum regurgitant orifice area (instantaneous) then This ROA and regurgitant volume is consistent with
is calculated from the RFR and MR jet velocity (where moderate mitral regurgitation.
4.6 m/s = 460 cm/s): If the jet is eccentric, quantitation should be per-
( ) formed using trans-aortic (forward) stroke volume and
ROAmax = RFR /VMR = 189cm3 /s /460cm/s trans-mitral (total) stroke volume calculations, as illus-
= 0.41cm2 trated for aortic regurgitation.

Quantitation of Mitral Regurgitation Severity


Measurements
Parameter Modality View(s) Recording and Calculations
Vena contracta Color flow Parasternal long Angulate, decrease Narrowest segment of regurgitant
width imaging axis depth, narrow sector, jet between proximal flow conver-
zoom gence and distal jet expansion
Color flow Color flow Parasternal and Narrow sector, decrease Central vs eccentric, anterior vs
imaging imaging apical depth posterior
CW Doppler CW Doppler Apical Careful positioning and Compare signal intensity of
signal transducer angulation retrograde to antegrade flow
to obtain clear signal
Proximal isove- Color flow A4C or A-long Decrease depth, narrow PISA = 2πr2
locity surface imaging axis sector, zoom, adjust RFR = PISA × Valiasing
area aliasing velocity ROAmax = RFR/VMR
Adjust aliasing velocity so RV = ROA × VTIMR
PISA is hemi-spherical,
measure from aliasing
boundary to orifice
Volume flow at 2D and pulsed Parasternal (2D) LVOT diameter and VTI TSV = SVMA = (CSAMA × VTIMA)
2 sites ­Doppler and apical Mitral annulus diameter FSV = SVLVOT = (CSALVOT × VTILVOT)
and VTI RV = TSV- FSV
ROA = RSV/VTIAR

2D LV total and 2D and pulsed Parasternal (2D) LVOT diameter and VTI TSV = EDV-ESV (on 2D LV volumes)
­Doppler LVOT ­Doppler and apical Apical biplane LV FSV = SVLVOT = (CSALVOT × VTILVOT)
­forward SV volumes RV = TSV- FSV
ROA = RSV/VTIAR
Pulmonary vein Pulsed A4C on TTE but Pulmonary vein flow in Qualitative systolic flow reversal
flow Doppler TEE often needed all four veins

A4C, Apical 4-chamber view; CSA, cross-sectional area; EDV, end-diastolic volume; ESV, end-systolic volume; FSV, forward stroke volume; LVOT, left
ventricular outflow tract; MA, mitral annulus; MR, mitral regurgitation; PISA, proximal isovelocity surface area; RFR, regurgitant flow rate; ROA, regurgi-
tant orifice area; RV, regurgitant volume; SV, stroke volume; TSV, total stroke volume; TTE, transthoracic echocardiography; VTI, velocity-time integral.

Quantitative Evaluation of Primary Mitral Regurgitant Severity


Mitral regurgitation Progressive (Mild to Moderate) Severe
Jet area (cm2) 20-40% >40% LA area
Vena contracta (cm) <0.7 ≥0.7
Regurgitant volume (mL) <60 ≥60
Regurgitant fraction (%) <50 ≥50
Regurgitant orifice area (cm2) <0.40 ≥0.40
258 CHAPTER 12  Valvular Regurgitation

SELF-ASSESSMENT QUESTIONS
Question 1 A.
Aortic regurgitation
Doppler recordings of valve flow were recorded in the B. Mitral regurgitation
same patient. For the Doppler signals shown in Fig. C. Pulmonic regurgitation
12-31, A-D, match each Doppler signal with the cor- D. Tricuspid regurgitation
rect diagnosis: E. Ventricular septal defect

1.0
2.0

m/s

m/s

2.0 3.0
A B

2.0
8.0

m/s

m/s

2.0 8.0
C D
Figure 12-31 

Question 2 2.0
This Doppler signal (Fig. 12-32) was recorded on a TTE.
The most likely diagnosis is:
m/s
A. Bicuspid aortic valve
B. Mechanical prosthetic valve
C. Hypertrophic cardiomyopathy
D. Mitral valve prolapse

6.0

Figure 12-32 
Valvular Regurgitation  CHAPTER 12 259

Question 3 Question 4
In patient undergoing TEE, the images in Fig. 12-33, TEE was requested in an asymptomatic 56-year-old
A-C were obtained. Identify the findings in each image man with mitral regurgitation for evaluation of valve
and indicate which image shows the most significant anatomy and quantitation of mitral regurgitant sever-
abnormality. ity. The color and CW Doppler signals are shown in
A. Aortic stenosis Fig. 12-34.
B. Aortic regurgitation Calculate the following:
  
C. Mitral stenosis Regurgitant orifice area _____________________
D. Mitral regurgitation Regurgitant volume ________________________
E. Pulmonic regurgitation
F. Tricuspid regurgitation

FR 16Hz PISA 0.8 cm


13cm
2D
75%
C 48
P Off
Res
CF
59%
3.3MHz
WF Med
Low

A
FR 16Hz 6.0
13cm
2D
75%
VTI 5.0
C 48 4.8 m/s
P Off 140 cm
Res 4.0
CF
59% 3.0
3.3MHz
WF Med
Low 2.0

1.0

m/s

B 75mm/s 91bpm
FR 20Hz
10cm Figure 12-34 
2D
61%
C 50
P Off
Question 5
Gen
CF Which of the following measures is least consistent
59%
3.3MHz
with severe mitral regurgitation?
WF Med A. Mitral E-wave velocity 2.5 m/s
Low
B. Mitral vena contracta 0.8 cm
C. Maximum regurgitant velocity 5.2 m/s
D. Regurgitant orifice area 0.5 cm2
Question 6
C In a patient with myxomatous mitral valve disease,
which of the following echocardiographic findings is
Figure 12-33 
most sensitive for an acute flail posterior leaflet?
A. Vena contracta 0.4 cm
B. Pulmonary vein systolic blunting of flow
C. Mitral regurgitation dP/dt 780 mmHg/s
D. Anteriorly directed jet
260 CHAPTER 12  Valvular Regurgitation

Question 7
A 40-year-old asymptomatic man with a known bicus- The most compelling finding to recommend cardiac
pid aortic valve, undergoes routine TTE, and the fol- surgery, based on these data, is:
lowing data are obtained. A. Vena contracta
B. LV function
LV dimensions, systole/diastole 4.9/7.1 cm C. Pulmonary pressure
LV ejection fraction 48% D. Ascending aorta size
Tricuspid regurgitation jet velocity 3.3 m/s
E. Pressure half-time (T½)
Aortic sinus diameter 4.2 cm
Aortic pressure half time (T½) 410 ms

Question 8
Identify the red color Doppler signal (seen in Fig. 12-35).
A. Aortic regurgitation
B. Pulmonic regurgitation
C. Ruptured sinus of Valsalva aneurysm
D. Ventricular septal defect
E. Coronary artery

38dB 3 •/1/0/ 1 4V1c-S


PW Depth  61mm H4.25MHz
UWMC
PW Gate  2.5mm UWMC/V
PW Gain  10dB Store in progress
HR  43bpm
Sweep  100mm/s
2.5

m/s
.50
Figure 12-35 

Question 9 6.0
In an acutely ill patient admitted to the intensive care
unit, a murmur is heard on physical examination, and
a TTE is performed (Fig. 12-36).
The most likely diagnosis is:
A. Severe mitral regurgitation
B. Severe tricuspid regurgitation
C. Severe aortic regurgitation
D. Severe pulmonic regurgitation m/s

2.0

Figure 12-36 
Valvular Regurgitation  CHAPTER 12 261

Question 10 Question 11
Echocardiographic data are recorded in a patient For each of the three Doppler flows (as shown in Fig.
with a diastolic murmur (Fig. 12-37). 12-38) indicate the most likely diagnosis using the fol-
lowing choices:
Mitral valve inflow VTI 15.0 cm
A. Aortic regurgitation
LV outflow tract VTI 29.0 cm B. Mitral regurgitation
LV outflow tract diameter 2.4 cm C. Pulmonic regurgitation
D. Tricuspid regurgitation
Mitral annular diameter 3.0 cm

Given these data, calculate the following for this patient:


   1.0
Aortic regurgitant volume _____________________
Aortic regurgitant fraction ____________________
Aortic regurgitant orifice area ___________________ m/s

1.0

I
AR VTI  2.91 m
1.0

m/s

m/s 2.0

II
1.0

5.0
m/s

Figure 12-37 

1.5

III
Figure 12-38 

Question 12 0
Cal  20mm

This M-mode tracing (as shown in Fig. 12-39) is most


consistent with:
A. Aortic regurgitation
B. Hypertrophic cardiomyopathy
C. Dilated cardiomyopathy
D. Mitral valve prolapse
E. Mitral stenosis

160

Figure 12-39 
262 CHAPTER 12  Valvular Regurgitation

ANSWERS

Answer 1 Cal  10mm

The answers within the identified sections of Fig. 9


12-31 are as follows:
  

Fig. 12-31, A: Pulmonic regurgitation


Fig. 12-31, B: Tricuspid regurgitation
Fig. 12-31, C: Aortic regurgitation
Fig. 12-31, D: Mitral regurgitation
  

The type of valve regurgitation for each of these


four Doppler signals, recorded in the same patient,
can be distinguished based on the timing of the flow
signal, the accompanying antegrade flow signal, and
the shape and velocity of the velocity signals. The
pulmonic valve in Fig. 12-1, A shows an ejection type
velocity in systole, with a diastolic pulmonic regur-
gitation signal. This patient has severe pulmonic
regurgitation with equal density of the antegrade and
retrograde flow signals, a steep diastolic deceleration 160
slope that reaches the zero baseline before the end of APX MV
diastole and an increased antegrade velocity due to
the increased transpulmonic volume flow rate. Tricus- 2.0
pid regurgitation in Fig. 12-1, B shows a slow rate or
rise in velocity, with a late peak in systole consistent
with the normal RV systolic pressure increase. The m/s
peak velocity of 2.8 m/s is consistent with mildly
increased RV systolic pressure. The antegrade dia-
stolic flow signal across the tricuspid valve is not well
demonstrated. The aortic regurgitant signal in Fig.
12-1, C shows a normal antegrade velocity across the
aortic valve, with the velocity (over 4 m/s) and dia-
stolic slope of the aortic regurgitant signal consistent
with the aortic to LV pressure difference in diastole.
Only mild aortic regurgitation is present as evidenced
by the relatively low density of diastolic compared to 6.0
systolic flow and the flat diastolic deceleration slope.
Only trace mitral regurgitation is present with a faint
signal seen at the beginning and end of systole in Figure 12-40 
Fig. 12-1, D. The duration of mitral regurgitation is
shorter than the duration of tricuspid regurgitation regurgitation coinciding with prolapse of the poste-
because RV ejection typically is slightly longer than rior mitral valve leaflet.
LV ejection. Aortic stenosis due to a bicuspid aortic valve
would show an ejection curve throughout systole
Answer 2: D and probably would show aortic regurgitation in
This is a CW Doppler signal of the mitral valve taken diastole. A dynamic LV outflow gradient due to
from the apical view. The LV inflow pattern shows hypertrophic cardiomyopathy does peak in late sys-
the early diastolic E-wave filling and the late diastolic tole, but flow starts with LV ejection with an upward
atrial A-wave, corresponding to atrial contraction. curving or “dagger-shaped” waveform, indica-
During systole, flow is directed away from the trans- tive of progressive systolic obstruction rather than
ducer. There is absence of flow in early systole, with the abrupt late-systolic flow onset characteristic of
abrupt onset of flow in mid- to late systole. This is mitral valve prolapse. A mechanical aortic or mitral
characteristic of mitral valve prolapse where the valve prosthetic valve Doppler signal would show a bright
is competent earlier in systole, until buckling and pro- mechanical click corresponding to opening and
lapse of the mitral valve into the LA leads to poor closing of the valve occluders. Shadowing of the
leaflet coaptation and mitral regurgitation. The color LA by a prosthetic mitral valve will hinder evalu-
M-mode tracing (as shown in Fig. 12-40) shows mitral ation of prosthetic regurgitation, but if prosthetic
Valvular Regurgitation  CHAPTER 12 263

regurgitation is present, occasionally a faint holosys- are equal and that volume flow rate equals the cross-
tolic regurgitation Doppler signal can be seen dur- sectional area of flow times the velocity at that site.
ing systole. Thus, for a single point in the cardiac cycle:
Answer 3 PISA × Valiasing = ROA × VMR (12-12)
Fig. 12-33, A: Mitral regurgitation. This TEE long-
axis image shows the aortic valve open in systole with In these images, the aliasing velocity in the direc-
normal antegrade flow across the aortic valve. How- tion of flow shown at the top of the color bar is 38.5
ever, in the LA, just posterior to the aorta, a systolic cm/s or 0.39 m/s. The maximum velocity of the
flow disturbance is seen consistent with an anteriorly mitral regurgitant jet is 4.8 m/s recorded with CW
directly jet of mitral regurgitation. Doppler. The ROA (at that point in the cardiac cycle),
Fig. 12-33, B: Aortic regurgitation. In the same long- then, is calculated as:
axis image as Fig. 12-3, A, the diastolic image shows a
small central jet of aortic regurgitation. The narrow ROA = PISA × (Valiasing /VMR )
vena contracta is consistent with mild regurgitation. = 4.02 cm2 × (0.39/4.8) = 0.33 cm2 (12-13)
Fig. 12-33, C: Tricuspid regurgitation. This view
shows the aortic valve in short axis (circle to the Regurgitant volume is the volume of blood that
right of the image) and is aimed at the tricuspid goes backward through the valve with each cardiac
valve. This systolic frame shows a normal appearing cycle. Regurgitant volume is calculated as the product
tricuspid valve that is closed in systole. An eccentric of the velocity time integral (VTI) of the regurgitant
jet of tricuspid regurgitation is directed towards the jet and the regurgitant orifice area:
atrial septum. The flow disturbance fills only about
one third of the atrium and the vena contracta RV = ROA × VTIMR = 0.33 cm2 × 140 cm
where the jet crosses the valve plane is narrow. = 46 cm3 or 46 mL (12-14)
These findings are consistent with mild tricuspid
regurgitation. These calculations are consistent with moderate
Most significant abnormality: mitral regurgitation. mitral regurgitation.
In this patient, there is only mild aortic and mild tri-
cuspid regurgitation. However, an eccentric anteriorly Answer 5: C
directed jet of mitral regurgitation usually indicates The maximum mitral regurgitant jet velocity reflects
significant mitral valve disease, with a flail or pro- the LV-to-LA pressure gradient and is not a useful
lapsed posterior mitral valve leaflet. Once the flow measure of regurgitant severity. Although with severe
disturbance is seen, the echocardiographer should acute regurgitation, the increased LA pressure and
carefully examine the valve to measure the vena con- decreased systolic blood pressure (and decreased LV
tracta, PISA, and CW Doppler of mitral regurgitant pressure) may result in a lower maximum regurgitant
jet for quantitation of regurgitant severity. Ideally, 3D velocity, with chronic regurgitation, velocity primarily
imaging of the valve should be done to evaluate the reflects systolic blood pressure. The increased regur-
likelihood of valve repair. gitant volume of severe mitral regurgitation results in
an increased antegrade transmitral stroke volume and
Answer 4 the mitral opening pressure (the LA-to-LV pressure
difference in early diastole) is increased; both these
Regurgitant orifice area 0.33 cm2
factors result in a high antegrade mitral flow velocity
Regurgitant volume 46 mL (with a steep deceleration slope) when severe mitral
regurgitation is present. The transmitral E-wave is
The first step in this calculation is to measure the typically increased over 1.5 m/s. The vena contracta
proximal isovelocity surface area (PISA). This is the is the narrowest diameter of the regurgitant jet at the
surface area of the 3D hemisphere on the LV side of valve plane, usually measured in the long-axis view
the valve where the velocity is the same everywhere of the valve. A wider vena contracta implies a larger
on the surface (e.g., isovelocity), defined by the color regurgitant orifice area and more severe regurgitation.
aliasing velocity. The radius of the hemisphere from A vena contracta width 0.7 cm or greater is consis-
the aliasing velocity to the valve plane is 0.80 cm. The tent with severe regurgitation. The regurgitant orifice
surface area of a hemisphere is 2πr2. Thus: area is a measure of the “hole” through which blood
is regurgitant. A regurgitant orifice area of 0.4 cm2
PISA = 2πr2 = 2(3.14)(0.80)2 = 4.02 cm2 (12-11) or larger is consistent with severe mitral regurgitation.

The regurgitant orifice area is calculated based on Answer 6: D


the continuity principle that the volume of blood flow A flail leaflet leads to failure of leaflet coaptation.
through the PISA and through the regurgitant orifice With posterior mitral leaflet prolapse, the tip of the
264 CHAPTER 12  Valvular Regurgitation

posterior leaflet prolapses into the LA. During sys-


tole, the flail tip is more cephalad and the regurgitant
jet enters the LA eccentrically across the back of the
anterior leaflet, directed anteriorly as seen in the api-
cal 4-chamber view (Fig. 12-41) in this patient.
Entrainment of the regurgitant jet along the wall
of the atrium may make the jet appear smaller. A
vena contracta of 0.4 cm is consistent with only mod-
erate regurgitation; a flail leaflet is typically associated
with severe regurgitation. Pulmonary vein systolic
blunting of flow is consistent with moderate severity
regurgitation. Pulmonary vein systolic flow reversal
is consistent with severe regurgitation but does not
help in determining the specific leaflet involved unless
systolic flow reversal is present in some, but not all,
pulmonary veins. In patients with LV systolic dysfunc-
tion, the poorly functioning LV does not generate a
transmitral gradient rapidly and the initial slope of
the mitral regurgitant jet decreases to below 1000
mmHg/s. A decreased dP/dT, indicative of decreased
systolic function, is not a sensitive finding for a flail
mitral valve leaflet alone.
Answer 7: B
Patients with symptomatic, severe valvular regurgi- Figure 12-41 
tation should be referred for surgical intervention.
In asymptomatic patients, evidence of the progressive when aortic diameter exceeds 5.5 cm. If the patient
effect of the regurgitant volume load with either LV is already undergoing valve replacement for stenosis
dilation or a decline in systolic function is the primary or regurgitation, replacement of the aorta should be
criteria to prompt earlier surgical intervention. Because considered at a diameter of 4.5 cm.
a small subset of patients do not develop cardiopul-
monary symptoms despite LV dilation or a decline in Answer 8: B
function, periodic imaging in asymptomatic patients This color Doppler image shows an eccentric jet of
is indicated once regurgitation is diagnosed. Current pulmonic valve regurgitation as confirmed by the
guidelines recommend valve surgery in asymptomatic pulsed Doppler tracing showing the characteristic low
patients with severe aortic regurgitation and an end- velocity diastolic flow signal. The systolic flow signal
systolic dimension of 50 mm or more, or an ejection represents normal antegrade aortic flow at a nonpar-
fraction less than 50%. Surgery is recommended in allel intercept angle. Aortic regurgitation would also
this case based on the decreased ejection fraction. be diastolic but would be higher in velocity reflect-
A tricuspid regurgitation jet velocity of 3.3 m/s ing the aortic to LV diastolic pressure difference and
suggests that the RV systolic pressure is at least 44 the color jet would go across the valve into the LV.
mmHg over RA pressure, supporting at least moder- A ruptured sinus of Valsalva aneurysm would show
ate pulmonary hypertension. However, pulmonary high velocity systolic (aortic to RV outflow tract pres-
hypertension is not a primary indication for aortic sure difference) and diastolic flow (because aortic dia-
valve replacement. The pressure half-time (T½) is stolic pressure is higher than RV diastolic pressure). A
the time interval between the peak transvalvular ventricular septal defect would be located on the ven-
pressure gradient and half the initial gradient. For tricular size of the aortic valve and would show high
the aortic regurgitant jet, a pressure half-time of less velocity systolic flow with the shape of the flow curve
than 200 ms indicates rapid equalization of pressures similar to mitral regurgitation. Coronary artery flow
between the aorta and LV, consistent with acute sig- occurs predominantly in diastole and is low velocity;
nificant regurgitation. However, with chronic disease, however an antegrade signal in systole typically also
the pressure half-time may be normal despite severe is present.
regurgitation. The vena contracta of 0.8 cm confirms
that this patient has severe aortic regurgitation, which Answer 9: C
by itself is not an indication for valve surgery in the This is a CW Doppler signal showing flow away
absence of symptoms. A subset of bicuspid aortic from the transducer in systole with a shape that is
valve patients have associated dilation of the aorta, consistent with antegrade flow in a great vessel
and current guidelines recommend root replacement (aorta or pulmonary artery) or a regurgitant flow
Valvular Regurgitation  CHAPTER 12 265

across an atrioventricular (mitral or tricuspid valve). The regurgitant fraction is the proportion of
However, the maximum velocity is less than 2 m/s, regurgitant volume compared with the total transaor-
which excludes mitral regurgitation, and the shorter tic flow:
duration of the systolic signal suggests that tricus-
RF = RVAR /SVLVOT = 25 mL/131 mL = 19 % (12-18)
pid regurgitation is unlikely. The antegrade ejection
velocity might be transaortic flow or transpulmonic The ROA is then calculated by dividing regurgi-
flow. However, the diastolic signal has an initial dia- tant stroke volume by the VTIAR.
stolic velocity of 4 m/s, indicating a 64 mmHg gradi- ROA = RVAR /VTIAR
ent between the great vessel and ventricle. Although = 25 cm3 /291 cm = 0.09 cm2 (12-19)
this might be pulmonic regurgitation if severe pul-
monary hypertension were present, the timing is An ROA less than 0.1 cm2, a regurgitant volume
more consistent with aortic regurgitation. These of 25 mL, and a regurgitant fraction of 19% are all
data were recorded in a patient with acute severe consistent with mild aortic regurgitation. These data
aortic regurgitation. The end-diastolic velocity of are congruent with the visual impression from the
the regurgitation jet, about 2 m/s, indicates the end- CW Doppler signal with a faint diastolic regurgitant
diastolic gradient between the aorta and LV is only signal compared with antegrade flow.
16 mmHg, consistent with a low aortic and high LV
diastolic pressure in this acutely ill patient. In addi- Answer 11
tion, the diastolic regurgitant signal is as dense as I: D. This is a pulsed Doppler sample taken from
antegrade systolic flow consistent with severe aortic the hepatic vein in a patient with severe tricuspid
regurgitation. The steep diastolic deceleration slope regurgitation. The image is acquired from the sub-
indicates rapid equalization of pressure between the costal view with flow from the hepatic vein to the
aorta and LV during diastole, consistent with acute, IVC directed antegrade, away from the transducer.
rather than chronic, regurgitation. The venous flow pattern is evident with a brief flow
Answer 10 curve toward the transducer after atrial contraction
and atrial filling during diastole. Peak flow veloci-
Regurgitant volume 25 mL
ties in the hepatic vein are low, and the scale is set
with a maximum velocity of 0.6 m/s. With severe
Regurgitant fraction 19% tricuspid regurgitation, there is systolic flow reversal
Regurgitant orifice area 0.09 cm2 in the hepatic vein, shown as flow in systole directed
toward the transducer following the QRS complex
In this patient with aortic regurgitation, the stroke vol- instead of the normal pattern of RA filling in sys-
ume across the aortic valve is the sum of anterograde tole, as well as diastole. Pulmonary vein flow also
flow and regurgitant flow or total stroke volume. shows a venous flow pattern, but diastolic filling
Assuming a competent mitral valve, the anterograde would be directed toward the transducer from the
flow across the mitral valve (SVMV) equals forward transthoracic approach.
stroke volume. To calculate the volume flow rate II: A. This is a pulsed Doppler recording taken
(stroke volume) at each valve, the area and velocity from the descending thoracic aorta in a patient with
time integral at each site are needed. Stroke volumes severe aortic regurgitation. The antegrade flow in sys-
are calculated by multiplying cross-sectional area tole at 1.3 m/s with an ejection type curve that iden-
(assumed to be circular using the measured diameter) tifies this as a great artery. This is unlikely to be the
and velocity time integral at that site. Thus: pulmonary artery because the velocity peaks in early
systole and is shorter in duration and higher in veloc-
SVMV = CSAMV × VTIMV ity than typical pulmonary artery flow. The holodia-

= 3.14(3.0/2)2 × 15 cm = 106 cm3 (12-15) stolic (extends continuously from the beginning to end
of diastole) flow reversal seen as flow directed toward
the transducer during diastole is consistent with mod-
erate to severe aortic regurgitation.
SVLVOT = CSALVOT × VTILVOT III: B. This is a pulsed Doppler sample of the
= 3.14(2.4/2)2 × 29 cm = 131 cm3 (12-16) pulmonary vein flow in a patient with severe mitral
regurgitation due to mitral valve prolapse. Diastolic
LV filling with flow directed toward the transducer
Regurgitant volume (RVAR) is the difference is seen, identifying this as pulmonary venous inflow.
between SVLVOT and SVMV: In systole, early systolic flow toward the transducer
is seen, consistent with normal LV filling. How-
ever, in late systole flow is reversed (directed away
RVAR = SVLVOT − SVMV from the transducer), suggesting late systolic mitral
= 131 cm3 − 106 cm3 = 25 cm3 or 25 mL (12-17) regurgitation.
266 CHAPTER 12  Valvular Regurgitation

Answer 12: A thickening of the septum compared with the poste-


rior wall is seen, and if there is obstruction, systolic
This is an M-mode tracing from the parasternal long-
anterior motion of the mitral leaflets is present. LV
axis view. The mitral valve is seen in the midportion
chamber size is normal, not consistent with a dilated
of the LV. During diastole, the mitral valve is open
cardiomyopathy; in addition, the separation between
and the anterior mitral valve leaflet shows a rapid flut-
the mitral E point and the septum is increased with
tering motion. This motion is the result of the aortic
LV dilation and systolic dysfunction. With mitral
regurgitant jet impinging on the thin flexible anterior
valve prolapse, the mitral leaflets are thickened and
mitral leaflet. Regurgitation is likely only mild because
there is late systolic buckling of the mitral valve leaflet
the E-point septal separation is normal and there is no
posteriorly on the M-mode tracing. If mitral stenosis
LV dilation. Other findings on the M-mode tracing are
were present, leaflet thickening with a reduced dia-
concentric LV hypertrophy (thick walls with a small
stolic opening and a flat diastolic leaflet slope would
chamber) and marked posterior pericardial thicken-
be seen.
ing. With hypertrophic cardiomyopathy, asymmetric
13 Prosthetic Valves
BASIC PRINCIPLES Transcatheter Aortic Valve Implantation
Step-by-Step Approach Valved Conduits or Valve Resuspension
Review the Clinical and Operative Data Mechanical Aortic Valves
Obtain Images of the Prosthetic Valve Mitral Valves
Record Prosthetic Valve Doppler Data Mitral Valve Repair
Evaluate LV Geometry and Function Bioprosthetic Mitral Valves
Measure Pulmonary Pressures and Evaluate Right Mechanical Mitral Valves
Heart Function Tricuspid Valve Prostheses and Rings
SPECIFIC VALVE TYPES Prosthetic Pulmonic Valves
Aortic Valves THE ECHO EXAM
Bioprosthetic Aortic Valves SELF-ASSESSMENT QUESTIONS

BASIC PRINCIPLES o A surgically implanted bioprosthetic valve typi-


n  valuation of prosthetic valves by echocardiogra-
E cally has three struts supporting the leaflets at
phy is based on the same principles as evaluation the commissures; a stentless bioprosthetic valve
of native valve disease. may be indistinguishable from a normal native
n F luid dynamics (and Doppler flows) depend on the valve.
specific valve type and size (Fig. 13-1). o Transcatheter aortic valve implantation can be
n Dysfunction of mechanical valves usually is due to performed either from the femoral artery retro-
valve thrombosis resulting in systemic embolism, grade across the valve or from the left ventricu-
incomplete closure (regurgitation), or inadequate lar (LV) apex antegrade across the valve.
opening (stenosis) (Table 13-1 and Table 13-2). o Transcatheter bioprosthetic valves currently
n Dysfunction of bioprosthetic valves usually is due include balloon expandable and self-expanding
to leaflet degeneration (regurgitation) or calcifica- types, both with a trileaflet bioprosthetic valve
tion (stenosis). mounted inside a metal mesh cage or stent
n All prosthetic valves are at risk of endocarditis, (Fig. 13-2; see Fig. 13-1).
which often primarily affects the annular ring o The most common mechanical valve now
rather than the valve leaflets. implanted is a bileaflet design with two semicir-
cular disks that open to form a central slit-like
v  KEY POINTS orifice and two larger lateral openings.
o There are several types of surgically implanted o Other types of mechanical valves include single
bioprosthetic valves, which can be classified as disk valves that “tilt” to open, either on a central
stented, stentless, or combined valve-root pros- strut or with hinges in the annular ring. Ball-
theses (including homograft valves). cage valves may still be seen in some patients.

A  Valve posts B C
Figure 13-1  Basic types of prosthetic valves.  Examples of a bioprosthetic aortic valve at the time of surgical implantation (A), bileaflet mechanical valve (B), and
transcatheter bioprosthetic valve (C). (A, From Oxorn D, Otto C: Atlas of Intraoperative Echocardiography. Philadelphia, Elsevier, 2007. B, Courtesy St Jude Medical, Inc,
St Paul, MN. C, Courtesy Edwards SAPIEN Investigational Transcatheter Heart Valve. Available at www.edwards.com/products/transcathetervalves/sapienthv.htm; from
Otto, CM: Textbook of Clinical Echocardiography, ed 5, Elsevier, 2013, Philadelphia.)

267
268 CHAPTER 13  Prosthetic Valves

TABLE 13–1  Prosthetic Valves: Clinical Echocardiographic Correlates


Mechanical Surgical Transcatheter Mechanical Bioprosthetic
AVR Bioprosthetic AVR Bioprosthetic AVR MVR MVR
Fluid • Complex fluid • Central orifice, • Central orifice, • Complex • Central ori­
dyanmics dynamics laminar flow, laminar flow, blunt fluid dynamics fice, laminar
depending on blunt flow profile flow profile depending on flow, blunt
valve type valve type flow profile
Echo • Shadowing • Echogenic • Increased • Shadowing • Stented valve,
imaging and reverbera­ sewing ring and echogeniticity of and reverbera­ flow directed
tions limit valve three struts aortic sinuses and tions limit valve towards
imaging • Trileaflet porcine annulus due to imaging on TTE septum
or pericardial supporting stent • Valve occluder • Trileaflet
bioprosthetic • Biologic valve leaf­ motion well porcine or
similar to a na­ lets appear similar seen on TEE pericardial
tive aortic valve to a native aortic bioprosthetic
valve similar to a
native aortic
valve
Nornal Doppler • Antegrade veloc­ • Antegrade • Antegrade veloc­ • Antegrade ve­ • Antegrade
findings ity < 3 m/s with velocity < 3 m/s ity < 3 m/s with locity < 1.9 m/s velocity < 1.9
triangular shaped with triangular triangular shaped with short T½ m/s with short
flow curve shaped flow flow curve • Mild eccentric T½
• Mild eccentric AR curve • Mild valvular or MR due to oc­ • No to trace
due to occluder • No to trace paravalvular AR cluder closure central MR
closure central AR
Advantages/ • Excellent long • Variable dura­ • Unknow durabilty • Excellent long • Variable dura­
disadvan- term durabilty bilty, longer in • Currently recom­ term durabilty bilty, longer in
tages • Requires chronic older patients mended in high • Requires older patients
anticoagulation • Does not require risk patients chronic antico­ • Does not
anticoagulation • Does not require agulation require anti­
anticoagulation coagulation
(but may be
needed for
atrial fibrilla­
tion)
Complications • Valve thrombosis • Leaflet degen­ • Leaflet degenera­ • Valve throm­ • Leaflet de­
• Pannus eration tion bosis generation
• Paravalvular AR • Stenosis • Stenosis • Pannus • Stenosis
• Endocarditis • Regurgitation • Regurgitation • Paravalvular • Regurgitation
• Pannus • Pannus MR • Pannus
• Paravalvular AR • Paravalvular AR • Endocarditis • Paravalvular
• Endocarditis • Endocarditis AR
• Endocarditis
Echo • Baseline post op • Baseline post op • Baseline post op • Baseline post • Baseline post
follow-up • Changing signs or • Changing signs • Changing signs or op op
(in addition symptoms or symptoms symptoms • Changing signs • Changing
to annual • Annual starting • Annual exams or symptoms signs or
clinical 5 years after recommended at symptoms
­evaluation) implantation this time • Annual
starting 5
years after
implantation

AR, Aortic regurgitation; AVR, aortic valve replacement; MR, mitral regurgitation; MVR, mitral valve replacement.
Prosthetic Valves  CHAPTER 13 269

TABLE 13-2  Prosthetic Stenosis and Regurgitation: Findings Suggestive of Significant Valve
Dysfunction With Stented Bioprosthetic and Mechanical Valves
Severe Stenosis Severe Regurgitation
Aortic valve Vmax> 4 m/s LV dilation
replacement Mean ΔP > 35 mmHg AR jet width ≥65% of LVOT diameter
Velocity ratio < 0.25 CW Doppler signal dense with T½ < 200 ms
Rounded, late peaking velocity Holodiastolic flow reversal in DA
curve shape Regurgitant volume (RV) > 60 mL
EOA < 0.8 cm2 Regurgitant fraction(RF) > 50%
Mitral valve Vmax> 2.5 m/s LV dilation
replacement Mean ΔP > 10 mmHg Large central MR jet or variable size wall-impinging jet
T ½ > 200 msec Large PISA with vena contracta ≥ 0.6 cm
VTImitral/VTILVOT> 2.5 CW Doppler signal dense with triangular shape
EOA < 1.0 cm2 Pulmonary vein systolic flow reversal
Pulmonary hypertesion (esp. if new)
RV ≥60 mL, RF ≥50%, EROA ≥0.50 cm2
Pulmonary valve Vmax> 3 m/s (or >2 m/s with a homograft) RV dilation
replacement with a progressive increase in velocity Jet width > 50% of pulmonic annulus
on serial studies CW Doppler signal dense, steep deceleration, flow
ends in mid to late diastole
Diastolic flow reversal in pulmonary artery
RF > 50%
Tricuspid valve Vmax> 1.7 m/s TR jet area > 10 cm2
replacement Mean ΔP ≥ 6 mmHg Vena contracta width > 0.7 cm
T ½ ≥ 230 msec CW Doppler signal dense with triangular shape
Holosystolic flow reveral in hepatic veins
Severe RA dilation

Data from Zoghbi WA, Chambers JB, Dumesnil JG, et al: Recommendations for evaluation of prosthetic valves with echocardiography and Doppler
ultrasound. J Am Soc Echocardiogr 2009;22:975-1014.
AR, Aortic regurgitaiton; DA, descending aorta; EOA, effective orifice area; EROA, effective regurgitant orifice area; LVOT, left ventricular outflow
tract; Mean ΔP, mean transvalvular pressure gradient; MR, mitral regurgutation; PISA, proximal isovelocity surfac area; RF, regurgitant fraction;
RV, regurgitant volume; Vmax, maximum antegrade transvalvular velocity; VTI, velocity time integral.

o 
On echocardiography, mechanical valves Valsalva, and surgical coronary reimplantation
result in ultrasound reverberations and shad- (with replacement of the aortic sinuses).
owing that limit direct visualization of valve o With mitral valve surgery, key features are valve
function. repair or replacement, preservation of the
mitral leaflets and chords with valve replace-
Step-by-Step Approach ment, amputation of the left atrial (LA) append-
age, and whether a concurrent atrial ablation
Step 1: Review Clinical and Operative Data (e.g., maze) procedure was done.
n Information on the operative procedure is reviewed o The valve type and size determine the expected
before the echocardiographic examination. hemodynamics and are important for distin-
n The valve type and size, obtained from the medical guishing normal prosthetic Doppler data from
record or the patient’s valve ID card, are included prosthetic valve stenosis or regurgitation.
on the echocardiographic report. o On early postoperative studies, unexpected
n 
Blood pressure and heart rate at the time of the findings are discussed directly with the surgeon
echocardiogram are recorded. to correlate with observations during the surgi-
cal procedure.
v  KEY POINTS
Information in the operative report helps guide
o
Step 2: Obtain Images of the Prosthetic
the echocardiographic image acquisition and Valve
improves the final interpretation. n Prosthetic aortic valves are imaged in parasternal
o With aortic valve surgery, key features are valve long- and short-axis views.
replacement versus resuspension, concurrent n 
Prosthetic mitral valves are imaged in parasternal
replacement of the aortic root either above the long- and short-axis views and in apical 4-chamber
sinotubular junction or including the sinuses of and long-axis views (Fig. 13-3).
270 CHAPTER 13  Prosthetic Valves

APX AV
RVOT
4.0

LV
Ao

m/s

LA
LA

A B C 4.0
Figure 13-2  Transcatheter aortic valve.  In the parasternal long-axis (A) and short-axis (B) views the trileaflet bioprosthetic valve is similar in appearance
to a native aortic valve. Increased paravalvular echogenicity is consistent with the mesh-cage around the valve. Paravalvular regurgitation is present with an
eccentric jet (arrow) seen in the long-axis view. The short-axis view shows regurgitation extending from about 8 to 11 o’clock (arrows) around the medial aspect
of the valve cage. CW Doppler shows a moderately dense diastolic signal (arrow). Ao, Aorta; RVOT, right ventricular outflow tract.

Ao

LV

LA

A B
Figure 13-3  Stented bioprosthetic mitral valve.  A, Parasternal long-axis view of a stented bioprosthetic mitral valve prosthesis with the typical appearance
of the struts (arrow) protruding into the LV; B, color Doppler shows the inflow stream directed toward the ventricular septum. Acoustic shadowing (S) from the
valve struts results in an anechoic region in the far field. Ao, Aorta; S, acoustic shadow.

n Transesophageal echocardiography (TEE) imag- the leaflets and annular region may be adequately
ing is needed to evaluate the left atrium (LA) side evaluated by this approach for most baseline or
of mechanical mitral prosthetic valves, due to follow-up studies in clinically stable patients.
shadowing from the transthoracic approach, when o Three-dimensional (3D) TEE imaging of pros-
valve dysfunction is suspected (Fig. 13-4). thetic valves provide better spatial resolution
n TEE also often provides better images of the pos- with lower temporal resolution (see Fig 13-4).
terior aspect of aortic valve prostheses. o Bioprosthetic valves have a trileaflet structure
similar to a native aortic valve. Mitral biopros-
v  KEY POINTS thetic valves are stented to provide support for
o Transthoracic imaging of mechanical valves is the leaflets, with the leaflets well seen in both
limited by reverberations and shadowing. Even so, parasternal and apical views.
Prosthetic Valves  CHAPTER 13 271

LA MVR

S R S

A B
Figure 13-4  TEE of mechanical mitral prosthetic valve.  A, Transesophageal 4-chamber view of a bileaflet mechanical mitral valve replacement (MVR)
showing that with the transducer on the LA side of the valve, acoustic shadows (S) and reverberations (R) obscure the LV but not the LA side of the valve. B,
Real-time zoom 3D imaging of the valve shows the open leaflets in diastole but at a frame rate of only 10 Hz, compared to 50 Hz for 2D imaging. (B, From Otto,
CM: Textbook of Clinical Echocardiography, ed 5, Elsevier, 2013, Philadelphia.)

With aortic bioprosthetic valves, support is pro-


o 
oIf a baseline examination is not available,
vided either by stents, by attachment directly to recorded data are compared with published
the aortic wall (stentless valves), or by implant- data for that valve type and size.
ing an intact valve and root (sometimes called o A small amount of regurgitation is normal with
a “mini-root” approach). The aortic biopros- most prosthetic valves.
thetic prosthesis is well seen in long- and short-
axis views. Step 3A: Evaluate for Prosthetic Valve Stenosis
o When prosthetic valve dysfunction is suspected n Maximum and mean gradients are calculated with
on clinical grounds or based on transthoracic the Bernoulli equation from transvalvular veloci-
echocardiography (TTE) findings, both TTE ties (Fig. 13-5).
and TEE are recommended. n  Continuity equation valve area can be calculated
for aortic valve prostheses (Fig. 13-6).
Step 3: Record Prosthetic Valve Doppler n The mitral pressure half-time is measured for pros-
Data thetic valves in the mitral position (Fig. 13-7).
n Antegrade velocities across the prosthetic valve are
recorded with pulsed and continuous wave (CW) v  KEY POINTS
Doppler. LV outflow tract diameter is measured from the
o
n 
Prosthetic valve regurgitation is evaluated using two-dimensional (2D) images for calculation of
CW and color Doppler. valve area. The valve size may differ from the
subaortic anatomy and so cannot be substituted
v  KEY POINTS for this diameter measurement.
Both bioprosthetic and mechanical valves are
o o When measurement of outflow tract diameter
inherently stenotic compared with normal is difficult, the ratio of the velocity proximal to
native valves. the valve and in the orifice is used as a measure
o The normal antegrade velocity and pressure of stenosis severity.
gradient depends on the specific valve type, o With bileaflet mechanical valves, the small cen-
valve size, heart rate, and cardiac output tral orifice often results in high velocities due to
(Table 13-3). local acceleration, which should not be mistaken
o Ideally, Doppler data are compared with the for prosthetic valve stenosis (see Fig. 13-5, A)
patient’s own baseline postoperative examina- o The mitral pressure half-time is used to cal-
tion, done when the patient had fully recovered culate valve area, as for native mitral stenosis.
from surgery and was clinically stable. Often the pressure half-time itself is reported.
272 CHAPTER 13  Prosthetic Valves

o “Patient-prosthesis mismatch” describes a nor- n  valuation of prosthetic mitral regurgitation


E
mally functioning prosthetic valve that has a requires TEE; transthoracic imaging is nondiag-
valve area inadequate for the patient’s body size nostic due to shadowing and reverberation by the
(Fig. 13-8). valve prostheses.
n A small amount of prosthetic regurgitation is nor-
Step 3B: Evaluate for Prosthetic Valve Regurgitation mal; moderate to severe prosthetic regurgitation
n 
Prosthetic valve regurgitation is evaluated using or any degree of paraprosthetic regurgitation is
CW and color Doppler (Fig. 13-9). pathologic.

TABLE 13-3  Normal Reference Values of Effective Orifice Areas for the Prosthetic Valves
Prosthetic Valve Size (mm)
No. of Patients (%) 19 21 23 25 27 29
Stented Bioprosthetic Valves
Medtronic Intact 129 (10.2) 0.85 1.02 1.27 1.40 1.66 2.04
Medtronic Mosaic 390 (30.8) 1.20 1.22 1.38 1.65 1.80 2.00
Hancock II 53 (4.2) … 1.18 1.33 1.46 1.55 1.60
Carpentier-Edwards Perimount 59 (4.7) 1.10 1.30 1.50 1.80 1.80 …
St. Jude Medical X-cell 21 (1.7) … … … … … …
Stentless Bioprosthetic Valves
Medtronic freestyle 368 (29.1) 1.15 1.35 1.48 2.00 2.32 …
St. Jude Medical Toronto SPV 60 (4.7) … 1.30 1.50 1.70 2.00 2.50
Mechanical Valves
St. Jude Medical Standard 151 (11.9) 1.04 1.38 1.52 2.08 2.65 3.23
St. Jude Medical Regent 13 (1.0) 1.50 2.00 2.40 2.50 3.60 4.80
MCRI On-X 18 (1.4) 1.50 1.70 2.00 2.40 3.20 3.20
Carbomedics 3 (0.2) 1.00 1.54 1.63 1.98 2.41 2.63

From Blais C, Dumesnil JG, Baillot R, et al: Impact of valve prosthesis-patient mismatch on short-term mortality after aortic valve replacement.
Circulation 2003;108:983-988.

APX AV APX AV
1.0
1.0

m/s
m/s

MVR

AVR
4.0
A B5.0
Figure 13-5  CW Doppler aortic valve flows.  A, CW Doppler of antegrade flow across a normally functioning bileaflet mechanical aortic valve soon after
aortic and mitral valve replacement shows an early peaking systolic antegrade velocity of 3.4 m/s. Opening and closing clicks from both valves are seen. B, CW
Doppler of a stenotic bioprosthetic valve shows a maximum velocity of 4.5 m/s. In addition, the maximum velocity occurs in mid-systole with a rounded velocity
curve, consistent with an elevated mean gradient and significant valve obstruction. AVR, Aortic valve replacement; MVR, mitral valve replacement.
Prosthetic Valves  CHAPTER 13 273

v  KEY POINTS The exact pattern of normal prosthetic regurgi-


o
Prosthetic regurgitation often is first detected
o tation depends on the valve type—for example,
with CW Doppler due to the high signal- central with bioprosthetic valves, two eccentric
to-noise ratio, excellent bioprosthetic pen- jets with bileaflet mechanical valves.
etration, and wide beam geometry of CW o Pathologic prosthetic valve regurgitation on
Doppler. color Doppler often is paravalvular, has a larger
o Normal prosthetic regurgitation has a weak vena contracta and jet area, and lasts longer
CW Doppler signal and typically is brief in during the cardiac cycle.
duration. o Significant prosthetic regurgitation, especially
o A dense regurgitant CW Doppler signal is an of the mitral valve, may not be detectable on
indication for further evaluation (Fig. 13-10). transthoracic imaging.
o Normal prosthetic regurgitation on color Dop-
pler is spatially localized adjacent to the valve, Step 4: Evaluate Geometry and Function
has a small vena contracta and jet area, is n  After valve surgery, LV dilation and hypertrophy
through the prosthesis (not paravalvular), and is typically regress, but many patients have persistent
brief in duration. abnormalities.
n Systolic ventricular dysfunction often improves
SVLVOT = SVAVR after valve surgery, but diastolic dysfunction may
CSALVOT × VTILVOT = EOA × VTIAVR be evident for many years.
n 
LV geometry and systolic and diastolic function
EOA = (CSALVOT × VTILVOT)/VTIAVR
are evaluated in patients with prior heart valve sur-
gery as detailed in Chapters 6 and 7.
AVR v  KEY POINTS
LVOT After aortic valve replacement for aortic stenosis,
o
Leaflet left ventricular hypertrophy regresses and sys-
Strut tolic function improves, but diastolic dysfunction
Stented tissue valve may be chronic.
Sewing ring o After valve replacement for aortic or mitral
Figure 13-6  Schematic drawing of the continuity equation with a regurgitation, LV dilation and systolic dysfunc-
stented bioprosthetic aortic valve replacement (AVR).  Stroke volume tion improve in most patients, but a subset have
(SV) proximal to the valve in the LV outflow tract (LVOT) equals SV through irreversible LV dilation and systolic dysfunction.
the aortic valve replacement. SV at each site is equal to the cross-sectional
area of flow (CSA) times the velocity-time integral (VTI) of flow at that site.
o In patients with isolated mitral stenosis, LV size
LVOT flow is measured with pulsed Doppler from an apical approach. CSALVOT and systolic function usually are normal both
is calculated as a circle from a mid-systolic LVOT diameter measurement, before and after valve surgery.
and the VTI of flow through the valve prosthesis is measured with CW Dop- o Comparison of the early postoperative study
pler, usually from the apical approach. This equation then is solved for the with the preoperative exam helps distinguish
aortic prosthetic effective orifice area (EOA).

1.5

m/s

Figure 13-7  Transmitral pulsed Doppler flow.  In the apical 4-chamber view (left), mitral inflow across a stented bioprosthetic valve is directed toward
the ventricular septum, with a reversed direction of the normal LV diastolic flow vortex. Pulsed or CW Doppler (right) shows a normal inflow pattern with an E
and A velocity (in sinus rhythm). The diastolic mean gradient and the pressure half time are only slightly higher than expected for a normal native mitral valve.
274 CHAPTER 13  Prosthetic Valves

Bioprosthetic valve Mechanical valve

Figure 13-8  Prosthetic valve effective


orifice area. View of a bioprosthesis and
a bileaflet mechanical valve with the leaflets
in a fully open position. The area highlighted
in pink is the effective orifice area. Patient–
prosthesis mismatch is present when the ef-
fective orifice area is smaller than needed to
maintain a normal cardiac output at rest and
with exercise without an excessive increase
in transvalvular pressure gradient. (From Internal diameter Internal diameter
Pibarot P, Dumesnil JG: Prosthesis-patient
mismatch: definition, clinical impact, and
prevention. Heart 2006;92:1022-1029.) External diameter External diameter

CW:2MHz APX AVR CW:2MHz APX MV


6.0

2.0

m/s

m/s

2.0

Figure 13-9  Prosthetic aortic valve regurgitation. CW Doppler re-


corded from an apical approach shows prominent valve clicks and aortic
regurgitation. The diastolic aortic regurgitant signal is much less dense than
the antegrade systolic signal, consistent with mild regurgitation. 6.0

Figure 13-10  Transthoracic apical CW Doppler after mitral valve re-


residual ventricular abnormalities from new pair.  The antegrade flow velocity is increased to over 2 m/s and a systolic
signal is present. The systolic signal is consistent with mitral regurgitation
ventricular dysfunction. (not LV outflow) based on the timing of flow. The density of the signal sug-
gests that more than trivial regurgitation is present. TEE is needed for further
Step 5: Measure Pulmonary Pressures evaluation to avoid shadowing by the annular ring.
and Evaluate Right Heart Function
n  There is an immediate decrease in pulmonary v  KEY POINTS
pressures after valve surgery that is directly related Measurement of pulmonary pressures on the
o
to the fall in LA pressure (e.g., the passive compo- early postoperative study serves as the baseline
nent of pulmonary hypertension). for subsequent studies.
n The late decrease in pulmonary pressures is vari- o After mitral valve surgery, recurrent pulmonary
able and depends on the extent of irreversible hypertension might be due to prosthetic regurgita-
changes in the pulmonary vasculature. tion, which otherwise might be missed on TTE.
n 
Pulmonary pressures and right heart function are o Right ventricular (RV) systolic function usually
evaluated in patients with prior heart valve surgery improves when pulmonary pressures decline
as detailed in Chapters 6 and 7. after valve surgery.
Prosthetic Valves  CHAPTER 13 275

SPECIFIC VALVE TYPES


Aortic Valves
Bioprosthetic Aortic Valves
n Bioprosthetic aortic valves have three thin leaflets,
similar to a native aortic valve. Ao
n  With stented bioprosthetic valves, the three stents
are seen in both long- and short-axis views. LV
n The flow profile and hemodynamics are similar to
a native valve with only a small degree of central LA
regurgitation.
v  KEY POINTS
o Aortic bioprosthetic valves are well visualized
in long- and short-axis views both on transtho- A
racic imaging from a parasternal window and
on TEE from a high esophageal window. .88
o Both TTE and TEE may be needed for com-
plete evaluation when endocarditis is suspected
because each approach visualizes the part of
the valve that is obscured by the ring shadow
from the other approach. .88
o Antegrade flow across the valve is recorded
from the apical window using CW Doppler. Ao
Alignment with flow is usually not optimal on a LV
TEE study.
o Valve regurgitation is evaluated by color Dop- LA
pler in the short- and long-axis views of the
valve, with measurement of vena contracta, as
described in Chapter 12 for native valves, when
possible (Fig. 13-11). B
o Valve regurgitation also is evaluated with CW
Doppler from the apical view, with the velocity Figure 13-11  Severe bioprosthetic aortic regurgitation.  A, This bio-
scale, gain, and filters adjusted to demonstrate prosthetic aortic valve has a flail leaflet (arrow) with color Doppler showing
regurgitation (arrow) that fills the LV outflow tract in diastole, consistent with
the regurgitant flow signal (Fig. 13-12). severe regurgitation (B). Ao, Aorta.

CW:2MHz APEX AV
40dB 2*/+1/0/1
PW Depth=103mm
6.0 PW Gate= 2/0mm
PW Gain= 12dB

PW:2MHz

1.0

m/s
m/s

2.0
A .50 B
Figure 13-12  CW and pulsed Doppler of severe prosthetic aortic regurgitation.  A, In the same patient as Fig. 13-11, the CW Doppler signal confirms
severe regurgitation with an equal signal density for antegrade systolic and retrograde diastolic aortic flow. B, Pulsed Doppler recording in the proximal abdomi-
nal aorta shows holodiastolic flow reversal, again consistent with severe aortic regurgitation.
276 CHAPTER 13  Prosthetic Valves

n  hen aortic root replacement includes the sinuses


W
Transcatheter Aortic Valve Implantation of Valsalva, the right and left coronary ostium are
n  TTE imaging often is used to help with sizing of reimplanted into the prosthetic aorta (Fig. 13-13).
the transcatheter valve prosthesis.
n TEE provides additional data on annulus size and
v  KEY POINTS
shape and often is used for correct positioning of a The structure and dimensions of the aorta at each
o
transcatheter valve at the time of implantation. level (annulus, sinuses, sinotubular junction, and
n 
The echocardiographic appearance of an aortic midascending aorta) are measured in patients
prosthesis implanted by the transcatheter approach who have undergone aortic valve surgery.
is similar to a stentless bioprosthetic valve. o The coronary reimplantation sites are best seen
on TEE imaging.
v  KEY POINTS o Postoperative echocardiographic findings may
Aortic annulus diameter measured in the long-
o include paraaortic edema, hematoma, or surgi-
axis view at the aortic cusp insertion in mid- cal material. Review of the images with the sur-
systole is used to choose the correct valve size gical team is helpful in distinguishing expected
for implantation. postoperative findings from infection or bleeding
o 3D TEE aortic annulus measurement is recom- (Fig. 13-14).
mended when transthoracic views are suboptimal
(see Fig. 11-8). Imaging with other modalities may
be needed to fully evaluate annulus size and shape. PLAX
o The normal antegrade velocity across a trans-
catheter implanted aortic valve is lower than
for most bioprosthetic valves, typically about
2 m/s.
o Paravalvular regurgitation is common after Ao
transcatheter aortic valve implantation and is a
predictor of adverse outcomes (see Fig. 13-2).
Valved Conduits or Valve Resuspension
n The native aortic valve may be sutured inside a
tube graft replacement of the aortic root (called
valve resuspension or the David procedure).
n 
A combined bioprosthetic aortic valve with an
attached segment of aorta (either a homograft
or heterograft) may be used when endocarditis is
present or there is involvement of the aorta. A

PSAX

B
Figure 13-14  Aortic valve resuspension.  Replacement of the ascending
Figure 13-13  Coronary reimplantation.  In a patient with replacement of aorta and resuspension of the aortic valve complicated by periaortic post-
the ascending aorta and sinuses with resuspension of the native aortic valve operative hematoma and bioprosthetic edema (arrows), seen in paraster-
within the conduit, TEE imaging in a short-axis view of the aorta just superior nal long-axis and short-axis views. Ao, Aorta; PLAX, parasternal long-axis;
to the aortic valve showing the reimplanted left main coronary (arrow). PSAX, parasternal short-axis.
Prosthetic Valves  CHAPTER 13 277

o With valve resuspension or any other subcoro- other diagnostic evaluations such as computed
nary stentless valve implantation, the height tomographic imaging or fluoroscopy of valve
and symmetry of the commissures affects valve motion.
function, so these valves are imaged using a o Prosthetic aortic valve regurgitation is evalu-
higher-frequency transducer and zoom mode. ated with color Doppler in short- and long-axis
views of the valve (TTE or TEE) with identi-
Mechanical Aortic Valves fication of jet origin (valvular or paravalvular)
n Both stenosis and regurgitation of a mechanical and vena contracta width.
valve in the aortic position can be evaluated on o Normal regurgitation of a bileaflet mechanical
transthoracic imaging. valve typically consists of two or more eccentric
n  TEE is needed when the indication for echocar- small jets that originate at the closure points of
diography is bacteremia, fever, or embolic events. the valve occluders with the sewing ring.
n A mechanical valve may be used in a composite o CW Doppler is used to evaluate prosthetic aor-
aortic root and valve replacement, with coronary tic regurgitation based on the density and time
reimplantation (the Bentall or modified Bentall course of the diastolic regurgitant signal.
procedure). o Diastolic flow reversal in the descending
aorta, as with native valve regurgitation, is
v  KEY POINTS also useful for evaluation of prosthetic aortic
An aortic mechanical prosthesis is best imaged
o regurgitation.
in long- and short-axis views from the paraster-
nal transthoracic or high esophageal windows Mitral Valves
by TEE.
o Infection typically involves the paravalvular Mitral Valve Repair
region so imaging includes evaluation of the n  The most common mitral valve repair involves
aortic wall thickness, identification of the coro- resection of a segment of the posterior leaflet,
nary ostium, and visualization of the paraval- with a suture line in the midsegment of the poste-
vular region (Fig. 13-15). rior leaflet and placement of an annuloplasty ring
o Antegrade velocity is recorded using CW Dop- (Fig. 13-16).
pler from the apical window. Prominent valve n Other procedures used for mitral valve repair
opening and closing clicks often are seen. include transfer of a segment of the anterior leaf-
o A high antegrade velocity (and small calcu- let to the posterior leaflet, use of artificial chords,
lated valve area) for a bileaflet valve in the aor- suturing of the anterior and posterior leaflets
tic position may be due either to normal valve together in their midsegments (Alfieri repair), and
function (with a high velocity in the central a variety of other techniques.
slit-like orifice), patient prosthesis mismatch, n 
Percutaneous approaches to mitral valve repair
or valve stenosis. These conditions are differ- include deployment of a device in the coronary
entiated based on clinical information, other sinus to mimic an annuloplasty ring and a clip or
echocardiographic findings, and, in some cases, suture to mimic an Alfieri-type repair.

PAn

LA

Ao
Figure 13-15  Aortic pseudoaneurysm
adjacent to mechanical aortic valve pros-
thesis.  In a TEE long-axis view, a mechani-
LV cal aortic valve prosthesis is present. There
is a spherical echo-free space just superior
to the valve plane, partly filled with crescent
shaped echodensity. This finding is consis-
tent with a partially thrombosed aortic pseu-
80 bpm
doaneurysm. Ao, Aorta.
278 CHAPTER 13  Prosthetic Valves

evaluated based on mean pressure gradient and


v  KEY POINTS pressure half-time valve area, as for a native
Knowledge of details of the repair procedure is
o valve.
helpful for interpreting the echocardiographic o With a successful repair, there is no more than
findings. trace to mild (1+) residual mitral regurgitation.
o The mitral annuloplasty ring causes shadows and o Recurrent mitral regurgitation after valve repair
reverberations that may obscure mitral regurgi- is evaluated as for a native valve.
tation from the transthoracic approach; TEE is o An infrequent complication of mitral valve
indicated when regurgitation is suspected. repair is subaortic obstruction due to systolic
o Mitral valve repair may be associated with anterior motion of the mitral leaflets. This
a mild degree of functional stenosis, which is complication is related to the size and rigidity
of the annuloplasty ring.
Bioprosthetic Mitral Valves
n Bioprosthetic mitral valves are oriented with the
stents typically directed slightly toward the ven-
tricular septum.
n 
Imaging and Doppler evaluation of a biopros-
AMVL thetic mitral valve are similar to evaluation of a
native valve.
•  Shadowing and reverberations from the sewing ring
and stents decrease the accuracy of TTE for evalu-
ation of valve dysfunction; TEE is more accurate
PMVL when a prosthetic mitral valve is present (Fig. 13-17).

v  KEY POINTS
With a prosthetic mitral valve, LV inflow is
o
Annular ring
directed toward the ventricular septum, the
opposite of the normal diastolic vortex in
Quadrangular resection the LV.
Figure 13-16  Mitral valve repair.  View of the mitral valve from the LA o Recording of antegrade flows and calculation
side after valve repair, showing the suture in the posterior leaflet at the site of of pressure gradient and valve area are no dif-
resection of the prolapse segment and the annuloplasty ring. AMVL,Anterior ferent than for a native mitral valve.
mitral valve leaflet; PMVL, posterior mitral valve leaflet. (From Zurick AO,
Stewart WJ, Griffin BP: Intraoperative echocardiography in surgical and o Although the apical window usually provides
transcatheter mitral valve repair. In Otto CM: The Practice of Clinical Echo- a parallel alignment for Doppler recordings, in
cardiography, ed 4. Philadelphia, Elsevier, 2012.) some cases, the mitral inflow can be recorded

LA

LA

Ao

LV

LV

A B
Figure 13-17  Flail bioprosthetic mitral valve leaflet.  A stented bioprosthetic mitral valve with a flail leaflet (arrow, left) and an eccentric jet of severe mitral
regurgitation (arrow, right), seen on transesophageal but not transthoracic imaging. Ao, Aorta.
Prosthetic Valves  CHAPTER 13 279

from a parasternal window, depending on the regurgitant signal that is obscured by shadow-
orientation of the valve inflow stream. ing on color Doppler flow imaging.
o Prosthetic regurgitation is evaluated with CW o Other clues that suggest mitral prosthetic
and color Doppler, as for a native valve, but regurgitation on TTE include a high antegrade
TEE is considered when valve dysfunction is velocity across the mitral valve and recurrent
suspected because significant regurgitation may (or persistent) pulmonary hypertension.
not be detected on TTE due to acoustic shad- o TEE provides superior imaging of the poste-
owing of the LA. rior aspects of the prosthetic mitral valve and is
o A small amount of central regurgitation is nor- more accurate than TTE for diagnosis of pros-
mal for a bioprosthetic valve. thetic regurgitation.
o Clear definition of the leaflets and annular ring
Mechanical Mitral Valves allows visualization of the normal regurgitant
n T he valve occluders are best seen from the api- jets that originate at the closure plane of the
cal transthoracic or high esophageal window, occluders with the sewing ring.
using zoom mode to focus on the mitral valve o Paravalvular regurgitation originates out-
(Fig. 13-18). side the sewing ring, often has an identifi-
n Antegrade flow across the valve is recorded from able proximal isovelocity surface area on the
the apical window, using pulsed or CW Dop- ventricular side of the valve, and typically
pler, depending on the maximum transvalvular has a very eccentric jet direction in the LA
velocity. (Fig. 13-19).
n Evaluation for regurgitation requires TEE because o TEE imaging is used to guide catheter based
the LA is shadowed by the prosthesis itself, both interventions for closure of paravalvular regur-
from the parasternal and apical windows. gitation (Fig. 13-20).
o Pulmonary venous flow patterns in patients
v  KEY POINTS with mechanical mitral valves are affected by
Adjustments in the rotation of the image plane
o atrial rhythm, atrial mechanical function, and
from the standard views may be needed to show mitral valve hemodynamics, as well as by the
both leaflets, from both the transthoracic and presence of mitral regurgitation.
transesophageal approach. o Paravalvular regurgitation may be clinically
o CW Doppler is especially important for detec- important regardless of hemodynamic severity
tion of mechanical mitral valve regurgitation because it may be a sign of infection or may be
because the broad CW beam may detect a a cause of hemolytic anemia.

TEE TEE
Systole Diastole
LV

LV

A B
Figure 13–18  Bileaflet mechanical mitral valve.  Transesophageal images of a bileaflet mechanical mitral valve in systole and diastole, showing the two
parallel (arrows) open occluders in diastole.
280 CHAPTER 13  Prosthetic Valves

LA
MVR

R S
Figure 13-19  Paravalvular mitral regur-
gitation.  In a TEE 2-chamber view, a jet of
mitral regurgitation is seen originating at the
lateral aspect of the mechanical mitral valve
replacement (MVR), outside the sewing ring.
Reverberations (R) and shadows (S) from the
prosthetic valve obscure the LV. (From Otto,
CM: Textbook of Clinical Echocardiography, 86 bpm
ed 5, Elsevier, 2013, Philadelphia.)

to the chest wall and because the RA can be


evaluated from the parasternal window, without
shadowing by the valve prosthesis.
o TEE imaging is helpful when transthoracic
images are non-diagnostic.
o Antegrade flows are recorded using pulsed or
CW Doppler from the apical window for calcu-
lation of pressure gradients and pressure half-
time valve area.
o Prosthetic tricuspid regurgitation is evaluated
by standard approaches using CW and color
Doppler.
LA
LV Prosthetic Pulmonic Valves
n  Most pulmonic valve replacements are seen in
patients with congenital heart disease.
n Pulmonic valve substitutes include homografts, bio-
Figure 13-20  TEE guided intervention for paravalvular mitral regurgi-
tation.  3D imaging during the procedure is used to position the guide wire
prosthetic valves (either in a conduit or isolated),
across the paravalvular leak before placement of a transcatheter closure and (increasingly) transcatheter valve implantation.
device in the same patient as in Fig. 13-19. (From Otto, CM: Textbook of Clinical n Transcatheter pulmonic valves are typically placed
Echocardiography, ed 5, Elsevier, 2013, Philadelphia.) in patients with a prior surgically implanted con-
duit. The transcatheter valve is analogous to a
trileaflet bioprosthetic bioprosthesis mounted in
Tricuspid Valve Prostheses and Rings an expandable stent.
n  Tricuspid valve replacement is uncommon, but n Mechanical valves are occasionally used in the pul-
either a stented bioprosthetic or a mechanical monic position.
valve is used (Fig. 13-21).
n  In patients undergoing mitral valve surgery, a tri- v  KEY POINTS
cuspid annuloplasty ring often is placed if severe 
oVisualization of prosthetic pulmonic valves
tricuspid regurgitation is present or if there is from either transthoracic or TEE approaches
annular dilation with moderate regurgitation. often is limited in adults. Alternate diagnostic
n Evaluation of a prosthetic tricuspid valve is similar procedures, such as cardiac magnetic resonance
to evaluation of a mitral valve replacement. imaging or cardiac catheterization and angiog-
raphy, often are needed.
v  KEY POINTS o Antegrade velocity is recorded with pulsed or
o Tricuspid valve prostheses often can be fully CW Doppler in the parasternal short-axis or
evaluated on TTE because the valve is close RV outflow view.
Prosthetic Valves  CHAPTER 13 281

RV

CW:2MHz APX TV
4.0

LV

RA LA

m/s

A 2.0 B
Figure 13-21  Apical view of a bioprosthetic tricuspid valve.  Color Doppler shows proximal acceleration (arrow) and a stenotic jet inflow signal. CW Dop-
pler shows a high velocity and long pressure half-time, consistent with severe prosthetic stenosis.

Pulsed and color Doppler are used to docu-


o pressure difference is low, there may be little
ment the level of obstruction. Many of these evidence of a flow disturbance.
patients also have subvalvular or supravalvular o On CW Doppler, severe prosthetic pulmonic
pulmonic stenosis. Stenosis also can occur at the regurgitation is seen as a diastolic signal with
distal anastomosis site of the conduit or in the a density equal to antegrade flow and a steep
branch pulmonary arteries. slope, often reaching the baseline before the end
o Severe prosthetic regurgitation is seen as to- of diastole.
and-fro flow on color Doppler; because the
282 CHAPTER 13  Prosthetic Valves

THE ECHO EXAM

Prosthetic Valves
Transthoracic Examination TEE for Prosthetic Valves
Imaging Imaging
Valve leaflet thickness and motion Valve leaflet thickness and motion
LV size, wall thickness, and systolic function Examine atrial side of mitral prostheses
LA size LV size, wall thickness, and systolic function
Doppler Doppler
Antegrade prosthetic valve velocity Antegrade prosthetic valve velocity
Evaluate for stenosis Evaluate for stenosis
Search carefully for regurgitation Search carefully for regurgitation
Pulmonary artery pressures Pulmonary artery pressures

TTE Evaluation of Prosthetic Valves


Components Modality View Recording Measurements
Antegrade flow Pulsed or Apical Antegrade transmitral or Peak velocity (compare
velocity CW Doppler transaortic velocity to normal values for
valve type and size)
Measures of valve Pulsed and Apical Careful positioning to Mean gradient
stenosis CW Doppler obtain highest velocity Aortic valves: ratio of
signal LVOT to aortic velocity
Mitral valve: pressure
half-time
Valve regurgitation Color imaging and CW Parasternal, apical, Jet origin, direction Vena contracta width
Doppler SSN and size on color Intensity of CW Doppler
CW Doppler of each signal
valve Pulmonary vein systolic
Pulmonary vein flow flow reversal (MR)
Descending aorta flow Descending aorta flow
reversal (AR)
Pulmonary pres­ CW Doppler RV inflow and apical TR-jet velocity Calculate PAP as 4v2 of
sures IVC size and variation TR jet plus estimated
right atrial pressure
Left ventricle 2D or 3D imaging Apical Apical biplane images Biplane LV volumes and
of LV or 3D volumetric ejection fraction
dataset

AR, Atrial regurgitation; IVC, inferior vena cava; LVOT, left ventricular outflow tract; MR, mitral regurgitation; PAP, pulmonary artery pressure;
TR, tricuspid regurgitation; SSN, suprasternal notch.
Prosthetic Valves  CHAPTER 13 283

TEE Evaluation of Prosthetic Valves


Components Modality View Recording Limitations
Valve imaging 2D and 3D echo High esophageal Mitral valve in high Aortic valve prosthesis may shadow
esophageal 4-chamber anteior segments of the aortic
view valve.
Aortic valve in high With both aortic and mitral pros­
esophageal long and theses, the aortic shadow may
short-axis views obscure the mitral prosthesis.
Antegrade flow Pulsed or High esophageal Antegrade transmitral or Alignment of Doppler beam with
velocity CW Doppler or transgastric transaortic velocity transaortic valve flow may be
apical problematic, compare with TTE
data.
Measures of Pulsed and CW High esophageal Careful positioning to Mean gradient
valve stenosis Doppler or transgastric obtain highest velocity Aortic valves: ratio of LVOT to
apical signal aortic velocity (alignment may be
suboptimal)
Mitral valve: pressure half-time
Valve Color imaging High esophageal Document origin of jet Measure vena contracta, record
regurgitation and CW Dop­ with rotational and proximal flow pulmonary venous flow pattern,
pler scan acceleration, and jet search carefully for eccentric jets.
size and direction
Pulmonary CW Doppler RV inflow and TR-jet velocity Calculate PAP as 4v2 of TR jet plus
pressures apical IVC size and variation estimated right atrial pressure.
May be difficult to align Doppler
beam parallel to TR jet, correlate
with TTE data.

IVC, Inferior vena cava; LVOT, left ventricular outflow tract; PAP, pulmonary artery pressure; TR, tricuspid regurgitation.

Echocardiographic Signs of Prosthetic TTE shows:


Valve Dysfunction
LA anterior-posterior dimension 5.7 cm
Increased antegrade velocity across the valve
LV dimensions (diastole/systole) 6.2/3.8 cm
Decreased valve area (continuity equation or T1/2)
Increased regurgitation on color flow
Ejection fraction 56%
Increased intensity of CW Doppler regurgitant signal
Transmitral E-velocity 1.8 m/s
Progressive chamber dilation
Mitral pressure half-time 100 ms
Persistent LV hypertrophy TR jet velocity 3.2 m/s
Recurrent pulmonary hypertension IVC size and variation Normal

Example Color flow imaging shows ghosting and reverbera-


A 62-year-old man with a mechanical mitral valve tions in the left atrial region but no definite regurgi-
replacement 2 years ago for myxomatous mitral valve tant jet can be identified. CW Doppler shows a mitral
disease presents with increasing heart failure symptoms regurgitant signal that is incomplete in duration and
and a systolic murmur. He is in chronic atrial fibrillation. not as dense as antegrade flow.
284 CHAPTER 13  Prosthetic Valves

This transthoracic study is difficult to interpret acceleration region seen at the lateral aspect of the
without a previous study for comparison. The left annulus, a vena contracta width of 7 mm, and an
atrial and LV dilation and the borderline ejection eccentric jet directed along the posterior-lateral left
fraction may be residual from before the valve sur- atrial wall. The left pulmonary veins show definite
gery or could represent progressive changes after systolic flow reversal; the right pulmonary veins
valve replacement. Pulmonary artery pressure (PAP) show blunting of the normal systolic flow pattern.
is moderately elevated: These findings are consistent with severe parapros-
thetic regurgitation.
PAP = 4(VTR )2 + RAP = 4(3.2)2 + 10 (13-1) On TEE imaging, the LV was not well visualized
= 41 + 10 = 51 mmHg due to shadowing and reverberations from the mitral
prosthesis, although transgastric short-axis views were
Again, pulmonary hypertension may be residual or obtained, ejection fraction could not be calculated.
recurrent after valve surgery but the presence of pul- The maximum TR jet obtained on TEE echocardiog-
monary hypertension suggests the possibility of signif- raphy was 2.9 m/s. Because a higher jet was obtained
icant prosthetic mitral regurgitation. Although a clear on transthoracic imaging, the TEE jet most likely
regurgitant jet is not demonstrated due to shadowing underestimates pulmonary pressures.
and reverberations from the valve prosthesis, the high In summary, this patient has severe paraprosthetic
antegrade flow velocity with a short pressure half-time mitral regurgitation with left atrial and LV dilation,
and detection of regurgitation with continuous wave moderate pulmonary hypertension and a borderline
Doppler indicate that further evaluation is needed. ejection fraction. As is typical with prosthetic valves,
TEE echocardiography demonstrates a para- the combination of transthoracic and TEE echocar-
valvular mitral regurgitant jet with a proximal diography was needed for diagnosis.
Prosthetic Valves  CHAPTER 13 285

SELF-ASSESSMENT QUESTIONS

Question 1
An 87-year-old asymptomatic woman has relocated A.
Endocarditis with paravalvular abscess
to your area and presents to establish care. She has a B. Prosthetic valve mismatch
history of aortic stenosis and had undergone recent C. Aorto-ventricular fistula
aortic valve replacement. You obtain a baseline echo- D. Normally functioning aortic valve replacement
cardiogram (Fig. 13-22).
Based on the images provided, you conclude which
of the following?

Figure 13-22 

Question 2
This echocardiographic image (Fig. 13-23) was
obtained at the postoperative baseline study after bio-
prosthetic mitral valve replacement.
What is the most likely diagnosis for the structure
indicated by the arrow?
A. Vegetation
B. Valve strut
C. Mitral valve
D. Ruptured papillary muscle
E. LV thrombus

Figure 13-23 
286 CHAPTER 13  Prosthetic Valves

Question 3 Question 5
A 66-year-old man who had undergone mechani- A patient with a mechanical aortic valve prosthe-
cal mitral valve replacement for endocarditis now sis presents with exertional dyspnea. A transthoracic
presents with progressive exertional dyspnea. Based echocardiogram is performed, demonstrating a peak
on your clinical examination, you are suspicious for aortic jet velocity of 3.6 m/s with an LV outflow tract
prosthetic valve regurgitation. What additional infor- velocity of 1.4 m/s. He has recently had his chronic
mation from transthoracic imaging will most likely oral anticoagulation held due to severe esophageal var-
confirm your suspicion? iceal bleeding. Prior TTE images showed a peak aortic
A. Proximal isovelocity surface area calculation jet velocity of 2.7 m/s with an LV outflow tract veloc-
B. Pulmonary venous pulse wave Doppler tracing ity of 1.0 m/s. These data are most consistent with:
C. Transmitral CW Doppler tracing A. Prosthesis mismatch
D. Apical 4-chamber view color Doppler imaging B. Valve thrombosis
C. Bacterial endocarditis
Question 4 D. Hyperdynamic cardiac function
A 34-year-old patient presents with progressive
exertional dyspnea over the last year. He has a his- Question 6
tory of congenital aortic stenosis and underwent a A 71-year-old woman presented for a second opinion
Ross procedure 15 years ago. A transthoracic echo- regarding possible prosthetic stenosis. She had under-
cardiogram is ordered. Based on the patient history, gone a 19 mm pericardial bioprosthetic aortic valve
which of the following will most likely be seen on the replacement 2 years ago. On exam, she is an older
echocardiogram? anxious woman with a blood pressure of 120/80
A. Anterograde velocity of 3.0 m/s distal to the mmHg, pulse of 72 beats/min, body surface area of
neoaorta 1.9 cm2, and an aortic ejection murmur but no evi-
B. Autograft pressure half-time of 200 ms dence of heart failure. Echocardiography shows a
C. Complete heart block with ventricular escape normal appearing prosthetic valve with the following
rate 40 beats/min Doppler data (Fig. 13-24, A to C).
D. Anterograde velocity of 1.5 m/s in the pulmo- The most likely diagnosis in this patient is:
nary conduit A. No prosthetic valve dysfunction
B. Prosthetic valve stenosis
C. Prosthetic valve regurgitation
D. Patient-prosthesis mismatch

A C
Figure 13-24 
Prosthetic Valves  CHAPTER 13 287

Question 7
A patient with a stentless bioprosthetic aortic valve and Based on the imaging provided, you recommend:
fevers is admitted to the inpatient service. He is referred A.
Administer local intravenous lytic therapy
for a TEE, and the following image is recorded from B. Broaden intravenous antibiotic therapy
the midesophageal long-axis view (Fig. 13-25): C. Referral to cardiac surgery
D. Serial monitoring, repeat TEE in 1 year

Figure 13-25 

Question 8
A patient with a history of prior mitral valve annulo- A.
Peak mitral E flow velocity 1.8 m/s
plasty ring repair presents with recurrent progressive B. Mitral vena contracta 0.3 cm
dyspnea. A TEE is performed (Fig. 13-26). Based on C. Mitral diastolic mean gradient 5 mmHg
the image provided, additional imaging would likely D. Proximal isovelocity surface area radius 0.5 cm
demonstrate:

Figure 13-26 
288 CHAPTER 13  Prosthetic Valves

Question 9
This 88-year-old woman presented with worsening sounds, and bilateral pulmonary rales. The following
heart failure and hemoptysis. She had undergone Doppler tracings were recorded on the current study
bioprosthetic mitral valve replacement 12 years ago (Fig. 13-27, A and B).
for severe mitral stenosis with an early postoperative The most likely cause of her current symptoms is:
baseline echocardiogram that showed normal LV and A. Pulmonary embolus
RV size and function, normal prosthetic valve func- B. LV systolic dysfunction
tion and a pulmonary systolic pressure of 40 mmHg. C. Severe mitral regurgitation
On exam now she has a blood pressure of 100/70 D. Rheumatic aortic valve disease
mmHg, heart rate of 74 bpm with an irregular pulse, E. Mitral stenosis
a jugular venous pressure of 20 cm H2O, distant heart

B
Figure 13-27 

Question 10
Using the data shown for Question 9, calculate:
Pulmonary systolic pressure: ___________________
Mitral valve area: ____________________________
Prosthetic Valves  CHAPTER 13 289

Question 11
For each of the following three images (Figs. 13-28 to
13-30), the valve shown is a:
A. Ball-cage mechanical valve
B. Bileaflet mechanical valve
C. Stentless bioprosthetic valve

Figure 13-30 

Question 12
In the patient with this echocardiographic image (Fig.
13-31), which of the following clinical findings is most
likely present?
A. Elevated reticulocyte count
B. Diastolic murmur
Figure 13-28  C. Wide pulse pressure
D. S4 gallop
E. Thrombocytopenia

Figure 13-29 

Figure 13-31 
290 CHAPTER 13  Prosthetic Valves

ANSWERS

Answer 1: D by acoustic shadowing, and if a signal is detected, a


This patient has undergone an aortic valve replace- diagnosis of (some) mitral regurgitation is confirmed.
ment with a transcatheter bioprosthetic aortic valve Quantification of mitral regurgitation severity is rela-
replacement (TAVR). Color Doppler images show tively limited with CW tracings, but if a dense signal
paravalvular regurgitation along the anterior portion is identified, significant mitral regurgitation should
of the valve annulus. TAVR valves are positioned be suspected (Fig. 13-32). In this image, bright valve
within the patient’s native, stenotic valve. In patients occluder motion is seen at the beginning and end of
with heavy native valve calcification, incomplete systole, and there is a dense, systolic regurgitant signal
apposition of the TAVR against the native valve may directed away from the transducer.
lead to small, paravalvular regurgitation. Often, these
jets improve or resolve with endothelialization of the
prosthesis.
Spectral Doppler images in this patient dem- m/s
onstrated mildly increased anterograde velocities
through the TAVR (not shown). This is common in
TAVR valves because the maximal valve orifice size
is within the native valve annulus. A peak velocity of
2.5 m/s is within expected valve hemodynamics, and
is not consistent with prosthetic valve mismatch. The
patient is asymptomatic without clinical evidence of
endocarditis, making a paravalvular abscess or aorto-
ventricular fistula less likely.
Answer 2: C
8.0
This images shows the native mitral valve chords and
part of the mitral leaflet which were retained at the APX MV
time of mitral valve replacement. Maintenance of Figure 13-32 
mitral annular-papillary muscle continuity helps pre-
vent loss of LV systolic contractile function with surgi-
cal mitral valve replacement. Typically the prosthetic Additionally, a peaked early diastolic velocity
valve is inserted centrally and the posterior leaflets would suggest higher left atrial pressures, raising sus-
and chords are left connected to the papillary muscle picion for significant mitral regurgitation. Another
behind the prosthetic valve sewing ring. The anterior potential indicator for significant mitral regurgitation
leaflet may be partially retained or may be resected, would be an elevated tricuspid regurgitant jet velocity,
leaving freely mobile chordal remnants. A mitral veg- suggesting pulmonary hypertension, particularly in
etation would be more likely on the left atrial side of patients who had demonstrated normal range pulmo-
the valve; infection of the sewing ring with annular nary pressures previously.
abscess formation instead of a typical vegetation is With significant acoustic shadowing of the left
common with a prosthetic valve. Valve struts are more atrium, pulmonary venous tracings would be dif-
uniform in appearance and do not protrude this far ficult to reliably record. However, if visualized, sys-
into the LV cavity. A ruptured papillary muscle results tolic flow reversal would be consistent with significant
in a disrupted muscle head moving freely in the LV, mitral regurgitation. Acoustic shadowing at the valve
attached to the mitral valve; normal attachments of occluders hinders visualization of the vena contracta
the chords to the papillary muscle can be seen on this and a discrete proximal isovelocity surface area hemi-
image. An LV thrombus usually occurs in an area of sphere is rarely visualized.
regional dysfunction, often the apex, and is adherent
to the LV myocardium. Answer 4: B
In the Ross procedure, the patient’s native pulmonic
Answer 3: C valve (pulmonary autograft) replaces the diseased
Mechanical mitral valve occluders acoustically aortic valve, and a pulmonary homograft is used to
shadow structures distal to the valve. This often results replace the native pulmonary valve. The pulmonary
in hindering visualization of mitral regurgitation, par- autograft may include a portion of the native main
ticularly in the apical 4-chamber view where the left pulmonary artery (neoaorta). Late complications of
atrium is in the far field relative to the mitral valve the procedure include dilation of the neoaorta,
prosthesis. CW Doppler tracings are less affected stenosis/regurgitation of the autograft, or stenosis/
Prosthetic Valves  CHAPTER 13 291

regurgitation of the implanted pulmonary homograft correction would require another surgical procedure.
conduit. An autograft pressure half-time of 200 ms Both short and long term outcomes are worse when
would be consistent with significant regurgitation, patient prosthesis mismatch is present. This patient’s
and could account for the patient’s symptoms. An transvalvular mean gradient is only 14 mmHg and the
anterograde velocity of 3.0 m/s distal to the neoaorta LV outflow to aortic velocity ratio is 1.4/3.0 = 0.47,
suggests obstruction in the aorta; aortic complications which does not support a significant hemodynamic
post-Ross procedure are dilation of the neoaorta, not effect from the small prosthesis. Thus, although she
stenosis. Complete heart block is not a common com- meets the definition for patient prosthesis mismatch,
plication following Ross procedure. Typical antero- there is no significant outflow obstruction at this time.
grade velocities in the pulmonary conduit are in the
1.5-2.5 m/s range. A normal (or even lower) antero- Answer 7: C
grade pulmonary conduit velocity would not result in In the images provided, the patient has a pseudoan-
progressive exertional dyspnea. eurysm of the aortic-mitral intervavular fibrosis, with
flow entering the psuedoaneurysm on the LV side of
Answer 5: D the aortic valve aortic pseudoaneurysm. The prosthe-
This patient presents with exertional dyspnea and sis is seen just anterior to the pseudoaneurysm with-
has a recent history of severe bleeding; he is likely out typical acoustic shadowing, as this is a stentless
anemic. The significant interval increase in peak aor- valve prosthesis. Although no obvious vegetation is
tic jet velocity raises suspicion for prosthetic stenosis seen, other views are needed to fully evaluate pros-
or obstruction. However, in anemia, cardiac func- thetic valve anatomy. While broadening intravenous
tion is hyperdynamic with proportional increases in antibiotic therapy should be considered, the patient
both LVOT and transaortic jet velocities. The rela- is demonstrating signs of active infection with fevers
tive ratio of the left ventricular outflow velocity to and paravalvular infection, so that cardiac surgery
the peak aortic jet velocity (“dimensionless index”) is should be consulted for prosthetic valve endocarditis
preserved. In this case, although the aortic velocity complicated by pseudoaneurysm. Valve thrombosis is
is increased, the LVOT velocity is increased as well, not the key finding, so lytic therapy is not indicated.
and the ratio, or, dimensionless index was compara-
ble, consistent with hyperdynamic cardiac function Answer 8: A
rather than an obstruction. For patients with aortic The image provided is from a midesophageal long-
prostheses and no other thrombosis risk factors, oral axis view. On the left side of the image, there is a
anticoagulation may be temporarily held for non- dehiscence of the ring, evidenced by an echolucent
cardiac procedures without need for anticoagulation discontinuity of the bright ring from the valve annu-
bridging therapy. lus. The patient has recurrent symptoms consistent
with mitral regurgitation. A peak mitral E wave veloc-
Answer 6: D ity of 1.8 m/s is consistent with increased transmi-
The Doppler data show an aortic velocity of 3.0 m/s tral volume flow. A mitral vena contracta of 0.3 cm
with an LV outflow tract diameter of only 1.5 cm and and a PISA radius of 0.5 cm are not consistent with
an LV outflow tract velocity of 1.4 m/s. The circu- significant mitral regurgitation. 3D imaging from the
lar LV outflow tract cross sectional area (CSA) is 1.77 same patient shows the valve dehiscence posterior to
cm2. Aortic valve area calculated with the continuity the ring (Fig. 13-33).
equation is:

AVA = CSALVOT × VLVOT /VAo


= 1.77 cm2 × (1.4 m/s/3.0 m/s) aortic valve
= 0.8 cm2 (13-2)

When indexed for body size:

Indexed AVA = 0.8 cm2 /1.9 m2 = 0.43 cm2 /m2 (13-3)


These data are consistent with severe patient pros-
thesis mismatch, with an aortic valve indexed area
less than 0.65 cm2/m2. Ideally, patient prosthesis mis-
match is avoided by calculating the expected valve
area divided by body size before valve implantation;
dehiscence
if the expected valve area is too small, an alternate
valve choice or an aortic root enlarging procedure
can be considered. Once patient prosthesis mismatch
is present, decision making is more difficult because Figure 13-33 
292 CHAPTER 13  Prosthetic Valves

Answer 9: E velocity to this point, provides a T½ of 370 msec.


These Doppler recordings show a high velocity tricus- Mitral valve area is 220/T½ or 0.6 cm2. These
pid regurgitant jet, consistent with pulmonary hyper- findings are consistent with severe prosthetic mitral
tension, and a transmitral flow signal consistent with stenosis. Although tissue valve durability usually is
an elevated transmitral gradient and small valve area. longer in older patients, at surgery the bioprosthetic
The tricuspid regurgitant signal is identified based on valve was severely calcified with restricted leaflet
systolic flow with a long flow period relative to the motion.
QRS and with the typical rapid, followed by slow, rate
of rise in velocity with a late peaking curve. The trans- Answer 11
mitral flow curve is in diastole with a typical passive Fig. 13-28: C. This zoomed parasternal long-axis
flow pattern of an early diastolic peak and linear fall diastolic image of the aortic valve might be mis-
off in velocity throughout diastole. Atrial fibrillation taken for a normal native valve, given the thin nor-
is present with no discernable a-velocity. The slow mal appearance of the aortic valve leaflets. However,
diastolic decline in velocity is consistent with mitral there is increased echodensity in the paravalvular
stenosis. region, both anteriorly and posteriorly, consistent
Pulmonary embolus might be associated with with the extra tissue of a stentless bioprosthetic valve.
pulmonary hypertension but transmitral flow would This image emphasizes the importance of complete
be normal. LV systolic dysfunction would result in and correct clinical information for interpretation of
a reduced dP/dt on the mitral regurgitant velocity echocardiographic data.
signal, which is not shown here. Severe mitral regur- Fig. 13-29: A. This TEE midesophageal view
gitation would result in an increased antegrade trans- shows a mechanical mitral valve. Transmitral color
mitral velocity but the diastolic slope would be steep. Doppler flow shows a spherical echodensity displac-
Rheumatic aortic valve disease is present in about one ing color flow in the ventricular side of the jet. There
third of patients with rheumatic valve disease and is acoustic shadowing on both sides adjacent to the
the mitral stenosis signal appears similar in shape to color jet in the left ventricle. A bileaflet mechanical
aortic regurgitation. However, diastolic velocities are prosthesis would acoustically shadow the far field to
lower across the mitral compared to aortic valve, with all prosthetic material, which would include a portion
a diastolic blood pressure of 70 mmHg. the initial dia- of the center of the valve. In this case, acoustic shad-
stolic velocity for aortic regurgitation would be about owing is seen distal to the “ball,” not above it.
4 m/s. Fig. 13-30: B. This TEE image shows the typical
appearance of a bileaflet mechanical valve. The disks
are closed in systole, forming a “tent-shaped” closure
Answer 10 within the sewing ring. Distal to the valve the LV is
Pulmonary systolic pressure: 79 mmHg obscured by shadowing from the sewing ring and
Mitral valve area: 0.6 cm2 reverberations from the valve disks. The small dense
The tricuspid regurgitant velocity is 4.0 m/s, reflect- echoes on the atrial side of the sewing ring are likely
ing an RV to RA systolic pressure difference of 64 sutures.
mmHg. Images of the inferior vena cava are not pro-
vided to estimate RA pressure; the central venous Answer 12: A
pressure was estimated to be about 20 cm H2O (or 15 This is a TEE 2-chamber view of a patient with a
mmHg) based on physical examination of the neck mechanical mitral valve prosthesis showing an eccen-
veins. The conversion factor for units of pressure is tric paravalvular mitral regurgitant jet. Paravalvular
1.36 cm H2O for each mmHg. Thus, adding RA pres- mitral regurgitation can cause hemolysis resulting in
sure to the RV-RA pressure difference, the estimated an elevated reticulocyte count. Most often, hemolysis
pulmonary systolic pressure is 79 mmHg. is well tolerated and the patient is able to maintain
With this bioprosthetic valve, mitral valve area is a relatively normal red blood cell count, although
calculated from the mitral pressure half-time (T½), sometimes vitamin and iron supplements are needed.
as for native mitral valve stenosis. On the first beat, Rarely surgical or percutaneous intervention is needed
the peak transmitral velocity is 2.3 m/s, correspond- to close the paravalvular leak unless there also is a
ing to an instantaneous pressure gradient of 21 large volume of regurgitant flow. This patient likely
mmHg. The T½ is measured on the time axis from has a systolic (not diastolic) murmur. A wide pulse
this point, to the point on the diastolic deceleration pressure is typical for aortic, not mitral, regurgitation.
slope where the pressure drop is ½ the initial gradi- An S4 gallop will not be present as the ECG shows
ent. A pressure gradient of 11 mmHg, corresponds atrial fibrillation. The platelet count should be nor-
to a velocity of 1.66 m/s. Finding this point on the mal, although blood clotting likely is abnormal due to
Doppler signal, drawing a vertical line to the time warfarin anticoagulation for a mechanical valve and
axis and then measuring the time interval from peak atrial fibrillation.
14 Endocarditis
BASIC PRINCIPLES Look for Other Complications of Endocarditis
STEP-BY-STEP APPROACH Special Situations
Review the Clinical Data Right-Sided Endocarditis
Choose Transthoracic and/or Transesophageal Prosthetic Valves
Echocardiography Pacer/Defibrillator Leads
Examine Valve Anatomy to Detect Valvular Vegetations Staphylococcus aureus Bacteremia
Evaluate Valve Dysfunction due to Endocarditis THE ECHO EXAM
Evaluate for the Possibility of a Paravalvular Abscess or Echocardiographic Approach
Fistula SELF-ASSESSMENT QUESTIONS
Measure the Hemodynamic Consequences of Valve
Dysfunction

BASIC PRINCIPLES Vascular phenomenon (arterial emboli,


o
n  chocardiographic evaluation for endocarditis uses
E mycotic aneurysm, conjunctival hemor-
an integrated approach with transthoracic echo- rhage, etc.)
cardiography (TTE) and transesophageal echocar- o  Immunologic phenomenon (glomerulone-
diography (TEE), depending on the clinical setting phritis, rheumatoid factor, etc.)
and the initial echocardiographic findings. o Other microbiologic evidence
n The modified Duke criteria for infective endocar- o Definite endocarditis is based on the presence
ditis are the current clinical standard. of two major, or one major plus three minor,
n The primary goals of the echocardiographic
 or all five minor criteria. Possible endocarditis
examination in a patient with suspected or known is based on one major plus one minor, or three
endocarditis are to: minor criteria.
• 
Detect and describe valvular vegetations
• 
Quantitate degree of valve dysfunction
• 
Identify paravalvular abscess or other complications STEP-BY-STEP APPROACH
• 
Evaluate hemodynamics effects of valve dysfunction
on ventricular size and function and on pulmonary Step 1: Review the Clinical Data
pressures n 
Key clinical data in the patient undergoing echo-
• 
Provide prognostic data on clinical course and need cardiography for suspected endocarditis are:
for surgical intervention • 
Blood culture results
• 
History of underlying cardiac disease or intravenous
v  KEY POINTS drug use

Definite endocarditis is present when blood
o • 
Other evidence of endocarditis (fever, embolic
cultures are positive and diagnostic findings events, PR interval prolongation)
are present on echocardiography; these are • 
Any contraindication to TEE
called “major criteria” for the diagnosis of n  he clinical data help focus the echocardiographic
T
endocarditis. examination so that particular attention is directed
o 
Diagnostic echocardiographic findings for toward:
endocarditis are one of the following: • 
Detection of right-sided vegetations in patients with a
o Typical vegetation on a valve or prosthetic material history of intravenous drug use
o Paravalvularabscess • 
Comparison of the current study with previous
o New prosthetic valve dehiscence examinations in patients with underlying valve
o New valvular regurgitation pathology
o 
In the absence of the major criteria (blood • 
Additional imaging, often with TEE, of prosthetic
cultures and echocardiographic findings), the valves and pacer leads
minor criteria for diagnosis of endocarditis are:
o Predisposing heart condition or intravenous v  KEY POINTS
drug use o The sensitivity of echocardiography for detec-
o Fever tion of valve vegetations depends as much on

293
294 CHAPTER 14  Endocarditis

the diligence of the exam as on image quality; n TTE imaging is followed by TEE if transtho-

therefore, a pretest estimate of the likelihood of racic images are nondiagnostic, if a prosthetic
disease is helpful to the sonographer. valve is present, or if the patient has a high risk of
o Review of previous imaging studies before per- endocarditis.
forming the exam allows quick recognition of n TEE is an appropriate initial diagnostic approach
new abnormalities. in patients with a prosthetic valve or other intra-
o Clinical data are critical for interpretation of cardiac devices (such as pacer leads).
echocardiographic data. The echo appearance n In a patient with suspected or known endocarditis,
of a cardiac tumor, thrombus, and infected veg- TEE is recommended if clinical data suggest para-
etation are similar—the final diagnosis is based valvular abscess.
on integration of echocardiographic and clini-
cal data. v  KEY POINTS
o Clinical data determine the urgency and most TEE is more sensitive for detection of valve
o
appropriate initial diagnostic modality as well vegetations compared with transthoracic imag-
as the need for any subsequent studies. ing (a sensitivity of ∼90% vs. ∼70%) (Fig. 14-1).
o TEE is more sensitive for detection of paravalvu-
Step 2: Choose Transthoracic and/or
lar abscess compared with transthoracic imaging
Transesophageal Echocardiography (sensitivity greater than 90% vs. about 50%).
(Table 14-1) o TEE is the preferred approach in patients with
n Most centers perform TTE before TEE in patients prosthetic valves or other intracardiac devices
with suspected endocarditis but this decision (such as pacer leads) for detection of vegetations
depends on the clinical situation. and evaluation of valve dysfunction (Fig. 14-2).

TABLE 14-1 Indications for Echocardiography in Infective Endocarditis


Additional
Diagnostic Approach Diagnostic Options
Suspected Endocarditis
1. Detection of valvular vegetations (with or without TTE TEE if image quality
positive blood cultures) suboptimal
2. Known valve disease with positive blood cultures and TEE
nondiagnostic TTE
3. Staphylococcal bacteremia without a known source TTE TEE if image quality
suboptimal
Known Endocarditis
1. Evaluation of valve hemodynamics TTE
2. Detect and assess complications (abscess, perforation, TEE CT or CMR imaging
shunt)
3. Reassessment of valve function in high risk patients (e.g., TTE or TEE
virulent organism, clinical deterioration, persistent or
recurrent fever, new murmur, persistent bacteremia)
Prosthetic Valve Endocarditis
1. Diagnosis and complications TTE and TEE for aortic prosthetic
valves.
TEE for mitral prostheses.
2. Patient with a prosthetic valve and a persistent fever TTE; consider TEE
without bacteremia or a new murmur
Peri-operative Management
1. Preoperative evaluation in patients with known TTE and TEE
infective endocarditis
2. Intraoperative TEE in patients undergoing valve surgery TEE
for infective endocarditis

From Otto CM: Textbook of Clinical Echocardiography, ed 5, 2013, Philadelphia, Elsevier, 2013.
Endocarditis  CHAPTER 14 295

Ao

LV

LA

A B
Figure 14-1  Detection of valvular vegetations.  TTE (A) and TEE (B) views of the mitral valve in a patient with bacteremia. The large mobile mitral valve
vegetation is easily seen on the TEE images but was barely visible on the transthoracic study. This typical vegetation is attached to the upstream side of the
valve (atrial side of mitral valve), is not as echodense as the valve tissue, is irregular in shape, and has a chaotic pattern of motion that is separate from the
normal motion of the valve tissue. Ao, Aorta.

LA

LV
LV

A B
Figure 14-2  Prosthetic valve vegetation.  In this patient with endocarditis of a mechanical bileaflet mitral valve, the transthoracic study did not show valve
vegetations due to shadowing and reverberations by the prosthesis. These TEE images show a large irregular mass, consistent with a vegetation, attached to
the valve that prolapses into the LV in diastole (A) and the LA in systole (B).


oTransthoracic imaging provides more reli- often results in a nonparallel intercept angle
able measurements of left ventricle (LV) size between the Doppler beam and high-velocity jet.
and ejection fraction, because images of the o 
When TEE is contraindicated but endocar-
LV are often oblique or foreshortened on TEE ditis is suspected on clinical grounds, a repeat
views. transthoracic study in 5 to 10 days has additive
o Transthoracic imaging provides more accurate value, with the prevalence of diagnostic find-
Doppler evaluation of stenotic valves and esti- ings increasing from 20% on the initial study to
mation of pulmonary pressures because TEE 40% on the repeat examination.
296 CHAPTER 14  Endocarditis

on whether 3D imaging increases diagnostic


Step 3: Examine Valve Anatomy to Detect accuracy for endocarditis.
Valvular Vegetations o Other valve masses may be mistaken for an
n Valvular vegetations on echocardiography are seen infected vegetation, including (Fig. 14-7):
as an abnormal, irregular mass attached to the o Papillary fibroelastoma
valve apparatus (Fig. 14-3). o Partial flail mitral leaflet or chord
n Valvular vegetations typically are attached to the o Nonbacterial thrombotic endocarditis
upstream side of the valve leaflet (e.g., atrial side o Valve thrombus (especially with prosthetic
of the mitral or tricuspid valve, LV side of aortic valves)
valve) (Fig. 14-4). o Normal valve variants (such as Lambl
n 
Motion of a vegetation typically is chaotic, with a excrescences)
spatial range in excess of normal valve excursion and o Ultrasound artifacts
a temporal pattern of rapid oscillations (Fig. 14-5). o Valvular vegetations tend to decrease in size and
increase in echogenicity with effective therapy.
v  KEY POINTS However, some vegetations may still be present
o In addition to standard views of each valve, the years after active infection.
image plane is slowly moved from side to side
(or in a rotational sweep on TEE) because veg-
etations often are seen only in oblique views.
o Zoom mode, a narrow sector, high transducer
frequency, and harmonic imaging are used to LV
enhance details of valve anatomy (Fig. 14-6).
o An M-mode recording through suspected veg-
etation seen on two-dimensional (2D) imaging
helps distinguish vegetation from an artifact or
valve tissue, based on the pattern and speed of
motion of the structure. LA
o Vegetations may be missed on TTE; transesoph-
ageal imaging has a higher sensitivity for detec-
tion of vegetation due to improved image quality.
o The relationship of the vegetation to valve
anatomy may be better appreciated with three-
dimensional (3D) imaging but there are no data

LV

LV

Ao

Ao
LA

LA

Figure 14-3  Lambl excrescences versus valve vegetation.  Small linear


mobile echo densities on the ventricular side of the aortic valve in diastole B
may be a vegetation or a normal variant called Lambl excrescences, small
fibrous strands attached near the tip of each valve cusp that are more preva- Figure 14-4  Mitral valve vegetation.  Parasternal (A) and apical (B) long-
lent with age. This patient had positive blood cultures and associated aortic axis zoomed images of the mitral valve show an echodensity on the atrial
regurgitation, with a clinical course consistent with endocarditis. Ao, Aorta. side of the posterior leaflet, consistent with a vegetation. Ao, Aorta.
Endocarditis  CHAPTER 14 297

Step 4: Evaluate Valve Dysfunction due to v  KEY POINTS


Endocarditis Vegetations may impede complete valve clo-
o
n Vegetations are associated with distortion of valve sure, resulting in regurgitation at the coaptation
anatomy and destruction of valve tissue, typically plane with either native or prosthetic valves.
resulting in valve regurgitation (Fig. 14-8). o Valve destruction results in regurgitation due to
n The presence and severity of valve dysfunction are leaflet perforation or deformity of the leaflet edge.
evaluated no differently than in a patient with valve
disease of any cause (see Chapters 12 and 13).
n 
Valve regurgitation in a patient with endocarditis
often is acute, rather than chronic, in duration.
Lambl

Cal=10mm
0

Ao

LV

Ao
LA

LA

140
Figure 14-5  M-mode of aortic valve vegetation. A small linear ech- Figure 14-7  Lambl excrescence on the aortic valve in diastole.  A Lambl
odensity was seen in a patient referred for possible endocarditis. The excrescence can be difficult to distinguish from a vegetation but often is
M-mode shows fine oscillations of the mass in diastole, suggesting a vegeta- smaller, more linear and echodense, is not associated with valve dysfunction,
tion rather than nonspecific leaflet thickening. Ao, Aorta. and does not change in size or appearance on sequential studies. Ao, Aorta.

Ao Ao
LV

LA
LA

A B
Figure 14-6  Aortic valve vegetation.  A, This long-axis view at a standard depth shows a thickened prolapsing aortic valve in diastole. B, Using zoom mode
the valve is seen in more detail with real-time images showing independent rapid oscillating motion of the prolapsing tissue, suggestive of a vegetation. Ao, Aorta.
298 CHAPTER 14  Endocarditis

MR PISA radius = 0.76 cm CW:2MHz


Alias velocity = –0.43 m/sec
2.0

LV
RV
m/s

RA

A B 4.0 C
Figure 14-8  Mitral valve vegetation.  A, A large mass is seen on the LA side of the mitral valve consistent with a vegetation. Although the mass is distant
from the coaptation point, color Doppler shows significant mitral regurgitation (MR), which is quantitated with standard approaches, including the proximal
isovelocity surface area (PISA) method (B) and the CW Doppler signal (C).

.74
Regurgitation of prosthetic valves often is para-
o
valvular due to infection in the annulus with AVR
valve dehiscence (Fig. 14-9).
AR
o About 10% of patients with endocarditis do not
have significant valve regurgitation due to the
location of the vegetation at the leaflet base, .74

which does not impair valve function.


o Rarely, a large vegetation causes stenosis due to
obstruction of the native or prosthetic valve ori-
fice by the vegetation mass.
o Prosthetic valve stenosis can result from a small
infected vegetation or thrombus impinging on
normal disk excursion.
Step 5: Evaluate for the Possibility of a A
Paravalvular Abscess or Fistula
n A paravalvular abscess is present in 20% to 60% of 39dB 4 •/+1/0/ 1
.74
CW Focus=103mm
native aortic valve endocarditis cases and in about CW Gain =15dB
15% of mitral valve infections (Fig. 14-10).
n Paravalvular infection occurs in more than 60% of
.74
prosthetic valve endocarditis cases.
n On echocardiography, a paravalvular abscess may
CW:2MHz

be echolucent or echodense. 6.0


n A paravalvular aortic abscess often communicates AR
with the aortic lumen, appearing as an aneurysm
of the sinus of Valsalva (Fig. 14-11).
n 
Rupture of paravalvular infection into adjacent m/s
chambers results in an infected fistula (Fig. 14-12).
v  KEY POINTS
B
TEE is indicated when a paravalvular abscess
o 6.0
is suspected because the sensitivity of transtho-
Figure 14-9  Paravalvular infection.  A, An eccentric jet of aortic regurgita-
racic imaging is low. tion that originates anterior to the valve sewing ring is seen in a parasternal
o Aortic paravalvular infection often is recognized long-axis view. B, CW Doppler recorded from the apical window shows a similar
based on distortion of the normal contours of density of antegrade and retrograde aortic flow, suggesting significant regurgi-
the sinuses of Valsalva. tation may be present. AR, Aortic regurgitation; AVR, aortic valve replacement.
Endocarditis  CHAPTER 14 299

RVOT

Ao

Figure 14-10  Paravalvular abscess.  In this parasternal short-axis view,


there is increased echogenicity in the paraaortic region due to abscess for- Figure 14-12  Fistula formation.  Infection of an aortic valve prosthesis
mation. resulting in a fistula from the aorta to the RV outflow tract seen on TTE in a
parasternal short-axis view. The appearance is similar to a ventricular septal
defect, but Doppler interrogation showed both diastolic and systolic flow
more consistent with flow from the aorta into the RV. Ao, Aorta; RVOT, right
ventricular outflow tract.

LA

Ao

LV

Figure 14-11  Sinus of Valsalva aneurysm.  Infection of the aortic sinuses


can present as an asymmetric dilation, as seen with the left coronary sinus
in this image. Comparison with previous images and TEE imaging both are
helpful in distinguishing an infected sinus from benign congenital dilation.
Figure 14-13  Mitral leaflet perforation.  In this patient with aortic valve
endocarditis, infection has extended into the base of the adjacent anterior
mitral leaflet with thickening and a small perforation. Ao, Aorta.

Paravalvular aortic infection may extend into


o regurgitation) or into the left atrium, right
the base of the anterior mitral leaflet, resulting atrium (RA), or right ventricular outflow tract
in mitral leaflet perforation (Fig. 14-13). (resulting in a fistula).
o An aortic paravalvular abscess may rup- o With prosthetic aortic valves, infection may
ture into the LV (resulting in severe aortic result in an aneurysm of the aortic-mitral
300 CHAPTER 14  Endocarditis

intervalvular fibrosa (a space between the


aortic and mitral valve that communicates
Step 6: Measure the Hemodynamic
with the LV) (Fig. 14-14). Consequences of Valve Dysfunction
o 
A mitral paravalvular abscess may extend n Valve dysfunction due to endocarditis may result in
into the pericardium, resulting in purulent ventricular dilation and dysfunction or in pulmo-
pericarditis. nary hypertension.
n 
Because regurgitation often is acute, evidence of
chronic volume overload may be absent even when
regurgitation is severe.
v  KEY POINTS
o Evaluation of the patient with endocardi-
tis includes measurement of LV dimensions,
volumes, and ejection fraction as detailed in
Chapter 6.
o Pulmonary systolic pressure is estimated as
described in Chapter 6.
o Early (mid-diastolic) closure of the mitral valve
may be seen on M-mode when acute severe
LV aortic regurgitation is present due to the rapid
rise in diastolic LV pressure.
o The time course of the continuous wave
(CW) Doppler recording of valve regurgita-
tion may show evidence of hemodynamic
decompensation:
o A rapid decline in velocity in late systole with
mitral (or tricuspid) regurgitation suggests a
LA left (or right) atrial v wave.
o A steep diastolic deceleration slope with aor-
Ao tic regurgitation suggests acute regurgitation
with an elevated LV end-diastolic pressure
(Fig. 14-15).
o LV systolic function may be impaired, without
LV dilation, with acute severe aortic regurgita-
A tion due to endocarditis, possibly due to the
effects of systemic infection, combined with a
shift to the steep segment of the LV pressure vol-
ume curve.
LV Step 7: Look for Other Complications of
Endocarditis
n  More than one cardiac valve may be affected, due
either to primary infection at more than one site or
by direct extension of infection to adjacent structures.
n  Septic coronary artery emboli, resulting in myo-
cardial infarction, occur in 10% of patients.
n Endocarditis may occur at intracardiac sites other
LA
Ao than valve leaflets, including a mitral or tricuspid
valve chord, a Chiari network or Eustachian valve,
or the RA wall in a region abraded by the tip of a
central catheter.
n 
A pericardial effusion may be present, either as
a nonspecific sign of systemic infection or due to
B direct extension of infection.
Figure 14-14  Aneurysm of aortic mitral intervalvular fibrosa.  A, In the v  KEY POINTS
apical long-axis view, a curved pulsatile echo-free space (arrow) is seen
between the posterior aortic root and LA. B, Using zoom mode, the narrow
o Once a vegetation has been detected on one
neck where this aneurysm of the aortic mitral intervalvular fibrosa commu- valve, careful evaluation for infection of other
nicates with the LV is seen (arrow). Ao, Aorta. valves is needed.
Endocarditis  CHAPTER 14 301

The presence of a regional wall motion abnor-


o SPECIAL SITUATIONS
mality in a patient with endocarditis is con-
sistent with a coronary embolus from a valve Right-Sided Endocarditis
vegetation. n Only 6% to 13% of febrile intravenous drug users
o Intracardiac sites subject to injury, such as have endocarditis.
the RA wall in a patient with a central cath- n  In intravenous drug users, infection affects the
eter or the tricuspid valve in a patient with right side of the heart (predominantly the tricuspid
an indwelling right heart catheter, should be valve) in 75% of cases (Fig. 14-17).
carefully examined for evidence of infection n  Most cases of right-sided endocarditis in drug
(Fig. 14-16). users are due to Staphylococcus aureus, and persistent
infection or abscess formation requiring surgery
occurs in less than 25% of cases.
n  Pulmonary emboli due to right heart vegetations
CW:2MHz may result in elevated pulmonary pressures.
n Left-sided involvement occurs in 25% to 35% of
6.0 endocarditis cases in patients with a history of
intravenous drug use.
n 
TTE often is adequate for evaluation of tricuspid
valve endocarditis, but TEE may be needed to
AR exclude left heart involvement.
Prosthetic Valves
n  Blood cultures should be drawn before any anti-
biotic therapy in febrile patients with a prosthetic
heart valve.
n  TEE is indicated in all patients with a prosthetic
m/s heart valve and positive blood cultures.
n TEE should be considered in patients with a
prosthetic heart valve and suspected endocarditis
because transthoracic imaging is inadequate to
2.0
exclude prosthetic valve infection.
A n 
More than 50% of patients with prosthetic valve
endocarditis require surgical intervention.
CW:2MHz APX MV

2.0
PSAX
RV

m/s Ao

RA
MR

6.0
B
Figure 14-15  Acute valve regurgitation.  Regurgitation due to endocar-
ditis often has an acute onset. A, Acute aortic regurgitation shows a dense Figure 14-16  RA mass.  A rounded mass is seen in the RA attached to the
signal with a steep deceleration slope due to rapid equalization of aortic and free wall, near the distal tip of a central venous line, in this zoomed para-
LV pressures in diastole. B, With acute mitral regurgitation, an early fall-off sternal short-axis view. This might be a thrombus or an infected vegetation,
from peak velocity is due to an increased LA systolic pressure and v wave. depending on blood culture results and clinical evidence of infection. Ao,
AR, Aortic regurgitation; MR, mitral regurgitation. Aorta; PSAX, parasternal short axis.
302 CHAPTER 14  Endocarditis

RA

Figure 14-17  Tricuspid valve vegetation. 


In a TEE 4-chamber view, a mass is seen
on the right atrial side of the tricuspid RV
valve consistent with a vegetation (left).
Color flow (right) shows only mild tricuspid
regurgitation.

Pacer/Defibrillator Leads
n Blood cultures should be drawn before any antibiotic
therapy in febrile patients with an intracardiac device.
n  If pacer/defibrillator leads are not optimally LA
seen by transthoracic imaging, TEE should be
performed.
n Vegetations on the pacer wire are detected in less IVC
than 25% of cases on transthoracic imaging but
are seen in more than 90% on TEE when infection RA
is present (Fig. 14-18).
n 
The differential diagnosis of a mobile mass
on a pacer lead includes thrombus. Throm-
bus and vegetation cannot be distinguished by
echocardiography.
Staphylococcus aureus Bacteremia
n TEE is reasonable in patients with persistently pos-
itive blood cultures for Staphylococcus aureus, even if
the transthoracic study is negative. Figure 14-18  Infected pacer lead.  This TEE bicaval view shows a pacer
lead (arrow), traversing the RA chamber with an attached echogenic mass
that showed independent mobility. The appearance is consistent with veg-
etation or a thrombus. IVC, Inferior vena cava.
Endocarditis  CHAPTER 14 303

THE ECHO EXAM

Endocarditis
Duke Criteria (Short Version)
Definite endocarditis 2 major, or
1 major + 3 minor, or
5 minor criteria
Major criteria Bacteremia with a typical
organism
Echo evidence of endocarditis
Minor criteria Predisposing condition
Fever
Vascular phenomenon
Immunologic phenomenon
Other microbiologic evidence

Echocardiographic Approach

Suspected Endocarditis

Low risk High Risk


Prosthetic valve,
congenital disease,
previous endocarditis,
TTE heart failure,
community acquired
Staph bacteremia,
new AV block

Positive Equivocal Negative


TEE + TTE
Aortic valve
infection, Good
persistent fever, Suboptimal quality
AV block, images images
recurrent
bacteremia

Evaluate for
other source
TEE of infection
304 CHAPTER 14  Endocarditis

Echo Findings in Endocarditis


Finding Definition TTE TEE
Valvular vegetations Mass attached to leaflet with Sensitivity 50-80% Sensitivity 90-100%
independent motion Specificity 90-100% Specificity 90-100%
Leaflet destruction New or worsening valve Accurate for detection May better define mechanism of
regurgitation and quantitation of regurgitation
regurgitation
Abscess Infected area adjacent to valve, Sensitivity low Sensitivity 90-100%
usually in the aortic or mitral Specificity 90-100% Specificity 90-100%
annulus
Aneurysm or pseu- Localized dilation of a valve leaflet, TTE may show TEE more sensitive for
doaneurysm aortic sinus or aortic-mitral abnormal echo-free detection of aneurysm or
­intervalvular fibrosa (aneurysm) space around aorta pseudoaneurysm but other
or a contained rupture or in mitral-aortic imaging modalities (computed
(pseudoaneurysm) intervalvular fibrosa. tomography, magnetic
resonance imaging) may be
needed
Fistula Abnormal communication between Color or CW Doppler TEE images allow visualization
cardiac chambers or great may show abnormal of the location and size of the
vessels flow signal with intracardiac fistula.
timing and velocity
diagnostic for
location.
Prosthetic valve Detachment (partial) of the Paravalvular TEE more sensitive for evaluation
dehiscence prosthetic valve from the annular regurgitation is of prosthetic mitral valves
tissue typical. Abnormal Combination of TTE (anterior
motion of valve is aspect of valve) and TEE (for
diagnostic but rare. posterior aspect) needed for
aortic prosthetic valves

Example abdominal aorta. The TTE findings are diagnos-


A 28-year-old man with a known bicuspid aortic tic for endocarditis so that this patient now has two
valve presents with a 2-week history of fevers and major Duke criteria and a diagnosis of definite endocar-
fatigue. Physical examination shows a blood pressure ditis. With his bicuspid aortic valve, there may have
of 120/40 mmHg and a harsh diastolic murmur at been some degree of underlying aortic regurgitation.
the left sternal border. Blood cultures (three sets) are However, the normal LV size and the steep decel-
positive for Streptococcus viridans. With a predisposing eration slope of the continuous wave aortic regurgi-
factor, fevers, and positive blood cultures with a typi- tant jet are consistent with superimposed acute aortic
cal organism, the pretest likelihood of endocarditis is regurgitation.
very high (>90%). Aortic regurgitation is severe as evidenced by a wide
Transthoracic echocardiography shows a bicus- vena contracta and holodiastolic flow reversal in the
pid aortic valve with a mass on the ventricular side abdominal aorta.
of the leaflets with independent motion. LV size and The following day, a prolonged PR interval (first
systolic function are normal. Color flow Doppler degree atrioventricular [AV] block) is noted on the
shows aortic regurgitation with an eccentric jet and a electrocardiogram and TEE is performed. The TEE
vena contracta width of 7 mm; CW Doppler shows a shows an echolucent area in the aortic annulus region
dense signal with a deceleration slope of 120 ms, and consistent with abscess and the patient is referred for
there is holodiastolic flow reversal in the proximal prompt surgical intervention.
Endocarditis  CHAPTER 14 305

SELF-ASSESSMENT QUESTIONS

Question 1
Which of the following indications for echocardiogra- A.
Bacteremia
phy least contributes to a definite diagnosis of endo- B. Systemic embolic events
carditis as per the Duke modified criteria? C. Cardiac murmur
D. Fever
E. Immunologic phenomenon

Question 2
A 39-year-old man presents to the emergency depart-
ment with lethargy and fatigue which has been pro-
gressive for several weeks. He has no prior cardiac
history. Three months ago, he had developed a cough
and was diagnosed with a community-acquired pneu-
monia. He had completed at least two courses of
oral antibiotic therapy, prescribed for ongoing symp-
toms, by his physician before presentation. The fol-
lowing images (parasternal long-axis 2D view, and
AV spectral Doppler tracing from the apical view)
are obtained. List four abnormal findings from the
images provided (Fig. 14-19), and summarize the pri- A
mary diagnosis findings.
  

_________________________________________
_________________________________________
_________________________________________
_________________________________________

Figure 14-19 
306 CHAPTER 14  Endocarditis

Question 3 Question 4
A 38-year-old woman presents with several days of Transthoracic echocardiography was requested in a
fevers and fatigue. She has a history of prior bio- 69-year-old man with end-stage liver disease and a
prosthetic mitral valve replacement 6 years ago for fever. He has no history of cardiac disease and blood
endocarditis. Her course at that time was compli- cultures are negative (Fig. 14-21).
cated by complete heart block and a permanent The most likely diagnosis is:
pacer was placed. Physical examination at this time A. Bacterial endocarditis
demonstrates erythema and mild fluctuance around B. Lambl excrescence
the pacer pocket. Her cardiac examination is other- C. Papillary fibroelastoma
wise unremarkable without evidence of heart failure. D. Nonbacterial thrombotic endocarditis
TTE is performed and the following image is taken E. Ultrasound artifact
(Fig. 14-20). Otherwise, the echo shows normal leaflet
motion of the prosthetic valve without obvious vege­
tation. Valve hemodynamics are normal.
The most appropriate next step is:
A. Reassurance with watchful waiting
B. Repeat transthoracic echo in 2 to 4 weeks
C. Transesophageal echocardiography
D. Two-week course of IV antibiotics

LV Ao
RV

RA LA

Figure 14-21 

Figure 14-20 

Question 5
A 73-year-old man with a prior history of biopros-
thetic aortic valve replacement for aortic stenosis now
presents with fever, chills, and serial blood cultures
positive for Streptococcus viridans. He has been success-
fully initiated on intravenous antibiotic therapy and
a TEE is ordered. The following image is obtained
(Fig. 14-22). Based on the image provided, you con-
clude that the patient has now developed which com-
plication of endocarditis?
A. Paravalvular abscess
B. Prosthetic valve stenosis
C. Aortic pseudoaneurysm
D. Aortic regurgitation
E. Ventricular septal defect Figure 14-22 
Endocarditis  CHAPTER 14 307

Question 6
A 44-year-old man presents with a several-day history
of fevers, tachycardia, and fatigue. A transthoracic
echo is obtained. Based on the image shown (Fig.
14-23), what additional findings would you expect on
subsequent clinical evaluation?
A. Progressive AV block on electrocardiogram
B. Oxygen saturation step-up from RA to RV on
right heart catheterization
C. Pulse pressure 75 mmHg on blood pressure cuff
D. Prominent internal jugular venous pulsation on
physical exam

Figure 14-23 

Question 7   

A 61-year-old man with a history of multiple Regurgitant orifice area_______________________


myeloma presents with a 2-week history of increas- Regurgitant volume__________________________
ing lower extremity edema and is diagnosed with new Overall regurgitation severity ___________________
onset mitral regurgitation and probable endocarditis.
An echocardiogram is performed (Fig. 14-24). Based
on the data provided, calculate the following:

A B
Figure 14-24 
308 CHAPTER 14  Endocarditis

Question 8
A 72-year-old patient presents for clinical evaluation A.
Cardiac magnetic resonance imaging
with complaints of exertional dyspnea. He has a his- B. 3D echocardiography
tory of mitral valve endocarditis several years ago, C. Cardiac computed tomography
treated with a prolonged course of intravenous anti- D. Bicycle ergometery stress echocardiogram
biotic therapy. An echocardiographic image is pro-
vided (Fig. 14-25). Additional quantitative data from
the echo study reveal a regurgitant orifice area of
0.8 cm2 with a regurgitant volume of 70 mL. Which
of the following procedures would best aid diagnostic
evaluation of his mitral regurgitation?

LA

LV

Figure 14-25 

Question 9
A 62-year-old man is admitted to the intensive care
unit with hypotension and pulmonary edema. After
endotracheal intubation and stabilization, a bedside
echocardiogram is performed. This Doppler signal is
recorded from an apical window (Fig. 14-26).
The most likely diagnosis is:
A. Moderate mitral stenosis
B. Severe aortic stenosis
C. Ventricular septal rupture
D. Severe pulmonary hypertensions
E. Acute mitral regurgitation

Figure 14-26 
Endocarditis  CHAPTER 14 309

Question 10
A 24-year-old woman presents for evaluation of a
murmur. She has a history of aortic valve endocardi-
tis complicated by an aortic annular abscess for which
she underwent homograft aortic valve replacement 3 RVOT
months ago.
The findings in this parasternal short-axis view
(Fig. 14-27) and the CW Doppler recording of the
flow disturbance are most consistent with: RA
A. Aortic regurgitation
B. Ventricular septal defect
C. Aorta to LA fistula
D. Perforated anterior mitral leaflet
LA
E. Aortic annular abscess

m/s

4.0

Figure 14-27 
310 CHAPTER 14  Endocarditis

ANSWERS

Answer 1: C There is a mildly elevated anterograde velocity of 2.7


The Duke criteria for endocarditis diagnosis are m/s which is indicative of increase volume flow across
grouped into major and minor diagnostic criteria ini- the valve rather than significant stenosis. In this case,
tially described in 1994 and then revised (“modified”) the LVOT velocity (not shown) was also increased due
in 2000. A definite diagnosis of endocarditis is present to the aortic regurgitation at 1.3 m/s, such that the
for one of the following three conditions: presence of aortic valve dimensionless ratio was only 0.48. Severe,
two major criteria, OR presence of one major and acute aortic regurgitation has led to LV overload, with
three minor criteria, OR presence of five minor cri- mild LV dilation. The distance measurement indica-
teria. While a new or changing heart murmur is an tors along the edge of the window show centimeter
important clinical finding in patients with endocardi- markings. The end-diastolic dimension is enlarged at
tis, murmur characteristics are not a component of ∼6 cm. Because the regurgitation is relatively acute,
the Duke criteria. cardiac remodeling, such as would occur with chronic,
Major criteria for endocarditis diagnosis are: severe regurgitation, has not occurred, and LV size is
1 . Bacteremia with a typical endocarditis organism, not maximally dilated.
such as Staph. or Strep. species, with two positive
blood cultures drawn at least 12 hours apart. Answer 3: C
2 . Echocardiographic evidence of endocarditis such This patient’s clinical presentation is concerning for
as a large vegetation, abscess, or dehisced pros- a systemic infection, given several days of fever. The
thetic valve. physical examination is consistent with a pacer pocket
Minor criteria for endocarditis diagnosis are: infection, and the transthoracic image shows a vegeta-
1 . Predisposing condition such as a prosthetic valve, tion attached to the pacer lead in the right atrium. In
or prior known endocarditis these cases, antibiotic treatment alone for the primary
2 . Fever source (pacer) will not adequately clear the infection,
3 . Evidence of systemic embolic events and the device and infected material, including the
4 . Immunologic phenomenon such as glomerular pacer leads, need to be extracted. Watchful waiting
nephritis or Osler’s nodes would not be the correct course of action. Patients
5 . Positive blood cultures that do not meet the major should be initiated on a prolonged course of intrave-
criterion. nous antibiotic therapy, at least 4 to 6 weeks in dura-
tion once blood cultures have been obtained. Given
Answer 2 infected prosthetic material (pacer lead/pocket), care
The primary diagnosis findings in this case are: must be made to exclude concurrent endocarditis of
1 . Aortic valve thickening with oblique closure con- the prosthetic valve. Functional assessment of pros-
sistent with bicuspid aortic valve thetic valves in the mitral position is challenging due
2 . Mild LV enlargement to the relatively posterior location relative to the trans-
3 . Large right sinus of Valsalva aneurysm ducer. To ensure a more definitive evaluation of the
4 . Severe aortic regurgitation valve and exclude vegetation or other sequelae of
5 . Aggregate findings consistent with endocarditis endocarditis, such as paravalvular abscess, TEE imag-
This patient has no known prior cardiac history but ing is needed, given its higher sensitivity for detecting
has a markedly abnormal echocardiogram. The aortic vegetations.
valve is bright and thickened with an oblique closure
plane consistent with a bicuspid aortic valve. In this Answer 4: B
case, the leaflet was thickened additionally due to veg- A faint linear echodensity is seen in the LV outflow
etation, not optimally shown in the images provided. tract with a normal aortic valve and normal size LV
The parasternal long-axis view shows distortion of and LA. This appearance is most consistent with a
the aortic sinuses. There is a large sinus of Valsalva Lambl excrescence—a small filamentous structure,
aneurysm at the right coronary sinus which protrudes more often seen on the ventricular side of the valve,
into the right ventricular outflow tract. Color Dop- a normal variant which increases in prevalence with
pler flow into this aneurysm does not demonstrate a age. Endocarditis typically results in larger vegetations
fistulous connection, or rupture of the sinus which, if on the downstream side of the valve (LV side of aor-
present, would show flow in both systole and diastole. tic valve) in association with leaflet tissue destruction
There is severe aortic regurgitation. Spectral Doppler resulting in valve regurgitation. A papillary fibroelas-
across the aortic valve from the apical view shows a toma is a benign tumor attached to the valve leaflet
dense regurgitant jet with a steep deceleration slope, that appears as a mobile mass; although typically an
indicative of a rapid decrease in the diastolic aortic incidental finding, a larger papillary fibroelastoma
pressure gradient to nearly baseline at end-diastole. can be associated with thrombosis and adverse clinical
Endocarditis  CHAPTER 14 311

events. Nonbacterial thrombotic endocarditis typi- catheterization, this would manifest as a step-up in
cally appears as a globular mass (or masses) attached oxygen saturation in blood sampled from the RV rela-
to the upstream side (aortic side of aortic valve) in tive to the RA. Prominent internal jugular venous
patients with systemic inflammatory disorders. These pulsation is a clinical examination finding in patients
vegetations tend to be multiple, sessile masses on the with severe tricuspid regurgitation, not seen in this
leaflets. This might be an ultrasound artifact given case.
the smooth linear appearance, but there is no obvious
structure between the mass and transducer that might Answer 7:
cause a reverberation artifact. Regurgitation orifice area: 1.38 cm2
Answer 5: D Regurgitant volume: 166 mL
Overall regurgitant severity: Severe
The spectral Doppler image provided was taken   

from a midesophageal window with the transducer The color Doppler image has been optimized for
set to 28°. The guide image shows an oblique view measurement of the PISA with the baseline moved
of the aortic valve and left ventricular outflow tract so that the aliasing velocity for flow away from the
in the midportion of the image. The left atrium is transducer is 0.43 m/s which then is the velocity at
adjacent to the transducer and the left ventricle is the color change that defines the PISA. The radius
not well seen. The Doppler tracing shows an early of the PISA (from color change to valve plane) is 1.6
peaking systolic signal with a peak velocity of ∼3 cm. The surface area of the PISA is 2πr2 = 2(3.14)(1.6
m/s. There is a dense diastolic signal with a steep cm)2 = 16 cm2. The instantaneous regurgitant orifice
deceleration slope directed towards the transducer. area (ROA) is this surface area multiplied by the alias-
The aortic root is not well visualized, but there is ing velocity and then divided by the maximum mitral
no clear fluid collection or echolucency adjacent to regurgitant jet velocity (units of m/s are converted to
the aortic valve to suggest an abscess or pseudoan- cm/s to match the units of the PISA measurement
eurysm. The Doppler signal is not consistent with in cm2):
VSD, which would show both systolic and diastolic
left-to-right shunt flow, not the bidirectional flow
shown. Although the signal is not optimally aligned ROA = (16 cm2 × 43 cm/s)/500 cm/s = 1.38 cm2
with left ventricular outflow, the anterograde veloc-
ity is increased, at least 3 m/s, which should raise Regurgitant volume is calculated by multiplying the
suspicion of prosthetic valve stenosis. However, the ROA by the mitral regurgitant velocity time integral
signal is early peaking (a normal flow pattern). In (VTI), in this case, 1.38 cm2 × 120 cm = 166 cm3 or
this case, the relatively mild increase in anterograde 166 mL. This is severe mitral regurgitation, defined
velocities is due to increased transvalvular flow from as a ROA ≥ 0.4 cm2 and a regurgitant volume greater
the significant aortic regurgitation. The aortic regur- than 60 mL.
gitant signal is nearly as dense as the anterograde
signal, and the steep deceleration slope suggests Answer 8: B
rapid equalization of the diastolic pressure gradient Quantitative data for this patient are consistent with
between the aorta and the left ventricle. severe mitral regurgitation. The TEE images pro-
vided were from a mildly anteflexed midesophageal
Answer 6: C view at 0°. In this view, the anterior and posterior
This patient has abnormal thickening and symmetry mitral valves should be well defined with a central
of the aortic valve consistent with a large vegetation, coaptation point. This patient’s mitral valve is struc-
and it is likely that he has significant aortic regurgita- turally abnormal with an irregular contour at the
tion. This would manifest clinically with a widened posterior mitral annulus. There are two regurgitant
pulse pressure (difference between systolic and dia- jets with flow that crosses just above the mitral valve
stolic blood pressures). Progressive AV nodal block leaflets. The posterior regurgitant jet originates from
on ECG may occur in the setting of intramyocardial the irregular region of the leaflet, raising suspi-
abscess extension with involvement of the conduction cion for a leaflet perforation. 3D echocardiography
system. This parasternal view does not show a para- would provide excellent enface views of the mitral
valvular abscess (typically shown as an echolucent valve, which could aid in surgical planning. A 3D
region adjacent to the affected valve). Paravalvular image of the mitral valve from this patient is pro-
abscess is a more common manifestation of endocar- vided (Fig. 14-28).
ditis associated with prosthetic valves. Patients with Cardiac magnetic resonance imaging and cardiac
intramyocardial extension of an abscess to the point CT imaging provide excellent tomographic planes,
of interventricular septal rupture or an aortoventricu- but cannot provide enface views of valve leaflets due to
lar rupture would have oxygenated blood returning motion through the cardiac cycle. This patient has severe
to the right ventricle via the defect. On right heart mitral regurgitation, and additional testing to evaluate
312 CHAPTER 14  Endocarditis

example. Echocardiography reliability identifies the


valve dysfunction and may demonstrate the cause of
regurgitation—for example, vegetations consistent
with endocarditis or a wall motion abnormality and
AMVL papillary muscle rupture consistent with myocardial
infarction. In addition to stabilizing the patient, blood
cultures should be obtained when new severe regur-
gitation is present. Mitral stenosis can present acutely
if there is a concurrent medical issue (such as infec-
tion or anemia) with increased cardiac demand, but
Doppler would show the typical diastolic flow signal. A
high-velocity tricuspid regurgitant jet due to pulmo-
PMVL nary hypertension is longer in duration than mitral
Perforation regurgitation and the associated flow in diastole is
lower-velocity tricuspid, not mitral, inflow. It can be
challenging to separate a ventricular septal defect sig-
Figure 14-28  nal from mitral regurgitation, but duration typically
is longer and the mitral inflow signal is usually not
for the presence of severe regurgitation, such as bicycle seen in diastole. Instead, low-velocity left-to-right flow
stress ergometry, is not necessary for the diagnosis. across the septal defect may be seen in diastole along
with the high velocity systolic jet.
Answer 9: E
This high-velocity systolic signal directed away from Answer 10: D
the apex might be due to aortic stenosis, mitral regur- These findings are consistent with a perforated ante-
gitation, tricuspid regurgitation (if severe pulmonary rior mitral leaflet. The color Doppler image demon-
hypertension is present), or a ventricular septal defect. strates a flow disturbance entering the LA from either
All of these are possible with this clinical presenta- the aorta or the LV outflow tract; it is difficult to be
tion; for example, this patient could have a postmyo- certain of the position of this image plane relative to
cardial infarction ventricular septal defect or papillary the aortic valve in the short-axis plane. The corre-
muscle rupture. This signal can be identified as mitral sponding long-axis view showed the flow disturbance
regurgitation based on the timing relative to the QRS originating at the base of the anterior mitral leaflet.
(starts early) and associated diastolic flow signal. In Review of the operative report showed that the aortic
diastole, an LV filling curve is seen with an E and A homograft was trimmed to retain a segment of the
wave, with velocities typical of left (not right) heart anterior mitral leaflet base, which was used to repair
filling. The systolic signal begins and ends exactly at the native mitral valve. The perforation is between the
the end and beginning of the diastolic flow signal, homograft and native leaflet tissue. The CW Doppler
confirming this is mitral regurgitation, not aortic is diagnostic showing a high-velocity systolic waveform
stenosis, which would have slight gaps in the onset consistent with mitral regurgitation. Although the
and offset of flow due to isovolumic contraction and Doppler signal for a ventricular septal defect might be
relaxation. In fact, the denser aortic signal can be seen similar, there usually is a low-velocity diastolic compo-
“underneath” the mitral regurgitant signal on the first nent because of the slight diastolic pressure difference
beat. The shape of the velocity curve is suggestive of between the ventricles, and the color flow image is not
acute regurgitation, with a rapid decline in velocity in consistent with a ventricular septal defect. An aortic
late systole, instead of the more rounded waveform to LA fistula would exhibit high-velocity systolic and
of chronic regurgitation. With acute regurgitation, diastolic flow due to the higher aortic, relative to LA,
the rise in LV pressure (or v wave) as the regurgitant diastolic pressure. An aortic annular abscess may have
flow fills the small noncompliant chamber results areas of flow, but the CW Doppler velocity is low and
in a smaller pressure difference (and lower velocity) the color signal is localized to the area adjacent to the
between the LV and LA. Endocarditis can present valve. Aortic regurgitation is a likely complication of
as pulmonary edema or cardiogenic shock if there is endocarditis but, of course, is a diastolic flow signal
valve destruction with severe regurgitation, as in this from the aorta into the LV.
Cardiac Masses and Potential Cardiac
15 Source of Embolus
BASIC PRINCIPLES Vegetations
STEP-BY-STEP APPROACH Benign Valve-Associated Lesions
Left Atrial Thrombi Patent Foramen Ovale
Left Ventricular Thrombi Evaluation for Cardiac Source of Embolus
Right Heart Thrombi THE ECHO EXAM
Nonprimary Cardiac Tumors SELF-ASSESSMENT QUESTIONS
Primary Cardiac Tumors

BASIC PRINCIPLES cardiac mass (e.g., a left atrial mass in a patient


n  he first step in evaluation of a cardiac mass on
T with severe rheumatic mitral stenosis likely is an
echocardiography is to determine if the findings atrial thrombus).
are due to an ultrasound artifact or an actual ana-
tomic finding (Fig. 15-1).
n A prominent normal cardiac structure of a normal STEP-BY-STEP APPROACH
anatomic variant may be mistaken for an abnor-
mal mass. Step 1: Left Atrial Thrombi
n Ultrasound has limited utility for determination of n  Left atrial thrombi most often form in the atrial
tissue type; diagnosis of a cardiac mass is based on appendage, particularly in patients with atrial
location, attachment, appearance, and any associ- fibrillation (Fig. 15-2).
ated abnormalities. n  Thrombi may be seen in the body of the left
atrium (LA) with severe stasis of blood flow (e.g.,
v  KEY POINTS with mitral stenosis).
o Image quality for evaluation of a cardiac mass n TEE is required to exclude LA thrombi when clini-
is optimized by using: cally indicated.
o 
Highest transducer frequency with adequate tis-
sue penetration
o Acoustic access adjacent to the structure of interest
(e.g., transthoracic apical for ventricular thrombi
versus transesophageal echocardiography [TEE]
for atrial thrombi)
o Visualization of the motion of the mass with the
cardiac cycle
o Use of a narrow sector and zoom mode once a
mass is identified
o Careful gain and processing adjustments (exces-
sive or inadequate gain can obscure a mass)
o Off-axis views from standard image planes
A detailed knowledge of cardiac anatomy and
o
normal variants allows recognition of structures
that may mimic a cardiac mass.
o Echocardiography cannot identify the etiology
of a cardiac mass based on appearance. A dif-
ferential diagnosis for the echocardiographic
finding is based on the location, appearance,
size, mobility, physiologic effects, and other Figure 15-1  Normal cardiac structure.  In this apical 4-chamber view, an
apparent mass is seen in the LV chamber. Given the relationship of this mass
findings associated with the mass. to the anterior mitral leaflet, this most likely is a normal papillary muscle tip,
o Clinical data and other echocardiographic find- seen in oblique view. This diagnosis can be confirmed by scanning posteri-
ings often provide clues about the identity of a orly to show its connection to the lateral LV wall.

313
314 CHAPTER 15  Cardiac Masses and Potential Cardiac Source of Embolus

LA

Ao

A B
Figure 15-2  LA appendage thrombus.  A, Transesophageal view of the LA appendage shows an ovoid echodensity consistent with an atrial thrombus.
B, This finding is confirmed in an orthogonal view at 118° rotation using a magnified image. Ao, Aorta.

v  KEY POINTS
Transthoracic echocardiography (TTE) is not
o
sensitive for the diagnosis of LA thrombi due
to the distance between the transducer and LA
(limiting image quality at that depth) and the
small size and location in the atrial appendage
of most thrombi.
o The LA appendage may be visualized on trans-
thoracic imaging in a parasternal short-axis
view or in an apical 2-chamber view, but image
quality often is limited.
o TEE images of the LA appendage are obtained
from a high esophageal position. Evaluation
includes:
o 
Use of a high transducer frequency (typically 7
MHz)
o A narrow image sector and zoom mode
o Visualization in at least two orthogonal views, typi-
cally in views rotated to 0° and 60°. Use of simul- Figure 15-3  Normal atrial appendage anatomy.  Transesophageal im-
taneous biplane imaging with a three-dimensional aging of the LA appendage at about 50° using a 7-MHz transducer frequen-
cy. The normal ridge between the LA appendage and left superior pulmonary
(3D) probe is optimal vein is clearly seen (arrow). The LA appendage was imaged in several planes
o Pulsed Doppler recording of atrial appendage to evaluate for possible thrombus. The small circular echolucent structure
flow with the sample volume about 1 cm from seen between the LV outflow tract and LA is a cross-section of the circum-
the junction of the atrial appendage with the LA flex coronary artery.
chamber

oThe normal Doppler velocity with atrial
contraction is more than 0.4 m/s; lower o Reverberation artifact from the ridge between
velocities in sinus rhythm suggest contractile the left upper pulmonary vein and LA append-
dysfunction. age may hinder definitive exclusion of an
o The LA appendage has normal trabeculations appendage thrombus.
that are distinguished from thrombus by their
continuity with and echogenicity similar to the Step 2: Left Ventricular Thrombi
appendage wall, as well as their lack of inde- n Left ventricular (LV) thrombus formation occurs in
pendent mobility (Fig. 15-3). regions of blood flow stasis or low-velocity flow.
Cardiac Masses and Potential Cardiac Source of Embolus  CHAPTER 15 315

RV

Ao

LV
LV

LA

A B
Figure 15-4  LV mass.  A, In a low parasternal long-axis view, an echogenic mass is seen in apical region of the LV. B, In the short-axis view, the rounded
mass is seen adjacent to the septum. This finding may be an LV thrombus but is atypical given the location and lack of an underlying wall motion abnormality.
Ao, Aorta.

n  V thrombi most often form in an akinetic or


L
dyskinetic apex after myocardial infarction (Fig.
15-4).
n LV thrombi also are seen in patients with severely
reduced LV systolic dysfunction.
v  KEY POINTS
o TTE from the apical window is the optimal
approach to detection of LV thrombi, with a
sensitivity of 92% to 95% and a specificity of
86% to 88%.
o Detection of LV apical thrombi is enhanced
by:
o A steep left lateral decubitus patient position
on a stretcher with an apical cutout
o Use of a high transducer frequency (typi-
cally 5 to 7 MHz)
o Standard and oblique image planes of the
apex, especially medial angulation from a
lateral transducer position
o A shallow depth setting
o Myocardial trabeculations are differentiated
from thrombi by their linear shape with an ech-
odensity similar to and attachment to the myo-
cardium (Fig. 15-5). Figure 15-5  LV web.  An echodensity was seen in the LV apex. The trans-
o Left echo contrast is helpful in identifying ducer frequency was increased to 4 MHz, the focal depth decreased, and the
thrombus when image quality is suboptimal. transducer moved medially and angulated laterally from the apical position
o Transesophageal imaging is not sensitive for for further evaluation. The linear echo traversing the apex is consistent with
prominent trabeculation, not thrombus, because it connects with myocar-
diagnosis of LV apical thrombi because the dium at both ends.
apex is in the far field of the image and the
true apex may not be included in the image
plane. n  hrombi also may form on permanent pacer leads
T
in the right atrium (RA) or right ventricle (RV).
Step 3: Right Heart Thrombi n Peripheral venous thrombi may embolize to the
n 
RA thrombi may be seen in patients with central right heart and become entangled in the tricuspid
lines that abrade the RA wall. valve chords or a RA Chiari network (Fig. 15-6).
316 CHAPTER 15  Cardiac Masses and Potential Cardiac Source of Embolus

LA
LA

RA
SVC

RA

A B
Figure 15-6  Thrombus in transit.  A, In this TEE long-axis view of the RA, a large, echogenic, tubular, mobile mass is seen. B, Slight medial turning of the TEE
probe demonstrates that the mass originates from the region of the superior vena cava. By imaging in multiple planes, the attachment of this mass to a chronic
indwelling catheter was demonstrated. The location, clinical setting, and appearance of the mass are most consistent with thrombus. SVC, Superior vena cava.

KEY POINTS
o Normal echogenic structures in the RA that
may be mistaken for a thrombus include:
o Eustachian valve or Chiari network (Fig. 15-7)
o Crista terminalis (Fig. 15-8) LA
Eustachian valves and Chiari networks are
o
IVC
thin filamentous structures that extend from
the region of the inferior vena cava toward the
superior vena cava. The bright mobile echoes SVC
of a Chiari network may look similar to echo
contrast in the RA.
o The RA and RV are examined in parasternal
short-axis and RV inflow views, in the api- RA
cal 4-chamber view, and from the subcostal
window.
o Transesophageal imaging provides improved
visualization of the right heart when thrombi
are suspected.
Step 4: Nonprimary Cardiac Tumors
n  Nonprimary cardiac tumors are 20 times more
common than primary cardiac tumors. Figure 15-7  Eustachian valve.  In this TEE bicaval view, a linear mobile
n Nonprimary tumors can involve the heart by: echo originating from the junction of the inferior vena cava (IVC) and RA
• Direct extension is seen, consistent with a normal eustachian valve. In some patients this
embryologic remnant is more extensive, forming a network of filamentous
• Metastatic spread of disease
strands extending from the region of the inferior to the superior vena cava
• Production of biologically active substances (SVC). This finding, called a Chiari network, may appear on transthoracic
• Side effects related to treatment of the primary tumor echocardiography imaging as bright mobile echoes with chaotic motion in
n  onprimary cardiac tumors most often involve the
N the RA often best appreciated in parasternal short-axis, RV inflow, and sub-
pericardium but also may invade the myocardium. costal 4-chamber views.
They rarely appear as intracardiac masses (Fig. 15-9).
o Breast
v  KEY POINTS o Leukemia
o The most common nonprimary cardiac tumors, o Stomach
in order of frequency, are: o Melanoma
o Lung o Liver
o Lymphoma o Colon
Cardiac Masses and Potential Cardiac Source of Embolus  CHAPTER 15 317

RV

SVC
RA RA

A B
Figure 15-8  Crista terminalis.  A, This TEE long-axis image of the superior vena cava (SVC) and RA demonstrates the crista (arrow), the ridge at the junction
of the trabeculated and smooth segments of the RA wall. B, The crista terminalis often is seen in the transthoracic apical 4-chamber view as a slight bump on
the superior aspect of the RA wall.

LV

RV

LA RA
RA

A B
Figure 15-9  RA mass.  A, An apical 4-chamber view shows an inhomogeneous mass either attached to or invading the RA free wall. B, A magnified view
shows the anatomy in more detail but does not provide a tissue diagnosis. This mass clearly is not an artifact, thrombus, vegetation, or normal variant. A benign
primary cardiac tumor is unlikely because the appearance is atypical for a myxoma or fibroma, given the apparent involvement of the atrial wall. Thus, this most
likely is a metastatic tumor of the heart or, less likely, a primary cardiac malignancy.

All these tumors may involve the pericardium by


o fraction by echocardiography often is recom­
direct extension (breast, lung) or by metastatic mended.
spread, presenting with a pericardial effusion, o Radiation therapy that included cardiac structures
sometimes with tamponade physiology. in the treatment field may have very late (20 years
o Renal cell carcinoma may extend up the infe- or greater) adverse cardiac effects, including valve
rior vena cava into the RA and may be removed disease, accelerated coronary atherosclerosis, peri-
surgically en bloc with the primary tumor. cardial constriction, and myocardial fibrosis.
o Carcinoid heart disease is characterized by o TTE in standard views usually is adequate for
thickening and shortening of the right heart evaluation of nonprimary cardiac tumors, but
valve leaflets, resulting in pulmonic and tricuspid TEE provides improved image quality when
regurgitation (Fig. 15-10). needed.
o Some forms of chemotherapy affect myocardial o 3D imaging may be helpful in defining the rela-
function, so periodic monitoring of ejection tionship of the mass to other cardiac structures.
318 CHAPTER 15  Cardiac Masses and Potential Cardiac Source of Embolus

RV

RA

A B

C
Figure 15-10  Carcinoid heart disease.  A, In a right ventricular inflow view the shortened thickened tricuspid leaflets (arrows) are pathognomonic for a
diagnosis of carcinoid disease. B, Color Doppler shows a wide jet of tricuspid regurgitation. C, CW Doppler shows low velocity forward and reverse (arrow) flow
across the tricuspid valve consistent with severe regurgitation and normal pulmonary pressures.

Step 5: Primary Cardiac Tumors o Angiosarcoma (malignant)


n Primary cardiac tumors in adults usually are histo- o Rhabdomyosarcoma (malignant)
logically benign. Myxomas most often are seen in the LA (75%
o
n 
Benign cardiac tumors result in adverse clinical of cases), attached by a narrow stalk to the
outcomes due to both: center of the interatrial septum. Myxomas less
• Obstruction of blood flow often are seen in the RA, LV, and RV.
• Embolization o A pericardial cyst is a single or multilobed sac
n  rimary cardiac tumors most often present on
P lined by mesothelium that communicates with
echocardiography as an intracardiac mass. the pericardial space. Pericardial cysts are rare
but, when present, most often are seen adjacent
v  KEY POINTS to the RA.
o The most common primary cardiac tumors in o Papillary fibroelastomas typically are small masses
adults, in order of frequency, are: attached to the downstream side of a cardiac valve.
o Myxoma (Fig. 15-11) The appearance is similar to a vegetation (except
o Pericardial cyst that vegetations usually are on the upstream side
o Lipoma of the valve) but blood cultures are negative and
o Papillary fibroelastoma (Fig. 15-12) clinical signs of endocarditis are absent.
Cardiac Masses and Potential Cardiac Source of Embolus  CHAPTER 15 319

LV

LV

/
RA

9
LA

9
Ao

i r
LA

a h
t
A B

r/
Figure 15-11 Atrial myxoma. The location and smooth contour of this LA mass, seen in an apical long-axis view (A) and a 4-chamber view (B), is consistent


with an atrial myxoma. Ao, Aorta.

s e
/r u
.t c
a
RVOT
PA

/: / k PA
RV

s
LA

tt p
A B

h
Figure 15-12 Pulmonic valve papillary fibroelastoma. In a biplane parasternal view, a small mobile mass (arrow) is seen attached to the pulmonic valve


but with independent motion. The patient had no systemic symptoms and blood cultures were negative. The size and shape of this mass have been stable on
annual examinations over the past 5 years suggesting a diagnosis of papillary fibroelastoma. RVOT, Right ventricular outflow tract.

o Lipomatous hypertrophy of the interatrial sep- o Evaluate any associated pericardial effusion and





tum is common, with a typical appearance of signs of tamponade
sparing of the fossa ovalis. If in doubt, computed o Often both transthoracic and transesophageal


tomographic imaging confirms adipose tissue. imaging are needed to fully evaluate a cardiac
o Malignant primary cardiac tumors are rare, tumor. Masses located in the LA may be missed


usually seen as an intracardiac mass. on transthoracic imaging (Fig. 15-13).
o The goals of echocardiography in patients with
Step 6: Vegetations


a cardiac tumor are:
o Define the location and extent of tumor involvement n Vegetations are infected or noninfected masses of





o Evaluate obstruction or regurgitation due to the platelets and fibrin debris, typically attached to a



tumor valve leaflet.
320 CHAPTER 15  Cardiac Masses and Potential Cardiac Source of Embolus

n The vegetations of nonbacterial thrombotic endo- combination of clinical and echocardiographic





carditis are small and attached to the downstream findings.
(compared with upstream with infective vegeta- o TEE is more accurate for diagnosis of nonbac-



tions) side of the valve (Fig. 15-14). terial valve vegetations compared with trans-
n Infective vegetations are discussed in Chapter 14. thoracic imaging.



o Valve involvement by noninfected vegetations
v KEY POINTS



is seen in patients with systemic inflammatory

o The most critical step in evaluation of a patient diseases (i.e., systemic lupus erythematosus) and

/


with an intracardiac mass, especially a valve some malignancies.
vegetation, is to obtain blood cultures for pos-

9
sible infective endocarditis. Step 7: Benign Valve-Associated Lesions
Like infective endocarditis, nonbacterial throm- n Nodules of Arantius are small nodules at the cen-

9
o





botic endocarditis is diagnosed based on a tral coaptation points of the semilunar valves.

i r
h
TEE TEE LA

a
LA

r/ t
e
LV

s
Ao

/r u
.t c
RVOT

A B

a
Figure 15-13 Aortic valve papillary fibroelastoma. TEE imaging in long- (A) and short- (B) axis views of the aortic valve shows a mobile mass attached

k


to the aortic valve. There was no evidence for endocarditis or a systemic inflammatory disease so the mass most likely is a papillary fibroelastoma. Ao, Aorta;

/: /
RVOT, right ventricular outflow tract.

s
tt p
LA
LA

h
Ao

LV

A B

Figure 15-14 Nonbacterial thrombotic endocarditis. A small mass (arrow) is seen on the right coronary cusp of the aortic valve on this TEE short-axis


image (A) with the long-axis view (B) showing small masses at the leaflet base and at the leaflet tip. These masses showed independent motion in real time
suggestive of vegetations. The patient had no clinical signs of endocarditis and blood cultures were negative, so these findings may be due to nonbacterial
thrombotic endocarditis. Ao, Aorta.
Cardiac Masses and Potential Cardiac Source of Embolus  CHAPTER 15 321

n Lambl excrescences are thin, mobile, linear ech- o Lambl excrescences also are more common in





odensities seen on the downstream side of a valve, older patients, are most often seen in the LV
most commonly the aortic valve. outflow tract, and may be mistaken for a veg-
n Calcification of the posterior mitral annulus is etation. They less often are seen on the LA side



common in the elderly. of the mitral valve.
o Caseous calcification of the mitral annulus
v KEY POINTS



is a rare variant of mitral annular calcifica-

o Nodules of Arantius typically are more promi- tion, appearing as a smooth, round perian-

/


nent with age on the aortic side of the valve. nular mass with a central echolucent zone by
echocardiography.

9
SALINE
Step 8: Patent Foramen Ovale

9
CONTRAST 1
n A small communication (patent foramen ovale)

r



i
between the RA and LA is present in 20% to 30%
LV of adults.

h
n In some patients, a patent foramen ovale is asso-




ciated with a contour abnormality of the septum

a
RV with bulging from the midline more than 15 mm

t
(atrial septal aneurysm) (Fig. 15-15).

r/
n There is a higher prevalence of patent foramen



ovale in patients with a cryptogenic stroke.
RA LA

e
v KEY POINTS

s
o Shunting at the atrial level is sometimes seen with


color Doppler but often requires a saline contrast

/r u
injection for detection. (Figs. 15-16 and 15-17)
o A patent foramen ovale allows blood flow from


RA to LA when RA pressure exceeds LA pres-
sure. In some patients, shunting occurs at rest;

.t c
Figure 15-15 Atrial septal aneurysm. An atrial septal aneurysm is seen
in others, a right-to-left shunt is seen only after


in this apical 4-chamber view with saline contrast used to opacify the right
heart. The atrial septum deviates from left to right in the region of the fossa Valsalva maneuver to transiently increase RA
ovalis with a radius of more than 15 mm at the maximum curvature point. pressure.

k a
/: /
s
tt p
RA RV
RA
LV

h LA

LA

A B
Figure 15-16 TTE of patent foramen ovale. A, In a subcostal 4-chamber view the atrial septum bulges towards the RA. B, Color Doppler demonstrates a small jet


of flow (arrow) from left to right across the interatrial septum consistent with a patent foramen ovale.
322 CHAPTER 15  Cardiac Masses and Potential Cardiac Source of Embolus

o Appearance of echo contrast in the LA within o At least two saline contrast injections are




three beats of right heart opacification is consis- needed, one at rest and one with Valsalva
tent with a patent foramen ovale. Later appear- maneuver. However, accuracy is optimized with
ance of contrast (after three to five cycles) may at least four contrast injections, two at rest and
be due to transpulmonary passage. two with Valsalva.
o Longer digital clip lengths, which include entry of o TEE is more sensitive than transthoracic imag-




contrast into the right heart and at least five beats ing for detection of a patent foramen ovale (Fig.
after RA opacification, are needed for evaluation 15-18).

/
of a saline contrast study. Review of videotaped o In patients with chronically elevated RA pressures



images may be needed to include an adequate (such as severe pulmonary hypertension with

9
number of cardiac cycles with each injection. right heart failure), persistent right-to-left shunt-
ing may result in arterial oxygen desaturation.

9
Echocardiography (TEE or TTE) can be used

r
o



i
SALINE to guide percutaneous closure of a patent fora-
CONTRAST men ovale (Fig. 15-19).

h
Step 9: Evaluation for Cardiac Source
LV of Embolus

ta
RV n Echocardiography requested to evaluate for a car-

r/



diac source of embolus should include a saline con-
trast study for detection of patent foramen ovale.
n A careful examination for cardiac thrombi,

e



tumors, valvular vegetations, and aortic atheroma,

s
RA LA
often with TEE, is needed when a cardiac source
of embolus is suspected (Table 15-1).

/r u
v KEY POINTS

o If atrial fibrillation is present, an LA thrombus


is a likely cause of clinical events, even if not

.t c
detected on TEE.
o Embolic events in patients with mechani-


cal prosthetic valves must be presumed to be
Figure 15-17 Saline contrast study. Saline contrast study in an apical

a
related to the prosthetic valve, regardless of


4-chamber view in a patient with a systemic embolic event shows a small
echocardiographic findings.

k
amount of contrast (arrow) in the left heart within three beats of contrast
appearance in the right heart consistent with a patent foramen ovale.

/: /
s
tt p LA

h
RA

A B
Figure 15-18 TTE of patent foramen ovale. TEE in a patient with a cryptogenic stroke shows the typical “flap valve” appearance of a patent foramen ovale


with color Doppler demonstrating a narrow red flow signal (arrow) in the slit-like orifice.
Cardiac Masses and Potential Cardiac Source of Embolus  CHAPTER 15 323

RA RA

/
LA

LA

9 9
i r
a h
A B

r/ t
RA

s e
/r u
RA

.t c
a
LA

k
LA

/: /
s
C

tt p D

h
Figure 15-19 Intracardiac echocardiographic guidance of percutaneous closure of a patent foramen ovale. The transducer tip (top of the sector) is


in the RA, with the septum in the midfield and LA in the far field of the images. A, The guiding catheter has been passed through the patent foramen ovale.
B, A sizing balloon is inflated to measure the defect size. C, The closure device is in position but still attached to the catheter seen in the RA. D, The guiding
catheter has been removed and contrast injected into the right heart. The parallel linear echoes of the closure device are seen positioned on the atrial septum
with no evidence for residual shunting.

o Aortic atheroma, detected on TEE, are associated o Cerebrovascular emboli events in patients




with an increased prevalence of embolic events. younger than 45 years old
o TEE to evaluate for a cardiac source of embolus o Cerebrovascular events without other evident




is recommended in patients with: causes in patients of any age
o Abrupt occlusion of a major peripheral or vis- o Whenever clinical management would be altered




ceral artery based on the echocardiographic findings
324 CHAPTER 15  Cardiac Masses and Potential Cardiac Source of Embolus

TABLE 15-1 ESE Recommendations for Echocardiography in Diagnosis and Management

 
of Cardiac Sources of Embolism
Clinical Condition TTE TEE Comments
Acute myocardial Evaluate LV and RV function Not useful for detection of Contrast may improve
infarction and detect LV thrombus LV thrombus detection of LV thrombus
­
on TTE
Cardiomyopathy Evaluate LV and RV Contrast may improve

/
dysfunction, detect LV detection of LV thrombus
thrombus on TTE

9
Atrial fibrillation Detect underlying structural Required to exclude atrial

9
heart disease thrombus in guiding
To indicate, guide, and cardioversion, preablation,

i r
follow-up invasive surgical recurrent embolism, and
procedures to determine risk of future

h
embolism
Detection of PFO May be sufficient to detect Highest sensitivity for detec- Factors that suggest an

a
PFO with good image tion and evaluation of PFO association between stroke

t
quality, saline contrast with and PFO include (1) a

r/
Valsalva maneuver temporal relationship with
a venous thrombosis, (2)
younger age (<55 years)

e
and absence of other
causes, (3) associated

s
atrial septal aneurysm,
and (4) large spontaneous

/r u
or provocable right-to-left
shunt
Aortic atherosclerosis Suprasternal TTE may help TEE may be indicated when

.t c
identify arch atheromas TTE images are suboptimal
or when plaque character-
ization is needed
Cardiac masses Recommended for patients TEE is appropriate when

a
with clinical syndromes TTE is nondiagnostic

k
suggesting a cardiac mass

/: /
or patients with conditions
known to predispose to
mass formation
Recommended for follow-up

s
after mass removal if
recurrence is likely

tt p
Endocarditis Recommended as first Recommended when TTE Repeat TTE or TEE
step in evaluation of is negative and clinical recommended in 7-10
­
endocarditis likelihood is high, with days if initial study is nega-
­
h
prosthetic valves or when tive but clinical likelihood
TTE provides inadequate remains high
imaging
Prosthetic valves TTE must be performed in TEE also must be performed Repeat TTE or TEE is
patients with a prosthetic in patients with a prosthetic recommended for follow-up
valve and embolic event. valve and embolic event, after thrombolytic or
even if TTE is negative anticoagulant therapy
Intracardiac devices TTE is recommended in TEE is also used for diagnosis Intracardiac devices include
patients with a device and a of device thrombosis or permanent pacemakers
pulmonary emboli event or infection and implantable
when paradoxical embolus cardioverters defibrillators
is suspected

Data from Pepi M, Evangelista A, et al: Recommendations for echocardiography use in the diagnosis and management of cardiac sources of embo-
 
lism: European Association of Echocardiography (EAE) (a registered branch of the ESC). Eur J Echocardiogr 2010;11:461-476.
PFO, Patent forame ovale.
Cardiac Masses and Potential Cardiac Source of Embolus  CHAPTER 15 325

THE ECHO EXAM

Cardiac Masses and Source of Embolus


Echocardiographic Findings Associated With Systemic Embolism (in Relative Order
of Clinical Indications for Echocardiography)
Potential Embolic Source Clinical Setting Echocardiographic Findings Caveats

/
Patent foramen ovale Cryptogenic stroke Saline contrast shows right to Cause-effect relationship
left shunt at the atrial level between PFO and stroke

9
Best visualized in TEE remains controversial
PFO present in 20-30% of

9
people

i r
LA thrombus Atrial fibrillation—before LA mass, most often located TEE required for diagnosis
­
cardioversion, AF in LA appendage, may be of LA thrombus due to low

h
ablation, or mitral mobile sensitivity of TTE
­
valvotomy
­
a
Endocarditis Bacteremia, clinical Valve vegetations with valve TEE often needed in addition

t
criteria for endocarditis destruction to TTE

r/
­
Prosthetic valve Mechanical or Mobile mass attached to A prosthetic valve is always
thrombosis bioprosthetic valve leaflets or sewing ring a potential embolic source,
­
­
­
Valve obstruction or even when echo findings

e
regurgitation are absent
­
s
LV thrombus Apical akinesis post Echo dense mass in LV apex Best seen on TTE apical
infarction or global views with high frequency

/r u
­
hypokinesis with transducer
dilated cardiomyopathy May be missed on TEE
Aortic atherosclerosis Evaulation for stroke Typical atheroma Even with TEE, asending

.t c
or intraopertive aortic and arch atheroma
evaluation of aorta for may be missed
­
graft placement Intraoperative direct
placement of a sterile

a
­
probe on the aorta may be
helpful

/: / k
Nonbacterial thrombotic Systemic inflammatory Valve masses with less Blood cultures are needed to
endocarditis disease independent motion than exclude infective endocar-
­
typical vegetations ditis
Lipomatous hypertrophy Benign incidental finding Bright smooth thickening of Echo appearance is typical

s
of the atrial septum the interatrial septum with but CT allows tissue char-
sparing of the fossa ovalis acterization if diagnosis is

tt p
unclear
Papillary fibroelastoma Cryptogenic stroke or Highly mobile small mass, Blood cultures are needed to
incidental echo finding usually attached to valve, exclude infective endocar-

h
­
­
often with a stalk ditis
Atrial myxoma TIA or stroke Well circumscribed mass Best seen on TEE but initial
attached to atrial septum, diagnosis often with TTE
­
most often in LA imaging
Secondary cardiac tumors Direct extension of lung Pericardial effusion and tumor Further evaluation for a
or breast cancer into involvement is most common specific diagnosis is
heart or metastatic needed
disease
Malignant primary cardiac Rare in adults Intracardiac mass with Imaging with CMR or CT
tumors invasion of chamber walls provides better definition of
­
the site and extent of tumor
involvement

AF, Atrial fibrillation; CMR, cardiac magnetic resonance imaging; PFO, patent foramen ovale; TIA, transient ischemic attack.
326 CHAPTER 15  Cardiac Masses and Potential Cardiac Source of Embolus

Distinguishing Characteristics of Intracardiac Masses


Characteristic Thrombus Tumor Vegetation
Location LA (especially when enlarged LA (myxoma) Usually valvular
or associated with MV Myocardium Occasionally on ventricular wall

­
disease) Pericardium or Chiari network
LV (in setting of reduced systolic Valves

­
function or segmental wall

/
abnormalities)
­
Appearance Usually discrete and somewhat Various: may be Irregular shape, attached to the

9
spherical in shape or laminated circumscribed or may proximal (upstream) side of the
against LV apex or LA wall be irregular valve with motion independent

9
from the valve

i r
Associated findings Underlying etiology usually Intracardiac obstruction Valvular regurgitation usually
evident depending on site of present

h
LV systolic dysfunction or seg- tumor
mental wall motion abnormalities Clinically: fevers, systemic

a
(exception: eosinophilic heart signs of endocarditis,

t
disease) positive blood cultures

r/
MV disease with LA enlargement

MV, Mitral valve.

s e
/r u
.t c
k a
/: /
s
tt p
h
Cardiac Masses and Potential Cardiac Source of Embolus  CHAPTER 15 327

SELF-ASSESSMENT QUESTIONS
Question 1 Question 3
A 72-year-old patient presents with an acute isch- A 42-year-old female patient has recently moved to
emic stroke and an embolic event is suspected clini- the area and is transferring care to you. She had an
cally. Which echocardiographic study is most likely echocardiogram for a clinical evaluation for palpita-
to be diagnostic to exclude a left ventricular apical tions and the following abnormality (arrow) (Fig. 15-21)

/
thrombus? was incidentally found, taken from a parasternal view.
A. TTE Otherwise, she has no significant medical history and

9



B. Agitated saline contrast study has no complaints on her visit today.



C. TEE Based on this clinical presentation, the abnormal-

9



D. 3D echocardiogram ity is most consistent with:

r



A. Pulmonic valve stenosis

i
Question 2




B. Papillary fibroelastoma




h
What is the most likely diagnosis for the mass (arrow) C. Bacterial endocarditis




seen in this echocardiographic image (Fig. 15-20)? D. Atrial myxoma




a
A. Central venous catheter

t



B. Pacer lead

r/



C. Moderator band



D. Apical thrombus



s e
/r u
.t c
k a
/: /
s
Figure 15-21

tt p
h
Figure 15-20

328 CHAPTER 15  Cardiac Masses and Potential Cardiac Source of Embolus

Question 4 Question 6
TEE was requested before elective cardioversion in A 36-year-old woman presents for clinical evalua-
a patient with symptomatic atrial fibrillation. The tion. She describes a history of cough, intermittent
following finding (arrow) was identified on the trans- fevers and joint pain lasting for several months. A
esophageal study (Fig. 15-22). TTE is ordered and the following image is recorded
Based on this image, the most appropriate next (Fig. 15-24):
step is: Based on the echocardiographic appearance of
A. Cancel cardioversion procedure the finding, which one of the following recommenda-

/



B. Proceed with cardioversion tions would you make?



C. Delay cardioversion and repeat TEE after 4 A. Parenteral antibiotic therapy

9






weeks of anticoagulation therapy B. Cardiac magnetic resonance imaging




C. Intravenous anticoagulation therapy

9



D. Cardiac surgery referral


r


h i
ta
e r/
s
/r u
.t c
Figure 15-22

Figure 15-24
Question 5

Question 7

a
Doppler data were recorded during an elective TEE
in an outpatient (Fig. 15-23). This M-mode tracing was obtained in a patient referred

k
This Doppler recording is most consistent with: for evaluation of an embolic stroke (Fig. 15-25).

/: /
A. Normal sinus rhythm This finding is most consistent with:



B. Atrial fibrillation A. Normal






C. Atrial flutter B. Mitral stenosis

s






D. Ventricular tachycardia C. Aortic stenosis






E. Ventricular fibrillation D. Atrial septal aneurysm

tt p






E. Atrial myxoma



45dB 3 •/1/1/1 Cal  10mm
0
PW Depth  68mm

h
PW Gate  5.0mm
PW Gain  12dB

.50

m/s

.50

140

Figure 15-23 Figure 15-25




Cardiac Masses and Potential Cardiac Source of Embolus  CHAPTER 15 329

Question 8 This finding is most consistent with:


A 39-year-old woman is referred for TEE for two epi- A. Eustachian valve




sodes of transient aphasia. Color Doppler imaging B. Atrial myxoma




shows the following flow pattern (Fig. 15-26). C. Crista terminalis




D. Thebesian valve




9/
i r 9
a h
r/ t
s e
/r u
Figure 15-26

.t c
Question 9
A 58-year-old male patient presents with progressive dys-

a
pnea. The image obtained (Fig. 15-27) suggests which of
the following:

k
A. Atrial myxoma

/: /



B. Bacterial endocarditis



C. Intracardiac thrombus



D. Papillary fibroelastoma

s



tt p
h
Figure 15-27

330 CHAPTER 15  Cardiac Masses and Potential Cardiac Source of Embolus

Question 10 Question 11
TEE is requested to evaluate for a potential cardiac A 41-year-old woman presents with cardiopulmonary
source of embolus in a 73-year-old woman with a symptoms and a TTE is ordered. The following image
transient ischemic event (Fig. 15-28). is obtained (Fig. 15-29).
This finding is diagnostic of: The most likely origin of this mass is:
A. Atrial septal aneurysm A. Intra-cardiac
B. Patent foramen ovale B. Pericardial
C. Atrial myxoma C. Abdominal
D. Lipomatous hypertrophy D. Intra-cranial
E. Atrial septal occluder device

LA

RA

Figure 15-29 

Figure 15-28 
Cardiac Masses and Potential Cardiac Source of Embolus  CHAPTER 15 331

ANSWERS

Answer 1: A collagenous branching, frond-like protrusions which


are avascular. If large enough, they may be associ-
The left ventricular apex is best seen from apical
ated with embolic events or symptoms consistent
windows on transthoracic imaging. Care must be
with valve outflow obstruction. Smaller, asymp-
taken by the sonographer not to foreshorten the
tomatic lesions may be followed with serial echo-
ventricle and exclude the apex. Even if not clearly
cardiographic imaging. This asymptomatic patient
visualized, suspicion for a thrombus is heightened
presented with serial imaging over several years
if there is apical hypokinesis with evidence of
blood stasis. Even with optimal imaging, the apex which showed stable appearance of the lesion. The
may be difficult to clearly visualize, due to artifacts appearance (attached to the pulmonic valve) is
such as near field clutter (ring-down artifact). In not consistent with an atrial myxoma. Her asymp-
these cases, microbubble transpulmonary contrast tomatic clinical presentation is not consistent with
can be used to aid endocardial LV opacification; acute bacterial endocarditis. Pulmonic valve steno-
apical thrombi appear as negative contrast zones sis is a congenital abnormality, typically associated
where contrast is unable to penetrate. Agitated with narrowing of the right ventricular outflow
saline contrast (answer “B”) will not aid in exclud- tract and dysplasia of the valve leaflets. Isolated
ing left sided thrombi. TEE imaging may produce globular thickening is not characteristic of pul-
improved image quality compared with transtho- monic stenosis.
racic views with absence of intervening tissue, but Answer 4: B
because of physical constraint of the probe in the
esophagus, the ventricle is often foreshortened and This TEE image taken at 100° view shows an enlarged
the apex not visualized. The imaging frame rate left atrium. The left atrial appendage is well seen and
for 3D echo imaging is not adequate to definitively there is prominent trabeculation present (arrow).
exclude a thrombus. Of the options provided, TTE Given the absence of thrombus, the elective cardio-
imaging is most likely to be diagnostic for LV apical version should proceed in this symptomatic patient.
thrombus. Trabeculations are differentiated from thrombus
typically by higher echogenicity (brighter), continuity
Answer 2: B with the atrial wall, and contractile motion. Although,
This bright dense “mass” seen in the right ventricle in patients with atrial fibrillation, contractile motion
is a pacer lead. This diagnosis can be confirmed by of the trabeculae will be decreased compared with
reviewing the clinical history (or asking the patient) patients in sinus rhythm. Multiple views of the left
and by imaging in other views to demonstrate the atrial appendage are needed to show continuity of tra-
length of the pacer lead as it traverses the RA and beculations with atrial myocardium. In patients with
RV. A central venous catheter should be in the supe- a prominent ridge between the left atrial appendage
rior vena cava or right atrium; in the rare event that a and left upper pulmonary vein, acoustic shadowing
central catheter extends into the RV, the thickness of of the appendage may hinder definitive evaluation for
the catheter is less than the metallic pacer leads. The thrombus. Characteristic appearance of thrombus is
moderator band is a normal muscle band closer to one of blood stasis, with increased density at the tip of
the RV apex, extending from the free wall to septum, the appendage. When a thrombus is present, several
with an echodensity similar to the rest of the myocar- weeks or months of anticoagulation are appropriate
dium. Ventricular thrombi are unusual in the RV but, before re-evaluation for resolution of thrombus.
when present, have an echodensity similar to myocar- Answer 5: C
dium and occur in regions of myocardial dysfunction.
Peripheral venous thrombi may embolize to the heart This Doppler signal was recorded in the left atrial
and lodge in the RV or tricuspid valve apparatus, with appendage, seen in the 2D image in the 90° view,
a tubular shape reflecting formation in a peripheral with the Doppler sample volume about 1 cm into
vein. the appendage. Regular flow in and out of the
atrial appendage at a rate about 300 bpm is seen
Answer 3: B with a velocity about 0.5 m/s, in both directions.
This image was taken at the base of the heart with This is consistent with atrial flutter with typical
the pulmonic valve in long axis. On the right ven- atrial (and appendage) contractions at a rate of 300
tricular outflow side of the pulmonic valve, there bpm. Normal sinus rhythm results in flow out of the
is an echogenic mobile mass. A papillary fibro- atrial appendage following the P-wave on the elec-
elastoma is a benign, primary tumor of the heart trocardiogram with a single velocity peak of at least
valves. Histologically, papillary fibroelastomas have 0.4 ms/s with each cardiac cycle. Atrial fibrillation
332 CHAPTER 15  Cardiac Masses and Potential Cardiac Source of Embolus

results in rapid, irregular, low velocity flow waves


from the atrial appendage. Ventricular tachycardia
and ventricular fibrillation affect the contraction of
the left ventricle, not the left atrium, and usually
are associated with hemodynamic compromise.
RVOT
Answer 6: D
This image was taken from a 4-chamber subcos- Ao
tal view. There is a large atrial myxoma in the left
atrium, with attachment along the inter-atrial sep- LV
tum. The mass abuts the mitral valve and nearly
obliterates the left atrium. Most myxomas (70% to
80%) occur in the left atrium, with an attachment LA
point at the interatrial septum. Generally, myxo-
mas are the most common primary cardiac tumor.
They are benign, derived from multipotential mes-
enchymal cells and are more common in women.
Clinical presentation may be asymptomatic or
there may be an array of symptoms, such as dizzi-
ness, cough fever, cachexia and arthralgias. Large
myxomas may occlude left ventricular inflow and
present with symptoms analogous to mitral valve Figure 15-30 
obstruction such as syncope, dyspnea, or palpita-
tions. Additional cardiac imaging is not needed; Answer 8: A
echocardiography was diagnostic and a referral to The eustachian valve overlies the inferior vena
cardiac surgery for resection is indicated. Cardiac cava, lies at the junction of the inferior vena cava
thrombi occur in areas of blood stasis such as the and right atrium, and is variable in size, length, and
left atrial appendage in atrial fibrillation or the LV prominence in individuals. In the image provided, it
apex with aneurysm. Isolated thrombi in the main protrudes into the right atrium and displaces color
chamber of the left atrium is unlikely and intra- Doppler flow (absence of color in the left side of the
venous anticoagulation is not indicated. The mass image). The function of the eustachian valve was to
is more consistent with myxoma than a vegetation direct intrauterine flow of oxygenated blood from
in its size and septal attachment point. While anti- the inferior vena cava to the fossa ovalis. The crista
biotic therapy should be considered, the correct terminalis is embryologic line of union between the
intervention is surgical resection. trabeculated appendage and the right atrium. When
echo imaging planes go through the midportion of the
Answer 7: E crista terminalis, a prominent junction line appears
This is an M-mode tracing of the mitral valve, as a circular echodensity along the right atrial wall.
with the structures seen, from anterior to posterior An atrial myxoma is a benign cardiac tumor, most
including the RV, septum, mitral valve, and posterior commonly in the left atrium with an attachment point
LV wall. An aortic valve M-mode would show the on or near the inter-atrial septum. The thebesian valve
parallel walls of the aorta anterior to the left atrium. is a membranous structure which originates at the
An atrial septal aneurysm is not usually seen well by superior vena cava, at the orifice of the coronary
M-mode given the orientation of the atrial septum sinus. It is highly variable in size among individuals
relative to the chest wall. The diastolic slope of the and is not commonly seen.
anterior leaflet is flat, as is seen with mitral stenosis,
but leaflet excursion and E-point septal separation Answer 9: C
are normal. In addition, there are multiple parallel The TEE image provided was taken from an upper
echoes moving with the mitral valve, seen filling the esophageal view of the base of the heart. In the mid-
space between the anterior and posterior leaflets in portion of the image, the aortic sinuses are seen cut
diastole. This finding is consistent with a mass, most in cross section. The take-off of the left main coro-
likely a left atrial myxoma (arrow) prolapsing in the nary artery is seen originating at the top of the aorta,
mitral orifice in diastole, as confirmed on 2D imag- and the take-off of the right coronary artery is seen
ing (Fig. 15-30). originating at the bottom of the aorta. Just above the
Cardiac Masses and Potential Cardiac Source of Embolus  CHAPTER 15 333

aorta, the inter-atrial septum is seen and there is a and right ventricle. Although the most common pri-
large thrombus in transit crossing a patent foramen mary cardiac tumors are myxomas, typically originat-
ovale. The thrombus diameter is ∼1 cm, consistent ing in the left atrium, secondary cardiac tumors are
with a proximal deep vein thrombosis vessel diam- much more common. Identification of an intracar-
eter. In this patient, there was heavy thrombus bur- diac mass should prompt clinical evaluation for an
den throughout the right heart and pulmonary artery, extracardiac malignancy source. Secondary tumors
consistent with prior pulmonary embolism. He had may originate from a variety of places, but are most
recently been in a motor vehicle accident and was in commonly related to the lungs, kidneys, melanoma, or
recovery when he developed progressive dyspnea. hematologic disorders such as lymphoma or leukemia.
A primary pericardial source is rare. Of the options
Answer 10: D listed, the most likely source is abdominal. Additional
In this TEE bicaval view of the LA and RA, the atrial imaging in this patient confirmed a uterine mass. Echo-
septum appears intact with a normal thin fossa ova- cardiographic imaging from the subcostal view showed
lis (between arrows) but with marked thickening and tumor invading the heart from the inferior vena cava
increased echodensity of the rest of the septum, diag- (dilated to 3.2 cm), marked with asterisks in the image
nostic for lipomatous hypertrophy of the interatrial below, and filled with tumor (Fig. 15-31).
septum. This benign normal variant is commonly
seen, with prevalence increasing with age and body
mass index. Although the fossa ovalis appears rela-
tively thin, it is normal thickness and is not deviated
towards either side. The definition of an atrial septum
aneurysm is deviation by 1.5 cm or more, which the
image does not demonstrate. A patent foramen ovale
may or may not be present but diagnosis requires
color Doppler and a saline contrast injection. If a
secundum or primum atrial septal defect were pres-
ent, the right heart chambers would be enlarged and
there would be a discontinuity in the atrial septum.
With a sinus venosus atrial septal defect, the septum
might appear intact in this view, but the right heart
would still be enlarged. An atrial septal occluder
device results in prominent echo densities on both
sides of the fossa ovalis, in a shape consistent with the
specific device implanted.
Answer 11: C
The image from the apical 4-chamber view shows a
heterogenous, large mass occupying the right atrium Figure 15-31 
16 Diseases of the Great Arteries
BASIC PRINCIPLES Evaluate Aortic Valve Anatomy and Function
STEP-BY-STEP APPROACH Examine the Descending Thoracic Aorta
Transthoracic Echocardiography SPECIAL CONSIDERATIONS
Record Blood Pressure and Ensure the Patient Is Chronic Aortic Dilation
Medically Stable Aortic Dissection
Assess the Aortic Sinuses and Ascending Aorta Use the Basic Approach for Evaluation of the Aorta
from the Parasternal Window to Identify the Dissection Flap
Assess the Descending Thoracic Aorta from the Look for Complications of Aortic Dissection
Parasternal and Apical Windows Sinus of Valsalva Aneurysm
Assess the Proximal Abdominal Aorta from the Aortic Pseudoaneurysm
Subcostal Window Atherosclerotic Aortic Disease
Assess the Aortic Arch and Proximal Descending Persistent Left Superior Vena Cava
Thoracic Aorta from the Suprasternal Notch Pulmonary Artery Abnormalites
Window Clinical Concerns
Decide if TEE or Other Imaging Procedures Are Basic Echocardiographic Approach
Needed THE ECHO EXAM
Transesophageal Echocardiography SELF-ASSESSMENT QUESTIONS
Image the Aortic Sinuses and Ascending Aorta
from a High Esophageal Position

BASIC PRINCIPLES STEP-BY-STEP APPROACH


n  systematic approach is needed for echocardio-
A
graphic evaluation of the great arteries. Transthoracic Echocardiography
n Transesophageal echocardiography (TEE) is more n  Examination of the aorta is based on visualization
sensitive than transthoracic imaging for detection of several segments from different acoustic windows.
of aortic aneurysm and dissection. n The sequence suggested here follows the sequence
n Wider field of view tomographic imaging tech- of a standard transthoracic study; other exam
niques, including chest computed tomography sequences may be appropriate with an acute clini-
(CT) or cardiac magnetic resonance (CMR) cal presentation.
imaging, provide optimal evaluation of the great
vessels. Step 1: Record Blood Pressure and Ensure
the Patient Is Medically Stable
v  KEY POINTS n  Aortic disease often presents as a medical/surgi-
o Many segments of the aorta and pulmonary cal emergency; appropriately trained health care
artery can be visualized on transthoracic imag- providers should be available during the study.
ing, but: n Blood pressure is recorded at the beginning of the
o 
Evaluation of branch pulmonary arteries and study because findings may change with altered
the branching of systemic arteries from the aorta loading conditions.
often is not possible
o 
Ultrasound imaging artifacts must be dis- v  KEY POINTS
tinguished from an intraluminal dissection When time is of the essence, limited imaging
o
flap and Doppler data should be focused on the spe-
The colloquial term aortic root includes the aortic
o cific clinical question.
annulus, sinuses of Valsalva, sinotubular junc- o It may be appropriate to proceed directly to
tion, and ascending aorta. TEE when aortic dissection is suspected; the
o When the echocardiogram is nondiagnostic echocardiographer should consult with the
or equivocal, additional imaging techniques referring provider to ensure that the most
should be recommended, based on the clinical appropriate test is performed in a timely
signs and symptoms. manner.

334
Diseases of the Great Arteries  CHAPTER 16 335

Ao

LV

LA

Figure 16-1  Parasternal imaging of the aorta.  Standard parasternal long-axis view showing the proximal ascending aorta (left). The transducer is moved
up an interspace to visualize additional segments of the ascending aorta (center), and then the image is zoomed to improve resolution of the aortic sinuses,
sinotubular junction, and ascending aorta for accurate measurements (right). Ao, Aorta.

ST junction

Sinus D

Septum C

LVOT

AMVL

Figure 16-2  Aortic diameter measurements. In addition to standard


measurement at the annulus and sinuses of Valsalva, the sinotubular junc-
tion and midascending aorta are measured when the aorta is dilated as
shown in this schematic illustration. AMVL, Anterior mitral valve leaflet;
LVOT, left ventricular outflow tract; ST, sinotubular. Figure 16-3  Example of aortic measurements.  Diameter is measured at
the sinuses (1), sinotubular junction (2), and midascending aorta (3) in a patient
with mild sinus dilation. By convention, measurements are made at end-diastole
Step 2: Assess the Aortic Sinuses and from the inner white-black border of the anterior wall to the black-white border
Ascending Aorta from the Parasternal Window of the posterior aortic wall. Care is needed to ensure that the image plane is cen-
tered in the aorta and that measurements are perpendicular to the long axis of
n The aortic sinuses are seen in the standard and high the vessel. An off-center image plane will underestimate aortic size; conversely,
parasternal long-axis view (Fig. 16-1 and Fig. 16-2). oblique measurements will overestimate aortic size.
n Diameter measurements are reported at end-
diastole for the aortic annulus, sinuses of Valsalva, The sinotubular junction is defined as the top
o
and sinotubular junction and in the midascending of the sinuses of Valsalva and is recognized by
aorta (Fig. 16-3). the acute angle at the transition from the curved
n 
Color Doppler allows detection of aortic regur- sinuses to the tubular ascending aorta.
gitation and evaluation of the flow pattern in the o Aortic dimensions are measured on two-­
ascending aorta. dimensional (2D) images from the inner white-
black border to the inner black-white border
v  KEY POINTS of the aortic lumen.
o 
After recording the standard parasternal o Comparison of measurements on serial studies
long-axis view of the aortic sinuses, the trans- or by different modalities should be made at the
ducer is moved up one or more interspaces to same time point in the cardiac cycle.
visualize as much of the ascending aorta as o In comparing measurements from different
possible. modalities (CT and MRI), both imaging physics
336 CHAPTER 16  Diseases of the Great Arteries

Ao
LV
LA

LA
DA

DA

A B
Figure 16-4  Parasternal views of descending aorta.  In a Marfan patient, the descending aorta is seen posterior to the LA. A long-axis view of the descend-
ing thoracic aorta can be obtained by rotating the transducer clockwise into a parasternal short-axis plane. Ao, Aorta; DA, descending aorta.

differ and measurement norms may vary, lead-


ing to apparent discrepancies.
Step 3: Assess the Descending Thoracic Aorta
from the Parasternal and Apical Windows
n  The midportion of the descending thoracic aorta
can be visualized from the parasternal long-axis
view, posterior to the left atrium (LA), by rotating
the image plane clockwise to obtain a longitudinal
view of the aorta (Fig. 16-4).
n The descending thoracic aorta also can be imaged
from the apical 2-chamber view by lateral angula-
tion of the image plane (Fig. 16-5).
n 
Color Doppler is helpful in distinguishing image
artifacts from an intraluminal flap in these views.
v  KEY POINTS DA
Dilation of the descending aorta and dissec-
o
tion flaps can be identified in these views when
image quality is adequate.
o Only some segments of the descending thoracic
aorta are visualized so that significant pathol-
ogy may be missed. Figure 16-5  Apical views of descending aorta.  Another segment of the
descending thoracic aorta is imaged in the same patient as Fig. 16-4 from
o The aorta is in the far field of the image in these the apical 2-chamber view with lateral angulation of the image plane. DA,
views, limiting evaluation for aortic atheroma. Descending aorta.
o In patients with a large left pleural effusion,
the descending aorta can be imaged from the
posterior chest wall, using the pleural effusion n  olodiastolic flow reversal is seen in the proximal
H
as an acoustic window. abdominal aorta in patients with severe aortic
valve regurgitation (see Fig. 12-5).
Step 4: Assess the Proximal Abdominal
Aorta from the Subcostal Window v  KEY POINTS
n The proximal abdominal aorta is seen in the sub- o Diameter of the proximal abdominal aorta is
costal view by medial angulation from the inferior routinely measured in patients with aortic dis-
vena cava image plane (Fig. 16-6). ease, such as Marfan syndrome.
Diseases of the Great Arteries  CHAPTER 16 337

.69 40dB 2 •/+1/0/ 1


PW Depth=115mm
PW Gate= 3.0mm
PW Gain= 12dB

.69

2.0

DA

m/s

.50

Figure 16-7  Normal flow in the proximal abdominal aorta. There


is antegrade flow in systole, brief early diastolic reversal (arrow), low-
velocity forward flow in mid-diastole, and then a slight flow reversal at
end-diastole.
Figure 16-6  Proximal abdominal aorta.  From the subcostal window the
image plane is turned medially from the inferior vena cava to visualize the The descending aorta appears to “taper,” even
o
aorta in a long-axis view. DA, Descending aorta.
when normal, due to the oblique plane of the
ultrasound image compared with the curvature
With a normal pattern of flow, antegrade sys-
o of the aorta.
tolic flow is followed by early diastolic flow o The distance from the aortic valve that holo-
reversal due to coronary blood flow, mid-dia- diastolic flow reversal persists correlates with
stolic low-velocity antegrade flow, and a very regurgitant severity. Thus, reversal in the proxi-
brief late diastolic flow signal due to elastic mal descending aorta is seen with moderate
recoil of the aorta (Fig. 16-7). regurgitation, but reversal in the abdominal
o In order to detect the holodiastolic flow reversal aorta indicates severe regurgitation.
seen with severe aortic regurgitation the filters o The right pulmonary artery is seen in cross-
are adjusted to show low-velocity flow. section, under the arch, in the long-axis view of
o Holodiastolic flow reversal also is seen with the aortic arch.
other aortic diastolic flow abnormalities, such o A longitudinal view of the left pulmonary
as a patent ductus arteriosus, surgical systemic- artery can be obtained by leftward rotation and
to-pulmonary shunt (e.g., Blalock-Taussig angulation of the image plane.
shunt) or aorto-pulmonary window, or other
large arteriovenous communication, such as an Step 6: Decide if TEE or Other Imaging
upper extremity dialysis fistula in patients with Procedures Are Needed
end-stage renal disease). n Echocardiographic interpretation should first
describe the imaging and Doppler findings, along
Step 5: Assess the Aortic Arch and Proximal with a differential diagnosis for these findings. The
Descending Thoracic Aorta from the level of confidence in any diagnosis should be
Suprasternal Notch Window indicated.
n  The aortic arch is visualized in long- and short-axis n 
Second, the interpretation should indicate any
views from the suprasternal notch window (Fig. 16-8) areas of uncertainly and suggest additional diag-
n Pulsed Doppler recordings of flow in the proximal nostic procedures in consultation with the referring
descending aorta show holodiastolic flow rever- physician.
sal when moderate to severe aortic regurgitation
is present (Fig. 16-9). The normal flow pattern is v  KEY POINTS
shown in Fig. 16-10. o Complete evaluation of the aorta may require
additional imaging procedures, including:
v  KEY POINTS o TEE
o The aortic arch is measured in its midsection, o Chest CT
where the ultrasound beam is perpendicular to o CMR imaging
the aortic walls. o Cardiac catheterization with aortography
338 CHAPTER 16  Diseases of the Great Arteries

.69

Ao

RPA
.69

1.0

m/s

1.0

Ao
Figure 16-9  Holodiastolic aortic flow reversal.  In a patient with severe
aortic regurgitation, holodiastolic flow is seen in the descending thoracic
aorta. The diastolic flow signal is above the baseline from the end of ejection
PA up to the start of the next ejection period.

LA

B m/s

Figure 16-8  Aortic arch.  A, From the suprasternal notch position, a long-
axis view of the aortic arch shows segments of the ascending and descend-
ing thoracic aorta, the arch, and the origins of the head and neck vessels.
The right pulmonary artery is seen under the curve of the arch. B, In the
short-axis view of the arch, the LA and pulmonary veins are seen inferior
to the right pulmonary artery. Ao, Aorta; PA, pulmonary artery; RPA, right
pulmonary artery. 1.0

Figure 16-10  Normal descending aortic flow.  Normal flow in the descend-
ing thoracic aorta shows early diastolic flow reversal (arrow, corresponding to
diastolic coronary blood flow), low-velocity forward flow in mid-diastole, and
slight reversal at end-diastole. This patient had no aortic regurgitation.
Diseases of the Great Arteries  CHAPTER 16 339

o Selection of the most appropriate diagnostic recorded from the aortic valve level to as high as
modality depends on several clinical factors, possible in the ascending aorta (Fig. 16-12).
including the differential diagnosis, acuity o Color Doppler in long- and short-axis views
of symptoms, concurrent diseases, and local may show flow in two lumens when a dissection
expertise in imaging. is present.
o The mid- and distal ascending aorta may be
Transesophageal Echocardiography better seen by decreasing the rotation to about
100° and slightly withdrawing the probe.
Step 1: Image the Aortic Sinuses and o The distal ascending aorta just proximal to the
Ascending Aorta from a High Esophageal arch is often not well seen due to the interposed
Position air column (trachea) between the esophagus
n  Long-axis views of the ascending aorta provide and aorta.
excellent image quality for detection of aortic dila-
tion or dissection.
n Short-axis images provide confirmation of find-
ings and are helpful in distinguishing artifact from
intraluminal abnormalities. LA
n 
Color Doppler evaluation of the flow pattern in
the aorta helps identify dissection flaps.
v  KEY POINTS
The long-axis view typically is obtained at 120°
o
rotation, but there is individual variability, so
the image plane should be adjusted to show
the ascending aorta in a long-axis orientation
(Fig. 16-11). Ao
o From the long-axis view, the transducer is
moved up in the esophagus to image as much
of the ascending aorta as possible.
o Slow medial and lateral turning of the image
plane from the long-axis view may identify abnor-
malities not seen in the centered long-axis view.
o The short-axis view is used to provide an orthog-
onal image plane. The short-axis view should be A

LA

LA

LV
Ao

B
Figure 16-12  Aortic dissection.  A complex dissection flap in the ascend-
Figure 16-11  TEE long-axis view of the ascending aorta.  This view is ob- ing aorta is seen in TEE long-axis (A) and short-axis (B) views of the ascend-
tained from a high TEE probe position at about 120° to 140° rotation. Ao, Aorta. ing aorta. Ao, Aorta.
340 CHAPTER 16  Diseases of the Great Arteries

o Imaging artifacts commonly seen in the ascend- v  KEY POINTS


ing aorta often include linear reverberations 
oThe risk of aortic dilation and dissection is
from the aorta itself or adjacent structures. Arti- higher in patients with a congenitally bicuspid
facts are distinguished from anatomic abnor- or unicuspid valve, compared with those with a
malities by examination in at least two imaging trileaflet valve (Fig. 16-13).
planes; their location, appearance, and pattern o Aortic disease may result in aortic regurgita-
of motion relative to the aortic wall; and cor- tion either due to dilation of the aortic sinuses
relation with color Doppler flow patterns. and central noncoaptation of the leaflets or due
to extension of a dissection flap into the valve,
Step 2: Evaluate Aortic Valve Anatomy resulting in a flail leaflet.
and Function o Aortic valve anatomy and function is evalu-
n  The presence of aortic valve disease prompts ated with 2D imaging and color Doppler in a
careful evaluation for associated disease of the long-axis view at about 120° rotation and in the
aortic sinuses and ascending aorta (Tables 16-1 short-axis view, at about 30° rotation.
and 16-2). o Additional scanning medially and laterally from
n Conversely, aortic disease may result in secondary the long-axis view and superiorly and inferiorly
aortic valve regurgitation. from the short-axis view also is helpful.
Step 3: Examine the Descending Thoracic
Aorta
TABLE 16-1 Upper Limits of Normal Aortic
Dimensions in Adults n The descending thoracic aorta is well visualized on
TEE by turning the image plane posteriorly.
Indexed for
n The short-axis view is recorded with the trans-
BSA (Men
Men Women and Women)
ducer slowly withdrawn from the transgastric level
to the high esophagus (Fig. 16-14).
Aortic annulus* 3.2 cm 2.7 cm 1.5 cm/m2 n 
The aortic arch is visualized from the high TEE
Sinus of Valsalva* 4.0 cm 3.6 cm 2.1 cm/m2 position by turning the image plane medially with
Sinotubular 3.6 cm 3.2 cm 1.9 cm/m2 inferior angulation.
junction*
v  KEY POINTS
Proximal ascend- 3.8 cm 3.5 cm 2.1 cm/m2
Long-axis views of the descending aorta supple-
o
ing aorta*
ment short-axis views and are helpful for evalu-
Aortic arch† 3.6 cm 3.2 cm ation of any abnormal findings. However, the
Descending 3.3 cm 3.0 cm long-axis view alone may miss abnormalities that
thoracic aorta† are located medial and lateral to the image plane.
Aorta at 3.2 cm 2.9 cm
o Color flow provides visualization of flow in the
diaphragm true and false lumens when dissection is present.
o Color flow also is helpful in distinguishing image
Data from Roman MJ, Devereux RB, Kramer-Fox R, et al, Am J artifacts from dissection flaps and atheroma.
Cardiol 1989; 64: 507-512 and Hannuksela M, Lundqvist S, Carlberg o When the aorta is tortuous, care is needed to
B. Scand Cardiovasc J 2006; 40: 175-178.
BSA, Body surface area.
distinguish intraluminal abnormalities from an
Upper limits are estimated from the mean diameter plus 2 standard oblique image plane.
deviations.
*Based on TTE data.
†Based on CT data.
SPECIAL CONSIDERATIONS
Chronic Aortic Dilation
n 
There are many causes of aortic dilation, includ-
TABLE 16-2 Equations for Calculation ing (Table 16-3):
of Expected Aortic Sinus •
Hypertension
Dimension Based on Body Size • Atherosclerosis
Patient Age Expected Sinus Dimension (cm) • Familial aortic aneurysm
<20 years 1.02 + 0.98 (BSA) • Marfan syndrome and other connective tissue disorders
• Aortic dilation associated with congenital unicuspid
20-39 years 0.97 + 1.12 (BSA)
and bicuspid aortic valves
40 years or older 1.92 + 0.74 (BSA) • Inflammatory diseases of the aorta including tertiary
syphilis, giant cell arteritis, and Takayasu arteritis
From Roman MJ, Devereux RB, Kramer-Fox R, et al, Am J Cardiol
1989; 64: 507-551.
• Systemic inflammatory diseases including ankylosing
spondylitis
Diseases of the Great Arteries  CHAPTER 16 341

Ao
LV

LA

A B
Figure 16-13  Bicuspid aortic valve disease.  Transthoracic imaging of a bicuspid aortic valve in short-axis (A) often is associated with dilation of the aortic
sinuses or ascending aorta as seen in the corresponding long-axis view (B). Ao, Aorta.

TL

FL

A B
Figure 16-14  TEE imaging of the descending thoracic aorta.  A, The short-axis view of the middescending thoracic aorta shows a dissection flap separat-
ing the true lumen from the false lumen. B, Color Doppler shows flow in the true lumen with a jet of flow through a fenestration in the flap into the false lumen.
FL, False lumen; TL, true lumen.

n chocardiography provides accurate measure-


E Early in the disease, this finding may be sub-
ment of the aortic sinuses and ascending aorta and tle; late in the disease the aortic sinuses appear
often provides anatomic clues about the cause of globular with no discernible sinotubular junc-
disease. tion (Fig. 16-15).
o Aortic dilation associated with congenital
v  KEY POINTS valve abnormalities is independent of valve
Hypertensive aortic dilation usually is mild
o hemodynamics.
and accompanied by left ventricular hypertro- o Aortic dilation due to an inflammatory process
phy, aortic valve sclerosis, and mitral annular usually is characterized by increased thickness
calcification. of the aortic walls.
o Marfan syndrome is characterized by loss of the
normal acute angle at the sinotubular junction.
342 CHAPTER 16  Diseases of the Great Arteries

TABLE 16-3 Aortic Disease: Clinical Echocardiographic Correlates


Clinical Correlation Echocardiographic Findings Imaging Recommendations
Hypertensive Chronic hypertension is Dilation of the ascending TTE imaging from a high
heart disease associated with mild aortic aorta with normal sinuses interspace allows measurement
dilation and STJ is typical of the ascending aorta
Aortic Atherosclerosis is associated Focal irregular thickening of TEE is needed for evaluation of
atherosclerosis with mild aortic dilation the aortic wall with areas of aortic atheroma
Aortic atheroma may cause calcification.
systemic emboli, esp. with Associated thrombus is seen
large protruding atheroma as a mobile echo-density
with mobile thrombus
Bicuspid aortic Associated aortopathy in Enlargement of the aortic CT or CMR is recommended for
valve disease some patients with an sinuses and/or the evaluation of the ascending
increased risk of progressive ascending aorta, usually aorta if not full imaged by
dilation and dissection with a preserved STJ echocardiography
Marfan (and Aortic dilation, ectopia lentis Dilated sinuses with Aortic imaging (TTE or CMR)
Loeys-Dietz) skeletal features, family enlargement (or effacement) recommended every 6-12 months
syndrome history of the STJ CMR recommended annually in
Long anterior mitral leaflet Loeys-Dietz patients, from
with prolapse cerebrovascular ­circulation to pelvis
TTE recommended in first degree
relatives of patients with thoracic
aneurysm/dissection or known
genetic mutation
Systemic Arthritis with systemic Dilated thick walled aorta TTE appropriate when aortic
inflammatory inflammation with characteristic regurgitant murmur is present
disease Aortic involvement in about thickening extending onto Routine surveillance not
(ankylosing 20% of patients base of the anterior mitral recommended.
spondylitis, etc.) valve leaflet
Syphilitic aortitis Aneurysm of the ascending Dilated aorta with calcification Rare in North America or Europe
aorta is seen 10-25 years May involve proximal
after initial spirochetal coronary arteries
infection

In ankylosing spondylitis, the increase in aor-


o
tic wall thickness extends into the base of the
anterior mitral valve leaflet, with the appear-
ance of a subaortic “bump” in a long-axis
view. Ao
o Takayasu arteritis typically involves the aortic
arch and branches, resulting in areas of stenosis
and dilation, but the descending aorta also may LV
be involved.
LA
Aortic Dissection
Step 1: Use the Basic Approach
for Evaluation of the Aorta to Identify
the Dissection Flap
n 
The characteristics of a dissection flap (Fig. 16-16)
are:
Figure 16-15  Effacement of the sinotubular junction.  In patients with

A thin, linear, mobile intraluminal echo Marfan syndrome, the normal acute angle between the curved sinuses and
• Motion independent of the aortic walls tubular ascending aorta at the sinotubular junction is attenuated (arrows),
• Separation of the lumen into two channels resulting in a “water balloon” appearance of the aorta. Ao, Aorta.
TABLE 16-3 Aortic Disease: Clinical Echocardiographic Correlates—cont’d
Clinical Correlation Echocardiographic Findings Imaging Recommendations
Takayasu or giant Loss of distal artery pulses in Dilation of thoracic and TTE and TEE show aortic dilation,
cell arteritis patient under age 40 years abdominal aorta aortic walls may be thickened
(Takayasu) or over age and irregular
50 years (giant cell)
Elevated systemic
inflammatory markers
Aortic ­dissection Acute onset of chest pain, Intimal flap with flow in true TEE, CMR, or CT recommended
often described as tearing and false lumen in patients at high risk of aortic
May radiate to neck or back Type A dissection involves dissection
ascending aorta Choice of procedures depends on
Type B dissection limited to patient variables and availability
descending thoracic aorta of each imaging modality
Intramural Acute presentation with chest Crescent-shaped thickening Intramural hematoma may be seen
hematoma or back pain of aortic wall on TTE but TEE has a higher
May be localized sensitivity for this diagnosis
Aortic pseudo­ Typically seen after complex Echo free space adjacent to TEE, CT, and/or MRI needed to
aneurysm ­aortic valve surgery or aorta evaluate known or suspected
­endocarditis Flow from aortic lumen in and aortic pseudoaneurysm
out of the pseudoaneurysm
may be seen
Traumatic aortic Deceleration injury can result Disruption of the aorta at the CT recommended for diagnosis
­disease in aortic rupture, most often junction between the arch Delayed diagnosis may result in
at the ligamentum arteriosus and descending thoracic pseudoaneurysm at the aortic
(45%) or the ascending aorta aorta may be difficult to isthmus
(23%) appreciate on TTE or TEE
Sinus of Valsalva Isolated congenital sinus of Rupture may occur into the Both color and spectral Doppler
aneurysm Valsalva aneurysms may RV outflow tract from the are helpful in determining
be a smooth dilation of left coronary cusp, into the which chamber is affected by
one sinus or may have a RA from the right coronary a ruptured sinus of Valsalva
“windsock” a­ ppearance cusp or into the LA from the aneurysm
Aneurysms due to endocarditis noncoronary cusp
are associated with
thickening of the aortic wall
and abscess formation
CMR, Cardiac magnetic resonance imaging; STJ, sinotubular junction.

TL
FL
LV

LA

A B

Figure 16-16  TTE view of aortic dissection.  A, Transthoracic parasternal long-axis view showing a linear echo in the lumen of a dilated ascending aorta
consistent with a dissection flap (arrow) separating the true lumen (TL) from the false lumen (FL). In real time, this linear echo showed motion separate from
the motion of the aortic wall and was seen in multiple image planes. B, Color Doppler shows flow only in the TL.
344 CHAPTER 16  Diseases of the Great Arteries

n  ites where there is communication between the


S •
Extension of the dissection flap into a coronary
true and false lumens may be identified with color artery
Doppler. • Involvement of arteries that arise from the aorta
n An ascending aortic dissection usually requires
 n Echocardiography may detect the central com-

surgical intervention, so it is especially important plications of aortic dissection, but evaluation of
to determine if a dissection flap is present in the branch vessels typically requires other imaging
ascending aorta. techniques.
n TEE is more sensitive than transthoracic imaging for
detection of aortic dissection and should be the initial v  KEY POINTS
echo procedure when this diagnosis is suspected. Pericardial effusion may be due to rupture
o
of the dissection into the pericardial space.
v  KEY POINTS Because the effusion is acute, tamponade physi-
o Imaging artifacts may be mistaken for a dissec- ology may be present with only a small effusion.
tion flap. Approaches to avoiding a false posi- o Aortic regurgitation may be due to aortic dila-
tive diagnosis include imaging the flap in more tion with central noncoaptation of the leaflets
than one imaging plane, demonstrating flow in or due to a flail leaflet from extension of the
two separate lumens, and demonstrating the dissection flap into the valve (Fig. 16-18).
three key characteristics of a dissection flap. o Extension of a dissection flap into the coronary
o There may be more than one entry site from artery may be visualized on TTE or TEE imag-
the true into the false lumen and multiple exit ing in some cases. More often, the key finding
sites may be detected distally (see Fig. 16-14). is a regional wall motion abnormality due to
o The false lumen may be thrombosed. In this ischemia in the myocardium supplied by the
situation the flap does not move and the false dissected vessel.
lumen is filled with an irregular echodensity, o The proximal segments of the left carotid
consistent with thrombus. and subclavian and right brachiocephalic
o Localized dissection into the aortic wall may artery may be seen by echocardiography in
result in a crescent-shaped intramural hema- some cases. However, accurate evaluation of
toma, without a dissection flap (Fig. 16-17). these vessels and more distal arteries (renal,
o Outcomes with intramural hematoma and dis- mesenteric, etc.) requires other imaging
section are similar. approaches.
o Extraluminal, periaortic hematoma in associa-
tion with aortic dissection is a poor prognostic Sinus of Valsalva Aneurysm
marker. n Congenital sinus of Valsalva aneurysms are irreg-
ularly shaped, thin-walled outpouchings of the
Step 2: Look for Complications of Aortic sinus.
Dissection
n Complications of aortic dissection include:

Pericardial effusion LA
• Aortic regurgitation

LV

Ao

IMH

Figure 16-17  Intramural hematoma in the descending thoracic aor-


ta on chest CT.  These findings can be seen on TEE but may be difficult Figure 16-18  Severe secondary aortic regurgitation. The aortic dis-
to distinguish from atheromatous or extraaortic disease. IMH, Intramural section flap has extended into the aortic valve leaflet, resulting in severe
hematoma. regurgitation. Ao, Aorta.
Diseases of the Great Arteries  CHAPTER 16 345

n  upture into adjacent chambers results in a fistula


R v  KEY POINTS
from the aorta into the RV, RA, or LA, depending Rupture of a right coronary sinus aneurysm is
o
on which sinus is affected. into the RV, left coronary sinus into the LA, and
n Acquired sinus of Valsalva aneurysms usually are noncoronary sinus into the RA.
due to endocarditis and typically have a rounded o Flow in the fistula from the aorta is continuous
symmetric shape (Fig. 16-19). with high-velocity flow in both systole and dias-
tole, reflecting the systolic and diastolic pressure
differences between the aortas and receiving
chamber.
o Acquired aneurysms due to infection may
extend below the aortic valve, into the base of
the septum. Imaging in long-axis views helps
determine the level and extent of involvement.
Aortic Pseudoaneurysm
n An aortic pseudoaneurysm is a contained aortic
rupture (Fig. 16-20).
n 
Pseudoaneurysms may occur after aortic surgery
due to dehiscence at the proximal or distal anasto-
mosis or at a coronary reimplantation site.
v  KEY POINTS
A pseudoaneurysm is detected as an echolucent
o
space adjacent to the aorta. The pseudoaneu-
rysm may be echodense if hematoma is present.
o A pseudoaneurysm should be suspected when a
paraaortic mass is found in a patient with recent
or remote surgery on the ascending aorta.
Figure 16-19  Sinus of Valsalva aneurysm.  This short-axis view of the aor-
tic valve shows the valve leaflets open to a triangular shape in systole. There o Although often initially diagnosed by echocar-
is asymmetric dilation of the sinuses of Valsalva, most prominently involving diography, evaluation of the size and origin
the noncoronary sinus (arrows), consistent with a sinus of Valsalva aneurysm. of the pseudoaneurysm often requires a wide

DISTAL ASC AO
2
1

pA 1

Ao

A B
Figure 16-20  Aortic pseudoaneurysm.  In a patient with a Dacron tube graft replacement of the ascending aorta, an abnormal echolucent space is seen
adjacent to the ascending aorta, near the distal anastomosis site. A, The 3.5 by 4.5 cm diameter (measurements 1 and 2 shown) space appears lined by
thrombus. B, Color Doppler demonstrates flow from the aorta into this space, consistent with a contained aortic rupture or pseudoaneurysm. Ao, Aorta; DISTAL
ASC AO, distal ascending aorta; pA, pseudoaneurysm.
346 CHAPTER 16  Diseases of the Great Arteries

n  theroma that protrude into the aortic lumen


A
and atheroma associated with mobile thrombus
are associated with an increased risk of embolic
events.
v  KEY POINTS
Atheroma is identified as irregular focal areas
o
of thickening of the aortic wall, with or without
associated calcification.
o Images of the aortic arch are limited, even with
TEE, but atheroma may be detected from a
high esophageal position in some cases.
Persistent Left Superior Vena Cava
n 
A persistent left superior vena cava is a normal
variant in which left upper extremity venous return
enters the RA via the coronary sinus.
v  KEY POINTS
A dilated coronary sinus is seen in cross section,
o

A posterior to the left atrium, in long-axis views,


and in long axis with posterior angulation from
a 4-chamber view.
o The persistent left superior vena cava may be
directly visualized in some cases; Doppler flow
shows low-velocity antegrade systolic and dia-
stolic flow.
o A persistent left superior vena cava can be con-
firmed with saline contrast injection into the left
upper extremity venous system, where saline
Ao contrast opacifies the coronary sinus before
entering the right atrium.

Pulmonary Artery Abnormalites


Clinical Concerns
n Isolated abnormalities of the pulmonary artery are
rare; most pulmonary artery disease is associated
with congenital heart disease (Fig. 16-22).
n  Idiopathic dilation of the pulmonary artery is an
uncommon abnormality in which a dilated pulmo-
nary artery is seen in the absence of other congeni-
B tal lesions.
n Thrombus in the pulmonary artery may be seen on
Figure 16-21  Aortic atheroma.  A, The descending thoracic aorta was transesophageal or transthoracic imaging in some
evaluated in sequential short-axis images as the transducer was withdrawn cases, but echocardiography is not an accurate
in the esophagus. B, From a high esophageal position, the image plane was
turned medially and angulated inferiorly to show the aortic arch and ascend- approach to diagnosis of pulmonary embolism.
ing aorta. The irregular thickening of the aortic wall with an area of calcifica-
tion (with shadowing) is consistent with an aortic atheroma. Ao, Aorta. v  KEY POINTS
Abnormalities of the pulmonary artery associ-
o
field of view imaging approach, such as CMR ated with other congenital heart disease include
imaging or CT. pulmonary artery dilation and branch pulmo-
nary artery stenosis.
Atherosclerotic Aortic Disease o Pulmonary artery dissection is rare.
n 
Aortic atheromas may be detected on TEE
imaging of the ascending and descending tho- Basic Echocardiographic Approach
racic aorta and are a marker of coronary disease n 
The pulmonary artery is visualized in the trans-
(Fig. 16-21). thoracic short-axis view by angulation superiorly
Diseases of the Great Arteries  CHAPTER 16 347

.70 37dB 4 •/+1/0/ 1


PW Depth=150mm
PW Gate= 3.0mm
PW Gain= 14dB

V = –2.09m/s
.70 PG = 17.5mmHg
HPRF
Ao
PA
1.0

m/s

2.0

Figure 16-23  Branch pulmonary artery stenosis.  Stenosis of the distal


right pulmonary artery in this patient with a repaired tetralogy of Fallot is
demonstrated using high-pulse repetition frequency (HPRF) Doppler to localize
Figure 16-22  Severe dilation of the pulmonary artery.  TEE in a patient the origin of the high velocity jet.
with congenital heart disease in a short-axis view of the aortic valve shows a
severely dilated pulmonary artery. Ao, Aorta; PA, pulmonary artery.
o A small amount of pulmonic regurgitation is
normal and is characterized by a narrow jet on
to demonstrate the bifurcation of the main pulmo- color Doppler and a low-intensity, low-velocity
nary artery. spectral Doppler signal.
n Images also can be obtained in the transthoracic o With pulmonary hypertension, the pulmonic
RV outflow view. regurgitant velocity is increased, reflecting the ele-
n TEE imaging of the pulmonary artery is challeng- vation of pulmonary diastolic pressure, and there
ing and requires high esophageal views. is a shortened time-to-peak velocity and mid-­
n Color, pulsed, and continuous wave (CW) Dop- systolic deceleration in the antegrade flow signal.
pler allow detection of pulmonic regurgitation and o With branch pulmonary artery stenosis, a high-
abnormal pulmonary artery flow patterns. velocity signal may be detected with spectral
Doppler, even when image quality is subopti-
v  KEY POINTS mal (Fig. 16-23).
In adults, visualization of the lateral wall of the
o o With a patent ductus arteriosus, the diastolic
pulmonary artery is difficult due to limitation of flow from the descending aorta into the pul-
the acoustic window by the adjacent lung. monary artery is seen with color and spectral
o Pulmonary artery diameter measurements are Doppler.
mainly helpful in adults with congenital heart o Full evaluation of the main pulmonary artery
disease, and may be better made by other and branches in patients with congenital heart
imaging techniques. disease usually requires CMR or CT imaging.
348 CHAPTER 16  Diseases of the Great Arteries

THE ECHO EXAM

Disease of the Great Vessels


Examination of the Aorta
Aortic
Segment Modality View Recording Limitations
Aortic TTE Parasternal long Images of sinuses of Valsalva, Shadowing of posterior aortic
sinuses axis aortic annulus, and sinotubular aortic sinuses
junction
TEE High esophageal Standard long-axis plane by
long axis rotating to about 120°-130°
Ascending TTE Parasternal long Move transducer superiorly to Only limited segments
axis image sinotubular junction and visualized, variable between
ascending aorta patients
TTE Doppler Apical LVOT and ascending aorta flow Velocity underestimation if the
recorded with pulsed or CW angle between the Doppler
Doppler from an anteriorly beam and flow is not parallel
angulated 4-chamber view
TEE High esophageal From long-axis view, move The distal ascending aorta may
long axis transducer superiorly to image not be visualized
ascending aorta
Arch TTE Suprasternal Long- and short-axis views of Descending aorta appears to
aortic arch taper as it leaves the image
plane
TEE High esophageal From the short-axis view of the View not obtained in all patients.
initial segment of the The aortic segment at the
descending thoracic aorta, turn junction of the ascending aorta
the probe towards the patient’s and arch may not be visualized
right side and angulate inferiorly
Descending TTE Parastenal and Rotate from long-axis view to Depth of thoracic aorta on TTE
thoracic modified apical image thoracic aorta in long axis limits image qualtiy
views posterior to LV TEE usually needed for
From apical 2-chamber view, use diangosis
lateral inferior/posterior angulation
and counter-clockwise rotation to
image aorta
TTE Doppler Suprasternal Descending aorta flow recorded Low wall filters needed to
with pulsed Doppler from SSN evaluate for holodiastolic flow
view reversal
TEE Short-axis aorta Sequential short-axis views of the Long-axis view allows further
aorta from the level of the evaluation of abnormal
diaphragm to the arch with the findings
image plane turned posteriorly
and the transducer slowly
withdrawn in the esophagus
Proximal TTE Subcostal Long axis of proximal abdominal Only the proximal segement is
abdominal aorta visualized
TTE Doppler Subcostal Proximal abdominal aorta flow Low wall filters needed
recorded with pulsed Doppler to evaluate for holodiastolic
flow reversal
TEE Transgastric From the transgastric position, Does not allow evaluation
portions of the abdominal aorta of entire abdominal aorta
may be seen posteriorly

LVOT, Left ventricular outflow tract; SSN, suprasternal notch.


Diseases of the Great Arteries  CHAPTER 16 349

Aortic Dissection Complications of Thoracic Aortic Dissection


Dissection flap Aortic valve regurgitation
In aortic lumen Due to aortic root dilation
Independent motion Due to leaflet flail
True and false lumen
Coronary artery occlusion due to dissection at the
Entry sites
orifice
Thrombosis of false lumen
Ventricular fibrillation
Intramural hematoma Acute myocardial infarction
Indirect findings Distal vessel obstruction or occlusion
Aortic dilation Carotid (stroke)
Aortic regugitation Subclavian (upper limb ischemia)
Coronary ostial involvement
Aortic rupture
Pericardial effusion
Into the pericardium
Pericardial effusion
Pericardial tamponade
Into the mediastinum
Into the pleural space
Pleural effusion
Sinus of Valsalva Aneurysm Exsanguination
Congenital
Complex shape
Protrusion into RV outflow tract
Fenestrations
Acquired
Aortic Atheroma
Infection or inflammation Complex (≥4 mm or mobile)
Symmetric shape Associated with:
Communication with aorta Coronary artery disease
Potential for rupture Cerebroembolic events
350 CHAPTER 16  Diseases of the Great Arteries

SELF-ASSESSMENT QUESTIONS
Question 1
An 82-year-old woman presents with dyspnea. A
transthoracic echo is ordered to evaluate for conges-
tive heart failure. The following image is obtained
from the subcostal window (Fig. 16-24).
You conclude the patient has:
A. Abdominal ascites
B. Atherosclerotic disease
C. Aortic dissection
D. Hepatic vein plethora

Figure 16-24 

Question 2
A transthoracic echocardiogram is ordered for a A.
Aortic coarctation
newly diagnosed murmur. No valvular abnormalities B. Aortic regurgitation
or dysfunction is identified. Doppler interrogation of C. Patent ductus arteriosus
the thoracic aorta reveals the following (Fig. 16-25). D. Aortic atherosclerosis
You conclude that the most likely diagnosis is:

3.0

m/s

2.0

Figure 16-25 
Diseases of the Great Arteries  CHAPTER 16 351

Question 3 Question 4
Identify the structures (Fig. 16-26, numbered 1 to 4) A patient is referred for an echocardiogram. Dur-
by matching them with the following list: ing scanning, the sonographer moves the transducer
A. Descending aorta to the apical windows, and the following image is
B. Ascending aorta obtained (Fig. 16-27).
C. Left ventricle Additional imaging is suggested to confirm the
D. Left atrium diagnosis. You recommend:
E. Right pulmonary artery A. Transpulmonary microbubble contrast study
F. Left pulmonary artery B. Repeat imaging following Valsalva maneuver
G. Brachiocephalic vein C. 3D TEE imaging
H. Azygous vein D. Imaging from left supraclavicular window

Figure 16-27 

Question 5
A 54-year-old man presents to the emergency
department following a motor vehicle accident. A
Figure 16-26  TEE is ordered and the following image is obtained
(Fig. 16-28). Based on this study, you make the fol-
lowing diagnosis:
A. Intramural hematoma
B. Aortic dissection
C. Penetrating aortic atheroma
D. Aortic transection

Figure 16-28 
352 CHAPTER 16  Diseases of the Great Arteries

Question 6 Question 8
Several echocardiographic findings may be associated The following image is obtained (Fig. 16-30). Based on
with an ascending aortic dissection. Which of the fol- this image, you are concerned about which diagnosis?
lowing is least likely of a complication of acute aortic A. Aortic dissection
dissection? B. Marfan syndrome
A. Pericardial effusion C. Sinus of Valsalva aneurysm
B. Periaortic hematoma D. Hypertensive heart disease
C. Myocardial infarction E. Bicuspid aortic valve
D. Mitral regurgitation
Question 7
A 56-year-old man with a long history of hyperten-
sion presented to the emergency department with
the sudden onset of severe tearing chest pain. His
ECG showed only nonspecific ST changes. Urgent
echocardiography at the bedside was performed (Fig.
16-29). Based on the clinical history and this image,
the echocardiographic exam should next focus on:
A. Calculation of LV ejection fraction
B. Evaluation of regional ventricular systolic

function
C. Imaging of the ascending aorta
D. Measurement of tricuspid regurgitant jet

velocity
E. Respiratory variation in RV and LV diastolic
filling
Figure 16-30 

Question 9
The most likely diagnosis in the 56-year-old man with
this aortic flow signal (Fig. 16-31) is:
A. Aortic coarctation
B. Aortic regurgitation
C. Patent ductus arteriosus
D. Branch pulmonary stenosis
E. Persistent left superior vena cava

1.0
Figure 16-29 

m/s

.50

Figure 16-31 
Diseases of the Great Arteries  CHAPTER 16 353

ANSWERS

Answer 1: B Answer 4: D
This image is taken from the subcostal view. The liver This patient has a severely enlarged coronary sinus,
is closest to the transducer, and a small portion of the raising clinical suspicion for a persistent left-sided
right atrium is seen adjacent to the liver on the right superior vena cava. 2D imaging from the left supra-
side of the image. If present, ascites is commonly seen clavicular window may show the left-sided superior
in this view, surrounding the liver. Distal to the liver, a vena cava (Fig. 16-32).
long-axis view of the abdominal aorta is seen. Dense
calcification is present along the length of the aorta,
consistent with atherosclerotic disease. A clear dissec-
tion flap is not present.
Answer 2: C
The Doppler tracing was taken of the descending
thoracic aorta from the suprasternal notch. Sys-
tolic flow is directed away from the transducer. In
both systole and diastole, there is continuous flow
directed towards the transducer. This is consis-
tent with a patent ductus arteriosus, where flow is
directed from the descending thoracic aorta to the
pulmonary artery. Flow is continuous because of
lower pressure in the pulmonary vasculature rela- Figure 16-32 
tive to the aorta. Aortic coarctation, if significant,
would result in increased anterograde velocity and
continued anterograde flow during diastole (away If this diagnosis is suspected, an agitated saline
from the transducer), “diastolic run-off.” With aortic contrast study can confirm the diagnosis. For an agi-
regurgitation of at least moderate severity, holodia- tated saline study, intravenous access is placed in the
stolic flow reversal may be present. However, with left upper extremity. Following injection of saline, a
aortic regurgitation, there is a demarcation between persistent left superior vena cava is present if contrast
systole (flow directed away from the transducer) opacifies the coronary sinus before the right-sided
and diastole (flow directed towards the transducer). chambers. Answer “A” is incorrect; transpulmonary
Additionally, the clinical vignette described no echo- microbubble contrast traverses the pulmonary circula-
cardiographic findings of valvular abnormalities or tion and is used to opacify left ventricular endocardial
dysfunction. Aortic atherosclerosis does not typically borders. 3D imaging of the interatrial septum may
affect the Doppler signal in diastole. If significant be helpful if an atrial septal defect is suspected, but
atherosclerosis is obstructive, this may affect antero- the image shown suggests a tubular, vascular structure
grade (systolic) velocities. rather than a septal defect. Imaging following a Val-
salva maneuver would not be helpful for diagnosis of
Answer 3: a persistent left superior vena cava.
This suprasternal notch long-axis view of the aortic
arch shows the following matches: Answer 5: A
1: B. Ascending aorta The image is a short-axis view of the descending tho-
2: A. Descending aorta racic aorta taken at 0°. The aortic lumen is echolucent.
3: D. Left atrium In the far field there is crescentic thickening of the wall
4: E. Right pulmonary artery (hematoma) which encompasses half of the diameter
The left ventricle is not well seen in this view. The of the aorta and is echodense relative to the lumen. No
right pulmonary artery is seen in cross-section under clear dissection flap is seen. Aortic atherosclerosis typi-
the aortic arch but the image plane does not include cally lies along the aortic wall. Prominent atherosclerotic
the left pulmonary artery. The azygous vein arises lesions may protrude into the lumen of the aorta, but the
from the inferior vena cava, courses superiorly adja- wall is not smooth as is seen in this image. Atherosclero-
cent to the spine receiving the drainage of the inter- sis is also echodense (calcium), commonly with acoustic
costal veins, and then enters the superior vena cava shadowing of the far field. An aortic transection shows
above the right mainstem bronchus. When normal as a discontinuity of the aorta from long-axis views. Peri-
in size, it is rarely seen by echocardiography and lies aortic hematoma is present with Doppler imaging show-
medial to the descending thoracic aorta. ing flow outside the aortic border.
354 CHAPTER 16  Diseases of the Great Arteries

Answer 6: D requires identification of the underlying cause—aortic


Proximal propagation of an ascending aortic dissec- dissection in this case.
tion to the aortic root may extend to the coronary
arteries and aortic valve. If the outer wall of the aorta Answer 8: E
is disrupted from the dissection, periaortic hematoma, The image shows a parasternal long-axis view of the
or pericardial effusion will be seen. Propagation down left ventricle and aortic root. The ascending aorta is
a coronary artery may occlude the vessel leading to dilated. An estimate of aortic diameter can be made
myocardial infarction. Involvement of the dissection comparing the diameter of the aorta with the centi-
to aortic valve may cause faulty leaflet coaptation meter tick marks along the edge of the image. Nor-
and aortic regurgitation. Mitral regurgitation is not a mal caliber ascending aorta is in the ∼3.3 to 3.8 cm
direct complication of aortic dissection. range at end-diastole. This patient’s aorta measures
over 4.0 cm, suggesting an ascending aortopathy. The
Answer 7: C aortic luminal diameter is comparable in size to the
This patient’s clinical history is strongly suggestive of LV internal dimensions, another indicator of aortic
acute aortic dissection and the presence of a small dilation. The valve closure plane is asymmetric with
pericardial effusion is an ominous sign, suggesting a smaller anterior cusp and a larger posterior cusp.
partial rupture into the pericardium. The immedi- Asymmetric aortic valve closure coupled with ascend-
ate next step should be evaluation of the ascending ing aortic enlargement is most consistent with a bicus-
aorta, which in this patient showed a definite dissec- pid aortic valve. In the image shown, there is a linear
tion flap (arrow) on a TTE parasternal long-axis view reverberation artifact in the posterior aspect of the
(Fig. 16-33). aortic lumen which may give the appearance of a dis-
section flap. It is important to image the aorta from
multiple views to exclude a dissection. In Marfan
syndrome, ascending aortopathy may lead to blunt-
ing or loss of the angle at the sinotubular junction. A
sinus of Valsalva aneurysm is focal dilation of one of
the coronary sinuses. In this case, dilation is diffuse
along the length of the ascending aorta. In chronic,
LV
systemic hypertension, patients may develop mild
Ao dilation of the ascending aorta, but the sinotubular
junction is typically preserved and aortic valve closure
plane is not affected.
Answer 9: B
LA This pulsed Doppler recording in a long-axis view of
the descending aorta on TEE shows antegrade flow
in systole with reversal of flow for the full duration of
diastole (e.g., holodiastolic). This flow signal indicates
continuous retrograde diastolic flow in the descend-
ing aorta, consistent with a communication from the
aorta to a chamber or vessel with a lower diastolic
Figure 16-33  pressure. Possible diagnoses include aortic regurgita-
tion (aorta to LV) or a patent ductus arteriosus (aorta
to pulmonary artery), but aortic regurgitation is most
When aortic dissection is suspected and transtho- likely given the patient’s age. This diagnosis can be
racic imaging is not diagnostic, prompt additional confirmed by color and CW Doppler evaluation of
imaging (CT angiography or TEE) is recommended the aortic valve and by confirming the absence of
as urgent surgical intervention is indicated. In a clini- diastolic flow in the main pulmonary artery. With
cally stable patient, quantitative evaluation of ven- aortic coarctation, the systolic velocity is higher than
tricular function is appropriate, but if the patient is normal and there is continuous forward flow in dias-
unstable, a qualitative assessment of function (visual tole. Branch pulmonary stenosis would result in high
estimate) is reasonable. Pulmonary embolism may velocity systolic flow in the right or left pulmonary
result in an acute elevation in pulmonary pressures, artery, which are not seen in this view. A persistent left
but is unlikely to cause a pericardial effusion. Tam- superior vena cava would have a typical venous flow
ponade physiology, resulting in respiratory variation pattern with low-velocity antegrade systolic and dia-
in RV and LV diastolic filling, can occur with aortic stolic flow and is associated with an enlarged coronary
dissection but would be evident clinically as a low sinus, as the left superior vena cava usually drains into
blood pressure or pulsus paradoxus and treatment the right atrium via the coronary sinus.
17 The Adult with Congenital Heart Disease
BASIC PRINCIPLES Basic Transesophageal Approach
Identification of Cardiac Chambers and Vessels Assess the Risk of TEE and Institute Appropriate
Valve Stenosis and Regurgitation Modifications in the Study Protocol
Intracardiac Shunts Determine the Objectives of the TEE Study
Complex Disease TEE Imaging Sequence
STEP-BY-STEP APPROACH SPECIAL CONSIDERATIONS IN COMMONLY SEEN
Basic Transthoracic Echo Exam CONDITIONS
Review the Clinical History Atrial Septal Defect
Acquire Images and Doppler Data from the Ventricular Septal Defect
­Parasternal Window Patent Ductus Arteriosus
Acquire Images and Doppler Data from the Apical Aortic Coarctation
Window Ebstein Anomaly
Acquire Images and Doppler Data from the Complex Congenital Heart Disease
­Subcostal Window Tetralogy of Fallot
Acquire Images and Doppler Data from the Congenitally Corrected Transposition of the Great
­Suprasternal Notch Window Arteries
Pulmonary Pressure Estimation Complete Transposition of the Great Arteries
Review and Report Study Results Fontan Physiology with Tricuspid Atresia
Determine Remaining Anatomic/Physiologic THE ECHO EXAM
Questions SELF-ASSESSMENT QUESTIONS

BASIC PRINCIPLES
Identification of Cardiac Chambers
and Vessels
n 
Identification of the chambers, great vessels, and
their connections is the first step in echocardio-
graphic evaluation of the patient with congenital
RV LV
heart disease (see The Echo Exam).
v  KEY POINTS
o The right ventricle (RV) and left ventricle (LV)
are distinguished based on the atrioventricular
valve anatomy and position, the presence or
absence of a moderator band, and the presence
or absence of a muscular infundibular region
(Fig. 17-1).
o The size and location of the ventricular cham-
ber is not reliable for distinguishing the ana-
tomic LV from RV.
o The RV tends to have a more triangular
shape, whereas the LV is more ellipsoid, but
shape can be unreliable when severe dilation is
present.
Figure 17-1  Identify the ventricles.  In this patient with congenital heart
o The aorta and pulmonary artery are identified
disease, the anatomic RV and LV are identified in the apical 4-chamber view
by their distal anatomy; the pulmonary artery based on morphologic features that include a more apical position of the
bifurcates into right and left pulmonary arteries tricuspid versus mitral annulus (arrow) and the moderator band and trabecu-
whereas the aorta supplies the head and neck lations in the RV. The atrioventricular valves are associated with the correct
anatomic ventricle, even when atrial or great vessel connections are discor-
arteries via the arch (Fig. 17-2). dant. Mitral and tricuspid valve anatomy were confirmed in other views. This
o In addition to anatomic definitions, the ventri- patient presents with a patent ductus arteriosus mild LV and LA enlargement
cle that pumps oxygenated blood into the aorta but an otherwise unremarkable 4-chamber view.

355
356 CHAPTER 17  The Adult with Congenital Heart Disease

is called the systemic ventricle, and the ventricle


that pumps systemic venous return into the pul- v  KEY POINTS
monary artery is called the pulmonic ventricle. Congenital valve stenosis may be valvular, sub-
o
valvular, or supravalvular.
Valve Stenosis and Regurgitation o Delineation of the level of obstruction using
n 
Valve stenosis and regurgitation are evaluated pulsed and color Doppler is necessary, in addition
using the same methods as for acquired valve to continuous wave (CW) Doppler measurements.
disease.
Intracardiac Shunts
n 
Intracardiac shunts are detected and quantitated
using multiple Doppler modalities.
Aortic arch v  KEY POINTS

oIntracardiac shunts are detected based on the
presence of a flow disturbance on the downstream
side of the shunt with color or pulsed Doppler.
o The velocity and shape of the CW Doppler
signal across an intracardiac shunt reflects the
pressure difference across the shunt and is a key
factor in determining shunt location.
o The pulmonic-to-systemic flow ratio is calcu-
lated based on transpulmonic stroke volume
and transaortic stroke volume.
Complex Disease
n Complex congenital heart disease should be evalu-
ated at centers with established Adult Congenital
Heart Disease programs.
v  KEY POINTS
A This chapter includes a basic approach for sim-
o
ple conditions or patients with a well-­established
diagnosis.
o More complex cases require additional data
PA acquisition by sonographers and physicians
with expertise in congenital heart disease.

STEP-BY-STEP APPROACH
Basic Transthoracic Echo Exam
n  A structured study sequence is needed to ensure
that all the images and Doppler flows needed for
diagnosis are recorded.
PA n Most adult echocardiography laboratories acquire
images and Doppler data in a similar sequence as
for a standard adult study.
n 
With complex disease, the sonographer and phy-
sician work together during image acquisition to
ensure the needed data are obtained.
Step 1: Review the Clinical History
B n Review the details of any previous surgical or per-
cutaneous procedures.
Figure 17-2  Identify the great arteries.  A, The aortic arch, in a supra- n Obtain reports (and images when possible) of any
sternal notch view, is identified by the arch and head and neck vessels.
B, The pulmonary artery (PA), from an apical transducer position angulated
previous diagnostic tests.
anteriorly from the 4-chamber view, is identified based on its bifurcation to n 
Determine the specific objectives of the current
the main PA branches (arrows). examination.
The Adult with Congenital Heart Disease  CHAPTER 17 357

v  KEY POINTS v  PARASTERNAL SHORT-AXIS VIEW


Knowledge of previous procedures and diag-
o The short-axis view shows the relationship of the
o
nostic studies ensures the current exam provides semilunar (aortic and pulmonic) valves (Fig. 17-4).
additional information and focuses on the key o Basal ventricular size, systolic function, and sep-
clinical issues. tal motion are evaluated.
o Complete evaluation of the adult with congenital o Ventricular and atrial septal defects are demon-
heart disease often requires multiple diagnostic strated using color Doppler.
modalities; the echocardiographic examination
provides only part of the needed information.

Step 2: Acquire Images and Doppler Data


from the Parasternal Window
v  PARASTERNAL LONG-AXIS VIEW

oThe position and angle of the transducer
needed to obtain a long-axis view help identify
Ao
an abnormal cardiac position in the chest (dex-
troversion or dextrocardia).
o Great vessel morphology and location are eval-
uated in the long-axis view. PA
o The connections of the great vessels to the ven-
tricles are determined.
o Doppler is used to evaluate for valve regurgita-
tion, areas of stenosis, and intracardiac shunts.
v  KEY POINTS
o Medial and lateral angulation of the transducer
shows the relationship of the great vessels to
each other and to the ventricular chambers. Figure 17-3  Anteriorly located aorta (Ao).  From a parasternal long-axis
o The pulmonary artery normally is anterior and image of a patient with transposition of the great arteries, the anterior-pos-
runs perpendicular to the aorta; an anteriorly terior locations of the aorta (larger and anterior) and pulmonary artery (PA)
located aorta that lies parallel with the pulmo- (smaller and posterior) are opposite the normal position, and the vessels lie
parallel to each, rather than in the normal perpendicular relationship.
nary artery suggests transposition of the great
vessels (Fig. 17-3).
o The transducer is moved up one or more inter-
spaces to follow the great vessel(s) seen in the
long-axis view, allowing differentiation of the
aorta from the pulmonary artery. Ao
o Ventricular and atrial septal defects can be iden-
tified with color Doppler while slowly scanning PV
from lateral to medial in the long-axis image
plane and from apex to base in the short-axis
plane.
o Standard measurements of great vessels and car-
diac chambers are recorded in the long-axis view.
o The transducer location for image acquisition
is recorded because this information cannot be
determined from the images themselves.
o An enlarged coronary sinus suggests a persis-
tent left superior vena cava. If needed, this can
be confirmed with intravenous administration
of saline contrast via the left upper extrem-
ity; after injection, contrast will appear in the
coronary sinus before it drains into the RA.
Two-dimensional (2D) imaging may confirm Figure 17-4  Normal aortic and pulmonary artery positions.  The nor-
mal aortic valve (Ao) and pulmonic valve (PV) planes are perpendicular to
a persistent left superior vena cava if a large each other so that the aortic valve is seen in short-axis when the pulmonic
vessel is seen branching off from the left sub- valve is seen in long axis. Normally, the most anterior great vessel at the
clavian vein. base of the heart is the pulmonary artery.
358 CHAPTER 17  The Adult with Congenital Heart Disease

o A cleft anterior mitral valve leaflet, commonly CW Doppler measurement of atrioventricular
o
associated with primum atrial septal defects, valve regurgitant velocity allows measurement
may be seen from a parasternal short-axis view of RV (or pulmonary) systolic pressure.
of the mitral valve. o The RV outflow view allows visualization of
RV outflow obstruction at the subvalvular, pul-
v  KEY POINTS monic valve, or supravalvular level.
o The normal relationship of the aortic and pul- o Doppler evaluation allows localization of the
monic valve planes is perpendicular to each level of RV outflow obstruction and calculation
other. When both are seen in short-axis view of the gradient between the ventricle and pul-
in the same image plane, transposition of the monary artery.
great vessels is present (Fig. 17-5).
o The anterior-posterior and medial-lateral loca- v  KEY POINTS
tions of the aorta and pulmonary artery at the Standard RV inflow and outflow views may
o
base are evaluated; the aortic root is anterior to the be difficult to obtain when transposition is
pulmonary artery when transposition is present. present.
o Atrial and ventricular septal defects typically o Slow angulation from the long-axis view toward
are well seen in the short-axis view. the RV inflow view, using color Doppler, may
o The location of a ventricular septal defect relative be helpful for diagnosis of atrial and ventricular
to the aortic valve helps distinguish a membra- septal defect.
nous from subpulmonic (or supracristal) defect. o When right (or pulmonic) ventricular out-
o Pulsed and CW Doppler interrogation of any flow tract obstruction is present, ventricular
abnormal color flow signal is helpful for diag- systolic pressure does not equal pulmonary
nosis based on the time course and velocity of systolic pressure; instead, the transpulmonic
flow. gradient is subtracted from ventricular sys-
tolic pressure to determine pulmonary systolic
v  PARASTERNAL RV INFLOW AND pressure (Fig. 17-7).
OUTFLOW VIEWS o Evaluation of RV outflow obstruction requires
o The RV inflow view is helpful for evaluation of color and pulsed Doppler to determine the ana-
the RA, pulmonary atrioventricular valve, and tomic location of the increase in velocity and
annulus (Fig. 17-6). CW Doppler to measure the peak velocity.

Aortic
valve RV

Pulmonic
valve

TV

RA

Figure 17-5  Abnormal aortic and pulmonary artery positions. With


transposition of the great arteries, the aorta is anterior to the pulmonary
­artery with a side-by-side (instead of crisscross) relationship of the great Figure 17-6  Ebstein anomaly. The RV inflow view in a patient with
arteries. The semilunar valves are both in the same image plane so that the ­Ebstein anomaly of the tricuspid valve (TV) shows the apical displacement of
aortic and pulmonic valves are both seen in cross-section in a short-axis the septal leaflet from the annulus (short arrow), with the atrialized portion of
view. the RV between the annulus and leaflet attachment level.
The Adult with Congenital Heart Disease  CHAPTER 17 359

Step 3: Acquire Images and Doppler Data n  nterior angulation from the 4-chamber view
A
from the Apical Window often allows imaging of the connection from each
n  The morphology, size, and function of both ven- ventricle to the great vessels (Fig. 17-8).
tricles are evaluated in the 4-chamber, 2-chamber, n Ventricular inflow and outflow signals are recorded
and long-axis views. using pulsed and CW Doppler.
n The atrioventricular valves are evaluated using 2D
imaging, color Doppler, and CW Doppler. v  KEY POINTS
o The normal transducer orientation is used to
ensure correct identification of the location and
anatomy of each ventricle.
V = –3.41m/s o The apical view often allows recognition of
CW:1.75MHz PG = 46.6mmHg the anatomic RV based on the shorter distance
1.0 from the annulus to the apex, compared with
the LV, as well as presence of the moderator
band.
m/s o The atrioventricular valves are evaluated using
standard Doppler approaches.
o With anterior angulation to image the great
vessels, the bifurcation of the pulmonary artery
and the curve of the aortic arch may be visual-
ized, helping with identification of the ventricu-
lar to great vessel connections.
o With posterior angulation, the size and location
of the coronary sinus are evaluated.
o Atrial anatomy and size may be evaluated,
4.0 although the distance of these chambers from
the transducer may limit detailed evaluation,
particularly in patients with an interatrial baffle
Figure 17-7  Pulmonic stenosis.  From a parasternal window, CW Doppler repair.
was used to record this high-velocity systolic ejection signal from the region
of the pulmonic valve in a patient with pulmonic stenosis. This signal might
o Evaluation of the interatrial septum may be
be due to subpulmonic or valvular stenosis; 2D imaging and color Doppler limited by ultrasound dropout because the
are helpful in localizing the level of obstruction. atrial septum is parallel to the ultrasound beam.

LV
RV

LV
RV

Ao PA

PA

A B
Figure 17-8  Transposition of the great arteries (TGA).  In this patient with TGA, from the apical 4-chamber view, the transducer is angulated anteriorly to
show the pulmonary artery (PA) with its bifurcation (A) and then the ascending aorta (Ao), which has coronary ostia and an arch in the far field (B). This view is
helpful for documenting which ventricle ejects into each great vessel. In this patient with transposition of the great vessels and an interatrial baffle repair, the
RV ejects into the anteriorly located aorta and the LV ejects into the more posteriorly located pulmonary artery.
360 CHAPTER 17  The Adult with Congenital Heart Disease

Step 4: Acquire Images and Doppler Data n  ortic coarctation is evaluated (or excluded) based
A
from the Subcostal Window on CW Doppler descending aortic flow (see Fig.
n  The subcostal 4-chamber view allows evaluation 17-10 and Fig. 17-11).
of the interatrial septum (Fig. 17-9). n Images and pulsed Doppler flows in the superior
n RV size and systolic function often are best evalu- vena cava are useful in many types of congenital
ated from the subcostal view. heart disease.
n 
The entrance of the inferior vena cava provides
clear identification of the RA chamber.
v  KEY POINTS
The ultrasound beam is perpendicular to the
o
interatrial septum from the subcostal view so
that ultrasound dropout, simulating an atrial
defect, is less likely. This view is optimal for 2D Ao
and color Doppler detection of an atrial septal
defect or patent foramen ovale.
o In adults, the free wall of the RV may not be
well seen in apical views. The subcostal view RPA
provides a more standard image plane of the
RV, with the ultrasound beam perpendicular to
the RV free wall, and thus is more reliable for
evaluation of RV size and function.
o The junction of the inferior vena cava and
RA provides anatomic information on atrial
situs, in addition to allowing estimation of RA
pressure.
Step 5: Acquire Images and Doppler Data
from the Suprasternal Notch Window Figure 17-10  Aortic arch imaging.  The suprasternal notch view is used to
evaluate aortic coarctation. However, on 2D imaging even a normal descend-
n 
A standard transducer orientation allows identifi- ing aorta (Ao) appears to taper (arrow) because the curvature of the vessel
cation of aortic arch position and anatomy. results in an oblique plane through the vessel. RPA, Right pulmonary artery.

RA
RV

LV
RA

LA

LA

A B
Figure 17-9  Secundum atrial septal defect.  The subcostal view is ideal to evaluate for a secundum atrial septal defect. A, In this patient, the RA and RV
are enlarged and there is a defect in the center of the atrial septum. Because the ultrasound beam is perpendicular to the atrial septum from this window, this
is a true defect and not echo dropout. B, Color flow imaging confirms the large defect with left to right flow.
The Adult with Congenital Heart Disease  CHAPTER 17 361

v  KEY POINTS
V = –3.61m/s APEX TV
o A right-sided aortic arch may be missed if the PG = 52.0mmHg
transducer or image orientation is reversed. 1.0
o Normal systolic and diastolic flow in the descend-
ing aorta excludes a diagnosis of aortic coarctation.
o The superior vena cava typically is to the right m/s
of the ascending aorta. Flow patterns are diag-
nostic if obstruction is present or when the
superior vena caval flow has been redirected
into the pulmonary artery.
o The right pulmonary artery is seen inferior to
the arch. When ultrasound penetration is opti-
mal, the left atrium (LA) and pulmonary veins
also may be identified.
o The branch pulmonary arteries may be seen in
the parasternal short-axis view in some patients.
o An aortic to pulmonary artery shunt may be
evaluated from the suprasternal notch view in
some cases (e.g., patent ductus arteriosis, aortic to
pulmonary window).
5.0
Step 6: Pulmonary Pressure Estimation
n In the absence of pulmonic stenosis, RV (and pul-
Figure 17-12  Causes of increased tricuspid regurgitant velocity. This
tricuspid regurgitant jet shows a maximum velocity of 3.6 m/s, consistent
monary) systolic pressure is calculated by the stan- with an RV to RA systolic pressure difference of 52 mmHg, or an estimated
dard approach based on tricuspid regurgitant jet RV systolic pressure of 62 mmHg assuming an RA pressure of 10 mmHg.
velocity and estimated RA pressure (Fig. 17-12). However, this patient has pulmonic stenosis, so the RV-to-pulmonary artery
n With complex congenital heart disease, estimation (PA) systolic gradient must be subtracted from the estimated RV pressure
to estimate PA systolic pressure. This is the same patient as in Fig. 17-7,
of pulmonary pressures depends on the exact car- so estimated PA systolic pressure is 62 mmHg − 47 mmHg = 15 mmHg.
diac anatomy.
v  KEY POINTS
o When pulmonic stenosis is present, the trans-
pulmonic systolic gradient is subtracted from
the estimated RV systolic pressure.

.88 45dB 1 •/+1/0/ 1 CW:2MHz DESC AO


PW Depth= 72mm
PW Gate= 3.0mm
PWGain= 3dB

m/s
.88

PW:1.75MHz

.50

m/s

3.0

A
2.0 B
Figure 17-11  Doppler flows in aortic coarctation.  Flow is recorded using pulsed Doppler in the descending aorta proximal to the coarctation (A) and with
CW Doppler as blood passes through the narrowed segment (B). When a severe coarctation is present, there is persistent antegrade flow in diastole (arrow)
due to a higher diastolic pressure proximal, compared with distal, to the coarctation.
362 CHAPTER 17  The Adult with Congenital Heart Disease

With a large unrestricted ventricular septal


o
v  KEY POINTS
defect and Eisenmenger physiology, pulmonary The findings in most patients with congenital
o
artery and aortic pressures are equalized, even disease can be described using a standard report
if there is no tricuspid regurgitation. format; however, with complex congenital heart
o With tricuspid atresia and Fontan physiology disease, a more detailed narrative description is
(direct connection of systemic venous return to needed.
the pulmonary artery pulmonary pressures are o The echocardiographic study should not be used
low with a venous type blood flow pattern. to deduce the surgical history; instead, the surgical
o Pulmonary diastolic pressure can be estimated history should be reviewed to ensure the echocar-
from the end-diastolic pulmonic regurgitant diographic study provides a complete evaluation.
velocity, plus an estimate of RV diastolic pres- o The echocardiographic study should seek to
sure (Fig. 17-13). answer the specific clinical question articulated
o The velocity through a ventricular septal by the referring physician.
defect reflects the LV to RV systolic pressure o The current study should be compared with
difference. previous examinations (with side-by-side review
of images when possible).
Step 7: Review and Report Study Results
n  ost adult congenital disease studies are reported
M Step 8: Determine Remaining Anatomic/
in the standard format with additional sections for Physiologic Questions
the congenital findings. n Initial evaluation of simple congenital heart dis-
n The report describes the anatomy and physiology, ease and follow-up studies of more complex disease
with an indication of the level of certainty of each may require only transthoracic echocardiography.
finding, depending on data quality. n Additional diagnostic procedures often are needed
n The echocardiographic findings are interpreted in for evaluation of complex congenital heart disease.
view of the clinical history, previous surgical proce-
dures, and current clinical indication. v  KEY POINTS
o 
Transthoracic echocardiography often cannot
fully evaluate atrial-level anatomy and flow in
patients with Fontan physiology or an interatrial
baffle repair, due to the distance of the trans-
.69 39dB 4 •/+1/0/ 1
CW Focus= 63mm
ducer from the structures of interest (Fig. 17-14).
CW Gain= 9dB

V = 2.81m/s
.69 PG = 31.5mmHg

CW:2MHz
6.0 RV LV

PVR SVR

m/s

2.0

Figure 17-14  Interatrial baffle. Apical 4-chamber view of a patient


with transposition of the great arteries (TGA) and an interatrial baffle re-
Figure 17-13  Pulmonic regurgitant velocity.  The end-diastolic pulmonic pair. The systemic (anatomic right, RV) ventricle fills from the pulmonary
regurgitant velocity reflects the diastolic pulmonary artery (PA)-to-RV pres- venous ­return (PVR) arm of the baffle. The RV is dilated and hypertrophied
sure gradient, 32 mmHg in this case. Assuming an RV diastolic pressure of as ­expected. The pulmonic (anatomic left, LV) ventricle fills from the systemic
10 mmHg, estimated PA diastolic pressure is 32 mmHg + 10 mmHg = 42 venous return (SVR) arm of the baffle. The LV is normal in size. TEE provides
mmHg consistent with severe pulmonary hypertension. better visualization of the atrial baffle when clinically indicated.
The Adult with Congenital Heart Disease  CHAPTER 17 363

o Extracardiac connections, such as arterial to v  KEY POINTS


pulmonary shunts, are difficult to assess by o TEE provides better visualization of posterior
transthoracic echocardiography. structures, such as the atrial septum, pulmonary
o Previous surgical procedures may result in veins, and interatrial baffle repairs (Fig. 17-15).
shadowing or reverberations due to prosthetic o Three-dimensional (3D) imaging allows bet-
valves, conduits, or patch material. ter visualization of the shape of the defect
o Evaluation of branch pulmonary artery and accurate measurement of defect size
stenosis usually requires other diagnostic (Fig. 17-16).
approaches. o TEE provides improved images when prosthetic
o Quantitation of RV volumes and systolic func- material shadows posterior structures from the
tion is problematic with standard echocardio- transthoracic approach.
graphic approaches. o Anterior and extracardiac structures, such as
o These areas of uncertainty or issues that can- systemic venous return conduits, branch pul-
not be addressed by echocardiography are monary stenosis, and arterial to pulmonic
identified at the end of the study with sug- shunts, may be difficult to visualize on TEE
gestions for appropriate additional diagnostic (Fig. 17-17).
approaches. o Be sure all individuals involved in the TEE
study understand the study objectives.

Basic Transesophageal Approach


Step 1: Assess the Risk of TEE and
Institute Appropriate Modifications
in the Study Protocol
n  The risk of conscious sedation is higher in some
patients with congenital heart disease.
n Additional monitoring and sedation by an anesthe- LA
siologist may be needed in some cases.
v  KEY POINTS
o Risk of sedation is highest in patients with cya-
nosis, severe pulmonary hypertension, or Eisen-
menger physiology.
o Concurrent pulmonary or other medical condi-
RA
tions also may be present that increase proce-
dural risk.
o Baseline oxygen saturation is assessed before
beginning the procedure because patients may
have significant chronic desaturation due to an
intracardiac shunt.
o When risk is high or uncertain, a cardiac anes-
thesiologist should be asked to assist with the
procedure. ASD
o All health care providers involved in the
study (e.g., physician, nurse, and sonographer)
should understand and review any potential
risks.
Ao
Step 2: Determine the Objectives of
the TEE Study
n In consultation with the referring physician, the
clinical data and transthoracic echocardiogram
are reviewed to determine the specific areas of
interest on the transesophageal echocardiography
(TEE) study.
Figure 17-15  TEE imaging of atrial septal defect.  Top, 2D TEE shows
n 
A complete TEE study is performed whenever a secundum atrial septal defect (ASD) with left-to-right flow seen on color
possible, putting priority on key elements if study Doppler. Bottom, A 3D image better shows the shape and size of the defect.
length is constrained. Ao, Aorta.
364 CHAPTER 17  The Adult with Congenital Heart Disease

ASD

RA Ao

3 planes

Figure 17-16  3D measurement of atrial septal defect (ASD) size.  From a 3D volumetric data acquisition in the same patient as in Fig. 17-15, the im-
age planes are adjusted to provide a planar view of the ASD (arrow), which allows accurate measurement of the area of the ASD and diameter for planning a
transcatheter closure. Ao, Aorta.

IVC
Conduit

PA

Conduit

A B
Figure 17-17  Tricuspid atresia with Fontan conduit.  The conduit from the inferior vena cava (IVC) to the pulmonary artery (PA) is seen on TEE in a vertical
image plane by turning the probe toward the patient’s right side. The junction of the IVC and the conduit (A) is seen and then the probe is slowly withdrawn in
the esophagus, keeping the conduit centered in the image plane (B) to show the flow from the conduit into the pulmonary artery.
The Adult with Congenital Heart Disease  CHAPTER 17 365

n  oth the anatomic size of an ASD (measured


B
Step 3: TEE Imaging Sequence directly) and the physiologic effects (based on the
n  The standard imaging sequence, described in amount of flow across the defect) are useful mea-
Chapter 3, is appropriate for adults with congeni- sures of disease severity.
tal heart disease.
n The examiner should ensure that all cardiac struc-
v  KEY POINTS
tures are evaluated by imaging and Doppler, pref- o A secundum or primum ASD may be visual-
erably in at least two orthogonal views. ized on transthoracic imaging in parasternal
short axis, apical 4-chamber, and subcostal
v  KEY POINTS 4-­chamber views (see Fig. 17-9).
Start with the standard TEE 4-chamber,
o o Color Doppler evidence of transatrial flow
2-chamber, long-axis rotational series of images avoids mistaking echo dropout from an ASD,
to provide an overview of cardiac anatomy. but care is needed to distinguish normal supe-
o Follow a checklist to ensure all structures are rior and inferior vena caval inflow from flow
evaluated: across the atrial septum.
o Systemic and pulmonic ventricles (including ana- o A primum ASD may be accompanied by a cleft
tomic identity, location, great vessel connections, anterior mitral leaflet (Fig. 17-18).
and systolic function) o An endocardial cushion defect is the association
o Aorta and pulmonary artery (including size, loca- of a primum ASD with an adjacent ventricular
tion, and connections to the ventricles) septal defect, often with associated abnormali-
o Aortic and pulmonic valves (including anatomy ties of the atrioventricular valves.
and Doppler flows) o A sinus venosus ASD may be difficult to
o Systemic and pulmonic atrioventricular valves visualize on transthoracic imaging and often
(anatomy and function) is suspected based on unexplained right-
o Left and right atria (or interatrial baffle, Fontan sided enlargement; on TEE imaging in the
conduit, etc.) bicaval view with the probe rotated slightly
o Atrial and ventricular septums
o Location and flow patterns in all four pulmonary
veins
o Superior and inferior vena cava
o Coronary sinus
o Descending aorta
Transgastric views may provide alternate views of
o
the ventricles, atrioventricular valves, and aortic
and pulmonic valves.
o Before completing the study, ask the nurse and
sonographer if any imaging views or Doppler
flows have been missed and if they have any
other suggestions.

SPECIAL CONSIDERATIONS IN COMMONLY


SEEN CONDITIONS
Atrial Septal Defect
n RV volume overload due to an atrial septal defect
(ASD) results in the characteristic findings of RV
enlargement and paradoxical septal motion.
n ASD are classified as:

Secundum (center of atrial septum)
• Primum (adjacent to the atrioventricular valves)
• Sinus venosus (near junction of superior or inferior
vena cava)
n  nomalous pulmonary venous drainage into the
A Figure 17-18  Cleft anterior mitral valve. In a parasternal short-axis
RA or cavae also results in right-sided volume ­image, the differing maximal excursion of the medial and lateral aspects of
overload. the cleft leaflet (arrows) are seen.
366 CHAPTER 17  The Adult with Congenital Heart Disease

rightward, the defect is seen at the junction of


the superior vena cava with the RA. Anoma-
lous pulmonary venous return may also be
seen.
o The presence of RV dilation mandates a care-
ful search for an ASD or anomalous pulmo-
nary venous return using 2D imaging, color
Doppler, and an intravenous saline contrast
study. Ao
o When visualized, the diameter of the ASD
LV
should be directly measured from two images.
o Shunt flow, defined as the ratio of pulmo-
nary blood flow (Qp) to systemic blood flow
(Qs), is determined by calculating stroke
volume in the pulmonary artery and aorta,
with a ratio of more than 1.5 : 1 considered LA
significant.
o Transesophageal imaging is more sensitive than
transthoracic imaging for detection of a sinus
venosus ASD or anomalous pulmonary venous
return.
Ventricular Septal Defect Figure 17-19  Ventricular septal defect imaging.  In a patient referred for
a systolic murmur, the region of the membranous septum (arrow) a­ ppears
n Ventricular septal defects (VSDs) are classified as: abnormal in the standard parasternal long-axis view. Ao, Aorta.

Membranous (from just beneath the aortic valve to
under the septal tricuspid leaflet)
• Supracristal (from just beneath the aortic valve to
under the pulmonic valve)
• Inlet (between the mitral and tricuspid valves)
• Muscular (anywhere in the muscular part of the ven-
tricular septum)
n Large uncorrected VSDs result in severe pulmo- RVOT
nary hypertension early in life with equalization of
pulmonary and systolic pressures and bidirectional
shunting (Eisenmenger physiology).
n Small VSDs are associated with a high-pressure
difference (and high velocity) between the LV and Ao
LV
RV in systole.
v  KEY POINTS
o The anatomic site of a VSD is detected using LA
the combination of 2D imaging (fig 17-19) and
color Doppler to demonstrate systolic turbu-
lence on the right side of the ventricular septum
(Fig. 17-20).
o The VSD is confirmed using CW Doppler to
demonstrate the high-velocity systolic ejection
type Doppler curve (Fig. 17-21). Figure 17-20  Ventricular septal defect (VSD) color Doppler flow.  In the
o The peak velocity of the VSD jet is related to same patient as in Fig. 17-19, color Doppler confirms left-to-right flow consis-
the left to RV systolic pressure difference as tent with a membranous VSD. Ao, Aorta; RVOT, right ventricular outflow tract.
stated in the Bernoulli equation: ΔP = 4V2
o A low-velocity diastolic signal also may be seen in the mitral and tricuspid regurgitant jets are
corresponding to the diastolic pressure differ- equal (Fig. 17-22).
ence between the LV and RV.
o With Eisenmenger physiology, a large VSD is
Patent Ductus Arteriosus
seen on 2D imaging with bidirectional flow on n 
Most patent ductus arteriosus (PDAs) are diag-
color and spectral Doppler. Right and LV size nosed and treated early in life, with only rare cases
and wall thickness are similar and the velocities diagnosed in adults.
The Adult with Congenital Heart Disease  CHAPTER 17 367

n  ontinuous systolic and diastolic flow into the


C
main pulmonary artery is the key findings in adults
with a PDA.
6.0
v  KEY POINTS
Color Doppler of the main pulmonary artery
o
shows the diastolic flow from the PDA origi-
nating near the pulmonary artery bifurcation
(Fig. 17-23).
o Pulsed Doppler evidence of continuous flow in
m/s the pulmonary artery is diagnostic; this flow dif-
fers from pulmonic regurgitation by being distal
to the pulmonic valve, and the diastolic compo-
Figure 17-21  Ventricular septal defect CW Doppler.  In the same patient nents extend into systole (Fig. 17-24).
as in Fig. 17-20, the CW Doppler recording shows a very high velocity signal
(arrow) consistent with a small defect and a large systolic pressure gradient
o Diastolic flow reversal also may be seen in the
between the LV and RV in systole. There also is low-velocity left-to-right descending thoracic aorta and should not be
flow in diastole. mistaken for aortic regurgitation.

RV RV

RA LV
LV

LA

A B
PW:2MHz HPRF

3.0

m/s

2.0
C
Figure 17-22  Eisenmenger ventricular septal defect (VSD).  A, With a large VSD, as seen in this low parasternal 4-chamber view in a patient with trisomy
21, Eisenmenger physiology is present with equalization of RV and LV pressures, significant systemic oxygen desaturation, and cyanosis. Color flow shows both
right-to-left and left-to-right flow (B), and spectral Doppler shows relatively low-velocity bidirectional flow (C).
368 CHAPTER 17  The Adult with Congenital Heart Disease

CW:5MHz

m/s

PA

Ao

4.0

Figure 17-25  Aortic coarctation. CW Doppler flow in the descending


DA aorta shows an antegrade velocity greater than 4 m/s consistent with a
pressure gradient of at least 64 mmHg (possibly higher if the intercept angle
is not parallel to blood flow). Persistent antegrade flow in diastole confirms
severe obstruction.

v  KEY POINTS

oDoppler may underestimate the severity of
coarctation due to nonparallel intercept angle
between the eccentric jet and ultrasound
beam.
Figure 17-23  Patent ductus arteriosus.  In a parasternal short-axis im-
age of the pulmonary artery (PA), a color jet of diastolic flow is seen entering
o When the proximal velocity is also increased,
the PA near its bifurcation, with the flow originating from the descending the proximal velocity should be included in the
aorta (DA) just posterior to the PA. Ao, Aorta. pressure gradient calculation:

CW:2MHz
ΔP = 4(Vmax 2 − Vprox 2 )

5.0
Imaging of the coarctation is rarely possible by
o
transthoracic imaging in adults; TEE may be
helpful in selected cases.
o Further evaluation of aortic coarctation with
computed tomographic imaging or cardiac
catheterization typically is needed.
Ebstein Anomaly
m/s n  Ebstein anomaly is characterized by apical dis-
placement of one or more tricuspid valve leaflets
1.0 (Fig. 17-26 and see Fig. 17-6).
n Imaging the tricuspid annulus and leaflets in para-
Figure 17-24  CW Doppler of patent ductus arteriosus.  There is contin-
uous flow from the aorta into the pulmonary artery with the shape and veloc- sternal RV inflow and apical 4-chamber views usu-
ity reflecting the pressure difference between the two vessels. For example, ally is diagnostic.
the increased systolic velocity corresponds to the increase in systemic blood n The segment of the RV between the annulus and
pressure during systole. displaced leaflet is “atrialized”; that is, ventricular
myocardium is physiologically part of the atrial
chamber.
Aortic Coarctation
n  An aortic coarctation results in an increased v  KEY POINTS
systolic velocity and persistent diastolic ante- o Ebstein anomaly may be isolated or associated
grade flow in the descending thoracic aorta (Fig. with an ASD and is common (about 15% to
17-25). 40%) in patients with congenitally corrected
n Descending aortic flow is recorded with CW Dop- transposition of the great vessels (see later
pler from the suprasternal notch view. section).
The Adult with Congenital Heart Disease  CHAPTER 17 369

RV

LV RV

Ao

LA
RA LV

Figure 17-26  Ebstein anomaly.  Severe apical displacement of the septal


tricuspid valve leaflet (arrow) is seen in an apical 4-chamber view.
Figure 17-27  Tetralogy of Fallot.  Parasternal long-axis view in a typical
patient with repaired tetralogy of Fallot. The RV outflow tract is enlarged; the
basal septum is intact but has increased echogenicity (arrow) suggestive of
Ebstein anomaly is often associated with ven-
o ventricular septal defect patch repair; and the aorta (Ao) is mildly enlarged
tricular preexcitation due to an accessory and slightly overrides the septum.
atrioventricular pathway (e.g., Wolf-Parkinson-
White syndrome).
o In the apical 4-chamber view, a distance greater than one site, including branch pulmonary
than 10 mm between the mitral and tricus- artery stenoses.
pid leaflet insertions is diagnostic of Ebstein o Occasionally an adult patient with an untreated
anomaly. tetralogy of Fallot will be diagnosed by echocar-
o Ebstein anomaly typically results in moderate diography because the RV outflow obstruction
or severe tricuspid regurgitation. prevents pulmonary hypertension.
o Severe pulmonic regurgitation typically is low
Complex Congenital Heart Disease velocity with a to-and-fro flow pattern by pulsed,
CW, and color Doppler that may be overlooked
Tetralogy of Fallot due to the absence of turbulence (Fig. 17-28).
n Tetralogy of Fallot is characterized by: o Additional evaluation with cardiac magnetic

A membranous (anteriorly misaligned) VSD resonance imaging often is needed for quantita-
• An aorta that straddles the ventricular septum and tion of RV size and function.
• RV outflow obstruction resulting in
• RV hypertrophy Congenitally Corrected Transposition
n Most adults with tetralogy of Fallot have under- of the Great Arteries
gone previous surgical repair with VSD closure n 
The pathway of blood flow with transposition of
and relief of RV outflow obstruction (Fig. 17-27). the great arteries (L-TGA) is:
n The most common long-term issue after tetralogy •
Systemic venous return to the RA, then into the LV
of Fallot repair is severe pulmonic regurgitation and out the pulmonary artery.
with progressive RV enlargement and eventual • Pulmonary venous return to the LA, then into the RV
dysfunction. and out the aorta.
n Patients with tetralogy of Fallot usually have mild n  ome patients with L-TGA remain undiagnosed
S
dilation of the aortic sinuses. until adulthood because the flow of oxygenated and
unoxygenated blood is physiologic, even though the
v  KEY POINTS anatomic RV serves as the systemic ventricle.
o 
RV outflow obstruction may be subvalvular, n Defects commonly associated with L-TGA include
valvular, or supravalvular or may occur at more a VSD, pulmonic stenosis, and Ebstein anomaly of
370 CHAPTER 17  The Adult with Congenital Heart Disease

the systemic (tricuspid) atrioventricular valve, and ventricles (and the associated atrioventricular
complete heart block. valves).
o 
Typically, the aortic annulus is anterior and to
v  KEY POINTS the left (the L in L-TGA) of the pulmonic valve.
o 
L-TGA also is called “ventricular inversion” o 
L-TGA is evident on echocardiography based
because the pattern of blood flow is nor- on the systemic ventricle having the anatomic
mal other than the reversed positions of the features of an RV (moderator band, apical
annulus, and tricuspid valve) (Fig. 17-29).
o 
The atrioventricular valves are associated with
CW:1.75MHz each ventricle so the systemic atrioventricular
valve is the tricuspid valve and the pulmonary
atrioventricular valve is the mitral valve.
o 
Long-term systolic dysfunction of the systemic
ventricle may complicate L-TGA.
2.0
o 
Dextroversion (apex pointed toward the right)
or mesocardia often is present with L-TGA,
which limits acoustic access due to the retroster-
nal cardiac position.
m/s
Complete Transposition of the Great Arteries
n Complete transposition of the great arteries
(D-TGA) requires intervention at birth to provide
2.0 mixing between the separated pulmonic and sys-
temic blood flow circuits.
n 
D-TGA then is treated by redirection of blood
flow in childhood with:

An interatrial baffle repair that redirects systemic and
Figure 17-28  Severe pulmonic regurgitation. CW Doppler signal of pulmonary venous inflow to restore a normal pattern
severe pulmonic regurgitation in a patient with repaired tetralogy of Fallot of circulation, but with the anatomic RV serving as
and persistent pulmonic regurgitation after surgical pulmonic valvotomy. The
retrograde flow in diastole has a signal density similar to antegrade flow, the systemic ventricle (Mustard or Senning repair) or
consistent with similar volume flow rates. The diastolic deceleration slope is • More recently, an arterial switch procedure with the
steep and reaches the baseline before the end of diastole (arrow), consistent aorta and pulmonary artery transected and recon-
with equalization of diastolic pulmonary artery and RV pressures. nected to the correct ventricles.

PLAX SAX

Ao Ao
RV
PA

LA

A B
Figure 17-29  Congenitally corrected transposition of the great arteries ventricular inversion.  A, The parasternal long-axis view shows the muscular
ridge (arrow) between the systemic atrioventricular valve and aorta. This muscular band in the RV outflow tract identifies the anatomic RV (and tricuspid valve),
which receives blood from the pulmonary veins and LA and ejects into the aorta. B, In the short-axis view, the position of the aorta (Ao) anterior to the pulmonary
artery (PA) and the side-by-side relationship of the great arteries is evident. PLAX, Parasternal long axis; SAX, short axis.
The Adult with Congenital Heart Disease  CHAPTER 17 371

n  he aorta is anterior and the great vessels are par-


T v  KEY POINTS
allel to each other when D-TGA is present (see Fig. o With an interatrial baffle repair, the circula-
17-3 and Fig. 17-4). tory pattern of oxygenated and unoxygenated
n With an interatrial baffle repair, a major long-term blood is normal but the systemic ventricle is the
issue is systolic dysfunction of the systemic (ana- anatomic RV and the pulmonic ventricle is the
tomic right) ventricle. anatomic LV (Fig. 17-31).
n With an arterial switch repair, a few patients
 o The atrioventricular valves are associated with
develop systemic semilunar valve regurgitation, each ventricle so the systemic atrioventricular
particularly of the neo-aortic valve with dilation of valve is the tricuspid valve and the pulmonic
the proximal “aortic” root (Fig. 17-30). atrioventricular valve is the mitral valve.
o Atrial baffle leaks and stenosis are difficult to
evaluate by transthoracic imaging, typically
requiring TEE or other imaging approaches.
o A lower Nyquist limit (i.e., signal aliasing at a
lower velocity) and use of variance mode on the
color display enhance detection of baffle leaks.
o With an arterial switch procedure, the neoaorta—
the systemic semilunar valve and sinuses—was the
Ao anatomic pulmonic valve. The coronary arteries
also were transposed to the neoaorta.
o Branch pulmonic stenosis can occur after an arte-

LV rial switch repair related to moving the pulmonary


artery anteriorly during the repair procedure.
Fontan Physiology with Tricuspid Atresia
LA
n 
Fontan physiology refers to a direct valveless
surgical connection from the systemic venous

RV

Ao

LV
LV

LA

B
Figure 17-30  Transposition the great arteries (D-TGA).  This 19-year- Figure 17-31  Transposition the great arteries (D-TGA).  Short-axis view
old man with complete TGA underwent a great vessel switch repair as an of the ventricles in a patient with TGA and an interatrial baffle repair. The RV
infant. The ventricular to great vessel relationships now are relatively nor- is located anteriorly but serves as the systemic ventricle and is appropriately
mal as seen in this long-axis view. However, the sinuses of the transposed dilated and hypertrophied. The smaller, posteriorly located LV is the low-
pulmonic valve (neo-aorta) are dilated to 4.6 cm (A) and central “aortic” pressure pulmonic ventricle with systolic curvature of the septum reflecting
regurgitation is present (B), with a vena contracta width of 0.5 cm. Ao, Aorta. the physiologic functions of each ventricle.
372 CHAPTER 17  The Adult with Congenital Heart Disease

return to the pulmonary artery, without an inter- o More recent repairs include a direct connection
vening RV. of the superior vena cava to the right pulmo-
n A Fontan repair is used for patients with only a nary artery with the inferior vena cava con-
single functional ventricle, including those with tri- nected to the pulmonary artery by a conduit.
cuspid atresia (Fig. 17-32). This repair leaves the small residual RA (with
n F low in a Fontan conduit is driven by the pressure the coronary sinus in communication with the
gradient from the systemic venous return to the LA via an ASD.
pulmonary artery, with a flow pattern similar to a Transthoracic and transesophageal echocar-
o
normal systemic venous inflow (Fig. 17-33). diographic evaluation of a Fontan conduit is
challenging and depends on the exact surgi-
v  KEY POINTS cal repair and location of the conduit (see
o 
There are many variations of the Fontan Fig. 17-17).
procedure: o Valved conduits connecting the RV to the pul-
o 
Early Fontan repairs connected the RA to the monary artery are used for other types of com-
pulmonary artery. These patients often have a plex congenital heart disease. These patients
severely enlarged RA with significant arrhythmias do not have Fontan physiology because the
and may have obstruction of the pulmonary veins RV provides pulsatile systolic pulmonary blood
posterior to the dilated atrium. flow.

.79 45dB 3 •/+1/0/ 1


PW Depth= 21mm
PW Gate= 3.0mm
PW Gain= 4dB

.79

PW:3.5MHz

.60

m/s

.40

Figure 17-32  Single ventricle physiology.  Apical view in a patient with tri- Figure 17-33  Fontan conduit flow.  The flow pattern is similar to systemic
cuspid and pulmonic atresia shows only one ventricle and atrioventricular valve. venous flow, with low-velocity forward flow in systole and diastole because
The tricuspid valve is absent, with the small residual RV chamber (not visible in pulmonary blood flow is driven by systemic venous pressure in the absence
this view) communicating with the LV via a ventricular septal defect. The right of the pumping action of a RV.
and left atria are connected by a large atrial septal defect. Pulmonary blood flow
in this patient is provided by a right subclavian artery to pulmonary artery shunt.
The Adult with Congenital Heart Disease  CHAPTER 17 373

THE ECHO EXAM

Adult Congenital Heart Disease


Categories of Congenital Heart Disease Approach to the Echocardiographic
Congenital Stenotic Lesions Examination in Adults with Congenital
Subvalvular Heart Disease
Valvular Before the Examination
Supravalvular Review the clinical history
Peripheral great vessels (aortic coarctation) Obtain details of any prior surgical procedures
Congenital Regurgitant Lesions Review results of prior diagnostic tests
Formulate specific questions
Myomatous valve disease
Ebstein anomaly Sequence of Examination
Abnormal Intracardiac Communications Identify cardiac chambers, great vessels, and their con-
nections
Atrial septal defect
Identify associated defects, and evaluate the physiology
Ventricular septal defect
of each lesion
Patent ductus arteriosus
Regurgitation and/or stenosis (quantitate as per Chapter
Abnormal Chamber and Great Vessel ­Connections 11 and Chapter 12)
Transposition of the great arteries Shunts (calculate Qp:Qs)
Ventricular inversion (congenitally corrected transposition) Pulmonary hypertension (calculate pulmonary pressure)
Tetralogy of Fallot Ventricular dysfunction (measure ejection fraction if
Tricuspid atresia anatomy allows)
Truncus arteriosus After the Examination
Integrate echo and Doppler findings with clinical data
Summarize findings
Identify which clinical questions remain unanswered,
suggest appropriate subsequent diagnostic tests

Clues to the Identification of Cardiac Structures in Adults with Congenital Heart Disease
Structure Anatomic Feature Echo Approach
RA • Inferior vena cava enters right atrium • Start with subcostal approach to identify
RA
RV • Prominent trabeculation • Apical 4-chamber view to compare annular
• Moderator band insertions of 2 ventricles, parasternal for
• Infundibulum valve anatomy and infundibulum
• Tricuspid valve
• Apical location of annulus
Pulmonary artery • Bifurcates • Parasternal long-axis view or apical
4-chamber view angulated very anteriorly
LA • Pulmonary veins usually enter left atrium • TEE imaging for pulmonary vein anatomy
LV • Mitral valve • Apical 4-chamber view and parasternal
• Basal location of annulus long- and short-axis views.
• Fibrous continuity between anterior mitral
leaflet and semilunar valve
Aorta • Gives rise to aortic arch and arterial • Start with parasternal long-axis view and
branches. move transducer superiorly to follow vessel
to its branches.
374 CHAPTER 17  The Adult with Congenital Heart Disease

Examples 2. A 26-year-old woman undergoes echocardiogra-


1. A 24-year-old with a history of a cardiac murmur phy for symptoms of decreased exercise tolerance.
has an echocardiogram which shows: She is found to have an enlarged right atrium and
ventricle with paradoxic septal motion and the fol-
Right ventricular 1.6 m/s lowing Doppler data:
outflow velocity
RV outflow velocity 1.8 m/s
Pulmonary artery 3.1 m/s
velocity Velocity-time integral (VTIRVOT) 32 cm
Tricuspid regurgitant jet 3.4 m/s Diameter 2.6 cm
Estimated right atrial 5 mmHg (small LV outflow velocity 1.1 m/s
pressure IVC with normal Velocity-time integral (VTILVOT) 16 cm
respiratory
variation) Diameter 2.4 cm

Because the RV outflow velocity is elevated, the maxi- The right heart enlargement suggests an ASD may be
mum pulmonic valve gradient should be calculated present. The shunt ratio is calculated from the ratio
using the proximal velocity in the Bernoulli equation: of pulmonary flow (Qp), measured in the RV outflow
( ) tract, and systemic flow (Qs), measured in the LV out-
ΔP = 4 V2jet − V2prox flow tract. At each site, cross-sectional area is calcu-
lated as the area of a circle:

[ ] CSARVOT = π (D/2)2 = 3.14(2.6/2)2 = 5.3 cm2


ΔP = 4 (3.1)2 − (1.6)2 = 4 [9.6 − 2.6]
= 28 mmHg
CSALVOT = π (D/2)2 = 3.14(2.4/2)2 = 4.5 cm2
If the proximal velocity is not included, the gradient
would be overestimated at 38 mmHg. Flow (stroke volume) at each site then is calculated:
Estimated pulmonary systolic pressure is calcu-
lated by subtracting the pulmonic valve gradient from Q p = CSARVOT × VTIRVOT
the estimated RV pressure because pulmonic stenosis = 5.3 cm2 × 32 cm = 170 cm3
is present:
Q s = CSALVOT × VTILVOT
PAP = (ΔPRV − RA + PRA ) − ΔPRV − PA
= 4.5 cm2 × 16 cm = 72 cm3
( )
PAP = [4V 2 TR + PRA] − ΔPRV − PA
so that Q  p/Q  s = 170/72 = 2.4
= 4 (3.4)2 + 5 − 28 = 23 mmHg These calculations are consistent with a significant
shunt that most likely will require closure to prevent
Thus, PA systolic pressure is normal even though the progressive right heart dysfunction.
tricuspid regurgitant jet indicates a RV systolic pres-
sure of 51 mmHg.
The Adult with Congenital Heart Disease  CHAPTER 17 375

SELF-ASSESSMENT QUESTIONS

Question 1 Question 3
A 23-year-old asymptomatic woman is referred for Identify the two cardiac chambers and two valves
echocardiography to evaluate a murmur (Fig. 17-34). numbered in Fig. 17-35:
The most likely diagnosis is: 1 . ________________________________________
A. Aortic coarctation 2 . ________________________________________
B. Bicuspid aortic valve 3 . ________________________________________
C. Ventricular septal defect 4 . ________________________________________
D. Ebstein anomaly
E. Pulmonic stenosis

2 4

Figure 17-35 

Question 4
A 23-year-old woman with a history of surgery for
Figure 17-34  repair of aortic coarctation in childhood is referred
for echocardiography. Which of the following is most
likely to be found in this patient?
Question 2 A. Cleft mitral valve
Regarding the patient in Question 1, the maximum B. Bicuspid aortic valve
velocity in the CW Doppler signal is expected to be C. Ventricular septal defect
closest to which of the following? D. Ebstein anomaly
A. 1 m/s E. Pulmonic stenosis
B. 2 m/s
C. 3 m/s
D. 4 m/s
E. 5 m/s
376 CHAPTER 17  The Adult with Congenital Heart Disease

Question 5 Question 6
A 27-year-old woman undergoes echocardiography A patient with an uncorrected tetralogy of Fallot pres-
for a murmur during pregnancy and is found to have ents for echocardiographic evaluation. His blood pres-
an ASD. The following measurements are made: sure is 115/75 mmHg pulse is 76 bpm, and estimated
Pulmonary Artery RA pressure is 8 mmHg. The following maximum
systolic velocities are recorded with CW Doppler:
 Velocity 1.8 m/s
 Velocity-time integral (VTIRVOT) 36 cm Aortic valve 1.3 m/s
 Diameter 2.6 cm Mitral valve 5.0 m/s
LV Outflow Pulmonic valve 3.6 m/s
 Velocity 1.1 m/s Tricuspid valve 4.2 m/s
 Velocity-time integral (VTILVOT) 25 cm Calculate:
  

 Diameter 2.2 cm LV systolic pressure ___________________mmHg


Mitral Valve RV systolic pressure ___________________mmHg
Pulmonary artery (PA) systolic pressure ____mmHg
 Velocity-time integral (VTIMV) 11 cm Velocity in the ventricular septal defect ______m/s
 Annulus diameter 3.3 cm
Calculate the pulmonic to systemic shunt ratio:
__________________________________________

Question 7
A 28-year-old patient presents to the adult congenital A.
Aortic coarctation
heart disease clinic with a history of heart surgery as B. Congenial aortic stenosis
an infant. Based on the image in Fig. 17-36, the most C. Tetralogy of Fallot
likely diagnosis is: D. Transposition of the great arteries
E. Ventricular septal defect

2.0

m/s

3.0
Figure 17-36 
The Adult with Congenital Heart Disease  CHAPTER 17 377

Question 8
The most likely diagnosis in the patient with the imag-
ing and Doppler data shown in Fig. 17-37 is:
A. Anomalous left coronary artery from the pul- C.
Coronary artery fistula
monary artery (ALCAPA) D. Patent ductus arteriosus
B. Aortic regurgitation E. Ventricular septal defect

4.0

m/s

Figure 17-37 

Question 9
The most likely diagnosis in the patient with the color A.
Aortic coarctation
and CW Doppler data shown in Fig. 17-38 is: B. Aortic regurgitation
C. Atrial septal defect
D. Patent ductus arteriosus
E. Ventricular septal defect

Figure 17-38 
378 CHAPTER 17  The Adult with Congenital Heart Disease

Question 10
Echocardiography is requested in a 24-year-old A.
Severe tricuspid regurgitation
woman with complex congenital heart disease with B. Anomalous pulmonary venous return
palliative surgery in childhood. Her aortic and mitral C. Aortopulmonary window
valves show normal function with no significant regur- D. Aortic coarctation
gitation, but this Doppler flow tracing is recorded (Fig. E. Branch pulmonary stenosis
17-39). This finding is most likely due to:

2.0

m/s

1.0

Figure 17-39 

Question 11
This echocardiographic image (Fig. 17-40) was obtained A.
Atrial septal defect
in a patient with congenital heart disease with a prior B. Cor triatriatum
surgical repair. C. Ebstein anomaly
The most likely diagnosis in this patient is: D. Tetralogy of Fallot
E. Transposition of the great arteries

Figure 17-40 
The Adult with Congenital Heart Disease  CHAPTER 17 379

Questions 12 to 15
Match each of the echocardiographic images in D.
Patent ductus arteriosus
Questions 12 to 15 with the most likely diagnosis from E. Persistent left superior vena cava
the following list: F. Tetralogy of Fallot
A. Atrial septal defect G. Transposition of the great arteries
B. Congenital aortic stenosis H. Ventricular septal defect
C. Ebstein anomaly

Question 12 Question 14

Figure 17-41 
Figure 17-43 

Question 13 Question 15

Figure 17-42  Figure 17-44 


380
380 CHAPTER 17  The Adult with Congenital Heart Disease

ANSWERS

Answer 1: C annulus usually is slightly more apical than the mitral


In this parasternal long-axis view, turbulent flow is valve annulus (as seen on this image), so this is a reli-
seen in systole (mitral valve closed, aortic valve open) able feature for identifying the RV. Occasionally, the
in the RV outflow tract. This is most likely due to a tricuspid and mitral valve annulus are at the same
membranous ventricular septal defect. Aortic coarcta- level so that other features must be used for anatomic
tion would result in high-velocity flow and turbulence identification of the ventricular chambers. The pres-
in the descending thoracic aorta, which is not seen ence of a moderator band is reliable for identifica-
on this image. A bicuspid valve would be associated tion of the RV, but the degree of trabeculation is not
with an asymmetric aortic valve, systolic doming of reliable. The normal triangular shape, smaller size,
the leaflets, and (possibly) an increased flow velocity and thinner walls of the RV are all altered when the
in the ascending aorta. Ebstein anomaly is associated anatomic RV serves as the systemic ventricle. With
with tricuspid regurgitation with a flow jet in the RA. systemic pressures, the RV appropriately enlarges,
Pulmonic stenosis would result in turbulent flow distal hypertrophies, and changes shape, similar to an
to the pulmonic valve, not in the RV outflow tract. anatomic LV. The location of the RV and LV in the
chest is abnormal in some patients with complex
Answer 2: E congenital heart disease and is not a reliable indica-
This patient has an asymptomatic small ventricular tor of anatomy.
septal defect. The restrictive size of the defect is simi-
lar to a stenotic valve so that the velocity in the defect Answer 4: B
reflects the pressure difference between the systolic
LV pressure (about 120 mmHg with a normal sys- In patients with an aortic coarctation, about 50%
tolic blood pressure) and RV systolic pressure (about have a bicuspid aortic valve. Conversely, in patients
20 mmHg in the absence of pulmonary hyperten- with a bicuspid aortic valve, about 10% have an
sion). Thus, based on the Bernoulli 4V2 equation, the aortic coarctation, so that the presence of either of
100 mmHg pressure difference between the LV and these conditions mandates a search for the other. A
RV is equivalent to a velocity of 5 m/s. cleft mitral valve is most often associated with a pri-
Pulmonary (and RV) systolic pressure should be mum atrial septal defect or an atrioventricular canal
normal in this patient with only a small volume of flow defect with a ventricular and atrial septal defect.
across the defect. Eisenmenger physiology, or equal- Ventricular septal defects are seen in several con-
ization of systemic and pulmonary pressures, only genital conditions, such as tetralogy of Fallot and
occurs with larger VSDs that results in equalization congenitally corrected transposition of the great
of left and RV systolic pressures due to unrestricted arteries, or as an isolated anomaly, but are not asso-
flow between the ventricles in systole. Eisenmenger ciated with aortic coarctation. Ebstein anomaly and
physiology typically occurs early in life and is associ- pulmonic stenosis often accompany congenitally
ated with significant clinical symptoms and physical corrected transposition of the great vessels. Ebstein
examination findings; echocardiography would not anomaly of the tricuspid valve is associated with
show turbulence in the RV outflow tract because the secundum atrial septal defect, arrhythmias due to
flow velocity across the defect is low in this situation. preexcitation and a bypass tract, and transposition
of the great vessels.
Answer 3
1.
Anatomic left ventricle Answer 5
2. Anatomic right ventricle Qp : Qs 2 : 1
3. Mitral valve
4. Tricuspid valve
The shunt ratio is calculated from the ratio of pulmo-
nary flow (Qp), measured in the RV outflow tract, and
This is an apical 4-chamber view in a patient with
systemic flow (Qs), measured in the LV outflow tract.
congenitally corrected transposition of the great ves-
At each site, cross-sectional area is calculated as the
sels. The transducer is in the normal orientation at
area of a circle:
the apex, with the anatomic RV on the right and
the anatomic LV on the left of the image. The RV
is the systemic ventricle, with the interatrial baffle CSARVOT = π(D/2)2 = 3.14(2.6/2)2 = 5.3 cm2
directing pulmonary venous return to the RV, which
then ejects into the aorta. The atrioventricular valves
go with the corresponding ventricles so that the ana-
CSALVOT = π(D/2)2 = 3.14(2.2/2)2 = 3.8 cm2
tomic RV has a tricuspid valve. The tricuspid valve
The Adult with Congenital Heart Disease  CHAPTER 17 381

Flow at each site then is calculated: The velocity in the VSD reflects the systolic LV to RV
pressure gradient.
Q p = CSARVOT × VTIRVOT = 5.3 cm2 × 36 cm Since
= 191 cm3 or mL LV pressure − RV pressure = 115 − 79 = 36 mmHg

and
Q s = CSALVOT × VTILVOT = 3.8 cm2 × 25 cm ΔPVSD = 4(VVSD )2
= 95 cm3 or mL
then,
so that √ √
VVSD = ΔPVSD /4 = (36/4) = 3.0 m/s
Q P : Q s = 191/95 = 2 : 1
Answer 7: C
These calculations are consistent with a large In the diastolic image recorded from the paraster-
shunt that likely is associated with right-sided heart nal RV outflow view (left), diastolic flow is seen fill-
enlargement and will require further evaluation and ing the RV outflow tract. The CW Doppler signal
consideration of closure after pregnancy. (right) confirms that this flow is severe pulmonic valve
The transmitral volume flow rate should equal regurgitation―the diastolic flow signal is equally
transaortic flow, in the absence of aortic or mitral dense compared to antegrade flow and the decelera-
regurgitation, and might provide an alternate site for tion slope is steep, reaching the baseline before the end
calculation of systemic blood flow. of diastole. The systolic antegrade flow is increased
in velocity (about 2 m/s) due to the forward stroke
QMV = CSAMV × VTIMV = 8.5 cm2 × 11 cm volume across the pulmonic valve. The color Dop-
= 94 cm3 or mL pler signal is towards the transducer but is displayed
In this case, there is only a slight (1 mL) difference as blue flow because the aliasing velocity is only 0.75
between transaortic and transmitral flow rates, which m/s, whereas the diastolic velocity is about 1.6 m/s.
is within measurement error. Transmitral flow is rarely Pulmonic regurgitation is the most common long term
used for shunt ratio calculations because reproducible residual defect in adults with a previous tetralogy of
measurement of mitral annulus diameter is problematic. Fallot repair, which is the correct diagnosis in this case.
Echocardiographic findings with aortic coarctation
Answer 6 are an increased antegrade velocity in the descending
thoracic aorta with persistent forward flow in dias-
LV systolic pressure 115 mmHg tole (sometimes called diastolic run-off) when obstruc-
RV systolic pressure 79 mmHg tion is severe. With congenital aortic stenosis, there
PA systolic pressure 27 mmHg is an increased velocity across the aortic valve, often
with some degree of aortic regurgitation. Compared
VSD velocity 3 m/s to pulmonic regurgitation, the velocities with aortic
regurgitation are much higher reflecting the aortic to
LV systolic pressure is the same as systolic blood pres- LV diastolic pressure difference. Transposition of the
sure (115 mmHg) because there is no significant trans- great arteries is not typically associated with severe
aortic systolic gradient. pulmonic regurgitation and there is a parallel relation-
RV systolic pressure is calculated by adding the ship between the aorta and pulmonary artery with an
RV-to-RA systolic pressure difference to estimated anterior aorta, not the normal great vessel relationship
RA pressure: seen here. A small ventricular septal defect is charac-
terized by high-velocity, left-to-right flow. In contrast,
RV pressure = [ΔPRV − RA + )PRA = 4V2 TR + PRA a large defect is characterized by low-­velocity flow
= 4 (4.2)2 + 8 = 79 mmHg across the defect (nonrestrictive flow).
Pulmonary systolic pressure is calculated by subtract- Answer 8: E
ing the pulmonic valve gradient (from the estimated In the parasternal long-axis view (left) a small color
RV systolic pressure) because pulmonic stenosis is jet is seen with flow crossing the septum from left to
present: right. The location is on the LV side of the aorta valve
and the CW Doppler signal (right) shows high veloc-
ΔPRV − PA = 4V2 PA = 52 mmHg ity systolic flow from left to right, with an ejection type
velocity curve, consistent with a small ventricular septal
PA pressure = RV pressure − ΔPRV − PA defect with normal LV and RV pressures. Low-­velocity,
= RV pressure − 4V2 PA passive, left-to-right flow in diastole also is present con-
= 79 − 52 = 27 mmHg sistent with higher LV than RV diastolic pressures.
382
382 CHAPTER 17  The Adult with Congenital Heart Disease

An anomalous left coronary artery arising from cava or RA junction and would show a typical low-
the pulmonary artery (ALCAPA) is an unusual velocity systolic and diastolic filling pattern. Aortic
anomaly in which the direction of flow in the left cor- coarctation results in continued forward flow in the
onary artery is reversed, with coronary flow supplied aorta in diastole, as well as an increased systolic veloc-
by the normal right coronary then going retrograde ity. Branch pulmonary stenosis results in a high sys-
in the left coronary and emptying into the pulmo- tolic velocity and the pulmonary artery is not seen on
nary artery. These patients usually have significant this subcostal view.
coronary ischemia resulting in heart failure. Echocar-
diography shows an abnormal diastolic flow signal Answer 11: E
in the pulmonary artery. Aortic regurgitation occurs This is an apical 4-chamber view, magnified to focus
in diastole and color flow would be directed into the on the atrium and atrioventricular valves. The 2D
LV. A coronary artery fistula may arise from either images show a linear echo across the atrial region with
coronary artery with drainage into the right heart in color Doppler showing laminar blood flow through
over 90% of cases. Coronary flow is increased with this channel and then across an atrioventricular valve
continuous diastolic and systolic flow when a fistula into a ventricle. These images are consistent with an
is present. A patent ductus arteriosus results in con- interatrial baffle surgical repair (Mustard or Senning)
tinuous diastolic and systolic flow in the pulmonary for D-TGA. The baffle directs systemic venous return
artery. to the pulmonary artery via the anatomic LV and pul-
monary venous return to the aorta via the anatomic
Answer 9: D RV. This corrects blood flow to the pulmonary and
The color Doppler image (left) shows the pulmonary systemic circuits but leaves the anatomic RV as the
artery in a parasternal RV outflow view with a dia- systemic ventricle. Although patients with this surgi-
stolic flow disturbance along the lateral wall of the cal procedure are still seen, the current approach for
pulmonary artery. CW Doppler confirms continu- surgical repair is an arterial switch (Jatene) procedure.
ous high-velocity systolic and diastolic flow consistent There is an apparent defect in the atrial septum
with a patent ductus arteriosus from the descending because of the anatomy of the baffle repair, but there
aorta (high pressure in systolic and diastole) to the low is no intracardiac shunt and no evidence for volume
pressure pulmonary artery. overload of the pulmonic ventricle. Cor triatriatum
Aortic regurgitation would occur only in dias- is a partial membrane across the LA chamber, with
tole with characteristic diastolic flow curve from the normal ventricular and great vessel relationships. In
aorta to LV. An atrial septal defect results in low- Ebstein anomaly, the anatomic tricuspid valve (the
velocity back-and-forth flow between the right and systemic atrioventricular valve in this case) is displaced
left atrium with flow predominantly from right to apically, whereas this image shows the atrioventricular
left as long as pulmonary pressures are not severely valve insertions at the same level. Tetralogy of Fallot
elevated. A ventricular septal defect results in high- includes a ventricular septal defect, an enlarged aorta,
velocity systolic flow from the left to the right ventri- and RV outflow obstruction with RV hypertrophy,
cle, with an ejection type curve similar in shape and none of which are seen on these images.
velocity to mitral regurgitation, but slightly longer in
duration. Answer 12: E
This parasternal long-axis image shows an enlarged
Answer 10: C coronary sinus, due to a persistent left superior vena
This is a Doppler recording of flow from the trans- cava. The enlarged coronary sinus also can be visu-
thoracic subcostal view in the proximal abdominal alized in a posteriorly angulated apical 4-chamber
aorta with normal antegrade systolic flow and abnor- view. This is a variant seen in about 0.5% of normal
mal holodiastolic flow reversal. Retrograde diastolic individuals that results in an enlarged coronary sinus
flow in the aorta may be due to any communication due to venous drainage via the coronary sinus into
from the proximal aorta into a lower-pressure vessel the RA. If needed, the diagnosis can be confirmed
or chamber; classically, this finding is seen with severe by saline contrast injected in a left arm vein, appear-
aortic regurgitation. Other causes of aortic diastolic ing first in the coronary sinus, then in the RA; saline
flow reversal include a systemic arterial to pulmonary contrast injected in a right arm vein will appear in the
arterial shunt such as a Blalock-Taussig shunt. This RA but not the coronary sinus.
patient had pulmonary atresia and single-ventricle
physiology with pulmonary blood flow supplied by a Answer 13: C
surgically created aortic-to-pulmonary window, which Ebstein anomaly is characterized by apical displace-
accounts for the flow pattern seen in the aorta. Severe ment of one or more of the tricuspid valve leaflet inser-
tricuspid regurgitation causes systolic flow reversal in tions, resulting in “atrialization” of the part of the RV
the inferior vena cava or hepatic veins. Anomalous between the leaflet insertion and tricuspid annulus,
pulmonary venous return may enter the inferior vena often resulting in significant tricuspid regurgitation.
The Adult with Congenital Heart Disease  CHAPTER 17 383

The diagnosis is based on relative apical displace- Answer 15: A


ment, relative to the mitral annular plane (>10 mm). This apical 4-chamber view shows a severely enlarged
Other abnormalities associated with Ebstein anomaly RV and RA consistent with chronic volume over-
are preexcitation resulting in arrhythmias and the load of the right heart. If RV enlargement were
presence of an atrial septal defect. Long term, vol- due to pressure overload (for example, with pulmo-
ume overload from tricuspid regurgitation may result nary hypertension), the RV free wall is expected to
in right ventricular dilation and dysfunction. be hypertrophied as well. The differential diagnosis
of chronic right heart volume overload is tricuspid
Answer 14: G or pulmonic valve regurgitation or an atrial septal
In this parasternal long-axis view, two semilunar defect. In this view, the atrial septum appears to show
valves are seen in short-axis simultaneously, consis- a defect; while echo-dropout might occur in this view
tent with the parallel orientation of the great arteries due to a parallel intercept angle between the atrial
when transposition is present. With normally related septum and the ultrasound beam, parasternal views
great arteries, the pulmonic valve is seen in long axis confirmed the presence of a secundum atrial septal
when the aortic valve is seen in short axis. With trans- defect with color Doppler showing left-to-right flow
position, the anterior great vessel is the aorta, which (Fig. 17-46).
can be confirmed by turning to a long-axis view (Fig.
17-45) and showing that this valve connects to the
aortic arch. The pulmonic valve is posterior, which
is confirmed by showing the connection to the pul-
monary artery, which bifurcates into right and left
branches.

Figure 17-46 

Figure 17-45 
Intraoperative and Interventional
18 Echocardiography
STEP-BY-STEP APPROACH Hypertrophic Cardiomyopathy
Basic Principles Advanced Heart Failure Therapies
Review the Preoperative Data Heart Transplantation
Consider Effects of Hemodynamic Changes Congenital Heart Disease
and Surgical Instrumentation TRANSCATHETER AND HYBRID PROCEDURES
Baseline data acquisition Atrial Septal Defect or Patent Foramen Ovale Closure
Postprocedure Data Acquisition Transcatheter Aortic Valve Implantation
Interpret and Communicate Findings Transcatheter Mitral Valve Repair
INTRAOPERATIVE TEE Mitral Balloon Valvotomy
Monitoring Left Ventricular Function Transcatheter Closure of Paravalvular Prosthetic
Mitral Valve Repair Regurgitation
Aortic or Mitral Valve Replacement THE ECHO EXAM
Endocarditis SELF-ASSESSMENT QUESTIONS
Aortic Disease

STEP-BY-STEP APPROACH conditions and to allow time for discussion and


procedural planning.
Basic Principles n All preprocedure images and diagnostic data

n  Transesophageal echocardiography (TEE) is should be reviewed, including coronary angiogra-
increasingly used to guide surgical and percutane- phy, hemodynamics, cardiac magnetic resonance
ous cardiac procedures (Table 18-1). imaging (MRI) and computed tomography (CT)
n TEE performance and interpretation occur simul- imaging, as well as echocardiographic data.
taneously, with immediate communication of
results to the physician performing the surgical or v  KEY POINTS
percutaneous procedure. o Intraoperative TEE provides:
n 
More detailed information on intraoperative TEE is o Confirmation of the diagnosis
provided in the Otto Companion Series book, Intraop- o Additional information on valve reparability
erative Echocardiography: A Volume in Practical Echocardiogra- o A baseline study for comparison to postprocedure
phy Series, edited by Don Oxorn, MD (Elsevier, 2011). imaging
o Monitoring of LV function
v  KEY POINTS o Guidance for optimization of intravascular cath-
o Time constraints may require a focused eter and device placement
examination. Unexpected findings on the baseline procedural
o
o Altered loading conditions may affect evalua- TEE may require a change in the intervention,
tion of valve and ventricular function. consultation with the referring cardiologist, or
o Loading conditions should be matched on base- rescheduling of the procedure.
line and postintervention studies. o In emergency situations, the baseline proce-
o Urgent decision-making based on TEE findings dural TEE may be the primary diagnostic test;
may be necessary. in this setting, a complete study should be per-
o Limitations of the TEE data must be promptly formed when allowed by time constraints.
recognized and communicated.
o Appropriate training and experience are needed
Step 2: Consider Effects of Hemodynamic
for procedural TEE; at many institutions, these Changes and Surgical Instrumentation
TEE studies are performed and interpreted by n Hemodynamic effects of sedation or anesthesia
qualified anesthesiologists. must be considered in the interpretation of proce-
dural TEE imaging and Doppler data.
Step 1: Review the Preoperative Data n 
Baseline and postprocedure data should be
n 
For elective procedures, a complete diagnostic recorded at similar loading conditions, using
evaluation is performed before the planned inter- volume infusion and pharmacologic agents, if
vention to ensure evaluation under normal loading needed, to match hemodynamic parameters.

384
Intraoperative and Interventional Echocardiography  CHAPTER 18 385

TABLE 18-1 Indications for Intraoperative


or Intraprocedural TEE
Monitoring Ventricular Function
Before and after cardiopulmonary bypass in high-risk
patients
During noncardiac surgery in high-risk patients
Cardiac Surgical Procedures
Mitral valve repair
Mechanism of regurgitation
Severity of regurgitation
Functional assessment after mitral valve repair
Complications after mitral valve repair
Prosthetic valve replacement
Evaluation after valve implantation
Detection of complications
Complex surgical valve procedures
Aortic valve resuspension and aortic root repair
Coronary artery reimplantation
Endocarditis
Valve involvement and dysfunction
Assessment after repair or valve replacement
Hypertrophic cardiomyopathy Figure 18-1  Electronic artifact.  TEE in the operating room often is com-
plicated by artifacts, such as the electronic interference pattern due to the
Evaluation before and after myectomy
use of electrocautery, seen on this transgastric short-axis view. Diagnostic
Aortic dissection repair imaging should be performed when this artifact is absent, as it may also
Congenital heart disease—before and after surgical repair affect the color Doppler signal.
Transcatheter Interventions
Transcatheter aortic valve replacement (TAVR)
Transcatheter mitral valve repair procedures
Balloon mitral valvotomy
Transcatheter closure for paravalvular regurgitation LA
Atrial septal defect or patent foramen ovale closure
Septal ablation for hypertrophic cardiomyopathy
Placement of Intracardiac Devices post
Cannula placement
Ventricular assist devices
Aortic cannulation (avoid atheroma)
LV
Pericardial Disease, Including Loculated Effusion
General Surgical Complications
Intracardiac air

From Otto CM: Textbook of Clinical Echocardiography, 5 ed, Elsevier,


2013, Philadelphia.

v  KEY POINTS
o Assessment of hemodynamics and ventricular
function is affected by:
o Positive pressure mechanical ventilation Figure 18-2  Intracardiac air.  When weaning from cardiopulmonary by-
o Volume status pass, TEE can assist in detection of intracardiac air. In this TEE 4-chamber
o Myocardial “stunning” (when aortic cross-clamp- view, isolated bubbles are seen in the LV chamber along with a denser area
ing is necessary) (arrows) due to an air collection in the apical aspect of the septum.
o Effects of cardiopulmonary bypass
o Pharmacologic therapy o 
With cardiac surgery, TEE images will be
Basic hemodynamic parameters (heart rate
o affected by:
and blood pressure) should be indicated on the o Inversion of the left atrium (LA) appendage (looks
recorded echocardiographic images to ensure like an LA mass) during mitral valve surgery
matched loading conditions. o Reverberations and shadowing by intracardiac
o When possible, cardiac output, filling pressures, cannulas
and systemic vascular resistance should also be o Electronic interference (Fig. 18-1)
recorded with the TEE images. o Intracardiac air (Fig. 18-2)
386 CHAPTER 18  Intraoperative and Interventional Echocardiography

FR 27Hz
15cm
xPlane
68%
68% 4-chamber 2-chamber
50dB
P Off
Gen

Figure 18-3  Basic TEE views of the left


ventricle. Whenever possible, the intraop-
erative TEE exam should start with standard
4-chamber, 2-chamber, and long-axis views
recorded at a depth to include the LV apex.
In this example, biplane imaging is used for
simultaneous acquisition of 4-chamber and
2-chamber view. Acquisition of these images
takes only 1 to 2 minutes and allows evalua- JPEG
tion of LV size, LV regional and global systolic PAT T:37.0C 76 bpm
function, and RV size and systolic function. TEE T:38.4C

M4 M4 M4 30.8
0 0

1 1

2
2

LA 3
3
92.4
4 cm/s
4
5
5
6
6
7
7
LV 8
8
9
JPEG 9 61 bpm

Figure 18-4  TEE color Doppler imaging.  In a patient undergoing surgical mitral valve repair, 2D imaging (left) shows a flail posterior mitral leaflet (arrow).
Color Doppler (right) demonstrates an anteriorly directed mitral regurgitant jet with a wide vena contracta and a large proximal isovelocity surface area radius
(color Doppler baseline moved in the direction of flow, toward the trasnducer).

Step 3: Baseline data acquisition v  KEY POINTS


n  A complete systemic TEE (see Chapter 3) is rec- As with any echocardiographic study, intraop-
o
ommended when possible, but time constraints erative and intraprocedural TEE images should
may require a limited study focused on the key be obtained in standard long-axis, short-axis,
diagnostic information. 2-chamber, and 4-chamber image planes (Fig.
n Optimal diagnosis is ensured by use of a protocol 18-3).
with a consistent image sequence, and acquisition o Three-dimensional (3D) imaging is helpful
of images is obtained in standard TEE views. for evaluation of mitral valve and atrial sep-
n Images may be acquired either by: tal defect anatomy. 3D echocardiographic

Obtaining all views from each transducer position
 guidance is essential for some transcatheter
(high esophageal, midesophageal, transgastric), first procedures.
with imaging and then with Doppler, or o Color Doppler is helpful for evaluation of intra-

Evaluation of each anatomic structure by both imag- cardiac flow patterns, particularly for evalua-
ing and Doppler in at least two orthogonal views— tion of mitral regurgitation. 3D color imaging
all four cardiac chambers, all four valves, both great is useful in selected cases (Fig. 18-4).
arteries, systemic and pulmonary venous return, atrial o Pulsed Doppler is useful for evaluation of ante-
appendage, and interatrial septum. grade pulmonary venous and transmitral flow
Intraoperative and Interventional Echocardiography  CHAPTER 18 387

FR 54 Hz Time 208 ms PW M4
13cm Slope 0.000 cm/s2 50%
Time 201 ms 2.9MHz
2D Slope 417 cms2 WF 150Hz
76% P½t 59 ms SV4.0mm
C 50 MV Peak A Vel 8.9cm
P Off Vel 47.9 cm/s
HGen PG 1 mmHg
MV Peak E Vel
Vel 84.1 cm/s
PG 3 mmHg
MV E/A 1.8 cm/s

20

40

60
Figure 18-5  Pulsed Doppler velocity
80
data. LV inflow velocity across the mitral
valve is recorded at baseline with a velocity
curve similar to that seen on transthoracic
100
PAT T:37.0C imaging except that flow is directly away from
75 mm/s 60 bpm
TEE T: 37.9C the transducer.

patterns (Fig. 18-5). The pulsed Doppler sig- of the procedure; obtaining data in similar
nal also may help in identification of anatomic views as the baseline study allows direct com-
structures when otherwise unclear. parison of the images.
o 
Continuous wave (CW) Doppler is helpful in o With surgical procedures, the postprocedure
selected cases but the possibility of a nonparallel TEE is performed after weaning from car-
intercept angle (and underestimation of veloc- diopulmonary bypass and after restoration of
ity) must be considered due to the constraints hemodynamics similar to the baseline study.
on transducer positioning in the esophagus. o With percutaneous interventions, TEE data
o 
Interference and technical artifacts that limit may be used continuously to guide the proce-
image quality may be reduced by reposition- dure with repeat evaluations at each stage of
ing the transducer or pausing electronic devices the intervention.
while images are obtained. o Instrument settings should be the same as on
o 
Standard instrument presets, including trans- the baseline study to avoid differences due to
ducer frequency, depth, gain, preprocessing and technical factors, rather than to the procedure
postprocessing, and sector scan width may be itself.
adequate for some images, but adjustment often
is needed during the examination. Step 5: Interpret and Communicate
o 
If the electrocardiogram signal is inadequate Findings
or subject to interference, cine loops can be n Unlike a conventional diagnostic TEE, intraopera-
acquired using a set time interval, instead of one- tive and procedural TEE studies are interpreted
or two-beat clips triggered by the electrocardio- and reported verbally simultaneously with image
gram QRS complex. acquisition.
o 
The structure of interest should be centered, n In addition to the results themselves, the degree of
with depth and zoom adjusted to optimize the certainty of each finding should be reported.
image.
v  KEY POINTS
Step 4: Postprocedure Data Acquisition TEE findings may prompt a change in surgical
o
n After the procedure, TEE is repeated to assess plans or additional procedures.
the results of the intervention and to evaluate for o If the findings are equivocal or images are low
potential complications. quality, this information should be communi-
n 
When possible, loading conditions on the postop- cated to avoid decision-making based on inad-
erative TEE should be similar to the baseline study. equate data.
o Qualitative data often are adequate for deci-
v  KEY POINTS sion-making; when quantitative approaches are
o The postprocedure TEE focuses on the views used, measurements and calculations that can
and Doppler flows needed to evaluate the effect be performed rapidly are preferred.
388 CHAPTER 18  Intraoperative and Interventional Echocardiography


TEE findings should be documented in the
o
permanent medical record, as well as providing
immediate results during the procedure.
o 
Intraoperative and intraprocedural TEE
images should be saved, as for any echocardio-
graphic study, to allow comparison with future post
studies.

INTRAOPERATIVE TEE
Monitoring Left Ventricular Function
n  Intraoperative TEE is useful for continuously
monitoring left ventricular (LV) function in high-
risk patients undergoing noncardiac surgery and
for evaluation of LV function after cardiac surgical
procedures (see Table 18-1).
n Images of the LV allow monitoring of:

Ventricular preload (LV volume)
• Global LV systolic function Figure 18-6  LV size and function.  The transgastric short-axis view of the
LV allows evaluation of overall LV size (reflecting preload or volume status),
• Regional LV function global ventricular systolic function, and regional dysfunction due to coronary
• Right ventricle (RV) size and systolic function disease.

v  KEY POINTS
Long axis
o In general, the size of the LV chamber reflects 2C
filling volume; monitoring allows optimization
of preload.
o Small LV volumes with adequate filling
pressures are seen with restrictive cardiomy- Inf
Post Cx
opathy, pericardial constraint, severe RV dys- PDA
function, or high contractility states.
o The transgastric short-axis view is often used Lat
4C
for continuous monitoring of LV size and Inf
global and regional function, as it includes Septum
myocardial segments supplied by the three
major coronary arteries (Fig. 18-6 and Fig. Ant
18-7). RV Ant
o Standard two-dimensional (2D) TEE images Septum
of the LV apex are usually suboptimal, as LAD
the LV is typically foreshortened due to the
constraints of transducer positioning in the Transgastric short axis LV
esophagus. Figure 18-7  LV regional function.  The wall segments seen on the trans-
o 3D LV volumes are recommended for quanti- gastric view, with the corresponding coronary artery supply, are shown on
tation of LV volumes and ejection fraction. this schematic drawing. 2C, 2 chamber; 4C, 4 chamber; Ant, anterior; Cx,
o Changes in wall motion usually reflect isch- circumflex coronary artery; Inf, inferior; LAD, left anterior descending coro-
emia; other causes include conduction defects, nary artery; Lat, lateral; PDA, posterior descending coronary artery; Post,
posterior. (From Otto, CM: Textbook of Clinical Echocardiography, ed 5, Else-
hypovolemia, and myocardial stunning after vier, 2013, Philadelphia.)
cardiopulmonary bypass.
o RV dysfunction may be due to ischemia, inad-
equate cardioplegia, or air embolism into the n 
Postrepair TEE allows evaluation of any
right coronary artery when separating from residual mitral regurgitation and detection of
cardiopulmonary bypass. complications.
Mitral Valve Repair v  KEY POINTS
n 
Baseline 2D and 3D TEE allows precise delinea- o Mitral valve anatomy is best evaluated using
tion of mitral valve anatomy and the mechanism 2D rotational scanning and 3D full volume
of regurgitation, which is helpful in planning the imaging from a midesophageal position (Fig.
surgical repair. 18-8).
Intraoperative and Interventional Echocardiography  CHAPTER 18 389

Superior/anterior o 
Proximal isovelocity surface area calculation of
regurgitant orifice area (if needed).
o 
Postrepair, anatomic, and functional results
LAA are evaluated using the same imaging views
P1
and Doppler measures as on the baseline study
Medial Ao A1
(Fig. 18-11).
A2
P2 o 
Loading conditions (especially blood pressure)
A2
0° Lateral should be similar to the baseline study.
P2
A3 o Use of the same imaging planes facilitates com-
parison of pre- and postprocedure data.
P3 o Doppler data are recorded with the same instru-

A3 A2 P1 ment settings to ensure detection of any residual


45° 135° regurgitation.
P2 A2 o The postprocedure TEE includes evaluation for
90° the complications of mitral valve repair.
P3 A2 P1
o Persistentmitral regurgitation
o Systolicanterior motion of the mitral leaflet
o Functional mitral stenosis

Inferior/posterior o Ventricular systolic dysfunction

Figure 18-8  Imaging the mitral valve. Reference view demonstrating Aortic or Mitral Valve Replacement
the relationship of the TEE rotational imaging planes to the mitral valve with
the probe positioned in the standard midesophageal position. A1, A2, A3, n  In patients undergoing aortic or mitral valve
Anterior mitral leaflet segments; P1, P2, P3, posterior mitral leaflet seg- replacement, anatomic and functional assessment
ments. (From Foster GP, Isselbacher EM, Rose GA, Torchiana DF, Akins CW,
Picard MH: Accurate localization of mitral regurgitant defects using mul-
should be completed before the patient is in the
tiplane transesophageal echocardiography. Ann Thorac Surg. 65(4):1025– operating room.
31, 1998; Otto, CM: Textbook of Clinical Echocardiography, ed 5, Elsevier, n Evaluation of the severity of valve stenosis by TEE
2013, Philadelphia.) is problematic for several reasons, so intraoperative
TEE decisions regarding stenosis severity should
o The central scallops of the anterior (A) and pos- be avoided (Fig. 18-12).
terior (P) leaflets are seen in the 4-chamber and n The postprocedure TEE allows detection of pros-
long-axis views. thetic valve dysfunction and assessment of LV sys-
o The lateral (P1) and medial (P2) scallops of the tolic function.
posterior leaflets are seen in the bicommissural n 
Knowledge of the structure and function of each
view at 60° to 90° rotation. prosthetic valve type is needed for correct interpre-
o Starting in the 4-chamber plane, the lateral scal- tation of postprocedure data.
lops of the anterior (A1) and posterior (P1) leaflets
can be seen by slightly withdrawing the probe, or v  KEY POINTS
tilting superiorly; medial segments (A3 and P3) are o A parallel alignment between the ultrasound
seen by advancing the probe or tilting posteriorly. beam and aortic velocity is rarely possible on
o From the 2-chamber plane, all three scallops of TEE, resulting in underestimation of stenosis
the anterior leaflet are seen when the probe is severity.
turned toward the patient’s right and all three o Direct imaging and planimetry of aortic valve
scallops of the posterior leaflet are seen when the area is limited by reverberations and shadowing
probe is turned leftward. due to valve calcification but may be possible
o A transgastric short-axis view of the mitral valve, with 3D imaging.
when possible, shows both leaflets. o Mitral stenosis can be evaluated using the Dop-
3D images of the mitral valve, showing the left
o pler pressure half-time method because the
atrial side of the valve, facilitate identification TEE probe position allows a parallel alignment
of prolapsing segments and flail chords and are with antegrade transmitral flow.
recommended, when available. o A small amount of valve regurgitation is seen
o The direction of the mitral regurgitant jet is with a normally functioning prosthetic valve.
helpful in defining the mechanism of regurgi- o Central regurgitation is common with a bio-
tation. Severity of regurgitation is evaluated by prosthetic valve; eccentric jets, with a variable
(Fig. 18-9): pattern of regurgitation depending on valve
o Vena contracta width type, are typical with mechanical valves.
o CW Doppler signal intensity of regurgitant com- o A small amount of paravalvular regurgitation
pared to antegrade flow may be seen but a large paravalvular leak may
o Pulmonary vein systolic flow reversal (Fig. 18-10) require immediate correction.
390 CHAPTER 18  Intraoperative and Interventional Echocardiography

FR 12Hz
13cm
0 128 180 2D color Doppler
2D
72%
C 50
P Off
Gen
CF
59%
4.4MHz LA
WF High
Med

JPEG
PAT T: 37.0C 60 bpm
TEE T: 40.5C

2-chamber 4-chamber Area(s)


3D A10.82 cm2
color I
Doppler
LA

LV

Vena contracta

Figure 18-9  3D Doppler evaluation of mitral regurgitation.  Mitral regurgitant severity is evaluated with measurement of vena contracta and calculation of re-
gurgitation orifice area by the proximal isovelocity surface area method using standard 2D color Doppler imaging. In selected patients, 3D Doppler imaging may be
helpful for better visualization of the vena contracta as shown here with 3D imaging used to obtain a short-axis view of the vena contracta area (lower left image).


oRarely, retained mitral leaflet tissue impairs
normal motion of a mechanical mitral occluder. v  KEY POINTS
o Recording images of leaflet/occluder motion o 
The baseline intraoperative TEE focuses on
and transvalvular Doppler flows in the operat- (Fig. 18-13):
ing room provides a useful comparison for sub- o Presence and location of vegetations
sequent studies. o Mechanism of valve dysfunction
o Severity of regurgitation
Endocarditis o Paravalvular abscess
n Intraoperative TEE is essential in assessment of o Detection of other complications (e.g., fistulas,
the degree of valve destruction and paravalvular pseudoaneurysm)
involvement due to endocarditis. o 
Extensive valve and paravalvular destruction
n 
Postprocedure evaluation in the operating room in endocarditis often requires a complex surgi-
allows assessment of valve function after repair cal repair; knowledge of the surgical details is
or replacement and provides a baseline for subse- needed for correct interpretation of the post-
quent imaging studies. procedure images.
Intraoperative and Interventional Echocardiography  CHAPTER 18 391

FR 18Hz M4M4
13cm Baseline PW 38.5
74 pulmonary vein 50%
2D 2.9MHz
69% WF 150Hz
C 50 SV4.0mm
P Off 2.8cm
Gen
38.5
CF cm/s
59%
4.4MHz 40
WF High
Med 20

cm/s

20

40

60

80

PAT T:37.0C 75mm/s


TEE T: 39.2C 60 bpm

FR 54Hz
14cm Pulmonary vein PW
70 after procedure 50%
2D 2.9MHz
69% WF 150Hz
C 50 SV4.0mm
P Off 3.6cm
Gen

40

20

cm/s

20

40

60

80

PAT T:37.0C 75mm/s


TEE T: 39.9C 60 bpm

Figure 18-10  Pulmonary vein flow.  In a patient undergoing mitral valve repair, the baseline pulmonary vein flow pattern (top) shows systolic reversal (arrow).
After valve repair (bottom), systolic flow reversal is no longer present with normal flow into the left atrium in systole (arrow).

LA

LV

Figure 18-11  Post–mitral valve repair.  In the same patient as Fig. 18-4, the after repair 2D images (left) show an annular ring (arrow) with normal mitral
leaflet coaptation and no residual regurgitation. The 3D view (right) from the left atrial perspective shows the annular ring with sutures appearing as small
bumps along the ring.
392 CHAPTER 18  Intraoperative and Interventional Echocardiography

A B
Figure 18-12  Aortic valve anatomy.  TEE short-axis images in two patients with aortic valve disease show a typical calcified trileaflet aortic valve in a
76-year-old patient with severe aortic stenosis showing marked restriction of systolic leaflet opening (A) and a bicuspid valve (arrows) in a 28-year-old man
with no prior cardiac history who presented with acute aortic dissection (B).

LA LA

LV
LV

A B
Figure 18-13  Endocarditis.  In a 28-year-old woman who presented with fever and bacteremia, intraoperative TEE demonstrates a vegetation on the left atrial
side of the valve in systole in the 4-chamber view (A), but the attachment site of the vegetation and the mechanism of regurgitation are not well defined. Rotating
the image plane toward the 2-chamber plane (B) now demonstrates the attachment of the 2 cm vegetation on the P2 scallop of the posterior leaflet. Color Doppler
demonstrated a leaflet perforation adjacent to the vegetation. This information assisted in a successful mitral valve repair (not replacement) in this young woman.

Aortic Disease
n  Atheroma in the ascending aorta can be detected n  EE distinguishes involvement of the ascending
T
by TEE or epicardial scanning, which allows place- aorta (type A dissection) from a more distal (type
ment of bypass grafts and aortotomy sites in areas B) dissection.
of normal aortic tissue. n Evaluation of aortic regurgitation is a key element
n TEE is essential for accurate diagnosis of the pres- of the examination because dilation of the sinuses
ence and extent of aortic dissection; in urgent or extension of the dissection into the valve can
cases, diagnostic imaging may be performed in the result in valve dysfunction.
operating room (Fig. 18-14).
Intraoperative and Interventional Echocardiography  CHAPTER 18 393

TL

FL

A B
Figure 18-14  Aortic dissection.  Intraoperative TEE imaging quickly demonstrates the presence of a dissection flap in the descending thoracic aorta (A) with
color Doppler (B) showing a fenestration with flow from the true lumen into the false lumen. FL, False lumen; TL, true lumen.

o LV regional wall motion abnormalities due to dis-


v  KEY POINTS section extending into the coronary artery

oThe ascending aorta often is best visualized o After surgical repair, a persistent distal dissec-
using a sterile epicardial transducer to ensure tion flap is common.
the absence of atheroma at the site of proximal
coronary bypass graft anastomoses and at aor- Hypertrophic Cardiomyopathy
totomy sites. n  Hypertrophic cardiomyopathy may be treated
o A complete examination for aortic dissection surgically by basal septal hypertrophy resection
includes: (myectomy) or may be treated percutaneously with
o 
Midesophageal views of the aortic sinuses and catheter-based ablation of basal septal hypertrophy.
ascending aorta n Echocardiography provides evaluation of the pattern
o Imaging of the descending thoracic aorta in a slow of hypertrophy, baseline and postprocedure hemody-
pull-back from the diaphragmatic level to the arch namics, and detection of procedural complications.
o High TEE views of the aortic arch
o Imaging of the os of the left subclavian artery v  KEY POINTS
(which delineates type A and type B dissections) Imaging allows assessment of the pattern and
o
o Color Doppler imaging improves identification severity of septal hypertrophy, which is useful in
of dissection flaps and provides visualization of procedural planning.
flow in the true and false lumens. o Color Doppler localizes the level of out-
o The aortic valve is examined in standard flow obstruction based on the location of
long- and short-axis views, with color Dop- the flow acceleration proximal to the aortic
pler to detect and quantitate aortic regurgi- valve plane.
tant severity (Fig. 18-15). o CW Doppler alignment with the outflow tract
o Pulsed Doppler evaluation of descending aor- velocity may be difficult by TEE; sterile epicar-
tic flow is helpful in evaluation of aortic valve dial scanning in the operating room (transtho-
regurgitation. racic scanning for percutaneous procedures)
o The coronary arteries are visualized, if possible, is helpful if these data are needed for clinical
because the dissection flap may extend into the decision-making.
(most often, right) coronary ostium. o After surgical resection, and at each stage of
o Other indirect signs of aortic dissection include: a percutaneous procedure, imaging and Dop-
o 
Pericardial effusion (from extension into the pler assessment of anatomic and hemodynamic
­pericardium, signaling impending aortic rupture) results are documented.
394 CHAPTER 18  Intraoperative and Interventional Echocardiography

LV

Ao
Ao
LV

A B
Figure 18-15  Complications of aortic dissection.  A, In the same patient as in Fig. 18-14, the ascending aorta is markedly dilated with a dissection flap
(arrow), distinct from the open aortic valve leaflets in this image. B, Color Doppler shows severe aortic regurgitation due to the combination of aortic dilation,
dissection, and a bicuspid aortic valve. Ao, Aorta.

38dB 2 /1/0/1
LA PW Depth  44mm
PW Gate  2.5mm
PW Gain  12dB

Aorta

RV LV
1.0

m/s

1.0
A B
Figure 18-16  LV assist device.  A, Intraoperative TEE in this patient with an LV assist device shows the inflow cannula in the LV apex (arrow) in the 4-chamber
view. B, In a high TEE long-axis view of the ascending aorta, a color Doppler signal is seen with pulsed Doppler confirming continuous flow from the LV assist
device outflow cannula. The aortic valve does not open in systole when an LV assist device completely unloads the LV.

Systolic anterior motion of the mitral valve


o
Advanced Heart Failure Therapies
leaflet and accompanying mitral regurgi- n LV assist devices (LVADs) and total artificial hearts
tation may resolve after relief of outflow (TAH) are increasingly used during complex car-
obstruction. diac procedures and long-term in patients with
o Postprocedure imaging allows detection of severe LV dysfunction.
complications, such as a ventricular septal n 
The echocardiographer should be familiar with
defect, which is a rare complication of surgical the specific type of LVAD or TAH being used;
myotomy-myectomy. continual design advances require regular updates
on implantation approaches and expected flow
patterns (Fig. 18-16).
Intraoperative and Interventional Echocardiography  CHAPTER 18 395

Heart Transplantation
n After orthotopic heart transplantation, intraopera-
tive TEE allows evaluation of the anastomoses to
the aorta, pulmonary artery, pulmonary veins, and
LA
vena cavae.
n Intraoperative TEE provides a baseline assessment
of LV and RV systolic function in the transplanted
RA heart.
v  KEY POINTS
LV
Evaluation of LV and RV function in the oper-
o

RV
ating room is helpful in managing acute hemo-
dynamics as the patient is stabilized after heart
transplantation (Fig. 18-17).
o Tricuspid regurgitation may be present early
after transplantation if RV systolic dysfunction
is present.
Congenital Heart Disease
Figure 18-17  Cardiac transplantation.  Intraoperative TEE was performed n In patients undergoing surgery for congenital heart
at baseline and after cardiac transplantation. This 4-chamber view shows the disease, it is essential that intraoperative TEE be
atrial anastomosis site (arrow). Other views were obtained to evaluate LV and performed by a skilled echocardiographer who
RV systolic function and to measure antegrade aortic flow velocity.
is knowledgeable about congenital heart disease
anatomy, physiology, and surgical approaches.
v  KEY POINTS
v  KEY POINTS Except for simple corrective procedures, such as
o
o TEE evaluation is helpful during implantation closure of an isolated atrial septal defect, intraop-
of an LVAD for: erative TEE for congenital heart disease should
o Placement of inflow and outflow cannula be performed by echocardiographers with addi-
o Ventricular volumes and systolic function tional training in congenital heart disease.
o LVAD inflow and outflow velocities and flow o Additional information on intraoperative TEE
patterns for congenital heart disease is available in the
o De-airing of the pump before activation Otto Companion Series book: Echocardiography
o Typically, the LVAD inflow cannula is in in Pediatric and Adult Congenital Heart Disease: A
the LV apex with the outflow cannula in the Volume in Practical Echocardiography Series edited by
proximal ascending aorta. Karen Stout and Mark Lewin (Elsevier, 2011).
o The aortic valve usually remains closed
throughout the cardiac cycle because cardiac
output now is directed through the LVAD; sig- TRANSCATHETER AND HYBRID PROCEDURES
nificant aortic regurgitation is a contraindica-
tion to LVAD placement.
Atrial Septal Defect or Patent Foramen
o The LV is small (decompressed) when LVAD Ovale Closure
function is normal. LV contraction reflects the n TEE or intracardiac echocardiography (ICE) guid-
underlying disease process, with improvement ance is used for transcatheter closure of an atrial
due to disease resolution, not the LVAD. septal defect or patent foramen ovale (Table 18-2).
o LVAD flow may be pulsatile or continuous (also n Baseline images allow visualization of the location,
called axial); each has a specific expected flow size, and shape of the defect using 2D rotational
pattern. scanning and 3D imaging.
o Complications of an LVAD that may be n 
Postprocedure imaging allows evaluation for cor-
detected by echocardiography include: rect placement of the device and detection of any
o Intracardiac thrombus formation residual shunt.
o Obstruction of inflow or outflow cannula due to
positioning or thrombus Transcatheter Aortic Valve Implantation
o Regurgitation of LVAD valves (in some devices) n 
Baseline images of LV global and regional func-
o Inadequate flow volumes tion are recorded in standard views for comparison
o Pericardial hematoma around the inflow cannula to postprocedure images.
396 CHAPTER 18  Intraoperative and Interventional Echocardiography

TABLE 18-2 Echocardiography in Transcatheter Interventions for Valve Disease


Evaluation
Procedures Preprocedural Evaluation Procedural Monitoring of Complications
Transcatheter aortic • Aortic stenosis severity • Aortic cusp length • Paraprosthetic regur-
valve implantation • Leaflet number and (< annular – coronary ostial gitation
(TAVR) calcification distance) • Mitral regurgitation
• Annulus diameter (2D TTE • LV outflow tract shape (avoid • Coronary ostial
or TEE) subaortic septal bulge) occlusion (LV wall
• Assess for aortic atheroma motion abnormalities)
• Positioning and function of • Cardiac tamponade
prosthetic valve • Aortic dissection or
rupture
Balloon mitral • Mitral stenosis severity • Transseptal puncture • Worsening mitral
valvotomy • Mitral valve morphology • Position of dilating balloon regurgitation
• Commissural fusion and • Changes in ΔP and MVA • Atrial septal defect
calcification • Mitral regurgitation severity
• Coexisting mitral
regurgitation
• Left atrial thrombus
Transcatheter mitral • Valve anatomy • Transseptal puncture • Leaflet or chordal
valve repair • Severity of mitral • Position of delivery system tears
regurgitation • Residual regurgitation • Pericardial effusion
• Mechanism of mitral • Suboptimal position
­regurgitation of device
• Mitral coaptation length
and depth
• Flail leaflet gap and width
Closure of prosthetic • Presence and severity of • Location and size of • Residual regurgitation
paravalvular paraprosthetic mitral paravalvular defects (3D TEE) • Device dislodgement
regurgitation ­regurgitation • Catheter placement • Pericardial effusion/
• Prosthetic valve function • Seating of device tamponade
• LA thrombus

From Otto CM: Textbook of Clinical Echocardiography, 5 ed, Elsevier, 2013, Philadelphia.
MR, Mitral regurgitation; MVA, mitral valve area; ΔP, pressure gradient.

n  alve anatomy and function are evaluated using


V
standard imaging views and Doppler approaches, Transcatheter Mitral Valve Repair
for comparison to postprocedure data. n  TEE imaging is used to evaluate patients for
n 3D TEE imaging is used to measure outflow tract transcatheter mitral valve repair with attention to
area and diameters to size the valve annulus, in con- the mechanism of mitral regurgitation and valve
junction with data from CT imaging (Fig. 18-18). anatomy.
n Transcatheter aortic valve implantation typically is n  Baseline measurements include annulus dimen-
monitored using TEE for: sions, leaflet tenting distances, and length of width

Correct placement of the valve at the time of implan- of leaflet noncoaptation.
tation (Fig. 18-19) n TEE guidance is used to monitor positioning of
• Evaluation of regurgitation (often paravalvular) after the mitral repair device (Fig. 18-22).
implantation n 
After the procedure, color and pulsed Doppler
• Evaluation of leaflet motion of the implanted valve allow detection of residual regurgitation and eval-
(Fig. 18-20 and Fig. 18-21) uation of antegrade flow velocities (Fig. 18-23).
Intraoperative and Interventional Echocardiography  CHAPTER 18 397

FR 29Hz 3D Beats 4Q
3D
9.0cm
2D
62%
C 50
P Off
Gen
Long axis Short axis

PAT T:37.0C
TEE T: 38.7C 52bpm

Figure 18-18  Aortic annulus size.  3D TEE imaging in a patient being considered for transcatheter aortic valve implantation allows measurement of aortic
annulus diameter and provides a short-axis view of the annulus for measurement of circumference and area to assist in choosing the correct prosthetic valve
size.

Aortic stenosis
0

Positioning LA 2
LA
valve
3

4 Ao
LV 5
LV
Ao
6

7
10
8
T:37.0C 65 bpm
JPEG 9 T:39.5C
M4M4
T:37.0C 68 bpm Post-TAVR 61.6
T: 38.4C 0
59.3
0

1
After valve
implantation 2
61.6
cm/s
3 59.3 5
cm/s

4
LV
5
Ao
6

10
7
JPEG
T:37.0C
JPEG 8 65 bpm
T:39.0C
T:37.0C 67 bpm
T: 38.7C
Figure 18-20  Aortic stenosis before and after transcatheter valve
Figure 18-19  Guidance of transcatheter aortic valve implanta- implantation.  In a long-axis view, the baseline image shows a calcified
tion.  TEE imaging is used during the procedure (top) to position the wire immobile aortic valve with high-velocity flow beginning at the valve level.
(arrow) and valve. After valve implantation (bottom), the thin leaflets (long The postprocedure image in the same view (bottom), now shows laminar
arrow) and mesh valve support (short arrows) are seen. Ao, Aorta. low-velocity flow across the prosthetic valve. Ao, Aorta.
3D Beats 4Q

JPEG
61 bpm

Figure 18-21  Transcatheter aortic valve prosthesis.  After implantation, 2D (left) and 3D (right) imaging show the trileaflet biological valve with thin leaflets
and normal systolic opening.

M4

AMVL

Figure 18-22  Transcatheter mitral valve repair.  Intraprocedural 3D TEE is


used to guide the clip device towards the mitral valve and ensure the device
is correctly positioned relative to the specific valve pathology in that patient. PMVL
AMVL, Anterior mitral valve leaflet; PMVL, posterior mitral valve leaflet. 60 bpm

LA

LV

JPEG

Color Doppler 3D Imaging 60 bpm

Figure 18-23  Posttranscatheter mitral valve clip.  The anterior and posterior leaflets are now clipped together resulting in two mitral orifices (arrows) as
seen on color Doppler (left) and 3D imaging (right).
Intraoperative and Interventional Echocardiography  CHAPTER 18 399


Guide the catheter across the defect for placement of
Mitral Balloon Valvotomy the closure device (Fig. 18-26)
n  Mitral stenosis often is treated by percutaneous • Ensure the device placement does not affect valve
balloon valvotomy rather than by surgical valve occluder motion (Fig. 18-27)
replacement using TEE or ICE guidance for posi- • Assess residual regurgitation
tioning of the transseptal catheter and balloon
position across the mitral valve.
n  Transmitral mean gradient and pressure half- M4M4
47.6
time valve area are monitored after each bal-
loon d ­ilation, along with invasive pressure
measurements.
LA
n  Doppler evaluation for mitral regurgitation using
standard approaches is repeated after each balloon
dilation; an increase in regurgitant severity pre- 47.6
cludes further dilation attempts. cm/s

n A small shunt across the atrial septum may be seen


RA LV
after any procedure involving a transseptal punc-
ture (Fig. 18-24). RV

Transcatheter Closure of Paravalvular


Prosthetic Regurgitation
n Newer transcatheter interventions include closure of
paravalvular leaks in patients with mechanical valves. JPEG
n 
Echocardiographic imaging is essential during 60 bpm
these procedures to: Figure 18-24  Small atrial septal defect.  After transseptal puncture for a

Identify the site, size, and shape of the paravalvular transcatheter procedure, a small left-to-right shunt (arrow) across the atrial
defect (Fig. 18-25) septum is often present.

FR 17Hz M4M4
10cm 61.6

2D
66%
C 50
P Off
Gen
CF
59%
4.4MHz
WF High LA
Med 61.6
cm/s

MVR

LV

JPEG

PAT T:37.0C 80 bpm


TEE T: 41.1C
Figure 18-25  Paravalvular prosthetic mitral regurgitation.  The 2D image rotation plane is adjusted to show the paravalvular jet (arrow) lateral to the
mitral valve replacement. If possible, acceleration on the LV size of the valve is visualized to precisely locate the defect although this may be challenging due to
reverberations from the mechanical prosthetic valve. MVR, Mitral valve replacement.
400 CHAPTER 18  Intraoperative and Interventional Echocardiography

FR 11Hz 3D Beats 1
6.6cm
3D
3D 47%
3D 40dB

MVR

catheter
JPEG
PAT T:37.0C TEE T: 39.7C

Figure 18-26  Guidance of transcatheter closure for paravalvular regurgitation.  3D imaging is used during the procedure to guide the catheter (arrow)
into the paravalvular defect. One device has already been deployed medial to the catheter position. MVR, Mitral valve replacement.

Figure 18-27  Postparavalvular leak closure.  3D imaging from a left atrial perspective shows a bileaflet mechanical mitral valve in the open position in
diastole (left) with two small closure devices (arrows). The systolic image with the leaflets closed is shown on the right. The images are rotated compared to
the orientation in Fig. 18-26.
Intraoperative and Interventional Echocardiography  CHAPTER 18 401

THE ECHO EXAM

Intraprocedural TEE
Basic Principles of Intraprocedural TEE
• Establish diagnosis preprocedure when possible
• The goals of the baseline TEE are to:
• Confirm the diagnosis
• Provide additional information on repairability
• Serve as comparison to postprocedure study
• Assess LV and RV function
• Check for other abnormalities
• Perform a complete study unless there are clinical or time constraints
• Record postprocedure images at similar loading conditions to baseline
• Communicate and discuss findings at time of study
• Report TEE findings in medical record and store TEE images

Factors that Affect Cardiac Hemodynamics During Procedures


Positive pressure mechanical ventilation
Volume status
Myocardial “stunning” secondary to aortic cross-clamping
Effects of cardiopulmonary bypass
Pharmacologic therapy

Transcatheter Interventions
Indications for Echocardiography
Transcatheter aortic valve replacement (TAVR)
Transcatheter mitral valve repair procedures
Balloon mitral valvotomy
Transcatheter closure for paravalvular regurgitation
Atrial septal defect or patent foramen ovale closure
Septal ablation for hypertrophic cardiomyopathy
Echocardiographic Approaches
Transesophageal imaging (with 3D)
Transthoracic imaging
Intracardiac imaging
402 CHAPTER 18  Intraoperative and Interventional Echocardiography

Key Data for Intraoperative TEE


Procedure Preprocedure During Procedure Postprocedure
Mitral valve repair • Valve anatomy • Baseline anatomy and • Assess for complications
• Repairability regurgitation pre-CPB • Persistent mitral
• Regurgitation • Residual mitral regurgitation regurgitation
• Mechanism post-CPB • Mitral SAM
• Severity • Functional mitral
stenosis
• Tricuspid regurgitation
• Circumflex artery injury
• LV function
Valve stenosis • Valve anatomy and • Baseline LV function • Paravalvular regurgitation
calcification • Baseline valve anatomy and • LV function
• Severity of stenosis function
• LV function • Post-CPB assess repair or
• PA pressures prosthetic valve
Endocarditis • Vegetations • Baseline valve anatomy and • Baseline postsurgery valve
• Abscess formation function function
• Valve function • Post-CPB valve function • LV function
• LV function
• PA pressures
Prosthetic valve • Valve thrombosis • Baseline study to guide • Baseline prosthetic valve
dysfunction • Pannus formation surgical intervention function
• Paravalvular • Post-CPB assess new • Follow-up depends on
regurgitation prosthetic valve residual lesions and valve
type
Aortic dissection • Confirm diagnosis • Post-CPB document • Long-term follow-up of
• Extent of dissection residual dissection flap aortic valve and LV function
distally distal to repair • Residual dissection flap
• Coronary ostial • Assess flow in true and false distally
involvement lumen
• Aortic valve function • Aortic valve function
• Pericardial effusion • LV function
Hypertrophic cardio- • Location and severity of • Residual subaortic • Long-term hemodynamic
myopathy septal thickening obstruction results
• Severity of subaortic • Ventricular septal defect • LV systolic and diastolic
dynamic obstruction • Residual mitral regurgitation function
• Mitral regurgitation
Congenital heart • Diagnosis of complex • Baseline evaluation of each • Long-term anatomic and
disease anatomy lesion anatomy and functional results
• Surgical planning ­hemo­dynamics • Ventricular function
• Integration with data from • Residual lesions after CPB • Pulmonary pressures
other imaging modalities

CPB, Cardiopulmonary bypass; PA, pulmonary artery; SAM, systolic anterior motion.
Intraoperative and Interventional Echocardiography  CHAPTER 18 403

SELF-ASSESSMENT QUESTIONS

Question 1 Question 3
During weaning from cardiopulmonary bypass in a Intraprocedural echocardiographic monitoring is
patient undergoing coronary artery bypass grafting performed in a patient with congenital heart dis-
surgery, the following image was obtained (Fig. 18-28). ease undergoing an invasive electrophysiologic abla-
The structure indicated by the arrow most likely is: tion procedure. The following image (Fig. 18-29) is
A. Air obtained. The finding (arrow) is consistent with:
B. Artifact A. Patent foramen ovale
C. Cannula B. Primum atrial septal defect
D. Tumor C. Unroofed coronary sinus
E. Thrombus D. Atrial septal puncture

RA

Figure 18-29 
Question 4
In this TEE image (Fig. 18-30), the arrow points at:
A. LA appendage
B. Coronary sinus
C. Left main coronary artery
D. Right coronary artery
Figure 18-28  E. RA appendage
Question 2
Intraoperative echocardiography is performed in
a patient undergoing coronary bypass graft sur-
gery and mitral annuloplasty repair. Preoperative
images show severe LV systolic dysfunction with
and ejection fraction of 25% and severe secondary
mitral regurgitation. Following surgery, LV func-
tion improves with an ejection fraction of 35% and
mitral regurgitation is now mild, central, and along
commissural edges. Hemodynamic status is tenu-
ous and a dose of epinephrine is given. LV function
remains comparable, but you note that the mitral
regurgitation jet seen previously has now increased
to moderate range severity. Based on the informa-
tion available, the most likely explanation for this
finding is:
A. Failed coronary bypass graft
B. Annuloplasty ring dehiscence
C. Expected function of mitral repair
D. Ruptured mitral valve leaflet Figure 18-30 
404 CHAPTER 18  Intraoperative and Interventional Echocardiography

Question 5 Question 7
3D images are obtained in a patient undergoing a You perform a TEE study on a patient who is under
catheter-based valve intervention. The following evaluation for mitral valve replacement. Cardiac
image of the deployment catheter in the left atrium is anesthesia is at the bedside and conscious sedation
obtained (Fig. 18-31). The echolucent finding (arrow) is administered. The probe is advanced through the
is consistent with: bite block and oropharynx. The patient is agitated
A. Acoustic shadowing and coughing; the probe cannot be advanced beyond
B. Microbubbles 10 cm. Which of the following is the next step?
C. Stitch artifact A. Refer the patient for barium swallow
D. Vegetation B. Withdraw probe and reattempt placement
C. Increase sedation and advance probe further
D. Rotate probe clockwise and retroflex probe tip
Question 8
A 64-year-old man is referred for coronary artery
bypass grafting surgery. He has a history of prior
cardiac surgery. During his procedure, the following
image is obtained (Fig. 18-33). You conclude that his
prior procedure was:
A. Coronary artery bypass graft surgery
B. LA appendage exclusion procedure
C. Mitral valve annuloplasty
D. Aortic valve replacement

PAT T:37.0C
TEE T: 39.6C

Figure 18-31 

Question 6
This Doppler tracing recording during an intraopera-
tive TEE study shows (Fig. 18-32):
A. Atrial fibrillation
B. Atrial flutter
C. Superior vena caval flow
D. Coronary artery flow
E. Pulmonary vein flow

Figure 18-33 
PW:3.5MHz
1.5

m/s

1.0

Figure 18-32 
Intraoperative and Interventional Echocardiography  CHAPTER 18 405

Question 9
A 54-year-old man with mitral valve prolapse is
referred for mitral valve repair. There is a known flail
component. The following TEE image is obtained
from the atrial side of the mitral valve with the aortic
valve rotated to the top of the image (Fig. 18-34). The
surgeon inquires as to what portion of the mitral valve
is most affected?
A. A1 segment/P1 segment coaptation
B. A2 segment/P2 segment coaptation
C. A3 segment/P3 segment coaptation

Figure 18-34 

Question 10
A 58-year-old presents to the emergency department A.
Resuspension of the aortic valve
with acute, severe chest and upper back pain and was B. Mechanical aortic valve replacement
diagnosed with an acute type A aortic dissection. A C. Bioprosthetic aortic valve replacement
transthoracic image obtained in the emergency depart- D. Transcatheter aortic valve implantation
ment and an intraoperative TEE recorded at the end of
the surgical procedure are shown (Fig 18-35, A and B).
The patient has undergone aortic root replacement with:

V .73
.79

5
post

2.73
.79
10

15

A B
Figure 18-35 
406 CHAPTER 18  Intraoperative and Interventional Echocardiography

Question 11 Question 12
You are asked to determine if there is systolic flow A patient is sent for urgent aortic valve replacement
reversal in the pulmonary veins in a patient under- and aortic root repair following diagnosis of an acute
going evaluation for mitral valve replacement (Fig. ascending aortic dissection. The cardiothoracic sur-
18-36). Label the two structures, choosing from the geon requests guidance in placing cannula to retro-
following list. grade deliver cardioplegia solution. You visualize and
A. Left upper pulmonary vein guide towards:

/



B. Left lower pulmonary vein A. Coronary sinus






C. Right upper pulmonary vein B. Descending abdominal aorta

9






D. Right lower pulmonary vein C. Right coronary artery






D. Left brachiocephalic vein

9



i r
a h
a

r/ t
b

s e
/r u
.t c
a
Figure 18-36

/: / k
s
tt p
h
Intraoperative and Interventional Echocardiography  CHAPTER 18 407

ANSWERS

Answer 1: A the left main coronary artery arising from the left
coronary cusp (Fig. 18-37).
This view of the RA shows microbubbles in the right

­
The LA appendage is just lateral to the coro-
heart consistent with intracardiac air. The dense mass
nary artery. The coronary sinus would be seen in a
of echoes indicated by the arrow is an air collection at
4-chamber view with posterior angulation or in a low
the superior aspect of the RA, near the RA append-

­
TEE view of the RA and RV. The RA appendage

/
age. Tumor or thrombus is unlikely given the clinical
is anterior to the aorta (Ao); in fact, a bit of the RA
setting and the density of the echo signal. The appear-

9
appendage is seen in this image anterior to the aorta
ance is not suggestive of an artifact. A cannula would
and medial to the RV outflow tract (RVOT).
show parallel smooth lines; the presence and position

9
of a cannula could be confirmed by direct inspec-

i r
tion in the operating room. TEE imaging is helpful in
ensuring that all intracardiac air is eliminated before

h
coming off bypass. LA

Answer 2: C

a
L-main

t
This patient has severe LV systolic dysfunction with

r/
severe secondary mitral regurgitation. Following coro- Ao
nary bypass grafting and mitral annuloplasty repair,
residual mitral regurgitation was only mild. Initially

e
following surgery, afterload on the ventricle is relatively

s
lower. Because of the patient’s tenuous hemodynamic
status epinephrine was given and afterload was tran- RVOT

/r u
siently increased. If the hemodynamic response to epi-
nephrine is excessive (systolic blood pressure over 160
to 180 mmHg), the increase in LV afterload and results
in an increase in mitral regurgitation, which is within

.t c
expected function of the repair. The regurgitation jet
remains central, so an annuloplasty ring dehiscence is
not suspected. A failed bypass graft is less likely given

a
that LV function is comparable to the immediate post- Figure 18-37

operative images (no new wall motion abnormalities

k
are identified). Spontaneous rupture of mitral valve Answer 5: A

/: /
leaflets is unlikely, but if present would present with This image was taken intraprocedurally in a patient
eccentric rather than centrally directed jets. undergoing a percutaneous mitral clip procedure.
Device delivery is via a transseptal puncture. In this
Answer 3: D

s
image, the catheter is seen crossing the interatrial
The image is taken from the midesophageal 4-chamber septum with the tip in the left atrium. As with any

tt p
­
view (0°), with the probe rotated slightly clockwise ultrasound image, ultrasound does not penetrate
(rightward). Closer to the transducer, the interatrial through bright reflectors. The field distal to the
septum is seen. There is left-to-right color Doppler reflector is acoustically shadowed. In this 3D image,
flow across the interatrial septum. Catheters are seen

h
there is acoustic shadowing along the length of the
in the right atrium. The location of the flow is distant catheter, with the largest under the catheter tip
from the fossa ovalis, where a secundum defect would where the device is located. Adjacent to the device,
be visualized. Similarly, a patent foramen ovale would injection of saline creates microbubbles just above
be visualized right at the juncture where the thinned the catheter tip, seen projecting in the top part of
secundum portion overlies the primum portion (in this the image. A vegetation would appear echodense,
case, thickened due to lipomatous hypertrophy). The not echolucent.
coronary sinus os is remote from where flow is identi-
fied in this image. For ablative procedures where left Answer 6: B
atrial access is needed, septal puncture is performed, This Doppler tracing of flow from the LA appendage
resulting in a small postprocedural defect. into the LA demonstrates regular low-velocity pul-
satile flow at a rate about 300 bpm, consistent with
Answer 4: C atrial flutter. Atrial fibrillation would result in rapid
This oblique short-axis view at the level of the aortic irregular LA appendage flow signals, usually with an
sinuses, just superior to the aortic valve leaflets, shows even lower velocity. Flow in the superior vena cava
408 CHAPTER 18  Intraoperative and Interventional Echocardiography

and in the pulmonary vein would show a typical


venous flow pattern with systolic and diastolic filling
velocities, and a small reversal of flow after atrial con-
traction. Coronary artery flow occurs predominantly
in diastole.
Answer 7: B
This patient is not tolerating the procedure. Cough- A3

/
ing during probe placement and difficulty advancing P3
the probe suggests that the probe may be errone-

9
ously placed in the trachea. Absence of diagnos-
tic echocardiographic images aids in confirming

9
wrong probe placement. Withdrawing the probe

i r
and reattempting placement is the correct answer.
When placing the probe, the tip should be midline

h
(not rotated), with the tip slightly anteflexed to aid
esophageal intubation. If resistance is encountered

a
during probe placement, the probe should never be Figure 18-38

t
advanced further; this may cause esophageal perfo-


r/
ration or other patient injury.
Answer 10: A
Answer 8: D The intraoperative image shows a normal appearing

e
The image provided is from a midesophageal view aortic valve with thin leaflets and without supporting

s
taken from the base of the heart. The aortic valve is struts or a sewing ring to suggest either a mechani-
seen in short axis in the middle of the image. At the cal or stented bioprosthetic valve. It is possible that

/r u
juncture of each of the aortic valve cusps, there is a a stentless valved conduit may have been used in this
bright echodensity (prosthetic) consistent with valve case, but it is more likely that the native valve has been
struts. There are thin, mobile tissue leaflets between resuspended in the aortic graft. With a transcatheter
the struts. The mitral valve and annulus are not seen bioprosthetic valve, echodensities along the aortic

.t c
in this view. The left atrial appendage is seen to the wall from the valve cage would be seen.
right of the aortic valve and is still intact. The tricus- With aortic dissection, aortic regurgitation may
pid valve, faint, is seen just to the left of the aortic be due to an intrinsic anatomic abnormality of the

a
valve in the image. The interatrial septum is seen at valve, such as a bicuspid aortic valve, or may be due
the top-left of the image bisecting one of the aortic to extension of the dissection into the base of the aor-

k
valve cusps. tic valve. In this case, aortic regurgitation was due to

/: /
prolapse of the dissected segment through a normal
Answer 9: C trileaflet aortic valve. The prolapsing tissue impeded
In the 3D image provided, the mitral valve is seen leaflet coaptation, resulting in severe aortic regurgita-

s
enface. The anterior valve is seen obliquely at the top tion. Replacement of the ascending aorta with elimi-
of the image. This is a standard view of the mitral nation of the dissection flap restored normal aortic

tt p
valve taken from a 3D volume. The anterior mitral leaflet closure. Thus, the aortic valve was resuspended
valve leaflet is closer to the aortic valve, and the pos- with only mild regurgitation postoperatively.
terior mitral valve is in the far field. Each mitral valve
leaflet is partitioned anatomically into three segments. Answer 11

h
On the lateral side of the valve (left on this image), A1 a. Right lower pulmonary vein



and P1 coapt. On the medial side of the valve (right b. Right upper pulmonary vein



side of image), A3 and P3 coapt. A2 and P2 coapt in the This 3D image is taken from an upper-esophageal
center of the valve. In this image there is redundancy window with the transducer angle at 8°. The trans-
with severe prolapse of A3/P3 with a flail tip of P3 ducer is rotated slightly rightward. From this view, the
(Fig. 18-38). right pulmonary veins are seen entering the left atrium
This appears as protrusion into the atrial side of on the left side of the image. The left pulmonary veins
the valve (right side of the coaptation in this image). It would be seen by turning the transducer toward the
is important to utilize anatomic landmarks to ensure patient’s left side. Often the left upper and left lower
accurate labeling for leaflet nomenclature. Leaflet ori- pulmonary veins are in different image planes, requir-
entation flips left-right with 3D imaging when viewing ing the probe be move up and down slightly in the
the valve from the ventricular size or in a short-axis esophagus, but they would enter the LA from the right
view on 2D transthoracic imaging. side of the image.
Intraoperative and Interventional Echocardiography  CHAPTER 18 409

Answer 12: A
Coronary
Administration of retrograde cardioplegia solution is sinus
aided by TEE visualization of the coronary sinus for
cannulation. The coronary sinus is best seen in the
distal esophagus from the 4-chamber view, just as the RA
transducer is at the gastroesophageal junction, and is
seen as a vessel vertically crossing the middle of the
scanning sector (Fig. 18-39).

/
LV
For minimally invasive surgery, smaller, steerable

9
catheters have been developed to increase success RV
of coronary sinus cannulation, but still often require

9
the aid of echocardiographic guidance in optimizing

r
placement. Flow in the descending abdominal aorta is

i
directed away from the heart and would not provide a
conduit for retrograde cardioplegia. Selective cannu-

h
lation of the right coronary artery would not provide
enough myocardial distribution for the cardioplegia

ta
solution.

r/
Figure 18-39

e

s
/r u
.t c
k a
/: /
s
tt p
h
Index

A Aorta. see also Ascending aorta;

9/
Aortic regurgitation (AR), 52, 52f, 61,

9
Abdominal aorta, proximal, 336–337, Descending thoracic aorta 61f, 239–244, 240f, 259f, 262–263,

r
337f atheroma of, 346 265–266, 354

i
Abscess, paravalvular, 298–300, 299f, 311 echocardiographic anatomy of, 21t antegrade aortic flow and stenosis
Accuracy, of echocardiography, 85, 85f identification of, 355, 356f evaluation, 243
Acoustic impedance (Z ), 1, 2t, 18 imaging measurements of, 25t aortic disease and, 340

h
Acoustic shadowing, 6, 6f, 19, 290, 407 midascending, 335f aortic dissection and, 344, 344f
Acute chest pain, causes of, 149t pseudoaneurysms of, 345–346, 345f chronic ventricular pressure evaluation,

a
Acute coronary syndromes, 97, 148–150, transesophageal echocardiography of, 250–251

t
165 337 clinical echocardiographic correlation,

r/
basic principles of, 148 transthoracic echocardiography of, 334, 241t
cardiac hemodynamic evaluation in, 335f–338f color Doppler of, 335
150 upper limits of normal dimensions in, CW Doppler evaluation of, 242, 243f,

e
chest pain in, alternate causes of, 150 340t 255
ejection fraction in, measurement of, Aortic annulus, area and diameter determine the etiology of, 239

s
149–150 measurement of, 217f endocarditis and, 311
regional ventricular function in, 148–149 Aortic arch, 337, 338f evaluation of, 217
Adult Congenital Heart Disease suprasternal notch window of, 337, examination of, 255–256, 255t

/r u
programs, 356 338f, 353 holodiastolic flow reversal in, 255, 336,
Advanced echocardiographic modalities, transesophageal echocardiography of, 338f
65–84 55–56 LV shape changes with, 244f
clinical utility, 71, 72t Aortic atheroma, 323 regurgitant volume calculated, 237f

.t c
contrast echocardiography, 74–75, Aortic atherosclerosis, as aortic dilation severity of, 239–243, 264–265
74f–75f cause, 342t–343t quantitation of, 256t
examination, 78–80, 78t Aortic coarctation, 360f–361f, 368, 368f, quantitative evaluation of, 255t
intracardiac echocardiography, 75–76, 382 small central jet on, 263

a
76f Aortic dilation, 92t–94t volume overload evaluation for, 243–244
myocardial mechanics, 71–74 chronic, 340–342, 342t–343t Aortic root, 334

k
point of care echocardiography, 76, 79f hypertensive, 341 Aortic sinus(es)

/: /
stress echocardiography, 65, 66t–67t Aortic disease, 342t–343t calculation of dimension, 340t
three-dimensional echocardiography, differential diagnosis of, 92t–94t parasternal window views of, 335–336,
65–71 Aortic dissection, 92t–94t, 96–97, 349t 335f
Advanced heart failure therapies, 394–395 as aortic dilation cause, 342t–343t three-dimensional echocardiography

s
Air, intracardiac, 385f, 407 ascending aorta in, 339f of, 72t
Akinesis, 145t as cause of chest pain, 148 transesophageal echocardiography of,
due to dobutamine, 167 complications of, 344 339–340

tt p
Alfieri repair, for mitral valve, 277 dissection flap identification, 342–344, Aortic stenosis (AS), 42, 213–218, 225t,
Aliasing velocity, 235, 236f 343f 232
A-mode echocardiography, recordings of intraoperative transesophageal aortic regurgitation evaluation and, 217
aortic valve motion, 5f echocardiography for, 392, ascending aorta evaluation and, 217

h
Amplitude (dB), 1, 2t 393f–394f, 403t causes of discrepancies in measures
Anatomy, echocardiographic, terminology with pericardial effusion, 354 of, 218t
for, 21t type A and type B, 392 of chronic pressure overload, 217
Anemia, 291 Aortic flow dobutamine stress echocardiography
Aneurysmal wall motion, 145t antegrade, 243 for, 233
Aneurysms, 154, 155f. see also holodiastolic, 242 low output, 66t–67t
Pseudoaneurysms Aortic jet velocity, 214, 214f, 234 etiology determination of, 213–214,
aortic-mitral intervalvular fibrosa, in atrial fibrillation, 218f 213f
299–300, 300f CW Doppler recording of, 214f examination of, 225t
sinus of Valsalva, 299f, 310 peak, 232 fluid dynamics of, 215f
Angina, 92t–94t Aortic-mitral intervalvular fibrosa, high-velocity systolic jets, 214t
unstable, as cause of chest pain, 148 aneurysm, 299–300, 300f low-output low-gradient, 217–218,
Ankylosing spondylitis, 342 Aortic pseudoaneurysm, 277f, 291 219f, 234
Annuloplasty rings, mitral, 278, 278f as aortic dilation cause, 342t–343t M-mode tracing, 232

Page numbers followed by f refer to figures, by b


to boxes, and by t to tables.

410
INDEX 411

Aortic stenosis (AS) (Continued) Ascending aorta (AA) Biphasic response, to dobutamine, 167
moderate, 234 aortic stenosis and, 217 Blood flow velocity, underestimation of, 10
severity of, 214–217 atheroma in, 392 Blood pressure. see also Hypertension;
categories of, 215t Dacron tube graft replacement of, 345f Hypotension; Pulmonary
classification, 225t dilated, 343f hypertension
quantitation, 226t long-axis view of, 26, 26f in pulsus paradoxus, 194f
transesophageal echocardiographic proximal, parasternal long-axis view recording of, 334
short-axis view of, 392f of, 335f Brachiocephalic artery, right, 344
Aortic valve replacement. see also superior vena cava and, 361 Burst length, 9t, 18
Bioprosthetic aortic valves; Prosthetic transesophageal echocardiography of,

/
aortic valves 51–52, 339f C
intraoperative transesophageal Atheroma, aortic, 346, 346f, 349t Calcific aortic stenosis, 192

9
echocardiography in, 389–390, Atherosclerotic disease, 353 Calcification
392f, 408 Atrial appendage. see also Left atrial in aortic atheroma, 346

9
Aortic valve(s), 275–277. see also Bicuspid appendage; Right atrial appendage of aortic valve, 389

r
aortic valve(s); Bioprosthetic aortic flow in, 48 of mitral annulus, 19, 181, 181f, 321

i
valves Atrial arrhythmias, from dobutamine, 146 of prosthetic valves, 267
anatomy and function of, 340 Atrial baffle leaks, 371 of trileaflet aortic valve, 392f
area (AVA), 215, 232, 291 Atrial fibrillation, 48f Carcinoid heart disease, 317, 318f

h
calcified, 217 aortic jet velocity, 218f Cardiac compression, from tamponade
echocardiographic anatomy of, 21t with aortic stenosis, 218 physiology, 198

a
equation, 234 conditions associated with, 90 Cardiac cycle, 103f

t
four-chamber view of, 31, 31f left atrial thrombi and, 313 Cardiac masses, 313–333

r/
imaging of, 242f prevalence of, 90 basic principles in evaluation of, 313,
TEE long-axis, 263 Atrial flutter, 407–408 313f
leaflet tip, 234 Atrial myocardial velocity (A′), 124, 126f benign valve-associated lesions,

e
long-axis view of, 51, 51f Atrial myxoma, 319f, 332, 332f 320–321
M-mode tracing, 25f Atrial septal aneurysm, 321, 321f, 332 cardiac tumors, 92t–94t

s
mechanical, 277, 277f Atrial septal defect (ASD), 62f, 357, intracardiac masses, distinguishing
regurgitation in, 275f 365–366, 365f, 383, 383f, 395 characteristics of, 326t
in hypertensive heart disease, 191 secundum, 360f left atrial (LA) thrombi of, 92t–94t,

/r u
resuspension of, 408 Atrial septal puncture, 407 313–323, 314f
short-axis view of, 28, 28f, 51, 52f Atrial septum, 29 left ventricular (LV) thrombi of,
systematic 3D study of, 69t systematic 3D study of, 69t 92t–94t, 314–315, 315f, 331
three-dimensional imaging of, 51–52, transesophageal echocardiography of, nonprimary cardiac tumors, 316–317,

.t c
52f, 72t 47–48, 47f 317f
transesophageal echocardiography of, Atrial velocity peak, 124 patent foramen ovale, 321–322
51–52 Atrioventricular valve, 355, 355f primary cardiac tumors, 318–319
trileaflet, calcified, 392f Atrium right heart thrombi of, 315–316,

a
vegetation on, 297f left. see Left atrium 316f–317f
zoomed parasternal long-axis diastolic right. see Right atrium vegetation, 319–320

k
image of, 292 Attenuation, 1, 2t, 18 Cardiac murmurs. see Heart murmurs

/: /
Apical ballooning, 179, 190 Automated implanted defibrillator, Cardiac output, 105–106, 115t, 150
Apical biplane method, for left ventricular placement of, 173 Cardiac sources of embolism, 322–323,
ejection fraction, 102 324t
Apical thrombus, 154, 155f B echocardiographic findings associated

s
Apical window, 29–32 Bacteremia, differential diagnosis of, 90, with, 325t
in congenital heart disease in adult, 91f Cardiac tamponade, pericardial, 97.
359, 359f Basal segments, hyperdynamic, 166, 166f see also Tamponade physiology,

tt p
Doppler data, 31–32, 32f Benign valve-associated lesions, 320–321 pericardial
four-chamber views, 29–31, 30f Bernoulli equation, 112, 221, 232, 366 Cardiac thrombi, 332
long-axis views, 29–31, 30f Bicuspid aortic valve disease, as aortic Cardiomyopathies, 168–179. see
two-chamber views, 29–31, 30f dilation cause, 342t–343t also Dilated cardiomyopathy;

h
Arrhythmias Bicuspid aortic valve(s), 262–264, 354, 380 Hypertrophic cardiomyopathy;
associated with myocardial infarction, as aortic dilation and dissection risk, 340 Restrictive cardiomyopathy
151 transthoracic imaging of, 341f definition of, 170
from dobutamine, 146 Bicycle exercise stress echocardiography, echocardiographic examination of,
Arrhythmogenic right ventricular 145, 166 184–185, 184t–185t
dysplasia (ARVD), 179 Bicycle stress ergometry, 96 general approach to, 168–173
Artifacts, 4–6 Bileaflet mechanical valves, 267f, 271, left atrial size evaluation in, 173
in ascending aorta, 340 274f, 292 left ventricle chamber size in, 168,
common causes of, 4–6 aortic, 272f 168f–169f
Doppler, 10–12, 11f mitral, 271f left ventricular diastolic function in,
in intraoperative transesophageal regurgitation in, 277 171–172, 172f
echocardiography, 385f Bioprosthetic aortic valves, 275, 275f left ventricular systolic function in, 169,
minimization of, 11f “patient-prosthesis mismatch”, 272, 291 170f
mirror image, 12 Bioprosthetic mitral valves, 278–279 mitral and tricuspid regurgitation
refraction, 6f imaging of, 278, 278f evaluation in, 173, 174f
reverberations, 6f, 354 mitral annular-papillary muscle pulmonary artery pressures in,
shadow, 11 continuity, 290 estimation of, 172–173, 172f–173f
412 INDEX

Cardiomyopathies (Continued) Continuity equation, 237, 271, 273f Dextroversion, 370


right ventricular size and function Continuity equation valve area, 215, 216f Diagnostic value, of echocardiography, 85
evaluation in, 173, 174f Continuous wave (CW) Doppler, 7t, 10, Diastolic deceleration slope, 239, 300,
takotsubo, 179 19 301f
ventricular hypertrophy in, 170–171 antegrade flow recording, 10f Diastolic flow reversal, 240, 243f
Cardiomyopathy, differential diagnosis of, aortic jet velocity, 214f Diastolic function
92t–94t for aortic regurgitation evaluation, 242, abnormal, 140
Cardioplegia solution, retrograde 243f, 255 Doppler evaluation of, 139
administration of, 409 for aortic stenosis, 214 Differential diagnosis, 85–86
Cardiopulmonary bypass, weaning from, intraoperative and interventional of cardiac murmurs, 87–88, 87f

/
385f echocardiography, 387 of chest pain, 88, 88f
Carotid artery, left, 344 of mitral regurgitant velocity, 250f of embolic event, 90

9
Catheters in mitral regurgitation, 263 of heart failure, 89, 89f
endocarditis from, 90 of mitral valve, 262 of palpitations, 89–90

9
as endocarditis risk factor, 301 optimization of Doppler recordings, 14t of valvular heart disease, 92t–94t

r
Cavitation, 11t in pulmonic regurgitation, 252f Dilated cardiomyopathy, 92t–94t, 120,

i
Chagas disease, 165, 179 retrograde flow recording, 10f 173–174, 175f, 175t–176t, 210, 246f
Chest pain signal, 239 functional mitral regurgitation in, 121
after myocardial infarction, 151–152 mitral regurgitation, 256 with mitral regurgitation, 171f

h
causes of, 150 pulmonary vein systolic flow reversal Distal flow reversals, 238, 238f
acute, 149t and density, 249–250 Dobutamine, arrhythmias from, 146

a
differential diagnosis of, 88, 88f transit time effect, 12 Dobutamine stress echocardiography, 97,

t
Chronic obstructive pulmonary disease, in tricuspid regurgitation, 254f 158t

r/
respiratory variation in, 211–212 in valve regurgitation, 300, 301f for aortic stenosis, 233–234
Circumflex coronary artery, 143 Contrast echocardiography, 74–75, decrease in chamber size with, 147f
Clinical data, integration with 74f–75f protocol for, 146, 146t

e
echocardiographic findings, 86, 86f definition of, 74 Doppler analysis, principles of, 1–20
Clinical indications. see Echocardiography; Contrast enhancement, for endocardial Doppler artifacts, 10–12, 11f, 11t

s
clinical indications for definition, 145, 145f Doppler effect, 7t
Color Doppler, 9–10, 19 Cor pulmonale, 181, 182f Doppler mitral valve area (MVADoppler),
baseline of, adjustment of, 61, 61f Coronary artery(ies) calculation of, 227

/r u
flow mapping, 9t, 10f anatomy of, 142–143, 142f–143f Doppler quantitation, principles of, 37t
intracardiac flow patterns, 386 left main, 53f Doppler signals, 262
intraoperative transesophageal, 386, 386f transesophageal echocardiography of, 53 recorded on TTE, 262
optimization of Doppler recordings, 14t transgastric apical view, 53f Doppler stroke volume, calculation of, 105f

.t c
with signal aliasing, 24f Coronary artery disease (CAD), 142–167, Doppler ultrasound, 6–12, 6f. see also
Color scale, 9t 149t Color Doppler; Continuous wave
Complete heart block, 369–370 anatomy review in, 142–143 (CW) Doppler; Pulsed Doppler
Congenital heart disease, 92t–94t differential diagnosis of, 92t–94t physics, 7t

a
in adult, 355–383 echocardiographic diagnosis of, 158t spectral tracing, 8f
apical window of, 359, 359f left ventricular systolic function wall filter settings of, 8f

k
basic principles of, 355–356 evaluation in, 144–145, 144f, 145t -dP/dt, calculation of, 140

/: /
basic transesophageal approach of, left ventricular wall segments review in, Drug users, intravenous, endocarditis in,
363–365, 363f–364f 142–143 301, 302f
basic transthoracic echo exam of, stress echocardiography for, 66t–67t, Dynamic range, 13t–14t
356–363, 357f–362f 143–148 Dyskinesis, 145t

s
categories of, 373t Coronary sinus, 31, 31f, 408f–409f Dyspnea, differential diagnosis of, 89
clues to identification with, 373t dilated, 346 Dyssynchronous motion, of myocardial
complex, 356, 369–372 Coronary stenosis, 142–143 segments, 82f

tt p
determination of anatomic/physiologic Coronary vasculopathy, after
questions of, 362–363 transplantation, 191 E
echocardiographic examination of, 373t Crista terminalis, 41, 316, 317f, 332 E/A ratio, 127, 128f, 129, 131, 135t, 140
intraoperative transesophageal Early diastolic velocity (E′), 72

h
echocardiography for, 395, 403t D Ebstein anomaly, 90, 190, 253f, 358f,
parasternal right ventricular inflow and Data integration, 86 368–369, 369f, 382–383
outflow views of, 358, 358f–359f Defibrillator, endocarditis from, 90 Echo exam, 78–80. see under specific
pulmonary artery evaluation for, 347 Defibrillator leads, endocarditis of, 302 echocardiographic modalities
as pulmonary hypertension cause, 183 Depth, 13t–14t basic principles, 13t–14t
pulmonary pressure estimation of, Descending aorta (DA) Echocardiographer, diagnostic thinking
361–362, 361f–362f diastolic flow reversal, 240, 243f of, 87
review and report study results in, 362 transesophageal echocardiography of, Echocardiographic image acquisition,
subcostal window of, 360, 360f 55–56, 55f principles of, 1–20
suprasternal notch window of, Descending thoracic aorta, 41, 336, 336f, Echocardiography
360–361, 360f–361f 353 clinical indications for, 85–97
Congenital valve abnormalities, aortic atheroma of, 346f bacteremia, 90, 91f
dilation associated with, 341 diastolic flow reversal in, 338f basic principles, 85–87
Constrictive pericarditis. see also hematoma of, 344f cardiac murmurs, 87–88, 87f
Pericardial constriction proximal, 337, 338f chest pain, 88, 88f
presentation of, 89 transesophageal echocardiography of, for common signs and symptoms,
restrictive cardiomyopathy and, 179 340, 341f 87–90
INDEX 413

Echocardiography (Continued) Endocarditis (Continued) Hemodynamics, cardiac


dyspnea, 89 transthoracic or transesophageal in acute coronary syndromes, 150
embolic event, 90 echocardiography for, 294–295, in end-stage ischemic disease, 157
fever, 90, 91f 294t, 295f in intraoperative transesophageal
heart failure, 89, 89f as tricuspid regurgitation, 252–253 echocardiography, 384–385, 385f,
palpitations, 89–90 valve anatomy examination of, 296, 401t
instrument panel, 22f 296f–297f regurgitation, 239f
E deceleration time, 127 valve dysfunction evaluation of, Hepatic vein systolic flow reversal, 238f
Eisenmenger physiology, 362, 366 297–298, 298f High pulse repetition frequency (HPRF),
Ejection fraction (EF), 46, 47f, 115t Endocardium in mitral stenosis, 221

/
in acute coronary syndrome, 149–150 echo contrast enhancement for Holodiastolic flow, 238f, 240
left ventricular, 102–103, 104f, 169, identification of, 145f reversal, 233, 336

9
169f posterior wall of, 99f Homograft valves, 267
less than 50%, 108 Endomyocardial fibrosis, 165 Hyperdynamic left ventricle response, to

9
quantitative, 46–47 Epicardial fat pad, 194, 195f dobutamine, 167

r
Electrocardiography (ECG), 13t–14t E-point septal separation (EPSS), 107, Hypertension. see also Pulmonary

i
in pericarditis, 193, 194f 107f, 170, 170f hypertension
Electrocautery, as transesophageal Eustachian valve, 316, 316f, 332 differential diagnosis of, 92t–94t
echocardiography artifact cause, 385f E wave deceleration slope, 141 Hypertensive heart disease, 180–181,

h
Embolic event, differential diagnosis of, 90 Exercise stress echocardiography, 83, 148f, 181f
Embolism 158t, 166 as aortic dilation cause, 342t–343t

a
of cardiac source, 322–323, 324t for myocardial infarction, 82 echocardiographic examination of,

t
echocardiographic findings associated protocol for, 145, 146f 184–185, 184t–185t

r/
with, 325t Exposure intensity (I), 11t hypertrophic cardiomyopathy
pulmonary, 192 and, 178
as chest pain cause, 148 F Hypertrophic cardiomyopathy, 92t–94t,

e
End-diastole, measurement in, 98, 98f False apex, 104f 96, 177–179, 232, 393–394, 403t
End-stage ischemic disease, 155–157, Fever, differential diagnosis of, 90, 91f dynamic subaortic outflow obstruction

s
156f–157f Fistula, paravalvular, 298–300, 299f, 311 in, 177, 178f
End-systole, measurement in, 98, 98f Flail posterior mitral leaflet, TEE imaging hypertensive heart disease and,
Endocardial borders, 99, 99f of, 247f 178–179

/r u
Endocarditis, 293–312 Focal depth, 13t–14t in left ventricle, 177, 177f
acute valve regurgitation due to, 300, Fontan conduit, 364f, 372, 372f with severe diastolic dysfunction, 132f
301f, 310, 312 Fontan physiology, 362, 371–372, 372f stress echocardiography for, 66t–67t
basic principles of, 293 Forward stroke volume, 236 Hypokinesis, 145t

.t c
clinical data in, review of, 293–294 Fossa ovalis, 333 Hypoperfusion, regional coronary, 165
clinical suspicion for, 96 French classification, for mitral valve Hypotension
diagnosis of morphology, 215t, 218 after myocardial infarction, 152–154
major criteria for, 310 Frequency ( f ), 2t, 3f in tamponade physiology, 193

a
minor criteria for, 293, 310
diagnostic echocardiographic findings G I

k
in, 293 Gain, 13t–14t Image acquisition, 1–20

/: /
differential diagnosis of, 92t–94t Gastric long-axis view, 63 Image resolution, 1
Duke criteria for, 303t, 310 Ghosting, 11 Imaging mode, 13t–14t. see also
echocardiographic approach to, Giant cell arteritis. see Takayasu arteritis M-mode; Three-dimensional (3D)
303–304, 303f Global longitudinal strain (GLS), 73, 82, echocardiography; Two-dimensional

s
echocardiographic examination 82f (2D) imaging; Zoom mode
of, 304t Great arteries, diseases of, 334–354 Inferior vena cava (IVC), 62
goals of, 293 basic principles of, 334 dilation of, 233

tt p
at intracardiac sites other than leaflets, echo exam of, 348t with Fontan conduit, 364f
300 transesophageal echocardiography for, in right atrial pressure estimation, 112,
intraoperative transesophageal 339–340, 339f, 341f 112t, 113f
echocardiography for, 390, 392f, transthoracic echocardiography for, in tamponade physiology, 200, 200f

h
403t 334–339, 335f–338f Informed consent, 44
in intravenous drug users, 301, 302f Grey scale, 13t–14t Instantaneous flow rate, 236
measuring hemodynamic consequences Instantaneous regurgitant flow rate
of valve dysfunction, 300 H (RFR), 257
with mitral leaflet perforation, 299, Harmonic imaging, 4, 6f, 13t–14t Interatrial baffle repair, 362, 362f
299f, 312 Heart, normal, 190–191 Interatrial septum, lipomatous
nonbacterial thrombotic, 310–311 Heart failure hypertrophy of, 319, 333
pacer/defibrillator leads and, 302, definition of, 89 Intercept angle, 7t, 18–19
302f, 310 differential diagnosis of, 89, 89f Interventricular septum, flattening
paravalvular abscess or fistula Heart murmurs, 87–88, 87f, 97 of, 191
evaluation in, 298–300, 299f, echocardiography measurements of, Intracardiac device, endocarditis
311 380–381 from, 90
prophylaxis against, 44 systolic, after myocardial infarction, 152 Intracardiac echocardiography, 75–76,
of prosthetic valves, 267, 301 Heart transplantation, intraoperative 76f
right-sided, 301 transesophageal echocardiography Intracardiac shunts, 356
Staphylococcus aureus and, 301–302 and, 395, 395f Intramural hematoma, as aortic dilation
Staphylococcus viridans and, 304 Hematoma, intramural, 344f, 353 cause, 342t–343t
414 INDEX

Intraoperative transesophageal Left atrium, 62 Left ventricular (LV) diastolic function


echocardiography echocardiographic anatomy of, 21t in cardiomyopathy, 171–172, 172f
in aortic dissection, 392, 393f–394f imaging measurements, 25t classification of, 135t
artifacts, 385f three-dimensional echocardiography evaluation of, in end-stage ischemic
in ascending aortic atheroma, 392 of, 72t disease, 156
baseline data acquisition of, 386–387 tracing of, mitral annular plane, quantitation of, 134t
basic principles of, 384–388 42, 43f Left ventricular (LV) diastolic relaxation,
in cardiopulmonary bypass weaning, transesophageal echocardiography of, 126, 127f
385f 47–48 Left ventricular (LV) dP/dt, 106,
2-chamber view in, 386, 386f volume measurement, 30, 30f 106f–107f, 115t, 120

/
4-chamber view in, 386, 386f Left carotid artery, 344 calculation of, 169, 170f
consideration of hemodynamic changes Left lower pulmonary vein, 408 Left ventricular ejection fraction (LVEF),

9
and surgical instrumentation, Left main coronary artery, 406f, 407 169, 169f
384–385 Left-sided heart disease, as pulmonary apical biplane calculation of, 120

9
in endocarditis, 390, 392f hypertension cause, 183 measurement of, 102–103, 104f

r
in heart transplantation, 395, 395f Left upper pulmonary vein, 408 Left ventricular (LV) filling velocities, in

i
in hypertrophic cardiomyopathy, Left ventricle, 41 pericardial disease, 199, 200f
393–394 anatomic, 380 Left ventricular (LV) function
indications for, 385t chamber dimensions of, 98–99, cardiac surgery and, 264

h
interpretation and communication 98f–101f, 102t–103t, 103f–104f intraoperative transesophageal
findings of, 387–388 chamber size, measurement of, 168, echocardiography monitoring of,

a
key data for, 403t 168f–169f 388, 388f

t
left ventricular assist devices, 394, in congenital heart disease, 355 regional, 73, 103–105, 105f

r/
394f dilation and dysfunction of, 170, 171f three-dimensional echocardiography
left ventricular function monitoring in, echocardiographic anatomy of, 21t of, 72t
388, 388f imaging measurements, 25t Left ventricular (LV) hypertrophy

e
long-axis view in, 386, 386f intraoperative transesophageal in cardiomyopathies, 170–171
in mitral valve repair, 388–389 echocardiography views of, 386f in hypertensive heart disease, 181

s
postprocedure data acquisition in, 387 M-mode tracing, 24, 25f pattern of, 171, 171f
preoperative data review of, 384 mass, 101–102 presence and severity of, 170–171
transcatheter interventions of, 401t measurement of, 98–102, 98f–99f Left ventricular (LV) inflow, in

/r u
valve replacement and, 389–390 parasternal views of, 29 diastole, 42
Intraprocedural echocardiography (ICE), rupture of, 152, 152f, 165 Left ventricular (LV) mass, calculation of,
96 shape changes with aortic regurgitation, 101–102
Intraprocedural transesophageal 244f Left ventricular (LV) noncompaction, 179,

.t c
echocardiography, basic principles short-axis view, 29f 180f, 190
of, 401t stroke volume and cardiac output Left ventricular outflow tract (LVOT),
Ischemia, right coronary artery calculation, 105–106, 105f–106f 63, 63f
distribution, 167 systematic 3D study of, 69t diameter, 216f

a
Ischemic cardiomyopathy, 155 transesophageal echocardiography measurement of, 105, 106f, 217f
Ischemic disease evaluation for, 46–47, 47f modal, systolic velocity, 232

k
cardiac, 158–161 transgastric short-axis view of, 388f obstruction in, 232

/: /
end-stage, 92t–94t, 155–157, wall stress of, 101–102 outflow velocity, 105, 106f
156f–157f wall thickness of, 100–101 three-dimensional measurement
Ischemic ventricular septal rupture, 167 Left ventricular (LV) apex, 386f of, 68f
Isovolumic relaxation time (IVRT), 125, transthoracic imaging of, 331 Left ventricular (LV) pseudoaneurysm,

s
126f, 135t, 140, 172f, 211 Left ventricular (LV) apical thrombus, in echocardiographic examination of,
dilated cardiomyopathy, 174, 175f 205t
L Left ventricular assist devices (LVADs), Left ventricular (LV) size, intraoperative

tt p
Lambl excrescences, 296f–297f, 310–311, 180, 181f, 394, 394f transesophageal echocardiography
321 Left ventricular (LV) chamber dilation, evaluation of, 388f
Late diastolic velocity (A′), 72 82f Left ventricular (LV) systolic dysfunction,
Left anterior descending coronary artery, Left ventricular (LV) diastolic filling 120

h
143 diastolic function, 127, 128f–129f Left ventricular (LV) systolic function,
Left atrial appendage, 48, 48f, 314, integration of data, 126, 127f 98–108, 107f
314f isovolumic relaxation time in cardiomyopathies, 169, 170f
Doppler signal recorded measurement, 125, 126f evaluation of
in, 331–332 left ventricular inflow velocity in coronary artery disease, 144–145,
three-dimensional echocardiography measurement, 123–124, 123f–124f 144f–145f, 145t
of, 72t in mild diastolic dysfunction, 127–129, in end-stage ischemic disease, 156,
thrombus of, 63–64 127f, 130f, 133f 159t
transesophageal echocardiography of, in moderate diastolic dysfunction, 127f, at peak heart rate, 146–147, 147f
314, 331 129, 130f, 133f global, 386f
Left atrial inflow velocity, 140 in normal diastolic function, 127, 128f, intraoperative transesophageal
Left atrial pressure 133f echocardiography in, 386f
estimation of, 131–132, 132f other useful measurements, 126, 127f left ventricular dP/dt, 106, 106f–107f,
increased, 139 in severe diastolic dysfunction, 127f, 115t, 120
Left atrial size, in cardiomyopathy, 173 131, 131f, 133f regional, 386f
Left atrial thrombi, 313–323, 314f tissue Doppler recording at mitral stroke volume and cardiac output
Left atrial volume, calculation of, 139 annulus, 124–125, 125f–126f calculation, 105–106, 105f–106f
INDEX 415

Left ventricular (LV) thrombi, 314–315, Mitral regurgitation (MR) (Continued) Mitral valve orifice, rheumatic, 219f
315f, 331 chronic ventricular volume overload Mitral valve prolapse, 246, 248f, 262f
apical, 47, 315 evaluation and, 250–251 continuous wave Doppler imaging,
and myocardial trabeculations, 315, clinical echocardiographic correlation, 262–263
315f, 331 245t murmur and, 192
Left ventricular (LV) volume, 99, 100f consequences evaluation, 251 Mitral valve repair, 277–278, 278f,
in pericardial disease, 199 etiology determination of, 244, 248f 388–389, 389f–391f, 403t
Left ventricular (LV) wall examination of, 256–262, 256t function of, 407
motion abnormalities of, 103 flow patterns in, 386 Mitral valve replacement (MVR),
segments of, 142–143, 142f in hypertrophic cardiomyopathy, 178f intraoperative transesophageal

/
thickness, calculation of, 100–101 ischemic, 246 echocardiography in, 389–390, 392f
Lipomatous hypertrophy, of interatrial jet direction determination, 246, 248f M-mode, 18

9
septum, 319, 333 left ventricular dilation and dysfunction aortic regurgitation, 243
Loculation, of pericardial fluid, 194, 196f and, 170, 171f aortic root motion in, 190

9
LV-to-LA pressure gradient, 263 M-mode tracing, 266 aortic stenosis, 232

r
maximum velocity from, 190 definition of, 4

i
M primary, 246f E-point septal separation (EPSS) in,
Marfan syndrome, 239, 336, 336f, 341, regurgitant jet velocity, 263 107, 107f, 170f
342f, 354 rheumatic, 250 left ventricular dimensions, 98, 99f, 102t

h
as aortic dilation cause, 342t–343t secondary, 246f mitral regurgitation, 266
Mean gradient three cardiac cycles, severity of, 244–250, 263, 389, 390f in mitral valve, 191–192, 191f

a
average, 233 quantitation of, 257t mitral valve coaptation, 232

t
Mean transaortic pressure gradient, 214, quantitative evaluation of, 257t in pericardial thickening, 201, 202f

r/
216f transthoracic echocardiography, in pulmonary hypertension, 191
Mean transmitral pressure gradient, in 247f recordings, 4, 5f
mitral stenosis, 221f vena contracta width measurement, sampling rate, 19

e
Mechanical index (MI), 12 244, 248f Murmurs. see Heart murmurs
Mechanical valves volume calculation, 249f Mustard repair, 370

s
aortic, 277, 277f TEE, 247f Myocardial infarction, 142, 150f
eccentric paravalvular mitral Mitral stenosis (MS), 218–224, 232, 233f cardiogenic shock after, 152–154
regurgitant jet in, 292 aortic and tricuspid valves for rheumatic complications of, 151–155, 151t, 160t

/r u
mitral, 279, 279f, 292 involvement, 223 late, 154–155
normal reference values of effective consequences evaluation, 220t, differential diagnosis of, 92t–94t
orifice areas for, 272t 223–224, 223f exercise echocardiography for, 82
Mesocardia, 370 evaluation of morphology, 218, 219f hypotension after, 152–154

.t c
Microbubbles examination, 226t left ventricular dilation and dysfunction
injection of. see Contrast jet, 221 due to, 170
echocardiography mean transmitral pressure gradient in, left ventricular walls and, 105f
solution, echocardiographic image 221f non-ST-elevation, 148

a
quality and, 82–83 with mitral regurgitation, 222–223 with recurrent chest pain, 151–152
Mirror image artifact, 12 percutaneous balloon valvotomy ST-elevation, 148

k
Mitral annulus of, 399 systolic murmur after, 152

/: /
apical motion of, 108 rheumatic, 219f, 223f Myocardial ischemia
calcification of, 19, 181, 181f, 321 severity of, 389 identification of, 97
in hypertensive heart disease, 191 classification, 227t postcardiac transplant, stress
motion of, 108, 108f evaluation, 220–222 echocardiography for, 66t–67t

s
tissue Doppler myocardial velocity quantitation, 227t resting wall motion in, 147f
recordings at, 124–125, 125f–126f Mitral valve, 277–279, 380 Myocardial mechanics, 71–74
Mitral A wave duration, 140 anatomy of, 388–389, 389f approaches and clinical applications,

tt p
Mitral balloon valvotomy, 396t, 399, 399f coaptation, A3 segment/P3 segment, 77t
Mitral inflow, 140f, 141 407f, 408 basic principles, 71–74
Mitral leaflets, 140, 140f echocardiographic anatomy of, 21t dyssynchrony, 74
cleft, 358, 365f long-axis view of, 26f speckle tracking strain, 73, 74f

h
intraoperative transesophageal M-mode tracing of, 332 strain and strain rate, 72–73
echocardiography imaging of, 389f mechanical, 279, 279f tissue Doppler velocities, 71–72, 73f
paravalvular aortic infection, 298, 298f scallops of, 42, 42f Myocardial segments, long-axis views, 42
perforation of, 299, 299f, 312 short-axis view of, 29f Myocardial shortening (systole), 73
prolapse, 263–264, 264f systematic 3D study of, 69t Myocardial trabeculations, thrombi and,
versus flail, 248f three-dimensional echocardiography of, 315, 315f, 331
Mitral regurgitant jet 50, 50f, 72t Myocardial velocity, tissue Doppler,
direction of, 389, 390f transesophageal echocardiography of, 124–125, 125f–126f, 139
eccentric paravalvular, 292 48–51, 50f Myocardial viability, stress
initial slope of, 121 transgastric view of, 51 echocardiography, 66t–67t
Mitral regurgitation (MR), 50–51, 51f, vegetation in, 296f, 298f Myocardial wall motion, patterns of, 146t
244–251, 259f, 262–266, 279, Mitral valve area, 292 Myxoma
290–291, 290f–291f, 311, 312f, 354 calculation of, 220 atrial, 319f
after myocardial infarction, 152, 153f measurement of, 220f left atrial (LA), 318, 332
antegrade mitral flow and stenosis Mitral valve disease Myxomatous disease
evaluation, 250 myxomatous, 244, 246f, 264 mitral valve, 244, 246f, 264
in cardiomyopathy, 171f, 173, 174f stress echocardiography for, 66t–67t as tricuspid regurgitation, 252–253
416 INDEX

N Pericardial constriction (Continued) Prosthetic mitral valves. see also


Near field clutter, 18 pericardial thickening in, evidence of, Bioprosthetic mitral valves
Negative data, 85 201, 202f imaging views of, 269, 270f–271f
Negative predictive value (NPV), of respiratory variation in, 211 mitral valve pressure half-time,
echocardiography, 85, 85f Pericardial cyst, 318 measurement of, 271, 273f
Nodules of Arantius, 320 Pericardial disease, 193–212. see also “patient-prosthesis mismatch”, 272,
Non-ST-elevation myocardial infarction Constrictive pericarditis; Pericardial 274f, 291
(NSTEMI), 148 cyst; Pericardial effusion; Pericarditis; regurgitation of, 269t, 274f
Nonlinear diastolic velocity curve, 222f Tamponade physiology, pericardial evaluation for, 272, 274f
Nonprimary cardiac tumors, 316–317, blood pressure and heart rate recording Prosthetic valves, 267–292

/
317f in, 193–194 aortic-mitral intervalvular fibrosa
Normal respiratory variation, 211, 212f differential diagnosis of, 92t–94t, 193t aneurysm of, 299–300, 300f

9
Nyquist limit, 7t echocardiography examination of, basic principles, 267–274
205–206, 205t basic types of, 267f

9
O pericardial effusion in, size of, 194–197, clinical and operative data, review of,

r
Oral suction, 44 197f–198f 269

i
Ostia, of coronary arteries, 142 pericardial fluid in, 194, 195f–196f clinical echocardiographic correlates
tamponade physiology in, 197–200 of, 268t
P Pericardial effusion, 41, 193–197, 195f, continuous wave Doppler recordings

h
Pacer leads, 331 210, 344 of, 273f
endocarditis of, 302, 302f, 310 echocardiographic examination of, 205t Doppler data for, 271–273

a
Palpitations, 89–90 endocarditis-related, 300 dysfunction of, echocardiographic signs

t
Papillary fibroelastoma, 318, 319f–320f, M-mode tracing of, 197f of, 283t

r/
331 size estimation of, 194–197, 197f–198f echo exam for, 282–284
Parasternal long-axis views, 23f, 24–26 subcostal view of, 198f endocarditis of, 301
aortic sinuses, 335f Pericardial fluid function of, 92t–94t

e
of congenital heart disease in adult, accumulation of, 193 key points in, 267–269
357, 357f distribution of, 194 left ventricular geometry and function,

s
left ventricular, 98, 99f presence of, 194, 195f–196f evaluation of, 273–274
Parasternal short-axis views, 29f Pericardial hematoma, 211 mechanical, 277, 277f
of congenital heart disease in adult, Pericardial sac, 194 normal reference values of effective

/r u
357–358, 357f–358f Pericardial stripping, 210–211 orifice areas for, 272t
of mitral valve, 29f Pericardiocentesis, 165, 200, 201f obtaining images of, 269–271, 270f
of trileaflet aortic valve, 28f Pericarditis, 193. see also Constrictive postprocedure dysfunction detection of,
Paravalvular abscess, 298–300, 299f, 311 pericarditis 389, 403t

.t c
Paravalvular fistula, 298–300, 299f, 311 as chest pain cause, 148 pulmonary pressures, measurement
Paravalvular regurgitation, 279, 280f purulent, 300 of, 274
prosthetic, transcatheter closure of, Pericardium, echocardiographic anatomy regurgitation of, 269t, 274f
396t, 399f–400f, 399 of, 21t evaluation for, 272, 274f

a
Partial flail leaflet, anteriorly directed Persistent left superior vena cava, 382 right heart function, evaluation of, 274
regurgitant jet in, 263–264, 264f Pleural effusion, 41 stenosis of, 269t, 298

k
Patent ductus arteriosus (PDA), 347, 353, Point of care echocardiography, 76, 79f evaluation for, 271, 272f

/: /
366–367, 368f, 382 in emergency department, 77t transcatheter aortic valve, 267, 270f
Patent foramen ovale, 321–322, 333 Positive data, 85 implantation of, 267, 276
closure of, 395 Positive likelihood ratio, 86 transesophageal echocardiographic
percutaneous, 323f Positive predictive value (PPV), of evaluation of, 282t

s
transesophageal echocardiography of, echocardiography, 85, 85f valve resuspension, 276f
322, 322f Positive pressure ventilation, 211–212 vegetations on, 295f
transthoracic echocardiography of, Posterior descending artery, myocardial Proximal isovelocity surface area (PISA),

tt p
321f ischemia in, 165 235–236, 236f
“Patient-prosthesis mismatch”, 272, 274f, Posttest probability, 86 calculation of regurgitant volume, 250f
291 Posttransplant heart disease, 180, 180f color Doppler measurement, 10
Peak aortic jet velocity, 232 Power output, 13t–14t in mitral regurgitation, 256

h
Peak diastolic velocity, 124 Prerevascularization, 92t–94t in mitral stenosis, 311
Peak systolic velocity, 124 Pressure half time (PHT), 222f, 232 radius, measurement of, 61, 61f
Percutaneous procedures valve area, 221, 221f Pseudoaneurysms
mitral valve repair, 277 Presyncope, 96 aortic, 277f, 291, 342t–343t, 345–346,
for patent foramen ovale closure, 323f Pretest probability, 86 345f
valvotomy, 221 Primary cardiac tumors, 318–319 left ventricle, 153, 154f, 165
valvuloplasty, 97 Probe Pulmonary arterial diastolic pressure,
Pericardial constriction, 200–204, 202f, position of, in transesophageal calculation of, 192
210. see also Constrictive pericarditis echocardiogram, 63 Pulmonary arterial systolic pressure,
after transplantation, 191 withdraw and reattempt placement of, 408 normal, 121–122
anatomic evidence of, 202 Propagation velocity, 126, 127f Pulmonary artery (PA)
comparison with pericardial Prosthetic aortic valves, 269. see also abnormalities of, 346–347, 347f
tamponade, 203t Bioprosthetic aortic valves basic echocardiographic approach of,
comparison with restrictive patient-prosthesis mismatch, 272, 274f, 346–347, 347f
cardiomyopathy, 203–204, 203t 291 branch, 361
Doppler studies for, 202–203, 203f regurgitation of, 269t stenosis of, 347, 347f, 371
echocardiographic examination of, 205t evaluation for, 272, 274f identification of, 355, 356f
INDEX 417

Pulmonary artery (PA) (Continued) Pulmonic valves Right atrium (Continued)


imaging measurements, 25t prosthetic, 280–281 three-dimensional echocardiography
right, 338f, 361 visualization of, 280 of, 72t
three-dimensional echocardiography stenosis of, 331 transesophageal echocardiography of,
of, 72t systematic 3D study of, 69t 54, 54f
thrombus of, 346 three-dimensional echocardiography Right brachiocephalic artery, 344
transesophageal echocardiography of, of, 72t Right coronary artery, 143
54–55, 55f Pulse repetition frequency (PRF), 7t Right heart enlargement, pulmonary
in transposition of great vessels, 357 Pulsed Doppler, 7t, 8–9, 19–20 heart disease and, 183
Pulmonary artery pressure (PAP), 112f, gain setting, 8f Right heart function, with prosthetic

/
115t intraoperative transesophageal, valves, 274
in cardiomyopathy, 172–173, 172f–173f 386–387, 387f Right heart thrombi, 315–316, 316f–317f

9
Pulmonary embolism, 192 LV outflow velocity recording, 8f Right supraclavicular window, imaging
as chest pain cause, 148 optimization of Doppler recordings, 14t from, 353, 353f

9
Pulmonary function testing, 82, 82f tissue, of tricuspid annulus, 190 Right ventricle, 41, 62

r
Pulmonary heart disease, 181–183, 182f velocity recording, 23, 23f anatomic, 380

i
echocardiographic examination of, Pulsus paradoxus, 193, 194f apical view, 31, 31f
184–185, 184t–185t in atrial septal defect, 360f
pulmonary hypertension and, 183 Q in congenital heart disease, 355, 355f

h
pulmonary pressure estimation in, 182, Q p measurement, 192 echocardiographic anatomy of, 21t
183f Q p/Qs ratio, 192 imaging measurements, 25t

a
right heart enlargement and, 183 Qualitative information, 86 inflow view, 27, 27f

t
right ventricular size and function in, Quantitative information, 86 outflow view, 27–28, 27f–28f

r/
182 systematic 3D study of, 69t
tricuspid regurgitation in, 182 R transesophageal echocardiography of,
Pulmonary hypertension, 89, 112, Radiation therapy, cardiac effects of, 317 53–54, 53f

e
210–211, 284, 300 Range ambiguity, 11–12 transgastric view of, 54, 54f
classification of, 113t Reflected backscatter frequency, 18 in transposition of great arteries, 359f

s
from congenital heart disease, 183 Reflection, 1, 2t Right ventricular chamber size, 108,
differential diagnosis of, 92t–94t Refraction, 1, 2t, 3f 109f–110f, 109t
from left-sided heart disease, 183 Refraction artifact, 6, 6f Right ventricular diastolic collapse,

/r u
in mitral stenosis, 223 Regional coronary hypoperfusion, 165 198–199, 199f
in pulmonary vascular disease, 181, Regional ventricular function, in acute Right ventricular diastolic pressure, 122
182f–183f, 183 coronary syndromes, 148–149, 150f calculation of, 192
pulmonic regurgitant velocity in, Regurgitant fraction, 255, 265 Right ventricular filling velocities, in

.t c
347 Regurgitant jet velocity, maximum, 263 pericardial disease, 199, 200f
Pulmonary pressure Regurgitant orifice area (ROA), 237–238, Right ventricular function, three-
in congenital heart disease, 361–362, 238f, 243, 255, 311 dimensional echocardiography of, 72t
361f–362f calculation of, 61, 263, 265 Right ventricular infarction, 153, 154f

a
in prosthetic valves, 274 clinically indicated, 247–249 Right ventricular outflow, 358
Pulmonary systolic pressure, 112 maximum, 257 Right ventricular outflow tract (RVOT), 28f

k
in congenital heart disease, 361 Regurgitant volume, 236, 243, 255, 311 pulmonic regurgitation, 252f

/: /
elevated. see Pulmonary hypertension calculation of, 263, 265 Right ventricular pressure overload, 108,
in pulmonary heart disease, 182, Regurgitation hemodynamics, 239f 111f
183f Reliability, of echocardiography, 85–86 Right ventricular size
in restrictive cardiomyopathy, 179 Resolution, 2t, 3f in cardiomyopathy, 173, 174f

s
Pulmonary valve, transesophageal axial, 19 in end-stage ischemic disease, 157
echocardiography of, 54–55, 55f lateral, 4, 5f in pulmonary heart disease, 182
Pulmonary veins, 338f Respiratory variation, normal, 211 Right ventricular systolic function,

tt p
diastolic flow, 128f Restrictive cardiomyopathy, 92t–94t, 108–114
flow, 48, 50f 121–122, 175t–176t, 179, 179f, 210 in cardiomyopathy, 173
flow systolic flow reversal, 250f, 264 comparison with constrictive chamber size and wall thickness
inflow, 141 pericarditis, 203–204, 203t evaluation, 108, 109f–110f, 109t

h
left, 48, 49f M-mode tracing, 121f in end-stage ischemic disease, 157, 157f
upper, 62–63, 63f respiratory variation in, 211 intraoperative transesophageal
right, 48, 49f, 62–63, 62f Reverberation artifacts, 314, 354 echocardiography evaluation of,
transesophageal echocardiography of, Reverberations, 6, 18 386f
48 Rheumatic mitral regurgitation, 222f in pulmonary heart disease, 182
Pulmonic regurgitant jet, 192, 192f Rheumatic valve disease, 232, 234 pulmonary pressure calculation, 112,
Pulmonic regurgitation, 251–252, 262, mitral, 218 112f–113f, 112t
264 Right atrial appendage, 62 pulsed tissue Doppler in, 190
after tetralogy of Fallot repair, 369 Right atrial filling pressures, in tamponade systolic contraction estimation,
color Doppler image of, 251f physiology, 200, 200f 109–111, 111f
etiology determination, 251 Right atrial mass, 301f ventricular septal motion, 108, 110f–111f
right ventricular volume overload, Right atrial pressure (RAP), 112, 112t, Right ventricular volume
252 113f overload, 108, 110f
severity evaluation, 251–252, 252f Right atrial systolic collapse, 198, 198f in pericardial disease, 199
Pulmonic stenosis, 224, 224f, 369–370 Right atrium pulmonic regurgitation, 252
branch, 371 echocardiographic anatomy of, 21t Ring-down artifact, 18
Pulmonic-to-systemic flow ratio, 356 imaging measurements, 25t Ross procedure, 290–291
418 INDEX

S Stress testing. see Stress echocardiography Three-dimensional (3D)


Saline contrast study, 322f Stroke volume (SV) echocardiography, 65–71
Sample line, 9t forward, 236 American Society of Echocardiography
Sample volume, 7t left ventricular, 105–106, 105f, 120 (ASE) and the European
Scattering, 1, 2t total, 236 Association of Echocardiography
Secondary cardiac tumors, 333, 333f Subaortic outflow obstruction, 177, 178f (EAE) recommendations for, 78t
Sector width, 9t, 13t–14t, 23 Subaortic stenosis, 232 clinical applications of, 72t
Secundum atrial septal defect, 84 Subclavian artery, left, 344 definition of, 4
“Senile calcification”, 232 Subcostal window views, 33 examination protocol, 65, 69t
Senning repair, 370 of congenital heart disease in adult, image acquisition and display, 65, 68f

/
Sensitivity, 85, 85f 360, 360f for mitral valve prolapsed, 83
Septal thickening, in hypertensive heart four-chamber, 33f modalities, 79t

9
disease, 191 hepatic vein flow, 33f quantitation from 3D datasets, 65–71,
Shadow artifacts, 11 inferior vena cava, 33f 70f–71f

9
Shock, cardiogenic, 152–154 of pericardial effusion, 198f realtime narrow sector imaging, 65, 67f

r
Shunts, intracardiac, 356 Subvalvular apparatus, 218 recordings of, aortic valve motion, 5f

i
Signal aliasing, 7t, 9f, 10, 11f Superior vena cava (SVC), 62 TEE, of mitral valve, 50, 50f
Sinotubular junction ascending aorta and, 361 Threshold approach, to clinical decision-
definition of, 335 persistent left, 346 making, 86

h
in Marfan syndrome, 342f Suprasternal notch window views, 232 Time gain compensation (TGC), 13t–14t
measurement of, 335, 335f of aortic arch, 337, 338f, 353 Tissue Doppler

a
Sinus of Valsalva, 299f, 310 of proximal descending thoracic aorta, normal heart in, 190–191

t
aneurysm, 344–345, 345f, 349t 337, 338f velocity, 72, 124–125, 125f–126f

r/
as aortic dilation cause, 342t–343t Suprasternal window views, 33–34, 34f of myocardial motion, 84
measurement of, 335, 335f Syphilitic aortitis, as aortic dilation cause, strain rate and strain, 73
paravalvular aortic abscess of, 298 342t–343t Tissue interaction, ultrasound, 1, 2t, 3f

e
Sinus rhythm, 48f Systemic inflammatory disease, as aortic Trabeculations
Sinus venosus atrial septal defect, 63 dilation cause, 342t–343t apical, 155, 156f

s
Small aperture size, 18 Systemic ventricle, 355–356 myocardial, thrombi versus, 315, 315f,
Specificity, 85, 85f Systolic anterior motion (SAM), 232 331
Speckle tracking, 83 mitral, 178, 178f Trachea, interposition of, 63

/r u
strain imaging, 73 Systolic function, normal, 140 Transaortic pressure gradient, 214
in dilated cardiomyopathy, 174 Systolic velocities, 381 mean, 216f
Spectral analysis, 7t Transaortic stroke volume, 233
ST-elevation myocardial infarction T Transaortic volume flow rate, 216f

.t c
(STEMI), 148 Tachycardia, in tamponade physiology, Transcatheter aortic valve implantation,
Staphylococcus aureus, endocarditis and, 302 193 395–396, 396t, 397f–398f
Stenosis. see also Aortic stenosis; Mitral Takayasu arteritis, 342 Transcatheter bioprosthetic aortic valve
stenosis; Tricuspid stenosis as aortic dilation cause, 342t–343t replacement, 290

a
coronary, 142–143 Takotsubo cardiomyopathy, 166, 166f, Transcatheter mitral valve repair, 396,
of prosthetic valves, 269t, 271, 272f, 298 179, 190 396t, 398f

k
Stented bioprosthetic valves, 272t Tamponade physiology, pericardial, 193, Transducers, 1–3, 3f, 13t–14t

/: /
Stentless prosthetic valves, 271 197–200, 210 characteristics of, 3
Strain, 72–73 comparison with pericardial configuration approaches of, 4t
tissue Doppler velocity, 73 constriction and restrictive moving of, to higher interspace, 41f
Strain rate (SR), 72–73 cardiomyopathy, 203t position, 13t–14t

s
tissue Doppler velocity, 73 echo-guided pericardiocentesis in, 200, Transesophageal echocardiography
Streptococcus viridans, endocarditis and, 304 201f (TEE), 44–64, 213, 214f, 217f. see
Stress cardiomyopathy, 166, 166f echocardiographic examination also Intraoperative transesophageal

tt p
Stress echocardiography, 65, 66t–67t of, 205t echocardiography
in coronary artery disease, 143–148, reciprocal respiratory changes in right aortic annulus measurement of, 276
158t ventricular and left ventricular for aortic dissection, 342–344
dobutamine, 146, 146t filling velocities, 199, 200f for aortic stenosis, 233

h
exercise, 145, 146f reciprocal respiratory changes in right of aortic valve and ascending aorta,
indications for discontinuing, 166 ventricular and left ventricular 51–52, 51f
left ventricular systolic function volumes in, 199 of atrial septum, 47–48, 47f
for, evaluation of, 144–145, respiratory variation in, 211, 212f basic examination principles, 44–45, 57t
144f–145f, 145t right atrial filling pressures in, 200, 200f of bioprosthetic valves, 275
patient recovery in, monitoring of, 147 right atrial systolic collapse in, 198, of cardiac source of embolus, 323
pharmacologic, 144 198f clinical data, 44
preparation for, 144 right ventricular diastolic collapse in, for congenital heart disease, 363–365,
principles of, 143, 143f 198–199, 199f 363f–364f
protocol for, 145–146 Tetralogy of Fallot (TOF), 224, 369, contraindications to, 44
review and interpretation of, 369f–370f, 381 of coronary arteries, 53, 53f
147–148 surgically repaired, 251 of descending aorta and aortic arch,
treadmill exercise, 145, 166 Thebesian valve, 332 55–56, 55f
dobutamine stress, 66t–67t Thermal bioeffects, 11t imaging sequence, 45–46
exercise, 66t–67t Thermal index (TI), 12 of left atrial appendages, 314
pharmacologic, 66t–67t Thoracic aortic dissection, complications of left atrium, 47–48, 270, 271f
three-dimensional, 72t of, 349t bicaval view of, 333
INDEX 419

Transesophageal echocardiography (TEE) Transesophageal echocardiography (TEE) V


(Continued) (Continued) Valsalva maneuver, 83
of left ventricle, 46–47, 47f standard acoustic windows, 24, 24f in cardiac cycle, 139
of mitral valve, 48–51, 50f subcostal window views, 33 E velocity and, 127
of nonprimary cardiac tumors, 317 suprasternal window views, 33–34, 34f for pseudo-normal transmitral flow
probe, 44, 45f threshold, 86 identification, 129
positioning, 45, 45f–46f Transvalvular volume flow rate, 236 Valve area, planimetry of, 220
of prosthetic valve, 282t Traumatic aortic disease, as aortic dilation Valve disease
protocol, 44 cause, 342t–343t echocardiography in transcatheter
of pulmonary valve and pulmonary Treadmill exercise stress interventions for, 396t

/
artery, 54–55, 55f echocardiography, 66t–67t, 145, 166 in end-stage ischemic disease, 157
report, 56 transfer to imaging stretcher in, Valve regurgitation, 235–266.

9
of right atrium, 54 165–166 see also Aortic regurgitation;
bicaval view of, 333 Tricuspid annular plane systolic excursion Mitral regurgitation; Pulmonic

9
of right ventricle and tricuspid valves, (TAPSE), 190 regurgitation; Tricuspid regurgitation

r
53–54, 53f Tricuspid annulus, pulsed tissue Doppler differential diagnosis of, 92t–94t

i
of thrombus in transit, 332–333 interrogation of, 190 distal flow reversals, 238, 238f
for transcatheter aortic valve Tricuspid atresia, 362, 364f, 371–372, evaluation of, 356
implantation, 276 372f fluid dynamics of, 235f

h
Transgastric short-axis view, 388f Tricuspid regurgitant jet, 112f, 292 proximal isovelocity surface area of,
Transgastric views, of congenital heart high-velocity, 312 235–236

a
disease, 365 velocity, measurement of, 354, 354f regurgitant orifice area of, 237–238

t
Transit time effect, 12 vena contracta of, 253f regurgitant volume, 236

r/
Transmitral flow, color Doppler M-mode Tricuspid regurgitation, 252–253, 259f, valve vegetation and, 297, 298f
tracing of, 139 262–263, 265 volume flow, 237f
Transmitral stroke volume, 222 in cardiomyopathy, 173 Valve replacement, intraoperative

e
Transplant rejection, 191 continuous wave Doppler recording transesophageal echocardiography
Transposition of great arteries (TGA), of, 254f in, 389–390

s
359f, 362f, 382–383, 383f eccentric jet, 263 Valvotomy
complete, 370–371, 371f etiology evaluation of, 252–253 mitral balloon, 399, 399f
congenitally corrected, 369–370, jet velocity, 263 percutaneous procedures, 221

/r u
370f in pulmonary heart disease, 182 Valvular heart disease, heart failure
Transpulmonary microbubble contrast, right ventricular volume overload in, in, 89
165–166 253 Valvular stenosis, 213–234, 403t. see
left anterior descending artery severity evaluation of, 253 also Aortic stenosis; Mitral stenosis;

.t c
distribution, 83–84 Tricuspid stenosis, 223, 224f Tricuspid stenosis
Transthoracic echo exam, in congenital Tricuspid valve, 380 differential diagnosis of, 92t–94t
heart disease, 356–363, 357f–362f echocardiographic anatomy of, 21t evaluation of, 356
Transthoracic echocardiography (TTE) leaflets of, 41, 41f Valvular vegetation

a
additional components, 37t prosthetic, 280, 281f detection of, 295f–296f, 296
aging changes on, 34 regurgitant velocity, 27f motion of, 296, 297f

k
apical window, 29–32 stenosis of, 233 prosthetic, 295f

/: /
clinical data, 21 systematic 3D study of, 69t Valvular vegetations, 319–320
core elements, 36–37, 36t three-dimensional echocardiography detection of, 90
data recording, 23–24 of, 72t of nonbacterial thrombotic
of echocardiographic anatomy, 21t transesophageal echocardiography of, endocarditis, 320, 320f

s
examination sequence, 24 53–54 Valvuloplasty, percutaneous mitral, 97
flow patterns on, 21–43 vegetation in, 302f Variance, 9t
of great arteries, 334–339, 335f–338f Tricuspid velocity curve, antegrade, 223, Vegetations. see Valvular vegetations

tt p
imaging measurements, 25t 224f Velocity of propagation (c), 2t
instrumentation principles, 22–23, Two-dimensional (2D) imaging, 18 Velocity ratio (dimensional index), 234
22f definition of, 4 Velocity-time integral, 236, 237f–238f
of left atrial thrombi, 314 left ventricular size, 99, 100f Vena contracta width, 182, 235, 235f

h
left ventricular, 102t recordings of, aortic valve motion, 5f measurement
of left ventricular thrombi, 315 system controls for, 4 aortic regurgitation, 239, 242f, 255
long-axis view, 24–26 mitral regurgitation, 244, 256
of mitral mechanical prostheses, 96 U tricuspid regurgitant jet of, 253f
for mitral regurgitation, 247f Ultrasound Ventricular arrhythmias, 90
normal anatomy on, 21–43 bioeffects, 12 from dobutamine, 146
parasternal long-axis view, 23f of cardiac masses, 313 Ventricular bigeminy, 166
parasternal window views, 24–29 exposure measurement, 12 Ventricular diastolic filling
of patent foramen ovale, 321f imaging modalities, 4–6 diastolic function, 127, 128f–129f
patient positioning, 22, 22f misaligned, 42–43, 43f integration of data, 126, 127f
for pulmonary arteriovenous principles of, 1 isovolumic relaxation time
malformation, 83 safety in, 11t, 12 measurement, 125, 126f
repeat, 233 terminology, 11t left atrial inflow recording, 124,
report, 34, 35t Ultrasound gel, 22f 124f–125f
right ventricular inflow view, 27 Ultrasound waves, 1, 2t left ventricular inflow velocity
right ventricular outflow view, 27–28 Unicuspid valve, 340 measurement, 123–124,
short-axis view, 28–29 Unstable angina, 148 123f–124f
420 INDEX

Ventricular diastolic filling (Continued) Ventricular septal defects (VSDs), W


in mild diastolic dysfunction, 127–129, 262, 357, 362, 366, 366f–367f, Wall motion
127f, 130f, 133f 369–370, 380–382 abnormality in, 148, 148f
in moderate diastolic dysfunction, 127f, after myocardial infarction, 152, echocardiographic views of, 159f
129, 130f, 133f 153f resting regional, 158t
in normal diastolic function, 127, 128f, maximum velocity from, 380 Wall motion score index, 167
133f Ventricular systolic function. see also Left Warfarin, 166–167
other useful measurements, 126, 127f ventricular (LV) systolic function; Wavelength (λ), 2t
in severe diastolic dysfunction, 127f, Right ventricular systolic function Wolf-Parkinson-White syndrome, 369
131, 131f, 133f echo exam of, 115–116

/
tissue Doppler recording at mitral Vitamin K antagonist therapy, Z
annulus, 124–125, 125f–126f 166–167 Zoom mode, 13t–14t

9
“Ventricular inversion”, 370 Volume flow rate, 236 of valvular vegetations, 297f, 300f
Ventricular preexcitation, 369 in vena contracta measurements, 235,

9
242f

i r
a h
r/ t
s e
/r u
.t c
k a
/: /
s
tt p
h
9/
i r9
a h
r/ t
s e
/r u
.t c
k a
/: /
s
tt p
h

Вам также может понравиться